You are on page 1of 517

The Most Accepted

Crash Course Programme

JEE Main in
Days
Physics
The Most Accepted
Crash Course Programme

JEE Main in
Days
Physics

ARIHANT PRAKASHAN (Series), MEERUT


ARIHANT PRAKASHAN (Series), MEERUT
All Rights Reserved

© Publisher
No part of this publication may be re-produced, stored in a retrieval system or by any means,
electronic, mechanical, photocopying, recording, scanning, web or otherwise without the
written permission of the publisher. Arihant has obtained all the information in this book from
the sources believed to be reliable and true. However, Arihant or its editors or authors or
illustrators don’t take any responsibility for the absolute accuracy of any information published
and the damage or loss suffered thereupon.
All disputes subject to Meerut (UP) jurisdiction only.

Administrative & Production Offices


Regd. Office
‘Ramchhaya’ 4577/15, Agarwal Road, Darya Ganj, New Delhi -110002
Tele: 011- 47630600, 43518550

Head Office
Kalindi, TP Nagar, Meerut (UP) - 250002
Tel: 0121-7156203, 7156204

Sales & Support Offices


Agra, Ahmedabad, Bengaluru, Bareilly, Chennai, Delhi, Guwahati,
Hyderabad, Jaipur, Jhansi, Kolkata, Lucknow, Nagpur & Pune.

ISBN 978-93-25796-29-4
PO No : TXT-XX-XXXXXXX-X-XX
Published by Arihant Publications (India) Ltd.
For further information about the books published by Arihant, log on to
www.arihantbooks.com or e-mail at info@arihantbooks.com
Follow us on
PREFACE
It is a fact that nearly 10 lacs students would be in the race with you in JEE Main, the gateway
to some of the prestigious engineering and technology institutions in the country, requires
that you take it seriously and head-on. A slight underestimation or wrong guidance will ruin
all your prospects. You have to earmark the topics in the syllabus and have to master them in
concept-driven-problem-solving ways, considering the thrust of the questions being asked in
JEE Main.

The book 40 Days JEE Main Physics serves the above cited purpose in perfect manner. At
whatever level of preparation you are before the exam, this book gives you an accelerated way
to master the whole JEE Main Physics Syllabus. It has been conceived keeping in mind the
latest trend of questions, and the level of different types of students.

The whole syllabus of Physics has been divided into day-wise-learning modules with clear
groundings into concepts and sufficient practice with solved and unsolved questions on that
day. After every few days you get a Unit Test based upon the topics covered before that day.
On last three days you get three full-length Mock Tests, making you ready to face the test. It is
not necessary that you start working with this book in 40 days just before the exam. You may
start and finish your preparation of JEE Main much in advance before the exam date. This will
only keep you in good frame of mind and relaxed, vital for success at this level.

Salient Features
Ÿ Concepts discussed clearly and directly without being superfluous. Only the required
material for JEE Main being described comprehensively to keep the students focussed.
Ÿ Exercises for each day give you the collection of only the Best Questions of the concept,
giving you the perfect practice in less time.
Ÿ Each day has two Exercises; Foundation Questions Exercise having Topically Arranged
Questions & Progressive Question Exercise having higher Difficulty Level Questions.
Ÿ All types of Objective Questions included in Daily Exercises (Single Option Correct,
Assertion & Reason, etc).
Ÿ Along with Daywise Exercises, there above also the Unit Tests & Full Length Mock Tests.
Ÿ At the end, there are all Online Solved Papers of JEE Main 2021; February, March, July &
August attempts.
We are sure that 40 Days Physics for JEE Main will give you a fast way to prepare for Physics
without any other support or guidance.

Publisher
CONTENTS
Preparing JEE Main 2022 Physics in 40 Days !
Day 1. Units and Measurement 1-11 Day 21. Magnetic Effect of Current 241-253
Day 2. Kinematics 12-24 Day 22. Magnetism 254-263
Day 3. Scalar and Vector 25-36 Day 23. Electromagnetic Induction 264-274
Day 4. Laws of Motion 37-53 Day 24. Alternating Current 275-287
Day 5. Circular Motion 54-62 Day 25. Electromagnetic Wave 288-295
Day 6. Work, Energy and Power 63-77 Day 26. Unit Test 5 (Magnetostatics,
Day 7. System of Particles and Rigid Body 78-87 EMI & AC, EM Wave) 296-301

Day 8. Rotational Motion 88-97


Day 27. Ray Optics 302-315
Day 9. Gravitation 98-109
Day 28. Optical Instruments 316-322
Day 10. Unit Test 1 (Mechanics) 110-116
Day 29. Wave Optics 323-333

Day 11. Oscillations 117-130 Day 30. Unit Test 6 (Optics) 334-339

Day 12. Waves 131-143


Day 31. Dual Nature of Matter 340-349
Day 13. Unit Test 2
Day 32. Atoms 350-358
(Waves and Oscillations) 144-150
Day 33. Nuclei 359-366
Day 14. Properties of Matter 151-166 Day 34. Electronic Devices 367-377
Day 15. Heat and Thermodynamics 167-184 Day 35. Gate Circuit 378-385
Day 16. Transfer of Heat 185-194 Day 36. Communication Systems 386-394
Day 17. Unit Test 3 Day 37. Unit Test 7 (Modern Physics) 395-401
(General Properties of Matter) 195-201
Day 38. Mock Test 1 402-408
Day 18. Electrostatics 202-218
Day 39. Mock Test 2 409-415
Day 19. Current Electricity 219-233
Day 20. Unit Test 4 (Electrostatics & Day 40. Mock Test 3 416-423
Current Electricity) 234-240
Online JEE Main Solved Papers 2021 1-72
SYLLABUS
PHYSICS
NOTE The syllabus contains two Sections - A & B. Section A pertains to the Theory Part, having 80%
weightage, while Section B contains Practical Component (Experimental Skills) having 20% weightage.

SECTION- A
UNIT 1 Physics and Measurement UNIT 4 Work, Energy and Power
Physics, technology and society, SI units, Fundamental Work done by a constant force and a variable force;
and derived units. Least count, accuracy and precision kinetic and potential energies, work-energy theorem,
of measuring instruments, Errors in measurement, power.
Significant figures. Dimensions of Physical quantities, Potential energy of a spring, conservation of
dimensional analysis and its applications. mechanical energy, conservative and nonconservative
forces; Elastic and inelastic collisions in one and two
UNIT 2 Kinematics dimensions.
Frame of reference. Motion in a straight line: Position-
time graph, speed and velocity. Uniform and non- UNIT 5 Rotational Motion
uniform motion, average speed and instantaneous Centre of mass of a two-particle system, Centre of
velocity. mass of a rigid body; Basic concepts of rotational
Uniformly accelerated motion, velocity-time, position motion; moment of a force, torque, angular
time graphs, relations for uniformly accelerated momentum, conservation of angular momentum
motion. and its applications; moment of inertia, radius of
Scalars and Vectors, Vector addition and Subtraction, gyration. Values of moments of inertia for simple
Zero Vector, Scalar and Vector products, Unit Vector, geometrical objects, parallel and perpendicular axes
Resolution of a Vector. Relative Velocity, Motion in a theorems and their applications.
plane, Projectile Motion, Uniform Circular Motion. Rigid body rotation, equations of rotational motion.

UNIT 3 Laws of Motion UNIT 6 Gravitation


Force and Inertia, Newton's First Law of motion; The universal law of gravitation.
Momentum, Newton's Second Law of motion; Impulse; Acceleration due to gravity and its variation with
Newton's Third Law of motion. Law of conservation altitude and depth.
of linear momentum and its applications, Equilibrium Kepler's laws of planetary motion.
of concurrent forces. Gravitational potential energy; gravitational potential.
Static & Kinetic friction, laws of friction, rolling friction. Escape velocity. Orbital velocity of a satellite.
Dynamics of uniform circular motion: Centripetal Geo-stationary satellites.
force and its applications.
UNIT 7 Properties of Solids & Liquids Wave motion Longitudinal and transverse waves,
Elastic behaviour, Stress-strain relationship, Hooke's. speed of a wave. Displacement relation for a
Law, Young's modulus, bulk modulus, modulus of progressive wave. Principle of superposition of
rigidity. waves, reflection of waves, Standing waves in strings
and organ pipes, fundamental mode and harmonics,
Pressure due to a fluid column; Pascal's law and its Beats, Doppler effect in sound.
applications.
Viscosity, Stokes' law, terminal velocity, streamline and UNIT 11 Electrostatics
turbulent flow, Reynolds number. Bernoulli's Electric charges Conservation of charge, Coulomb's
principle and its applications. law-forces between two point charges, forces
Surface energy and surface tension, angle of contact, between multiple charges; superposition principle
application of surface tension - drops, bubbles and and continuous charge distribution.
capillary rise.
Electric field Electric field due to a point charge,
Heat, temperature, thermal expansion; specific heat Electric field lines, Electric dipole, Electric field due
capacity, calorimetry; change of state, latent heat. to a dipole, Torque on a dipole in a uniform electric
Heat transfer-conduction, convection and radiation, field.
Newton's law of cooling. Electric flux, Gauss's law and its applications to find
field due to infinitely long, uniformly charged
UNIT 8 Thermodynamics straight wire, uniformly charged infinite plane sheet
Thermal equilibrium, zeroth law of thermo-dynamics, and uniformly charged thin spherical shell.
concept of temperature. Heat, work and internal Electric potential and its calculation for a point
energy. First law of thermodynamics. charge, electric dipole and system of charges;
Second law of thermodynamics: reversible and Equipotential surfaces, Electrical potential energy of
irreversible processes. Camot engine and its efficiency. a system of two point charges in an electrostatic
field.
UNIT 9 Kinetic Theory of Gases Conductors and insulators, Dielectrics and electric
Equation of state of a perfect gas, work done on polarization, capacitor, combination of capacitors in
compressing a gas. series and in parallel, capacitance of a parallel plate
capacitor with and without dielectric medium
Kinetic theory of gases - assumptions, concept of
between the plates, Energy stored in a capacitor.
pressure. Kinetic energy and temperature: rms speed
of gas molecules; Degrees of freedom, Law of UNIT 12 Current Electricity
equipartition of energy, applications to specific heat
Electric current, Drift velocity, Ohm's law, Electrical
capacities of gases; Mean free path, Avogadro's
resistance, Resistances of different materials, V-I
number.
characteristics of Ohmic and nonohmic conductors,
Electrical energy and power, Electrical resistivity,
UNIT 10 Oscillations and Waves Colour code for resistors; Series and parallel
Periodic motion - period, frequency, displacement as combinations of resistors; Temperature
a function of time. Periodic functions. Simple harmonic dependence of resistance.
motion (S.H.M.) and its equation; phase; oscillations of Electric Cell and its Internal resistance, potential
a spring - restoring force and force constant; energy in difference and emf of a cell, combination of cells in
S.H.M. - kinetic and potential energies; Simple series and in parallel.
pendulum - derivation of expression for its time
Kirchhoff's laws and their applications.
period; Free, forced and damped oscillations,
Wheatstone bridge, Metre bridge.
resonance.
Potentiometer - principle and its applications.
UNIT 13 Magnetic Effects of Current Interference, Young's double slit experiment and
expression for fringe width, coherent sources and
and Magnetism
sustained interference of light. Diffraction due to
Biot-Savart law and its application to current carrying a single slit, width of central maximum.
circular loop. Ampere's law and its applications to Resolving power of microscopes and
infinitely long current carrying straight wire and solenoid. astronomical telescopes, Polarisation, plane
Force on a moving charge in uniform magnetic and polarized light; Brewster's law, uses of plane
electric fields Cyclotron. polarized light and Polaroids.
Force on a current-carrying conductor in a uniform
magnetic field. Force between two parallel current- UNIT 17
carrying conductors-definition of ampere. Torque Dual Nature of Matter and Radiation
experienced by a current loop in uniform magnetic field,
Dual nature of radiation. Photoelectric effect,
Moving coil galvanometer, its current sensitivity and
Hertz and Lenard's observations; Einstein's
conversion to ammeter and voltmeter.
photoelectric equation; particle nature of light.
Current loop as a magnetic dipole and its magnetic Matter waves-wave nature of particle, de Broglie
dipole moment. Bar magnet as an equivalent solenoid, relation. Davisson-Germer experiment.
magnetic field lines; Earth's magnetic field and magnetic
elements. Para, dia and ferro-magnetic substances UNIT 18 Atoms and Nuclei
Magnetic susceptibility and permeability, Hysteresis, Alpha-particle scattering experiment;
Electromagnets and permanent magnets. Rutherford's model of atom; Bohr model, energy
levels, hydrogen spectrum.
UNIT 14 Electromagnetic Induction
Composition and size of nucleus, atomic masses,
and Alternating Currents isotopes, isobars; isotones. Radioactivity-alpha,
Electromagnetic induction; Faraday's law, induced emf beta and gamma particles/rays and their
and current; Lenz's Law, Eddy currents. Self and mutual properties; radioactive decay law. Mass-energy
inductance. relation, mass defect; binding energy per nucleon
Alternating currents, peak and rms value of alternating and its variation with mass number, nuclear
current/ voltage; reactance and impedance; LCR series fission and fusion.
circuit, resonance; Quality factor, power in AC circuits,
wattless current. AC generator and transformer. UNIT 19 Electronic Devices
Semiconductors; semiconductor diode: I-V
UNIT 15 Electromagnetic Waves characteristics in forward and reverse bias; diode
Electromagnetic waves and their characteristics. as a rectifier; I-V characteristics of LED,
Transverse nature of electromagnetic waves. photodiode, solar cell, and Zener diode; Zener
Electromagnetic spectrum (radio waves, microwaves, diode as a voltage regulator. Junction transistor,
infrared, visible, ultraviolet, X-rays, gamma rays). transistor action, characteristics of a transistor
Applications of e.m. waves. transistor as an amplifier (common emitter
configuration) and oscillator. Logic gates (OR,
UNIT 16 Optics AND, NOT, NAND & NOR). Transistor as a switch.
Reflection and refraction of light at plane and spherical
UNIT 20 Communication Systems
surfaces, mirror formula, Total internal reflection and
its applications, Deviation and Dispersion of light by a Propagation of electromagnetic waves in the
prism, Lens Formula, Magnification, Power of a Lens, atmosphere; Sky and space wave propagation,
Combination of thin lenses in contact, Microscope and Need for modulation, Amplitude and Frequency
Astronomical Telescope (reflecting and refracting) and Modulation, Bandwidth of signals, Bandwidth of
their magnifying powers. Transmission medium, Basic Elements of a
Communication System (Block Diagram only)
Wave optics wave front and Huygens' principle, Laws of
reflection and refraction using Huygen's principle.
SECTION- B
UNIT 21 Experimental Skills 13. Potentiometer
Familiarity with the basic approach and (i) Comparison of emf of two primary cells.
observations of the experiments and activities (ii) Internal resistance of a cell.
1. Vernier callipers - its use to measure internal 14. Resistance and figure of merit of a
and external diameter and depth of a vessel. galvanometer by half deflection method.
2. Screw gauge - its use to determine thickness/ 15. Focal length of
diameter of thin sheet/wire.
(i) Convex mirror
3. Simple Pendulum - dissipation of energy by (ii) Concave mirror
plotting a graph between square of amplitude (iii) Convex lens
and time.
16. Using parallax method. Plot of angle of deviation
4. Metre Scale - mass of a given object by vs angle of incidence for a triangular prism.
principle of moments.
17. Refractive index of a glass slab using a travelling
5. Young's modulus of elasticity of the material of microscope.
a metallic wire.
18. Characteristic curves of a p-n junction diode in
6. Surface tension of water by capillary rise and forward and reverse bias.
effect of detergents.
19. Characteristic curves of a Zener diode and finding
7. Coefficient of Viscosity of a given viscous liquid reverse break down voltage.
by measuring terminal velocity of a given
spherical body. 20. Characteristic curves of a transistor and finding
current gain and voltage gain.
8. Plotting a cooling curve for the relationship
21. Identification of Diode, LED, Transistor, IC, Resistor,
between the temperature of a hot body and
Capacitor from mixed collection of such items.
time.
22. Using multimeter to
9. Speed of sound in air at room temperature
using a resonance tube. (i) Identify base of a transistor.
(ii) Distinguish between npn and pnp type
10. Specific heat capacity of a given (i) solid and (ii) transistor.
liquid by method of mixtures.
(iii) See the unidirectional flow of current in case of
11. Resistivity of the material of a given wire using a diode and an LED.
metre bridge. (iv) Check the correctness or otherwise of a given
12. Resistance of a given wire using Ohm's law. electronic component (diode, transistor or IC).
HOW THIS BOOK IS
USEFUL FOR YOU ?
As the name suggest, this is the perfect book for your recapitulation of the whole syllabus, as it
provides you a capsule course on the subject covering the syllabi of JEE Main, with the smartest
possible tactics as outlined below:

1. REVISION PLAN
The book provides you with a practical and sound revision plan.
The chapters of the book have been designed day-wise to guide the students in a planned
manner through day-by-day, during those precious 35-40 days. Every day you complete a
chapter/a topic, also take an exercise on the chapter. So that you can check & correct your
mistakes, answers with hints & solutions also have been provided. By 37th day from the date
you start using this book, entire syllabus gets revisited.
Again, as per your convenience/preparation strategy, you can also divide the available 30-35
days into two time frames, first time slot of 3 weeks and last slot of 1 & 1/2 week. Utilize first
time slot for studies and last one for revising the formulas and important points. Now fill the
time slots with subjects/topics and set key milestones. Keep all the formulas, key points on a
couple of A4 size sheets as ready-reckner on your table and go over them time and again. If you
are done with notes, prepare more detailed inside notes and go over them once again. Study
all the 3 subjects every day. Concentrate on the topics that have more weightage in the exam
that you are targeting.

2. MOCK TESTS
Once you finish your revision on 37th day, the book provides you with full length mock tests
for day 38th, 39th, & 40th, thereby ensures your total & full proof preparation for the final
show.
The importance of solving previous years' papers and 10-15 mock tests cannot be
overemphasized. Identify your weaknesses and strengths. Work towards your strengths i.e.,
devote more time to your strengths to be 100% sure and confident. In the last time frame of 1
& 1/2 week, don't take-up anything new, just revise what you have studied before. Be exam-
ready with quality mock tests in between to implement your winning strategy.

3. FOCUS TOPICS
Based on past years question paper trends, there are few topics in each subject which have
more questions in exam than other. So far Physics is concerned it may be summed up as
below:
Electricity, Magnetism, Modern Physics, Mechanics, Radioactivity, Wave Options and Heat
Transfer. More than 80% of questions are normally asked from these topics.
However, be prepared to find a completely changed pattern for the exam than noted above as
examiners keep trying to weed out 'learn by rot practice'. One should not panic by witnessing a
new pattern , rather should be tension free as no one will have any upper hand in the exam.
4. IMPROVES STRIKE RATE AND ACCURACY
The book even helps to improve your strike rate & accuracy. When solving practice tests or
mock tests, try to analyze where you are making mistakes-where are you wasting your time;
which section you are doing best. Whatever mistakes you make in the first mock test, try to
improve that in second. In this way, you can make the optimum use of the book for giving
perfection to your preparation.
What most students do is that they revise whole of the syllabus but never attempt a mock and
thus they always make mistake in main exam and lose the track.

5. LOG OF LESSONS
During your preparations, make a log of Lesson's Learnt. It is specific to each individual as to
where the person is being most efficient and least efficient. Three things are important - what
is working, what's not working and how would you like to do in your next mock test.

6. TIME MANAGEMENT
Most candidates who don't make it to good medical colleges are not good in one area- Time
Management. And, probably here lies the most important value addition that's the book
provides in an aspirant's preparation. Once the students go through the content of the book
precisely as given/directed, he/she learns the tactics of time management in the exam.
Realization and strengthening of what you are good at is very helpful, rather than what one
doesn't know. Your greatest motto in the exam should be, how to maximize your scoring with
the given level of preparation. You have to get about 200 plus marks out of a total of about 400
marks for admission to a good NIT (though for a good branch one needs to do much better
than that). Remember that one would be doomed if s/he tries to score 400 in about 3 hours.

7. ART OF PROBLEM SOLVING


The book also let you to master the art of problem solving. The key to problem solving does
not lie in understanding the solution to the problem but to find out what clues in the problem
leads you to the right solution. And, that's the reason Hints & Solutions are provided with the
exercises after each chapter of the book. Try to find out the reason by analyzing the level of
problem & practice similar kind of problems so that you can master the tricks involved.
Remember that directly going though the solutions is not going to help you at all.

8. POSITIVE PERCEPTION
The book put forth for its readers a 'Simple and Straightforward' concept of studies, which is
the best possible, time-tested perception for 11th hour revision / preparation.
The content of the book has been presented in such a lucid way so that you can enjoy what
you are reading, keeping a note of your already stressed mind & time span.
Cracking JEE Main is not a matter of life and death. Do not allow panic and pressure to create
confusion. Do some yoga and prayers. Enjoy this time with studies as it will never come back.
EXAM BITES

This Pdf Is
Downloaded From
www.exambites.in

Visit www.exambites.in for


More Premium Stuffs,Latest
Books,Test Papers,Lectures etc.
jeeneetadda
jeeneetadda_official
jeeneetadda

VISIT NOW !!
DAY ONE

Units and
Measurement
Learning & Revision for the Day

u Physics u Accuracy and Precision u Dimensions of Physical


u Units u Errors in Measurement Quantities
u Significant Figures

Physics
Physics is the study of matter and its motion, as well as space and time using concepts
such as energy, force, mass and charge. It is an experimental science, creating theories
that are tested against observation.

Scope and Excitement


Scope of Physics is very vast, as it deals with a wide variety of disciplines such as
mechanics, heat, light, etc.
It also deals with very large magnitude of astronomical phenomenon as well as very
small magnitude involving electrons, protons, etc.

Nature of Physical Laws


Physics is the study of nature and natural phenomena. All observations and experiments
in physics lead to certain facts. These facts can be explained on the basis of certain laws. PREP
MIRROR
Physics, Technology and Society Your Personal Preparation Indicator

Connection between physics, technology and society can be seen in many examples like u No. of Questions in Exercises (x)—
working of heat engines gave rise to thermodynamics. Wireless communication u No. of Questions Attempted (y)—
technology arose from basic laws of electricity and magnetism. Lately discovery of u No. of Correct Questions (z)—
silicon chip triggered the computer revolution. (Without referring Explanations)

u Accuracy Level (z / y × 100)—


Units u Prep Level (z / x × 100)—
Measurement of any physical quantity involves comparison with a certain basic, widely
accepted reference standard called unit. In order to expect good rank in JEE,
your Accuracy Level should be above
85 & Prep Level should be above 75.
02 40 DAYS ~ JEE MAIN PHYSICS DAY ONE

Basic Units
Fundamental and Derived Units Base
Name and
Quantity Definintion
Fundamental units are the units which can neither be Symbol
derived from one another, nor they can be further resolved Thermodyn Kelvin (K) 1
The kelvin is th fraction of
into more simpler units. -amic 273.16
These are the units of fundamental quantity. However, temperature the thermodynamic temperature of
derived units are the units of measurement of all physical the triple point of water.
quantities which can be obtained from fundamental units. Amount of mole (mol) The mole is the amount of substance
substance of a system, which contains as many
System of Units elementary entities as there are
atoms in 0.012 kg of carbon-12.
A complete set of these units, both fundamental and derived Luminous candela (cd) The candela is the luminous
unit is known as the system of units. The common systems intensity intensity in a given direction of a
are given below: source emitting monochromatic
1. CGS System (Centimetre, Gram, Second) are often used radiation of frequency 540 × 1012 Hz
in scientific work. This system measures, Length in and having a radiant intensity of
centimetre (cm), Mass in gram (g), Time in second (s). 1
W sr −1 in that direction.
2. FPS System (Foot, Pound, Second) It is also called the 683
British Unit System. This unit measures, Length in foot Supplementary Units
(foot), Mass in gram (pound), Time in second (s). Supplementary Name and
Quantity Symbol Definition
3. MKS System In this system also length, mass and time
have been taken as fundamental quantities and Plane angle radian It is angle subtended at the centre by
corresponding fundamental units are metre, kilogram (rad) an arc of a circle having a length
and second. equal to the radius of the circle.
4. International System (SI) of Units It is an extended Solid angle steradian It is the solid angle which is having
version of the MKS (Metre, Kilogram, Second) system. It (sr) its vertex at the centre of the sphere,
has seven base units and two supplementary units. it cuts-off an area of the surface of
Seven base quantities and two supplementary sphere equal to that of a square with
quantities, their units along with definitions are the length of each side equal to the
tabulated below. radius of the sphere.

Basic Units NOTE


Base • Angle subtended by a closed curve at an inside points
Name and is 2π rad.
Quantity Definintion
Symbol • Solid angle subtended by a closed surface at an inside
Length metre (m) The metre is the length of path point is 4 π steradian.
travelled by light in vacuum during
a time interval of 1/299,792,458 part
of a second.
Significant Figures
Mass kilogram (kg) It is the mass of the international In the measured value of a physical quantity, the digits about
prototype of the kilogram (a the correction of which we are sure, plus the last digits which
platinum iridium alloy cylinder) is doubtful, are called the significant figures.
kept at International Bureau of Larger the number of significant figures obtained in a
Weights and Measures, at Sevres measurement, greater is the accuracy of the measurement.
(France).
Time second (s) The second is the duration of Rules for Counting Significant Figures
9,192, 631,770 periods of the l
All the non-zero digits are significant. In 2.738 the number
radiation corresponding to the
of significant figures is 4.
transition between the two hyperfine
levels of the ground state of cesium-
l
All the zeros between two non-zero digits are significant,
133 atom. no matter where the decimal point is. As examples 209 and
Electric Ampere (A) The ampere is that constant current,
3.002 have 3 and 4 significant figures, respectively.
current which if maintained in two straight, l
If the measured number is less than 1, the zero (s) on the
parallel conductors of infinite length right of decimal point and to the left of the first non-zero
placed 1 m apart in vacuum would digit are non-significant. In 0.00807, first three underlined
produce a force equal to 2 × 10−7 zeros are non-significant and the number of significant
Nm −1 on either conductor. figures is only 3.
DAY ONE UNITS AND MEASUREMENT 03

l
The terminal or trailing zero (s) in a number without a (i) Least count of vernier callipers
decimal point are not significant. Thus, 12.3 m = 1230 cm Value of 1 main scale division
= 12300 mm has only 3 significant figures. LC =
Total number of vernier scale division
l
The trailing zero(s) in number with a decimal point are
significant. Thus, 3.800 kg has 4 significant figures. (ii) Least count of screw gauge
Value of 1 pitch scale reading
l
A choice of change of units does not change the number LC =
of significant digits or figures in a measurement. Total number of head scale division
l
To remove ambiguities in determining number of
significant figures, a measurement is usually expressed Errors in Measurement
as ‘a × 10b ’, where 1 ≤ a ≤ 10 and b is the order of
magnitude. The difference in the true value (mean value) and measured
value of a quantity is called error of measurement. Different
types of error are given below:
Rules for Arithmetic Operations (i) Absolute error,
with Significant Figures a + a2 + a3 + ... an 1 i = n
amean = a0 = 1 = ∑ ai
n n i= 1
l
In addition or subtraction, the final results should retain as
many decimal places as there are in the number with the ∆a1 = mean value − observed value
least decimal place. As an example sum of 423.5 g, 164.92 g ∆a1 = a0 − a1
and 24.381 g is 612.801 g, but it should be expressed as ∆a2 = a0 − a2
612.8 g only because the least precise measurement (423.5 :. :. :.
g) is correct to only one decimal place. ∆an = a0 − an
l
In multiplication or division, the final result should
(ii) Mean absolute error, n
retain as many significant figures as there are in the
original number with the least significant figures. ∑| ∆ai|
[| ∆a1| + | ∆a2| + | ∆a3| + .... + | ∆an|] i= 1
For example Suppose an expression is performed like ∆amean = =
n n
. × 1243) / (44 . 65) = 676 . 481522
(243 ∆amean
(iii) Relative or fractional error =
Rounding the above result upto three significant figures amean
result would become 676.
(v) Percentage error,
∆amean
δ a = Relative error × 100 % = × 100%
Rules for Rounding off the amean
Uncertain Digits
l
The preceding digit is raised by 1 if the insignificant Combination of Errors
digit to be dropped is more than 5 and is left unchanged (i) If X = A + B, then (∆X ) = ± (∆A + ∆B)
if the latter is less than 5. e.g. 18.764 will be rounded off
to 18.8 and 18.74 to 18.7.  ∆X   ∆A ∆B ∆C 
(ii) If X = ABC, then   =± + +
 X  max  A B C 
l
If the insignificant figure is 5 and the preceding digit is
even, then the insignificant digit is simply dropped.  ∆X   ∆A ∆B ∆C 
(iii) If X = A k B l C n , then   = ± k +l +n
However, if the preceding digit is odd, then it is raised  X   A B C 
by one so as to make it even. e.g. 17.845 will be rounded
off to 17.84 and 17.875 to 17.88.
Dimensions of Physical Quantities
Accuracy and Precision The dimensions of a physical quantity are the powers to which
the fundamental (base) quantities are raised, to represent that
The accuracy of a measurement is a measure of how close
quantity.
the measured value is to the true value of the quantity.
However, precision tells us to what resolution or limit, the Consider the physical quantity force.
quantity is measured by a measuring instrument. ‘Force = mass × acceleration = mass × length × (time)−2 ’
Thus, the dimension of force are 1 in mass [M]
Least Count 1 in length [L] and −2 in time [ T−2 ], that is [MLT−2 ].
The least count of a measuring instrument is the least l
Dimensions of a physical quantity do not depend on its
value, that can be measured using the instrument. It is
magnitude or the units in which it is measured.
denoted as LC.
04 40 DAYS ~ JEE MAIN PHYSICS DAY ONE

Principle of Homogeneity of Physical Quantity SI Unit


Dimensional
Formula
Dimensions and Applications Moment of inertia kg m2 [ ML2 ]
According to this principle, a correct dimensional equation
must be homogeneous, i.e. dimensions of all the terms in a Acceleration, acceleration due ms −2 [LT −2 ]
physical expression must be same. to gravity

LHS (dimension) = RHS (dimension) Force, thrust, tension, weight Newton (N) [MLT −2 ]

Linear momentum, impulse kg ms −1 or Ns [MLT −1 ]


Uses of Dimensions Work, energy, KE, PE, thermal Joule (J) [ML2 T −2 ]
(i) To check the correctness of a given physical equation. energy, internal energy, etc.
(ii) Derivation of formula. Surface area, area of m2 [L2 ]
(iii) Dimensional formula is useful to convert the value of a cross-section
physical quantity from one system to the other. Physical Electric conductivity Sm −1 [M −1 L−3 T3 A2 ]
quantity is expressed as a product of numerical value
and unit. In any system of measurement, this product Young’s modulus, Pa [ML−1 T −2 ]
remains constant. Bulk modulus

Let dimensional formula of a given physical quantity be Compressibility m2 N −1 [M −1 LT2 ]


[Ma Lb T c ].If in a system having base units [M1L1T1] the Magnetic Flux Wb [ML2 T −2 A −1 ]
numerical value of given quantity be n1 and numerical
value n2 in another unit system having the base units Magnetic Flux density (σ ) Wb / m2 [MT −2 A −1 ]
[ M2 , L2 , T2 ], then Intensity of a wave Wm −2 [MT −3 ]
Q = n1u1 = n2u2 Photon flux density m −2 s −1 [L−2 T −1 ]
n1[M 1a L b1 T c1] = n2 [Mb2 L b2 T c2]
Luminous energy Lm s [ML2 T −2 ]
a b c
M  L  T  Luminance Lux [MT −3 ]
n2 = n 1  1   1   1 
 M2   L2   T2 
Specific heat capacity Jkg −1 K −1 [L2 T −2 K −1 ]
Dimensions of Important Physical Quantities
Latent heat of vaporisation Jkg −1 [L2 T −2 ]
Dimensional
Physical Quantity SI Unit Coefficient of Thermal Wm −1 K −1 [MLT −3 K −1 ]
Formula
conductivity
Power Watt (W) [ML2 T −3 ]
Electric voltage JC −1 [ML2 T −3 A −1 ]
Pressure, stress, coefficient of Pascal (Pa) [ML−1 T −2 ]
elasticity (ρ, σ, η) or Nm −2 Magnetisation Am −1 [L−1 A]
Frequency, angular frequency Hz or s −1 [T −1 ] Magnetic induction T [MT −2 A −1 ]
2 −1
Angular momentum kg m s [ML2 T −1] Planck’s constant J-s [ML2 T −1 ]
2 −2
Torque Nm [ML T ] Radioactive decay constant Bq [T −1 ]
Gravitational constant (G) N m2 kg −2 [M −1 L3 T −2 ] Binding energy MeV [ML2 T −2 ]
DAY ONE UNITS AND MEASUREMENT05

DAY PRACTICE SESSION 1

FOUNDATION QUESTIONS EXERCISE


1 In which of the following systems of units, a Weber is the 9 A man runs 100.5 m in 10.3 s. Find his average speed up
unit of magnetic flux? to appropriate significant figures.
(a) CGS (b) MKS (c) SI (d) FPS (a) 9.71 ms −1 (b) 9.708 ms −1
(c) 9.7087 ms −1 (d) 9.70874 ms −1
2 In an experiment, the angles are required to be
measured using an instrument. 29 divisions of the main 10 If the length of rod A is 3. 25 ± 0.01 cm and that of B is
scale exactly coincide with the 30 divisions of the vernier . ± 0.01 cm, then the rod B is longer than rod A by
419
scale. If the smallest division of the main scale is (a) (0.94 ± 0.00) cm (b) (0.94 ± 0.01) cm
half-a-degree (0.5°), then the least count of the (c) (0.94 ± 0.02) cm (d) (0.094 ± 0.005) cm
instrument is j AIEEE 2009
11 You measure two quantities as A = 1. 0 m ± 0.2 m,
(a) one minute (b) half minute B = 2.0 m ± 0.2 m. We should report correct value for
(c) one degree (d) half degree AB as
3 A student measured the length of a rod and wrote it as (a) 1.4 m ± 0.4 m (b) 1.41m ± 0.15 m
3.50 cm.Which instrument did he use to measure it? (c) 1.4 m ± 0.3 m (d) 1.4 m ± 0.2 m
(a)A meter scale j JEE Main 2014
12 A student measured the length of the pendulum 1.21 m
(b) A vernier calliper where the 10 divisions in vernier scale using a metre scale and time for 25 vibrations as 2 min
matches with 9 divisions in main scale and main scale 20 sec using his wrist watch, absolute error in g is
has 10 divisions in 1 cm (a) 0.11 ms − 2 (b) 0.88 ms − 2
(c) A screw gauge having 100 divisions in the circular scale (c) 0.44 ms − 2 (d) 0. 22 ms − 2
and pitch as 1 mm
13 The absolute error in density of a sphere of radius
(d) A screw gauge having 50 divisions in the circular scale
10.01 cm and mass 4.692 kg is
and pitch as 1 mm
(a) 3.59 kgm − 3 (b) 4.692 kgm − 3
4 N division on main scale of a vernier callipers coincide (c) 0 (d) 1.12 kgm − 3
with (N + 1) division of the vernier scale if each division on
14 A sphere has a mass of 12.2 kg ± 0.1 kg and radius
main scale is of “a” units, least count of instrument is
10 cm ± 0.1 cm, the maximum % error in density is
N+ 1 a N −1 a
(a) (b) (c) (d) (a) 10% (b) 2.4% (c) 3.83% (d) 4.2%
a N+ 1 a N −1
15 If error in measurement of radius of sphere is 1%, what
5 One 8 centimetre on the main scale of a vernier calliper is
will be the error in measurement of volume?
divided into 10 equal parts. If 10 of the divisions of the
1
vernier coincide with small divisions on the main scale, (a) 1% (b) % (c) 3% (d) 10%
3
the least count of the callipers is
16 What is the percentage error in the measurement of the
(a) 0.005 cm (b) 0.02 cm
(c) 0.01 cm (d) 0.05 cm time period T of a pendulum, if the maximum errors in the
measurements of l and g are 2% and 4%, respectively?
6 The respective number of significant figures for the
(a) 6% (b) 4% (c) 3% (d) 5%
numbers 23.023, 0.0003 and 2.1 × 10−3 are
(a) 5, 1, 2 (b) 5, 1, 5 (c) 5, 5, 2 (d) 4, 4, 2
17 The density of a material in the shape of a cube is
determined by measuring three sides of the cube and its
7 A bee of mass 0.000087 kg sits on a flower of mass mass. If the relative errors in measuring the mass and
0.0123 kg. What is the total mass supported by the stem
length are respectively 1.5% and 1% , the maximum error
of the flower up to appropriate significant figures?
in determining the density is j
JEE Main 2018
(a) 0.012387 kg (b) 0.01239 kg
(a) 2.5% (b) 3.5%
(c) 0.0124 kg (d) 0.012 kg
(c) 4.5% (d) 6%
8 The radius of a uniform wire is r = 0.021 cm. The value of π
18 Which one of the following represents the correct
is given to be 3.142. What is the area of cross-section of
dimensions of the coefficient of viscosity?
the wire up to appropriate significant figures?
2 2 (a) [ML−1T−2 ] (b) [MLT−1 ]
(a) 0.0014 cm (b) 0.00139 cm
(c) 0.001386 cm2 (d) 0.0013856 cm2 (c) [ML−1T−1 ] (d) [ML−2 T−2 ]
06 40 DAYS ~ JEE MAIN PHYSICS DAY ONE

19 Which of the following sets share different dimensions? a a − t2


29 The dimensions of in the equation p = where, p
(a) Pressure, Young’s modulus, stress b bx
(b) Emf, potential difference, electric potential is pressure, x is distance and t is time, are
(c) Heat, work done, energy (a) [M2LT−3 ] (b) [MT−2 ] (c) [ML3 T−2 ] (d) [LT−3 ]
(d) Dipole moment, electric flux, electric field
30 The velocity of a particle is given as v = a + b t + c t 2. If
20 Out of the following pairs, which one does not have the velocity is measured in ms −1, then units of a and c are
identical dimensions?
(a) ms −1 and ms−3 (b) ms−2 and ms
(a) Angular momentum and Planck’s constant (c) m2 s and ms2
–1
(d) ms and ms−1
(b) Impulse and momentum
(c) Moment of inertia and moment of a force 31 In the following dimensionally consistent equation, we
(d) Work and torque X
have, F = + Y , where F = force.
2 Linear density
e
21 The dimensions of , where e, ε 0, h and c are the
4πε 0hc The dimensional formula for X and Y are
electronic charge, electric permittivity, Planck’s constant (a) [M2L0 T−2 ]; [MLT−2 ] (b) [M2L−2 T−2 ]; [MLT−2 ]
and velocity of light in vacuum respectively, are (c) [MLT−2 ]; [ML2 T−2 ] (d) [M0L0 T0 ]; [ML0 T0 ]
0 0 0 0 0 0 0 0 0
(a) [M L T ] (b) [ML T ] (c) [M LT ] (d) [M L T] 32 The dimensions of self-inductance are
22 In the relation X = 3YZ , X and Z represent the
2
(a) [ML−2 T−2 A −2 ] (b) [ML2 T−2 A −2 ]
dimensions of capacitance and magnetic induction (c) [ML2 T−2 A −1 ] (d) [ML−2 T−2 A −1 ]
respectively, dimensions ofY are 33 Match List I with List II and select the correct answer
(a) [MT −1Q−1] (b) [M−3 T 4L−2Q4 ] using the codes given below the lists. j 2013 Main
(c) [M−3 T −1L−1Q4 ] (d) [ML2 T −2 A −2 ]
23 The dimensions of magnetic field in M,L,T and C Column I Column II
(coulomb) is given as A. Boltzmann constant p. [ML2T −1 ]
(a) [MLT−1C −1 ] (b) [MT2C −2 ] (c) [MT−1C −1 ] (d) [MT−2C −1]
B. Coefficient of viscosity q. [ML−1T −1 ]
1
24 Dimensions of , where symbols have their usual
µ 0ε 0 C. Planck constant r. [MLT −3K −1 ]
meaning, are
D. Thermal conductivity s. [ML2T −2K −1 ]
(a) [L−1T] (b) [L2 T2 ] (c) [L2 T−2 ] (d) [LT−1 ]
25 If the acceleration due to gravity is 10 ms –2 and units of Codes
length and time are changed to kilometre and hours A B C D A B C D
respectively, the numerical value of acceleration is (a) p q r s (b) s q p r
(a) 360000 (b) 72000 (c) 36000 (d) 129600 (c) s r p q (d) r s q p
26 If E = energy, G = gravitational constant, I = impulse and
GIM 2 Direction (Q. Nos. 34-37) Each of these questions contains
M = mass, then dimensions of are same as that of two statements : Statement I (Assertion) and Statement II
E2
(Reason). Each of these questions also has four alternative
(a) time (b) mass (c) length (d) force
choices, only one of which is the correct answer. You have to
27 Let [ ε 0 ] denotes the dimensional formula of the select one of the codes (a), (b), (c), (d) given below
permittivity of vacuum. If M = mass, L = length, T = time (a) Statement I is true, Statement II is true; Statement II is
and A = electric current, then j
JEE Main 2013 the correct explanation for Statement I
(a) [ε0 ] = [ M −1 L−3 T 2 A] (b) [ε0 ] = [ M −1 L−3 T 4 A 2 ] (b) Statement I is true, Statement II is true; Statement II is
(c) [ε0 ] = [ M −2 L2 T −1 A −2 ] (d) [ε0 ] = [ M −1 L2 T −1 A 2 ] not the correct explanation for Statement I
28 With the usual notations, the following equation (c) Statement I is true; Statement II is false
1 (d) Statement I is false; Statement II is true
s = u + a ( 2 t − 1) is
2 34 Statement I The order of accuracy of measurement
(a) only numerically correct depends on the least count of the measuring instrument.
(b) only dimensionally correct Statement II The smaller the least count, the greater is
(c) Both numerically and dimensionally correct the number of significant figures in the measured
(d) Neither numerically nor dimensionally correct
value.
DAY ONE UNITS AND MEASUREMENT 07

35 Statement I The dimensional method cannot be used to Statement II The same mass when expressed in
obtain the dependence of the work done by a force F on grams as 13200 g has five significant figures.
the angle θ between force F and displacement x.
37 Statement I Method of dimensions cannot be used for
Statement II Angle can be measured in radians but it
has no dimensions. deriving formula containing trigonometrical ratios.

36 Statement I The mass of an object is 13.2 kg in the Statement II This is because trigonometrical ratios
measurement there are 3 significant figures. have no dimensions.

DAY PRACTICE SESSION 2

PROGRESSIVE QUESTIONS EXERCISE


1 The dimensions of angular momentum, latent heat and 7 The length and breadth of a rectangular sheet are 16.2
capacitance are, respectively. j JEE Main (Online) 2013 cm and 10 .1 cm , respectively. The area of the sheet in
2 1 2 2 −2 −1 −2 2 appropriate significant figures and error is
(a) [ML T A ], [L T ], [M L T ]
(b) [ML2 T−2 ], [L2 T2 ], [M−1 L−2 T4 A 2 ] (a) 164 ± 3 cm2 (b) 163.62 ± 2.6 cm2
(c) [ML2 T−1 ], [L2 T−2 ], [ML2 TA 2 ] (c) 163.6 ± 2.6 cm2 (d) 163.62 ± 3 cm2
(d) [ML2 T−1 ], [L2 T−2 ], [M−1L−2 T4 A 2 ] 8 Resistance of a given wire is obtained by measuring the
2 The speed (v ) of ripples on the surface of water depends current flowing in it and the voltage difference applied
on surface tension ( σ ), density (ρ) and wavelength (λ ). across it. If the percentage errors in the measurement of
The square of speed (v ) is proportional to the current and the voltage difference are 3% each, then
σ ρ λ error in the value of resistance of the wire is
(a) (b) (c) (d) ρλσ
ρλ σλ σρ (a) 6% (b) zero (c) 1% (d) 3%

3 Dimensions of resistance in an electrical circuit, in terms 9 A screw gauge gives the following reading when used to
of dimensions of mass M, length L, time T and current I, measure the diameter of a wire.
would be Main scale reading : 0 mm
Circular scale reading : 52 divisions
(a) [ML2 T−3I−1 ] (b) [ML2 T−2 ]
Given that 1 mm on main scale corresponds to
(c) [ML2 T−1I−1 ] (d) [ML2 T−3I−2 ]
100 divisions of the circular scale.
αz

α kθ The diameter of wire from the above data is
4 In the relation p = e , p is pressure, z is distance, k
β (a) 0.052 cm (b) 0.026 cm
is Boltzmann constant and θ is the temperature. The (c) 0.005 cm (d) 0.52 cm
dimensional formula of β will be 10 A screw gauge with a pitch of 0.5 mm and a circular
(a) [M0L2 T 0 ] (b) [M L2 T] scale with 50 divisions is used to measure the thickness
(c) [ML0 T –1 ] (d) [M0L2 T−1 ] of a thin sheet of aluminium. Before starting the
measurement, it is found that when the two jaws of the
5 The dimensions of σb 4( σ = Stefan’s constant and screw gauge are brought in contact, the 45th division
b = Wien’s constant) are coincides with the main scale line and that the zero of the
(a) [M0L0 T0 ] (b) [ML4 T−3 ] (c) [ML−2 T] (d) [ML6 T−3 ] main scale is barely visible. What is the thickness of the
sheet, if the main scale reading is 0.5 mm and the 25th
6 If Planck’s constant (h) and speed of light in vacuum (c) division coincides with the main scale line?
are taken as two fundamental quantities, which one of the (a) 0.75 mm (b) 0.80 mm
following can, in addition,be taken to express length, (c) 0.70 mm (d) 0.50 mm
mass and time in terms of the three chosen fundamental
11 The following observations were taken for determining
quantities?
surface tensionT of water by capillary method. Diameter
(i) Mass of electron (m e ) of capillary, d = 1.25 × 10−2 m rise of water,
(ii) Universal gravitational constant (G) h = 1.45 × 10−2m. Using g = 9.80 m/s 2 and the simplified
(iii) Charge of electron ( e) dhg
relation T = × 103N/m, the possible error in surface
(iv) Mass of proton (m p ) 4
(a) (i),(ii) and (iii) (b) (i) and (iii) tension is closest to j
JEE Main 2017 (Offline)
(c) (i), (ii) and (iv) (d) (i) only (a) 1.5% (b) 2.4% (c) 10% (d) 0.15%
08 40 DAYS ~ JEE MAIN PHYSICS DAY ONE

12 A student measures the time period of 100 oscillations of Direction (Q. Nos. 15-16) Each of these questions contains
a simple pendulum four times. The data set is 90s, 91s, two statements : Statement I (Assertion) and Statement II
92s and 95s. If the minimum division in the measuring (Reason). Each of these questions also has four alternative
clock is 1s, then the reported mean time should be choices, only one of which is the correct answer. You have to
j JEE Main 2016 (Offline) select one of the codes (a), (b), (c), (d) given below
(a) Statement I is true, Statement II is true; Statement II is
(a) (92 ± 2) s (b) (92 ± 5 ) s (c) (92 ± 18
. ) s (d) (92 ± 3) s
the correct explanation for Statement I
13 The period of oscillation of a simple pendulum is (b) Statement I is true, Statement II is true; Statement II is
T = 2π L / g . Measured value of L is 20.0 cm known to not the correct explanation for Statement I
(c) Statement I is true; Statement II is false
1mm accuracy and time for 100 oscillations of the
(d) Statement I is false; Statement II is true
pendulum is found to be 90 s using a wrist watch of
resolution.The accuracy in the determination of g is 15 Statement I The value of velocity of light is 3 × 108 ms −1
j JEE Main 2015 and acceleration due to gravity is
(a) 2% (b) 3% (c) 1% (d) 5% 10 ms −2 and the mass of proton is 1.67 × 10−27 kg.
14 The current voltage relation of diode is given by Statement II The value of time in such a system is
I = (e − 1) mA, where the applied voltage V is in volt
1000V /T 3 × 107 s.
and the temperature T is in kelvin. If a student makes an 16 Statement I The distance covered by a body is given by
error measuring ± 0.01V while measuring the current of 1a
s =u + , where the symbols have usual meaning.
5 mA at 300K, what will be the error in the value of 2 t
current in mA? j JEE Main 2013
Statement II We can add, substract or equate
(a) 0.2 mA (b) 0.02 mA (c) 0.5 mA (d) 0.05 mA quantities which have same dimensions.

ANSWERS
SESSION 1 1 (c) 2 (a) 3 (b) 4 (b) 5 (b) 6 (a) 7 (d) 8 (a) 9 (a) 10 (c)
11 (d) 12 (d) 13 (a) 14 (c) 15 (c) 16 (c) 17 (c) 18 (c) 19 (d) 20 (c)
21 (a) 22 (b) 23 (c) 24 (c) 25 (d) 26 (a) 27 (b) 28 (d) 29 (b) 30 (a)
31 (a) 32 (b) 33 (b) 34 (b) 35 (a) 36 (c) 37 (a)

SESSION 2 1 (d) 2 (a) 3 (d) 4 (a) 5 (b) 6 (a) 7 (a) 8 (a) 9 (a) 10 (b)
11 (a) 12 (a) 13 (b) 14 (a) 15 (b) 16 (d)
DAY ONE UNITS AND MEASUREMENT 9

Hints and Explanations


SESSION 1 9 Average speed = 100.5 m 14 ∴ Density, ρ = M = M
,
10.3 s 4 3 V
1 A weber is the unit of magnetic flux in πr
SI system. = 975728
. ms −1 3
dρ ∆M 3∆r 
× 100 =  +  × 100
2 Least count The distance has four significant figures ρ  M r 
Value of main scale division but the time has only three. Hence, the
= 
= 01
. . 
01
result must be rounded off to three + 3×  × 100
Number of divisions on  12.2 10 
vernier scale significant figure
= 3 . 83 %
to 9.71 ms −1 .
1 1 1° 1°
=
30
MSD =
30
×
2
=
60
= 1 min
10 As, A = 3.25 ± 0.01 cm 15 As, V = 4 πr 3
3
3 If student measures 3.50cm, it means and B = 4.19 ± 0.01 cm ∆V ∆r
Hence, × 100 = 3 × 100
that there is an uncertainly of
1
cm. ∴ Y =B − A V r
100 = 4.19 − 3.25 = 0.94 cm = 3 × 1% = 3%

For vernier scale with 1 MSD = 1 mm and ∆Y = ∆B + ∆A 16 Since, the time period,
and 9 MSD = 10 VSD = 0.01 cm + 0.01 cm = 0.02 cm 1 l
T =
LC of vernier calliper = 1 MSD – 1VSD 2π g
∴ Y = (0.94 ± 0.02) cm
=  1 −
1 9 1 Thus, for calculating the error, we get
 = cm 11 Here, A = 1.0 m ± 0.2 m,
10  10  100 ∆T  1 ∆l 1 ∆g 
B = 2.0 m ± 0.2 m =±  + 
4 (N + 1) VSD = N MSD x = AB = (1. 0)(2. 0) = 1.414 m
T 2 l 2 g 

= ±  × 2% + × 4%  = ± 3%
N 1 1
∴ 1 VSD = MSD Rounding off to two significant digits,  2 2 
N +1 x = AB = 1.4 m
Least count
∆x 1  ∆ A ∆ B  17 Q Density, ρ = Mass = M3 or ρ = M3
= (1 MSD − 1VSD) (value of MSD) Now, =  +
B 
Volume L L
x 2 A
 N  ∆ρ ∆M 3∆L
= 1 −
a 1  0.2 0.2  ⇒ Error in density = +
 × a= = + =
0.6
ρ M L
 N + 1 N +1 2  1.0 2 . 0  2 × 2.0
So, maximum % error in measurement
0.6 x
5 1 MSD = 1 cm = 0.1 cm, ∆x= = 0 .15 × 1.414 of ρ is
10 2 × 2.0 ∆ρ ∆M 3∆L
= 0.2121 × 100 = × 100 + × 100
10 VSD = 8 MSD ρ M L
Hence, we get Rounding off to one significant digit,
or % error in density = 1.5 + 3 × 1
8 8 ∆ x = 0.2 m
1 VSD = MSD = × (0.1) = 0.08 cm % error = 4.5%
10 10
Hence, AB = x ± ∆x = (1. 4 ± 0. 2) m F
Thus, the least count = 1 MSD – 18 By Newton’s formula η =
1VSD A ( ∆V / ∆Z )
12 As, T = 2π l or g = 4 π2 l
2

= 01. − 0.08 = 0.02 cm g T ∴ Dimensions of η


Dimensions of force
6 Number of significant figures in  ∆g ∆L 2∆T  =
23.023 = 5 ⇒  = +  (Dimensions of area × Dimensions
 g L T 
Number of significant figures in of velocity gradient)
0.0003 = 1 So, absolute error in g is
[MLT −2 ]
∆L 2∆T  = = [ML−1 T −1 ]
Number of significant figures in ∆g =  +  g [L2 ][T −1 ]
2.1 × 10−3 = 2  L T 
0.01 2 × 1  19 Dipole moment = charge × distance
7 The mass of the bee has 2 significant =  +  × 9.8
 1.21 140  Electric flux = electric field × area
figures in kg, whereas the mass of the
flower has three significant figures. = (0.0227 × 9.8) = 0.22 ms −2 20 I = mr 2
Hence, the sum (0.012387) must be ∴ [I ] = [ML2 ]
M 4.692 × 3
rounded off to the third decimal place. 13 ∴ ρ = =
4 3 4 × 3.14 × (10.01)3 × 10−6 τ moment of force = r × F
Therefore, the correct significant figure πr
is 0.012. 3 ∴ [τ] = [L][MLT −2 ] = [ML2 T −2 ]
ρ = 1.12 × 103 kg - m −3
8 A = πr 2 = 3.142 × (0.021) 2  2 
∆ρ ∆M
= +
3∆r 21  e 
= 0.00138562 cm .2
ρ M r  4πε 0hc 
Now, there are only two significant
0.001 3 × 0.01  [AT]2
∴ ∆ρ =  +  × 1.12 × 10
3 =
figures in 0.021 cm. Hence, the result
 4.692 10.01  [M L T A ] ⋅ [ML2 T −1 ] ⋅ [LT −1 ]
−1 −3 4 2

must be rounded off to two significant


= 3.59 kgm −3 = [M 0L0T 0]
figures as A = 0.0014 cm2 .
10 40 DAYS ~ JEE MAIN PHYSICS DAY ONE

22 X = [C ] = [M −1 L−2 T2Q 2 ],  X  37 It is true that trigonometrical ratios do


31 [F ] =   + [Y ].
Z = [B ] = [MT −1Q −1 ]  Linear density  not have dimensions. Therefore, method
−1 2 −2 2 of finding dimensions cannot be utilized
X [M T L Q ] So, the dimensions of Y are the same as
Y = = = [M −3 T 4 L−2Q 4 ] for deriving formula involving
Z2 [MT −1Q −1 ]2 that of F, i.e. [Y ] = [F ] = [MLT −2 ]
trigonometrical ratio.
Now, [MLT −2 ] = 
X 
23 From the relation F = qvB  ML− 1 
SESSION 2
⇒ [MLT −2 ] = [C][LT −1 ][B ] ⇒ X = [M 2 L0T − 2 ]
⇒ [B ] = [MC −1 T −1 ] 1 Angular momentum = r × P = [LM LT −1 ]
32 The self-inductance L of a coil in which = [ML2 T −1 ]
24 As we know that, formula of velocity is dI Q
1 1 the current varies at a rate and is Latent heat, L = = [L2 T −2 ]
v = ⇒ v2 = = [LT −1 ]2 ∴ dt M
µ 0ε0 µ 0ε0 dI q ( AT )2
given by e = − L , where e is the Capacitance, C = =
1 dt
= [L2 T −2 ] V W
µ 0ε0 electromotive force (emf) induced in the  W
coil.  as, V = 
  
−2  q 
25 n2 = n1  L1   T1  Now, the dimensions of emf are the same
= [A2 T2 M −1 L−2 T +2 ]
 L2   T2  as that of the potential difference,
−2 i.e. [ML2 T −3 I −1 ] = [M −1 L−2 T 4 A2 ]
= 10 
metre   sec 
 km   h  −e 2 Let v ∝ σ aρb λc . Equating dimensions on
Now, L = .
−2 dI / dt both sides
n2 = 10  3  
m sec 
 10 m   3600sec  Hence, the dimensions of L are [M 0 L1 T −1 ] = k [MT −2 ] a [ML−3 ] b [L] c
= 129600 [ L] =
dimensions of e [M 0LT −1 ] = k [M] a +b [L] −3b +c [T] − 2 a
dimensions of I / dimensions of t Equating the powers of M, L, T on both
 2 −1 3 −2 −1 [ML2 T −3 I −1 ]
26  GIM2  = [M L T ] × 2[MLT ] × [M]2 = sides, we get
 E 
−2 2
[ML T ] [I / T] a + b = 0 and −3b + c = 1; − 2a = − 1
= [ML2 T −2 I −2 ] 1 1 1
= [M 0L0T] Solving, we get a = , b = − , c = −
2 2 2
GIM 2 33 (A) U = 1 kT σ
So, dimensions of are same as that ∴ v ∝ σ 1 /2ρ−1 /2 λ−1 /2 ∴ v2 ∝
E2 2 ρλ
of time. ⇒ [ML2 T −2 ] = [k ] K Potential difference
3 Resistance, R =
27 Electrostatic force between two charges, ⇒ [K] = [ML2 T −2 K −1 ] Current
V W
F =
1 q1 q2 dv = =
(B) F = ηA I qI
4 πε0 R2 dx
(Q Potential difference is equal to work
⇒ ε0 =
q1 q2 [MLT −2 ]
⇒ [η] = = [ML−1 T −1 ] done per unit charge)
4 πFR 2 2
[L LT L ] −1 −1
So, dimensions of R
Substituting the units. [Dimensions of work]
(C) E = hν ⇒ [ML2 T2 ] = [h] [T −1 ] =
C2 [AT ]2 [Dimensions of charge]
Hence, ε0 = = ⇒ [h] = [ML2 T −1 ]
N - m2 [MLT −2 ] [L2 ] [Dimensions of current]
= [M −1 L−3 T 4 A2 ] dQ k A∆θ [ML2 T −2 ]
(D) = = = [ML2 T −3 I −2 ]
dt l [IT] [I]
28 s = distance travelled, u = velocity. [ML2 T −3 L]
So, dimensionally it is not a correct ⇒ [k] = = [MLT −3 K −1 ] αz
[L2 K] 4 In the given equation, should be
equation. kθ
dimensionless
a − t2 34 The least count of a measuring device is

29 p = the least distance (resolution/accuracy), ∴ [α] =  
bx
that can be measured using the device.  z 
⇒ pbx = a − t 2
Greater is the number of significant [ML2 T −2 K −1 × K]
⇒ [α] = = [MLT – 2 ]
⇒ [ pbx] = [a] = [T2 ] figures obtained in a measurement, [L]
[T2 ] [T2 ] greater is its precision and for this the α 
or [b] = = least count of the measuring instrument and [ p] =  
−1 −2
[ p][ x] [ML T ][L] β 
should be smaller.
= [M −1 T 4 ]  α  [MLT −2 ]
35 Work done is W = Fx cos θ. Since, θ is ⇒ [ β] =   = –1 –2
= [M 0L2 T 0]
[T2 ]  p  [ML T ]
∴  =
a
= [MT −2 ] dimensionless, the dependence of W
 b  [M −1 T 4 ]
on θ cannot be determined by the 5 λ mT = b or b 4 = λ4mT 4
−1 dimensional method.
30 Unit of a = unit of v = m/s = ms energy
and = σT 4
and unit of c = unit of
v 36 The degree of accuracy (and hence the area × time
t2 number of significant figures) of a energy
m /s or σ =
= = m / s 3 = ms −3 measurement cannot be increased by (area × time) T 4
s2 changing the unit.
DAY ONE UNITS AND MEASUREMENT 11

∴ LC = 0.01 ∆g ∆L 2∆T ∆g
σb 4 = 
energy  4
 λm ∴ = + or × 100%
 area × time  Negative zero error = −5 × LC = −0.005 mm g L T g
[ML2 T −2 ] Measured value = main scale reading ∆L   2∆T  × 100%
or [σb 4 ] = [L4 ] = [ML4 T −3 ]. =   × 100% +  
+ screw guage reading − zero error  L   T 
[L2 ][T]
= 0.5 mm + {25 × 0.01 − (−0.05)} mm
=  × 100 % + 2 ×
1 1
6 h = [ML2 T −1 ]; c = [LT −1 ], me = [M ], = 0.80 mm × 100%
 200  90
G = [ M −1 L3 T −2 ], e = AT; m p = M , 11 By given formula, we have surface = 2.72% = 3%
hc [M 1 L2 T −1 ] [LT −1 ] tension,
Thus, accuracy in the determination of g
= = [ M 2] dhg N Q r = d 
G [M −1 L3 T −2 ] T = × 103   is approx 3 %.
4 m  2
⇒ M =
hc ∆T ∆d ∆h 14 Given, I = (e 1000V / T − 1) mA
⇒ = + [given, g is constant]
G T d h dV = ± 0.01V, T = 300 K, I = 5mA,
h [ML2 T −1 ] So, percentage error is I = e 1000V / T − 1, I + 1 = e 1000V / T
= = [ML]
c [LT −1 ] ∆T
= × 100 Taking log on both sides, we get
h h G Gh T
⇒ L = = = 3 /2 ∆d ∆h  1000V
=  + log (I + 1) =
cM c hc c  × 100 T
 d h 
From c = [ LT −1 ], d (I + 1) 1000
 0.01 × 10−2 0.01 × 10−2  ⇒ = dV
L Gh Gh = +  × 100 I +1 T
T = = 3 /2 = 5/2  1.25 × 10
−2
1.45 × 10−2 
c c c c dI 1000
= dV
= 1.5% I +1 T
Hence, out of (i), (ii) and (iii) any one ∆T
can be taken to express L, M, T in terms ∴ × 100 = 1.5% 1000
of three chosen fundamental quantities.
T ⇒ dI = × (I +1) dV
T
7 Here, l = (16.2 ± 0.1 ) cm ; 12 Arithmetic mean time of a oscillating
Σ xi dI =
1000
× (5 + 1) × 0.01
b = (10.1 ± 0.1) cm simple pendulum =
N 300
A = l × b = 16.2 × 10.1 = 163.62 90 + 91 + 92 + 95
= = 92 s = 0.2 mA
Rounding off to three significant digits, 4
A = 164 cm2 Mean error is
So, error in the value of current is
∆ A  ∆ l ∆b  0 .1 0 .1 Σ | x − xi | 2 + 1 + 3 + 0
0.2 mA.
= +  = + = = = 1.5
A  l b  16.2 10.1 N 4 15 [ c ] = [LT −1 ] = 3 × 108 ms −1
1.01 +1.62
= = 2.63 cm2 Given, minimum division in the and [g ] = [LT −2 ] = 10 ms −2
16.2 × 10.1 measuring clock, is 1 s. Thus, the
c [LT −1 ]
Rounding off to one significant figure, reported mean time of a oscillating So, = =T
∆ A = 3cm2 simple pendulum = (92 ± 2) s. g [LT −2 ]
∴ A = (164 ± 3) cm2 3 × 108
13 Given, time period, T = 2π L ∴ T= = 3 × 107 s
g 10
8 From Ohm’s law, R = V
I Thus,changes can be expressed as 16 The physical quantities can be equated,
⇒ ln R = ln V − ln I 2T ∆L ∆g added or subtracted only when they
∆R ∆V ∆I = =± ±
⇒ = + = 3% + 3% = 6% T L g have same dimensions. The distance
R V I 1a
According to the question, we can write covered by a body is s = u +
9 Diameter of wire, d = MSR + CSR × LC ∆L 0.1cm 1 2t
= = [LT −2 ]
1 −1
= 0 + 52 × = 0.52 mm = 0.052 cm L 20.0cm 200 [L] = [LT ] +
100 90 [T]
Again time period T = s
pitch 100 [L] = [LT −1 ] + [LT −3 ]
10 Least count = 1 ∆T 1
(number of division on and ∆T = s ⇒ = As every term of equation is not is not
circular scale) 200 T 90 having same dimensions, so it is a
L wrong expression for distance.
=
0.5
mm Now, T = 2π
50 g
DAY TWO

Kinematics
Learning & Revision for the Day
u Frame of Reference u Elementary Concept of Differentiation and
u Motion in a Straight Line Integration for Describing Motion
u Uniform and Non-uniform Motion u Graphs
u Uniformly Accelerated Motion

Frame of Reference
The frame of reference is a suitable coordinate system involving space and time used as a
reference to study the motion of different bodies. The most common reference frame is
the cartesian frame of reference involving (x, y, z and t).
(i) Inertial Frame of Reference A frame of reference which is either at rest or moving
with constant velocity is known as inertial frame of reference. Inertial frame of
reference is one in which Newton’s first law of motion holds good.
(ii) Non-Inertial Frame of Reference A frame of reference moving with some
acceleration is known as non-inertial frame of reference. Non-inertial frame of
reference in one which Newton’s law of motion does not hold good.

Motion in a Straight Line


The motion of a point object in a straight line is one dimensional motion. During such a
motion the point object occupies definite position on the path at each instant of time.
Different terms used to described motion are defined below:

Distance and Displacement


l
Distance is the total length of the path travelled by a particle in a given interval of PREP
time. It is a scalar quantity and its SI unit is metre (m).
Displacement
MIRROR
l
Displacement is shortest distance between initial and final Your Personal Preparation Indicator
Y Distance
positions of a moving object. It is a vector quantity and its SI u No. of Questions in Exercises (x)—
unit is metre. A
u No. of Questions Attempted (y)—
From the given figure, mathematically it is expressed as, r1 ∆r u No. of Correct Questions (z)—
B
∆r = r2 – r1 r2 (Without referring Explanations)
l
Displacement of motion may be zero or negative O X
u Accuracy Level (z / y × 100)—
but path length or distance can never be negative.
u Prep Level (z / x × 100)—
l
For motion between two points displacement is single valued while distance depends
on actual path and so can have many values. In order to expect good rank in JEE,
your Accuracy Level should be above
l
Magnitude of displacement can never be greater than distance. However, it can be 85 & Prep Level should be above 75.
equal, if the motion is along a straight line without any change in direction.
DAY TWO KINEMATICS 13

Average and Instantaneous Acceleration If velocity of a


Speed and Velocity particle at instant t is v1 and at instant t 2 is v2 , then
l
Speed is defined as the total path length (or actual distance v2 − v1 ∆v
l
Average acceleration, aav = =
covered) by time taken by object. t2 − t 1 ∆t
Distance ∆v dv
Speed = l
Instantaneous acceleration, a = lim =
Time taken ∆t→ 0 ∆t dt
It is scalar quantity. Its SI unit is m/s.
Total distance travelled
l
Average Speed, vav = Uniformly Accelerated Motion
∆t
l
A motion, in which change in velocity in each unit of time
l
When a body travels equal distance with speeds v1 and v2 , is constant, is called an uniformly accelerated motion. So,
the average speed (v) is the harmonic mean of the two for an uniformly accelerated motion, acceleration is constant.
speeds.
2 1 1
l
For uniformly accelerated motion are given below
= +
v v1 v2 Equations of motion, v = u + at …(i)
1
l
When a body travels for equal time with speeds v1 and v2 , s = ut + at 2 …(ii)
the average speed v is the arithmetic mean of the two speeds. 2
v + v2 and v2 = u2 + 2 as …(iii)
vav = 1
2 where, u = initial velocity, v = velocity at time t
l
Velocity is defined as ratio of displacement and and s = displacement of particle at time t .
corresponding time interval taken by an object. l
Equation of uniformaly accelerated motion under gravity
Displacement are
i.e. velocity =
time interval 1
(i) v = u − gt (ii) h = ut − gt 2 (iii) v2 = u2 − 2 gh
Total displacement x2 − x 1 ∆x 2
l
Average velocity = = =
Total time taken t2 − t 1 ∆t
Here, x2 and x 1 are the positions of a particle at the time t 2 Elementary Concept of Differentiation
and t 1 respectively, with respect to a given frame of and Integration for Describing Motion
reference.
l
At an instant t , the body is at point P( x, y, z).
l
For a moving body speed can never be negative or zero
dx
while velocity can be negative and zero. Thus, velocity along X-axis, vx =
dt
l
The instantaneous speed is average speed for infinitesimal
dvx
small time interval (i.e. ∆t → 0) Acceleration along X -axis is ax =
∆s ds dt
i.e. Instantaneous speed, v = lim =
∆t → 0 ∆ t dt dy
Velocity along Y-axis is v y =
l
The instantaneous velocity (or simply velocity) v of a dt
∆x dx dvy
moving particle is v = lim = Acceleration along Y-axis is a y =
∆t → 0 ∆ t dt dt
It (at a particular time) can be calculated as the slope (at dz dv
Similarly, vz = and az = z
that particular time) of the graph of x versus t . dt dt
l
For a accelerating body
Uniform and Non-uniform Motion (i) If ax variable, x = ∫ vx dt ,∫ dvx = ∫ ax dt
l
An object is said to be in uniform motion if its velocity is
uniform i.e. it undergoes equal displacement in equal may (ii) If a y is variable, y = ∫ v y dt , ∫ dv y = ∫ a y dt
be intervals of time, howsoever small these interval.
(iii) If a z is variable, z = ∫ vz dt , ∫ dvz = ∫ az dt
l
An object is said to be in non-uniform motion if its
undergoes equal displacement in unequal intervals of Also, distance travelled by a particle is s = ∫ | v| dt
time., howsoever small these intervals may be.
(i) x-component of displacement is ∆x = ∫ vx dt
Acceleration (ii) y-component of displacement is ∆y = ∫ v y dt
Acceleration of an object is defined as rate of change
of velocity. It is a vector quantity having unit m/s2 or ms −2 . (iii) z-component of displacement is ∆z = ∫ vz dt
It can be positive, zero or negative.
14 40 DAYS ~ JEE MAIN PHYSICS DAY TWO

Graphs Velocity-Time Graph


During motion of the particle, its parameters of kinematical (i) Velocity-time graph gives the instantaneous value of
analysis changes with time. These can be represented on the velocity at any instant.
graph, which are given as follows: (ii) The slope of tangent drawn on graph gives instantaneous
acceleration.
Position-Time Graph (iii) Area under v-t graph with time axis gives the value of
(i) Position-time graph gives instantaneous value of displacement covered in given time.
displacement at any instant. (iv) The v-t curve cannot take sharp turns.
(ii) The slope of tangent drawn to the graph at any instant of Different Cases in Velocity-Time Graph
time gives the instantaneous velocity at that instant.
(iii) The s-t graph cannot make sharp turns. Different Cases v- t Graph The main Features of Graph

Different Cases of Position-Time Graph Uniform motion v (i) θ = 0º


(ii) v = constant
Different Cases s- t Graph The main Features of Graph v = constant (iii) Slope of v-t graph = a = 0

At rest s Slope = v = 0 t
Uniformly v So slope of v-t graph is
accelerated constant u = 0 i.e.
motion with v = at so, a = constant u = 0
t u = 0 and s = 0 i.e. v = 0 at t = 0
at t = 0
Uniform motion s Slope = constant, t
v = constant
Uniformly v Positive constant
a=0 accelerated acceleration because θ is
s =νt v = u + at
motion with u constant and <90º but the
t u ≠ 0 but s = 0 initial velocity of the particle
at t = 0 is positive
Uniformly s u = 0, i.e. t
accelerated Slope of s-t graph Uniformly v Slope of v-t graphs = – a
motion with 1
s = at 2 at t = 0, should be zero. decelerated (retardation)
u = 0, s = 0 at 2
motion
u
t =0 v = u – at
t

s t
Uniformly Slope of s-t graph gradually t0
accelerated goes on increasing
motion with 1 Non-uniformly v Slope of v-t graph increases
s =ut + at 2 accelerated with time.
u ≠ 0 but s = 0 2
at t = 0 motion θ is increasing, so,
t acceleration is increasing
t
Uniformly s θ is decreasing
retarded motion so, v is decreasing, a is Non-uniformly v θ is decreasing, so
negative decelerating acceleration decreasing
motion
t
t0 t
DAY TWO KINEMATICS 15

DAY PRACTICE SESSION 1

FOUNDATION QUESTIONS EXERCISE


1 An aeroplane flies 400 m from North and then flies 300 m 7 A body is thrown vertically upwards in air, when air
South and then flies 1200 m upwards, then net resistance is taken into account, the time of ascent is t1
displacement is and time of descent is t 2, then which of the following is
(a) 1200 m (b) 1300 m (c) 1400 m (d) 1500 m true?
(a) t1 = t 2 (b) t1 < t 2
2 The correct statement from the following is
(c) t1 > t 2 (d) t1 ≥ t 2
(a) A body having zero velocity will not necessarily have
zero acceleration 8 A stone falls freely from rest and the total distance covered
(b) A body having zero velocity will necessarily have zero by it in the last second of its motion equals the distance
acceleration covered by it in the first three seconds of its motion. The
(c) A body having uniform speed can have only uniform stone remains in the air for
acceleration
(d) A body having non-uniform velocity will have zero (a) 6 s (b) 5 s (c) 7 s (d) 4 s
acceleration 9 The motor of an electric train can give it an acceleration
3 A vehicle travels half the distance L with speed v1 and of 1 ms – 2 and brakes can give a negative acceleration of
the other half with speed v 2, then its average speed is 3 ms – 2. The shortest time in which the train can make a
v1 + v 2 2v1 + v 2 2v1v 2 2 (v1 + v 2 ) trip between the two stations 1215 m apart is
(a) (b) (c) (d)
2 v1 + v 2 v1 + v 2 v1v 2 (a) 113.6 s (b) 56.9 s
(c) 60 s (d) 55 s
4 A particle moves along the sides AB, BC, CD of a square
10 A train is moving along a straight path with a uniform
of side 25 m with a velocity of 15 m/s. Its average
acceleration. Its engine passes a pole with a velocity of
velocity is
60 kmh −1 and the end (guard’s van) passes across the
B C same pole with a velocity of 80 kmh −1. The middle point
of the train will pass the same pole with a velocity
(a) 70 kmh −1 (b) 70.7 kmh −1
v
(c) 65 kmh −1 (d) 75 kmh −1
11 The acceleration experienced by a moving boat after its
engine is cut-off, is given by a = −kv 3, where k is a
A D
constant. If v 0 is the magnitude of velocity at cut-off,
(a) 5 m/s (b) 7.5 m/s (c) 10 m/s (d) 15 m/s then the magnitude of the velocity at time t after the
5 A body sliding down on a smooth inclined plane slides cut-off is
down 1/4th of plane’s length in 2 s. It will slide down the v0 v0
(a) (b)
complete plane in 2 kt v 02 1 + 2 kt v 02
(a) 4 s (b) 5 s (c) 2 s (d) 3 s v0 v0
(c) (d)
6 Three particles P , Q and R are situated at corners of an 1 − 2 kv 02 1 + 2 kt v 02
equilateral triangle of side length (d). At t = 0, they 12 A body moving with an uniform acceleration describes
started to move such that P is moving towards Q, Q is 12 m in the 3rd second of its motion and 20 m in the
moving towards R and R is moving towards P at every 5th second. Find the velocity after the 10th second.
instant. After how much time (in second) will they meet
(a) 40 ms–1 (b) 42 ms–1
each other? (c) 52 ms–1 (d) 4 ms–1
Q
13 A train accelerating uniformly from rest attains a
u maximum speed of 40 ms –1 in 20 s. It travels at this
speed for 20 s and is brought to rest with an uniform
u retardation in the next 40 s. What is the average velocity
P R during this period?
u
80
d 2d 2d d (a) ms–1 (b) 25 ms–1
(a) (b) (c) (d) 3
u 3u (3) u (3) u (c) 40 ms–1 (d) 30 ms–1
16 40 DAYS ~ JEE MAIN PHYSICS DAY TWO

14 A frictionless wire AB is fixed on a sphere of radius R. A 21 Look at the graphs (i) to (iv) in figure carefully and
very small spherical ball slips on this wire. The time taken choose, which of these can possibly represent
by this ball to slip from A to B is one-dimensional motion of particle?
A x x

q
B O
t t
R

C
(i) (ii)
2 gR cosθ R gR
(a) (b) 2 gR (c) 2 (d) x x
g cosθ g g g cosθ

15 A balloon is going upwards with velocity12 ms –1. It


releases a packet when it is at a height of 65 m from the
t t
ground. How much time the packet will take to reach the
ground if g = 10 ms –2 ?
(a) 5 s (b) 6 s (c) 7 s (d) 8 s
(iii) (iv)
16 A ball is dropped from the top of a building. The ball (a) Both (i) and (ii)
takes 0.5 s to fall past the 3 m length of window some (b) Only (iv)
distance below from the top of building. With what speed (c) Only (iii)
does the ball pass the top of window? (d) Both (iii) and (iv)
(a) 6 ms–1 (b) 12 ms–1 (c) 7 ms–1 (d) 3.5 ms–1 22 Figure shows the time-displacement curve of the particles
17 A body starts from the origin and moves along the axis P and Q . Which of the following statement is correct?
such that the velocity at any instant is given by x P
v = 4 t 3 − 2 t where, t is in second and the velocity in Q
ms −1. Find the acceleration of the particle when it is at
a distance of 2 m from the origin. O t
(a) 28 ms –2 (b) 22 ms – 2 (c) 12 ms –2 (d) 10 ms –2
18 A point initially at rest moves along the x-axis. Its
(a) Both P and Q move with uniform equal speed
acceleration varies with time as a = ( 5t + 6) ms –2. If it
starts from the origin, the distance covered by it in 2 s is (b) P is accelerated and Q moves with uniform speed but
the speed of P is more than the speed of Q
(a) 18.66 m (b) 14.33 m (c) 12.18 m (d) 6.66 m
(c) Both P and Q moves with uniform speeds but the speed
19 A rod of length l leans by its upper end against a smooth of P is more than the speed of Q
vertical wall, while its other end leans against the floor.
(d) Both P and Q moves with uniform speeds but the speed
The end that leans against the wall moves uniformly
of Q is more than the speed of P
downwards. Then,
(a) the other end also move uniformly 23 The velocity versus time curve of a moving point is shown
(b) the speed of other end goes on increasing in the figure below. The maximum acceleration is
(c) the speed of other end goes on decreasing Velocity (ms–1)
(d) the speed of other end first decreases and then increases
D E
80
20 Which of the following distance-time graphs is not possible? 70
x x 60
50
40
30
B C
t t 20
(a) (b) 10
x x F
A
O 10 20 30 40 50 60 70 80 90 Time (s)
t

t (a) 1 ms−2 (b) 6 ms −2


(c) (d) (c) 2 ms−2 (d) 1.5 ms−2
DAY TWO KINEMATICS 17

24 The velocity-time graph of a body in a straight line is as 27 When two bodies move uniformly towards each other the
shown in figure. distance between them decreases by 8 ms –1. If both
bodies move in the same direction with different speeds,
v (ms–1)
2 the distance between them increases by 2 ms −1. The
speeds of two bodies will be
1
(a) 4 ms−1 and 3 ms−1
0
1 2 3 4 5
t(s) (b) 4 ms−1 and 2 ms−1
–1 (c) 5 ms−1 and 3 ms−1
(d) 7 ms−1 and 3 ms−1
–2

The displacement of the body in five seconds is Direction (Q. Nos. 28-30) Each of these questions contains
(a) 2 m (b) 3 m (c) 4 m (d) 5 m two statements : Statement I (Assertion) and Statement II
(Reason). Each of these questions also has four alternative
25 All the graphs below are intended to represent the same
choices, only one of which is the correct answer. You have to
motion. One of them does it incorrectly. Pick it up.
select one of the codes (a), (b), (c), (d) given below.
ª JEE Main 2018
(a) Statement I is true, Statement II is true, Statement II is
Velocity Distance
the correct explanation for Statement I
(b) Statement I is true, Statement II is true, Statement II is
(a) Position (b) Time
not the correct explanation for Statement I
(c) Statement I is true, Statement II is false
Position Velocity (d) Statement I is false, Statement II is true

28 Statement I A particle moving with a constant velocity,


(c) Time (d) Time
changes its direction uniformly.
Statement II In a uniform motion, the acceleration is zero.
26 A body is thrown vertically upwards. Which one of the 29 Statement I Two objects moving with velocities v1 and v 2
following graphs correctly represent the velocity versus in the opposite directions, have their relative velocity
time? ª JEE Main 2017 (Offline) along the direction of the one with a larger velocity.
Statement II The relative velocity between two bodies
v v
(a) (b) moving with velocity v1 and v 2 in same direction is given
t t
by v = v1 − v 2
30 Statement I Acceleration of a moving particle can be
change without changing direction of velocity.
v v
(c) (d) Statement II If the direction of velocity changes, so the
t
direction of acceleration also changes.
t
18 40 DAYS ~ JEE MAIN PHYSICS DAY TWO

DAY PRACTICE SESSION 2

PROGRESSIVE QUESTIONS EXERCISE


1 The velocity-time plot for a particle moving on a straight 4 A particle located at x = 0 at time t = 0, starts moving
line is as shown in figure, then along the positive x-direction with a velocity v that varies
as v = α x . The displacement of the particle varies
with time as
10
v (ms–1) (a) t 2 (b) t (c) t 1/ 2 (d) t 3
10 20 30
5 A stone is dropped from a certain height and reaches the
t (s)
ground in 5 s. If the stone is stopped after 3 s of its fall
–10
and then allowed to fall again, then the time taken by the
stone to reach the ground after covering the remaining
distance is
(a) the particle has a constant acceleration (a) 2 s (b) 3 s
(b) the particle has never turned around (c) 4 s (d) None of these
(c) the average speed in the interval 0 to 10 s is the same 6 A point moves with a uniform acceleration and v1, v 2, v 3
as the average speed in the interval 10 s to 20 s denote the average velocities in three successive
(d) Both (a) and (c) are correct intervals of time t1, t 2, t 3. Which of the following relations is
2 A body is at rest at x = 0. At t = 0, it starts moving in the correct?
positive x-direction with a constant acceleration. At the (a) (v1 − v 2 ) : (v 2 − v 3 ) = (t1 − t 2 ) : (t 2 + t 3 )
same instant another body passes through x = 0 moving (b) (v1 − v 2 ) : (v 2 − v 3 ) = (t1 + t 2 ) : (t 2 + t 3 )
in the positive x-direction with a constant speed. The (c) (v1 − v 2 ) : (v 2 − v 3 ) = (t1 − t 2 ) : (t1 − t 3 )
position of the first body is given by x1(t ) after time t and (d) (v1 − v 2 ) : (v 2 − v 3 ) = (t1 − t 2 ) : (t 2 − t 3 )
that of the second body by x 2(t ) after the same time
7 From the top of a tower of height 50 m, a ball is thrown
interval. Which of the following graphs correctly
vertically upwards with a certain velocity. It hits the
describes ( x1 − x 2 ) as a function of time t?
ground 10 s after it is thrown up. How much time does it
(x1 – x2) (x1 – x2) take to cover a distance AB where A and B are two points
20 m and 40 m below the edge of the tower?
( take,g = 10 ms – 2)
(a) (b)
(a) 2.0 s (b) 1 s (c) 0.5 s (d) 0.4 s
−1
8 Car A is moving with a speed of 36 kmh on a two lane
O t O t
road. Two cars B and C, each moving with a speed of
(x1 – x2) (x1 – x2) 54 kmh −1 in opposite directions on the other lane are
approaching car A. At certain instant of time, when the
distance AB = AC = 1 km, the driver of car B decides to
(c) (d)
overtake A before C does. What must be the minimum
acceleration of car B, so as to avoid an accident ?
O t O t
(a) 1 ms−2 (b) 4 ms−2 (c) 2 ms−2 (d) 3 ms−2
9 The displacement x of a particle varies with time,
3 The velocity of a particle is v = v 0 + gt + ft 2. If its position a
according to the relation x = (1 − e −bt ). Then
is x = 0 at t = 0, then its displacement after unit time b
(t = 1) is (a) the particle can not reach a point at a distance x from its
(a) v 0 + 2g + 3f starting position, if x > a / b
g f (b) at t = 1/ b, the displacement of the particle is nearly
(b) v 0 + +
2 3 (2 / 3) (a /b)
(c) v 0 + g + f (c) the velocity and acceleration of the particle at t = 0 are a
g and − ab respectively
(d) v 0 + +f
2 (d) the particle will come back its starting point as t → ∞
DAY TWO KINEMATICS 19

10 From the top of a tower, a stone is thrown up which 16 From a tower of height H, a particle is thrown vertically
reaches the ground in time t1. A second stone thrown upwards with a speed u. The time taken by the particle to
down, with the same speed, reaches the ground in time hit the ground, is n times that taken by it to reach the
t 2. A third stone released from rest, from the same highest point of its path. The relation between H, u and n
location, reaches the ground in a time t 3. Then, is ª 2014 JEE Main
1 1 1 (a) 2gH = n 2u 2 (b) gH = (n − 2)2 u 2
(a) = − (b) t 32 = t12 − t 22
t 3 t 2 t1 (c) 2gH = nu 2 (n − 2) (d) gH = (n − 2)2 u 2
t + t2
(c) t 3 = 1 (d) t 3 = t1 t 2 17 An object, moving with a speed of 6.25 ms −1, is
2 dv
decelerated at a rate given by = − 2.5 v , where, v is
11 A bullet moving with a velocity of 100 ms −1 can just dt
penetrate two plancks of equal thickness. The number of the instantaneous speed. The time taken by the object, to
such plancks penetrated by the same bullet, when the come to rest, would be
velocity is doubled, will be (a) 2 s (b) 4 s (c) 8 s (d) 1 s
(a) 4 (b) 6 (c) 8 (d) 10 18 A ball is released from the top of a tower of height
12 The acceleration in ms − 2 of a particle is given by, h metre. It takes T second to reach the ground. What is
a = 3 t + 2 t + 2 where, t is time. If the particle starts out
2 T
the position of the ball in s?
with a velocity v = 2 ms −1 at t = 0, then the velocity at the 3
end of 2 s is h 7h
(a) m from the ground (b) m from the ground
−1 −1 −1 −1 9 9
(a) 36 ms (b) 18 ms (c) 12 ms (d) 27 ms
8h 17h
13 A car, starting from rest, accelerates at the rate f through (c) m from the ground (d) m from the ground
9 18
a distance s, then continues at constant speed for time t
f 19 Two stones are thrown up simultaneously from the edge
and then decelerates at the rate to come to rest. If the
2 of a cliff 240 m high with initial speed at 10 ms −1 and
total distance travelled is 15 s, then 40 ms −1, respectively. Which of the following graph best
1 2 represents the time variation of relative position of the
(a) s = f t (b) s = ft
6 second stone with respect to the first? (Assume stones
1 2 1
(c) s = ft (d) s = f t 2 do not rebound after hitting the ground and neglect air
72 4 resistance, take g = 10 ms −2. The figures are schematic
14 The displacement of a particle is given by x = (t − 2)2 and not drawn to scale ª JEE Main 2015
where, x is in metres and t in seconds. The distance (y2 – y1)m (y2 – y1)m
240

240

covered by the particle in first 4 seconds is


(a) 4 m (b) 8 m (c) 12 m (d) 16 m (a) (b)
15 A metro train starts from rest and in five seconds
achieves 108 kmh −1. After that it moves with constant t 8 12
t(s)
12
t(s)
velocity and comes to rest after travelling 45 m with (y2 – y1)m (y2 – y1)m
240
240

uniform retardation. If total distance travelled is 395 m,


find total time of travelling.
(c) (d)
(a) 12.2 s (b) 15.3 s
(c) 9 s (d) 17.2 s t(s) t(s)
8 12 8 12

ANSWERS

SESSION 1 1 (a) 2 (a) 3 (c) 4 (a) 5 (a) 6 (b) 7 (b) 8 (b) 9 (b) 10 (b)
11 (d) 12 (b) 13 (b) 14 (c) 15 (a) 16 (d) 17 (b) 18 (a) 19 (c) 20 (c)
21 (d) 22 (c) 23 (b) 24 (b) 25 (b) 26 (b) 27 (c) 28 (d) 29 (d) 30 (c)

SESSION 2 1 (d) 2 (b) 3 (b) 4 (a) 5 (c) 6 (b) 7 (d) 8 (a) 9 (b) 10 (d)
11 (c) 12 (b) 13 (c) 14 (b) 15 (d) 16 (c) 17 (a) 18 (c) 19 (c)
20 40 DAYS ~ JEE MAIN PHYSICS DAY TWO

Hints and Explanations


SESSION 1 6 The person at P will travel a distance and s2 = vt 2 −
1
× 3 × t 22
1 Displacement along North PO, with velocity along PO = u cos 30° 2
3
= 400 − 300 = 100 m Here, = t 1 t 2 − t 22
Q 2
Upward displacement = 1200 m
From Eqs. (i) and (iii), we get
∴ Net displacement t
u t 1 = 3t 2 or t 2 = 1
= (100)2 + (1200)2 3
= 120415
. m ≈ 1200 m C A t 12 t 1 3 t 12 2 2
s1 + s2 = + t1 × − × = t1
2 When a body is projected vertically 2 3 2 9 3
u 30° O 2
upwards, at the highest point of its 1215 = t 12
motion, the velocity of the body 3
30°
becomes zero but acceleration is not P R 3 × 1215
B u ⇒ t1 =
zero. PO = PB sec 30° 2
d 2 d = 42.69 s
3 Time taken to travel first half distance, = × =
2 3 3 t1
t1 =
L /2
=
L Total time = t 1 + t 2 = t 1 + = 56.9 s
∴ Time of meeting, 3
v1 2v 1
distance d/ 3
Time taken to travel second half t = = 10 From v 2 − u2 = 2as
velocity u cos 30°
distance, 802 − 602
d/ 3 ⇒ =s
L /2 L =
t2 = = u 3 /2 2a
v2 2v 2
2d 6400 − 3600
Total time = t 1 + t 2 = s ⇒ s=
3u 2a
L L
= + 7 When a body is thrown up, its velocity 1400
2v 1 2v 2 =
goes on decreasing as air resistance is a
L1 1 small. When a body falls down, its
= + The middle point of the train has to
2  v 1 v 2 
 velocity goes on increasing as air
resistance is large, t 2 increases. cover a distance
Total distance g s 700
∴ Average speed = 8 As, sn = u + (2n − 1) =
Total time 2 2 a
L g From v 2 − u2 = 2as
= = 0 + (2n − 1)
L1 1 2 v 2 − 602 = 2a ×
700
= 1400
 + 
2  v1 v2  Distance travelled in the first three a
second v 2 = 1400 + 3600
2v 1 v 2 1
= From s = ut + at 2 v = 5000
v1 + v2 2
1 9 . kmh −1
= 707
4 Since, average velocity s3 = 0 × 3 + × g × 32 = g
2 2 11 Given, acceleration a = − kv 3
total displacement
= As, S n = s3 Initial velocity at cut-off, v 1 = v 0
total time taken
25
g
(2n − 1) =
9
g Initial time of cut-off, t = 0 and final time
= 2 2 after cut-off, t 2 = t
 75
  2n − 1 = 9 dv dv
 15 Again, a = = − kv 3 or 3 = − kdt
n= 5s dt v
25 × 15
= = 5m/s Integrating both sides, with in the
75 9 Let s1 be the distance travelled by the
condition of motion.
train moving with acceleration 1ms – 2
5 As, u = 0 and a is a constant v dv t

l 1
for time t 1 and s2 be the distance ∫v 0 v 3 = − ∫0 k dt
= a(2) 2 …(i) travelled by the train moving with
4 2 retardation 3 ms – 2 for time t 2 . If v is the or  − 1  = −[kt ] t
 2v 2 
1 0
l = at 2 …(ii) velocity of the train after time t 1 , then
2 1 1
On dividing Eq. (ii) by Eq. (i), we get v = 1 × t1 …(i) or − = kt
1 t2 2v 2 2v 20
l t2 s1 = × 1 × t 12 = 1 …(ii)
= , v0
l / 4 (2) 2 2 2 or v =
Also, v = 3t 2 …(iii) 1 + 2 kt v 20
t = 4s
DAY TWO KINEMATICS 21

12 Using, sn = u + a (2n − 1) 15 a = + g = 10 ms – 2 , At t = 2s,


2 5
s = 65m, t = ? x = × 8 + 3 × 4 = 18.66 m
a 6
12 = u + (2 × 3 − 1) …(i) 1 2
2 As, s = ut + at
a 2 19 If ( x, 0) and ( y , 0) are the coordinates of
20 = u + (2 × 5 − 1) …(ii)
2 ⇒ 65 = − 12t + 5 t 2 the end points of the rod at a given
location, then x2 + y 2 = l 2
On subtracting Eq. (i) from Eq. (ii), 5t 2 − 12t − 65 = 0
we get This gives, (y, 0)
a
8 = (10 − 6) = 2a 12 ± 144 + 1300 12 ± 38
2 t = = = 5s
10 10
a = 4 ms – 2
4
From Eq. (i), 12 = u + × 5
16 From s = ut + 1 at 2 , (x, 0)
2 Differentiating it w.r.t. t, we get
2
1
u = 2 ms −1 x = 0 + × 10t 2 = 5 t 2 …(i) 2x
dx
+ 2y
dy
=0
2 dt dt
From v = u + at = 2 + 4 × 10 1
Also, x + 3 = 0 + × 10(t + 0.5) 2 dx dy / dt
= 42 ms –1 =−y
2 dt x
= 5 t + + t 

13 As, v = u + at 1 …(i) 2 1 y
…(ii) and vx = − v y
 4  x
40 = 0 + a × 20
As, y decreases, x increases, so v x
a = 2 ms –2 Subtract Eq. (i) from decreases.
Eq. (ii), we get v x becomes zero when y is zero.
Now, v − u = 2as
2 2

3 = 5  + t  = + 5 t
1 5 x
402 − 0 = 2 × 2 × s1 20 The distance travelled can never be
4  4
negative in one dimensional motion.
s1 = 400 m 5
3− = 5t 21 In one dimensional motion, there is a
s2 = v × t 2 …(ii) 4 3m single value of displacement at one
= 40 × 20 = 800 m 7 7 particular time.
= 5 t or t = s
and v = u + at …(iii) 4 20
22 As x-t graph is a straight line in either
0 = 40 + a × 40,
From v = u + at , case, velocity of both is uniform. As the
a = − 1 ms – 2 slope of x - t graph for P is greater,
7
Also, v 2 − u2 = 2as v = 0 + 10 × = 3.5 ms − 1 therefore, velocity of P is greater than
20
that of Q.
02 − 402 = 2(−1) s3 17 v = 4t 3 − 2t …(i)
23 Maximum acceleration is represented by
s3 = 800 m dx the maximum slope of the velocity-time
= 4t 3 − 2t
∴Total distance travelled dt graph. Thus, it is the portion CD of the
= s1 + s2 + s3 80 − 20
On integration, we get, graph, which has a slope =
= 400 + 800 + 800 x = 2 = t 4 − t2 40 − 30

= 2000 m Let t2 = α = 6 ms −2 .
and total time taken = 20 + 20 + 40 ∴ 2=α −α 2
…(ii) 24 Displacement is the algebraic sum of area
= 80 s Let t2 = α under velocity-time graph.
2000
∴ Average velocity = = 25ms −1 α −α −2= 0
2 As, displacement = area of triangles
80 + area of rectangle
14 Acceleration of the body down the (α − 2) (α + 1) = 0
∴ α = 2, α = − 1, v (ms–1)
plane = g cos θ
Distance travelled by ball in time t which is not possible A
2
second is t 2 = α = 2 or t = 2, E F
1
AB = (g cos θ) t 2 ...(i) Differentiating Eq. (i) w.r.t. t,
2 dv O C H
= 12t 2 − 2 2 3 4 5
t (s)
From ∆ ABC, dt 1 B G
AB = 2R cos θ ...(ii)
a = 12 × 2 − 2 = 22 ms – 2 D
From Eqs. (i) and (ii), we get
1
2R cos θ = g cos θ t 2 18 Acceleration, a = dv = 5t + 6
2 dt
∆ OAB + ∆ ABC + ∆CDH + HEFG
On integrating, we get
4R
t =
2
5 dx 1 1 1
g v = t2 + 6 t = = ×2×2+ ×1×2+
2 dt 2 2 2
R Integrating again, × 1 × (− 2) + 1 × 1
or t =2 5 6
g x = t3 + t2 =2+ 1−1+ 1=3m
6 2
22 40 DAYS ~ JEE MAIN PHYSICS DAY TWO

25 If velocity versus time graph is a straight 4 Given, v = α x 7 Given, v = − u, a = g = 10 ms –2 ,


line with negative slope, then dx Q v = dx 
acceleration is constant and negative. or =α x   s = 50 m, t = 10 s
dt  dt  1
With a negative slope distance-time As, s = ut + at 2 ,
dx 2
= α dt
graph will be parabolic  s = ut − at 2  .
1 or
1
 2  x ⇒ 50 = − u × 10 + × 10 × 102
On integration, 2
So, option (b) will be incorrect. On solving,
x dx t −1
26 Initially velocity keeps on decreasing at ∫0 x
= ∫ α dt
0
u = 45ms
a constant rate, then it increases in If t 1 and t 2 are the timings taken by the
[Q at t = 0, x = 0 and let at any time t,
negative direction with same rate. ball to reach the points A and B
particle is at x]
27 Case I x respectively, then
x1 /2
Relative velocity is v 1 + v 2 = 8 ⇒ = αt 1
1/2 0 20 = − 45 t 1 + × 10 × t 12
Case II 2
α
Relative velocity is v 1 − v 2 = 2 or x1 /2 = t 1
2 40 = − 45 t 2 + × 10 × t 22
On solving, α2 2
or x= × t2 On solving, we get
v 1 = 5 ms −1 , v 2 = 3 ms −1 4
t 1 = 9.4 s and t 2 = 9.8 s
∴ x ∝ t2
28 When a particle moves with constant Time taken to cover the distance AB,
velocity, then acceleration of particle is 5 From s = ut + 1 at 2 , = (t 2 − t 1 )
zero and hence particle is not able to 2 = 9.8 − 9.4 = 0.4 s
change the direction. Hence, statement I 1
s= 0+ × 10 × 52 = 125m 8 Let us suppose that the cars A and B are
is false while statement II is true. 2
Hence, correct answer is (d). moving in the positive x-direction. Then,
Distance covered in 3 s, car C is moving in the negative
1
29 When two objects moving in opposite = 0 + × 10 × 32 = 45m x-direction.
direction, then their relative velocity 2 Therefore,
becomes (v 1 + v 2 ), hence statement I is Distance to be covered = 125 − 45 v A = 36 kmh −1 = 10 ms −1
false. When moves in same direction, = 80 m
v B = 54 kmh −1 = 15 ms −1
then relative velocity v = (v 1 − v 2 ), From s = ut + at
1 2
hence statement II is true. Hence, 2 and v C = − 54 kmh −1 = − 15 ms −1
correct answer is (d). 1 Thus, the relative speed of B with respect
80 = 0 + × 10t 2
30 Without changing direction of velocity, 2 to A is,
it is possible to change the acceleration ⇒ t =
2 80
= 16 v BA = v B − v A
of a moving particle, hence statement I 5 = 15 − 10 = 5 ms −1
is true, while statement II is false. ∴ t = 4s and the relative speed of C with respect
Hence, correct answer is (c). 6 Suppose velocity at O = zero to A is,
As average velocity in interval t 1 v CA = v C − v A = − 15 − 10
SESSION 2 is v 1 , = − 25ms −1
1 The slope of velocity-time graph gives ∴ Velocity at A = v 1 At time t = 0, the distance between A
acceleration. Since, the given graph is a As average velocity in interval t 2 and B = distance between A and
straight line and slope of graph is is v 2 , C = 1 km = 1000 m.
constant. Hence acceleration is ∴ Velocity at B = (v 2 − v 1 ) The car C covers a distance AC = 1000 m
constant. Thus, (a) is correct. The area and reaches car A at a time t given by
As average velocity in interval t 3
of v-t graph between 0 to 10 s is same as
is v 3 , AC
between 10 s to 20 s. t =
Velocity at C = (v 3 − v 2 + v 1 ) |v CA|
2 As, x1 ( t ) = 1 at 2 and x2 ( t ) = vt Using v = u + at
1000 m
2 v 1 = 0 + at 1 …(i) = = 40 s
1 25 ms −1
∴ x1 − x2 = at 2 − vt (parabola) (v 2 − v 1 ) = 0 + a(t 1 + t 2 ) …(ii)
2 (v 3 − v 2 + v 1 ) = 0 + a(t 1 + t 2 + t 3 ) Car B will overtake car A just before car
Clearly, graph (b) represents it correctly. …(iii) C does and the accident can be avoided if
3 As, v = v 0 + gt + ft 2 or Subtract Eq. (i) from Eq. (iii), we get it acquires a minimum acceleration a
dx
= v 0 + gt + ft 2 (v 3 − v 2 ) = a (t 2 + t 3 ) …(iv) such that it covers a distance,
dt s = AB = 1000 m in time t = 40 s
Divide Eq. (ii) by Eq. (iv), we get
⇒ dx = (v 0 + gt + ft 2 ) dt travelling with a relative speed of
(v 2 − v 1 ) a(t 1 + t 2 )
x 1 = u = v BA = 5ms −1 .
So, ∫0 dx = ∫0
(v 0 + gt + ft 2 ) dt (v 3 − v 2 ) a(t 2 + t 3 )
This gives, from
g f (v 1 − v 2 ) t 1 + t2
⇒ x = v0 + + = 1 2
2 3 (v 2 − v 3 ) t 2 + t3 s = ut + at , a = 1 ms −2
2
DAY TWO KINEMATICS 23

9 Velocity of the particle is given by We know that the relation for the or s + ( f t 1 ) t + 2 s = 15 s
dx d  a stopping distance s is  s = 1 f t2 
−bt  −bt  1
v = =  (1 − e ) = ae  
dt dt  b  v 2 = u2 + 2as 2
or ( f t 1 ) = 12s …(ii)
Acceleration of the particle is given by Since, v = 0,
From Eqs. (i) and (ii), we have
dv d So, 2as = − u2
α = = (ae −bt ) = − abe −bt 12s ( f t 1 )t
dt dt As, s ∝ u2 =
s 1
At t = 1/ b , the displacement of the 2 2 ( f t 1 )t 1
s1  u1 
=   = 
100  1 2
particle is Hence,  =
s2  u2   200  4
t
x = (1 − e −1 ) ≈  1 −  =
a a 1 2 a or t1 =
b b  3 3 b Thus, s2 = 4 s1 = 8 x 6
Q e −1 ≈ 1  Hence, the number of plancks From Eq. (i), we get
  s
 3 = n2 = 2 = 8 1
s = f (t 1 ) 2
x 2
Thus, choice (b) is correct. At t = 0, the dv 2
value v and α are v = ae −0 = a 12 Given, a = = 3t 2 + 2t + 2 1 t  1 2
= f  = ft
dt 2  6 72
and α = − ab e −0 = ab ⇒ dv = (3t 2 + 2t + 2) dt
The displacement x is maximum, when On integrating, this gives
14 Here, x = ( t − 2)2
t → ∞, v 2 v
∫u dv = ∫0(3t + 2t + 2) dt
2

4 ms-1
a a
i.e. x max = (1 − e − ∞ ) = 2
C
b b  3t 3 2t 2 
⇒ v −u=  + + 2t 
10 We know that, h = ut + 1 gt 2  3 2 0
2 B D
1 ⇒ v = u + [ t 3 + t 2 + 2t ] 20 O t
2s 4s
⇒ h = − ut + gt 2
2 v = 2 + [ 2 + 2 + 2 × 2]
3 2

– 4 ms-1
2u
and t = = t1 − t2 …(i) = 2 + 16 A
g
= 18 ms −1
A
C 13 The velocity-time graph for the given dx
Velocity, v = = 2 ( t − 2) ms −1
B situation can be drawn as below. dt
A Magnitudes of slope of OA = f dv
h Acceleration, a = = 2 ms −2
Y dt
(i.e. uniform)
When t = 0, v = −4 ms −1 ,
1 2 v (ms–1) A B t = 2 s, v = 0, t = 4s, v = 4 ms −1
Case I h = − ut 1 + gt 1 …(ii)
2
Velocity (v ) - time (t) graph of this motion
1
Case II h = + ut 2 + gt 22 …(iii) is as shown in figure.
2 C
1 O X
Case III h = gt 32 …(iv) t1 t t2 Distance travelled
2 D E = Area AOB + Area BCD
t (s)
This gives, 4×2 4×2
2h 2u f = + = 8m
= t 2 + t 22 …(v) and slope of BC = 2 2
g g 2
Solving these, give us f 15 Given, v = 108 kmh −1 = 30 ms −1
v = f t1 = t2
t 32 = (t 1 − t 2 ) t 2 + t 22 2 From first equation of motion
t 2 = 2t 1 v = u + at
⇒ t3 = t1 t2 ∴ 30 = 0 + a × 5 (Q u = 0)
In graph area of ∆ OAD gives distance,
11 Given that the initial velocity of the 1 or a = 6 ms − 2
s = f t 12 …(i)
bullet in the first case is u1 = 100 ms −1 . 2 So, distance travelled by metro train in 5
s
Initial number of plancks, n1 = 2 Area of rectangle ABED gives distance
1 1
travelled in time t s1 = at 2 = × (6) × (5)2 = 75 m
Initial stopping distance 2 2
s2 = ( f t 1 )t
= s1 = n1 x = 2 x, Distance travelled in time t 2 ,
Distance travelled before coming to rest
with x as the thickness of one planck. = 45m
1f
s3 = (2t 1 ) 2 So, from third equation of motion
Similarly, Initial velocity of the bullet in 22
second case, Thus, s1 + s2 + s3 = 15 s 02 = (30)2 − 2a′ × 45
30 × 30
u2 = 2 × 100 = 200 ms −1 ⇒ s + ( f t 1 ) t + f t 12 = 15 s or a′ = = 10 ms −2
2 × 45
24 40 DAYS ~ JEE MAIN PHYSICS DAY TWO

Time taken in travelling 45 m is 18 From equation law of motion gives, Consider the stones thrown up
30 simultaneously as shown in the diagram
t3 = = 3s 1
10 s = ut + gT 2 below. As motion of the second particle
2
Now, total distance = 395 m with respect to the first we have relative
1
i.e. 75 + s ′ + 45 = 395 m or h = 0 + gT 2 (Q u = 0) acceleration
2 |a21 |=|a2 − a1 |= g − g = 0.
or s′ = 395 − (75 + 45) = 275 m
∴ t2 =
275
= 9.2 s  2h  Thus, motion of first particle is straight
⇒ T =   line with respect to second particle till
30  g 
Hence, total time taken in whole the first particle strikes ground at a time
journey = t 1 + t 2 + t 3 t=0 is given by
u=0 1
= 5 + 9 . 2 + 3 = 17. 2 s −240 = 10t − × 10 × t 2
s T 2
16 Time taken to reach the maximum t=
h 3 or t 2 − 2 t − 48 = 0
u
height, t 1 =
g or t − 8t + 6t − 48 = 0
2
O t=T
or t = 8, − 6
t1 u Ground
[As, t = − 6s is not possible]
H T
t2 At, t = s, i.e., t = 8s
3
2 Thus, distance covered by second
1 T 
If t 2 is the time taken to hit the ground, s= 0+ g  particle with respect to first particle in 8
2  3 s is
1
i.e. − H = ut 2 − gt 22 1 T2 s 12 = (v 21 ) t = (40 − 10) (8s)
2 or s= g⋅
But t 2 = nt 1 [Given] 2 9 = 30 × 8
g 2h  2h  = 240 m
nu 1 n2u2 ⇒ s= × ∴ T = 
So, − H = u − g
g 2 g2 18 g  g  Similarly, time taken by second particle
h to strike the ground is given by
nu2 1 n2u2 or s= m
−H = − 1
g 2 g
9 −240 = 40 t − × 10 × t
Hence, the position of ball from the 2
1 n2u2 nu2 n2u2 − 2nu2 − 240 = 40 t − 5t 2
H = − = h 8h
ground = h − = m
or
2 g g 2g 9 9 or 5 t 2 − 40 t − 240 = 0
2gH = n2u2 − 2nu2
19 Central idea concept of relative motion or t 2 − 8 t − 48 = 0
2gH = nu2 (n − 2) can be applied to predict the nature of
motion of one particle with respect to t 2 − 12 t + 4 t − 48 = 0
17 Given, dv = − 2.5 v
dt the other. or t (t − 12) + 4 (t − 12) = 0

dv
= − 2.5dt or t = 12, − 4
v 40 ms-1
(As, t = − 4 s is not possible)
0 −1 /2 t 10 ms-1 t = 12 s
⇒ ∫ 6.25 v dv = − 2.5 ∫ dt i.e.
240 m

0
Thus, after 8 s, magnitude of relative
⇒ − 2.5 [ t ] t0 = [2 v 1 /2 ] 0625
. Cliff
velocity will increase upto 12 s when
second particle strikes the ground.
= 2(− 6.25) = 2 × 2.5 Hence, graph (c) is the correct
description.
⇒ t = 2s
DAY THREE

Scalar
and Vector
Learning & Revision for the Day
u Scalar and Vector Quantities u Multiplication or Division of u Relative velocity
u Laws of Vector Addition a Vector by a Scalar u Motion in a Plane
u Subtraction of Vectors u Product of Vectors u Projectile Motion
u Resolution of a vector

Scalar and Vector Quantities


A scalar quantity is one whose specification is completed with its magnitude only.
e.g. mass, distance, speed, energy, etc.
A vector quantity is a quantity that has magnitude as well as direction. e.g. Velocity,
displacement, force, etc.

Position and Displacement Vectors


A vector which gives position of an object with reference to the origin of a coordinate
system is called position vector.
The vector which tells how much and in which direction on object has changed its
position in a given interval of time is called displacement vector.

General Vectors and Notation


l
Zero Vector The vector having zero magnitude is called zero vector or null vector. It PREP
is written as 0. The initial and final points of a zero vector overlap, so its direction is
arbitrary (not known to us).
MIRROR
Your Personal Preparation Indicator
l
Unit Vector A vector of unit magnitude is known as an unit vector. Unit vector for A
is A$ (read as A cap). u No. of Questions in Exercises (x)—
A=AA Direction
u No. of Questions Attempted (y)—
u No. of Correct Questions (z)—
(Without referring Explanations)
Magnitude
Y u Accuracy Level (z / y × 100)—
l
Orthogonal Unit Vectors The unit vectors along X -axis,s, Y-axis
j u Prep Level (z / x × 100)—
and Z-axis are denoted by $i, $j and k$ . These are the orthogonal unit
i
vectors. In order to expect good rank in JEE,
X
$i = x , $j = y , k$ = z your Accuracy Level should be above
k 85 & Prep Level should be above 75.
x y z Z
26 40 DAYS ~ JEE MAIN PHYSICS DAY THREE

l
Parallel Vector Two vectors are said to be parallel, if they Thus, vector subtraction is B
have same direction but their magnitudes may or may not really a special case of vector A
be equal. addition. The geometric
l
Antiparallel Vector Two vectors are said to be anti-parallel construction for subtracting
when two vectors is shown in the A – B –B
(i) both have opposite direction above figure.
(ii) one vectors is scalar non zero negative multiple of If θ be the angle between A
another vector. and B,
l
Collinear Vector Collinear vector are those which act along
then | A − B| = A2 + B2 − 2 AB cos θ
same line.
l
Coplanar Vector Vector which lies on the same plane are If the vectors form a closed n sided polygon with all the sides
called coplanar vector. in the same order, then the resultant is zero.
l
Equal Vectors Two vectors A and B are equal, if they have
the same magnitude and the same direction.
Multiplication or Division
Laws of Vector Addition of a Vector by a Scalar
The multiplication or division of a vector by a scalar gives a
1. Triangle Law vector. For example, if vector A is multiplied by the scalar
If two non-zero vectors are represented by the two sides of a number 3, the result, written as 3A, is a vector with a magnitude
triangle taken in same order than the resultant is given by the three times that of A, pointing in the same direction as A. If we
closing side of triangle in opposite order, i.e. multiply vector A by the scalar − 3, the result is − 3 A , a vector
R=A+B with a magnitude three times that of A, pointing in the direction
The resultant R can be calculated as opposite to A (because of the negative sign).
| A + B| = R = A2 + B2 + 2 AB cos θ
Products of Vectors
B The two types of products of vectors are given below
A+ B sin θ
R= B
α θ Scalar or Dot Product
A B cos θ
The scalar product of two vectors A and B is defined as the
If resultant R makes an angle α with vector A, then product of magnitudes of A and B multiplied by the cosine of
B sin θ smaller angle between them. i.e. A ⋅ B = AB cos θ
tan α =
A + B cos θ
Properties of Dot Product
2. Parallelogram Law B
l
Dot product or scalar product of two
B
C vectors gives the scalar two vectors
According to parallelogram law of
vector addition, if two vector acting given the scalar quantity.
on a particle are represented in Q R l
It is commutative in nature.
magnitude and direction by two β
i.e. A ⋅ B = B ⋅ A .
adjacent side of a parallelogram, then α θ
P A
l
Dot product is distributive over the
the diagonal of the parallelogram O A
addition of vectors. B cos θ
represents the magnitude and direction of the resultant of the
i.e. A ⋅ (B + C) = A ⋅ B + A ⋅ C Component of
two vector acting as the particle.
l $i ⋅ i$ = $j ⋅ $j = k$ ⋅ k$ = 1, because angle vector B along A
i.e. R = P+Q
between two equal vectors is zero.
Magnitude of the resultant R is given by
l
If two vectors A and B are perpendicular vectors, then
| R | = P2 + Q2 + 2 PQ cos θ A ⋅ B = AB cos 90 ° = 0 and $i ⋅ $j = $j ⋅ k$ = k$ ⋅ i$ = 0
Q sin θ P sin θ
tan α = ⇒ tan β =
P + Q cos θ Q + P cos θ
The Vector Product
The vector product of A and B, written as A × B, produces a
Subtraction of Vectors third vector C whose magnitude is C = AB sin θ. where, θ is the
Vector subtraction makes use of the definition of the negative smaller of the two angles between A and B.
of a vector. We define the operation A − B as vector − B added Because of the notation, A × B is also known as the cross
to vector A. A − B = A + ( − B) product, and it is spelled as ‘A cross B’.
DAY THREE SCALAR AND VECTOR 27

Properties of Cross Product Relative Velocity


l
Vector or cross product of two vectors gives the vector The time rate of change of relative position of one object with
quantity. respect to another is called relative velocity.
l
Cross product of two vectors does not obey the
commutative law. i.e. A × B ≠ B × A ; Different Cases
Here, A × B = − B × A Case I If both objects A and B move along parallel straight
l
Cross product of two vectors is distributive over the lines in the opposite direction, then relative velocity of B
addition of vectors. w.r.t. A is given as,
A × (B + C) = A × B + A × C vBA = vB − (− v A ) = vB + v A
l
Cross product of two equal vectors is given by A × A = 0 If both objects A and B move along parallel staight lines in the
Similarly, $i × $i = (1 × 1 × sin 0 ° ) n$ = 0 same direction, then
$j × $j = (1 × 1 × sin 0 ° ) n$ = 0 v AB = v B − v A
$k × k$ = (1 × 1 × sin 0 ° ) n$ = 0 Case II Crossing the River To cross the river over shortest
distance, i.e. to cross the river straight, the man should swim
l
Cross product of two perpendicular vectors is given as
upstream making an angle θ with OB such that, OB gives the
A × B = ( AB sin 90 ° ) n$ = ( AB) n$
direction of resultant velocity ( vmR ) of velocity of swimmer v M
l
For two vectors A = a $i + a $j + a k$
x y z and velocity of river water v R as shown in figure. Let us
and B = b x i$ + b y $j + b z k$ . consider
A vR B
i$ $j k$
A × B = ax a y az
bx b y bz vm vmR
θ
l
Cross product of vectors $i , $j and k$ are following cyclic rules
as follows $i × $j = k,
$ $j × k$ = $i and k$ × $i = $j O
vR
j In ∆OAB, sin θ = and vmR = v2m − v2R
vm
The time taken to cross the river given by
d d
i ⊕ t1 = =
vmR v2 − v2 m R

Case III To cross the river in possible shortest time The


k man should go along OA. Now, the swimmer will be going
Cyclic representation for unit vectors $i , $j and k$ along OB, which is the direction of resultant velocity v mR of
vm and vR .
NOTE • Vector triple product is given by
AB vR
A × (B × C) = B (A ⋅ C) − C (A ⋅ B) In ∆OAB, tan θ = =
OA vm

Resolution of a Vector and vmR = v2m + v2R


The process of splitting of a single Y A vR B
vector into two or more vectors in x
different direction is called resolution A
of a vector. Consider a vector A in the d vm –vmR
A
X -Y plane making an angle θ with the y v θ
X -axis. The X and Y components of
A are Ax and A y respectively. downstream
θ upstream O
O X
Thus A = A = ( A cos θ)i$
x xi Ax Time of crossing the river,
along X -direction
d OB x2 + d 2
A y = A yj = ( A sin θ)$j along Y-direction t = = =
vm vmR v2m + v2R
From triangle law of vector addition
The boat will be reaching the point B instead of point A. If
|A|= | A xi + A yj| = A2x + A2y AB = x,
Ay  Ay  v x dv
and tan θ = = θ = tan −1   then, tan θ = R = ⇒ x= R
Ax  Ax  vm d vm
28 40 DAYS ~ JEE MAIN PHYSICS DAY THREE

gx2
Motion in a Plane l
Equation of trajectory, y = u
2u2 O X
Let the object be at position A y y
2 B 2h
and B at timing t 1 and t 2 , where A l
Time of flight, T =
y P
OA = r1 , and OB = r2 1 g h x vx
β
Suppose O be the origin for r1 r2 2h
measuring time and position of
l
Horizontal range, R = u vy v
g Y
the object (see figure). A B
O x1 x2 l
Velocity of projectile at any
l
Displacement of an object time, v = u2 + g2t 2
form position A to B is
AB = r = r2 − r1 = ( x2 − x1 ) i$ − ( y2 − y1 ) $j
Projectile Motion Up an Inclined Plane
r −r
l
Velocity, v = 2 1 Let a particle be projected
t2 − t1 up with speed u from an Y X
α P
inclined plane which in
l
A particle moving in X -Y plane (with uniform velocity) u – gs t=T
then, its equation of motion for X and Y axes are makes an angle α with the a x=
horizontal and velocity of ay = g cos α
v = v i$ + v $j, r = x i$ + y $j and r = xi$ + y$j
x y 0 0 0 projection makes an angle θ
α g
x = x0 + vx t , y = y0 + v y t θ with the inclined plane. O t=0
l
A particle moving in xy-plane (with uniform acceleration),
2u sin θ
then its equation of motion for X and Y-axes are l
Time of flight on an inclined plane, T =
vx = ux + ax t , v y = u y + a yt g sin α
1 1 u2 sin2 θ
x = x 0 + ux t + ax t 2 , y = y0 + u yt + a yt 2 l
Maximum height, h =
2 2 2 g cos α
a =a i+a i
$ $
x y 2u2 sin θ cos (θ + α )
l
Horizontal range, R =
g cos2 α
Projectile Motion π α
l
Maximum range occurs when θ = −
2 2
Projectile is an object which Y
once projected in a given u2
A u cos θ l
Rmax = when projectile is thrown upwards.
g(1 + sin α )
uy = u sin θ

direction with given velocity


and is then free to move u
u2
under gravity alone. The h
l
Rmax = when projectile is thrown downwards.
path described by the g (1 − sin α )
θ
projectile is called its
X
trajectory. O ux = u cos θ D
R
B
Projectile Motion Down
Let a particle is projected at
an angle θ from the ground with initial velocity u. an Inclined Plane
Resolving u in two components, we have A projectile is projected down the plane from the point O with
ux = u cos θ, u y = u sin θ, ax = 0, a y = − g. an initial velocity u at an angle θ with horizontal. The angle of
inclination of plane with horizontal α. Then,
g
l
Equation of trajectory, y = x tan θ − x2 α
2u cos2 θ
2
x sin
u g
u2 sin2 θ θ α)
=
l
Vertical height covered, h = ° –
2g O 90
s
(
α g sin (90°– α)
α

co
u sin 2θ 2
= g cos α
°–

g
Horizontal range, R = OB = ux T , R =
90

l
α α
g A A

NOTE Maximum range occurs when θ = 45° 2u sin(θ + α )


l
Time of flight down an inclined plane, T =
g cos α
Projectile Motion in Horizontal
u2
Direction From Height ( h) l
Horizontal range, R = [sin (2θ + α ) + sin α ]
g cos2 α
Let a particle be projected in horizontal direction with speed u
from height h.
DAY THREE SCALAR AND VECTOR 29

DAY PRACTICE SESSION 1

FOUNDATION QUESTIONS EXERCISE


1 If A and B are two non-zero vectors having equal 10 A ball is thrown from the ground with a velocity of
magnitude, the angle between the vectors A and A − B is 20 3 ms −1 making an angle of 60° with the horizontal.
(a) 0° (b) 90° (c) 180° The ball will be at a height of 40 m from the ground
(d) dependent on the orientation of A and B after a time t equal to (take, g = 10 ms –2 )
2 A vector having magnitude of 30 unit, makes equal (a) 2 s (b) 3 s (c) 2 s (d) 3 s
angles with each of the X , Y and Z -axes. The 11 A body is projected with a velocity v1 from the point A as
components of the vector along each of X , Y and Z -axes shown in the figure. At the same time, another body is
are projected vertically upwards from B with velocity v 2 . The
10 point B lies vertically below the highest point. For both
(a) 10 3 unit (b) unit (c) 15 3 unit (d) 10 unit v
3 the bodies to collide, 2 should be
v1
3 A particle has an initial velocity 3 $i + 4$j and an
acceleration of 0.4 $i + 0.3 $j. Its speed after 10 s is v1
v2
(a) 10 units (b) 7 2 units (c) 7 units (d) 8.5 units
30°
4 Unit vector perpendicular to vector A = 3 $i + $j and A
B
B = 2 $i − $j − 5 k$ both, is
3
(a) 2 (b) 0.5 (c) (d) 1
3 $j − 2 k$ ($i − 3 $j + k$ )
(a) ± (b) ± 2
11 11
12 A person aims a gun at a bird from a point, at a
− $j + 2 k$ $i + 3 $j − k$
(c) ± (d) ± horizontal distance of 100 m. If the gun can induce a
13 13
speed of 500 ms −1 to the bullet, at what height above the
5 A force F = ( 5 $i + 3 $j + 2 k$ ) N is applied over a particle bird must he aim his gun in order to hit it?
which displaces it from its origin to the point r = (2 $i − $j ) m. (take, g = 10 ms – 2 )
The work done on the particle (in joule) is (a) 10 cm (b) 20 cm (c) 50 cm (d) 100 cm
(a) − 7 (b) + 7 (c) + 10 (d) + 13 13 A cannon ball has the same range R on a horizontal
6 If A × B = B × A, then the angle between A and B is plane for two angles of projection. If h1 and h2 are the
π π π greatest heights in the two paths for which this is
(a) π (b) (c) (d)
3 2 4 possible, then
7 A ball rolls off the top of a stair way with a horizontal (a) R = (h1h2 )1 / 4 (b) R = 3 h1h2
−1
velocity of u ms . If the steps are h metre high and b (c) R = 4 h1h2 (d) R = h1h2
metre wide, the ball will hit the edge of the nth step, 14 A projectile is thrown in the upward direction making an
where n is angle of 60° with the horizontal direction with a velocity of
(a)
2hu
(b)
2hu 2
(c)
2 hu 2
(d)
hu 2 147 ms −1. Then, the time after which its inclination with
gb 2 gb 2 gb gb 2 the horizontal is 45°, is
8 Two paper screens A and B are separated by a distance (a) 15 s (b) 10.98 s (c) 5.49 s (d) 2.745 s
of 200 m. A bullet pierces A and B. The hole in B is 40 cm 15 A projectile projected with a velocity u at an angle θ
below the hole in A. If the bullet is travelling horizontally passes through a given height h two times at t1 and t 2 .
at the time of hitting A, then the velocity of the bullet at Then,
A is T
(a) t1 + t 2 = T (time of flight) (b) t1 + t 2 =
(a) 200 ms − 1 (b) 400 ms − 1 (c) 600 ms − 1 (d) 700 ms − 1 2
9 A projectile is fired at an angle of 30° with the horizontal (c) t1 + t 2 = 2T (d) t1t 2 = T
such that the vertical component of its initial velocity is 16. A particle is projected at angle of 60° with the horizontal
T
80 ms − 1. Its time of flight is T . Its velocity at t = has a having a kinetic energy K. The kinetic energy at the
4 highest point is
magnitude of nearly (take, g =10 ms – 2 ) (a) K (b) zero
(a) 180 ms − 1 (b) 155 ms − 1 (c) 145 ms – 1 (d) 140 ms − 1 (c) K / 4 (d) K / 2
30 40 DAYS ~ JEE MAIN PHYSICS DAY THREE

17 A boy playing on the roof of a 10 m high building throws express train applied brakes to avoid collision. If the
a ball with a speed of 10 ms −1 at an angle of 30° with retardation due to brakes is 4 ms −2 , the time in which the
the horizontal. How far from the throwing point, will the accident is avoided after the application of brakes is
ball be at the height of 10 m from the ground? (a) 4.25 s (b) 5.25 s (c) 6.25 s (d) 7.25 s
1 3 26 A boat takes 2 h to travel 8 km and back in a still water lake.
(take, g = 10 ms – 2 , sin 30° = , cos 30° = )
2 2 If the velocity of water is 4 kmh −1, the time taken for going
(a) 5.20 m (b) 4.33 m (c) 2.60 m (d) 8.66 m upstream of 8 km and coming back is
18 A ball projected from ground at an angle of 45° just (a) 2 h (b) 2 h and 40 min
(c) 1 h and 20 min
clears a wall in front. If point of projection is 4 m from
(d) Cannot be estimated from the given information
the foot of wall and ball strikes the ground at a distance
of 6 m on the other side of the wall, the height of the 27 A car is travelling with a velocity of 10 kmh −1 on a straight
wall is j JEE Main (Online) 2013
road. The driver of the car throws a parcel with a velocity
(a) 4.4 m (b) 2.4 m (c) 3.6 m (d) 1.6 m of 10 2 kmh −1 when the car is passing by a man
19 Neglecting the air resistance, the time of flight of a standing on the side of the road. If the parcel is to reach
projectile is determined by the man, the direction of throw makes the following angle
with the direction of the car.
(a) U vertical (b) U horizontal
(d) tan  
1
(c) U = U 2
+U 2
(d) U = (U vertical
2
+ U horizontal
2
)1 / 2 (a) 135° (b) 45° (c) tan−1 ( 2 )
vertical horizontal  2
20 The horizontal range of a projectile is 4 3 times its 28 A point P moves in counter- clockwise direction on a
maximum height. Its angle of projection will be circular path as shown in the figure.
(a) 45° (b) 60° (c) 90° (d) 30° Y
21 A projectile is fired at an angle of 45° with the horizontal. P (x, y)
Elevation angle of the projectile at its highest point as

m
seen from the point of projection is
(a) 60° (b) tan−1 ( 3 / 2) 20 X
(c) tan−1 (1 / 2) (d) 45° O

22 A man can swim with a speed of 4 kmh−1 in still The movement of p is such that it sweeps out a length
water.How long does he take to cross a river 1km wide, if s = t 3 + 5, where, s is in metre and t is in second.The
the river flows steadily 3 kmh−1 and he makes his strokes radius of the path is 20 m. The acceleration of P when
normal to the river current.How far down the river does t = 2s is nearly
he go, when he reaches the other bank? (a) 13 ms −2 (b) 12 ms −2 (c) 7.2 ms−2 (d) 14 ms −2
(a) 800 m (b) 900 m 29 For a particle in uniform circular motion the acceleration
(c) 400 m (d) 750 m a at a point P (R , θ ) on the circle of radius R is (here, θ is
23 A swimmer crosses a flowing stream of width d to and fro measured from the X -axis)
in time t1. The time taken to cover the same distance up v2 v2 v2 v2
(a) − cos θ $i + sin θ $j (b) − sin θ $i + cos θ $j
and down the stream is t 2 . Then, the time the swimmer R R R R
would take to swim across a distance 2 d in still water is v2 v2 v2 $ v2 $
(c) − cos θ $i − sin θ $j (d) i + j
t12 t 22 R R R R
(a) (b) (c) t1 t 2 (d) (t1 + t 2 )
t2 t1
Direction (Q. Nos. 30-34) Each of these questions contains
24 A man standing on a road has to hold his umbrella at 30° two statements : Statement I (Assertion) and Statement II
with the vertical to keep the rain away. He throws the (Reason). Each of these questions also has four alternative
umbrella and starts running at 10 kmh −1. He finds that the choices, only one of which is the correct answer. You have to
raindrops are hitting his head vertically. The actual select one of the codes (a), (b), (c), (d) given below :
speed of raindrops is (a) Statement I is true, Statement II is true; Statement II is the
−1 −1 correct explanation for Statement I
(a) 20 kmh (b) 10 3 kmh
(c) 20 3 kmh −1 (d) 10 kmh −1 (b) Statement I is true, Statement II is true; Statement II is not
the correct explanation for Statement I
25 A passenger train is moving at 5 ms −1 . An express train
(c) Statement I is true; Statement II is false
is travelling at 30 ms −1, on the same track and rear side of
(d) Statement I is false; Statement II is true
the passenger train at some distance. The driver in
DAY THREE SCALAR AND VECTOR 31

30 Statement I Rain is falling vertically downwards with a 33 Statement I The resultant of three vectors OA, OB and
velocity of 3 kmh −1. A man walks with a velocity of 4 kmh −1. OC as shown in the figure is R (1 + 2 ). R is the radius of
Relative velocity of rain w.r.t. man is 5 kmh −1. the circle. C
Statement II Relative velocity of rain w.r.t. man is given B
by v rm = v r − v m 45°
45°
31 Statement I For the projection angle tan−1( 4) , the O A
horizontal and maximum height of a projectile are equal.
Statement II The maximum range of a projectile is
directly proportional to the square of velocity and
inversely proportional to the acceleration due to gravity. Statement II OA + OC is acting along OB and
(OA + OC ) + OB is acting along OB.
32 Statement I In order to hit a target, a man should point
his rifle in the same direction as the target. 34 Statement I Angle between $i + $j and $i is 45°.
Statement II The horizontal range of bullet is dependent Statement II $i + $j is equally include to both $i and $j and
on the angle of projection with the horizontal. the angle between $i and $j is 90°.

DAY PRACTICE SESSION 2

PROGRESSIVE QUESTIONS EXERCISE


1 If F1 and F2 are two vectors of equal magnitude F such 4 A projectile is given an initial velocity of (i + 2 j) ms −1
that | F1 ⋅ F2 | = | F1 × F2 |, then | F1 + F 2 | is equal to where, i is along the ground and j is along the vertical.If
g = 10 ms −2 , the equation of its trajectory is
u
j
JEE Main 2013
(a)Y = X − 5 X 2
(b)Y = 2 X − 5 X 2
h (c) 4Y = 2 X − 5 X 2 (d) 4Y = 2 X − 25 X 2
θ 5 A particle of mass m is projected with a velocity v making
d an angle of 30° with the horizontal.The magnitude of
(a) (2 + 2) F (b) 2F angular momentum of the projectile about the point of
(c) F 2 (d) None of these projection when the particle is at its maximum height h is
3 mv 2
2 If a stone is to hit at a point which is at a distance d away (a) (b) zero
2g
and at a height h above the point from where the stone
mv 3 3 mv 3
starts, then what is the value of initial speed u if stone is (c) (d)
2g 16 g
launched at an angle θ?
g d d g 6 The coordinates of a moving particle at any time t are
(a) (b) given by x = αt 3 and y = βt 3 . The speed of the particle at
cosθ 2 (d tanθ − h) cosθ 2 (d tanθ − h)
time t is given by
gd 2 gd 2 (a) t 2 α 2 + β 2 (b) α 2 + β 2
(c) (d)
h cos2 θ (d − h)
(c) 3 t α 2 + β 2 (d) 3 t 2 α 2 + β 2
3 A projectile can have the same range R for two angles of
7 A ball whose kinetic energy is E , is projected at an angle
projection. If T1 and T2 be the time of flight in the two
of 45° with respect to the horizontal. The kinetic energy of
cases, then the product of the two time of flight is directly
the ball at the highest point of its flight will be
proportional to
E
1 (a) E (b)
(a) R (b) 2
R
1 E
(c) (d) R 2 (c) (d) zero
R2 2
32 40 DAYS ~ JEE MAIN PHYSICS DAY THREE

8 A river is flowing from West to East with a speed of v4 π v4


(a) π (b)
−1
5 m - min . A man on the South bank of the river, is g2 2 g2
capable of swimming at 10 m - min−1 in still water, he v2 v2
(c) π 2 (d) π
wants to swim across the river in the shortest time. He g g
should swim in a direction 13 Two fixed frictionless inclined plane making an angle 30°
(a) due to North (b) 30° East of North and 60° with the vertical are as shown in the figure. Two
blocks A and B are placed on the two planes. What is the
(c) 30° West of North (d) 60° East of North
relative vertical acceleration of A with respect to B?
9 A ship A is moving Westwards with a speed of 10 kmh −1
and a ship B, 100 km South of A is moving Northwards A
with a speed of 10 kmh −1. The time after which the
distance between them is shortest and the the shortest
B
distance between them are
(a) 0 h, 100 km (b) 5h, 50 2 km
(c) 5 2 h, 50 km (d) 10 2 h, 50 2 km
60° 30°
10 A particle is moving Eastwards with a velocity of 5 ms −1.
In 10s , the velocity changes to 5 ms −1 Northwards. The (a) 4.9 ms −2 in horizontal direction
average acceleration in this time is (b) 9.8 ms −2 in vertical direction
1 (c) zero
(a) ms −2 towards North-East
2 (d) 4.9 ms −2 in vertical direction
1
(b) ms −2 towards North 14 A particle is moving with velocity v = k (Y i+ X j), where k
2
(c) zero is a constant.The general equation for its path is
(d)
1
ms −2 towards North-West (a)Y = X 2 + constant (b)Y 2 = X + constant
2 (c) XY = constant (d)Y 2 = X 2 + constant
11 A boy can throw a stone upto a maximum height of 10 m. 15 The maximum range of a bullet fired from a toy pistol,
The maximum horizontal distance that the boy can throw mounted on a car at rest is R 0 = 40 m. What will be the
the same stone upto will be acute angle of inclination of the pistol for maximum range
(a) 20 2 m (b) 10 m when the car is moving in the direction of firing with
(c) 10 2 m (d) 20 m uniform velocity v = 20 ms −1, on a horizontal surface?
12 A water fountain on the ground sprinkles water all around (take,g = 10 ms−2 ) j
JEE Main (Online) 2013
it. If the speed of water coming out of the fountain is v, (a) 30° (b) 60°
the total area around the fountain that gets wet is (c) 75° (d) 45°

ANSWERS
SESSION 1 1 (d) 2 (a) 3 (b) 4 (b) 5 (b) 6 (a) 7 (b) 8 (d) 9 (c) 10 (c)
11 (b) 12 (b) 13 (c) 14 (c) 15 (a) 16 (c) 17 (d) 18 (c) 19 (a) 20 (d)
21 (c) 22 (d) 23 (a) 24 (a) 25 (c) 26 (b) 27 (b) 28 (d) 29 (c) 30 (a)
31 (b) 32 (d) 33 (a) 34 (a)

SESSION 2 1 (a) 2 (b) 3 (a) 4 (b) 5 (d) 6 (d) 7 (c) 8 (a) 9 (b) 10 (d)
11 (d) 12 (a) 13 (d) 14 (d) 15 (b)
DAY THREE SCALAR AND VECTOR 33

Hints and Explanations


SESSION 1 5 Work done in displacing the particle 10 As, s = u sinθt − 1 g t 2
W = F⋅ r = (5i$ + 3 $j + 2 k$ ) ⋅ (2 i$ − $j ) 2
1 Suppose angle between two vectors A
and B of equal magnitude is θ. Then, 3t 1
= 5 × 2 + 3 × (−1) + 2 × 0 = 10 − 3 So, 40 = 20 3 × − × 10 × t 2
angle between A and A − B will be = 7J 2 2
180° − θ θ or 5 t 2 − 30t + 40 = 0
or 90° − . 6 ( A × B) = (B × A ) (given)
2 2 or t 2 − 6t + 8 = 0 or t = 2 or 4
⇒ ( A × B) − (B × A ) = 0
Hence, this angle will depend on the The minimum time t = 2 s
angle between A and B or θ. or ( A × B) + (A × B) = 0
[Q(B × A ) = −( A × B)] 11 The two bodies will collide at the
B or 2( A × B) = 0 highest point if both cover the same
A
a ⇒ 2 AB sinθ = 0 vertical height in the same time. So,
or sinθ = 0 v 12 sin2 30° v 22
q =
A –B A– B [Q|A| = A ≠ 0, |B| = B ≠ 0] 2g 2g
⇒ θ = 0 or π v2 1
2 Ax = Ay = Az or = sin 30° = = 0.5
7 Let the ball strike the, nth step after t v1 2
Now, A = A2x + A2y + A2z = 3 A x second. Vertical distance travelled by
1 12 Let the gun be fired with a velocity
A 30 the ball = nh = gt 2 ...(i) u from the point O on the bird at B,
∴ Ax = = = 10 3 2
making an angle θ with the horizontal
3 3
Horizontal distance travelled by the ball direction. Therefore, the height of the
Similarly A y = A x = 10 3 unit nb aim of the person be at height BA (h )
= nb = ut or t =
u above the bird.
3 u = 3 i + 4 j; a = 0.4 $i + 03
. $j
So from Eq (i), we get A
v = u+a t 2
2u2 h
nh = g 
1 nb 
= 3 $i + 4 $j + (0.4 i$ + 03
. $j ) 10  or n =
2  u  gb 2
u
=3i + 4j + 4i + 3j
$ $ $ $
8 Refering to the figure,
= 7 $i + 7 $j O
q
B
Speed = 7 + 7 = 7 2 units
2 2 u
A B Here, horizontal range
4 Given A = 3 i$ + $j, B = 2 i$ − $j − 5k$ 40 cm u2 sin 2θ
= = 100
The unit vector in the normal direction g
is (500) 2 sin 2θ
200 m or = 100
A×B 10
n$ = ±
|A| |B| sin θ 100 × 10 1
or sin 2θ = = = sin 14 ′
200 (500) 2 250
$i $j k$ 200 = ut or t =
u or 2θ = 14 ′ or θ = 7 ′
Here, A × B = 3 1 0 2
40 1  200  7 π
2 −1 − 5 Also, = × 9.8   = × rad
100 2  u  60 180
On solving, u = 700 ms −1 arc
= − 5$i + 15$j − 5k$ As, angle =
radius
|A | = 32 + 12 = 10 9 Vertical component of the initial
AB
velocity, ∴ θ=
|B| = (2)2 + (−1)2 + (−5)2 = 30 u y = u sin30° OB

A ⋅B uy 80 or AB = θ × OB
cos θ = or u = = = 160 ms −1 7 π
|A ||B| sin30° 1 / 2 = × × (100 × 100) cm
T 2u sin 30° 2 × 80 60 180
5 1 t = = = = 4s
= − = 20 cm
10 3 2 3 4 4× g 4 × 10
11 3 13 The cannon ball will have the same
∴ sin θ = 1 − cos 2 θ = v x = u cos 30° = 160 ×
2 3 horizontal range for the angle of
2
projection θ and (90° − θ) . So,
−5$i + 15$j − 5k$ = 80 3 ms −1
u2 sin2 θ u2 cos 2 θ
∴ n$ = ± h1 = and h2 =
11 2g 2g
10 ⋅ 30 ⋅ ∴ v = v 2x + v 2y = u2x + (u y − gt ) 2
2 3 2
1  u2 sin θ cos θ  1 R2
($i − 3 $j + k$ ) = (80 3 )2 + (80 − 10 × 4) 2 h1 h2 =   = ×
=± 4 g  4 4
11 = 144.3 ms −1
or R = 4 h1 h2
= 145ms −1
34 40 DAYS ~ JEE MAIN PHYSICS DAY THREE

14 Horizontal component of the velocity at u2 sin 2θ 22 Given, speed of man v m = 4 km/h


18 As, range = 10 =
an angle 60° g
= Horizontal component of the velocity ⇒ u2 = 10 g A D
at an angle 45°. v
i.e. u cos 60° = v cos 45°
1 1 Wall
147 × = v × 45º v
2 2 1 km
4m 6m
147 vm β
or v = ms −1 10 m
2 ∴ u = 10ms −1
(as, g = 10ms )−2

Vertical component of u = u sin 60° 1 gx2 O vr


Y = x tan θ − B
147 3
= m 2 2v 20 cos 2 θ Speed of river, v r = 3 kmh −1
2
1 g × 16 Width of the river d = 1 km
Vertical component of v = v sin 45° = 4 tan 45° −
2 2 v 20 cos 2 45° Time taken by the man to cross the
147 1 river,
= × 1 10 × 16
2 2 = 4×1− Width of the river 1km
2 2 × 10 × 10 × 1 t = =
=
147
m Speed of the man 4 kmh −1
2 2
1 1
But v y = u y + at = 4 − 0.8 = 3.2 ≈ 3.6 m = h = × 60 = 15 min
4 4
147 147 3 19 Time of flight (T ) is 2t
∴ = − 9. 8 t Distance travelled along the river
2 2 2u sinθ = vr × t
147 ∴ T = 2t =
or 9. 8 t = ( 3 − 1) g 1 3
= 3 × = km =
3000
= 750 m
2 4 4 4
2
∴ t = 5.49 s = × U vertical
g 23 Let u be the velocity of the swimmer and
15 For a projectile fired with a velocity u v be the velocity of the river flow. Then
inclined at an angle θ with the 20 Let u be initial velocity of projection at 2d
horizontal. angle θ with the horizontal. t1 =
1 Then horizontal range, u2 − v 2
h = u sinθ t − g t 2
2 u2 sin 2θ d d 2 ud
R= and t 2 = + =
⇒ g t 2 − 2u sinθ t + 2h = 0 g u−v u + v u2 − v 2
u2 sin2 θ If t is the time taken by the swimmer to
2u sin θ ± 4 u2 sin2 θ − 8 gh and maximum height H =
∴ t = 2g swim a distance 2d in still water, then
2g
Given, R = 4 3H t =
2d
2 u sin θ + 4 u2 sin2 θ − 8 gh
⇒ t1 = u2 sin 2θ u2 sin2 θ u
2g ∴ =4 3
g 2g 2ud 2d 4d 2
t2 × t = 2 × = 2
2 u sin θ − 4 u2 sin2 θ − 8 gh ∴ 2sin θ cos θ = 2 3 sin2 θ u − v2 u u − v2
and t 2 =
2g cos θ t2
or = 3 From above, t 2 × t = t 12 or t = 1
2 u sinθ sin θ t2
∴ t1 + t2 = =T
g or cot θ = 3 = cot 30° 24 When the man is at rest w.r.t. to the
16 Kinetic energy at highest point, θ = 30° ground, the rain comes to him at an
1
( KE )H = mv 2 cos 2 θ = K cos 2 θ 21 Maximum height, angle 30° with the vertical. This is the
2 u2 sin2 45° u2 direction of the velocity of raindrops
K H = = = AC
= K (cos 60° ) 2 = 2g 4g w.r.t. to the ground.
4
Horizontal range, Here, v rg = velocity of the rain w.r.t. the
17 10 ms–1 u2 sin 2 × 45° u2
R= = ground.
g g
O 30° P vm g
X A
u
30°
10 m

H
45°
Ground a
O B vr m vr g
u2 sin 2θ R/2 C
OP = R =
g R v rm = v rg + v gm = v rg
∴ OC = R/2 = u2 / (2 g ) − v mg = v rg + (− v mg )
10 × sin(2 × 30° )
2
= AC u2 /4 g 1 v mg = velocity of the man w.r.t the
10 tan α = = 2 =
OC u /2 g 2 ground,
10 3 and v r m = velocity of the rain w.r.t.
= = 5 3 = 8. 66 m ∴ α = tan −1 (1 /2)
2 the man.
DAY THREE SCALAR AND VECTOR 35

Here, v rg sin 30° = v mg = 10 kmh −1 29 For a particle in uniform circular motion ( i$ + $j ) ⋅ i$ 1


10 34 cos θ = = = cos 45°
⇒ v rg = = 20 kmh −1 y |( $i + $j )| ⋅ |$i| 2
sin 30°
So, θ = 45°
25 Initial relative velocity of the express P (R, q) Hence, angle between $i and $j is 90°.
ac
train w.r.t. the passenger train,
uep = ue − u p = 30 − 5 = 25ms −1
Final relative velocity of the express O
x SESSION 2
train w.r.t. the passenger train, v ep = 0 1 F F cos θ = F F sin θ
(because express train comes to rest or tanθ = 1 or θ = 45°
relative to passenger train)
v2 ∴|F1 + F2 |
From first equation of motion, a = towards centre of circle
R = F 2 + F 2 + 2 F F cos 45°
v ep = uep − at ⇒ 0 = 25 − 4t 2
⇒ 4t = 25 ⇒ t = 6.25 s v = (2 +
∴ a = (− cos θ $i − sin θ $j ) 2) F
R
26 Total distance travelled by the boat in 2 v2 v2 2 Vertical distance covered by a projectile
h in still water = 8 + 8 = 16 km or a = − cos θ $i − sin θ $j
R R is given by
Therefore, speed of boat in still water, 1 2
16 30 Velocity of rain h = (u sinθ) t − gt …(i)
vb = = 8 km h −1 2
2 vr = − 3 $j (vertically downward) d = u cos θ × t
Effective velocity when boat moves Velocity of man d
upstream = v b − v w = 8 − 4 or t =
vm = 4 i$ u cos θ
= 4 km h −1 , From Eq. (i), we get
∴ Relative velocity of rain w.r.t. man
Therefore, time taken to travel from one d 1 d2
8 v = v − v = −3 $j − 4 i$ h = u sin θ × − g 2
end to other = = 2 h rm r m
u cos θ 2 u cos 2 θ
4 ∴ |vrm | = (−3)2 + (−4) 2 g
d
Effective velocity when boat moves or h =
= 5kmh −1 cos θ 2(d tan θ − h )
downstream = v b + v w = 8 + 4
= 12 km h −1 31 Horizontal range of the projectile, 3 We know that the range is
The time taken to travel 8 km distance 2u sin θ
u2 sin 2θ T =
8 2 R= g
= = h = 40 min g
12 3 According to the question, the range of
Total time taken = 2 h + 40 min u2 the projectile is the same for
R max = (θ = 45° )
= 2 h and 40 min g complementary angles,
27 Let v 1 be the velocity of the car and v 2 2u sin θ
The maximum height attained by the So, T1 =
be the velocity of the parcel. The projectile g
parcel is thrown at an angle θ from Q, it 2u sin (90° − θ)
u2 sin2 θ ⇒ T2 =
reaches the man at M. H = g
M 2g
Again, the range of the projectile is
If H = R, then
u2 sin 2θ
v2
u2 sin2 θ u2 sin 2θ R=
= g
2g g 2u sin θ 2u sin (90° − θ)
θ
Q A ∴ T1 T2 = ×
v1 sin2 θ g g
= 2sin θ cos θ
∴ cos θ =
v1
=
10
=
1
= cos 45° 2 2u2 sin 2θ 2R
= =
v 2 10 2 2 So, tanθ = 4 g2 g
So, θ = 45° ∴ θ = tan −1 (4) ∴ T1 T2 ∝ R
28 Given, s = t 3 + 5 32 To hit a target, the man should aim his 4 Initial velocity, u = i + 2 j ms −1
ds rifle at a point higher than the target as
∴ Speed v = = 6t Magnitude of velocity
dt the bullet suffers a vertical deflection
1
and rate of change of speed ( y = gt 2 ) due to acceleration due to u = (1)2 + (2)2 = 5ms −1
dv 2
at = = 6t gravity. Equation of trajectory of projectile
dt
∴ Tangential acceleration at t = 2s 33 OA = OC g X2
Y = X tan θ − (1 + tan2 θ)
2u2
a t = 6 × 2 = 12 ms −2 OA + OC is along OB (bisector) and its
 tanθ Y = 2 = 2
and at t = 2s, v = 3(2)2 = 12 ms −1 magnitude is  X 1 
∴ Centripetal acceleration, 2R cos 45° = R 2
10( X )2
v 2
144 (OA + OC ) + OB is along OB and its ∴ Y = X ×2− [1 + (2)2 ]
ac = = ms −2 2( 5)2
R 20 magnitude is
10( X 2 )
Net acceleration = a2c + a2t ≈ 14 ms −2 R 2 + R = R(1 + 2) =2X − (1 + 4) = 2 X − 5X 2
2× 5
36 40 DAYS ~ JEE MAIN PHYSICS DAY THREE

5 Angular momentum of the projectile, 9 Let the ships A and B be at positions as Range is maximum when projectile is
L = mv h r ⊥ = m(v cos θ)h shown in figure when the distance thrown at an angle of 45°.
where, h is the maximum height between them is shortest. u2 (10 2 )2
Thus, R max = = = 20 m
 v 2 sin2 θ  Relative velocity of B w.r.t. A is g 10
= m (v cos θ)  ;
 2g  v r = v 12 + v 22 = 102 + 102
12 Maximum range of water coming out of
mv 3 sin2 θ cos θ 3 mv 3 = 10 2 kmh −1 along BC the fountain,
L = = [Qθ = 30°]
2g 16 g The shortest distance between A and C v2
is d given by, d = AC = AB sin 45° Rm =
6 Since, x = αt and y = βt 3 3
g
∴ r = x i$ + y $j = αt 3 i$ + βt 3 $j N
∴Total area around fountain,
dr v4
Now, v= = α t 2 × 3 i$ + βt 2 × 3 $j A = πR2m = π 2
dt v1=10 kmh-1 A D g
W E
Thus,|v| = (3 α t ) + (3 βt )
2 2 2 2
d 90° 13 mg sinθ = ma
= 9 α2t 4 + 9 β2t 4 v2=10 kmh-1 45° C
∴ a = g sinθ
– v1=10 kmh-1 where, a is along the inclined plane.
= 3t 2
α +β2 2
B
∴ Vertical component of acceleration is
7 At the highest point of its flight, vertical S g sin2 θ.
component of velocity is zero and only 1 ∴ Relative vertical acceleration of A
= 100 × = 50 2 km with respect to B is
horizontal component is left which is 2
u x = u cos θ g
d 50 2 g (sin2 60° − sin2 30° ) = = 4.9 ms −2
Shortest time, t = = = 5h 2
Given, θ = 45° vr 10 2
u (in vertical direction)
∴ u x = u cos 45° =
2 10 v1 = + 5$i , v2 = + 5 $j 14 Given, velocity v = kY i + kX j
Hence, at the highest point kinetic ∆v = v2 − v1 = 5$j − 5$i dX dY
= kY , = kX
energy |∆ v |= 5 2 dt dt
2
1 1  u  |∆ v | 5 2 dY dY dt kX
E′ = mu2x = m   ∴ a= = =
1
ms −2 = × =
2 2  2 t 10 2 dX dt dX kY
1  u2  E Q 1 mv 2 = E  YdY = XdX ;
= m  =   N y
2 2 2  2  Dv Y2 = X 2 + C
v2 where, c = constant.
8 Let the swimmer starts swimming with x 2
velocity v along AC in a direction
15 According to question, u = 40
making an angle θ with AB as shown in g
W E
the figure. If d is the width of the river, – v1 v1 2u sin 45°
−1
∴ u = 20 ms and T =
time taken by the swimmer to cross the g
–v2
river will be
1 1
C B S = 2 × 20 × ×
2 10
For direction, 4
5 = =2 2
v d tan α = −= −1 2
q 5
1 When car is moving with speed,
Average acceleration is ms −2 towards
2 v = 20 ms −1 , then
A vr
North-West.
t = d / v cos θ (v cos θ + 20) × t = 40
As component of AB will be v cos θ, This 11 Maximum speed with which the boy
can throw stone is (20cos θ + 20) × 2 2 = 40
time will be minimum, when
cos θ = max = 1, i.e. θ = 0°. u = 2gh = 2 × 10 × 10 θ 1
⇒ cos 2 = ≈ 60°
So, the swimmer should swim in North 2 2 2
direction. = 10 2 ms −1 .
DAY FOUR

Laws
of Motion
Learning & Revision for the Day

u Concept of Forces u Principle of Conservation of u Connected Motion


u Inertia Linear Momentum u Equilibrium of concurrent Forces
u Newton’s Laws of Motion u Free Body Diagram u Friction

Concept of Forces
A push or a pull exerted on any object, is defined to be a force. It is a vector quantity.
Force can be grouped into two types:
l
Contact forces are the forces that act between two bodies in contact, e.g. tension,
normal reaction, friction etc.
l
Non-contact forces are the forces that act between two bodies separated by a distance
without any actual contact. e.g. gravational force between two bodies and electrostatic
form between two charges etc.

Inertia
The inability of a body to change by itself its state of rest or state of uniform motion along
a straight line is called inertia of the body.
As inertia of a body is measured by the mass of the body. Heavier the body, greater the PREP
force required to change its state and hence greater is its inertia. There are three type of
inertia (i) inertia of rest (ii) inertia of motion (iii) inertia of direction.
MIRROR
Your Personal Preparation Indicator

Newton’s Laws of Motion u No. of Questions in Exercises (x)—


u No. of Questions Attempted (y)—
First Law of Motion (Law of Inertia) u No. of Correct Questions (z)—
(Without referring Explanations)
It states that a body continues to be in a state of rest or of uniform motion along a straight
line, unless it is acted upon by some external force the change the state. This is also u Accuracy Level (z / y × 100)—
called law of inertia. u Prep Level (z / x × 100)—
If F = 0, ⇒ v = constant ⇒ a = 0
l
This law defines force. In order to expect good rank in JEE,
your Accuracy Level should be above
l
The body opposes any external change in its state of rest or of uniform motion. 85 & Prep Level should be above 75.
l
It is also known as the law of inertia given by Galileo.
38 40 DAYS ~ JEE MAIN PHYSICS DAY FOUR

Linear Momentum Applications of Conservation


It is defined as the total amount of motion of a body and is
measured as the product of the mass of the body and its
of Linear Momentum
velocity. The momentum of a body of mass m moving with a The propulsion of rockets and jet planes is based on the
velocity v is given by p = mv. principle of conservation of linear momentum.
u dm
Its unit is kg-ms–1 and dimensional formula is [ML T –1] l
Upward thrust on the rocket, F = − − mg and if effect
dt
u dm
of gravity is neglected, then F = − .
Second Law of Motion dt
The rate of change of momentum of a body is directly l
Instantaneous upward velocity of the rocket
proportional to the applied force and takes place in the m 
v = u ln  0  − gt
direction in which the force acts.  m
dp dp and neglecting the effect of gravity
According to second law, F ∝ or F = k
dt dt m  m 
v = u ln  0  = 2.303 u log 10  0 
where, k is constant.  m  m
dp d m dv where, m0 = initial mass of the rocket including that of the
as, = (mv) = ma or = ma
dt dt dt fuel,
i. e., second law can be written as u = initial velocity of the rocket at any time t ,
dp m = mass of the rocket left,
F = = ma
dt v = velocity acquired by the rocket,
The SI unit of force is newton (N) and in CGS system is dyne. dm
= rate of combination of fuel.
1 N = 10 5 dyne dt
l
Burnt out speed of the rocket is the speed attained by the
Impulse rocket when the whole of fuel of the rocket has been burnt.
Burnt out speed of the rocket
Impulse received during an impact is defined as the product
of the average force and the time for which the force acts. m  m 
vb = u loge  0  = 3.303 u log 10  0 
Impulse, I = Fav t  mr   mr 
Impulse is also equal to the total change in momentum of the
body during the impact. Apparent Weight of a Boy in a Lift
Impulse, I = p2 − p1 Actual weight of the body is mg. Here we consider the
apparent weight of a man standing in a moving lift.
Impulse = Change in momentum
l
If lift is accelerating upward with acceleration a, then
apparent weight of the body is R = m(g + a).
Third Law of Motion l
If lift is accelerating downward at the rate of acceleration a,
To every action, there is an equal and opposite reaction. then apparent weight of the body is R = m(g − a)
F12 = − F21 l
If lift is moving upward or downward with constant
l
Action and reaction are mutually opposite and act on two velocity, then apparent weight of the body is equal to
different bodies. actual weight.
l
The force acting on a body is known as action.
l
If the lift is falling freely under the effect of gravity, then it
is called weightlessness condition.
l
When a forc acts on a body, then the reaction acts normally
to the surface of the body.
Free Body Diagram
Principle of Conservation A free body diagram (FBD) consists of a diagramatize
representation of a single body or sub-system of bodies
of Linear Momentum isolated from its surroundings showing all forces acting on it.
It states that if no external force is acting on a system, the While sketching a free body diagram the following points
momentum of the system remains constant. should be kept in mind.
dp Normal reaction (N) always acts
According to second law of motion, F =
l

A B
dt normal to the surface on which the
If no force is acting, then F = 0 body is kept.
dp When two objects A and B are connected by a string, the
∴ = 0 ⇒ p = constant
l

dt tension for object A is towards B and for object B, it is


or m1v 1 = m2 v2 = constant towards A.
DAY FOUR LAWS OF MOTION 39

Tension in the string,


Connected Motion F (m3 + m2 )
If two blocks of masses m1 and m2 are placed on a perfectly T1 = (m2 + m3 )a =
(m1 + m2 + m3 )
l

smooth surface and are in contact, then


F m3
A B and tension T2 = m3 a =
m1 + m2 + m3
F f f
m1 m2
l
For a block system shown in the figure, acceleration of the
system
F F
Acceleration of the blocks, a = a=
m1 + m2 m1 + m2 + m3
and the contact force (acting normally) between the two
F m2 T1 T2 m3
blocks is f = m2 a = . m1 m2 F
(m1 + m2 )
l
A block system is shown in the figure Tension in the string,
m1F
m2 F T1 =
m1 m1 + m2 + m3
F (m1 + m2 )F
Acceleration of the blocks a = and T2 =
m1 + m2 m1 + m2 + m3
Contact force between two blocks
m1F
l
For a block system suspended freely from a
f = m1a = rigid support as shown in the figure, the T1
(m1 + m2 )
acceleration of the system a = 0.
m1 A
l
For a block of mass m placed on a fixed, perfectly smooth String tension,
inclined plane of angle θ, the forces acting on the block are T1 = (m1 + m2 + m3 )g T2
as shown in the figure. Obviously, here a = g sin θ m2 B
N
T2 = (m2 + m3 )g
m F and T3 = m3 g T3

a m3 C

θ θ
sin
mg mg cos θ
θ mg l
For a block system and a pulley as shown in the figure,
value of the acceleration of the system
l
If a block of mass m is placed on a smooth movable wedge of
mass M, which in turn is placed on smooth surface, then a (m + m2 − m3 )g
a= 1 T
force F is applied on the wedge, horizontally. (m1 + m2 + m3 )
The acceleration of the wedge and the block is 2 m1m3 g
Tension, T1 =
N cos θ (m1 + m2 + m3 ) T2
T2
a
N 2 m3 (m1 + m2 )g
θ Tension, T2 =
m (m1 + m2 + m3 ) C m3 m2 B

N sin θ and tension T


ma T1 a
F
4 m3 (m1 + m2 )g
T = 2 T2 =
(m1 + m2 + m3 ) m1
A
θ mg M
F
l
For the pulley and block arrangement as shown in the
a= figure, we have
(M + m)
Force on the block, F = (M + m)a = (M + m)g tan θ T2 T1
M
l
For a block system shown in the figure, acceleration of the
system
F T2
T1
a=
m1 + m2 + m3 a
a
m2 (m 1 > m 2 )
A B C m1
m1 m2 m3
F T1 T2
40 40 DAYS ~ JEE MAIN PHYSICS DAY FOUR

Net acceleration, Lami Theorem For three concurrent forces in equilibrium


Net accelerating force (m1 − m2 )g position.
a= = P R
Total mass (m1 + m2 + M ) β
(M + 2 m2 )m1g γ α
Tension, T1 = m1(g − a) =
(M + m1 + m2 )
(M + 2 m1)m2 g
and Tension, T2 = m2 (g + a) =
(M + m1 + m2 )
Q
l
For the system of block and pulley, with a smooth inclined P Q R
= =
plane as shown in the figure, we have sin α sin β sin γ

N a
T Friction
m2 T
Whenever an object actually slides or rolls over the surface of
θ
in another body or tends to do so, a force opposing the relative
gs
m2 θ a motion starts acting between these two surfaces in contact. It
m2 g cos θ m1
θ m2g is known as friction or the force due to friction. Force of
friction acts in a tangential direction to the surfaces in contact.
Net acceleration,
(m − m2 sin θ)g
a= 1 , if m1g > m2 g sin θ Types of Friction
(m1 + m2 )
The four types of friction are given below
(m2 sin θ − m1)g
and a= , if m1g < m2 g sin θ 1. Static Friction It is the opposing force that comes into
m1 + m2
play when one body is at rest and a force acts to move it
and tension in the string over the surface of another body.
m m (1 + sin θ) It is a self adjusting force and is always equal and opposite
T = m1(g − a) = 1 2
(m1 + m2 ) to the applied force.
l
For a pulley and block system on a smooth double inclined 2. Limiting Friction It is the limiting (maximum) value of
plane as shown in the figure, we have static friction when a body is just on the verge of starting
its motion over the surface of another body.
N
a T N
T
N
m2
m1
Applied
θ1 force F
in a
gs
m2

m1
g

f = µN
sin

m1g cos θ1 m2g cos θ2 θ2


θ1
θ2

(m2 sin θ2 − m1 sin θ 1)g


Net acceleration, a = , The force of limiting friction f l between the surfaces of
(m1 + m2 ) two bodies is directly proportional to the normal reaction
for θ2 > θ 1, m2 > m1 at the point of contact. Mathematically,
and tension in the string, fl ∝ N or fl = µ l N
m1m2 (sin θ 1 + sin θ2 )g f
T = ⇒ µl = l
(m1 + m2 ) N
where, µ l is the coefficient of limiting friction for the
given surfaces in contact.
Equilibrium of Concurrent Forces 3. Kinetic Friction It is the opposing force that comes into
If a number of forces act at the same point, they are called play when one body is actually slides over the surface of
concurrent forces. another body. Force of kinetic friction fk is directly
The necessary condition for the equilibrium of a body under f
proportional to the normal reaction N and the ratio k is
the action of concurrent forces is that the vector sum of all the N
forces acting on the body must be zero. called coefficient of kinetic friction µ k , value of µ k is
Mathematically for equilibrium, slightly less than µe (µ k < µ l ).
Σ Fnet = 0 or ΣFx = 0, ΣF y = 0 and ΣFz = 0 Whenever limiting friction is converted into kinetic
friction, body started motion with a lurch.
DAY FOUR LAWS OF MOTION 41

4. Rolling Friction It is the opposing force that comes into In limiting condition, f1 = mg sin φ
play when a body of symmetric shape (wheel or cylinder ⇒ N = mg cos φ
or disc, etc.) rolls over the surface of another body. Force
f1
of rolling friction fr is directly proportional to the normal = tan φ
reaction N and inversely proportional to the radius (r ) of N
the wheel. f1
∴ = µ s = tan φ
N N N
Thus, fr ∝ or fr = µ r
r r
The constant µ r is known as the coefficient of rolling Acceleration of a Block on
friction µ r has the unit and dimensions of length.
Applying a Force on a Rough Surface
Magnitudewise µ r << µ k or µ l .
l
Acceleration of a block on a horizontal surface is as shown
l
The value of rolling friction is much smaller than the in the figure.
value of sliding friction.
N
l
Ball bearings are used to reduce the wear and tear and
energy loss against friction.
f = µN F
Angle of Friction
Angle of friction is defined as the angle θ which the resultant R
mg
of the force of limiting friction fl and normal reaction N,
subtends with the normal reaction. F − f F − µmg
The tangent of the angle of friction is equal to the coefficient a= =
m m
of friction. i.e. µ = tan θ F
or a= − µg
R N m
where, µ = coefficient of kinetic friction between the two
surfaces in contact.
θ
fl l
Acceleration of block sliding down a rough inclined plane
Applied force F as shown in the figure is given by
a = g(sin α − µ cos α )
Angle of Repose N
µN
Angle of repose is the least angle of the inclined plane f=
(of given surface) with the horizontal such that the given body ma
placed over the plane, just begins to slide down, without α α
getting accelerated. sin
mg mg mg cos α
N α
fl
l
Retardation of a block sliding up a rough inclined plane as
φ φ shown in the figure is a = g(sin α + µ cos α )
sin
mg mg mg cos φ
φ N
F
The tangent of the angle of repose is equal to the coefficient of
friction. ma
α α
Hence, we conclude that angle of friction (θ) is equal to the sin
mg µN mg mg cos α
angle of repose (φ). α
42 40 DAYS ~ JEE MAIN PHYSICS DAY FOUR

DAY PRACTICE SESSION 1

FOUNDATION QUESTIONS EXERCISE


1 Five forces inclined at an angle of 72° w.r.t. each other
act on a particle of mass m placed at the origin. Four
(a) F1 (b) F1 F3
forces are of magnitude F1 and one has a magnitude F2.
F3
Find the resultant acceleration of the particle.
F2 − F1
(a) (b) Zero F2 F2
m
F2 + F1 F2 − 4F1 F1
(c) (d)
m m
(c) F2 (d) F1 F3
2 A ball of mass 0.2 kg is thrown vertically upwards by F3
applying a force by hand. If the hand moves by 0.2 m
while applying the force and the ball goes upto 2 m
F2
height further, find the magnitude of the force.
(take, g = 10 ms −2) 8 A string of negligible mass, going over a clamped pulley
(a) 4 N (b) 16 N (c) 20 N (d) 22 N of mass m supports a block of mass M as shown in the
3 A player catches a cricket ball of mass 150 g, moving at figure. The force on the pulley by the clamp is given by
a rate of 20 ms −1. If the catching process is completed in m
0.1 s, the force of the blow exerted by the ball on the
hand of the player is equal to
(a) 150 N (b) 3 N (c) 30 N (d) 300 N
4 A ball of mass m is thrown vertically upwards with a
velocity v. If air exerts an average resisting force F , the
M
velocity with which the ball returns to the thrower is
mg F
(a) v (b) v
mg + F mg + F (a) 2Mg (b) [(M + m)2 + m 2 ] g
mg − F mg + F (c) 2Mg (d) [(M + m) + m]2 g
(c) v (d) v
mg + F mg
9 A bullet is fired from a gun. The force on the bullet is
5 A block of mass 200 g is moving with a velocity of 5 ms −1 given by F = ( 600 − 2 × 10 5t ), where F is in newton and t
along the positive x-direction. At time t = 0, when the is in second. The force on the bullet becomes zero as
body is at x = 0, a constant force 0.4 N is directed along soon as it leaves the barrel. What is the average impulse
the negative x-direction , is applied on the body for 10 s. imparted to the bullet?
What is the position x of the body at t = 2.5 s?
(a) 9 N-s (b) Zero (c) 0.9 N-s (d) 1.8 N-s
(a) x = 6.75 m (b) x = 6.25 m
(c) x = 6 m (d) x = 6.50 m 10 Two wooden blocks are moving on a smooth horizontal
6 Two trains A and B are running in the same direction on surface, such that the mass m remains stationary with
parallel tracks such that A is faster than B. If packets of respect to the block of mass M as shown in the figure.
equal weight are exchanged between the two, then The magnitude of force P is
(a) A will be retarded but B will be accelerated
(b) A will be accelerated but B will be retarded
(c) there will not be any change in the velocity of A but B m
P
will be accelerated
M
(d) there will not be any change in the velocity of B, but A
will be accelerated b
7 Which of the four arrangements in the figure correctly
shows the vector addition of two forces F1 and F2 to yield (a) g tan β (b) mg cos β
(c) (M + m) cosec β (d) (M + m) g tan β
the third force F3?
DAY FOUR LAWS OF MOTION 43

11 The figure below shows a uniform rod of length 30 cm 18 A satellite in a force-free space sweeps out stationary
dM
having a mass of 3.0 kg. The strings as shown in the interplanetary dust at a rate = αv , where M is the
figure are pulled by constant forces of 20 N and 32 N. dt
Find the force exerted by the 20 cm part of the rod on mass and v is the velocity of the satellites and α is a
constant. The deceleration of the satellite is
the 10 cm part. All the surfaces are smooth and the
2 αv 2 αv 2 αv 2
strings are light (a) (b) − (c) − (d) −αv 2
M M 2M
F F
19 A lift is moving down with an acceleration a. A man in the
lift drops a ball inside the lift. The acceleration of the ball
20 N 10 cm 20 cm 32 N as observed by the man in the lift and a man standing
(a) 36 N (b) 12 N (c) 64 N (d) 24 N stationary on the ground are respectively
(a) g, g (b) a, a (c) (g − a), g (d) a, g
12 Two masses m1 = 5 kg and m2 = 4.8 kg, tied
to a string, are hanging over a light 20 A spring balance is attached to the ceiling of a lift. A man
hangs his bag on the string and the balance reads 49 N,
frictionless pulley. What is the acceleration of
when the lift is stationary. If the lift moves downwards
the masses produced when system is free to with an acceleration of 5 ms −2 , the reading of the spring
move? (take, g = 9.8 ms −2 ) m1 balance would be
(a) 0.2 ms −2 (b) 9.8 ms −2 (a) 24 N (b) 74 N (c) 15 N (d) 49 N
(c) 5 ms −2 (d) 4.8 ms −2 m2
21 A block A is able to slide on the frictionless incline of
13 A light string passing over a smooth light angle θ and length l, kept inside an elevator going up
pulley, connects two blocks of masses m1 and m 2 with uniform velocity v. Time taken by the block to slide
(vertically). If the acceleration of the system is ( g/8), then down the length of the incline, if released from rest is
the ratio of masses is
(a) 8 : 1 (b) 9 : 7 (c) 4 : 3 (d) 5 : 3
v
14 Two bodies of equal masses are connected by a light
inextensible string passing over a smooth frictionless
pulley. The amount of mass that should be transferred
q
from one to another, so that both the masses move 50 m
in 5 s is 2l 2l
(a) (b)
(a) 30% (b) 40% (c) 70% (d) 50% (g + a) sin θ g
15 A man slides down a light rope, whose breaking strength 2l 2l
(c) (d)
is η times his weight. What should be his maximum g sin θ sinθ
acceleration, so that the rope does not break?
g g 22 A plane is inclined at an angle θ with the horizontal. A
(a) g(1 − η) (b) ηg (c) (d)
1+ η 1− η body of mass m rests on it. If the coefficient of friction is
µ, then the minimum force that has to be applied parallel
16 Two blocks of mass M1 = 20kg and M 2 = 12 kg 480 N
to the inclined plane, so as to make the body to just
are connected by a metal rod of mass 8 kg. The
M1 move up the inclined plane, is
system is pulled vertically up by applying a force
of 480 N as shown in figure. The tension at the (a) mg sinθ (b) µ mg cosθ
(c) µ mg cosθ − mg sinθ (d) µ mg cosθ + mg sinθ
mid-point of the rod is
M2 23 A 40 kg slab rests on a frictionless floor. A 10 kg block
ª JEE Main (Online) 2013
rests on the top of the slab. The static coefficients of
(a) 144 N (b) 96 N (c) 240 N (d) 190 N friction between the block and the slab is 0.60, while the
kinetic coefficient is 0.40.
17 Two blocks of masses m and M are The 10 kg block is acted upon by a horizontal force of
connected by means of a metal wire of 100 N. If g = 9.8 ms −2 , the resultant acceleration of the
cross-sectional area A passing over a slab will be
frictionless fixed pulley as shown in the T
T 10 kg 100 N
figure. The system is then released. If
m
M = 2 m , then the stress produced in the
M 40 kg
wire is ª JEE Main (Online) 2013

(a)
2 mg
(b)
4 gm
(c)
gm
(d)
3 mg (a) 0.98 ms −2 (b) 1.47 ms −2
3A 3A A 4A (c) 1.52 ms −2 (d) 6.1 ms −2
44 40 DAYS ~ JEE MAIN PHYSICS DAY FOUR

24 A horizontal force of 10 N is necessary to f coefficient of friction between the blocks is 0.1 and
just hold a block stationary against a wall. between block B and the wall is 0.15, the frictional force
10 N
The coefficient of friction between the applied by the wall in block B is ª JEE Main 2015
block and the wall is 0.2. The weight of
the block is F A B
(a) 20 N (b) 50 N
(c) 100 N (d) 2 N
25 What is the maximum value of the force F , such that the 20 N 100 N
block as shown in the arrangement, does not move?
F m = Ö3 kg
1 (a) 100 N (b) 80 N (c) 120 N (d) 150 N
60° m=
2Ö3 31 A point particle of mass m, moves along the uniformly
rough track PQR as shown in the figure. The coefficient
(a) 20 N (b) 10 N (c) 12 N (d) 15 N of friction between the particle and the rough track
equals µ . The particle is released, from rest , from the
26 A block of mass M is held against a rough vertical wall
point P and it comes to rest at a point R. The energies,
by pressing it with a finger. If the coefficient of friction
lost by the ball, over the parts, PQ and QR , of the track,
between the block and the wall is µ and the acceleration
are equal to each other, and no energy is lost when
due to gravity is g, then minimum force required to be
particle changes direction from PQ to QR. The values of
applied by the finger to hold the block against the wall?
the coefficient of friction µ and the distance x(= QR), are
Mg Mg 2M 2 Mg
(a) (b) (c) (d) respectively close to
2µ µ µg µ
P
27 A wooden block of mass M resting on a rough horizontal
surface, is pulled with a force F at an angle with the h=2m
horizontal. If µ is the coefficient of kinetic friction between
30º R
block and the surface, then acceleration of the block is O Q
F F Horizontal
(a) (cos φ + µ sin φ) − µg (b) sin φ surface
M M ª JEE Main 2016 (Offline)
(c) µ F cos φ (d) µ F sinφ
(a) 0.2 and 6.5 m (b) 0.2 and 3.5 m
28 A block of mass 10 kg is placed at a distance of 5 m (c) 0.29 and 3.5 m (d) 0.29 and 6.5 m
from the rear end of a long trolley as shown in the figure. 32 Two masses m1 = 5 kg and m2 = 10 kg connected by an
The coefficient of friction between the block and the inextensible string over a frictionless pulley, are moving
surface below is 0.2. Starting from rest, the trolley is as shown in the figure. The coefficient of friction of
given an uniform acceleration of 3 ms −2. At what horizontal surface is 0.15. The minimum weight m that
distance from the starting point will the block fall off the should be put on top of m2 to stop the motion is
trolley? (take, g = 10 ms −2) ª JEE Main 2018
5m
m
T
f F Rear m2
end
a=3 ms–2
Trolley
T

(a) 15 m (b) 25 m (c) 20 m (d) 10 m m1

29 A body starts from rest on a long inclined plane of slope m 1g


45°. The coefficient of friction between the body and the
(a) 18.3 kg (b) 27.3 kg (c) 43.3 kg (d) 10.3 kg
plane varies as µ = 0.3 x , where x is distance travelled
down the plane. The body will have maximum speed (for 33 The minimum force required to start pushing a body up a
g = 10 m s −2) when x is equal to ª JEE Main (Online) 2013 rough (frictional coefficient µ) inclined plane is F1 while
the minimum force needed to prevent it from sliding
(a) 9.8 m (b) 27 m (c) 12 m (d) 3.33 m
down is F2. If the inclined plane makes an angle θ from
30 Given in the figure are two blocks A and B of weight 20 N F
the horizontal such that tan θ = 2 µ, then the ratio 1 is
and 100 N, respectively. These are being pressed F2
against a wall by a force F as shown in figure.If the (a) 4 (b) 1 (c) 2 (d) 3
DAY FOUR LAWS OF MOTION 45

34 A box of mass 80 kg kept on a horizontal weighing choices, only one of which is the correct answer. You have to
machine of negligible mass, attached to a massless select one of the codes (a), (b), (c), (d) given below
platform P that slides down at 37° incline. The weighing (a) Statement I is true, Statement II is true, Statement II is
machine read 72 kg. Box is always at rest w.r.t. weighing the correct explanation for Statement I
machine. Then, coefficient of friction between the (b) Statement I is true, Statement II is true, Statement II is
platform and incline is not the correct explanation for Statement I
(c) Statement I is true, Statement II is false
P (d) Statement I is false, Statement II is true
36 Statement I When the car accelerates horizontally along
a straight road, the accelerating force is given by the
push of the rear axle on the wheels.
37°
Statement II When the car accelerates, the rear axle
12 13 3 11 rotates with a greater frequency.
(a) (b) (c) (d)
13 24 5 24
37 Statement I It is easier to pull a heavy object than to
35 A block of mass m is placed against a vertical surface by push it on a level ground.
a spring of unstretched length l. If the coefficient of Statement II The magnitude of frictional force depends
friction between the block and the surface is µ, then on the nature of the two surfaces in contact.
choose the correct statement.
38 Statement I A cloth covers a table. Some dishes are
2mg
(a) If spring constant k = , kept on it. The cloth can be pulled out without dislodging
µd d the dishes from the table.
block will not be in equilibrium.
m Statement II For every action there is an equal and
(b) Minimum spring constant kmin to
keep the block of mass m in m
opposite reaction.
mg 39 Statement I A bullet is fired from a rifle. If the rifle recoils
equilibrium is .
µd l freely, the kinetic energy of the rifle is less than that of the
2mg bullet.
(c) If spring constant is k = ,
µd Statement II In the case of a rifle-bullet system, the law
mg of conservation of momentum is violated.
the normal reaction is .
µ 40 Statement I Newton’s second law is applicable on a
(d) In the part (c), force of friction is 2mg. body with respect to an inertial frame of reference.
Statement I In order to apply Newton’s second law on a
Direction (Q. Nos. 36-40) Each of these questions contains body observed from a non-inertial frame of reference. We
two statements : Statement I (Assertion) and Statement II apply line pseudo force an imaginary force.
(Reason). Each of these questions also has four alternative

DAY PRACTICE SESSION 2

PROGRESSIVE QUESTIONS EXERCISE


1 A flexible uniform chain of mass m and length l
suspended vertically, so that its lower end just touches A
the surface of a table. When the upper end of the chain
is released, it falls with each link coming to rest the
instant it strikes the table. The force exerted by the chain B
on the table at the moment when y part of the chain has
already rested on the table is
3myg 3mg 2 mg 1 mg
(a) (b) (c) (d) 60° 30°
l l 3 l 3 l
−2
2 Two fixed frictionless inclined plane making the angles (a) 4.9 ms in horizontal direction
30° and 60° with the vertical are shown in the figure. Two (b) 9.8 ms−2 in vertical direction
(c) zero
blocks A and B are placed on the two planes. What is the
(d) 4.9 ms−2 in vertical direction
relative vertical acceleration of A with respect to B?
46 40 DAYS ~ JEE MAIN PHYSICS DAY FOUR

3 If M is the mass of a rocket, r is the rate of ejection of  g sin2 θ   g cos2 θ 


(a)  2 
(b)  2 
gases with respect to the rocket, then acceleration of the  1+ sin θ   1+ cos θ 
rocket,
dv
is equal to  g tan2 θ   g sin2 θ 
(c)  2 
(d)  
dt
 1+ tan θ   1+ sinθ 
ru (M − r t ) ru ru
(a) (b) (c) (d)
(M − r t ) ru (M + r t ) M 9 A carriage of mass M and length l is joined to the end of
a slope as shown below. A block of mass m is released
4 A person 40 kg is managing to be at rest between two from the slope from height h. It slides till end of the
vertical walls by pressing one wall A by his hands and carriage. The coefficient of friction between block and
feet and B with his back. The coefficient of friction is 0.8 carriage is µ (the friction between other surfaces are
between his body and the wall. The force with which the negligible). Then, minimum height h is
person pushes the wall is m
(a) 100 N (b) 50 N (c) 150 N (d) 200 N
h
5 An insect crawls up a hemispherical surface very slowly.
The coefficient of friction between the insect and the M
surface is 1/3. If the line joining the centre of the
Smooth
hemispherical surface to the insect makes an angle α
with the vertical, the maximum possible value of α is
(a) µ  1 +  l (b) µ  2 +  l
M m
given by  m  M
 m
(c) µ  1 +  l (d) 2 µ  1 +  l
m
a
 M  M
10 System as shown in figure is in equilibrium
(a) cotα = 3 (b) sec α = 3 and at rest. The spring and string are
(c) cosec α = 3 (d) None of these massless, now the string is cut. The
6 A given object takes n times more time to slide down a acceleration of the masses 2 m and m just
45° rough inclined plane in comparison to slides down a after the string is cut, will be 2m
perfectly smooth 45° incline. The coefficient of kinetic g
(a) upwards, g downwards
friction between the object and the incline is 2
g
(b) g upwards, downwards
1 1 1 1 2
(a) (b) 1 − (c) 1− (d) m
1 − n2 n2 n2 1 − n2 (c) g upwards, 2g downwards
(d) 2g upwards, g downwards
7 When a body slides down from rest along a smooth
11 A block of mass m is at rest under the
inclined plane making an angle of 45° with the
action of a force F , acting against a wall,
horizontal,it takes timeT . When the same body slides as shown in the figure. Which of the F
down from rest along a rough inclined plane making the following statement is incorrect?
same angle and through the same distance,it is seen to (a) f = mg (where, f is the frictional force)
take time pT , where p is some number greater than 1. (b) F = N (where, N is the normal force)
The coefficient of friction between the body and the (c) F will not produce torque
rough plane will be (d) N will not produce torque
1
(a) 1 − p 2 (b) 1 − (c) p 2 − 1 (d) p 2 12 The upper-half of an inclined plane with an inclination φ, is
p2
perfectly smooth, while the lower half is rough. A body
8 Block A of mass m is placed over a wedge of the same starting from rest at the top will again come to rest at
mass m. Both the block and wedge are placed on a fixed the bottom, if the coefficient of friction for the lower half is
inclined plane. Assuming all surfaces to be smooth. given by
Then, displacement of the block A in ground frame in (a) 2 sin φ (b) 2 cos φ (c) 2 tan φ (d) tan φ
1 s is
13 A block of mass m is placed on a surface with a vertical
A cross-section given by y = x 3/ 6. If the coefficient of
B
friction is 0.5, the maximum height above the ground at
Fixed which the block can be placed without slipping is
incline
ª JEE Main 2014
q
(a) 1 / 6 m (b) 2 / 3 m (c) 1 / 3 m (d) 1 / 2 m
DAY FOUR LAWS OF MOTION 47

14 A particle of mass m is at rest at the origin at time t = 0. It (a)T = 2250N, F = 1125 N


is subjected to a force F( t ) = F0 e −bt in the x -direction. Its (b)T = 1125 N, F = 2250N
speed v ( t ) is depicted by which of the following curves? (c)T = 1125 N, F = 375 N
(d)T = 1125 N, F = 750N
ª JEE Main 2013
F0 F0
17 In the system shown in figure, masses of the blocks are
mb mb such that when system is released, acceleration of pulley
P1 is a upwards and acceleration of block 1 is a1,
(a) (b) upwards. It is found that acceleration of block 3 is same
as that of 1 both in magnitude and direction.
v (t) v (t)
t t

F0 F0
mb mb
P1 P2
a
(c) (d)

v (t) v (t) a1 1 3
t t

2 4
15 The figure shows the position-time ( x -t ) graph of a
one-dimensional motion of a body of mass 0.4 kg. The Given that, a1 > a > 1.
2
magnitude of each impulse is
Match the following.
2
Column I Column II
x (m) A. Acceleration of 2 1. 2 a + a1
B. Acceleration of 4 2. 2 a − a1
0 2 4 6 8 10 12 14 16 C. Acceleration of 2 w.r.t.3 3. Upwards
t (s)
D. Acceleration of 2 w.r.t. 4 4. Downwards
(a) 0.4N - s (b) 0.8 N - s
(c) 1.6N - s (d) 0.2 N - s Codes
16 A worker is raising himself and the crate A B C D
on which he stands with an acceleration (a) 2,3 1,4 4 3
of 5 ms −2 by a massless rope and pulley (b) 2,4 1 4 1,3
arrangement. Mass of the worker is 100 (c) 4 1,3 2 1,2
kg and that of the crate is 50 kg. If T is (d) No above matching is correct
the tension in the rope and F be the force
of contact between the worker and the
floor and if g = 10 ms −2, then

ANSWERS

SESSION 1 1 (a) 2 (d) 3 (c) 4 (c) 5 (b) 6 (a) 7 (a,c) 8 (b) 9 (c) 10 (d)
11 (d) 12 (a) 13 (b) 14 (b) 15 (a) 16 (d) 17 (b) 18 (b) 19 (c) 20 (a)
21 (c) 22 (d) 23 (a) 24 (d) 25 (a) 26 (b) 27 (a) 28 (a) 29 (d) 30 (c)
31 (c) 32 (b) 33 (d) 34 (b) 35 (b) 36 (a) 37 (b) 38 (b) 39 (c) 40 (a)

SESSION 2 1 (a) 2 (d) 3 (a) 4 (d) 5 (a) 6 (b) 7 (b) 8 (a) 9 (c) 10 (a)
11 (d) 12 (c) 13 (a) 14 (c) 15 (b) 16 (c) 17 (a)
48 40 DAYS ~ JEE MAIN PHYSICS DAY FOUR

Hints and Explanations


SESSION 1 ⇒ F × 01. = |p f − pi | reaching train A being of lower
As the ball will stop after catching, momentum will retard the train A but
1 According to polygon law, resultant of
four forces, each of magnitude F1 acting p i = mv i = 015
. × 20 = 3 and p f = 0 packet reaching train B, being of higher
at an angle of 72°, is along the fifth side ⇒ F × 01. =3 momentum will accelerate B.
of the polygon taken an in opposite ⇒ F = 30 N 7 Parallelogram law of vector addition
order. As F2 is acting along this side of 4 For an upward motion F3 = F1 + F2
polygon, therefore the net force on the
Retarding force = mg + F So, option (a) and (c) both can be
particle = F2 − F1
mg + F satisfied.
F − F1 Retardation (a) =
Acceleration (a) = 2 m
m 8 Force on the pulley, by the clamp
v2 v2m = Resultant of forces (M + m ) g acting
Distance, s = = … (i)
2 The situation is as shown in the figure. 2a 2(mg + F ) along horizontally and mg acting
At an initial time, the ball is at P, then
For the downward motion, net force vertically downwards
under the action of a force (exerted by
= mg − F = (Mg + mg )2 + (mg )2
hand) from P to A and then from A to B, mg − F
let acceleration of ball during the ∴ Acceleration (a ′ ) =
m = [(M + m )2 + m 2 ] g
motion from P to A is ams −2 [assumed
v ′2 v ′2 m
to be constant] in an upward direction Distance (s ′ ) = = As, 9 As, F = 600 − 2 × 105t
2a ′ 2(mg − F )
and velocity of ball at A is v ms −1 . At, t = 0, F = 600 N
s = s′
According to question,
mg − F
∴ v′= v F = 0, on leaving the barrel,
B mg + F ⇒ 0 = 600 − 2 × 105t
2m
5 Given, u = 5ms −1 , ∴ t =
600
= 3 × 10−3 s
along positive x-direction 2 × 105
A F = − 0.4 N, This is the time spent by the bullet in the
0.2 m barrel
along negative x-direction
600 + 0
P M = 200g = 0.2 kg Average force = = 300 N
2
Thus, the acceleration
Average impulse imparted = F × t
Then for PA, v 2 = 02 + 2a × 0.2 F 0.4
a= =− = − 2 ms −2 = 300 × 3 × 10−3 = 0.9 N-s
M 0.2
For AB, 0 = v2 − 2 × g × 2
The negative sign showing the 10 Different forces involved are as shown in
⇒ v = 2g × 2
2
the figure.
retardation.
From above equations, R
The position of the object at time t is f co
a = 10 g = 100 ms −2 given by
sb
1 b
F x = x 0 + ut + at 2 f
2 mg
At t = 0, the body is at x = 0, therefore sin
b
x 0 = 0. P
M
b
1
a=10g Hence, x = ut + at 2 mg cos b
mg
2 b
Since, the force acts during the time
interval from t = 0 to t = 10 s, the Observing the figure, we have
mg
motion is accelerated only within this acceleration of the system,
Then for PA, from FBD of ball is time interval. The position of the body P
a=
F − mg = ma at t = 2.5 s is given by M + m
[F is the force exerted by hand on the 1
x = 5 × 2.5 + × (−2) × (2.5)2 Pm
ball] 2 Force on block of mass (m ) =
M + m
⇒ F = m(g + a) = 0.2(11 g ) = 22 N = 6.25m
If f is pseudo force on m in the direction
3 This is the question based on 6 Initially, the momentum of the packet opposite to force P, then
impulse-momentum theorem. in train A is more than in train B. When Pm
packets are changed, the packet f =
|F∆t | = | Change in momentum| M + m
DAY FOUR LAWS OF MOTION 49

As it is clear from the figure, or 8 m1 − 8 m2 = m1 + m2 Now, for the metal rod, tension at both of
f cos β = mg sin β or 7m1 = 9 m2 its end are dissimilar and
Pm
cos β = mg sin β
m1
=
9 T1 − T2 = 80 …(ii)
(M + m ) m2 7 (Q g = 10 ms −2 )
sin β Now, from Eqs. (i) and (ii), we get
P = g (M + m ) 14 As, s = ut + 1 at 2
cos β 2 T1 = 230 N and T2 = 150 N
or, P = (M + m ) g tanβ 1 ∴ Tension at mid-point
⇒ 50 = 0 × 5 + × a × (5)2
2 = T1 − 4 g = 190 N
11 Net force on the rod,
100  2m1 m2 
f = 32 − 20 = 12 N ∴ a= = 4 ms −2 17 Tension, T = 
25 g
Acceleration of the rod  m1 + m2 
f 12 Let, mass of one become m1 and that of
= = = 4 ms −2 other m2 , where m1 > m2 . As m1 moves  2m × 2m 
m 3 =  g
downwards with acceleration  m + 2m 
Equation of motion of the 10 cm part is
a = 4 ms −2 (where, m1 = m and m2 = 2m)
F − 20 = m × a = 1 × 4 ,
F = 4 + 20 = 24 N  m − m2  4
a=  1 = mg
g 3
Similarly, equation of motion of 20 cm  m1 + m2 
Force ( Tension)
part is  m − m2  ∴ Stress =
32 − F = m × a = 2 × 4, So, 4=  1  10 Area
 m1 + m2  4
F = 32 − 8 = 24 N mg
4 mg
 m1 − m2  a 4 2 = 3 =
12 On releasing, the motion of the system   = = = A 3A
will be according to the figure  m1 + m2  g 10 5
18 It is known that the thrust
∴ Percentage of mass transferred
= − v 
dM 
 m − m2   = − v (αv )
= 1  × 100  dt 
 m1 + m2  Hence, the retardation produced
2 thrust αv 2
= × 100 = 40% = =−
a 5 mass M
a 15 As, mg − R = ma 19 When ball dropped, acceleration of the
m1 ball is g as will be observed by a man
standing stationary on the ground. The
Man man inside the lift is having its own
m 1g m2 downward acceleration, a. Therefore,
relative acceleration of the ball as
observed by the man in the lift will must
m2 g be = (g − a).
mg − ηmg = ma
The equations of motion of blocks mg (1 − η ) = ma ⇒ a = g(1 − η ) 20 When the lift is stationary, then
are, R = mg
m1 g − T = m1 a … (i)
16 For block of mass M1 ,
49 = m × 9.8
T − m2 g = m2 a 480 − T1 − 20 g
and … (ii) =a 49
20 m= kg = 5kg
On solving, 9.8
 m − m2  480 N T2 If a is the downward acceleration of the
a=  1 g … (iii)
 m1 + m2  lift then, R = m(g − a)
Here, m1 = 5 kg, M1 M2 = 5(9.8 − 5) = 24 N
m2 = 4.8 kg 21 The situation is given in figure below
g = 9.8 ms −2 T1
M2
5 − 4.8 
∴ a =   × 9.8 q
 5 + 4.8  Also, for block of mass M2 , in
gs
T2 − 12 g q
=
0.2
× 9.8 =a g cos q
12 q g
9.8
Since, a is common for all the
= 0.2 ms −2 From equation of motion,
individuals of the system
1
(m − m2 ) g g 480 − T1 − 20 g T2 − 12 g g sinθ t 2 = l
13 As, a = 1 = , ⇒ = 2
m1 + m2 8 20 12
2l
m1 − m2 1 After taking g = 10 ms −2 this gives ⇒ t =
= g sinθ
m1 + m2 8 5T2 + 3T1 = 1440 …(i)
50 40 DAYS ~ JEE MAIN PHYSICS DAY FOUR

22 To move the body up the inclined f The distance covered by the trolley in
plane, the force required, this time,
= mg sinθ + µR 1 1
s ′ = ut + at 2 = 0 + × 3 × 10 = 15m
= mg sin θ + µmg cos θ F 2 2
N
23 Limiting force of friction of block on 29 From Newton’s second law,
slab µ m1 g = 0.6 × 10 × 9.8 = 58.8 N mg sin θ − µ mg cos θ
=a
Since, the applied force = 100 N on m
block, which is greater than the force of Mg Now, distance covered by the particle,
limiting friction, the block will f = Mg …(i) v 2 = u2 + 2as
accelerate on the slab, due to which, the F =N mg sin θ − µ mg cos θ 
⇒ v = 2
and …(ii)
force acting on the slab will be that due But force of friction ( f ) = µN x
 m 
to the kinetic friction (µ k m1 g ).
= µ F …(iii)
Hence, acceleration of the slab, = 2 gx sin θ − 0.6 x2 g cos θ
[using Eq. (ii)]
µ m g 0.4 × 10 × 9.8 dv
a= k 1 = = 0.98 ms–2 From Eqs. (i) and (iii), we get v should be maximum when =0
m2 40 Mg dx
µF = Mg or F =
µ d 2gx sin θ − 0.6 x2 g cos θ )
24 Let, R be the normal contact force by ⇒ =0
wall on the block. dx
27 Here, R = Mg − F sin φ
By differentiating, we get x = 3 .33 m
f ∴ f = µR = µ(Mg − F sin φ)
30 In vertical direction, weights are
F sin f balanced by frictional forces.
10 N R R F Consider FBD of block A and B as shown
f in diagram below.
F cos f
fA fB
w
R = 10N, f L = w and f = µR
∴ µR = w or w = 0.2 × 10 = 2 N mg
F N
25 Free body diagram (FBD) of the block Net force, F cos φ − f = Ma
1
(shown by a dot) is as shown in the ∴ a= [F cos φ − f ]
figure M 20 N fA 100 N
1
N Vertical ⇒ a= [F cos φ − µ (Mg − F sin φ)]
M As the blocks are in equilibrium, balance
F µF forces are in horizontal and vertical
f
= cos φ − µg + sin φ
F cos 60° Horizontal M M direction.
F
= (cos φ + µ sin φ) − µg The system of blocks (A + B )
M
mg + F sin 60° F =N
28 Given, acceleration of the trolley
For vertical equilibrium of the block −2 For block A, f A = 20 N
(a) = 3 ms .
N = mg + F sin 60° and for block B,
F Therefore, the force acting on the block
= 3g + 3 … (i) is F = ma = 10 × 3 = 30 N. f B = f A + 100 = 20 + 100 = 120 N
2
For no motion, force of friction The weight mg of the block is balanced 31 Energy lost over path PQ = µ mg cos θ × 4
f ≥ F cos 60° by the normal reaction R. The force of
limiting friction is given by P
or µN ≥ F cos 60°
f f
1  3F  F µ = = 4m
or  3g + ≥ R mg h=2m
2 3 2  2
f = µmg = 0.2 × 10 × 10 = 20 N
F 30º
or g≥ or F ≤ 2 g or 20 N The net force on the block is towards
2 O 23m Q x R
right and is given by
Therefore, maximum value of F is 20 N. F ′ = F − f = 30 − 20 = 10 N Energy lost over path QR = µ mg x
26 Given, mass of the block = M F ′ 10 i.e. µ mg cos 30° × 4 = µ mg x
So, a′ = = = 1 ms −2 .
Coefficient of friction between the block m 10 (Q θ = 30° )
and the wall = µ Let, t be the time taken for the block to x = 2 3 = 3.45 m
Let, a force F be applied on the block to fall off from the rear end for the trolley. From Q to R energy loss is half of the
hold the block against the wall. The Then, the block has to travel a distance total energy loss.
normal reaction of mass be N and force s′ = 5m to fall off. Now, since the 1
i.e. µ mg x = × mgh ⇒ µ = 0.29
of friction acting upward be f . In trolley starts from rest. So, u = 0 and 2
1
equilibrium, vertical and horizontal using s = ut + at 2 , we can determine The values of the coefficient of friction µ
forces should be balanced separately. 2 and the distance x(= QR ) are 0.29 and 3.5.
t as 10 s.
DAY FOUR LAWS OF MOTION 51

32 Motion stops when pull due to 35 Free body diagram of the block is SESSION 2
m1 ≤ force of friction between m and m2
and surface. f 1 Suppose, F = force on the table due to
⇒ m1 g ≤ µ(m2 + m )g the weight of the chain on the table
⇒ 5 × 10 ≤ 01510. ( + m ) × 10 + momentum of the chain transmitted
⇒ m ≥ 2333
. kg Fs N on the table
Here, nearest value is 27.3 kg ⇒ F = F1 + F2 (Let)
So, m min = 27.3 kg m
mg Now, F1 = yg ,
l
33 F1 = mg (sin θ + µ cos θ) [as body just in m
position to move up, friction force Here, N = F s = kd dp = dmv = dy 2gy dy
l
downward] and mg = f ≤ µ N = µ kd dp m dy
F2 = mg (sin θ − µ cos θ) [as body just in mg = F2 = 2gy
or k ≥ dt l dt
position to slide down, friction upward] µd
(2gy )  = 2gy 
m dy y
=
Hence, (b) is the correct option. l  dt 
F2
36 When a car accelerates, the engine m 2 myg
m ∴ F = yg +
F1 rotates the rear axle which exerts a push l l
on the wheels to move. 3 myg
=
l
37 Both Statements are correct. But
Statement II does not explain correctly, 2 Force applying on the block
q
Statement I. F = mg sinθ
F1 sin θ + µ cos θ Correct explanation is there is increase or mg sinθ = ma
∴ = ∴ a = g sinθ
F2 sin θ − µ cos θ in normal reaction when the object is
where, a is along the inclined plane.
tan θ + µ 2µ + µ pushed and there is decrease in normal
= = =3
tan θ − µ 2µ − µ reaction when the object is pulled (but ∴ Vertical component of acceleration is
strictly, not horizontally). g sin2 θ.
34 Here, (80 − 72)g = ma y
or a y = 1 m/s2 38 The cloth can be pulled out without ∴ Relative vertical acceleration of A with
dislodging the dishes from the table due
5 respect to B is
∴asin37° = 1 m/s2 or a = m/s2 to law of inertia, which is Newton’s first
3 law. While, Statement II is true, but it is g (sin2 60° − sin2 30° ]
R Newton’s third law. g
= = 4.9 ms −2
2
m mg 39 If the bullet is fired from the rifle, the
momentum of bullet-rifle system is [in vertical direction]
conserved. 3 Here, initial mass of the rocket = M
mg
It means, Mb v b = M r v r … (i)
37° dm
1 2
=r
(a) M v dt
E k(b ) 2 b b
and = Relative velocity of gases w.r.t. rocket
E k( r ) 1 M v 2
r r =v
ax= a cos 37° 2
then, acceleration of the rocket
37° M
= r a=
F
=
u(dm/dt )
Mb m 
× t 
dm
a M −
As, M r > Mb (mass of rifle is greater  dt 
ay=a sin 37°
than the mass of bullet). ur
(b) =
Now, we apply Newton’s second law of Hence, E k(b ) > E k( r ). So, the kinetic (M − r t )
motion on the box in the direction of energy of bullet is greater than the
acceleration, kinetic energy of rifle. 4 Balanced horizontal force, N 1 = N 2
5 Balanced vertical force, 2µN 1 = mg
mg sin 37° − µ mg cos 37° = m × 40 In order to apply Newton’s second law
3 mg 40 × 10
on a body observed from a non-inertial µN 1 = =
13 2 2
or µ = frame of reference pseudo force is
24 considered in a direction opposite to = 200 N
Hence, (b) is the correct option. real acceleration.
52 40 DAYS ~ JEE MAIN PHYSICS DAY FOUR

∴ Man pushes the wall with 200 N ∴Displacement of block A in 1 s is


1  2g sin2 θ 
s = 01
. +   × (1)
2
2  1 + sin2 θ 
a
 g sin2 θ 
mN1 or s=  2 
θ = 45°  1 + sin θ 
mg
Hence, (a) is the correct option.
Here, θ = 45°
g 9
N1 N2 ∴ a = g sin 45° =
2
N
5 As, it is clear from the figure Let the travelled distance be s.
F Using equation of motion,
R a 1 a1
s = ut + at 2 , we get mmg
2
1 g 2 gT 2
s = 0.t + T or s = …(i) mg
2 2 2 2
mg sin a N
mg On rough inclined plane Acceleration
F = mg sinα of the body mmg
a2
and R = mg cos α a = g (sin θ − µ cos θ) mmg
F g(1 − µ )
⇒ = tanα = g(sin 45°− µ cos 45° ) =
R mg
2
1
i.e. µ = tan α =  As,sin 45° = cos 45° = 1 
3   Velocity of block, just before reaching
 2 the carriage is
⇒ cotα = 3
Again using equation of motion v = 2gh
6 On smooth inclined plane, the 1
s = ut + at 2 , we get Now, acceleration of block,
acceleration of body, 2 − µmg
a = g sinθ 1 g (1 − µ ) a1 = = −µ g
s = 0( pT ) + ( pT )2 m
If s be the distance travelled, then 2 2
Acceleration of carriage,
1
s = g sinθt 12 g (1 − µ )p2T 2 µ mg
…(i) or s = …(ii) a2 =
2 2 2 M
On rough inclined plane, the From Eqs. (i) and (ii), we get At t = 0, motion of block as seen from the
acceleration is given by carriage is
gT 2 g (1 − µ )p2T 2
a = g sin θ − µg cos θ = u rel = v = 2gh
2 2 2 2
1
∴ s = (g sin θ − µg cos θ)t 22 arel = a1 − a2 = − µg  1 + 
…(ii) or (1 − µ )p2 = 1 m
2 and
 M
From Eqs. (i) and (ii), we get 1  1 
or 1−µ = or µ =  1 − 2 
2 Now, relative velocity of block when
t 22 sin θ p  p  block moves through distance x with
=
t 12 g sin θ − g cos θ 8 Free body diagram for A and B is respect to carriage,
t 22 N v 2rel = u2rel + 2 arel x
But t 1 = nt 1 or =n 2 N¢
02 = 2gh − 2µg  1 +  ⋅ l
t 12 N m

B  M
sin θ
∴ n2 = (when x = l , then v rel = 0)
sin θ − µ cos θ A
2 gh = 2µg  1 +  ⋅ l
m
Solving, we get ⇒
 M
n2 − 1 sin θ mg mg
µ = × ⇒ h = µ  1 +  ⋅ l
m
n2 cos θ For A  M
 1  mg − N = m (asin θ) …(i)
Hence, (c) is the correct option.
n2 − 1 sin 45° n2 − 1  
2 [as block has only vertically downward
= × = ×
n2 cos 45° n2  1  acceleration] 10 Initially under the equilibrium of mass

 2 For B m, T = mg
n2 − 1 1 (N + mg )sin θ = ma …(ii) Now, the string is cut. Therefore, T = mg
= = 1− On solving Eqs. (i) and (ii), we get force is decreased on the mass m
n2 n2
 2g sin θ  upwards and downwards on the mass
µ = 1−
1 a=  
 1 + sin θ 
2
n2 2m.
The acceleration of block A is mg
7 On smooth inclined plane Acceleration ∴ am = =g (downwards)
2g sin2 θ m
of a body sliding down a smooth a A = asin θ = mg g
inclined plane, a = g sinθ 1 + sin2 θ and a2m = = (upwards)
2m 2
DAY FOUR LAWS OF MOTION 53

11 This is the equilibrium of coplanar increasing curve with decreasing slope F


forces. with time.
F F dv a=5 m/s–2
Hence, ΣF x = 0 a= = 0 e −bt = T
∴ F =N m m dt
v tF −bt
∴ ΣF y = 0, f = mg , Στc = 0 ⇒ ∫ dv = ∫ 0 e
0 0m
∴ τN + τ f = 0
t 0
Since, τf ≠ 0 mW g
F  1  −bt F
∴ τN ≠ 0 ⇒ v = 0 e = 0 e −bt
m  −b  mb T + F − mwg = mwa
Thus, N will produce torque. t
0
[Q m w = mass of worker]
12 According to the work-energy theorem, =
F0 0
(e − e −bt ) ⇒ T + F − 1000 = 500
∑ W = ∆K = 0 mb ⇒ T + F = 1500 …(ii)
⇒ Work done by friction + Work F
= 0 (1 − e −bt ) Solving Eqs. (i) and (ii), we get
done by gravity = 0 mb T = 1125N and F = 375N
l F
⇒ − (µ mg cos φ) + mgl sin φ = 0 with v max = 0 [ a + t = ∞ ]
2 mb 17 Let the accelerations of various blocks are
µ as shown in figure. Pulley P2 will have
or cos φ = sin φ or µ = 2 tan φ 15 From the graph, it is a straight line so, downward acceleration a.
2
motion is uniform because of impulse
13 A block of mass m is placed on a surface direction of velocity changes as can be
with a vertical cross-section, then seen from the slope of the graph.
2
y Initial velocity, v 1 = = 1 ms −1
m 2 P1 P2
a
Final velocity, v 2 = −2 / 2 = −1 ms −1
y a
p i = mv 1 = 0.4 N - s
θ a1 1 3
x and p f = mv 2 = −0.4 N - s
Now, impulse, a4
 x3 
d  J = p f − p i = − 0.4 −0.4 a2
 6 2 4
dy x2
tanθ = = = = − 0.8 N - s
dx dx 2 Analysing the diagram,
⇒ |J|= 0.8 N - s a1 + a2
At limiting equilibrium, we get a= ⇒ a2 = 2a − a1 > 0
µ = tan θ 16 Free body diagram for crate 2
x2 So, acceleration of block 2 is upward
0.5= ⇒ x2 = 1 ⇒ x = ±1
2 T T Hence, (A)→ (2,3)
a + a4
Now, putting the value of x in and a= 1
x3 a=5 ms–2 2
y= , we get F F ⇒ a4 = 2a + a1 > 0
6
MC g So, acceleration of block 4 is downward.
When x = 1 When x = − 1
Hence, (B)→ (1,4).
3
(1) 1 (−1)3
−1 mC g This is downwards.
y = = y = =
6 6 6 6 Hence, (c)→ (4)
So, for vertical equilibrium of the crate.
So, the maximum height above the Acceleration of block 2 with respect to
T − F − Mc g = Mc a block 4.
ground at which the block can be
placed without slipping is 1/6 m. [Q Mc = mass of crate = 50 kg] a2 /4 = a2 − (− a4 ) = 4 a > 0
⇒ T − F − 500 = 250 This is upward.
14 As the force is exponentially decreasing, Hence (D)→(3) (C)→ (4)
so it’s acceleration, i.e. rate of increase ⇒ T − F = 750 …(i)
Hence, A → (2,3), B → (1,4), C → (4),
of velocity will decrease with time. Free body diagram for worker D → (3)
Thus, the graph of velocity will be an
EXAM BITES

This Pdf Is
Downloaded From
www.exambites.in

Visit www.exambites.in for


More Premium Stuffs,Latest
Books,Test Papers,Lectures etc.
jeeneetadda
jeeneetadda_official
jeeneetadda

VISIT NOW !!
DAY FIVE

Circular
Motion
Learning & Revision for the Day
u Concept of Circular Motion u Forces in Circular Motion
u Dynamics of Uniform Circular Motion u Applications of Centripetal and Centrifugal Forces

Concept of Circular Motion


Circular motion is a two dimension motion. To bring circular motion in a body it must be
given some initial velocity and a force. Circular motion can be classified into two types-
Uniform circular motion and Non uniform circular motion.
When an object moves in a circular path at a constant speed then the motion is said to
be a uniform circular motion.
When an object moves in a circular path with variable speed, then the motion is said to
be non-uniform circular motion.

Terms Related to Circular Motion


1. Angular Displacement
It is defined as the angle turned by the particle from some reference line. Angular
displacement ∆θ is usually measured in radians. PREP
Finite angular displacement ∆θ is a scalar but an infinitesimally small displacement is MIRROR
a vector. Your Personal Preparation Indicator

2. Angular Velocity u No. of Questions in Exercises (x)—


It is defined as the rate of change of the angular v2 u No. of Questions Attempted (y)—
displacement of the body. u No. of Correct Questions (z)—
B (Without referring Explanations)
From figure a particle moving on circular track of radius r is
showing angular displacement ∆θ in ∆t time and in this time r2
u Accuracy Level (z / y × 100)—
period, it covers a distance ∆s along the circular track, then ∆θ v1
O u Prep Level (z / x × 100)—
 ∆θ  dθ
∴ Angular velocity, ω = lim   = r1
∆t→ 0  ∆t  dt In order to expect good rank in JEE,
A your Accuracy Level should be above
It is an axial vector whose direction is given by the right 85 & Prep Level should be above 75.
hand rule. Its unit is rad/s.
DAY FIVE CIRCULAR MOTION 55

3. Angular Acceleration Applications of Centripetal and


It is the rate of change of angular velocity.
Centrifugal Forces
dω d2θ
Thus, α= = Some of the most important applications of centripetal and
dt dt 2 centrifugal forces are given below
Its unit is rad/s2 .
Motion of a Vehicle on a Level Circular Road
Dynamics of Uniform When a vehicle negotiates a circular path, it requires a
centripetal force.
Circular Motion In such cases the lateral force of friction may provide the
If a particle, is performing circular motion with a uniform required centripetal force. Thus, for maintaining its circular
speed, then motion of the particle is called uniform circular path required centripetal force.
motion. In such a case,
 mv2 
dv v2   ≤ frictional force (µ mg)
= 0 and ac = ω2 r = [Q v = rω]  r 
dt r
Maximum speed vmax = µrg
Thus, if a particle moves in a circle of radius r with a uniform
v2 where, µ = coefficient of friction between road and vehicle
speed v, then its acceleration is , towards the centre. This
r tyres and r = radius of circular path.
acceleration is termed as centripetal acceleration.
NOTE • In non-uniform circular motion Resultant acceleration of
Bending of a Cyclist
v4
the body is a = ar2 + aT2 = + r2 α2 When a cyclist goes round turns in a circular track, then angle
r2
made by cyclist with vertical level is given by
aT
v2
tan θ = ≈θ
a Tangential rg
acceleration
 v2 
= tan −1  
O ar  rg 
Radial
acceleration
Banking of a Curved Road
For the safe journey of a vehicle on a curved (circular) road,
Forces in Circular Motion without any risk of skidding, the road is slightly raised
towards its outer end.
In circular motion of an object two kinds of forces occur
Let the road be banked at an angle θ from the horizontal, as
which are described below
shown in the figure.
Centripetal Force If b is width of the road and h is height of the outer edge of
the road as compared to the inner edge, then
The centripetal force is required to move a body along a
v2 h
circular path with a constant speed. The direction of the tan θ = =
centripetal force is along the radius, acting towards the centre rg b
of the circle, on which the given body is moving.
Centripetal force,
mv2 4π 2 R R cos θ
F = = mrω2 = mr 4π 2 ν2 = mr 2 [Q v = rω]
r T θ
Work done by centripetal force is always zero as it is R sin θ A
Outer
perpendicular to velocity and hence instantaneous edge
displacement. F raised
θ mg
O X
B
Centrifugal Force
In case of friction is present between road and tyre, then
‘Certrifugal force can be defined as the radially directed
Maximum speed,
outward force acting on a body in circular motion, as observed
by a person moving with the body.’ rg (µs + tan θ)
vmax = ,
mv2 1 − µs tan θ
Mathematically, centrifugal force = = mrω2 [Q v = rω]
r where µs = coefficient of static friction.
56 40 DAYS ~ JEE MAIN PHYSICS DAY FIVE

Motion of a Cyclist in a Death Well l


In general, if the revolving particle, at any instant of time,
is at position C, inclined at an angle θ from the vertical, then
For equilibrium of cyclist in a death
well, the normal reaction N provides v2 = v2L − 2 gr (1 − cos θ)
the centripetal force needed and the f mv2
force of friction balances his weight mg. N and T = mg cos θ +
Cyclist r
mv2
Thus, N = mg l
In the critical condition of just looping the vertical loop,
r
(i.e. when the tension just becomes zero at the highest point
and f = µN = mg B), we obtain the following results
rg r
TH = 0, TL = 6 mg, vL = 5 rg
⇒ vmax =
µ
and vH = rg
Motion along a Vertical Circle In general, TL − TH = 6 mg
In non-uniform circular motion speed of vH B
object decreases due to effect of gravity • When a vehicle is moving over a convex bridge, the
TH NOTE
as the object goes from its lowest mg maximum velocityv = rg , where r is the radius of the road.
position A to highest position B.
O v • When the vehicle is at the maximum height, the reaction of
l
At the lowest point A, the tension TL mv 2
θ the road, is N1 = mg −
and the weight mg are in mutually TL C r
opposite directions and their mg N1
resultant provides the necessary A vL
centripetal force, mg
A
mv2L mv2L
i.e. TL − mg = or TL = mg + N2
r r
l
At the highest point B, tension TH and the weight mg are in
the same direction and hence, B
mv2H mv2H
TH + mg = or TH = − mg • When the vehicle is moving in a dip B, then
r r mv 2
Moreover, vL and vH are correlated as v2H = v2L − 2 gr . N2 = mg +
r

DAY PRACTICE SESSION 1

FOUNDATION QUESTIONS EXERCISE


1 A particle is moving along a circular path of radius 5 m, 4 A wheel is rotating at 900 rpm about its axis. When the
moving with a uniform speed of 5 m s −1. What will be the power is cut-off, it comes to rest in 1 min. The angular
average acceleration, when the particle completes half retardation in rads – 2 is
revolution? (a) π / 2 (b) π / 4 (c) π / 6 (d) π / 8
−2
(a) zero (b) 10/ π ms 5 A roller is made by joining together B D
(c) 10 ms −2 (d) None of these two corners at their vertices O. It is
2 A cyclist goes round a circular path of circumference kept on two rails AB and CD which
34.3 m in 22 s, the angle made by him with the vertical are placed a symmetrically (see the O
will be figure), with its axis perpendicular
(a) 45 ° (b) 40° (c) 42° (d) 48°
to CD and its centre O at the centre A C
of line joining AB and CD (see the
3 A particle undergoes a uniform circular motion. About figure). It is given a light path, so that it starts rolling with
which point on the plane of the circle, will the angular
its centre O moving parallel to CD in the direction shown.
momentum of the particle, remain conserved?
As it moves, the roller will tend to ª JEE Main 2016 (Offline)
(a) About centre of the circle
(a) turn left (b) turn right
(b) On the circumference of the circle
(c) go straight
(c) Inside the circle
(d) turn left and right alternately
(d) Outside the circle
DAY FIVE CIRCULAR MOTION 57

6 A particle moves in a circular path with decreasing 13 A cyclist starts from centre O of a circular Q
speed. Choose the correct statement. park of radius 1 km and moves along the R

(a) Angular momentum remains constant path OPRQO as shown in figure. If he


(b) Acceleration a is acting towards the centre maintains constant speed of10 ms −1, what O
P
(c) Particle moves in a spiral path with decreasing radius is his acceleration at point R in magnitude
(d) The direction of angular momentum remains constant and direction?
7 A mass of 2 kg is whirled in a horizontal circle with the (a) 0.1 ms−2 along RO (b) 0.01ms−2 along OR
help of a string, at an initial speed of 5 rev/min. Keeping (c) 1ms−2 along RO (d) 0.1 rad s−2 along RO
the radius constant, the tension in the string is doubled.
14 A car is moving in a circular horizontal track of radius 10
The new speed is nearly
m with a constant speed of 10 ms −1. A plump bob is
(a) 14 rpm (b) 10 rpm suspended from the roof of the car by a light rigid rod of
(c) 2.25 rpm (d) 7 rpm
length 1.00 m. The angle made by the rod with the track
8 A cyclist is riding with a speed of 27 kmh−1. As he is
approaches a circular turn on the road of radius 80 m, he (a) 60° (b) 30°
applies brakes and reduces his speed at the constant (c) 45° (d) zero
rate of 0.5 ms −1 every second. What is the magnitude
15 A frictionless track ABCD ends in a semi-circular loop of
and direction of the net acceleration of the cyclist on the
radius R. A body slides down the track from point A which
circular turn?
−2 is at a height h = 5 cm. Maximum value of R for the body
(a) 0.86 ms at 54° to the velocity
to successfully complete the loop is
(b) 0.6 ms−2 at 54° to the velocity
(c) 0.3 ms−2 at 75° to the velocity A
(d) 0.7 ms−2 at 68° to the velocity
9 Two cars of masses m1 and m2 are moving in circles of D
h
radii r1 and r2, respectively. Their speeds are such that
2R C
they make complete circles in the same time t. The ratio
of their centripetal acceleration is ª AIEEE 2012 B
(a) m1r1 : m2r2 (b) m1 : m2 15 10
(c) r1 : r2 (d) 1 : 1 (a) 5 cm (b) cm (c) cm (d) 2 cm
4 3
10 A ball of mass 0.25 kg attached to the end of a string of 16 A weightless thread can bear a tension upto 3.7 kg-wt. A
length 1.96 m is moving in a horizontal circle. The string stone of mass 500 g is tied to it and revolved in a circular
will break if the tension is more than 25 N. What is the path of radius 4 m in a vertical plane. If g = 10 ms −2 , then
maximum speed with which the ball can be moved? the maximum angular velocity of the stone will be
(a) 14 ms −1 (b) 3 ms −1 (c) 3.92 ms −1 (d) 5 ms −1
(a) 4 rad s −1 (b) 16 rad s −1
11 A car is moving along a straight horizontal road with a (c) 21 rad s −1 (d) 2 rad s −1
speed ν 0. If the coefficient of friction between the tyres 17 A bob of mass m suspended by a light C
and the road is µ, the shortest distance in which the car string of length L is whirled into a
can be stopped is vertical circle as shown in figure. What
v 02 v0 will be the trajectory of the particle, if B
(a) (b)
2 µg µg the string is cut at B. L
2
v  v0 (a) Vertically upward
(c)  0  (d) A
 µg  µ (b) Vertically downward
(c) Horizontally towards left
12 A racing car travel on a track A F (d) Horizontally towards right
(without banking) ABCDEFA. ABC is R 18 A particle is moving in a vertical circle.The tensions in the
a circular arc of radius 2R . CD and E
90° R D string when passing through two positions at angles 30°
FA are straight paths of length R and C and 60° from vertical (lowest positions) areT1 and T2 ,
O 2R
DEF is a circular arc of radius
respectively.Then
R = 100 m. The coefficient of friction
(a) T1 = T2
on the road is µ = 0 .1. The maximum B
−1
speed of the car is 50 ms . The (b) T2 > T1
minimum time for completing one round is (c) T1 > T2
(a) 89.5 s (b) 86.3 s (c) 91.2 s (d) 41.3 s (d) tension in the string always remains the same
58 40 DAYS ~ JEE MAIN PHYSICS DAY FIVE

19 A small body of mass m slides down from the top of a Statement II Components of the frictional force are
hemisphere of radius R. The surface of block and providing the necessary tangential and centripetal
hemisphere are frictionless. The height at which the acceleration, in the above situation.
body lose contact with the surface of the sphere is 21 Statement I A particle moving in a vertical circle, has a
3 2 1 1 maximum kinetic energy at the highest point of its
(a) R (b) R (c) R (d) R
2 3 2 3 motion.
Statement II The magnitude of the velocity remains
Direction (Q. Nos. 20-24) Each of these questions contains constant for a particle moving in a horizontal plane.
two statements : Statement I (Assertion) and Statement II
(Reason). Each of these questions also has four alternative 22 Statement I The centripetal force and the centrifugal
choices, only one of which is the correct answer. You have to force never cancel out.
select one of the codes (a), (b), (c), (d) given below : Statement II They do not act at the same time.
(a) Statement I is true, Statement II is true; Statement II is the
correct explanation for Statement I 23 Statement I Improper banking of roads causes wear and
(b) Statement I is true, Statement II is true; Statement II is not tear of tyres.
the correct explanation for Statement I Statement II The necessary centripetal force in that event
(c) Statement I is true; Statement II is false is provided by friction between the tyres and roads.
(d) Statement I is false; Statement II is true 24 Statement I When a particle moves in a circle with a
20 Statement I A car is moving in a horizontal circular plane uniform speed, there is a change in both its velocity and
with varying speed, then the frictional force is neither acceleration.
pointing towards the radial direction nor along the Statement II The centripetal acceleration in circular
tangential direction. motion is dependent on the angular velocity of the body.

DAY PRACTICE SESSION 2

PROGRESSIVE QUESTIONS EXERCISE


1 A heavy sphere of mass m is 3 Two small spherical balls are free to move on the inner
suspended by a string of length l. surface of the rotating spherical chamber of radius
The sphere is made to revolve q R = 0.2 m. If the balls reach a steady state at angular
about a vertical line passing T T cosq position θ = 45°, the angular speed ω of device is
through the point of suspension, q w
in a horizontal circle such that the
string always remains inclined to 3m
the vertical making an angle θ. mg
What is the period of revolution?
l l cosθ q
(a)T = 2 π (b)T = 2 π
g g
l sinθ l tanθ
(c)T = 2 π (d)T = 2 π (a) 8 rad s−1 (b) 2 rad s−1
g g (c) 3.64 rad s−1 (d) 9.34 rad s−1
2 Two wires AC and BC are tied at C of A 4 The skate board negotiates the circular
small sphere of mass 5 kg, which surface of radius 4.5 m. At θ = 45°, its q
revolves at a constant speed v in the speed of centre of mass is 6 ms −1. The
30°

horizontal circle of radius 1.6 m. The B combined mass of skate board and the
minimum value of v is v
45° person is 70 kg and his centre of mass is
−1 0.75 m from the surface. The normal reaction between the
(a) 8.01 ms
C surface and the skate board
(b) 1.6 ms−1
(c) 0 wheel is
1.6 m
(d) 3.96 ms−1 (a) 500 N (b) 2040 N
(c) 1157 N (d) zero
DAY FIVE CIRCULAR MOTION 59

5 A particle is moving with a uniform speed in a circular


orbit of radius R in a central force inversely proportional R/4
to the nth power of R. If the period of rotation of the
particle is T , then ª JEE Main 2018
n
+1 q R
(a)T ∝ R 3 / 2 for any n (b)T ∝ R 2
(c)T ∝ R (n + 1)/ 2 (d)T ∝ R n / 2 O

(b) cos−1 
5 
(a) cos−1  
6 A bob of mass M is suspended by a massless string of 2

length L. The horizontal velocity v at position A is just  3  3
sufficient to make it reach the point B. The angle θ at
(d) cos−1 
5 
(c) cos−1  
5
which the speed of the bob is half of that at A, satisfies 
 6 2 3
B
10 A point moves along a circle with a speedV = kt , where
k = 0.5 m/s 2. Then total acceleration of the point at the
moment when it has covered the nth fraction of the circle
O
1
after the beginning of motion, where n =
L θ 10
(a) 0.8 m/s 2 (b) 1. 2 m/s 2
v
A (c) 1.6 m/s 2 (d) 2.0 m/s 2
π π π
(a) θ = (b) < θ< 11 A circular tube of mass M is m m
4 4 2
π 3π 3π placed vertically on a horizontal
(c) < θ< (d) < θ< π
2 4 4 surface as shown in the figure.
q
Two small spheres, each of
7 A roller coaster is designed such that riders experience
mass m, just fit in the tube, are
‘weightlessness’ as they go round the top of a hill whose
released from the top. If θ gives
radius of curvature is 20 m. The speed of the car at the
the angle between radius vector
top of the hill is between
of either ball with the verticle,
(a) 14 ms −1 and 15 ms −1 (b) 15 ms −1 and 16 ms −1 M
(c) 16 ms −1 and 17 ms −1 (d) 13 ms −1 and 14 ms −1 then for what value of the ratio , tube breaks its contact
m
8 Two particles revolve concentrically in a horizontal plane with ground when θ = 60º. (Neglect any friction).
in the same direction. The time required to complete one 1 2 3
(a) (b) (c) (d) None of these
revolution for particle A is 3 min, while for particle B is 2 3 2
1 min. The time required for A to complete one revolution
12 A particle is moving in a circle of radius R in such a way
relative to B is
that at any instant the normal and tangential component
(a) 2 min (b) 1.5 min (c) 1 min (d) 1.25 min
of its acceleration are equal. if its speed at t = 0 is vo . The
9 A skier plans to ski on smooth fixed hemisphere of radius time taken to complete the first revolution is
R. He starts from rest from a curved smooth surface of R R −2 π
(a) (b) e
R  v0 v0
height   . The angle θ at which he leaves the
 4 R R
(c) (1 + e −2 π ) (d) (1 − e −2 π )
hemisphere is v0 v0

ANSWERS
SESSION 1 1 (b) 2 (a) 3 (a) 4 (a) 5 (a) 6 (c) 7 (d) 8 (a) 9 (c) 10 (a)
11 (a) 12 (b) 13 (a) 14 (c) 15 (d) 16 (a) 17 (b) 18 (c) 19 (b) 20 (a)
21 (d) 22 (c) 23 (a) 24 (b)

SESSION 2 1 (b) 2 (d) 3 (c) 4 (c) 5 (c) 6 (d) 7 (a) 8 (b) 9 (c) 10 (a)
11 (a) 12 (d)
60 40 DAYS ~ JEE MAIN PHYSICS DAY FIVE

Hints and Explanations


SESSION 1 8 Speed of the cyclist (v ) = 27 kmh −1 12 Balancing frictional force for centripetal
1 The change in velocity, when the 5 force,
= 27 ×
particle completes half the revolution is 18 mv 2
= f = µN = µmg
given by Q1 kmh −1 = 5 ms −1  r
 
∆v = 5ms −1 − (−5ms −1 ) = 10ms −1  18  where, N is normal reaction
15 −1 −1 ∴ v = µrg
Now, the time taken to complete half = ms = 7.5 ms
2
the revolution is given by (where, r is radius of the circular track)
πr π ×1 Radius of the circular turn (r ) = 80 m For path ABC, Path length
t = = = πs aT
v 1 3
= (2 π 2R ) = 3 πR = 3 π × 100
So, the average acceleration a q 4
∆v 10 O = 300 π m
= = ms −2 ac
t π v 1 = µ 2 Rg

2 Here, 2πr = 34.3 ⇒ r = 34.3 = . ms −1


. × 2 × 100 × 10 = 1414
01
2π ∴ Centripetal acceleration acting on the
2 πr 2 πr 300 π
and v = =
cyclist ∴t 1 = = 66.6 s
T 22 v 2 (15 / 2)2 225 14.14
ac = = = ms −2
Angle of banking, r 80 4 × 80 For path DEF, Path length
= 0.70 ms −2 1 π × 100
v  2 = (2 πR ) = = 50 π
θ = tan −1   = 45° Tangential acceleration applied by 4 2
 rg  v 2 = µRg = 01 . × 100 × 10
brakes
3 In uniform circular motion, the only a T = 0.5ms −2 = 10 ms −1
force acting on the particle is centripetal Centripetal acceleration and tangential 50 π
t2 = = 5 πs = 157 . s
(towards centre). Torque of this force acceleration act perpendicular to each 10
about the centre is zero. Hence, angular other. For paths CD and FA,
momentum about centre remains ∴ Resultant acceleration, Path length = R + R = 2R = 200 m
conserved. 200
a= a2c + a2T t3 = = 4. 0 s
4 Use ω = ω0 + α t …(i) 50
= (0.7)2 + (0.5)2
Here, ω 0 = 900 rpm ∴ Total time for completing one round
= 0.49 + 0.25 t = t1 + t2 + t3
= (2 π × 900)/60 rad s − 1
= 0.74 = 0.86 ms −2 = 66. 6 + 157. + 4. 0 = 86.3 s
ω = 0 and t = 60 s
If resultant acceleration makes an angle
Then, Eq. (i) gives 13 Acceleration of the cyclist at point R
π θ with the direction of velocity, then
α = − rad s −2 a 0.7 = centripetal
2 tanθ = c = = 1.4 = tan 54° 28 ′ acceleration (a c )
a T 0.5
5 As, the wheel rolls forward the radius of v2 (10)2 100
θ = 54° 28 ′ ac = = =
the wheel decreases along AB, hence for r 1000 1000
the same number of rotations it moves 9 As their period of revolution is same, so, . ms −2 , along RO
= 01
less distance along AB, hence it turns their angular speed is same centripetal
left. acceleration is a = ω 2 r . 14 Centrifugal force on the rod,
mv 2
6 A particle moves in a spiral path with a1 ω2 r1 r F = along BF .
Thus, = = 1 r
decreasing radius. a2 ω2 r2 r2
O
7 The tension in the string will provide 10 As, T = mv 2 /r
necessary centripetal force. q
Hence, v = Tr /m
∴ T = mrω 2

= mr 4 π2 n2 = 25 × 1.96 / 0.25 = 14 ms −1
∴ T ∝ n2 B
11 Retarding force, C F
2
T1  n1  F = ma = µR = µ mg A
⇒ = 
T2  n2  a=µg mg
2 Now, from equation of motion,
∴ T  =  5 Let θ be the angle, which the rod makes
    v 2 = u2 − 2as
 2T   n2  with the vertical.
∴ 0 = u2 − 2as Forces parallel to the rod,
n22 = 25 × 2
u2 u2 v2 mv 2
n2 = 5 2 ≈ 7 rpm or s = = = 0 mg cos θ + sin θ = T
2a 2µg 2µg r
DAY FIVE CIRCULAR MOTION 61

Force perpendicular to the rod 2 (R − h ) sin θ 4 π2


cos θ = = l sin θ [Qr = l sinθ]
mv 2 R cos θ gT 2
mg sin θ − cos θ
r h 2 (R − h ) cos θ = h  4 π2
= ⇒ T2 = l cos θ
The rod would be balanced if R R  R  g
mv 2 2
mg sin θ − cos θ = 0 ∴ h= R l cos θ
r 3 ⇒ T = 2π
g
mv 2
mg sin θ = cos θ 20 In the present case, the tangential
r component of frictional force is 2 T1 cos 30° + T2 cos 45°
2 2
v (10) responsible for changing the speed of
This gives tan θ = = car while component along the radial
rg 10 × 10
direction is providing necessary
= 1 = tan 45° centripetal force, hence net friction
Here θ = 45° force is neither towards radial nor along T1 sin 30° + T2 sin 45°
tangential direction. mg
15 Velocity at the bottom is 2gh 21 As the kinetic energy at the highest From the figure,
point is zero. T1 cos 30° + T2 cos 45° = mg
For completing the loop,
mv 2
22 We know that centripetal and T1 sin 30° + T2 sin 45° =
2gh = 5gR r
centrifugal forces act at the same time
Hence, R = 2h/5 mv 2
on two different bodies. Thus, they mg −
= (2 × 5)/5 = 2 cm never cancel out. ⇒ T1 = r
( 3 − 1)
16 As, T max = mrω2 + mg 23 If the roads are not properly banked, the 2
force of friction between tyres and road
3.7 × 10 = 0.5 × 4ω + 0.5 × 10 2 But T1 ≥ 0
provides the necessary centripetal force,
O
32 which causes the wear and tear of tyres.
or ω2 = = 16 or ω = 4 rad s − 1 θ
2 24 A particle in a circular motion has the
shown feature. The velocity of particle T
17 When bob is whirled into a vertical in circular motion
circle, the required centripetal force is v = rω e t … (i) B
obtained from the tension in the string. C F
y
When string is cut,tension in string A
et er mg
becomes zero and centripetal force is
not provided, hence bob start to move q mv 2
x mg −
in a straight line path along the r P r ≥0
direction of its velocity. q
x 3−1
At point B, the velocity of B is vertically 2
downward therefore, when string is cut mv 2
⇒ mg ≥ ⇒ v≤ rg
at B, bob moves vertically downward. r
Thus, we see that velocity of the
2
particle is r ω along e t or in tangent ∴ v max =
18 T = mv + mg cos θ rg
r direction. So, it changes as the particle = 1.6 × 9.8 = 3.96 ms −1
rotates the circle. Acceleration of the
∴ T ∝ cos θ particle 3 Given, R = 0.2 m
T1 cos 30° 3 /2 3 a = − ω2 r e r +
dv
= = = et …(ii) From the figure,
T2 cos 60° 1 / 2 1 dt
r = 3 R + R sin 45°
or T1 > T2 . Thus, acceleration of a particle moving
in a circle has two components one w
19 Suppose body slips at point B along e t (along tangent) and the other
along e r (or towards centre). Of these
N sin 45°
A the first one is called the tangential
B a t and other is called centripetal a r .
mv 2 N
R From Eq. (ii), it is obvious that
°

mrw2
45

sq
acceleration depends on angular
h q co mg mg sinq velocity (ω ) of the body. N cos 45°
g
m R sin 45°
3R
O R SESSION 2 mg
2 r
mg cos θ =
mv 2 1 Here, mv = T sinθ and mg = T cos θ
R r In the frame of rotating spherical
[Q v = 2g (R − h )] Dividing these two, we get chamber,
v2 r ω2 4 π2 r N cos 45° = mrω2
2g (R − h ) tan θ = = =
g cos θ = rg g gT 2
R N sin 45° = mg
62 40 DAYS ~ JEE MAIN PHYSICS DAY FIVE

mg g Putting the values, 11 Speed of each particle at angle θ is


⇒ tan 45° = =
mrω2 rω2 R = 20 m , g = 10 ms −2 v = 2gh
g g −1 So, v = 20 × 10 = 14.14 ms −1
⇒ω= = = 3.64 rad s [After applying conservation of energy]
r R Thus, the speed of the car at the top of where, h = R(1 − cos θ)
3R +
2 the hill is between 14 ms−1 and 15 ms−1 . ∴ v = 2gR (1 − cos θ)

4
O 8 Here, (ω1 − ω2 )t = 2 π or t = If N is normal reaction, then
ω1 − ω2 mv 2
N N + mg cos θ =
θ 2π 2π
C where, ω1 = and ω2 = R
T1 T2 m
θ 2π T1T2 3×1 or N + mg cos θ = × 2gR(1 − cos θ)
∴t = or t = = R
 2π 2π  T2 − T1 3 − 1 or N + mg cos θ = 2 mg (1 − cos θ)
 − 
mg cos θ  T1 T2  or N = 2m − 3mg cos θ …(i)
mg or t = 1.5 min The tube will breaks its contact with
From the diagram, Hence, (b) is the correct option. ground when,
mv 2 9 At the time of leaving contact, normal 2N cos θ ≥ Mg
N − mg cos θ =
r reaction must be zero. where, we put the value of N from Eq. (i)
mv 2 v is above relation, then we get
∴ N = + mg cos θ 4 mg cos θ − 6mg cos 2 θ = Mg
r
From the figure, q Put θ = 60º (given)
q
θ = 45° , m = 70 kg, g = 9.8 ms −2 , ∴ 4 mg cos 60º −6 mg cos 2 60º = Mg
v = 6 ms −2 , r = CO = 4.5 − 0.75= 3.75 m O 3mg M 1
or 2mg − = Mg ⇒ =
Putting these values in above equation, 2 m 2
we get mg Hence, (a) is the correct option.
N = 1157 N i.e. N =0
mv 2 12 Given, at = ar
5 Q Force = Mass × Acceleration = mω2 R ∴mg cos θ = =
m(2gh )
R R dv v2 dv 1
and given, F ∝
1
⇒ F =
k or = or 2 = dt
2h dt R v R
Rn Rn or cos θ =
R v t
So, we have −2 1
2π 
2
cos θ =  + R(1 − cos θ)
2 R or ∫ v dv = R ∫ dt
= m 
k or
T 
 R R  4  v0 0
Rn 1 1 t
n +1 5 5 − =
4 π2 m n or 3cos θ = or cos θ = or 
+1
⇒ T2 = ⋅R ⇒T ∝ R 2 2 6 v0 v  R
θ = cos −1  
k 5 1 1 t R − v 0t
∴ or = − =
6 Velocity of the bob at the point A  6 v v0 R Rv 0
v = 5gL …(i) Hence, (c) is the correct option. R. v 0
or v =
From conservation of energy, 10 Q v = ds = kt R − v 0t
i.e. KE + PE = constant dt dx
2 s t As v =
 v  = v 2 − 2gh 1
⇒   …(ii) ∴ ∫ ds = k ∫ t dt ⇒ s = kt 2 …(i) dt
 2 2 dx Rv 0
0 0 ∴ =
h = L (1 − cos θ) …(iii) dt R − v 0t
After completion of n th fraction of circle
Solving Eqs. (i), (ii) and (iii), we get x t
s = 2πrn …(ii) Rv
cos θ = −
7
From Eqs. (i) and (ii), we get
or ∫ dx = ∫ R − v00 t dt
8 0 0
4 πrn 
⇒ θ = cos −1  −  = 151 °
7 t2 =   −1 
 k  or x = Rv 0   [ln(R − v 0 t )] 0
t
 8  v0
i.e. 3π / 4 < θ < π Now, tangential acceleration,
dv d  v t
7 Balancing the force, we get aT = = (kt ) = k or x = − R ln  1 − 0 
dt dt  R 
v2
Mg − N = M and normal acceleration,  1 − v 0t  x
R or ln   =−
N v2 k 2t 2  R  R
an = =
v t
r r or 1 − 0 = e −x /R
n
k 2 4 πrn R
or an = × = 4 πnk −x /R v t
Mg r κ or 1−e = 0
2 2 R
∴a = at + an = [k 2 + 16π2 n2 k 2 ] R
or t = (1 − e − x / R ).
= k 1 + 16 π2 n2 v0
After completing one revolution, x = 2πR
For weightlessness, N = 0 = 0.50 1 + 16 × (314
. )2 × (01
. )2 R
∴ t = (1 − e −2 π ).
Mv 2
= 0.8 m / s 2 v0
∴ = Mg or v = Rg
R Hence, (a) is the correct option. Hence, (d) is the correct option.
DAY SIX

Work, Energy
and Power
Learning & Revision for the Day
u Work u Energy u Power
u Conservative and u Work-Energy Theorem u Collision
Non-conservative Force u Law of Conservation of Energy

Work
Work is said to be done, when a body a displaced through some distance in the direction
of applied force. The SI unit work is joule (J) and in CGS it is erg.
1 joule (J) = 107 erg
The work done by the force F in displacing the body through a distance s is
W = (F cos θ)s = Fs cos θ = F ⋅ s
where, F cos θ is the component of the force, acting along the direction of the
displacement produced. SI unit of work is joule (J).
1 J = 1 N-m
Work is a scalar quantity. Work can be of three types
(i) Positive work (ii) Negative work and (iii) Zero work.
l
Positive work If value of the angle θ between the directions of F and s is either zero or PREP
an acute angle. MIRROR
l
Negative work If value of angle θ between the directions of F and s is either 180° or Your Personal Preparation Indicator
an obtuse angle.
u No. of Questions in Exercises (x)—
l
As work done W = F ⋅ s = F s cos θ, hence work done can be zero, if u No. of Questions Attempted (y)—
(i) No force is being applied on the body, i.e. F = 0. u No. of Correct Questions (z)—
(Without referring Explanations)
(ii) Although the force is being applied on a body but it is unable to cause any
displacement in the body, i.e. F ≠ 0 but s = 0.
u Accuracy Level (z / y × 100)—
(iii) Both F and s are finite but the angle θ between the directions of force and u Prep Level (z / x × 100)—
displacement is 90°. In such a case
W = F ⋅ s = F s cos θ = F s cos 90 ° = 0 In order to expect good rank in JEE,
your Accuracy Level should be above
85 & Prep Level should be above 75.
64 40 DAYS ~ JEE MAIN PHYSICS DAY SIX

Kinetic energy for a system of particle will be


Work Done by Variable Force
l

1
Work done by a variable force is given by W = ∫ F ⋅ ds K = Σ mi v2i
2 i
B
l
Relation between kinetic energy and force is
Force KE v ×t
A =
Force 2
where, v is velocity and t is time.
D C
Displacement
Potential Energy
It is equal to the area under the force-displacement graph,
along with proper sign. Potential energy is the energy stored in a body or a system by
virtue of its position in a field of force or due to its
Work done = Area of ABCDA configuration. Potential energy is also called mutual energy or
energy of the configuration.
Value of the potential energy in a given position can be
Conservative and defined only by assigning some arbitrary value to the
Non-conservative Force reference point. Generally, reference point is taken at infinity
and potential energy at infinity is taken as zero. In that case,
A force is said to be conservative if work done by or against r
the force in moving a body depends only on the initial and U = − W = − ∫ F ⋅ dr

final positions of the body and not on the nature of path
followed between the initial and the final position. Potential energy is a scalar quantity. It may be positive as well
Gravitational force, force of gravity, electrostatic force are as negative.
some examples of conservative forces (fields). Different types of potential energy are given below.
A force is said to be non-conservative if work done by or
against the force in moving a body from one positions to
Gravitational Potential Energy
another, depends on the path followed between these two It is the energy associated with the state of separation between
positions. Force of friction and viscous force are the examples two bodies which interact via the gravitational force.
of non-conservative forces. l
The gravitational potential energy of two particles of
masses m1 and m2 separated by a distance r is
− Gm 1 m2
Energy U= .
r
Energy is defined as the capacity or ability of a body to do
work. Energy is scalar and its units and dimensions are the
l
If a body of mass m is raised to a height h from the surface
same as that of work. Thus, SI unit of energy is J. of the earth, the change in potential energy of the system
(earth+body) comes out to be
Some other commonly used units of energy are
mgh
1 erg = 10 −7 J, ∆U =
 h
1 cal = 4.186 J ~= 4.2 J, 1 + 
 R
1 kcal = 4186 J,
or ∆U ≈ mgh if h << R
1 kWh = 3.6 × 10 6 J,
Thus, the potential energy of a body at height h, i.e. mgh is
and 1 electron volt (1 eV) = 1.60 × 10 −19 J really the change in potential energy of the system for
h << R.
Kinetic Energy l
For the gravitational potential energy, the zero of the
l
Kinetic energy is the capacity of a body to do work by potential energy is chosen on the ground.
virtue of its motion. A body of mass m, moving with a
1
velocity v, has a kinetic energy, K = mv2 .
Electric Potential Energy
2 The electric potential energy of two point charges q1 and q2
l
Kinetic energy of a body is always positive irrespective of separated by a distance r in vacuum is given by
the sign of velocity v. Negative kinetic energy is impossible. 1 q1q2
U=
Kinetic energy is correlated with momentum as, 4 π ε0 r
p2 1 N-m2
K = or p = 2 mK where, = 9.1 × 10 9 = constant
2m 4 π ε0 C2
DAY SIX WORK, ENERGY AND POWER 65

Potential Energy of a Spring l


Instantaneous power,
dW F ⋅ ds
Whenever an elastic body (say a spring) is either stretched or Pinst = = = F⋅v
compressed, work is being done against the elastic spring dt dt
1 Some other commonly used units of power are
force. The work done is W = kx2 ,
l

2 1 kW = 103 W,
where, k is spring constant and x is the displacement.
1 1 MW = 10 6 W,
And elastic potential energy, U = k x2
2 1 HP = 746 W
If spring is stretched from initial position x1 to final position
x2 , then
work done = Increment in elastic potential energy
Collision
1 The physical interaction of two or more bodies in which each
= k ( x22 − x12 ). equal and opposite forces act upon each other causing the
2
exchange of energy and momentum is called collision.
Collisions are classified as
Work-Energy Theorem (i) elastic collisions and (ii) inelastic collisions.
Accordingly, work done by all the forces (conservative or
non-conservative, external or internal) acting on a particle or
an object is equal to the change in its kinetic energy of the Elastic Collision in One Dimension
particle. Thus, we can write In a perfectly elastic collision, total energy and total linear
W = ∆K = K f − K i momentum of colliding particles remains conserved.
Moreover, the forces involved in interaction are conservative
We can also write, K f = K i + W
in nature and the total kinetic energy before and after the
 Kinetic energy after  collision, remains unchanged.
Which says that  
 the net work is done u1 u2 v1 v2
m1 m2 m1 m2
 Kinetic energy before  The net 
=  + 
 the net work done   work done A B A B
Before collision After collision

In above figure, two bodies A and B of masses m1 and m2 and


Law of Conservation of Energy having initial velocities u1 and u2 in one dimension, collide
The mechanical energy E of a system is the sum of its kinetic elastically and after collision move with velocities v1 and v2 ,
energy K and its potential energy U. then we find that
E = K +U l
Relative velocity of approach = Relative velocity of
When the forces acting on the system are conservative in separation, i.e. u1 − u2 = v2 − v1
nature, the mechanical energy of the system remains constant,  m − m2   2 m2 
l
v1 =  1  u1 +   u2
K + U = constant ⇒ ∆K + ∆U = 0  m1 + m2   m1 + m2 
There are physical situations, where one or more non-  2 m1   m − m1 
conservative force act on the system but net work done by and v2 =   u1 +  2  u2
 m1 + m2   m1 + m2 
them is zero, then too the mechanical energy of the system
remains constant.
If Σ Wnet = 0 Elastic Collision in Two Dimensions
Mechanical energy, E = constant. l
In this type of collision, the two particles or objects moving
along different directions collide with each other.

Power v1
m1
l
It is a quantity that measures the rate at which work is
done or energy is transformed. A
W A B θ
Average power (P)av = m1
t u1 u1
l
The shorter is the time taken by a person or a machine in Before collision
m2 φ
performing a particular task, the larger is the power of that B
person or machine. m2
l
Power is a scalar quantity and its SI unit is watt, where, v2
1 W = 1 J/s After collision
66 40 DAYS ~ JEE MAIN PHYSICS DAY SIX

l
As linear momentum is conserved. (1 + e)m1  m − em1 
and v2 = u1 +  2  u2
∴ Along the x-axis (m1 + m2 )  m1 + m2 
m1u1 + m2u2 = m1 v1 cos θ + m2 v2 cos φ …(i) l
If a particle of mass m, moving with velocity u, hits an
and along the y-axis identical stationary target inelastically, then final velocities
0 = m1 v1 sin θ − m2 v2 sin φ …(ii) of projectile and target are correlated as
l
As the total kinetic energy remains unchanged. i.e. m1 = m2 = m and u2 = 0;
1 1 1 1 v1 1 − e
Hence, m1u12 + m2u22 = m1 v12 + m2 v22 …(iii) =
2 2 2 2 v2 1 + e
We can solve these equations provided that either the value l
In case of extreme inelastic collision (in which colliding
of θ and φ is known to us. objects stick together after collision)

Inelastic Collision in One Dimension m1


v1
m2
rest
m1 m2
v2

In an inelastic collision, the total linear momentum as well as


total energy remain conserved but total kinetic energy after Before collision After collision
collision is not equal to kinetic energy before collision.
For inelastic collision,  m1 
l
Final velocity (v2 ) =   v1
Common speed, v =
m1 v1  m1 + m2 
m1 + m2 l
Ratio of kinetic energy before and after collision is
m m (v − v )2 KE1  m1 
and loss of kinetic energy ∆K = 1 2 1 2 =
2(m1 + m2 ) 
KE2  m1 + m2 
Here v2 = 0 l
Fraction of kinetic energy lost in the collision is
m1 m2 v12 KE1 − KE2  m2 
∴ ∆K = =
2(m1 + m2 ) 
KE1  m1 + m2 
v1 v2 v
Medium m1 m2 m1 m 2
Rebounding of a Ball on
After collision
Before collision v2=0
Collision with the Floor
l
Speed of the ball after the nth rebound
Coefficient of Restitution (e)
v n = e n v 0 = e n 2gh0
l
It is defined as the ratio of relative velocity of separation to
the relative velocity of approach. l
Height covered by the ball after the nth rebound
v2 − v1 hn = e2 n h0
l
Coefficient of restitution, e =
u1 − u2 l
Total distance (vertical) covered by the ball before it stops
l
For a perfectly elastic collision, e = 1. bouncing

For a perfectly inelastic collision, e = 0.  1 + e2 


H = h0 + 2h 1 + 2h 2 + 2h 3 + … = h0  
l

l
If 0 < e < 1, the collision is said to be partially elastic.  1 − e2 
l
In a perfectly inelastic collision, e = 0 which means that
l
Total time taken by the ball before it stops bouncing
v2 − v1 = 0 or v2 = v1 . T = t 0 + t 1 + t 2 + t 3 +…
It can be shown that for an inelastic collision the final 2 h0 2 h1 2 h2
= +2 +2 + ...
l

velocities of the colliding bodies are given by g g g


 m − em2  (1 + e)m2 2 h0  1 + e 
v1 =  1  u1 + u2 =  
 m1 + m2  (m1 + m2 ) g  1 − e
DAY SIX WORK, ENERGY AND POWER 67

DAY PRACTICE SESSION 1

FOUNDATION QUESTIONS EXERCISE


1 A body of mass 2 kg, initially at rest, is acted upon KE KE
simultaneously by two forces, one of 4N and other of 3N,
acting at right angles to each other. The work done by (c)
t
(d)
the body after 20 s is
(a) 500 J (b) 1250 J (c) 2500 J (d) 5000 J t

2 A body of mass 500 g is taken up an inclined plane of 8 A particle is moving in a circular path of radius a under
length 10 m and height 5 m and then released to slide k
the action of an attractive potentialU = − . Its total
down to the bottom. The coefficient of friction between 2r 2
the body and the plane is 0.1. What is the amount of energy is
work done in the round trip? k k 3 k
(a) − (b) (c) zero (d) −
5 4a 2 2a 2 2 a2
(a) 5 J (b) 15 J (c) 5 3 J (d) J
3
9 The potential energy of a 1 kg particle free to move along
3 A block of mass 5 kg is initially at rest on a horizontal  x 4 x 2
frictionless surface. A horizontal force F = ( 9 − x 2 ) $i the x-axis is given by V ( x ) =  −  J.
 4 2
Newton acts on it, when the block is at x = 0. The
maximum work done by the block between x = 0 and The total mechanical energy of the particle is 2 J. Then,
x = 3 m in joule is the maximum speed (in ms −1) is
3 1
(a) 18 J (b) 15 J (c) 20 J (d) 24 J (a) (b) 2 (c) (d) 2
2 2
4 An object is displaced from point A (2m, 3m, 4m) to a
10 A particle is placed at the origin and a force F = kx acts
point B (1m, 2m, 3m) under a constant force
on it (where, k is a positive constant). If U( 0) = 0, the
F = ( 2$i + 3 $j + 4 k$ ) N, then the work done by this force in graph of U( x ) versus x will be (where, U is the potential
this process is energy function)
(a) 9 J (b) −9 J (c) 18 J (d) −18 J U(x) U(x)

5 An open water tight railway wagon of mass 5 × 10 kg 3

coasts with an initial velocity of 1.2 ms −1 on a railway (a) x (b) x


track without friction. Rain falls vertically downwards on
the wagon.
What change occurs in the kinetic energy of the wagon, U(x) U(x)
after it has collected 103 kg of water?
(a) 900 J (b) 300 J (c) 600 J (d) 1200 J
(c) x (d) x
6 A particle moves in a straight line with retardation
proportional to its displacement. The loss in kinetic
energy of the particle, for any displacement x, is
proportional to 11 A bullet fired into a fixed target losses half of its velocity
2 x
after penetrating distance of 3 cm. How much further it
(a) x (b) e (c) x (d) loge x
will penetrate before coming to rest, assuming that it
7 Which of the diagrams as shown in figure most closely faces constant resistance to its motion?
shows the variation in kinetic energy of the earth as it (a) 3.0 cm (b) 2.0 cm (c) 1.5 cm (d) 1.0 cm
moves once around the sun in its elliptical orbit? 12 A 1.5 kg block is initially at rest on a horizontal frictionless
surface, when a horizontal force in the positive direction
KE KE
of x-axis, is applied to the block. The force is given by
F = ( 4 − x 2 ) i Newton, where x is in metre and the initial
(a) (b) position of the block is at x = 0. The maximum kinetic
energy of the block between x = 0 and x = 2. 0 m is
t t
(a) 6.67 J (b) 5.33 J (c) 8.67 J (d) 2.44 J
68 40 DAYS ~ JEE MAIN PHYSICS DAY SIX

13 A block of mass M moving on a 18 Two balls of masses m1 and m2 are separated from each
M
frictionless horizontal surface, other and a charge is placed between them. The whole
collides with a spring of spring constant k and system is at rest on the ground. Suddenly, the charge
compresses it by length L. The maximum momentum of explodes and the masses are pushed apart. The mass m1
the block, after collision is travels a distance S1 and then it stops. If the coefficient of
kL2 ML2 friction between the balls and the ground are the same,
(a) L Mk (b) (c) zero (d) mass m2 stops after covering the distance
2M k
m1 m2
(a) S2 = S1 (b) S2 = S1
14 A cyclist rides up a hill with a constant velocity. m2 m1
Determine the power developed by the cyclist, if the m2 m 2
(c) S2 = 1 2 S1 (d) S2 = 22 S1
length of the connecting rod of the pedal is r = 25 cm, the m2 m1
time of revolution of the rod is t = 2 s and the mean force
exerted by his foot on the pedal is F = 15 kgf. 19 A shell is fired from a cannon with a velocity v ms −1 at an
angle θ with the horizontal direction. At the highest point in
(a) 115.6 W (b) 215.6 W (c) 15.6 W (d) 11.56 W
its path, it explodes into 2 pieces of equal masses. One of
15 Power supplied to a particle of mass 2 kg varies with time the pieces retraces its path to the cannon. The speed in
as P = 3 t 2/ 2 W, where t is in second. If velocity of the ms −1 of the other piece, immediately after the explosion is
particle at t = 0 is v = 0, the velocity of the particle at t = 2 s, 3
(a) 3 v cosθ (b) 2 v cosθ (c) v cosθ (d) v cosθ
will be 2
(a) 1 ms −1 (b) 4 ms −1 (c) 2 ms −1 (d) 2 2 ms −1 20 A particle of mass m moving with a velocity u makes an
16 A body is moving unidirectionally under the influence of a elastic one dimensional collision with a stationary particle
source of constant power supplying energy. Which of the of mass m. They are in contact for a very short interval of
diagrams as shown in figure correctly shows the time T .
displacement-time curve for its motion?
d d F0

(a) (b) F

t t T/2
Time T
d d
T
The force of interaction increases from zero to F0 in and
(c) (d) 2
T
then decreases linearly to zero in further time interval .
2
t t The magnitude of F0 is
mu 2mu mu
17 Two identical ball bearings in contact with each other (a) (b) (c) (d) None of these
T T 2T
and resting on a frictionless table are hit head-on by
another ball bearing of the same mass moving initially 21 A block of mass 0.50 kg is moving with a speed of
with a speed v as shown in 2.00 ms −1 on a smooth surface. It strikes another mass of
1 2 3
1.00 kg and then they move together as a single body.
The energy loss during the collision is
v (a) 0.16 J (b) 1.00 J (c) 0.67 J (d) 0.34 J
If the collision is elastic, which of the following is a 22 A ball hits the floor and rebounds after an inelastic
possible result after collision?
collision. In this case
1 2 3 1 2 3 (a) the momentum of the ball just after the collision is the
(a) (b) same as that just before the collision
v=0 v/2 v=0 v (b) the mechanical energy of the ball remains the same in
the collision
(c) the total momentum of the ball and the earth is
1 2 3 1 2 3
conserved
(c) (d) (d) total mechanical energy of the ball and the earth is
v/3 conserved
v/1 v/2 v/3
DAY SIX WORK, ENERGY AND POWER 69

23 Two particles of masses m1 and m2 in projectile motion A B C


have velocities v1 and v2 respectively at time t = 0. They (a) 3 2 1
collide at time t 0. Their velocities become v1′ and v′2 at (b) 3 1 2
time 2t 0 while still moving in air. The value of (c) 1 2 3
(d) 2 3 1
| (m1 v′1 + m2 v′2 ) − (m1 v1 + m2 v2 )| is
(a) zero (b) (m1 + m2 )gt 0 Direction (Q. Nos. 29-32) These question consists of two
1
(c) 2 (m1 + m2 )gt 0 (d) (m1 + m2 ) gt 0 statements each printed as Statement I and Statement II.
2
While answering these questions you are required to choose
24 A ball is dropped on the ground from a height of 2m. If any one of the following five responses.
the coefficient of restitution is 0.6, the height to which the (a) If both Statement I and Statement II are correct and
ball will rebound is Statement II is the correct explanation of Statement I.
(a) 0.72 m (b) 1.72 m (c) 2.72 m (d) 1 m (b) If both Statement I and Statement II are correct but
25 A body of mass m is accelerated uniformly from rest to a Statement II is not correct explanation of Statement I.
speed v in a time interval T . The instantaneous power (c) If Statement I is correct but Statement II is incorrect.
delivered to the body as a function of time (t ), is given by (d) If Statement I is incorrect but Statement II is correct.
mv 2
mv 2
1 mv 2 2
1 mv 2 (e) If both Statement I and Statement II are incorrect
(a) 2
t (b) 2
t2 (c) t (d) t
T T 2 T2 2 T2 29 Statement I A body cannot have energy without having
26 A person trying to lose weight by burning fat lifts a mass momentum but it can have momentum without having
of 10 kg upto a height of 1 m, 1000 times. Assume that energy.
the potential energy lost each time he lowers the mass is Statement II Momentum and energy have same
dissipated. How much fat will he use up considering the dimensions.
work done only when the weight is lifted up? Fat supplies 30 Statement I When a machine-gun fires n bullets per
3.8 × 107 J of energy per kg which is converted into second with kinetic energy K, then the power of the
mechanical energy with a 20% of efficiency rate. machine-gun is P = nK .
(Take, g = 9.8 ms −2) ª JEE Main 2016 (Offline) Work nk
Statement II Power = =
(a) 2.45 × 10−3 kg (b) 6.45 × 10−3 kg Time 1
(c) 9.89 × 10−3 kg (d) 12.89 × 10−3 kg
31 Statement I A quick collision between two bodies is more
27 A time dependent force F = 6t acts on a particle of mass violent than a slow collision; even when the initial and the
1 kg. If the particle starts from rest, the work done by the final velocities are identical.
force during the first 1 s will be ª JEE Main 2017 (Offline) Statement II The momentum is greater in the first case.
(a) 22 J (b) 9 J
(c) 18 J (d) 4.5 J
32 Statement I A point particle of mass m moving with speed v
collides with stationary point particle of mass M. If the
28 Match the categories of energy given in column I with 1 
their formula given in column II and select the correct maximum energy loss possible is given as f  mv 2 , then
2 
option from the choices given below.
 M 
f = .
Column I Column II  M + m
A. Spring energy 1. 1
mv 2
2 Statement II Maximum energy loss occurs when the
B. Kinetic energy 2. mgh particles get stuck together as a result of the collision.
C. Potential energy 3. 1 2 ª JEE Main 2013
kx
2
70 40 DAYS ~ JEE MAIN PHYSICS DAY SIX

DAY PRACTICE SESSION 2

PROGRESSIVE QUESTIONS EXERCISE


1 A block of mass 0.5 kg has an initial velocity of 10 ms −1 5 A force F = − k ( y i + x j), acts on a particle moving in the
while moving down an inclined plane of angle 30°, the xy-plane. Starting from the origin, the particle is taken
coefficient of friction between the block and the inclined along the positive x-axis to the point (a , 0) and is then
surface is 0.2. The velocity of the block, after it covers a taken parallel to the y-axis to the point (a , a ). The total
distance of 10 m, is work done by the force is
(a) 17 ms −1 (b) 13 ms −1 (a) −2 ka 2 (b) 2 ka 2 (c) −ka 2 (d) ka 2
(c) 24 ms −1 (d) 8 ms −1
6 A uniform chain of length 2 m is kept on a table such that
2 Two inclined frictionless tracks, one gradual and the a length of 60 cm hangs freely from the edge of the table.
other steep meet at A from where two stones are allowed The total mass of the chain is 4 kg. What is the work done
to slide down from rest, one on each track as shown in in pulling the entire chain on the table?
figure. (a) 7.2 J (b) 3.6 J (c) 120 J (d) 1200 J
A
7 A 70 kg man leaps vertically into the air from a crouching
I II position. To take the leap the man pushes the ground
h
with a constant force F to raise himself.
q1 q2 The centre of gravity rises by 0.5 m before he leaps. After
B
C the leap the centre of gravity rises by another 1 m. The
Which of the following statement is correct ? maximum power delivered by the muscles is (take,
(a) Both the stones reach the bottom at the same time but g = 10 ms −2 ). ª JEE Main (Online) 2013
not with the same speed (a) 6. 26 × 103 W at the start (b) 6. 26 × 103 W at take off
(b) Both the stones reach the bottom with the same speed (c) 6. 26 × 104 W at the start (d) 6. 26 × 104 W at take off
and stone I reaches the bottom earlier than stone II 8 If two springs S1 and S 2 of force constants k1 and k 2,
(c) Both the stones reach the bottom with the same speed respectively are stretched by the same force, it is found
and stone II reaches the bottom earlier than stone I that more work is done on spring S1 than on spring S 2.
(d) Both the stones reach the bottom at different times and The correct option is
with different speeds
(a) w1 = w 2 (b) k1 < k2
3 Suppose the average mass of raindrops is 3 × 10−5 kg and (c) k1 > k2 (d) None of these
their average terminal velocity 9 ms −1. Then the energy
9 At time t = 0 s particle starts moving along the x-axis. If its
transferred by rain to each square metre of the surface at
kinetic energy increases uniformly with time t, the net
a place which receives 100 cm of rain in a year is
force acting on it must be proportional to
(a) 302 × 102 J (b) 102 × 105 J 1
(c) 4 . 05 × 104 J (d) 9. 2 × 103 J (a) t (b) constant (c) t (d)
t
4 A ball of mass m is dropped from a height h on a
10 It is found that, if a neutron suffers an elastic collinear
massless platform fixed at the top of a vertical spring as
collision with deuterium at rest, fractional loss of its
shown below. The platform is depressed by a distance x.
energy is Pd ; while for its similar collision with carbon
What will be the value of the spring constant?
nucleus at rest, fractional loss of energy is Pc . The values
m of Pd and Pc are respectively
h
(a) (.89, .28) (b) (.28, .89)
(c) (0, 0) (d) (0, 1)
11 A particle of mass m moving in the x-direction with speed
2v is hit by another particle of mass 2m moving in the
y-direction with speed v. If the collision is perfectly
inelastic, the percentage loss in the energy during the
(a) 2mg/x (b) 2mgh/x collision is close to ª JEE Main 2015
(c) 2 mg (h + x) / x 2 (d) 2 mg (h) + 2mghx/x 2 (a) 44% (b) 50% (c) 56% (d) 62%
DAY SIX WORK, ENERGY AND POWER 71

12 Consider a rubber ball freely falling from a height 17 A mass m moves with a velocity v and collides
h = 4.9 m onto a horizontal elastic plate. Assume that the inelastically with another identical mass. After collision,
duration of collision is negligible and the collision with the v
the 1st mass moves with velocity in a direction
plate is totally elastic. Then, the velocity as a function of 3
time the height as function of time will be perpendicular to the initial direction of motion. Find the
v v speed of the second mass after collision.
v1
2 v
(a) v (b) 3v (c) v (d)
+v1 3 3
(a) (b)
O t 18 The potential energy function for the force between two
O t
–v1 atoms in a diatomic molecule is approximately given by
a b
U( x ) = 12 − 6 , where a and b are constants and x is the
v v x x
distance between the atoms. If the dissociation energy of
+v1 +v1 the molecule is D = [U( x = ∞ ) − Uat equilibrium ], D is
(c) (d)
O
t1 3t1
t O
3t1 b2 b2 b2 b2
2t1 4t1 t1 2t1 4t1
t (a) (b) (c) (d)
2a 12a 4a 6a
–v1 –v1
19 Under the action of force, 3 kg body moves such that its
13 A body of mass m = 10 −2
kg is moving in a medium and t3
position x as a function of time t is given by x =
, where
experiences a frictional force F = −kv 2. Its initial speed is 3
1 x in meter and t in second. Then, work done by the force
v 0 = 10 ms −1. If after 10 s, its energy is mv 02, the value of
8 in the first 2 s is
k will be ª JEE Main 2017 (Offline) (a) 8 J (b) 16 J (c) 24 J (d) 30 J
(a) 103 kgs−1 (b) 10−4 kgm−1 20 When a rubber band is stretched by a distance x, it
(c) 10−1 kgm−1 s−1 (d) 10−3 kgm−1 exerts a restoring force of magnitude F = ax + bx 2, where,
14 A uniform chain of length l and weight w is hanging from a and b are constants. The work done in stretching the
its ends A and B which are close together. At a given unstretched rubber band by L is
instant end B is released. The tension at A when B has ª JEE Main 2014
l 1 2
fallen a distance x < is (a) aL + bL
2 3
(b) (aL + bL )
3
2 2
w  3x aL2 bL3 1  aL2 bL3 
− 2 3 − 3x 
w
(a) (b) (c) + (d)  + 
2  l  2  4  2 3 2 2 3 

⋅ 1 +
3x  w  3x
+ 4
w
(c) (d) 21 A spring gun having a spring of spring constant k is
2  l  2  l 
placed at a height h. A ball of mass m is placed in its
15 A body is moved along a straight line by a machine barrel and compressed by a distance x. Where shall we
delivering a constant power. The distance moved by the place a box on the ground, so that the ball lands in the
body in time t is proportional to box?
(a) t 3 / 4 (b) t 3 / 2 kh 2kh kh kh
(a) x (b) x (c) x (d)
1/ 4 1/ 2 mg mg 2mg 2mg
(c) t (d) t
16 In a collinear collision, a particle with an initial speed v 0 22 A particle of mass m is projected from the ground with an
strikes a stationary particle of the same mass. If the final initial speed u 0 at an angle α with the horizontal. At the
total kinetic energy is 50% greater than the original highest point of its trajectory, it makes a completely
kinetic energy, the magnitude of the relative velocity inelastic collision with another identical particle, which
between the two particles after collision, is was thrown vertically upward from the ground with the
ª JEE Main 2018 same initial speed u 0. The angle that the composite
v system makes with the horizontal immediately after the
(a) 0 (b) 2 v 0 collision is
4
v v0 π π π π
(c) 0 (d) (a) (b) +α (c) −α (d)
2 2 4 4 4 2
72 40 DAYS ~ JEE MAIN PHYSICS DAY SIX

23 Two small particles of equal masses start moving in (a) Statement I is correct, Statement II is correct; Statement
opposite directions from a point A in a horizontal circular II is the correct explanation for Statement I
orbit. Their tangential velocities are v and 2 v (b) Statement I is correct, Statement II is correct; Statement
respectively, as shown in the figure. Between collisions, II is not the correct explanation for Statement I
the particles move with constant speeds. After making (c) Statement I is correct; Statement II is incorrect
how many elastic collisions, other than that at A, these (d) Statement I is incorrect; Statement II is correct
two particles will again reach the point A? 24 Statement I Two bodies of different masses have the
v A same momentum, and their kinetic energies are in the
2v inverse ratio of their masses.
Statement II Kinetic energy of body is given by the
relation.
1
KE = mv 2
2
25 Statement I An object is displaced from point
(a) 4 (b) 3 (c) 2 (d) 1 A ( 2 m , 3 m , 4 m ) to a point B (1 m , 2 m , 3 m ) under a
Direction (Q. Nos. 24-25) Each of these questions constant force F = ( 2 i + 3 j + 4 k ) N. The work done by the
contains two statements : Statement I (Assertion) and force in this process is −9 J.
Statement II (Reason). Each of these questions also has four Statement II Work done by a force, an object can be
alternative choices, only one of which is the correct answer. given by the relation,
You have to select one of the codes (a), (b), (c), (d) given r2
W =∫ F⋅d r or W = F⋅s
below r1

ANSWERS

SESSION 1 1 (c) 2 (c) 3 (a) 4 (b) 5 (c) 6 (a) 7 (d) 8 (c) 9 (a) 10 (a)
11 (d) 12 (b) 13 (a) 14 (a) 15 (c) 16 (b) 17 (b) 18 (c) 19 (a) 20 (b)
21 (c) 22 (c) 23 (c) 24 (a) 25 (a) 26 (d) 27 (d) 28 (b) 29 (e) 30 (a)
31 (a) 32 (a)

SESSION 2 1 (b) 2 (c) 3 (c) 4 (c) 5 (c) 6 (b) 7 (b) 8 (b) 9 (d) 10 (a)
11 (c) 12 (c) 13 (b) 14 (c) 15 (b) 16 (b) 17 (c) 18 (c) 19 (c) 20 (c)
21 (b) 22 (a) 23 (c) 24 (a) 25 (a)
DAY SIX WORK, ENERGY AND POWER 73

Hints and Explanations


SESSION 1 Change in KE
or
mv 2
=
9
1 Resultant force, 1 1 2 4
= (6 × 103 ) × 12 − (5 × 103 )(1.2)2
2 2 3
F = 32 + 42 = 5N or v = ms −1
= − 600 J [minus sign for the loss 2
F 5
a= = = 2.5 m/s2 in kinetic energy]
m 2
vdv 10 From F = − dU
u = 0, t = 20 s 6 a = − kx or = − kx dx
dx U(x ) x x
1
∴ s = ut + at 2
⇒ v dv = − kxdx
∫0 dU = − ∫ F dx = − ∫ (kx )dx
0 0
2
1 Let the velocity change from v 0 to v. kx2
= 0 × 20 + × 2.5 × (20)2 = 500 m ∴ U ( x) = − as U(0) = 0
v x
2
Hence, work done
⇒ ∫v 0
dv = − ∫ k xdx
0
2
11 According to the work-energy theorem,
W = F ⋅ s = 5 × 500 = 2500 J v − v 20
2
k x2
⇒ =− W = ∆K
2 Work done in the round trip = total 2 2
Case I
work done against friction while  v 2 − v 20  mk x2 2
⇒ m  =− 1  v0 1
moving up and down the plane  2  2 −F ×3= m   − mv 20
= 2( µ mg cos θ) × s 2  2 2
⇒ ∆k ∝ x2 where, F is the resistive force and v 0 is
3
= 2 ( 01
. × 0.5 × 10 × × 10) [∆k is loss in kinetic energy] the initial speed.
2
 5 3 3 7 As, the earth moves ones around the Case II Let, the further distance
= 5 3 J Q cos θ = =  travelled by the bullet before coming to
10 2  sun in its elliptical orbit, its KE, is
 rest is s.
A maximum when it is closest to the sun
1
and minimum when it is farthest from ∴ − F (3 + s ) = K f − K i = − mv 20
the sun. Also, KE of the earth is never 2
10 zero during its motion choice (d) is 1 1
5 ⇒ − mv 20(3 + s ) = − mv 20
correct. 8 2
θ 1
8 ∴ Force = − dU or (3 + s ) = 1
O 4
5√3 B dr
3 s
d  −k  k or + =1
3 Work done among horizontal is ⇒ F =−   =− 3 4 4
dr  2r 2  r
dW = F ⋅ dx or s = 1 cm
As particle is on circular path, this force
or dW = (9 − x2 ) ⋅ dx must be centripetal force. 12 From the work-energy theorem, kinetic
For total work done, integrate both sides mv 2 energy of the block at a distance x is
with proper limit ⇒ |F |= x x x3
W
∫0 dW =
3
∫0(9 − x
2
)dx k
r
mv 2 1 k
K = ∫ 0 F dx = ∫ 0 (4 − x2 )dx = 4 x −
3
So, = ⇒ mv 2 = 2
3 r3 r 2 2r For kinetic energy to be maximum,
 x3  dK
or W = 9 x −  ∴ Total energy of particle = KE + PE =0
 3 0 dx
k k
= 2 − 2
d  x3 
=  27 −  − (0 − 0) = 18 J
27 2r 2r  4x −  =0
 3 =0 dx  3 
∴ Maximum work done is 18 J. Total energy = 0 4 − x2 = 0 or x = ± 2 m
4 Since, F = constant, 4  2
d2K
9 V ( x) =  x − x  J At x = + 2m , = negative
we can also use  4 2  dx2
W = F ⋅s For minimum value of V ,
dV
=0
i.e. Kinetic energy is maximum.
Here, s = r f − ri = (i$ + 2 $j + 3 k$ ) dx x3
K max = 4 x −
4 x3 2 x 3
−(2i$ + 3 $j + 4 k$ ) ⇒ − =0
4 2 23
= (− $i − $j − k$ ) = 4 (2) − = 5.33 J
⇒ x = 0, x = ± 1 3
∴ W = (2$i + 3 $j + 4 k$ ) ⋅ (− $i − $j − k$ ) 1 1 −1
So, V min ( x = ± 1) = − = J 13 According to the conservation of energy,
= −2 − 3 − 4 = −9 J 4 2 4
1 2 1
Now, K max + V min kL = Mv 2
5 If v ′ is the final velocity of the wagon, = total mechanical energy
2 2
then applying the principle of (Mv )2
⇒ kL =
⇒ K max =   + 2
2
1
conservation of linear momentum, we M
 4
get or MkL2 = p2 [Q p = Mv ]
5 × 103 × 1.2 = (5 × 103 + 103 ) × v ′ 9
or K max =
4 ⇒ p = L Mk
v ′ = 1 ms–1
74 40 DAYS ~ JEE MAIN PHYSICS DAY SIX

14 v = rω = r 2 π = 1 × 2 π = π ms −1 1
= 3 kg  
m
1

t 4 2 4
mu =
2
F0 × T ⇒ p= ∫0 6t dt s 
π mu
P = F × v = (15 × 9.8) × = 115.6 W ⇒ F0 = 2 Also, change in kinetic energy
4 T ∆p2 32
∆k = = = 4.5
15 From the work-energy theorem, 21 m1 u1 + m2 u2 = (m1 + m2 )v 2m 2×1
∆KE = W net ⇒ v = 2/3 ms −1 From work-energy theorem,
Kf − K i = ∫ P dt Energy loss Work done = change in kinetic energy
2 So, work done = ∆k = 4.5 J
mv 2 − 0 = ∫  t 2  dt = (0.5) × (2)2 − (1.5) ×  
1 2 3 1 1 2
2 0 2  2 2  3 28 Kinetic energy = 1 mv 2
1 3 2 = 0.67 J 2
(2)v 2 = ∫ t 2dt
2 2 0 Potential energy = mgh
2 22 In an inelastic collision only
1
3 t 3  momentum of the system may remain Spring energy = kx2
= ×  = 4 2
2  3 0 conserved. Some energy can be lost in
the form of heat, sound, etc. 29 We know that a body may not have
⇒ v = 2 ms −1
23 |(m1 v1′ + m2 v2′ ) − (m1 v1 + m2 v2) | momentum but may have potential
16 Here, P = [ ML2 T −3 ] = constant = |change in momentum of the two
energy by virtue of its position as in case
As mass M of body is fixed of a stretched or a compressed spring.
particles | But when the body does not contain
[L 2 T −3 ] = constant = |External force on the system | energy then its kinetic energy is zero
[L2 ] × time interval
⇒ = constant hence, its momentum is also zero.
[T3 ] = (m1 + m2 ) g (2t 0 ) = 2 (m1 + m2 ) gt 0 Dimensions of momentum
⇒ [L] ∝ [T3 /2 ] 24 Let v is the final velocity of ball on (mv ) = [MLT −1 ]
⇒ Displacement ∝t 3 /2 reaching the ground, then Dimensions of energy
v = 2 gh = 2 × g × 2  1 mv 2  = [ML2 T −2 ]
17 As the ball bearings are identical, their  
2 
masses are equal. In elastic collision, or v =2 g
n×K
their velocities are interchanged. In For upward motion 30 ∴Power = Work = =
nK
collision between 1 and 2; velocity of 1 u = 2 g × e and v = 0 Time 1 1
becomes zero and velocity of 2 becomes 31 Momentum p = mv or p ∝ v ,
∴ Height upto which the ball will
v. In collision between 2 and 3, velocity i.e. momentum is directly proportional to
rebound is
of 2 becomes zero and velocity of 3 the velocity, so the momentum is greater
u2 (2 g × e )
2
becomes v. H = = in a quicker collision between two bodies
2g 2g than in a slower one. Hence, due to
18 From the conservation of momentum, greater momentum, quicker collision
we get 4g × e 2
= = 2 × 0.6 × 0.6 = 072
. m between two bodies will be more violent
m1 v 1 = m2 v 2 …(i) 2g even if the initial and the final velocities
1 v − 0 are identical.
m1 v 12 = f1 S 1 = µ m1 g S 1 …(ii) 
also,
2 25 F = ma = mv Q a =  p2
T  T  32 Energy E = , where p is momentum,
1
and m2 v 22 = f2 S 2 = µm2 gS 2 …(iii) Instantaneous power = Fv = mav 2m
2
mv m is the mass moving of the particle.
where, µ = coefficient of friction = at
T Maximum energy loss occurs when the
On dividing Eq. (ii) by Eq. (iii), we get particles get stuck together as a result of
mv v mv 2
m1 v 12 mS = t= t the collision.
= 1 1 …(iv) T T T2
m2 v 22 m2S 2 Maximum energy loss (∆E)
Using Eqs. (i) and (iv), we get 26 Given, potential energy burnt by lifting p2 p2
= −
v1 mS weight = mgh = 10 × 9. 8 × 1 × 1000 2 m 2 (m + M )
= 1 1
v2 m2 S 2 = 9. 8 × 104
where, (m + M ) is the resultant mass
m2 mS m2 If mass lost by a person be m, then when the particles get stuck.
or = 1 1 ⇒ S 2 = 12 S 1 2
m1 m2S 2 m2 energy dissipated = m × × 3.8 × 107 J p2  m  p2  M 
10 ∆E = 1 − =  
2m  m + M  2m  m + M 
19 Velocity at the highest point = v cos θ 1
⇒ 9.8 × 104 = m × × 3.8 × 107
5 Also, p = mv
Applying the principle of conservation
5 m2 v 2  M 
of linear momentum, we get ⇒ m= × 10−3 × 9.8 ∴ ∆E =  
2m (v cos θ) = m(− v cos θ) + mv ′ 3.8 2m  m + M 
v ′ = 3 v cos θ = 12. 89 × 10−3 kg mv  M 
2
=  
20 The collision will cause an exchange of 27 From Newton’s second law, 2 m + M 
velocities. The change in momentum of ∆p Comparing the expression with
any particle = mu, which is equal to the = F ⇒ ∆p = F∆t
∆E = f  mv 2  , f =
∆t 1 M
impulse = area under the force-time 1 2  m+ M
graph ∴ p = ∫ dp = ∫0 F dt
DAY SIX WORK, ENERGY AND POWER 75

a a
So, coefficient of restitution, e = 1
SESSION 2 W2 = ∫ 0 F ⋅ d y = ∫ 0 − k ( yi +
$ x$j ) $j dy
Velocity of separation
−1 ∴ e =1=
1 Here, m = 0.5 kg, u = 10 ms , θ = 30°, a
= ∫ 0 − k(ai +
$ a$j )$j dy Velocity of approach
µ = 0.2 , s = 10 m v2 − v1
a
= − ka∫ dy = − ka2 ⇒ 1=
0 v0 − 0
⇒ v 0 = v2 − v1
m

Total work done, …(ii)


W = W1 + W2 = 0 − ka2 = − ka2 On adding Eqs. (i) and (ii), we get
θ=30° 2v 0
6 Mass per unit length = M 2v 0 = 3v 2 ⇒ = v2
Acceleration down the plane, 3
L
a = g (sin θ − µ cos θ) =
4
= 2 kg m –1 So, from Eq. (i), we get
= 10 (sin 30° − 0.2 cos 30° ) 2 v 1 = v 0 − 2v 2 = v 0 −
4v 0
1.4 m
= 3.268 ms −2 3
v
From second equation of motion, ⇒ v1 = − 0
v 2 = u2 + 2as 3

0.6 m
Fractional loss of energy of neutron
= 102 + 2 ( 3.268 ) × 10
 −K f + K i 
= 165.36 = 
 Ki  for neutron
v = 165.36
≈ 13 ms −1 1 1 v2
The mass of 0.6 m of chain = 0.6 × 2 − mv 12 + mv 20 − 0 + v 20
= 1.2 kg = 2 2 = 9
2 As, both surfaces I and II are frictionless 1 v 20
and two stones slide from rest from the The height of the centre of mass of the mv 20
2
same height, therefore, both the stones hanging part
=  − + 1 =
0.6 + 0 1 8
reach the bottom with same speed h= = 0.3 m = 0.88
 9  9
 1 mv 2 = mgh  . 2
  = 0.89
2  Hence, work done in pulling the chain
As acceleration down plane II is larger on the table = work done against the
m v0 12m m
(a2 = g sinθ2 is greater than force of gravity v1
a1 = g sin θ1 ), therefore, stone II reaches i.e. W = mgh = 1.2 × 10 × 0.3 = 3.6 J
the bottom earlier than stone I. + 12m v2
7 As, P = F ⋅v
3 Given, average mass of rain drop dP dv Similarly, for neutron-carbon atom
(m ) = 3. 0 × 10−5 kg So, =F⋅
dt dt collision;
Average terminal velocity = (v ) dP
To deliver the maximum power = 0, Momentum conservation gives;
= 9 ms −1 dt v 0 = v 1 + 12v 2 and e = 1
which gives ⇒ v 0 = v2 − v1
Height (h ) = 100 cm = 1m
Pmax = 6.26 × 103 W 11
Density of water ( ρ) = 103 kgm −3 So, v1 = v0
8 As no relation between k1 and k2 is 13
Area of the surface ( A ) = 1 m2 = A × h
∴ Loss of energy =  − + 1 = 0.28
given in the question. 121
= 1×1 Then, for same force  169 
= 1 m3 F F2 1 So, Pd = 0.89 and Pc = 0.28
W = Fx = F = ⇒W ∝
Mass of the water due to rain (M ) k k k
= Volume×density
11 Conservation of linear momentum can be
i.e. W1 > W2 ⇒ k1 < k2 applied but energy is not conserved.
= V × ρ = 1 × 103
9 Given, dk = constant ⇒ k ∝ t Consider the movement of two particles
= 103 kg dt as shown below :
∴ Energy transferred to the surface ⇒ v∝ t
y
1 1
= mv 2 = × 103 × (9)2 Also, p = Fv =
dk
= constant m 2v
2 2 dt x
= 40. 5 × 103 J 1 1
⇒ F∝ ⇒ F∝ v
= 4.05 × 104 J v t
2m
4 Here, mg (h + x ) = 1 kx2 10 Neutron-Deuterium collision;
2 (Just before collision)
⇒ kx2 = 2mg (h + x ) m 2m m 2m vy
2mg(h + x) v0 v=0 v1 v2 vx
⇒ k =
x2 Momentum conservation gives;
a a mv 0 = mv 1 + 2mv 2 3m
5 W1 = ∫ 0 F dx = ∫0 − k( yi +
$ x$j ) i$ dx
⇒ v 0 = v 1 + 2v 2 …(i)
a
= ∫ 0 − k(0i + x$j ) $i dx = zero
$ Collision given is elastic .
(Just after collision)
76 40 DAYS ~ JEE MAIN PHYSICS DAY SIX

According to conservation of linear Collision is perfectly elastic, then ball = Weight of half the chain
momentum in x-direction we have reaches to same height again and again x
+ Weight of length + F
( p1 )x = ( p1 )x or 2mv = (2m + m )v x with same velocity. 2
2 Hence, option (c) is true. w wx wx
or v 2 = v = + +
3 13 Given force, F = − kv 2 2 2l l
As, conserving linear momentum in −k 2 w  3x 
∴ Acceleration, a = v = 1+
y-direction, we get m 2  l 
( p i )y = ( p t )y dv −k 2 15 P = constant
or = v
or 2 mv = (2m + m )v x dt m ⇒ Fv = P [Q P = force × velocity]
2 dv k
or vy = v ⇒ =− dt ⇒ Ma × v = P
3 v2 m
P
Initial kinetic energy of the two ⇒ va =
Now, with limits, we have M
particles system is
v dv k t  v dv   v dv 
1 1
E = m(2v )2 + (2m )(v )2 ∫10 v 2 =−
m ∫0
dt ⇒v ×  =
P
Q a = ds 
2 2  ds  M
1 1 v v s P
− 1 = − k t
= × 4 mv 2 + × 2mv 2 ⇒ ∫ 0 v dv = ∫ 0 M ds
2
⇒  
2 2  v  10 m
= 2 mv 2 + mv 2 = 3 mv 2 [Assuming at t = 0 it starts from rest, i.e.
1 kt
Final energy of the combined two ⇒ = 01
. + from s = 0]
v m v3 P
particles system is 1 1 or = s
1 ⇒ v = = 3 M
E t = (3 m )(v 2x + v 2y ) kt 01
. + 1000 k
. +
01 1 /3
v = 
2 3P 
m ⇒  s1 /3
1  4 v2 4 v2  1 1 M
= (3 m )  +  ⇒ × m × v 2 = mv 20
2  9 9  2 8 ds   3P  
1 /3
v ⇒ = ks1 /3 k =   
3 m  8 v  4 mv
2 2 ⇒ v = 0 =5 M 
=
dt  
 = 2
2  9  3 1 s ds t

Loss in the energy,



. + 1000 k
01
=5 ⇒ ∫ 0 s1 /3 = ∫ 0 k d t
∆E = E i − E f ⇒ 1 = 0.5 + 5000 k s2 /3
0.5 or = kt
= mv 2 3 −  = mv 2
4 5 ⇒ k = 2/ 3
 3 3 5000 2
⇒ s2 /3 = k t
Percentage loss in the energy during the ⇒ k = 10−4 kg/m 3
3 /2
s =  k 
collision 14 Mass per unit length, 2
5 or × t 3 /2
mv 2 m w 3 
∆E 3 λ= =
× 100 = × 100 l lg or s ∝ t 3 /2
Ei 3 mv 2
v 2 = 2g  
x 16 Momentum is conserved in all type of
5 Velocity,
= × 100 = 56%  2 collisions,
9
or v = gx Final kinetic energy is 50% more than
12 As we know that for vertical motion, initial kinetic energy
1 2 Change in momentum when an element
h= gt [parabolic] w 1 1
2 dx falls is gx dx ⇒ mv 22 + mv 12
lg 2 2
v dp w dx w 150 1
F = = gx = x = × mv 20 …(i)
+v1 dt lg dt l 100 2
Q dx = gx 
3t1   m m
t  dt 
t1 2t1 v0
A B A
–v1
Before collision
m m
y l/2 x v1 v2
h
After collision
B
Conservation of momentum gives,
t
x/2 mv 0 = mv 1 + mv 2
v 0 = v2 + v1 …(ii)
v = − gt and after the collision,
v = gt (straight line). [Q v = u + gt ] Tension at A
DAY SIX WORK, ENERGY AND POWER 77

From Eqs. (i) and (ii), we have dv ∴ m(u 0 cos α )i$ + m( u20 − 2gH )$j
or = 2t [on difference above relation]
v 12 + v 22 + 2v 1 v 2 = v 02 dt
= (2m )v …(i)
or a = 2t as dv = a = acceleration 
− v 02  dt  u20 sin2 α
⇒ 2v 1 v 2 = H = …(ii)
2 ∴ F = ma = 6t 2g
∴ (v 1 − v 2 ) = (v 1 + v 2 ) − 4 v 1 v 2 =
2 2
2v 02 Hence, work done, put the value of F From Eqs. (i) and (ii), we get
and dx in Eq. (ii), we get
or v rel = 2v 0 2 u 0 cos α $ u 0 cos α $
2  t4  6 4 v= i + j
W = ∫0 6t × t dt =  6 × 4  = 4 [2 − 0 ]
2 4
17 In x-direction, 2 2
 0 Since, both components of v are equal.
Apply conservation of momentum, we 3
get = × 16 = 24 J Therefore, it is making 45° with
2 horizontal.
mu1 + 0 = mv x
⇒ mv = mv x 20 We know that change in potential 23 At first collision one particle having
energy of a system corresponding to a 4π 
⇒ vx = v conservative internal force as, speed 2v will rotate 240°  or  while
 3 
1 2 U f − U i = −W = − ∫ F ⋅ dr
other particle having speed v will rotate
2π 
F = ax + bx2 120°  or
Before collision Given,
 . At first collision they will
v/ 3 m We know that work done in stretching  3 
the rubber band by L is exchange their velocities. Now as shown
1 2 vx
|dW |=|Fdx| in figure, after two collisions they will
L again reach at point A.
vy |W|= ∫ (ax + bx2 )dx
After collision 0 A
L L v 2v
In y-direction, apply conservation of  ax2   bx3 
=  + 
momentum, we get  2 0  3 0
 v  v  aL2 a × (0)2 
0+ 0= m  − mv y ⇒ v y =
 3 3 = −  2p/3 2p/3
Velocity of second mass after collision,  2 2  2v v
2  b × L3 b × (0)3  2p/3
 v  4 2 + − 
v′=   + v =
2
v 1st collision 2nd collision
 3 3  3 3 
2 aL2 bL3 v 2v
or v′= v |W|= +
3 2 3
24 According to the principle of
18 U ( x ) = a12 − b6 21 From law of conservation of energy, we conservation of momentum
x x have v m
m1 v 1 = m2 v 2 ⇒ 1 = 2
U(x = ∞) = 0 v2 m1
dU  12a 6b  v 1
As, F =− = −  13 + 7  m1 v 12
x x  KE1
dx Again, = 2
At equilibrium, F = 0 KE2 1
m2 v 22
2a h 2
∴ x6 = 2
b m m  m
= 1 ×  2 = 2
a b − b2 m2  m1  m1
∴ U at equilibrium = − =
2
 2a  4a d
 2a    ∴ KE ∝
1
  b  1 1 k
b  mv 2 = kx2 ⇒ v = x m
2 2 2 m r2
b
∴ D = [U ( x = ∞ ) − U at equilibrium ] =
4a 2h
25 W = ∫r1
F ⋅ dr
Time taken to fall, t = (1m , 2m , 3m)
g
∫ (2m , 3m , 4m) (2 i + 3 j + 4 k$ ) ⋅
$ $
19 Here, m = 3 kg, t = 2 s, So, horizontal distance travelled is
t3 (dx $i + dy $j + dz k$ )
x= …(i) k 2h 2kh (1m , 2m , 3m)
3 d = vt = x x= x = [2 x + 3 y + 4z] (2m , 3m , 4m)
= −9 J
Work done, m g mg
Alternate
W = ∫ dW = ∫ F ⋅ dx …(ii) 22 From momentum conservation Since, F = constant, we can also use
Differentiate Eq. (i) w.r.t. time, then we equation, we have, W = F ⋅s
get Here, s = r f − ri
dx 3t 2 j = ( $i + 2$j + 3k$ ) − (2$i + 3$j + 4k$ )
= or dx = t 2dt
dt 3 u02 – 2gH
u0 cos a = (− $i − $j − k$ )
or v = t 2 m
as dx = v = velocity  i ∴ W = (2$i + 3$j + 4k$ )(− i$ − $j − k$ )
m
 dt  = − 2 − 3 − 4 = − 9J
pi = p f
DAY SEVEN

System of Particles
and Rigid Body
Learning & Revision for the Day
u Centre of Mass u Moment of Inertia u Moment of Inertia of Some
u Rigid Bodies u Theorems on Moment of Inertia Geometrical Oojects

Centre of Mass
Centre of mass of a system (body) is a point that moves when external forces are applied
on the body as though all the mass were concentrated at that point and all external forces
were applied there.

Centre of Mass of Two Particle System


Centre of mass of a two particles system consisting of two particles of masses m1, m2 and
respective position vectors r1 , r2 is given by
m r + m2 r2
rCM = 1 1
m1 + m2
r1 + r2
If m1 = m2 = m (say), then rCM =
2

Centre of Mass of n-Particle System


Centre of mass of rCM particles system which consists n-particles of masses m1, m2 ,..., mn PREP
with r1, r2 ,..., rn as their position vectors at a given instant of time is given by
n MIRROR
Σ mi ri Your Personal Preparation Indicator
m r + m2 r2 + K + mn rn i= 1
rCM = 1 1 =
m1 + m2 + K + mn M u No. of Questions in Exercises (x)—
u No. of Questions Attempted (y)—
No. of Correct Questions (z)—
Cartesian Components of the Centre of Mass
u

(Without referring Explanations)


The position vectors rCM and ri are related to their cartesian components by
Accuracy Level (z / y × 100)—
rCM = xCM i$ + yCM $j + zCM k$ and ri = x i i$ + yi $j + zi k$
u

u Prep Level (z / x × 100)—


The cartesian components of rCM are given by
n n n In order to expect good rank in JEE,
Σ mi x i Σ mi yi Σ mi zi your Accuracy Level should be above
i= 1 i= 1 i= 1
xCM = , yCM = and zCM = 85 & Prep Level should be above 75.
M M M
DAY SEVEN SYSTEM OF PARTICLE AND RIGID BODY 79

Motion of Centre of Mass Rigid Bodies


The position vector rCM of the centre of mass of n particle A rigid body is defined as that body which does not undergo
system is defined by any change in shape and volume when external forces are
m r + m2 r2 + m3 r3 + ... + mn rn applied on it. When a force is applied on a rigid body, the
rCM = 1 1
m1 + m2 + m3 ... mn distance between any two particles of the body will remain
unchanged, however, larger the forces may be.
1
= (m1r1 + m2 r2 + m3 r3 + ... + mn rn ) Coordinates of centre of mass of a rigid body are
M
1
drCM 1  dr dr dr  X CM = x dm,
=  m1 1 + m2 2 + ... + mn  M
dt M dt dt dt 
1
1 YCM = y dm
Velocity of centre of mass vCM = (m1v 1 + m2 v2 + ... mn v n ) M
M 1
n and ZCM = z dm
Σ mi vi M
i= 1
vCM =
M
Centre of Mass of Some Rigid Bodies
l
Similarly, acceleration of centre of mass is given by
n l
The centre of mass of a uniform rod is located at its
Σ mi a i mid-point.
i= 1
a CM =
M
l
From Newton’s second law of motion, CM
M a CM = F1 + F2 + ... Fn ⇒ M a CM = FExt
l
Centre of mass of a uniform rectangular, square or circular
plate lies at its centre.
For an isolated system, if external force on the body is
zero.
d
F = Ma CM = M (vCM ) = 0 ⇒ vCM = constant
dt
i.e. Centre of mass of an isolated system moves with CM CM
CM
uniform velocity along a straight line path and momentum
remain conserved. l
Centre of mass of a uniform semi-circular ring lies at a
2R
NOTE • If some mass or area is removed from a rigid body, then the distance of h = from its centre, on the axis of symmetry,
position of centre of mass of the remaining portion is π
obtained from the following formula where R is the radius of the ring.
m r − m2r2 A r − A2r2
rCM = 1 1 or rCM = 1 1
m1 − m2 A1 − A2
R CM 2R
Momentum Conservation π
O
Let us consider a system of particles of masses m1, m2 ... mn
are respective velocities v 1, v2 ... v n . The total linear l
Centre of mass of a uniform
momentum of the system would be the vector sum of the semi-circular disc of radius R
momentum of the individual particles. 4R R CM 4R
lies at a distance of h = from
3π 3p
i.e. p = p1 + p2 + p3 ... pn = m1v 1 + m2 v2 + .....+ mn v n
the centre on the axis of O
Velocity of centre of mass of a system
1 symmetry as shown in figure.
vCM = (m1v 1 + m2 v2 + ...... + mn v n ) l
Centre of mass of a
M
where, M is the total mass of the system, therefore hemispherical shell of radius R
R
p = MvCM lies at a distance of h = from
2
Thus, total linear momentum of a system of particles is equal
its centre on the axis of
to the product of the total mass of the system and the velocity
symmetry as shown in figure.
of its centre of mass.
dp dv l
Centre of mass of a solid
Again =M = Ma = Fext hemisphere of radius R lies at
dt dt
3R
If Fext = 0, then
dp
= 0, i.e. p = constant a distance of h = from its 3R
dt 8 8
centre on the axis of
If external force of a system is zero, then momentum of
symmetry.
system of particle remain constant.
80 40 DAYS ~ JEE MAIN PHYSICS DAY SEVEN

Moment of Inertia 2. Theorem of Perpendicular Axes


Moment of inertia of a rotating body is its property to oppose The sum of moment of inertia of a plane laminar body about
any change in its state of uniform rotation. two mutually perpendicular axes lying in its plane is equal to
If in a given rotational system particles of masses its moment of inertia about an axis passing through the point
m1, m2 , m3 ,K be situated at normal distances r1, r2 , r3 ,... from of intersection of these two axes and perpendicular to the
the axis of rotation, then moment of inertia of the system plane of laminar body.
about the axis of rotation is given by Z
I = m1r12 + m2 r22 + m3 r32 + ... = Σ mr 2 Planar Body
For a rigid body having continuous mass distribution
I = ∫ dm r 2
O N
Y
SI unit of moment of inertia is kg m2 . It is neither a scalar nor M
P(x, y)
a vector i.e. it is a tensor.
X
Radius of Gyration If I x and I y be moment of inertia of the body about two
Radius of gyration of a given body about a given axis of perpendicular axes in its own plane and I z be the moment of
rotation is the normal distance of a point from the axis, where inertia about an axis passing through point O and
if whole mass of the body is placed, then its moment of inertia perpendicular to the plane of lamina, then
will be exactly same as it has with its actual distribution of Iz = Ix + I y
mass. Thus, radius of gyration
1/ 2 In theorem of perpendicular axes, the point of intersection of
I  r 2 + r22 + r32 + K + rn2  the three axes ( x, y and z) may be any point on the plane.
K = or K =  1 
M  n 

Moment of Inertia of Some


m1 r3
m3 Geometrical Objects
r1
m4 M
l
Uniform Ring of Mass M and Radius R About an axis
r4 passing through the centre and perpendicular to plane of
m2
r2 P 1
K ring I = MR2 . About a diameter I = MR2
2
Axis of rotation l
Uniform Circular Disc of Mass M and Radius R About an
axis passing through the centre and perpendicular to plane
SI unit of radius of gyration is metre. 1 1
Radius of gyration depends upon shape and size of the body, of disc I = MR2 . About a diameter I = MR2
2 4
position and configuration of the axis of rotation and also on
distribution of mass of body w.r.t. axis of rotation. l
Thin Uniform Rod of Mass M and Length l About an axis
passing through its centre and perpendicular to the rod,
Theorems on Moment of Inertia I =
1
Ml 2
12
There are two theorems based on moment of inertia are given
below: l
Uniform Solid Cylinder of Mass M, Length l and Radius R
About its own axis,
1. Theorem of Parallel Axes 1
I = MR2 . About an axis passing through its centre and
I ICM 2
Moment of inertia of a body about a
given axis I is equal to the sum of  l 2 R2 
perpendicular to its length I = M  + 
moment of inertia of the body about a 12 4
parallel axis passing through its centre of A
CM 2
mass ICM and the product of mass of l
Uniform Solid Sphere About its diameter I = MR2 .
body (M ) and square of normal distance 5
between the two axes. Mathematically, 7
About its tangent I = MR 2
I = ICM + Md2 d 5
DAY SEVEN SYSTEM OF PARTICLE AND RIGID BODY 81

DAY PRACTICE SESSION 1

FOUNDATION QUESTIONS EXERCISE


1 A body A of mass M while falling vertically downwards string passed through the hand of the man while the
under gravity breaks into two parts; a body B of mass cylinder reached his hands is of
1 2
M and, a body C of mass M. The centre of mass of
3 3
bodies B and C taken together shifts compared to that of
body A towards
(a) depends on height of breaking
l 3l
(b) does not shift (a) (b) l (c) 2l (d)
(c) body C 2 2
(d) body B 7 A wheel has mass of the rim 1 kg, having 50 spokes each
of mass 5 g. The radius of the wheel is 40 cm. The
2 Distance of the centre of mass of a solid uniform cone
from its vertex is z 0. If the radius of its base is R and its moment of inertia
height is h, then z 0 is equal to ª JEE Main 2015 (a) 0.273 kg- m2 (b) 1.73 kg- m2
h 2
3h 5h 3h 2 (c) 0.173 kg- m2 (d) 2.73 kg- m2
(a) (b) (c) (d)
4R 4 8 8R 8 The surface density of a circular disc of radius a depends
3 A circular disc of radius R is removed from a bigger on the distance as ρ(r ) = A + Br . The moment of inertia
circular disc of radius 2 R, such that the circumference of about the line perpendicular to the plane of the disc is
(a) πa 4  +
A 2a 
(b) πa 4  +
the discs coincide. The centre of mass of the new disc is A 2B 
B 
α R from the centre of the bigger disc. The value of α is 2 5  2 5 
(c) 2 πa 3  +
1 1 1 1 A Ba 
(a) (b) (c) (d)  (d) None of these
3 2 6 4 2 5 

4 Which of the following points is the likely position of the 9 Four point masses, each of value m, are placed at the
centre of mass of the system as shown in figure? corners of a square ABCD of side l. The moment of inertia
of this system about an axis passing through A and
Hollow sphere
Air
parallel to BD is
(a) 2 ml 2 (b) 3 ml 2 (c) 3 ml 2 (d) ml 2
R/2 10 The ratio of the radii of gyration of a circular disc and a
A
B circular ring of the same radii about a tangential axis
C perpendicular to plane of disc or ring is
R/1 (a) 1 : 2 (b) 5 : 6
D 3
(c) 2 : 3 (d)
2
Sand
11 A uniform square plate has a small piece Q of an irregular
(a) A (b) B (c) C (d) D
shape removed and glued to the centre of the plate
5 Consider a two particles system with particles having leaving a hole behind. The moment of inertia about the
masses m1 and m2. If the first particle is pushed towards z-axis, then
the centre of mass through a distance d, by what
Y Y
distance should the second particle be moved, so as to
keep the centre of mass at the same position?
Q Hole
m2 m1 m
(a) d (b) d (c) − 1 d (d) d
m1 m1 + m2 m2 X X
6 A string of negligible thickness is wrapped several times
around a cylinder kept on a rough horizontal surface. A (a) increased
man standing at a distance lm from the cylinder holds (b) decreased
one end of the string and pulls the cylinder towards him. (c) same
There is no slipping anywhere. The length in (m ) of the (d) changed in unpredicted manner
82 40 DAYS ~ JEE MAIN PHYSICS DAY SEVEN

12 Consider a uniform square plate of side a and mass m. The moment of inertia of the system of four spheres about
The moment of inertia of this plate about an axis diagonal AB is
perpendicular to its plane and passing through one of its m m
(a) (8r 2 + 5 a 2 ) (b) (7r 2 + 4 a 2 )
corners is 5 5
m m
(a)
5
ma 2 (b)
1
ma 2 (c)
7
ma 2 (d)
2
ma 2 (c) (5 r 2 + 8 a 2 ) (d) (3 r 2 + 5 a 2 )
6 12 12 3 5 5

13 For the given uniform square lamina ABCD, whose centre 17 From a uniform circular disc of radius R and mass 9 M, a
R
is O. small disc of radius is removed as shown in the figure.
3
F
D C The moment of inertia of the remaining disc about an axis
perpendicular to the plane of the disc and passing
through centre of disc is ª JEE Main 2018
O

A B 2R
E 3

(a) 2 I AC = IEF (b) I AD = 3IEF


R
(c) I AC = IEF (d) I AC = 2 IEF
14 A solid sphere of radius R has moment of inertia I about
its geometrical axis. It is melted into a disc of radius r 40 37
(a) 4MR 2 (b) MR 2 (c) 10MR 2 (d) MR 2
and thickness t. If it’s moment of inertia about the 9 9
tangential axis (which is perpendicular to plane of the 18 A child with mass m is standing at the edge of a
disc), is also equal to I, then the value of r is equal to merry-go-round having moment of inertia I, radius R and
I initial angular velocity ω 0 as shown in the figure.
v
ω0
R

2 2 3 3
(a) R (b) R (c) R (d) R
15 5 15 15
The child jumps off the edge of the merry-go-round with
15 The moment of inertia of a system of four rods each of tangential velocity v w.r.t. ground. The new angular
length l and mass m about the axis shown is velocity of the merry-go-round is
1 1
I1 I4  I ω2 − mv 2  2  (I + mR 2 )ω20 − mv 2  2
(a)  0  (b)  
 I   I 
I ω0 − mvR   (I + mR 2 ) ω0 − mvR 
(c)   (d)  
 I  I
I2 I3  

(a)
2 2
ml (b) 2 ml 2 (c) 3 ml 2 (d)
8 2
ml Direction (Q. Nos. 19-20) are the Assertion and Reason
3 3 type. Each of the these contains two Statements; Assertion
16 Four solid spheres each of mass m and radius r are and Reason. Each of these question also has four alternative
choice,only one of which is correct. You have to select the
located with their centres on four corners of a squares
correct choices from the codes (a),(b), (c) and (d) given below:
ACBD of side a as shown in the figure.
(a) If both Assertion and Reason are true and the Reason is
A C correct explanation of the Assertion
(b) If both Assertion and Reason are true but Reason is not
a correct explanation of the Assertion
(c) If Asserion is true but Reason is false
D B (d) If both Assertion and Reason are false
DAY SEVEN SYSTEM OF PARTICLE AND RIGID BODY 83

19 Assertion (A) A constant force F is applied on the two the axis is moved downwards, moment of inertia of the
blocks and one spring system as shown in the figure. slab will first decreases, then increases.
Velocity of centre of mass increases linearly with time. Reason (R) Axis is first moving towards its centre of
F mass, then it is receding from it.
m 2m
A
Smooth

Reason (R) Acceleration of centre of mass is constant.


20 Assertion (A) There is a triangular plate as shown in the B C
figure A dotted axis is lying in the plane of the slab. As

DAY PRACTICE SESSION 2

PROGRESSIVE QUESTIONS EXERCISE


1 Mass of bigger disc having radius 2R is M. A disc of 4 Seven identical circular planar discs, each of mass M
radius R is cut from bigger disc as shown in figure. and radius R are welded symmetrically as shown in the
Moment of inertia of disc about an axis passing through figure. The moment of inertia of the arrangement about
periphery and perpendicular to plane is the axis normal to the plane and passing through the
point P is
2R
P
M
m
R O

27 MR 2 29 MR 2
(a) (b)
8 8
ª JEE Main 2018
(c) 3.5 MR (d) 2MR 2 19 55
(a) MR 2 (b) MR 2
2 Four spheres of diameter 2a and mass M are placed with 2 2
their centres on the four corners of a square of side b. 73 181
(c) MR 2 (d) MR 2
Then the moment of inertia of the system about an axis 2 2
along one of the sides the square is
5 The moment of inertia of a uniform cylinder of length l and
A B radius R about its perpendicular bisector is I. What is the
ratio l/R such that the moment of inertia is minimum?
b ª JEE Main 2017 (Offline)
3
(a) (b) 1
D C 2
3 3
4 8 (c) (d)
(a) Ma 2 + 2Mb 2 (b) Ma 2 + 2Mb 2 2 2
5 5
8 4 6 A particle moves parallel to X-axis with constant velocity v
(c) Ma 2 (d) Ma 2 + 4 Mb 2
5 5 as shown in the figure. The angular velocity of the particle
about the origin O is
3 From a solid sphere of mass M and radius R, a cube of
maximum possible volume is cut. Moment of inertia of Y
cube about an axis passing through its centre and
perpendicular to one of its faces is ª JEE Main 2015
v
MR 2 MR 2
(a) (b)
32 2 π 16 2 π q
X
2 2 O
4 MR 4 MR
(c) (d)
9 3π 3 3π (a) remains constant (b) continuously increasing
(c) continuously decreasing (d) oscillates
84 40 DAYS ~ JEE MAIN PHYSICS DAY SEVEN

7 Let I1 and I 2 be the moment of inertia of a uniform square k m2 x0


(a) (b) km2
plate about an axis as shown in the figure. Then, the ratio m1 + m2 m1 + m2
I1 : I 2 is  m + m2  km1
(c) x 0k  1  (d) x 0
1  m2  m1 + m2
2 10 A racing bike is travelling along a straight track at a
constant velocity of 40 m/s. A fixed CCTV camera is
recording the event as shown in the figure. In order to
keep the bike in view, in the position shown, the angular
velocity of the CCTV camera should be

1 12 7
(a) 1 : (b) 1 : (c) 1 : (d) 1 : 7
7 7 12
8 A thin rod of length 4l, mass 4m is bent at the points as 40 m/s
shown in the figure. What is the moment of inertia of the
rod about the axis passing through O and perpendicular
to the plane of the paper? 30m

O 30º
l
90º 90º
l CCTV

(a) 3 rad/s
ml 2 10 ml 2 ml 2 ml 2 (b) 2 rad/s
(a) (b) (c) (d)
3 3 12 24 (c) 1 red/s
(d) 4 rad/s
9 Two bars of masses m1 and m2 connected by a
weightless spring of stiffness k, rest on a smooth 11 A rod of length L is placed along the X-axis between
horizontal plane. Bar 2 is shifted by a small distance x = 0 and x = L. The linear mass density (mass/length) ρ
x 0 to the left and released. The velocity of the centre of of the rod varies with the distance x from the origin as
mass of the system when bar 1 breaks off the wall is ρ = a + bx. Here, a and b are constants. The position of
centre of mass of the rod is
 3aL + 2bL2   6aL + 3bL2 
(a)  , 0, 0 (b)  , 0, 0
 6a + 3bL   3a + 2bL 
1 2  aL + bL2 
(c)  , 0, 0 (d) None of these
m1 m2  2a + 3bL 

ANSWERS
SESSION 1 1 (b) 2 (b) 3 (a) 4 (c) 5 (c) 6 (c) 7 (c) 8 (a) 9 (c) 10 (d)
11 (b) 12 (d) 13 (c) 14 (a) 15 (d) 16 (a) 17 (a) 18 (d) 19 (a) 20 (a)

SESSION 2 1 (b) 2 (b) 3 (c) 4 (d) 5 (d) 6 (c) 7 (d) 8 (b) 9 (a) 10 (c)
11 (a)
DAY SEVEN SYSTEM OF PARTICLE AND RIGID BODY 85

Hints and Explanations


SESSION 1 dr1 and dr2 represent the change in 3
displacement of particles. K disc 2 = 3
1 The position of centre of mass remains =
unaffected because breaking of mass Let second particle has been displaced K ring 2 2
into two parts is due to internal forces. by distance x. 11 According to the theorem of
md perpendicular axes, I z = I x + I y with the
2 We know that centre of mass of a ⇒ m1 (d ) + m2 ( x ) = 0 or x = − 1
uniform solid cone of height (h ) is at m2 hole, I x and I y , both decreases (gluing the
h removed piece at the centre of square
height from base, therefore 6 If velocity of 2v
4 plate does not
h h 3h centre of mass
As, h − z 0 = or z 0 = h − = is v, then affect I z ).
4 4 4 v Hence, I z decreases overall.
velocity of
3 In this question, distance of centre of contact is 0
12 Moment of inertia of square plate about
mass of new disc from the centre of and that of the
ma2
mass of remaining disc is αR. top is 2 v, xy is . Moment of inertia about zz′
hence when 6
centre of mass covers a distance l, can be computed using parallel axes
thread covers a distance 2l. theorem
2 z¢ y
O
R 7 I = mr 2 + 50 ml
O2 O1 3
50 (5 × 10−3 ) (0.4)2
αR = 1 × (0. 4) +
2

3 z
= 016
. (1.083)
Mass of original disc = M
= 0173
. kg- m2
Mass of disc removed a
x
=
M
× π R2 =
M 8 dm = 2 πr dr (ρ) = ( A + Br ) (2 πr dr )
π (2R)
2 2
π Aa4 2 πBa5
I zz ′ = I xy + m 
4 a
a
I = ∫ dmr 2 = + 
2 5  2
Mass of remaining disc 0

M 3M 4  A 2a  ma2 ma2
=M − = = πa  + B = +
4 4 2 5  6 2
2
3M M 2ma
∴ − α R+ R=0 9 The situation is as shown in figure. =
4 4 3
1 x
⇒ α = Q 13 Let the each side of square lamina
3
is d.
Note In this question, the given distance
So, I EF = IGH (due to symmetry)
must be αR for real approach to the A
solution.
B and I AC = I BD (due to symmetry)

4 Centre of mass of a system lies towards D F C


P
that part of the system, having bigger
mass. In the given diagram, lower part x¢
is heavier, hence CM of the system lies
D C G H
below the horizontal diameter. O
Hence, (c) is correct option. I xx ′ = m × DP + m × BQ + m × CA
2 2 2

5 To keep the centre of mass at the same  2l


= m×2×   + m × ( 2 l )2 A E
B
position, velocity of centre of mass is  2  d
zero, so = 3 ml 2
m1 v1 + m 2 v2 Now, according to theorem of
=0
m1 + m 2 10 Radius of gyration, perpendicular axes,
l I AC + I BD = I 0
where, v1 and v2 are velocities of K =
m
particles 1 and 2 respectively. ⇒ 2 I AC = I 0 …(i)
dr dr 1
⇒ m1 1 + m2 2 = 0 mR2 + nR2 and I EF + IGH = I 0
dt dt 2 3
⇒ K disc = = R
Q v = dr1 and v = dr2  R 2 ⇒ 2 I EF = I 0 …(ii)
 dt 
1 2
dt From Eqs. (i) and (ii), we get
mR2 + mR2
⇒ m1 dr1 + m2 dr2 = 0 K ring = = 2R
m I AC = I EF
86 40 DAYS ~ JEE MAIN PHYSICS DAY SEVEN

14 2 MR2 = 1 Mr 2 + Mr 2 or v CM =
F
× t or v CM ∝ t
5 2 3m
S R
2 3 Hence, both Assertion and Reason are
or MR2 = Mr 2 2R R M
5 2 true and the Reason is the correct O
2 explanation of Assertion. Q v
∴ r = R
15 20 Moment of inertia (I ) = mr 2 , where r is
Cross-sectional view of the
15 Consider a square lamina, then distance from the axis of rotation to the
cube and sphere
l l 
2 2
centre of mass. When dotted axis moved
4m  +  about
 12 12  downward (towards centre of mass), r Using geometry of the cube
2R
2ml 2 decreases result moment of inertia PQ = 2R = ( 3 ) a or a =
Centre of mass = decrease and when dotted axis cross the 3
3
centre of mass and moved further Volume density of the solid sphere,
downwards then r increases result Y
moment of inertia increases. Hence, I P
Ö2 l both Assertion and Reason are true and R
S
the Reason is the correct explanation of a
l/Ö2 Assertion.
O
W
SESSION 2 V
Q V
1 Surface density of motional disc is
Apply perpendicular axis theorem, M 3 M
σ =
M
=
M ρ= =  3
 
πR 3 3 π R
2
4
+ 4m 
2ml 2 l  8 π(2R )2 4 πR 2
=  = ml
2

3  2 3 3
Mass of cutting portion is
M Mass of cube (m ) = (ρ)(a)3
16 The moment of inertia of the system of m1 = σ × πR2 = 3

=  × 3   
4 3 M 2R
four spheres about diagonal AB is
I = I1 − I2  4 π R   3 
I AB = MI of A about AB + MI of B
where, 3M 8R3 2M
about AB + MI of C = × =
about AB + MI of D about AB I1 = Moment of inertia of disc about 4 πR 3
3 3 3π
given axis without cutting portion Moment of inertia of the cube about
= mr 2 + mr 2 +  mr 2 + ma2 
2 2 2 1
5 5 5 2  I2 = Moment of inertia due to given axis is
cutting portion m 2 ma2
+  mr 2 + ma2  ly = (a + a2 ) =
2 1
5 2  M (2R )2  m R2  12 6
I = + M (2R )2 −  1 + m1 (3R )2  ma2 2M 1 4R2 4MR2
8  8r2  2  2  ⇒ Iy = = × × =
= mr 2 + ma2 = m  + a2  6 3π 6 3 9 3π
5  5  19MR 2 29MR 2
= 6MR − 2
=
m 8 8 4 Moment of inertia of an outer disc about
= ( 8 r + 5a )
2 2
the axis through centre is
5 2 a b
MR2
+ M (2R )2 = MR2  4 +  = MR2
a 1 9
A B =
17 Moment of inertia of remaining solid 2  2 2
= Moment of inertia of complete b b

solid − Moment of inertia of


a
removed portion D
a b C M
9MR2  M (R / 3)2 2R  
2

∴ I = − + M   
Moment of inertia of each of the spheres
2 2  3  A and D about
 
2
⇒ I = 4MR 2 AD = Ma2
5 R R
18 Since, in this condition, Moment of inertia of each of the spheres
Initial angular momentum B and C about AD
For 6 such discs,
= Final angular momentum =  Ma2 + Mb 2 
2
9
∴(I + mR2 ) ω 0 = (mvR ) + I ω ′ 5  Moment of inertia = 6 × MR2
2
(I + mR2 )ω 0 − mvR Using theorem of parallel axis, we get
or ω′ = = 27MR2
I Total moment of inertia So, moment of inertia of system
Hence, (d) is the correct option.
= I  Ma2  × 2 +  Ma2 + Mb 2  × 2 MR2
2 2 55
 5   5  = + 27MR2 = MR2
19 Total mass of the system 2 2
= m + 2m = 3m Hence, I P =
55
8 MR2 + (7M × 9R2 )
Force applied on the system is F . I = Ma2 + 2Mb 2 2
5
F 181
∴ a CM = ⇒ IP = MR2
3m 3 Consider the cross-sectional view of a 2
= constant as F is constant diametric plane as shown in the 181
I system = MR2
∴ v CM = a CM × t adjacent diagram. 2
DAY SEVEN SYSTEM OF PARTICLE AND RIGID BODY 87

2
5 MI of a solid cylinder about its 7 Here, I1 = ma and I2 = I1 + mr 2 10 Here, tan θ = x or x = 30 tan θ
perpendicular bisector of length is 12 30
x
 l2 R2  From figure, we get
I =m + 
 12 4 a
r =
mR2 ml 2 2
⇒ I = + 1 30 m
4 12
m2 ml 2 a q
= + [Q ρπR2 l = m] 2
4 πρl 12
For I to be maximum, Now, differentiate both sides w.r.t. time,
r
dI m2  1  ml dx dθ
=−   + =0 = 30 × sec2 θ ⋅
dl 4 πρ  l 2  6 dt dt
dx dθ
m2 ml 3 But = v bike and =W
⇒ = dt dt
4 πρ 6
ma2 ma2 7 v bike 40
Now, putting m = ρπR2 l ∴ I2 = + = ma2 ∴ W = =
12 2 12 30sec2 θ 30 sec2 30°
3
∴ l3 = ⋅ ρπR2 l Hence, I1 : I2 = 1 : 7 or W = 1 rad/s
2 πρ
Thus, (d) is the correct option. Hence, (c) is the correct option.
l2 3
=
R2 2 8 From figure, 11 Let dm is the mass of element AB of
l 3 O length dx at distance x
∴ =
R 2 r ∴ dm = ρ ⋅ dx = (a + bx ) ⋅ dx
l/2 A B
6 Since, ω = v l l l
r
But, here l2 5 x=0 dx x=L
v = v sin θ r = l2 + = l
4 2 The centre of mass of the element has
and r = b / sinθ Due to symmetry, coordinates ( x, 0, 0) .
 ml 2  ml 2  Therefore, x-coordinate of centre of mass
q

I = 2 [ I1 + I2 ] = 2  +  + mr 2  
os

 3  12  of the rod will be


vc

L L
q
v =2
 ml 3
+
ml 2
+
5ml  10 2
2
= ml xCM =
∫ 0
x ⋅ dm
=
∫ x (a + bx ) dx
0
4  3 L L
r
b
 3 12
∫ 0
dm ∫ (a + bx ) dx
0
v sin q Hence, (b) is the correct option. L
q  ax2 bx3 
9 Here, 1 kx20 = 1 m2 v 22  2 + 3 
2 2 =  0
L
v sin θ k  bx2 
∴ ω= ∴ v2 = x0  ax + 2 
b / sin θ m2  0
v sin2 θ m v + m2 v 2 3aL + 2bL2
= Since, v CM = 1 1 or xCM =
b m1 + m2 6a + 3bL
or ω ∝ sin2 θ y CM = 0 ,
k
m1 × 0 + m2 x0 zcom = 0
[as v and b are constants] m2
∴ v CM = ∴Centre of mass of the rod will be
∴When particle moves further m1 + m2  3aL + 2bL2 
parallel to the X-axis, θ decreases result k m2  6a + 3bL , 0, 0
= ⋅ x0  
angular velocity decreases. m1 + m2 Hence, (a) is the correct option.
Hence, (c) is the correct option. Hence, (a) is the correct option.
EXAM BITES

This Pdf Is
Downloaded From
www.exambites.in

Visit www.exambites.in for


More Premium Stuffs,Latest
Books,Test Papers,Lectures etc.
jeeneetadda
jeeneetadda_official
jeeneetadda

VISIT NOW !!
DAY EIGHT

Rotational
Motion
Learning & Revision for the Day
u Concept of Rotational Motion u Angular Momentum u Equilibrium of a Rigid
u Equation of Rotational Motion u Law of Conservation Bodies
u Moment of Force or Torque of Angular Momentum u Rigid Body Rotation

Concept of Rotational Motion


In rotation of a rigid body about a fixed axis, every particle of the body moves in a circle,
which lies in a plane perpendicular to the axis and has its centre on the axis.
l
Rotational motion is characterised by angular displacement dθ and angular velocity

ω= .
dt
l
If angular velocity is not uniform, then rate of change of angular velocity is called the
angular acceleration.

Angular acceleration, α = .
dt
SI unit of angular acceleration is rad/s2.
l
Angular acceleration α and linear tangential acceleration a t are correlated
PREP
as a t = α × r. MIRROR
Your Personal Preparation Indicator

Equation of Rotational Motion u

u
No. of Questions in Exercises (x)—
No. of Questions Attempted (y)—
If angular acceleration α is uniform, then equations of rotational motion may be written u No. of Correct Questions (z)—
as (Without referring Explanations)
1
(i) ω = ω 0 + α t (ii) θ = ω 0 t + α t 2
2 u Accuracy Level (z / y × 100)—
α u Prep Level (z / x × 100)—
(iii) ω − ω 0 = 2 αθ
2 2
(iv) θ nth = ω 0 + (2 n − 1)
2
In order to expect good rank in JEE,
your Accuracy Level should be above
85 & Prep Level should be above 75.
DAY EIGHT ROTATIONAL MOTION 89

Moment of Force or Torque Law of Conservation


Torque (or moment of a force) is the turning effect of a force of Angular Momentum
applied at a point on a rigid body about the axis of rotation.
According to the law of conservation of angular momentum, if
n = r F sin θ ^
Mathematically, torque, τ = r × F = | r × F|^ n no external torque is acting on a system, then total vector sum
where, ^ n is a unit vector along the axis of rotation. Torque is of angular momentum of different particles of the system
an axial vector and its SI unit is newton-metre (N-m). remains constant.
dL
l
The torque about axis of rotation is F We know that, = τext
dt
independent of choice of origin O, so long
as it is chosen on the axis of rotation AB. dL
Hence, if τext = 0, then = 0 ⇒ L = constant
l
Only normal component of force O r dt
θ A
contributes towards the torque. Radial Therefore, in the absence of any external torque, total angular
B
component of force does not contribute momentum of a system must remain conserved.
towards the torque.
Comparison of Linear and Rotational Motion
l
A torque produces angular acceleration
in a rotating body. Thus, torque, τ = Iα Linear Motion Rotational Motion
l
Moment of a couple (or torque) is given by product of 1. Linear momentum, p = mv Angular momentum, L = Iω, L = 2 IE
position vector r between the two forces and either force F. 2. Force, F = ma Torque, T = Iα
Thus, τ = r × F 3. Electric energy, E = 1 mv 2 1 2
Rotational energy, E = Iω
l
If under the influence of an external torque, τ the given 2 2
body rotates by dθ, then work done, dW = τ ⋅ dθ.
l
In rotational motion, power may be defined as the scalar
product of torque and angular velocity, i.e. Power P = τ ⋅ ω. Equilibrium of a Rigid Bodies
For mechanical equilibrium of a rigid body, two condition
Angular Momentum need to be satisfied.
The moment of linear momentum of a given body about an axis 1. Translational Equilibrium
of rotation is called its angular momentum. If p=mv be the
linear momentum of a particle and r is its position vector from A rigid body is said to be in translational equilibrium, if it
the point of rotation, then remains at rest or moving with a constant velocity in a
particular direction. For this, the net external force or the
Angular momentum, L = r × p = r psin θ ^
n = mvr sin θ n$ vector sum of all the external forces acting on the body must
where, ^
n is a unit vector in the direction of rotation. Angular be zero,
momentum is an axial vector and its SI unit is kg-m2s −1 or J-s. i.e. F = 0 or F = ΣFi = 0
l
For rotational motion of a rigid body, P
angular momentum is equal to the product 2. Rotational Equilibrium
of angular velocity and moment of inertia of A rigid body is said to be in rotational equilibrium, if the body
the body about the axis of rotation. does not rotate or rotates with constant angular velocity. For
Mathematically, L = Iω. r this, the net external torque or the vector sum of all the
l
According to the second law of rotational torques acting on the body is zero.
motion, the rate of change of angular For the body to be in rotational equilibrium,
momentum of a body is equal to the external torque dL
τext = 0, = 0 or ∑ τ i = 0
applied on it and takes place in the direction of torque. dt
Thus,
dL d dω Q α = dω  Rigid Body Rotation
τ= = (Iω) = I = Iα 
dt dt dt dt 
l
Total effect of a torque applied on a rotating body in a Spinning
given time is called angular impulse. Angular impulse is When the body rotates in such a manner that its axis of rotation
equal to total change in angular momentum of the system does not move, then its motion is called spinning motion.
in given time. Thus, angular impulse, 1
In spinning rotational kinetic energy is given by, K R = Iω2 .
∆t 2
J =∫ τ dt = ∆L = L f − Li Rotational kinetic energy is a scalar having SI unit joule (J).
0
Rotational kinetic energy is related to angular momentum as
l
The angular momentum of a system of particles about the per relation,
n
origin is L = ∑ ri × pi L2
KR = or L = 2 IK R
2I
90 40 DAYS ~ JEE MAIN PHYSICS DAY EIGHT

2 gh
Pure Rolling Motion l
Velocity at the lowest point v =
K2
Let a rigid body, having symmetric surface ω 1+ 2
R
about its centre of mass, is being spined at a
certain angular speed and placed on a surface, CM v l
Acceleration in motion From second equation of motion,
so that plane of rotation is perpendicular to v2 = u2 + 2 as
the surface. If the body is simultaneously h 2 gh
given a translational motion too, then the net By substituting u = 0, s = and v = , we get
sin θ K2
motion is called rolling motion. 1+ 2
R
The total kinetic energy in rolling motion,
g sin θ
K N = K R + KT a=
K2
1 1 1 1  K2  1+ 2
= mv2 + Iω2 = mv2 + mv2  2  R
2 2 2 2 R  l
Time of descent From first equation of motion, v = u + at
1  K2 
KN = mv2 1 + 2  By substituting u = 0 and value of v and a from above
2  R 
1 2h  K2 
expressions t = 1 + 2 
sin θ g  R 
Rolling Without Slipping
From the above expressions, it is clear that,
If the given body rolls over a surface such
that there is no relative motion between the 1 1 K2
v∝ ;a ∝ ; t ∝ 1 +
body and the surface at the point of contact, K2 K2 R2
1+ 1+ 2
then the motion is called rolling without P
Rω v R2 R
slipping.
Important Terms Related to Inclined Plane
Impure Rolling Motion  K2 
l
Here, factor  2  is a measure of moment of inertia of a
In impure rolling motion, the point of contact of the body with R 
the platform is not relatively at rest w.r.t. platform on which, body and its value is constant for given shape of the body
it is performing rolling motion, as a result sliding occurs at and it does not depend on the mass and radius of a body.
point of contact.
l
Velocity, acceleration and time of descent (for a given
For impure rolling motion, v AB ≠ 0 i.e. v − Rω ≠ v0
K2
If platform is stationary, i.e. v0 = 0, then v ≠ Rω inclined plane) all depends on 2 . Lesser the moment of
R
K2
inertia of the rolling body lesser will be the value of 2 . So,
R
ω v ⇒ greater will be its velocity and acceleration and lesser will
be the time of descent.
A
v = Rω
v0 v0
Rotation about Axis through
B
Centre of Mass (Centroidal Rotation)
When an object of given mass is tied to a
R
light string wound over a pulley whose
Rolling on an Inclined Plane moment of inertia is I and radius R as shown O
When a body of mass m and radius R in the figure. The wheel bearing is
rolls down on inclined plane of R frictionless and the string does not slip on T
height h and angle of inclination θ, it the rim, then
S h m a
loses potential energy. However, it Tension in the string is
acquires both linear and angular
θ I
speeds and hence gain kinetic energy T = × mg mg
of translation and that of rotation. I + mR2
 K2  mR2
1
By conservation of mechanical energy, mgh = mv2 1 + 2  and acceleration, a = ⋅g
2  R  I + mR2
DAY EIGHT ROTATIONAL MOTION 91

DAY PRACTICE SESSION 1

FOUNDATION QUESTIONS EXERCISE


1 When a body is projected at an angle with the horizontal 6 A particle of mass 5 g is moving with a uniform speed of
in the uniform gravitational field of the earth, the angular 3 2 cm s –1 is in the xy-plane along the line y = 2 5 cm.
momentum of the body about the point of projection, as it The magnitude of its angular momentum about the origin
proceeds along the path in g- cm 2 s – 1 is
(a) remains constant (a) zero
(b) increases continuously (b) 30
(c) decreases continuously (c) 30 2
(d) initially decreases and after reaching highest point (d) 30 10
increases
7 A bob of mass m attached to an inextensible string of
2 A force of −F k$ acts on O, the origin of the coordinate length l is suspended from a vertical support.The bob
system. The torque about the point (1, − 1) is rotates in a horizontal circle with an angular speed
z ω rad/s about the vertical support. About the point of
suspension ª JEE Main 2014
(a) angular momentum is conserved
y (b) angular momentum changes in magnitude but not in
O direction
(c) angular momentum changes in direction but not in
x magnitude
(a) F ($i − $j) (b) −F ($i + $j) (c) F ($i + $j) (d) −F ($i − $j) (d) angular momentum changes both in direction and
magnitude
3 Angular momentum of the particle rotating with a central
8 A particle of mass 2 kg is moving such that at time t, its
force is constant due to
position, in metre, is given by r (t ) = 5 $i − 2 t 2 $j. The
(a) constant force (b) constant linear momentum
angular momentum of the particle at t = 2 s about the
(c) zero torque (d) constant torque
origin in kg m −2 s −1 is ª JEE Main (Online) 2013
4 A particle of mass m = 3 kg moves along a straight line
(a) − 80 k$
4y − 3x = 2, where x and y are in metre, with constant
(b) (10 $i − 16 $j )
velocity v = 5 ms −1. The magnitude of angular
(c) − 40 k$
momentum about the origin is
(d) 40 k$
(a) 12kg - m2 s−1 (b) 8.0 kg - m2 s−1
(c) 6.0 kg - m2 s−1 (d) 4.5 kg - m2 s−1 9 A uniform disc of radius a and mass m, is rotating freely
with angular speed ω in a horizontal plane, about a
5 A particle of mass m moves along line PC with velocity v
smooth fixed vertical axis through its centre. A particle,
as shown in the figure. What is the angular momentum of
also of mass m, is suddenly attached to the rim of the disc
the particle about O?
and rotates with it. The new angular speed is
C
(a) ω / 6
(b) ω / 3
(c) ω / 2
L (d) ω /5
r 1
P
10 If the radius of the earth contracts of its present day
n
l value, length of the day will be approximately.
O 24 24
(a) h (b) h
(a) mvL (b) mv l (c) mvr (d) zero n n2
(c) 24 nh (d) 24 n 2 h
92 40 DAYS ~ JEE MAIN PHYSICS DAY EIGHT

11 A particle of mass m is moving along the side of a square Direction (Q. Nos. 16-18) Each of these questions contains
of side a , with a uniform speed v in the X-Y plane as two statements : Statement I and Statement II. Each of these
shown in the figure. questions also has four alternative choices, only one of which
is the correct answer. You have to select one of the codes (a),
a
Y D
v C (b), (c), (d) given here.
a v v a (a) Statement I is true, Statement II is true; Statement II is
A v B
the correct explanation for Statement I
a (b) Statement I is true, Statement II is true; Statement II is
R
45º
not the correct explanation for Statement I
O X (c) Statement I is true; Statement II is false
Which of the following statements is false for the angular (d) Statement I is false; Statement II is true
momentum L about the origin? ª JEE Main 2016 (Offline) 16 Statement I A disc A moves on a smooth horizontal plane
mv $ and rebounds elastically from a smooth vertical wall (Top
(a) L = − R k, when the particle is moving from A to B.
2 view is as shown in the figure) in this case about any
(b) L = mv  + a  k$ , when the particle is moving from B
R point on line xy. The angular momentum of the disc
 2  remains conserved.
to C.
A
(c) L = mv  − a  k$ , when the particle is moving from C
R
 2 
x y
to D.
mv $
(d) L = R k, when the particle is moving from D to A.
2
Statement II About any point in the plane, the torque
12 A uniform rod AB of mass m and length l at rest on a experienced by disc is zero as gravity force and normal
smooth horizontal surface. An impulse P is applied to the contact force balance each other.
end B. The time taken by the rod to turn through at right
angle is 17 Statement I A block is kept on a rough horizontal
ml πP π ml πP surface, under the action of a force F as shown in the
(a) 2 π (b) 2 (c) (d)
P ml 12 P ml figure. The torque of normal contact force about centre of
13 A solid cylinder of mass 20 kg rotates about its axis with mass is having zero value.
angular speed 100 rad/s. The radius of the cylinder is
0.25 m. Then the kinetic energy associated with the
F
rotation of the cylinder and the magnitude of angular C
momentum of the cylinder about its axis is respectively
(a) 3200 J, 62.5 J-s (b) 3125 J, 62.5 J-s Statement II The point of application of normal contact
(c) 3500 J, 68 J-s (d) 3400 J, 63.5 J-s force may pass through centre of mass.
14 A ring and a disc having the same mass, roll without 18 Statement I Two cylinders, one hollow (metal) and the
slipping with the same linear velocity. If the kinetic energy other solid (wood) with the same mass and identical
of the ring is 8 J, that of the disc must be dimensions are simultaneously allowed to roll without
(a) 2 J (b) 4 J (c) 6 J (d) 16 J slipping down an inclined plane from the same height.
15 A round uniform body of radius R, mass M and moment The solid cylinder will reach the bottom of the inclined
of inertia I, rolls down (without slipping) an inclined plane plane first.
making an angle θ with the horizontal. Then, its Statement II By the principle of conservation of energy, the
acceleration is total kinetic energies of both the cylinders are equal, when
g sin θ g sin θ g sin θ g sin θ they reach the bottom of the incline.
(a) (b) (c) (d)
1 + I /MR 2 1 + MR 2 /I 1 − I /MR 2 1 − MR 2 /I
DAY EIGHT ROTATIONAL MOTION 93

DAY PRACTICE SESSION 2

PROGRESSIVE QUESTIONS EXERCISE


1 A thin circular ring of mass m and radius R is rotating 5 A ring of mass M and radius R is rotating about its axis
about its axis with a constant angular velocity ω. Two with angular velocity ω. Two identical bodies each of
objects each of mass M are attached gently to the mass m are now gently attached at the two ends of a
opposite ends of a diameter of the ring. The ring now diameter of the ring. Because of this, the kinetic energy
rotates with an angular velocity ω′ loss will be ª JEE Main (Online) 2013
ω(m + 2M ) ω (m − 2M ) m (M + 2m) 2 2 Mm
(a) (b) (a) ωR (b) ω2R 2
m (m + 2M ) M (M + 2m)
ωm ωm Mm (M + m)M 2 2
(c) (d) (c) ω2R 2 (d) ωR
(m + M ) (m + 2M ) (M − 2m) (M + 2m)
2 A T shaped object with dimensions as shown in the 6 A hoop of radius r and mass m rotaing with an angular
figure, is lying on a smooth floor. A force is applied at the velocity ω 0 is placed on a rough horizontal surface. The
point P parallel to AB, such that the object has only the initial velocity of the centre of the hoop is zero. What will
translational motion without rotation. Find the location of be the velocity of the centre of the hoop, when it ceases
P with respect to C. to slip? ª JEE Main 2013
l rω0 rω0
(a) (b)
A B 4 3

(c) 0 (d) rω0
2
P
2l 7 An annular ring with inner and outer radii R1 and R 2 is
F
rolling without slipping with a uniform angular speed. The
ratio of the forces experienced by the two particles
F
situated on the inner and outer parts of the ring, 1 is
C F2
2 3 4 2
(a) l (b) l (c) l (d) l R2 R 
3 2 3 (a) (b)  1 
R1  R2 
3 A thin uniform rod of length l and mass m is swinging
freely about horizontal axis passing through its end. Its R1
(c) 1 (d)
maximum angular speed is ω. Its centre of mass rises to R2
a maximum height of 8 A slender uniform rod of mass M and length l is pivoted
1 l2ω2 1 lω 1 l2ω2 1 l2ω2 at one end so that it can rotate in a vertical plane (see the
(a) (b) (c) (d)
3 g 6 g 2 g 6 g figure). There is negligible friction at the pivot. The free
4 A small object of uniform density rolls up a curved end is held vertically above the pivot and then released.
surface with an initial velocity v. It reached up to a The angular acceleration of the rod when it makes an
3v 2 angle θ with the vertical, is
maximum height of with respect to the initial z
4g
position. The object is

q
v
x

ª JEE Main 2017 (Offline)


(a) ring (b) solid sphere 2g 3g 2g 3g
(c) hollow sphere (d) disc (a) sin θ (b) cos θ (c) cosθ (d) sinθ
3l 2l 3l 2l
94 40 DAYS ~ JEE MAIN PHYSICS DAY EIGHT

9 A horizontal force F acts on the sphere at its centre as


shown. Coefficient of friction between ground and sphere
is µ. What is the maximum value of F , for which there is no
slipping?
37º

F (a) 5.4 s (b) 2.4 s


O (c) 1.4 s (d) 1.2 s
12 Angular acceleration of the Cylinder (B) shown in the
figure is (all strings and pulley are ideal)
5 7
(a) F ≤ µmg (b) F ≤ µmg
2 2 A
9 3 m
(c) F ≤ µmg (d) F ≤ µmg
2 2
10 A solid uniform disc of mass m rolls without slipping down m
B R
a fixed inclined plank with an acceleration a. The frictional
force on the disc due to surface of the plank is
1 3 1 m
(a) ma (b) ma (c) ma (d) ma
4 2 2
2g 2g
(a) (b)
11 A cylinder having radius 0.4 m, initially rotating (at t = 0) 3R 5R
with ω 0 = 54 rad/s is placed on a rough inclined plane with 2g g
(c) (d)
θ = 37° having frictional coefficient µ = 0.5. The time taken R 2R
by the cylinder to start pure rolling is [g = 10m / s 2 ].

ANSWERS
SESSION 1 1 (b) 2 (c) 3 (c) 4 (c) 5 (b) 6 (d) 7 (c) 8 (a) 9 (b) 10 (b)
11 (c,d) 12 (c) 13 (b) 14 (c) 15 (a) 16 (a) 17 (a) 18 (b)

SESSION 2 1 (d) 2 (c) 3 (d) 4 (d) 5 (b) 6 (c) 7 (d) 8 (d) 9 (b) 10 (d)
11 (d) 12 (b)
DAY EIGHT ROTATIONAL MOTION 95

Hints and Explanations


SESSION 1 6 L = mvr1 9 Conserving angular momentum
1 τ 0 = mg × r⊥ y I1 ω1 = I2 ω2
 ma2 
As, r ⊥ is continuously increasing or  
m I1  2  ω
torque is continuously increasing on the v ∴ ω2 = ω1 = ω=
particle. Hence, angular momentum is r1 = y = 2Ö5 cm I2  ma2  3
 + ma
2

continuously increasing. x  2 
2 τ =r×F 10 I ω = constant
= (5)(3 2 )(2 5)
y R2
= 30 10 g- cm2s – 1 or = constant
T
 As, I ∝ R 2 and ω ∝ 1 
7 Angular momentum of the pendulum  
F  T
about the suspension point O is
∴ T ∝ R2
O x O O 1
As, R′= R
r n
z (1,–1) T 24
I ∴ T ′= 2 = 2 h
n n
τ = (i$ − $j ) × (− F k$ )
m vrad Lrad 11 For a particle of mass m is moving along
= F [(− i$ × k$ ) + ($j × k$ )]
the side of a square of side a. Such that
vtan v
= F [ $j + $i] Ltan Angular momentum L about the origin
=L=r ×p
= F [ $i + $j] O
= rpsin θ n$
3 Torque due to central force is zero, or L = r ( p ) n$
d
τ = (L ) = 0 When a particle is moving from D
dt L L to A,
∴ L = constant R
L= mv (− k$ )
4 Equation of straight line AB is 2
L = m (r ´ v)
A particle is moving from A to B,
R
B Then, v can be resolved into two L= m v (− k$ )
components, radial component r rad and 2
53º tangential component r tan . Due to v rad , and it moves from C to D,
0.5
L =  + a mv ( k$ )
L will be tangential and due to v tan , L R
r⊥
37º will be radially outwards as shown. So,  2 
A O net angular momentum will be as For B to C, we have
L =  + a mv ( k$ )
shown in figure, whose magnitude will R
4y − 3x = 2 be constant (|L |= mvl ). But its direction  2 
3 will change as shown in the figure.
or y = x + 0.5 Hence, options (c) and (d) are incorrect.
4
L = m (r × v)
∴ Slope = tanθ =
3 12 Angular impulse = P × l
4 where, r = radius of circle. 2
or θ = 37° 8 Given, m = 2 kg, r (t ) = 5i$ − 2t 2 $j = change in angular
r ⊥ = 0.5 sin 53° momentum
Angular momentum (L ) = r × p
= 0.4 m dr d
∴Velocity, v = = (5$i − 2t 2 $j ) P
∴Angular momentum (L ) = mv r ⊥ dt dt
or L = 3 × 5 × 0.4 = − 8 $j (at t = 2s )
= 6kg - m2 s−1 ∴ p = mv
5 Angular momentum of particle about O, = 2 × (−8 $j ) = − 16 $j v
Therefore, w
L = m( r × v)
L =r ×p
|L| = mr v sin θ
= (5i$ − 2t 2 $j ) × (−16) $j (at t = 2s )
= mv (r sin θ)
= mvl = − 80 k$
96 40 DAYS ~ JEE MAIN PHYSICS DAY EIGHT

Pl  ml2  If s be the distance covered along the


SESSION 2
∴ = Iω =  ω
2  12  plane, then
1 As, no external torque is acting on the
6P h = s sinθ
∴ ω= system, angular momentum of system
ml 2gs sin θ remains conserved.
θ π/2 ∴ v2 =
Now, t = = 1 + I /MR2
ω 6P /ml w w¢
π ml Now, v 2 = 2 as M
=
12 P 2gs sin θ g sin θ
∴ 2 as = ⇒a=
13 Moment of inertia of the solid cylinder 1 + I /MR2 1 + I /MR2 m, R
m, R M
about its axis of symmetry,
1 16 The forces experienced by disc are
I = MR2 gravity and normal contact force. In
2 i.e. I1 ω = I2 ω ′
1 addition to these, impact force (during
= × 20 × (0.25)2 collision) will act on the disc along line ⇒ mR 2ω = (mR 2 + 2MR 2 )ω ′
2
xy. Gravity and normal contact force  m 
= 10 × 0.0625 ⇒ ω′ =  ω
= 0.625kg-m2 = 62.5 J-s
balance each other (in terms of force  m + 2M 
and torque both), but impact force
Kinetic energy associated with the causes non-zero torque acting on disc 2 For pure translatory motion, net torque
rotation of the cylinder is given by about all points except the points lying about centre of mass should be zero.
1
K = Iω2 on its line of action i.e. xy. So, angular Thus, F is applied at centre of mass of
2 momentum remains conserved about system.
1 any point on xy.
= × 0.625 × (100)2 l
2 X
17 Here, as the block is kept in
= 0.3125 × 10000 = 3125J (0, 0)
equilibrium, the net torque experienced
KR by the body about any point has to be
14 For ring, =1 P (0, 2l/3)
KT zero. Here, due to F and Mg about C, the
KR 1 torque is zero but friction is providing Y
For disc, =
KT 2 non-zero torque in clockwise direction,
now other force is N only which can
For ring, total kinetic energy produce non-zero force in
= 2(K T ) = 8 J (given) (0, 2l)
anti-clockwise direction to make net C
Hence, KT = 4 J torque zero and it is possible only when
m1 × 0 + m2 × l
For disc, total kinetic energy =
3
KT N does not pass through C. OP =
2 m1 + m2
3 where, m1 and m2 are masses of
= × 4= 6 J
2 horizontal and vertical section of the
F
object. Assuming object is uniform,
15 Assuming that no energy is used up m2 = 2 m1
against friction, the loss in potential f Mg 2l
⇒ OP =
energy is equal to the total gain in the 3
N
kinetic energy. 
∴ PC =  l −
2l
+ l =  2l −  =
2l 4l
18 In case of pure rolling on inclined  3   3 3
C plane,
3 If centre of mass rises to a maximum
g sin θ
s a= height, then loss in KE = Gain in PE, we get
1 + I /mR 2

h
w ISolid < I Hollow
C ∴ a Solid > a Hollow
q v ∴ Solid cylinder will reach the bottom h
first. Further, in case pure rolling on
1 v  1
2
stationary ground, work done by
i.e. Mgh = I  + Mv
2
2  R2  2 friction is zero. Therefore, mechanical
energy of both the cylinders will remain
1 2
v  M + 2  = Mgh
I constant. 1 2
or Iω = mgh
2  R 
∴ (KE) Hollow = (KE) Solid 2
2Mgh 2gh 1 1 1 l2ω2
or v 2 = = = decrease in PE = mgh × ml2ω2 = mgh ⇒ h =
M + I/R 2
1 + I/MR 2 2 3 6 g
DAY EIGHT ROTATIONAL MOTION 97

2
 3 v2  1
4 1 mv 2 + 1 I  v  = mg  
8 As the rod rotates in vertical plane so a [For uniform solid disc, I =
2
mR2 ]
2 2  R  4g  torque is acting on it, which is due to
the vertical component of weight of 2
1 = g sinθ
∴ I = mR2 rod. 3
2 3
or g sinθ = a
Therefore, the body is a disc. 2
mg sinθ
5 By conservation of angular momentum, l/2 Now, force of friction ( f ) =
mR2
I1ω1 = I2ω2 …(i) 1+

q
s
I

co
l/2
where, I1 = mR2 …(ii) m (3a / 2) 1

g
or f = = ma

m
mg sin q
and I2 = 2mR2 + MR2 …(iii) q q 1+ 2 2
mg
Now change in, Hence, (d) is the correct option.
1 1 At any time t
KE = L1 I12 − L2 I22 …(iv) Initial condition
11 a = (µg cos θ) + (g sin θ)
2 2
Now, torque τ = force × perpendicular = 0.5 × 10 × 0.8 + 10 × 0.6
distance of line of action of force from or a= 10 m / s 2
axis of rotation a
ω l
= mg sin θ ×
2 a
Again, torque τ = Iα 7º º
3 37 37º
Substituting the values from Eqs. (i), (ii) where, I = moment of inertia sin cos mg cos 37º
and (iii) into Eq. (iv), we get, g
m mg mg
ml2 µ
 Mm  2 2 =
Change in KE =  ω R 3 37º
 M + 2m  [Force and torque frequency
along axis of rotation passing through in And angular acceleration (α )
6 R end] (µ mg cos θ)R
=
α = angular acceleration 1
mR2
l ml2 2
v ∴ mg sin θ × = α 2µ g cos θ
2 3 or α=
3g sin θ R
∴ α= 2 × 0.5× 10 × 0.8
From conservation of angular
2l or α= = 20 rad/s2
0.4
momentum 9 Here, F − f = ma … (i)
Pure rolling will start, when
v
mr 2ω 0 = mvr + mr 2 × v = Rω
r
where, v = a × t and ω = (ω 0 − α t )
ω 0r F
⇒ v = ∴ at = R (ω 0 − α t )
2 O
or 10t = 0.4(54 − 20t )
7 Since ω is constant, v would also be f or t = 12
. s
constant. So, no net force or torque is
τ = I α = I  
a Hence, (d) is the correct option.
acting on ring. The force experienced by and
 R
any particle is only along radial 12 Linear acceleration of cylinder (B) is
2 a
direction, or we can say the centripetal ⇒ f . R = mR2 × given by the relation
force. 5 R mR2
2 a= ⋅g
or f = ma …(ii) I + m ′ R2
5
1
F2 From Eqs. (i) and (ii), we get where, I = mR2 (for solid cylinder) and
R1 2 2
R2 F1 f= F m ′ = 2m
7
v m R2
w or
2
F ≤ µmg ∴ a= ⋅g
1
7 m R2 + 2m R2
7 2
or F ≤ µmg
2 2 m R2 2g
= ⋅g =
The force exprienced by inner part, Hence, (b) is the correct option. 5m R 2 5
F1 = mω2 R1 and the force experienced g sin θ ∴ Angular acceleration (α ) of the cylinder
10 Since, a =
by outer part, 1 + I / mR2 B is
a 2g
F2 = mω2 R2 g sin θ α= or α =
or a= ; R 5R
F1 R 1
= 1 1+ Hence, (b) is the correct option.
F2 R2 2
DAY NINE

Gravitation
Learning & Revision for the Day
u Universal Law of Gravitation u Gravitational Potential u Kepler’s Laws of Planetary Motion
u Acceleration due to Gravity u Gravitational Potential Energy u Satellite
u Gravitational Field u Escape Velocity u Geostationary Satellite

Universal Law of Gravitation


In this universe, each body attracts other body with a force that is directly proportional
to the product of their masses and inversely proportional to the square of the distance
between them.
Let m1 and m2 be the masses of two bodies and r be the separation between them.
m m
F = G 12 2 .
r
The proportionality constant G is called universal gravitational constant. In SI system,
value of gravitational constant G is 6.67 × 10 −11 Nm2 kg−2 . Dimensional formula of G is
[M −1L3 T −2 ].

Acceleration due to Gravity


m
The acceleration of an object during its free fall towards the earth is
called acceleration due to gravity.
If M is the mass of earth and R is the radius, the earth attracts a mass R
m on its surface with a force F given by
GMm Centre PREP
F = MIRROR
R2
Your Personal Preparation Indicator
This force imparts an acceleration to the mass m which is known as
acceleration due to gravity (g). u No. of Questions in Exercises (x)—
By Newton’s law, we have u No. of Questions Attempted (y)—
GMm u No. of Correct Questions (z)—
F 2 GM (Without referring Explanations)
Acceleration (g) = = R = 2
m m R
u Accuracy Level (z / y × 100)—
GM
On the surface of earth, g = 2 u Prep Level (z / x × 100)—
R
Substituting the values of G, M, R, we get g = 9.81 ms −2 . In order to expect good rank in JEE,
24
your Accuracy Level should be above
Mass of the earth m = 6 × 10 kg and radius of the earth R = 6.4 × 10 m.
6
85 & Prep Level should be above 75.
DAY NINE GRAVITATION 99

Gravitational field intensity due to a solid sphere


Variation in g with Altitude and Depth
l

(e.g. earth) of mass M and radius R at a point distant r from


with respect to Earth GM
its centre (r > R) is I = 2
r
The value of g is variable and can vary in same cases as
GM
mentioned below and at the surface of solid sphere, I = 2 .
1. Value of acceleration due to gravity ( g ) at a height ( h ) R
GMr
gR2 However, for a point r < R, we find that I = 3
from the surface of the earth is given by g′ = R
(R + h)2
 2h 
I =GM
If h << R, then g′ = g 1 − 
 R R2
I
2. Value of acceleration due to gravity (g) at a depth (d) I∝r I ∝ 12
r
from the surface of the earth is given by
 d r<R r>R
g′ = g  1 −  O
 R R r

At the centre of the earth d = R and hence, g′ = 0. Variation of gravitational field intensity in solid sphere

Due to a body in the form of uniform shell gravitational


Variation in the Value of g Due to
l

field intensity at a point outside the shell (r > R) is given


Rotation of the Earth by
GM
I= 2
r
Due to rotation of the earth, the value of g decreases as the
speed of rotation of the earth increases. The value of But at any point inside the shell, gravitational intensity is
acceleration due to gravity at a latitude is zero.
g′λ = g − Rω2 cos2 λ l
Gravitational intensity at a point due to the combined
effect of different point masses is given by the vector sum
Following conclusions can be drawn from the above
of individual intensities.
discussion
Thus, I = I 1 + I2 + I3 + ...
(a) The effect of centrifugal force due to rotation of the earth
is to reduce the effective value of g.
(b) The effective value of g is not truely in vertical direction. Gravitational Potential
(c) At the equators, λ = 0 ° Gravitational potential at any point in a gravitational field is
defined as the work done in bringing a unit mass from
Therefore, g′ = g − Rω 2
(minimum value)
infinity to that point.
(d) At the poles, λ = 90 ° W
Gravitational potential, V = lim .
Therefore, g′ = g (maximum value) m 0→ 0 m0

GM
Gravitational Field Gravitational potential due to a point mass is V = − .
r
The space surrounding a material body in which its Gravitational potential is always negative. It is a scalar term
gravitational force of attraction can be experienced is called and its SI unit is J kg −1.
its gravitational field.
For Solid Sphere
Gravitational Field Intensity (I) l
At a point outside the solid sphere, (e.g. earth), i.e.
Gravitational field intensity at any point is defined as the GM
r > R, V = − .
force experienced by any test mass devided by the magnitude r
of test mass when placed at the desired point. GM
Mathematically,
l
At a point on the surface, V = − .
R
F
Gravitational field intensity, I = l
At a point inside the sphere, (r < R).
m0
GM GM  r2 
where, m0 is a small test mass. The SI unit of gravitational V =− (3 R2 − r 2 ) = − 3 − 2 
intensity is N kg −1. 2 R3 2R  R 
l
Gravitational intensity at a point situated at a distance r l
At the centre of solid sphere,
GM 3GM 3
from a point mass M is given by I = 2 V =− = Vsurface
r 2R 2
100 40 DAYS ~ JEE MAIN PHYSICS DAY NINE

For Spherical Shell B


l
At a point outside the shell, A
C
GM
V =− where, r > R. II
r
l
At a point on the surface of spherical shell, D
GM
V =−
R
3. Law of Periods The square of the planet’s time period of
l
At any point inside the surface of spherical shell
revolution is directly proportional to the cube of
GM semi-major axis of its orbit.
V =− = Vsurface
R
T 2 ∝ a3
where a is the semi-major axis.
Relation between Gravitational Field
and Gravitational Potential Gravitational Potential Energy
If r1 and r2 are position of two points in the gravitation field Gravitational potential energy of a body or system is negative
with intensity (I), then change in gravitational potential of work done by the conservative gravitational forces F in
r2 bringing it from infinity to the present position.
V (r2 ) − V (r1) = − ∫ I ⋅ dr Mathematically, gravitational potential energy
r1

U = − W = − ∫ F ⋅ dr
r
⇒ dV = − I ⋅ dr ∞
dr = dx $i + dy $j + dz k$ l
The gravitational potential energy of two particles of
and I = I x $i + I y $j + I z k,
$ then masses m1 and m2 separated by a distance r is given by
Gm1m2
dV = − I.dr = −I x dx − I y dy − I z dz U=−
r
∂V $ ∂V $ ∂V $
Thus, I=− i− j− k The gravitational potential energy of mass m at the surface
∂x ∂y ∂z
l

of the earth is
Remember that partial differentiation indicates that variation GMm
of gravitational potential in counter along the variation of U=−
R
x-coordinate, then other coordinates (i.e. y and z) are
assumed to be constant.
l
Difference in potential energy of mass m at a height h from
the earth’s surface and at the earth’s surface is
mgh ~
U( R + h ) − UR = − mgh, if h << R
Kepler’s Laws of Planetary 1+
h
R
Motion l
For three particles system,
Kepler discovered three empirical laws which accurately
describe the motion of planets. Gm1m2 Gm1m3 Gm2 m3 
U = − + + 
These laws are  r 12 r 13 r23 
1. Law of Orbits All the planets move around the sun in an n (n − 1)
elliptical orbit with sun at one of the focus of ellipse.
l
For n-particles system, pairs form and total
2
Minor axis potential energy of the system is sum of potential energies
of all such pairs.

Planet Swept area


Escape Velocity
Major axis
It is the minimum velocity with which a body must be
S projected from the surface of the earth so that it escapes the
Perihelion Apehelion
or Perigee or Apogee gravitational field of the earth. We can also say that a body,
a projected with escape velocity, will be able to go to a point
2. Law of Areas The line joining the sun to the planet which is at infinite distance from the earth.
sweeps out equal areas in equal intervals of time, i.e. areal The value of escape velocity from the surface of a planet of
velocity of the planet w.r.t. sun is constant. This is called mass M, radius R and acceleration due to gravity g is
the law of area, which indicates that a planet moves faster 2GM
near the sun and slowly when away from the sun. vescape = = 2gR
R
DAY NINE GRAVITATION 101

Escape velocity does not depend upon the mass or shape or Height of Satellite in Terms of Period
size of the body as well as the direction of projection of the
The height of the satellite (from the earth planet) can be
body. For earth value of escape velocity is 11.2 kms−1.
determined by its time period and vice-versa.
Satellites As the height of the satellite in terms of time period,
1/ 3
Anybody that revolves around earth or any planet is called  gR2T 2 
h=r −R= 2 
− R.
satellite. These can be natural (e.g. Moon) or artificial. The  4π 
artificial satellites are man made satellites launched from the
earth. The path of these satellites are elliptical with the centre of Energy of Satellite
earth at a foci of the ellipse. However, as a first approximation
1 GMm
we may consider the orbit of satellite as circular. Kinetic energy of satellite, K = mv20 = .
2 2r
Orbital Velocity of Satellite Potential energy of satellite, U = −
GMm
Orbital velocity of a satellite is the velocity required to put r
the satellite into its orbit around the earth. The orbital GMm
and total energy of satellite E = K + U = − = − K.
velocity of satellite is given by 2r

GM gR2 gR2 Binding Energy of Satellite


vo = = =
r r (R + h) It is the energy required to remove the satellite from its orbit
If h << R or r ~
− R, then and take it to infinity.
GMm
GM Binding energy = − E = +
vo = = gR = 7.9 kms – 1 2r
R

vescape = 2 vorbital
Angular Momentum of Satellite
Angular momentum of a satellite, L = mv0r = m2GMr
Period of Revolution
It is the time taken by a satellite to complete one revolution
around the earth. Geostationary Satellite
2 πr r3 r3 3π If an artificial satellite revolves around the earth in an
Revolution period, T = = 2π = 2π = equatorial plane with a time period of 24 h in the same sense
vo GM gR2 G. e
as that of the earth, then it will appear stationary to the
R observer on the earth. Such a satellite is known as a
− R, then T = 2π
If r ~ = 84.6 min.
g geostationary satellite or parking satellite.

DAY PRACTICE SESSION 1

FOUNDATION QUESTIONS EXERCISE


1 A mass M is split into two parts m and (M − m ), which are (a) M will remain at rest
separated by a certain distance. The ratio m/M which (b) M will move towards M
maximizes the gravitational force between the parts is (c) M will move towards 2M
(d) M will have oscillatory motion
(a) 1 : 4 (b) 1 : 3
(c) 1 : 2 (d) 1 : 1 3 If one moves from the surface of the earth to the moon,
2 Particles of masses 2M, m and M are respectively at what will be the effect on its weight?
1 (a) Weight of a person decreases continuously with height
points A, B and C with AB = (BC ), m is much-much
2 from the surface of the earth
smaller than M and at time t = 0, they are all at rest as (b) Weight of a person increases with height from the
given in figure. At subsequent times before any collision surface of the earth
takes place. (c) Weight of a person first decreases with height and then
A B C increases with height from the surface of the earth
2M m M (d) Weight of a person first increases with height and then
decreases with height from the surface of the earth
102 40 DAYS ~ JEE MAIN PHYSICS DAY NINE

g g
4 At the surface of a certain planet acceleration due to
gravity is one-quarter of that on the earth. If a brass ball
(c) (d)
is transported on this planet, then which one of the
following statements is not correct?
d d
(a) The brass ball has same mass on the other planet as on O R O R
the earth
(b) The mass of the brass ball on this planet is a quarter of 11 The gravitational field, due to the ‘left over part’ of a
its mass as measured on the earth uniform sphere (from which a part as shown has been
(c) The weight of the brass ball on this planet is a quarter of ‘removed out’) at a very far off point, P located as shown,
the weight as measured on the earth would be (nearly) ª JEE Main 2015
(d) The brass ball has the same volume on the other planet
Mass of complete
as on the earth Removed
part sphere = M
5 If both the mass and radius of the earth, each decreases P
R
by 50%, the acceleration due to gravity would R
(a) remain same (b) decrease by 50%
x
(c) decrease by 100% (d) increase by 100%
5 GM 8 GM 7 GM 6 GM
6 A research satellite of mass 200 kg circles the earth in an (a) (b) (c) (d)
6 R2 9 R2 8 R2 7 R2
3R
orbit of average radius ,where R is the radius of the
2 12 A solid sphere is of density ρ and radius R. The
earth. Assuming the gravitational pull on a mass of 1kg gravitational field at a distance r from the centre of the
on the earth’s surface to be 10 N, the pull on the satellite sphere, when r < R , is
will be ρπGR 3 4 π Gρ r 2 4 π Gρ R 3 4 π Gρ r
(a) (b) (c) (d)
(a) 880 N (b) 889 N r 3 3r 2 3
(c) 885 N (d) 892 N 13 The maximum vertical distance through which a full
7 The height at which the acceleration due to gravity dressed astronaut can jump on the earth is 0.5 m.
g Estimate the maximum vertical distance through which he
becomes (where, g = the acceleration due to gravity
9 can jump on the moon, which has a mean density 2/3rd
on the surface of the earth) in terms of R, the radius of that of the earth and radius one quarter that of the earth.
the earth is (a) 1.5 m (b) 3 m (c) 6 m (d) 7.5 m
h h 14 The mass of a spaceship is 1000 kg. It is to be launched
(a) 2h (b) (c) (d) 2 h
3 2 from the earth’s surface out into free space. The value of
8 The change in the value of g at a height h above the g and R (radius of earth) are 10 m/s 2 and 6400 km
surface of the earth is the same as at a depth d below respectively. The required energy for this work will be
the surface of the earth. When both d and h are much (a) 6.4 × 1011 J (b) 6.4 × 108 J
smaller than the radius of the earth, then which one of the (c) 6.4 × 109 J (d) 6.4 × 1010 J
following is correct? 15 If g is the acceleration due to gravity on the earth’s
h 3h surface, the gain in the potential energy of an object of
(a) d = (b) d = (c) d = 2h (d) d = h
2 2 mass m raised from the surface of the earth to a height
9 Average density of the earth equal to the radius R of the earth, is
1 1
(a) does not depend on g (a) 2mgR (b) mgR (c) mgR (d) mgR
(b) is a complex function of g 2 4
(c) is directly proportional to g 16 Two bodies of masses m and 4 m are placed at a
(d) is inversely proportional to g distance r. The gravitational potential at a point on the line
joining them where the gravitational field is zero is
10 The variation of acceleration due to gravity g with 4Gm 6Gm 9Gm
distance d from centre of the Earth is best represented (a) − (b) − (c) − (d) zero
r r r
by (R = Earth’s radius) ª JEE Main 2017 (Offline)
g g 17 A planet in a distant solar system is 10 times more
massive than the earth and its radius is 10 times smaller.
Given that the escape velocity from the earth is 11 kms −1,
(a) (b) the escape velocity from the surface of the planet would
d d be
O R O R
(a) 1.1 kms −1 (b) 11 kms −1
(c) 110 kms −1 (d) 0.11 kms −1
DAY NINE GRAVITATION 103

18 A projectile is fired vertically upwards from the surface 26 The gravitational force exerted by the sun on the moon is
of the earth with a velocity kve , where ve is the escape about twice as great as the gravitational force exerted by
velocity and k < 1. If R is the radius of the earth, the the earth on the moon, but still the moon is not escaping
maximum height to which it will rise measured from the from gravitational influence of the earth. Mark the option
centre of the earth will be which correctly explains the above system.
1− k2 R (a) At some point of time the moon will escape from the earth
(a) (b)
R 1− k2 (b) Separation between the moon and sun is larger than the
R separation between the earth and moon
(c) R (1 − k 2 ) (d)
1+ k2 (c) The moon-earth system is bounded one and a minimum
amount of energy is required to escape the moon from the
19 Two cars of masses m1 and m2 are moving in circles of earth
radii r1 and r2, respectively. Their speeds are such that (d) None of the above
they make complete circles in the same time t. The
ratio of their centripetal acceleration is 27 What is the minimum energy required to launch a satellite of
mass m from the surface of a planet of mass M and radius R
(a) m1r1 : m2 r2 (b) m1 : m2 (c) r1 : r2 (d) 1 : 1
in a circular orbit at an altitude of 2R ?
20 What is the direction of areal velocity of the earth ª JEE Main 2013
around the sun? 5 GmM 2GmM GmM GmM
(a) (b) (c) (d)
(a) Perpendicular to positon of the earth w.r.t. the sun at 6R 3R 2R 3R
the focus Energy
(b) Perpendicular to velocity of the earth revolving in the
28 The curves for potential energy
elliptical path (E P ) and kinetic energy (E K ) of
EK
(c) Parallel to angular displacement two particles system are as
A B
(d) Both (a) and (b) shown in the figure. At what O C
r
points the system will be bound D
21 The time period of a satellite of the earth is 5 h. If EP
the separation between the earth and the satellite is (a) Only at point A
(b) Only at point D
increased to 4 times the previous value, the new time
(c) At points A and D
period will become
(d) At points A, B and C
(a) 10 h (b) 80 h (c) 40 h (d) 20 h
29 The earth moves around the sun in B Earth
22 A satellite goes along an elliptical path around earth.
an elliptical orbit as shown in the
The rate of change of area swept by the line joining OA
earth and the satellite is proportional to figure. The ratio = x . The ratio A
OB O Sun
1/ 2 3/ 2 2
(a) r (b) r (c) r (d) r of the speeds of the earth at B and
23 A satellite of mass m revolves around the earth of at A is
radius R at a height x from its surface. If g is the (a) x (b) x (c) x 2 (d) x x
acceleration due to gravity on the surface of the earth, 30 A satellite of mass ms revolving in a circular orbit of radius rs
the orbital speed of the satellite is around the earth of mass M, has a total energy E . Then its
1/ 2
gR gR 2  gR 2  angular momentum will be
(a) g x (b) (c) (d)  
R−x R+ x R + x  (a) (2E msrs )1/ 2 (b) (2E msrs )

24 Two particles of equal mass m go around a circle of (c) (2E msrs2 )1/ 2 (d) (2E msrs2 )
radius R under the action of their mutual gravitational 31 Match the term related to gravitation given in Column I with
attraction. The speed of each particle with respect to their formula given in Column II and select the correct
their centre of mass is ª AIEEE 2011 option from the choices given below :
Gm Gm Gm Gm
(a) (b) (c) (d) Column I Column II
R 4R 3R 2R
A. Gravitational potential at a point 1. 3 GM
25 Suppose the gravitational force varies inversely as the outside the solid sphere 2R
nth power of distance. Then the time period of a planet
B. Gravitational potential at a point on 2. GM
in circular orbit of radius R around the sun will be
the surface of sphere r
proportional to
 n + 1  n − 1  n − 2 C. Gravitational potential at the centre 3. GM
     
(a) R  2 
(b) R  2 
(c) R n (d) R  2  of solid sphere R
104 40 DAYS ~ JEE MAIN PHYSICS DAY NINE

A B C Statement II An object moving around the earth under


(a) 1 3 2 the influence of the earth’s gravitational force is in a state
(b) 2 3 1 of ‘free-fall’.
(c) 1 2 3
33 Statement I Kepler’s laws for planetary motion are
(d) 3 2 1
consequence of Newton’s laws.
Direction (Q. Nos. 32-33) Each of these questions contains Statement II Kepler’s laws can be derived by using
two statements : Statement I and Statement II. Each of these Newton’s laws.
questions also has four alternative choices, only one of which 34 Assertion (A) If the earth were a hollow sphere,
is the correct answer. You have to select one of the codes (a), gravitational field intensity at any point inside the earth
(b), (c), (d) given below would be zero.
(a) Statement I is true, Statement II is true; Statement II is Reason (R) Net force on a body inside the sphere is
the correct explanation for Statement I
zero.
(b) Statement I is true, Statement II is true; Statement II is
(a) If both Assertion and Reason are correct and Reason is
not the correct explanation for Statement I
the correct explanation of Assertion
(c) Statement I is true; Statement II is false
(b) If both Assertion and Reason are true but Reason is not
(d)Statement I is false; Statement II is true
the correct explanation of Assertion
32 Statement I An astronaut in an orbiting space station (c) If Assertion is true but Reason is false
above the earth experiences weightlessness. (d) If Assertion is false but Reason is true

DAY PRACTICE SESSION 2

PROGRESSIVE QUESTIONS EXERCISE


1 Satellites orbitting the earth have finite life and sometimes   1 1   1 1  
(a) Gm  A  −  + BL  (b) Gm  A  −  − BL 
debris of satellites fall to the earth. This is because   a + L a     a a + L  
(a) the solar cells and batteries in satellites run out   1   1 
1 1 
(b) the laws of gravitation predict a trajectory spiralling (c) Gm  A  −  − BL  (d) Gm  A  −  + BL 
inwards   (a + L) a    a a + L 
(c) of viscous forces causing the speed of satellite and 5 A ring having non-uniform distribution of mass having
hence height to gradually decrease
mass M and radius R is being considered. A point mass
(d) of collisions with other satellites
m0 is taken slowly from A to B along the axis of the ring. In
2 A body is released from a point distance r from the doing so, work done by the external force against the
centre of earth. If R is the radius of the earth and r > R , gravitational force exerted by ring is
then the velocity of the body at the time of striking the
M
earth will be
(a) gR (b) 2gR
R
A B
2gRr 2gR (r − R) O
(c) (d) R R
r −R r

3 Two small satellites move in circular orbits around the


earth, at distances r and r + ∆r from the centre of the (a)
GMm0
earth. Their time periods of rotation areT and T + ∆T , 2R
( ∆r << r , ∆T << T ). Then, ∆T is equal to (b)
GMm0  1

1 
3 ∆r 2 ∆r − 3 ∆r ∆r R  2 5 
(a) T (b) T (c) T (d)T
2 r 3 r 2 r r GMm0  1 1 
(c) −
4 A straight rod of length L extends from x = a to x = L + a . R  5 2 
The gravitational force, it exerts on a point mass m at (d) It is not possible to find the required work as the nature
x = 0, if the mass per unit length is A + Bx 2, is of distribution of mass is not known
DAY NINE GRAVITATION 105

6 With what minimum speed should vmin 12 From a solid sphere of mass M and
m be projected from point C in the m 30° radius R, a spherical portion of radius
presence of two fixed masses M R 
  is removed as shown in the figure.
each at A and B as shown in the  2
figure, such that mass m should Taking gravitational potentialV = 0 at
escape the gravitational attraction I = ∞, the potential at the centre of the
M M cavity thus formed is (I = gravitational
of A and B ?
A R O R B constant). ª JEE Main 2015
2GM 2 2 GM
(a) (b) −GM −GM −2GM −2GM
R R (a) (b) (c) (d)
2R R 3R R
GM GM
(c) 2 (d) 2 2 13 A satellite is revolving in a circular orbit at a height h from
R R
the Earth’s surface (radius of Earth R , h < < R ). The
7 A particle of mass 10 g is kept on the surface of a minimum increase in its orbital velocity required, so that
uniform sphere of mass 100 kg and radius 10 cm. Find the satellite could escape from the Earth’s gravitational
the work to be done against the gravitational force field, is close to (Neglect the effect of atmosphere)
between them, to take the particle far away from the ª JEE Main 2017 (Offline)
sphere, (you may take G = 6.67 × 10−11 Nm 2 / kg2) (a) 2gR (b) gR (c) gR / 2 (d) gR ( 2 − 1)
−10 −10
(a) 13.34 × 10 J (b) 3.33 × 10 J 14 Four particles, each of mass M and equidistant from each
(c) 6.67 × 10−9 J (d) 6.67 × 10−10 J other, move along a circle of radius R under the action of
8 Two metallic spheres each of mass M are suspended by their mutual gravitational attraction, the speed of each
two strings each of length L. The distance between the particle is ª JEE Main 2015
upper ends of strings is L. The angle which the strings GM GM
(a) (b) 2 2
will make with the vertical due to mutual attraction of the R R
spheres is GM 1 GM
(c) (1 + 2 2 ) (d) (1 + 2 2 )
 GM  R 2 R
 GM 
(a) tan−1   (b) tan−1  
 gL   2 gL  Direction (Q. Nos. 15-16) Each of these questions contains
two statements : Statement I and Statement II. Each of these
 GM   2GM 
(c) tan−1  2  (d) tan−1  2 
questions also has four alternative choices, only one of which
 gL   gL  is the correct answer. You have to select one of the codes (a),
(b), (c), (d) given below
9 A body of mass 2 kg is moving under the influence of a
(a) Statement I is true, Statement II is true; Statement II is
central force whose potential energy is given by the correct explanation for Statement I
U = 2r 3 joule. If the body is moving in a circular orbit of (b) Statement I is true, Statement II is true; Statement II is
5 m, its energy will be not the correct explanation for Statement I
(a) 250 J (b) 125 J (c) Statement I is true; Statement II is false
(c) 625 J (d) 650 J (d) Statement I is false; Statement II is true
2
10 A mass m is placed at P at a distance 15 Statement I Three orbits are marked 3
r as 1, 2 and 3. These three orbits 1
h along the normal through the centre P
O of a thin circular ring of mass M and O h m
have same semi-major axis although
their shapes (eccentricities) are
radius r as shown in figure. If the mass
different. The three identical
is moved further away such that OP M
satellites are orbiting in these three orbits, respectively.
becomes 2h, by what factor, the force These three satellites have the same binding energy.
of gravitation will decrease, if h = r ?
Statement II Total energy of a satellite depends on the
3 2 5 2 4 3 4 2 semi-major axis of orbit according to the expression,
(a) (b) (c) (d)
4 3 3 5 5 5 − GMm
E=
11 An artificial satellite of the earth is launched in circular orbit 2r
in equatorial plane of the earth and satellite is moving from 16 Statement I Two satellites are following one another in the
West to East. With respect to a person on the equator, the same circular orbit. If one satellite tries to catch another
satellite is completing one round trip in 24 h. Mass of the (leading one) satellite, then it can be done by increasing
its speed without changing the orbit.
earth is, M = 6 × 10 24kg. For this situation, orbital radius of the
satellite is Statement II The energy of the earth satellites system in
− GMm
(a) 2.66 × 104 km (b) 6400 km circular orbits is given by E = , where, r is the
2r
(c) 36,000 km (d) 29,600 km radius of the circular orbit.
106 40 DAYS ~ JEE MAIN PHYSICS DAY NINE

ANSWERS

SESSION 1 1 (c) 2 (c) 3 (c) 4 (b) 5 (d) 6 (b) 7 (c) 8 (c) 9 (c) 10 (c)
11 (c) 12 (d) 13 (b) 14 (d) 15 (b) 16 (c) 17 (c) 18 (b) 19 (c) 20 (d)
21 (c) 22 (a) 23 (d) 24 (b) 25 (a) 26 (c) 27 (a) 28 (d) 29 (b) 30 (c)
31 (b) 32 (a) 33 (d) 34 (a)

SESSION 2 1 (c) 2 (d) 3 (a) 4 (d) 5 (b) 6 (b) 7 (d) 8 (c) 9 (c) 10 (d)
11 (a) 12 (b) 13 (d) 14 (d) 15 (a) 16 (d)

Hints and Explanations


SESSION 1 6 Here, mg = 10 or 1 × g = 10 ⇒ ρ = 
3 
 g;
Gm(M − m )  4 πGR 
1 As, F = ⇒ g = 10 ms – 2
x2 R2 R2 40 ∴ ρ∝ g
For maximum gravitational force Now, g ′ = g 2 = 10 × = (where, ρ = average density of the earth)
r (3R/2) 2 9
dF G m 1
= 2 (M − 2m ) = 0 or = Pull on satellite = m ′ g ′ 10 Inside the earth’s surface,
dm x M 2
40 GM
= 200 × g = d i.e. g ∝ d
2 The particle B will move towards the 9 R3
greater force between forces by A ≈ 889 N Outside the earth’s surface,
and B. Gm 1
G (2Mm ) 7 Acceleration due to gravity at height h g = 2 i.e. g ∝ 2
Force on B due to A = F BA = GM d d
( AB )2 is, g ′ = So, till earth’s surface g increases linearly
(R + h )2
towards BA with distance r, shown only in graph (c).
2

Force on B due to C = F BC =
GMm g GM R2  R 
⇒ = 2 ⋅ =g   11 We have,
(BC )2 9 R (R + h )2  R + h 3
towards BC M 4  R M
2 M′ = × π  =
As, (BC ) = 2 AB 1  R  4 3 3  2 8
⇒ =  πR
GMm GMm 9  R + h 3
⇒ F BC = = < F BA
(2 AB )2 4( AB )2 R 1 ∴ Gravitational field at
⇒ = GM GM
Hence, m will move towards BA R+ h 3 P = 2 −
(i.e. 2M). R 8R2
⇒ 3R = R + h
= G × 2  1 −  =
M  1 7 GM
3 The gravitational attraction on a body h
⇒ R= R  8 8 R2
due to the earth decreases with height 2
and increases due to the moon at a 12 Intensity,
certain height. 8 g h = g  1 − 2h  …(i) GM Gr  4 π R3ρ = 4 π Gρr
 R I = r = 3  
At one point it becomes zero and with R3 R 3  3
and g d = g  1 − 
d
further increase in height the …(ii)
 R
gravitational attraction of the moon 13 On the moon, g m = 4 π G (R/4)(2ρ/3)
As per statement of the problem, 3
becomes more than that of the earth.
g h = gd 14  1
=  π GRρ = g
4 The mass of a body is always constant 63  6
i.e. g  1 −
and does not change with position. 2h   d
 = g 1 −  Work done in jumping
 R  R
5 Here, g = GM /R 2
= m × g × 0.5 = m × (g /6) h 1
⇒ 2h = d h 1 = 0.5 × 6 = 3.0 m
G (M /2) 2GM
and g ′ = = = 2g GM
(R/2)2 R2 9 As, g = 2 ; 14 Potential energy on the earth surface is
R − mgR while in free space it is zero. So,
 g′ − g 
∴ % increase in g =   × 100 to free the spaceship, minimum required
M =  π R 3  ρ
4
 g  energy is
3 
 2g − g  K = mgR = 103 × 10 × 6400 × 103 J
=  × 100 4G πR 3
 g  ∴ g = ρ = 6.4 × 1010 J
3 R2
= 100%
DAY NINE GRAVITATION 107

15 Gravitational potential energy of body 1 mg (r − R )  gR2   gR2 


1 /2
m k 2 ⋅ 2gR = , vo =
on the earth’s surface is 2 1+
r −R or  ( R + x ) =  ( R + x )
   
GMe m R
U =−
r − R
R k 2 R 1 + =r −R 24 As gravitational force provides necessary
At a height h from earth’s surface, its  R  centripetal force.
value is k2r = r − R
GMe m GMe m R
Uh = − =− [as h = R] ⇒ r =
(R + h ) 2R 1 − k2
R R
where, Me = mass of earth, m m mv2
19 As their period of revolution is same, so F R
m = mass of body their angular speed is also same.
and R = radius of earth. Centripetal acceleration is circular path,
∴ Gain in potential energy a = ω2 r .
= Uh − U ω2 r Gm2 mv 2
a
Thus, 1 = 2 1 = 1
r i.e. F = =
GMe m  GMe m  ω r2 (2R )2 R
=− − −  a2 r2
2R  R  Gm
20 Areal velocity of the earth around the ⇒ v =
GMe m GMe m 4R
=− + sun is given by
2R R
dA L 25 The necessary centripetal force required
GMe m gR2 m =
= = as g = GMe  dt 2m for a planet to move around the sun
2R 2R  R2  where, L is the angular momentum and = gravitational force exerted on it
1 /2
1 m is the mass of the earth. But angular
or v =  n −1e 
= mgR mv 2 GMe m GM
momentum i.e. =
2 R R n
 R 
L = r × p = r × mv 1 /2
16 Let gravitational field is zero at P as
∴ Areal velocity 2πR  R n −1 
shown in figure. Now, T = = 2 πR ×  
 dA  = 1 (r × m v) v  GMe 
m P 4m  
 dt  2m  R2 × R n −1 
1 /2
A B = 2π 
x r–x 1 
= (r × v)  Gme 
r 2
 R ( n + 1 )/2 
Gm G (4m ) Therefore, the direction of areal velocity = 2π  
∴ =  (Gme )1 /2 
x2 (r − x )2  d A  is the direction of (r × v), i.e.
 
 dt  or T ∝ R( n + 1 )/2
⇒ 4 x2 = (r − x )2 ⇒ 2x = r − x
r perpendicular to the plane containing r 26 Option (c) is correct, a minimum amount
⇒ x= and v.
3 of energy equal to|TE |of the moon-earth
Gm G (4m ) 9Gm 21 According to Kepler’s law, T 2 ∝ r 3 system has to be given to break
∴ VP = − − =− (unbound) the system, the sun is exerting
x r − x r 2 3

∴  1  =  1 
T r 25 r3
⇒ = force on the moon but not providing any
(∴ x = r / 3) T  r  (T ′ )2
64r 3 energy.
17 Mass of planet, M p = 10Me , or T ′ = 1600 or T ′ = 40 h
27 From conservation of energy
where, Me is mass of earth. 22 Areal velocity = dA = L = mvr = vr Total energy at the planet = Total energy
R dt 2m 2m 2 at the altitude
Radius of planet, R p = e ,
10 r GM 1 − GMm
= = GMr + (KE) surface
where Re is radius of earth. 2 r 2 R
Escape velocity is given by, dA − GMm 1
So, ∝ r = + mv 2A …(i)
2GM dt 3R 2
ve =
R In its orbit, the necessary centripetal force
2G × M p
23 The gravitational force exerted on is provided by gravitational force.
For planet, v p = satellite at a height x is
Rp mv 2A GMm
GMe m ∴ =
FG = (R + 2R ) (R + 2R )2
100 × 2GMe (R + x )2
= GM
Re where, Me = mass of the earth. ⇒ v 2A = …(ii)
3R
= 10 × v e Since, gravitational force provides the From Eqs. (i) and (ii), we get
= 10 × 11 necessary centripetal force, so 5 GMm
= 110 kms −1 (KE) surface =
GMe m mv o2 6 R
=
18 From law of conservation of energy, (R + x )2 (R + x )
28 The system will be bound at all these
1 mgh where, v o is orbital speed of satellite
mv 2 = points where the total energy = (E P + E K )
2 h GMe m
1+ ⇒ = mv o2 is negative.
R (R + x )
Here, v = kv e = k 2gR gR2 m In the given curve, at points A, B and C,
= mv o2 Q g = GMe 
or   the E P > E K .
and h = r − R (R + x )  R2 
108 40 DAYS ~ JEE MAIN PHYSICS DAY NINE

r − R
= GM 
29 According to law of conservation of v 2 GM GM 7 The initial potential energy of the
⇒ = − 
angular momentum; 2 R r  rR  particle = Work done.
mv A × OA = mv B × OB , r − R GMm
v B OA = gR  
Q GM = g 
  Ui = −
= = x  r   R2  r
v A OB 6.67 × 10−11 × 100 × 10−2
∴ v = 2 gR (r − R ) /r Ui = −
01
.
30 Total energy of satellite,
3 According to Kepler’s law, T 2 ∝ r 3 6.67 × 10−11
GMm s Ui = −
E =− …(i) T 2 = kr 3 … (i) 01
.
2r s
Orbital velocity of satellite,
Differentiating it, we have = − 6.67 × 10−10 J
GM 2T ∆T = 3kr 2 ∆r
vs = m = 10 × 10–3 kg
rs Dividing it by Eq. (i), we get
Angular momentum of satellite is given 2T ∆T 3kr 2 ∆r 3 ∆r
= ⇒ ∆T = T R = 0.1 m
by T 2
kr 3 2 r
1 /2 M = 100 kg
 GM  4 Mass of the element of length dx at a
L = m sv srs = m s   r s = (GM m2s r s )1 /2
 rs  distance x from the origin
= (2 Em s r s2 )1 /2 [from Eq. (i)] = ( A + Bx2 ) dx
We know that, work done = Difference in
31 Gravitational potential, Gm ( A + Bx2 )dx
dF = potential energy
GM x2
At outside point (solid sphere) = ∴ W = ∆U = U f − U i
r On integrating,
⇒ W = −Ui [Q U f = 0]
GM ( A + Bx2 )dx
a+L
At the surface (solid sphere)= F = Gm ∫ = 6.67 × 10−10 J
R a
x2
3GM a+L A  8 The metallic spheres will be at positions
At the centre (solid sphere)= = Gm ∫  2 + B  dx as shown in the figure.
2R a x 
Hence, (b) is correct.  1 1  
= Gm  A  −  + BL 
32 Force acting on astronaut is utilised in   a a + L   T q
providing necessary centripetal force, T cos q q
thus he feels weightlessness, as he is in 5 Even though the distribution of mass is
F
the state of free fall. unknown we can find the potential due T sin q
to ring on any axial point because from
33 Kepler’s laws are based on observations, mg
any axial point the entire mass is at the
hit and trial method and already same distance (whatever would be the From the figure,
recorded data but later on Newton nature of distribution). GM × M GM 2
proved their correctness using his laws. Potential at A due to ring is, T sinθ = F = = 2
L2 L
GM
34 It is clear that the net force on the body VA = − and T cos θ = Mg
2R GM
inside the hollow sphere is zero hence, ⇒ tanθ = 2
then net gravitational field intensity Potential at B due to ring is, gL
GM
 E = F  at any point inside the earth VB = −  GM 
  5R ⇒ θ = tan −1  2 
 m  gL 
dU = U f − U i
must also be zero.
= U B − U A = m 0 (V B − V A )
9 Given, U = 2r 3
GMm 0  1 1 
SESSION 2 = − + − dU
R  5 2  ∴ F = = −6r 2
dr
1 As the total energy of the earth satellite Wgr = − Wext
− GM  mv 2
bounded system is negative  . ⇒ Wgr = − dU = − Wext Now, =|F |= 6r 2
 2a  GMm 0  1 1  r
∴ Wext = dU = − 1 1
Where, a is radius of the satellite and M R  2 5  or mv 2 = (6r 3 ) = 3r 3
is mass of the earth. Due to the viscous 2 2
force acting on satellite, energy 6 Here, K i + U i = K f + U f As, TE = K + U
decreases continuously and radius of 1 2GMm = 3r 3 + 2r 3
∴ mv 2 − = 0+ 0
the orbit or height decreases gradually. 2 2R = 5r 3
2 Using law of conservation of energy, 1 2GMm 2 2 GM ∴ At r = 5m,
GMm 1 GMm or mv 2 = or v =
− = mv 2 − 2 2R R TE = 625J
r 2 R Hence, (b) is the correct option. Hence, (c) is the correct option.
DAY NINE GRAVITATION 109

10 Gravitational force acting on an object 12 Consider cavity as negative mass and ∴ Minimum increase in its orbital
of mass m, placed at point P at a apply superposition of gravitational velocity required to escape from the
distance h along the normal through the potential. Consider the cavity formed in Earth’s gravitational field.
centre of a circular ring of mass M and
a solid sphere as shown in figure. 2GM GM
radius r is given by v ′− v = −
V (∞ ) = 0 R R
r = 2gR − gR
P
= gR ( 2 − 1) Q g = GM 
h  
O m P ºº +  R2 
R R/2 R/2 -P
P R
M
14 Net force acting on any one particle M.
GMmh According to the equation, potential at v M
F = v
(r 2 + h2 )3 /2 an internal point P due to complete
When mass is displaced upto distance sphere. R
GM  R 
2

V3 = − 3 3R2 −   
2h, then R 45°
m m
GMm × 2h 2GMmh 2R   2  R O 45°
F ′= 2 =
(r + (2h )2 )3 /2 (r 2 + 4h2 )3 /2 R
− GM  2 R  − GM  11R2 
2

When h = r , then 3R − =
2R3  4  2R3  4  v
GMm × r GMm M v
F = 2 = −11 GM
(r × r 2 )3 /2 2 2r 2 =
8R GM 2 GM 2
and F ′ = 2
2GMmr
=
2GMm = + cos 45°
Mass of removed part (2R ) 2
(R 2 ) 2
(r + 4r 2 )3 /2 5 5r2 3 GM 2
+ cos 45°
× π   =
M 4 R M
F′ 4 2 4 2 = (R 2 ) 2
∴ = or F ′= F 4 3  2 8
F 5 5 5 5 × πR 3

3
=
GM 2 1 + 1 
 
11 Here, time period of satellite w.r.t. Potential at point P due to removed part R2 4 2
observer on equator is 24 h and the −3 GM 8 −3GM
V2 = × = This force will equal to centripetal force.
satellite is moving from West to East, so 2 R2 8R Mv 2 GM 2  1 1 
angular velocity of satellite w.r.t. the So, =  + 
Thus, potential due to remaining part at R R2  4 2
earth’s axis of rotation (considered as point P. GM
fixed) is, − 11GM  3GM  v = (1 + 2 2 )
2π 2π V P = V3 − V2 = − −  4R
ω= + , 8R  8R 
Ts Te 1 GM
(−11 + 3) GM − GM = (2 2 + 1)
= = 2 R
where, T s and Te are time periods of 8R R
satellite and the earth, respectively Hence, speed of each particle in a
π 13 Given, a satellite is revolving in a circular motion is
ω = rad/h
6 circular orbit at a height h from the 1 GM
(2 2 + 1)
= 1.45 × 10−4 rad s –1 Earth’s surface having radius of Earth R, 2 R
From v =
GM i.e. h < < R. 15 Total energy of earth (planet)-satellite
r Orbital velocity of a satellite, system is independent of eccentricity of
GM GM GM orbit and it depends on semi-major axis
⇒ rω = v = = (as h < < R )
r R+ h R and masses of the planet and satellite.
GM
⇒ r = 3 /2
Velocity required to escape, 16 Here, Statement I is wrong because as
ω 1 GMm speed of one satellite increases, its
mv ′ 2 =
6. 67 × 10−11 × 6 × 1024 2 R+ h kinetic energy and hence total energy
=
1. 45 × 10−4 2GM 2GM increases, i.e. total energy becomes less
v′ = =
⇒ r = 2.66 × 107 m R+ h R negative and hence r increases, i.e. orbit
= 2.66 × 104 km changes.
(h < < R )
110 40 DAYS ~ JEE MAIN PHYSICS DAY TEN

DAY TEN

Unit Test 1
(Mechanics)

1 Taking into account the significant figures, what should 7 A particle had a speed of 18 ms −1. After 2.4 s, its speed
be the value of 9.99 + 0.0099 ? was 30 ms −1 in the opposite direction. What were the
(a) 9.9999 (b) 10.00 (c) 10.0 (d) 10 magnitude and direction of the average acceleration of
the particle during this 2.4 s interval?
2 The maximum error in the measurement of mass and
length of the cube are 3% and 2%, respectively. The (a) 10 ms −2 (b) 15 ms −2 (c) 20 ms −2 (d) 25 ms −2
maximum error in the measurement of density will be 8 A rock is dropped from a 100 m high cliff. How long does it
(a) 5% (b) 6% (c) 7% (d) 9% take to fall first 50 m and the second 50 m?
3 If C and L denote the capacitance and inductance, then (a) 2 s, 3 s (b) 1.5 s, 2 .5 s
(c) 1.2 s, 3.2 s (d) 3.2 s , 1.3 s
the dimensional formula for C-L is same as that for
(a) frequency (b) time period 9 Two bodies of masses M1 and M 2 are dropped from
(c) (frequency) 2 (d) (time period) 2 heights H1 and H2 , respectively. They reach the ground
after time T1 and T2 , respectively. Which of the following
4 The dimensions of (velocity) 2 ÷ radius are the same as
relation is correct?
that of
1/ 2
(a) Planck’s constant (b) Gravitational constant T1  H1  T1 H1
(a) = (b) =
(c) Dielectric constant (d) None of these T2  H 2  T2 H 2
5 A soap bubble oscillates with time periodT , which in turn T1  M1 
1/ 2
T1 M1
depends on the pressure ( p ), density ( ρ ) and surface (c) = (d) =
T2  M 2  T2 M 2
tension ( σ ). Which of the following correctly represents
the expression for T 2 ? 10 As a rocket is accelerating vertically upwards
at 9.8 ms −2 near the earth’s surface, it releases a
ρσ 2
ρp 3
(a) (b) projectile with zero speed relative to rocket.
p3 σ
Immediately after release, the acceleration (in ms – 2 ) of
p 3σ ρ
(c) (d) 3 the projectile is [take, g = 9.8 ms −2 ]
ρ p σ
(a) zero (b) 9.8 ms−2 , up
6 An automobile travels on a straight road for 40 km at 30 km (c) 9.8 ms−2 , down (d) 19.6 ms−2 , down
h−1. It then continues in the same direction for another 40
11 A bullet is fired in a horizontal direction with a muzzle
km at 60 km h−1. What is the average velocity of the car
velocity of 300 ms −1. In the absence of air resistance,
during its 80 km trip?
how far will it drop in travelling a horizontal distance of
(a) 30 km h−1 (b) 50 km h−1
150 m? [take, g = 10 ms −2 ]
(c) 40 km h−1 (d) 60 km h−1
(a) 1.25 cm (b) 12.5 cm (c) 1.25 m (d) 1.25 mm
DAY TEN UNIT TEST 1 (MECHANICS) 111

12 A fixed mortar fires a bomb at an angle of 53° above the 18 A 20 kg block is placed on top of a 50 kg block as
horizontal with a muzzle velocity of 80 ms −1. A tank is shown. A horizontal force F acting on A causes an
advancing directly towards the mortar on level ground at acceleration of 3 ms −2 to A and 2 ms −2 to B as shown.
a constant speed of 5 ms −1. The initial separation (at the For this situation mark out the correct statement.
instant mortar is fired) between the mortar and tank, so
2 ms–2
that the tank would be hit is [take,g = 10 ms −2 ] B
Rough 20 kg
(a) 678.4 m (b) 614.4 m 3 ms–2
(c) 64 m (d) None of these A
Smooth F
13 A vector a is turned without a change in its length 50 kg
through a small angle dθ. The value of | ∆a | and ∆a are
respectively (a) The friction force between A and B is 40 N
(b) The net force acting on A is 150 N
(a) 0,adθ (b) adθ,0
(c) The value of F is 190 N
(c) 0, 0 (d) None of these
(d) All of the above
14 The vectors from origin to the points A and B are
19 The figure below shows a man standing stationary w.r.t. a
A = 3$i − 6$j + 2k$ and B = 2$i + $j − 2k,
$ respectively. The
horizontal conveyor belt which is accelerating at
area of the ∆OAB be 1 ms −2 . What is the net force on the man in this situation?
5 2
(a) 17 sq units (b) 17 sq units
2 5
3 5
(c) 17 sq units (d) 17 sq units
5 3
15 The sum of the magnitudes of the two forces acting at a
point is 18 and the magnitude of their resultant is 12. If
the resultant is at 90° with the force of smaller Take mass of the person to be as 70 kg. If the maximum
magnitude, what are the magnitude of the forces? acceleration of the belt, for which the man remains
(a) 12, 5 (b) 14, 4 (c) 5, 13 (d) 10, 8 stationary w.r.t. the belt, is 3 ms −2 , then the coefficient of
static friction between the man’s shoes and the belt
16 A metal sphere is hung by a string fixed to a wall. The
would be
sphere is pushed away from the wall by a stick. The
forces acting on the sphere are shown in the second (a) 70 N, 0.2 (b) 70 N, 0.3
(c) 700 N, 0.1 (d) 700 N, 0.3
diagram. Which of the following statements is wrong?
20 A parachutist is in free fall before opening her parachute.
T The net force on her has a magnitude F and is directed
θ
θ downwards. This net force is somewhat less than, her
P weight w because of air resistance. Then, she opens her
parachute. At the instant after her parachute fully inflates,
w the net force on her would be
(a) greater than F and still directed downwards
(a) P = w tanθ (b) T + P + w = 0
(c)T 2 = P 2 + w 2 (d)T = P + w (b) less than F and still directed downwards
(c) zero
17 Three blocks A , B and C of masses 5 kg, 3 kg and 2
(d) directed upwards but could be more or less than F
kg respectively are placed on a horizontal surface.
The coefficient of friction between C and surface is 0.2 21 The drive shaft of an automobile rotates at 3600 rpm and
while between A and surface is zero and between B transmits 80 HP up from the engine to the rear wheels. The
and surface is zero. If a force F = 10 N is first applied on torque developed by the engine is
A as shown and then in second case on C (shown (a) 16.58 N-m (b) 0.022 N-m
dotted), then the ratio of normal contact force between (c) 158.31 N-m (d) 141.6 N-m
B and C in first with respect to the second case is 22 A disk starts rotating from rest about its axis with an
[Take, g = 10 ms −2 ] angular acceleration equal to α = 10 rads – 2 , where t is
time in seconds. At t = 0, the disk is at rest. The time
F A taken by the disk to make its, first complete revolution is
B C 1/ 3 1/ 3 1/ 2 1/ 3
6π 3π 2π 6π
(a)   (b)  (c)  (d)  
 5   10   5   13 
13 12 1 2
(a) (b) (c) (d)
12 13 2 1
112 40 DAYS ~ JEE MAIN PHYSICS DAY TEN

23 A rectangular block of mass 29 If an object weighs 270 N at the earth’s surface, what will
M and height a is resting on a 3F be the weight of the object at an altitude equal to twice
smooth level surface. A force the radius of the earth?
F is applied to one corner as a (a) 270 N (b) 90 N
shown in the figure. At what (c) 30 N (d) 60 N
a/2
point should a parallel force 30 At its aphelion, the planet mercury is 6.99 × 1010 m from
F
3 F be applied in order that the sun, and at its perihelion it is 4.6 × 1010 m from the sun.
the block undergoes pure If its orbital speed at aphelion is 3.88 × 104 ms −1, then its
translational motion? Assume, the normal contact force perihelion orbital speed would be
between the block and surface, passes through the (a) 3.88 × 104 ms −1 (b) 5.90 × 104 ms −1
centre of gravity of the block.
(c) 5.00 × 104 ms −1 (d) 5.5 × 104 ms −1
a
(a) , vertically above centre of gravity
3 31 If R is the radius of the orbit of a geosynchronous satellite
a and another satellite is orbiting around the earth in a
(b) , vertically above centre of gravity
6 R
(c) No such point exists circular orbit of radius , then its time period would be
2
(d) It is not possible
(a) 6 2 h (b) 6 h
24 A helicopter takes off along the vertical with 3 ms −2 with (c) 12 h (d) Cannot be determined
zero initial velocity. In a certain time t, the pilot switches
32 A cylinder of mass M and radius r is mounted on a
off the engine. The sound dies away at the point of take
frictionless axle over a well. A rope of negligible mass is
off in 30 s. When engine is switched off, velocity of the
wrapped around the cylinder and a bucket of mass m is
helicopter is
suspended from the rope. The linear acceleration of the
(a) 80 ms −1 (b) 30 ms −1 (c) 25 ms −1 (d)100 ms −1 bucket will be
25 To maintain a rotor at an uniform angular speed of r
−1
200 rads , an engine needs to transmit a torque of
180 N-m. What is the power required by the engine? w
(Assume efficiency of the engine to be 80%) v

(a) 36 kW (b) 18 kW (c) 45 kW (d) 54 kW h


26 When a ball is whirled in a circle and the string Bucket
supporting the ball is released, the ball flies off M
tangentially. This is due to
mg 2mg
(a) the action of centrifugal force (a) (b)
M + 2m m + 2M
(b) inertia for linear motion mg 2mg
(c) centripetal force (c) (d)
2M + m M + 2m
(d) some unknown cause
27 When a particle is moving in a vertical circle, 33 A merry-go-round, made of a ring-like platform of radius
R and mass M, is revolving with angular speed ω. A
(a) its radial and tangential acceleration both are constant
person of mass M is standing on it. At one instant, the
(b) its radial and tangential acceleration both are varying
person jumps off the round, radially away from the centre
(c) its radial acceleration is constant but tangential
acceleration is varying of the round (as seen from the round). The speed of the
(d) its radial acceleration is varying but tangential round of afterwards is
acceleration is constant (a) 2ω (b) ω (c) ω / 2 d) 0
28 A particle of mass m slides on a O R m
A 34 A canon ball is fired with a velocity of 200 ms − 1 at an
30°
quarter part of a smooth sphere of angle of 60° with the horizontal. At the highest point, it
radius R as shown in the figure. It is explodes into three equal fragments. One goes vertically
released from rest at A , the normal P upwards with a velocity of 100 ms − 1 and other goes
contact force exerted by surface on vertically downwards with 100 ms − 1. The third one moves
the particle, when it reaches P is with a velocity of
(a) 100 ms − 1, horizontally
mg 3 mg
(a) (b) (b) 300 ms − 1, horizontally
2 2
(c) 200 ms − 1, at 60° with horizontal
3 mg 3
(c) mg × + mg (d) (d) 300 ms − 1, at 60° with horizontal
2 2
DAY TEN UNIT TEST 1 (MECHANICS) 113

35 A rocket of initial mass (including fuel) 15000 kg ejects P. If the velocity of the car at an instant is v, then after
mass at a constant rate of 25 kgs −1 with a constant travelling how much distance, it becomes double?
relative speed of 15 kms −1. The acceleration of the
rocket, 5 min after the blast is [Neglect gravity effect] m F
(a) 40 ms −2 (b) 50 ms −2
(c) 60 ms −2 (d) 45 ms −2
36 An elevator of total mass (elevator + passenger) 7mv 3 4 mv 3 mv 3 18 mv 3
−1
(a) (b) (c) (d)
1800 kg is moving up with a constant speed of 2 ms . 3P 3P P 7P
Frictional force of 2000 N is opposing its motion. The
minimum power delivered by the motor to the elevator is Direction (Q. Nos. 39-40) Each of these questions contains
[take, g = 10 ms −2 ] two statements Statement I and Statement II. Each of these
questions also has four alternative choices, only one of which is
(a) 36 kW (b) 4 kW the correct answer. You have to select one of the codes (a), (b),
(c) 40 kW (d) − 40 kW
(c) and (d) given below
37 A bead of mass 1 / 2 kg starts from rest from a point A to (a) Statement I is true, Statement II is true; Statement II is
B move in a vertical plane along a smooth fixed quarter the correct explanation for Statement I
ring of radius 5 m, under the action of a constant (b) Statement I is true, Statement II is true; Statement II is
horizontal force F = 5 N as shown. The speed of the bead not the correct explanation for Statement I
as it reaches the point B is (c) Statement I is true; Statement II is false
[Take g = 10 ms −2 ] (d) Statement I is false; Statement II is true
F 39 By considering the earth to be non-spherical
A
Statement I As, one moves from equator to the pole of
the earth, the value of accelaration due to gravity
increases.
R=5m
Statement II If the earth stops rotating about its own axis,
the value of accelaration due to gravity will be same at
B pole and at equator.
(a) 14.14 ms (b) 7.07 ms −1 (c) 5 ms −1
−1
(d) 25 ms −1 40 Statement I Total torque on a system is independent of
38 A car (treat it as particle) of mass m is accelerating on a the origin if the total external force is zero.
level smooth road under the action of a single force F . Statement II Torque due to a couple is independent of
The power delivered to the car is constant and equal to the origin.

ANSWERS
1. (d) 2. (d) 3. (d) 4. (d) 5. (a) 6. (c) 7. (c) 8. (d) 9. (a) 10. (c)
11. (c) 12. (a) 13. (b) 14. (a) 15. (c) 16. (d) 17. (a) 18. (d) 19. (b) 20. (d)
21. (c) 22. (c) 23. (b) 24. (a) 25. (c) 26. (b) 27. (b) 28. (b) 29. (c) 30. (b)
31. (a) 32. (d) 33. (a) 34. (b) 35. (b) 36. (c) 37. (a) 38. (a) 39. (c) 40. (a)
114 40 DAYS ~ JEE MAIN PHYSICS DAY TEN

Hints and Explanations


1 9.99 + 0.0099 = 9.9999 = 10 The average velocity is Let t be the time taken by the bullet to
80
v av = = 40 kmh −1 cover a horizontal distance of 150 m,
2 Here, ∆M = 3% = ± 0.03 2 then
M
∆l 1
and = 2% = ± 0.02
7 The average acceleration in the given 150 = 300t ⇒ t = s
l interval is 2
∆V 3∆l (v 2 − v 1 ) 1 2 1 1
Hence, = = ± 0.06 aav = y = gt = × 10 × = 1.25 m
V l (t 2 − t 1 ) 2 2 4
∆ρ ∆ M ∆V Take, v 1 = 18 ms −1 , v 2 = − 30 ms −1 ,
Now, = + 12 The situation is shown clearly in figure.
ρ M V t 1 = 0 and t 2 = 2.4s 2u sin θ
Time of flight of bomb is T =
∴ Density = Mass  − 30 − 18 g
 aav = = −20 ms −2
Volume  2.4 80 ms–1
= ± 0.09 = 9% The − ve sign indicates that the 5 ms–1
3 Time period of C - L oscillations is given acceleration is opposite to the initial 53°
by 2π LC . direction of motion.
Hence, [LC ] = [time period]2 8 Using second equation of motion, Mortar Tank
1 s1 s2
4 Dimensional formula of (velocity)2 ÷ y = ut + at 2 , at v 0 = 0
2 2 × 80 × 4
[M 0 LT −1 ]2 = = 12.8s
radius = = [M 0 LT −2 ] 1
and solve y = v 0 t + gt 2 for t 10 × 5
[M 0 LT 0 ] 2
= [acceleration] Distance travelled by the tank in T
So, t = 2 y /g
Note In circular motion, centripetal seconds is
For y = 50 m s1 = 5T = 5 × 12.8 = 64 m
V2
acceleration is . 2(50)
R So, t1 = − = 3.2 s The horizontal distance travelled by
9.8 bomb in T seconds is
5 Here, T = p ρ σ
2 a b c
…(i) For y = 100 m
u2 sin 2θ
Putting the dimensions of the quantities t2 =
2 (100)
= 4.5 s s2 =
in RHS, we get 9.8 g
= [ML−1 T −2 ] a [ML−3 ]b [MT −2 ] c 3 4
Hence, the difference is the time taken 802 × 2 × ×
= [M a + b +c
L− a − 3 b T −2 a − 2 c ] to fall the second 50 m = 5 5
Hence, a+ b + c = 0 …(ii) = 4.5 − 3.2 = 1.3 s 10
= 614.4 m
− a − 3b = 0 …(iii) 9 Distance of fall is independent of the [Q sin 2θ = 2sin θ ⋅ cos θ]
and − 2a − 2c = 2 …(iv) mass of the bodies
1 1 So, required separation
On solving, Eqs. (ii) (iii) and (iv), we get
H 1 = gT12 and H 2 = gT22 s = s1 + s2 = 678.4 m
a = − 3 b, b = 1 and c = 2 2 2
So, after putting the values of a, b and c T1  H 1 
1 /2 13 From the figure|OA | = a and|OB| = a.
in Eq. (i), we get Hence, =
T2  H 2  Also, from the triangle rule
ρσ 2
T2 = B
p3 10 As nothing has been mentioned that ∆a
w.r.t. which frame of reference is to be
6 Average velocity (v av ) = ∆x found, it means we have to compute a A
∆t w.r.t frame of reference of earth. As the
where, ∆x is the displacement in a given object is released, its acceleration w.r.t dq a
time interval. For first part of the ground is only due to the influence of
O
journey, time taken by the car, gravity of the earth and hence is equal
∆t 1 =
40
= 1.33 h to 9.8 ms – 2 in the downward direction. OB − OA = AB = ∆a
30 ⇒ |∆a | = AB
11 Let the bullet, dropped by y metre Arc
For second part of the journey time while covering a horizontal distance of Using, Angle =
taken by the car Radius
150 m.
40 ⇒ AB = a⋅ dθ
∆t 2 = = 0.67 h 300 ms –1
60 So, |∆ a | = a ⋅ dθ
Hence, the total displacement 150 m ∆a means a change in magnitude of the
∆x = ∆x1 + ∆x2 vector, i.e.,
= 40 + 40 = 80 km y |OB| − |OA|⇒ a − a = 0
and the total time interval So, ∆a = 0
∆t = ∆t 1 + ∆t 2 = 1.33 + 0.67 = 2 h
DAY TEN UNIT TEST 1 (MECHANICS) 115

14 Given OA = a = 3$i − 6$j + 2k$ and 10 − 4 6 22 As, ω = ω0 ± α t


⇒ a= =
10 10
OB = b = 2 $i + $j − 2k$ ⇒ ω 0 = 10 t
a

i$ $j k$ or = 10 t
dt
∴ (a × b) = 3 − 6 2
N2 F On integrating, (for one complete
2 1 −2 f
revolution)
= (12 − 2)$i + (4 + 6)$j + (3 + 12) k$ For C, F − N 2 − f = 2a 2π T
6 24 ∫0 dθ = ∫0 10 t dt
= 10$i + 10$j + 15k$ ⇒ N 2 = 10 − 4 − 2 × = N
10 5 10T 2
⇒ 2π =
⇒ |a × b| = 10 + 10 + 15
2 2 2 N 1 26 13 2
Required ratio is, = =
= 425 = 5 17 N 2 24 12 1
2π  2
⇒ T =  
1 18 As the acceleration of A and B are  5
Area of ∆OAB = |a × b|
2 different, it means there is a relative
5 17 motion between A and B. The free body 23 The free body diagram of the block can
= sq units. diagram of A and B can be drawn as be drawn as shown. As body has to move
2
in a pure translational motion, the torque
15 Let P be the smaller force and Q be the B
about the centre of gravity must be zero.
greater force, then according to the f
problem
N1 mg N 3F
P + Q = 18 … (i)
x
R= P 2 + Q 2 + 2PQ cos θ = 12 … (ii) N1 a
Q sin θ
tan φ = = tan 90° = ∞
P + Q cos θ f Mg
A F F
∴ P + Q cos θ = 0 … (iii)
By solving Eqs. (i), (ii) and (iii), we get a a
3F × x = F × ⇒ x=
P = 5 and Q = 13 N2 Mg 2 6

16 For A, F − f = Ma A = 50 × 3 = 150 N 24 The altitude of the helicopter when


T T cos q
For B, f = maB = 20 × 2 = 40 N at 12
q engine is switched off h = . Sound is
So, F = 150 + 40 = 190 N 2
at 12
T sin q 19 As person remains stationary w.r.t. belt, not heard after t 2 = t 1 + , where
P 2c
so acceleration of the person w.r.t.
ground is the same as that of belt w.r.t. c = speed of sound.
w
ground. So, net force acting on the at 12 + 2ct 1 − 2ct 2 = 0
As the metal sphere is in equilibrium person is, f = ma = 70 × 1 = 70 N. Let
under the effect of the three forces coefficient of static friction between the −2 c ± 4c 2 + 8cat 2
⇒ t1 =
therefore, T + P + w = 0 man’s shoes and belt is µ S , then 2a
From the figure, T cos θ = w …(i) amax = µ s g c c 2
2c
T sin θ = P … (ii) t1 = − + + t2
3 a a2 a
µS = = 03
.
From Eqs. (i) and (ii), we get 10
P = W tan θ ∴ v = at 1 = − c + c 2 + 2cat 2
T 2 = P2 + w 2 20 As the parachute inflates fully, the force
and = − 320 + (320)2 + 2 × 320 × 3 × 30
of air friction increases by a large
[as, sin θ + cos θ = 1]
2 2
amount and the parachutist starts = 1600 × (10)2 − 320
17 Friction force between C and surface is decelerating, i.e. net force acting on her
is in upward direction but the = 400 − 320 = 80 ms −1
f = µ × 2g = 0.2 × 2 × 10 = 4 N
magnitude of the net force cannot be
Case I (5 + 3 + 2) a = F − f 25 Power required for rotor, P = τ ⋅ ω
determined from given information.
10 − 4 6 = 180 × 200 = 36 kW
⇒ a= =
10 10 21 From P = τω P
Power of engine, P0 = = 45 kW
P 0.8
a τ =
ω [as efficiency is 80%]
F
N1 It is given, 26 This is due to Intertia for linear motion of
P = 80 HP = 80 × 746 W ball.
f
= 59680 N- ms – 1
For C, N 1 − f = 2a 3600 27 In a vertical circle, both radial and
6 26 ω = 3600 rpm = × 2 π rads – 1 tangential components of the
⇒ N1 = 2 ×+ 4= N 60
10 5 acceleration change direction at every
= 120 π rad s − 1 instant.
Case II F − f = (5 + 3 + 2) a
So, τ = 15831
. N-m
116 40 DAYS ~ JEE MAIN PHYSICS DAY TEN

28 Apply Work-Energy theorem at A and P, or T =


Ma Mass of rocket at t = 5min after the
2 2 blasting starts, is
mv R
− 0 = mg × mg − T m = 15000 − 25 × 5 × 60 = 7500
2 2 ∴ a=
m So, F = ma
O R A Ma
mg − F
⇒ a=
30°
⇒ a= 2 = 2mg − Ma m
R/2 m 2m 15 × 25000
=
⇒ 2ma + Ma = 2mg 7500
P
60° 2mg
mg cos 60° ⇒ a= = 50 ms −2
mg 2m + M
mg sin 60° Note If gravity is not neglected, then
33 As no external torque acts on the F − mg = ma check whether acceleration
Use dynamical equations at P, system, angular momentum should be is constant here.
mv 2 conserved.
N − mg cos 60° = 36 The net downward force on the elevator
Hence, Iω = constant …(i)
R is,
mg 3 mg where, I is moment of inertia of the
⇒ N = mg + = system and ω is angular velocity of the F1 = mg + f = 18000 + 2000 = 20000 N
2 2
system. So, the motor has to work against this
29 Let m be the mass of the object and g is From Eq. (i), we get force.
the acceleration due to gravity at the I1ω1 = I2ω2
earth’s surface, then mg = 270 N. To move the elevator with a constant
[where ω1 and ω2 are angular velocities speed, the minimum power delivered by
The acceleration due to gravity at an before and after jumping] the motor to the elevator must be,
altitude of 2Re is, I P = F ⋅ v = 20000 × 2 = 40 kW
⇒ Iω = × ω2
GM GM 1 2
g′ = = = ×g
(R e + 2R e )2 9Re2 9 [as mass reduced to half, hence moment 37 Applying the work-energy theorem,
1
So, required weight of inertia also reduced to half] × mv 2 − 0 = F × R + mg × R
⇒ ω2 = 2ω 2
mg 270
= mg ′ = = = 30 N 1 1 1
9 9 ⇒ × × v 2 = 5 × 5 + × 10 × 5 = 50
34 At the highest point, velocity before 2 2 2
explosion is v cos 60° along X-axis.
30 From the conservation of angular v = 200
momentum, By law of conservation of momentum,
m vA rA = mv p r p (mv cos 60° ) i$ =
m
(100$j ) = 14.14 ms −1
3
3.88 × 104 × 6.99 × 1010
⇒ vp = m mv ′ 38 As, P = Fv = mv dv × v
4.6 × 1010 + (− 100 $j ) + ds
3 3
= 5.90 × 104 ms −1
2v s

⇒ v′ =
3v 3 × 200 $
= i
⇒ ∫v mv 2dv = ∫0 P ds
31 From the Kepler’s law, T 2 ∝ r 3 2 2 7mv 3
3 /2 = 300 $i ms − 1 ⇒ s=
T1  r1  3P
⇒ = 
T2  r2  or 300 ms − 1 along X-axis or horizontally.
39 As one go from equator to pole of the
3 /2 35 Thrust force acting on the rocket is, earth, the value of g increase due to
24  R 
⇒ =  =2 3 /2
dm decrease in latitude (λ ). Also, the earth is
T2  R/2  F = v rel non-spherical, this implies the value of g,
dt
24 24 at the poles and equitorial point on the
⇒ T2 = = = 6 2h F = 15 × 1000 × 25N earth’s surface are unequal due to its
23 /2 2 2
different distances from earth’s centre.
32 Weight of bucket acts downwards while 40 If net force on the system is zero, it can
tension T in opposite direction a be resolved into two equal and opposite
mg − T = ma forces which can be considered to form a
Also, τ = I α = rT couple.
1 1
⇒ Mr 2α = rT ⇒ M (rα ) = T F
2 2
DAY ELEVEN

Oscillations
Learning & Revision for the Day
u Periodic Motion u Force and Energy in SHM u Free, Damped, Forced and
u Simple Harmonic Motion u Composition of Two SHMs Resonant Vibrations
u Oscillations of a Spring u Simple Pendulum

Periodic Motion
A motion which repeats itself over a regular interval of time is called a periodic motion.
A periodic motion in which a body moves back and forth repeatedly about a fixed point
(called mean position) is called oscillatory or vibratory motion.
l
Period The regular interval of time after which periodic motion repeats itself is called
period of the motion.
l
Frequency The number of times of motion repeated in one second is called frequency
of the periodic motion. Every oscillatory motion is periodic but every periodic motion
is not an oscillatory motion.
l
Displacement as a Function of Time In a periodic motion each displacement value is
repeated after a regular interval of time, displacement can be represented as a
function of time.
y = f (t )
l
Periodic Function A function which repeats its value after a fix interval of time is
called a periodic function.
y(t ) = y(t + T )
where, T is the period of the function.
Trigonometric functions sin θ and cos θ are simplest periodic functions having period PREP
of 2π.
MIRROR
Your Personal Preparation Indicator
Simple Harmonic Motion
u No. of Questions in Exercises (x)—
Simple Harmonic Motion (SHM) is that type of oscillatory motion in which the particle
u No. of Questions Attempted (y)—
moves to and fro or back and forth about a fixed point under a restoring force, whose
magnitude is directly proportional to its displacement u No. of Correct Questions (z)—
(Without referring Explanations)
i.e. F ∝ x or F = − kx
where, k is a positive constant called the force constant or spring factor and x is u Accuracy Level (z / y × 100)—
displacement. u Prep Level (z / x × 100)—
d2 y
Differential equations of SHM, for linear SHM, 2 + ω2 y = 0, In order to expect good rank in JEE,
dt your Accuracy Level should be above
d2 θ
for angular SHM, 2 + ω2 θ = 0 85 & Prep Level should be above 75.
dt
118 40 DAYS ~ JEE MAIN PHYSICS DAY ELEVEN

Phase Difference If two particles perform S.H.M and their


Terms Related to SHM
l

equations are
The few important terms related to simple harmonic motion y1 = a sin(ωt + φ1 ) and y2 = a sin(ωt + φ2 )
are given as
phase difference ∆φ = (ωt + φ2 ) − (ωt + φ1 ) = φ2 − φ1
l
Displacement The displacement of a particle executing
SHM is, in general, expressed as y = A sin (ωt − φ).
l
Time Period The time taken by a particle to complete one
oscillation is called time period. It is denoted by T.
where, A is the amplitude of SHM, ω is the angular
 2π  ∴ Time period of SHM,
frequency  where ω = = 2 πν and φ is the initial phase
 T  2π | y| Displacement
T = = 2π = 2π
of SHM. However, displacement may also be expressed as ω | a| Acceleration
x = A cos (ω t − φ). l
Frequency and Angular Frequency It is defined as the
Displacement number of oscillations executed by body per second. SI
a
unit of frequency is hertz.
T/2 T
O Time
Angular frequency of a body executing periodic motion is
equal to product of frequency of the body with factor 2π.
–a Angular frequency, ω = 2 πn.

l
Amplitude The maximum displacement on either side of Oscillations of a Spring
mean position is called amplitude of SHM. If the mass is once pulled, so as to stretch the spring and is
l
Velocity The velocity of a particle executing SHM at an then released, then a restoring force acts on it which
instant is defined as the time rate of change of its continuously tries to restore its mean position.
displacement at that instant.
dy Elongation
Velocity, = v = ω A2 − y 2
dt x1
At the mean position ( y = 0), during its motion
v = Aω = vmax and at the extreme positions ( y = ± A), v = 0. Natural position
∴ Velocity amplitude, v max = Aω
Compression
Velocity
T x2

T/2
O Time Restoring force F = − k l,
where k is force constant and l is the change in length of the
spring.
Here, x1 = x2 = l
l
Acceleration The acceleration of a particle executing SHM
at an instant is defined as the time rate of change of
l
The spring pendulum oscillates simple harmonically
velocity at that instant. having time period and frequency given by
d2 y m
Acceleration, = a = − ω2 y T = 2π
dt 2 k
The acceleration is also a variable. 1 k
and ν=
At the mean position ( y = 0), acceleration a = 0 and at the 2π m
extreme position ( y = ± A), the acceleration is amax = − Aω2 . l
If the spring is not light but has a mass ms , then
∴ Acceleration amplitude, amax = Aω 2
m + 1 / 3 ms
Acceleration T = 2π
k
l
If two masses m1 and m2 , k
m1 m2
connected by a spring, are made
T/2 T
O Time to oscillate on a horizontal
µ
surface, then its period will be T = 2π
k
m1 m2
l
Phase Phase is that physical quantity which tells about the where, µ = = reduced mass of the system.
position and direction of motion of any particle at any m1 + m2
moment. It is denoted by φ.
DAY ELEVEN OSCILLATIONS 119

and r1 + r2 = r = resultant position of the particle


Series Combination of Springs where, m = mass of the particle.
If two springs of spring constants k1 and k2 are joined r1 , r2 = positions of the particle under two forces.
in series (horizontally or vertically), then their
equivalent spring constant ks is given by k1 There are two cases
1 1 1 k k l
When two SHM are in same
= + ⇒ ks = 1 2 direction the resultant is given by
ks k1 k2 k1 + k2 k2
A
A2
x = x1 + x2 = A sin(ωt + β) β φ
m m(k1 + k2 )
∴ T = 2π = 2π m where, x1 = A1 sin ωt , A1
ks k1 k2
x2 = A2 sin(ωt + φ)
A= A21 + 2 A 1 A 2 cos φ + A 22
Parallel Combination of
A2 sin φ
Springs and tan β =
A 1 + A 2 cos φ
If two springs of spring constants k1 and k2 are k1 k2
joined in parallel as shown in figure, then their For any value of φ other than 0 and π resultant amplitude is
equivalent spring constant k p = k1 + k2 hence, between| A 1 − A 2| and A 1 + A 2 .
m m l
When two SHM are mutually y
T = 2π = 2π m
D C
kp (k1 + k2 ) perpendicular to each other.
The resultant SHM is given by 2A2 x

x2
y 2
2 xy cos φ
Force and Energy in SHM + − A B
A12 A22 A1 A2 2A1
l
Force For an object executing SHM, a force always acts on
= sin2 φ (ellipse)
it, which tries to bring it in mean position, i.e. it is always
directed towards mean position. where, x = A1 sin ωt and y = A2 sin(ωt + φ)
The equation of motion, F = ma , Here, x is always between − A1 to + A1 and y is always
∴ F = − mω x 2
[Q a = − ω x ]
2 between − A2 to + A2 .

 k  NOTE Special Cases in Composition of y


= − kx Q ω =  Two SHMs D C
 m A A2
• When φ = 0, y = 2 x x
Here, negative sign shows that direction of force is always A1 O A1
opposite to the direction of displacement. A B
y
l
Energy If a particle of mass m is executing SHM, then at a D C
A2
displacement x from the mean position, the particle • When φ = π, y = − x A2
possesses potential and kinetic energy. A1 x
O A1
At any displacement x, A B
1 1 y
Potential energy, U = m ω2 x2 = k x2
2 2 • When φ = π / 2. If A1 = A2 = A, D F C
1 1 x2 y2 A2
Kinetic energy, K = m ω2 ( A2 − x2 ) = k ( A2 − x2 ) then + =1 x
2 2 A12 A 22 G A1 E

1 x 2 + y 2 = A2 (circle) A H B
Total energy, E = U + K = m ω A = 2 π 2 mν2 A2
2 2
2
If there is no friction, the total mechanical energy,
E = K + U, of the system always remains constant even
Simple Pendulum
though K and U change. A simple pendulum, in practice, consists of a heavy but small
sized metallic bob suspended by a light, inextensible and
flexible string. The motion of a simple pendulum is simple
Composition of Two SHMs harmonic for very small angular displacement (θ) whose time
If a particle is acted upon two separate forces each of which period and frequency are given by
can produce a simple harmonic motion. The resultant motion l 1 g
of the particle would be a combination of two SHMs. T = 2π and ν =
g 2π l
d2 r
For which F1 + F2 = m where, l is the effective length of the string and g is
dt
acceleration due to gravity.
120 40 DAYS ~ JEE MAIN PHYSICS DAY ELEVEN

l
If a pendulum of length l at temperature θ°C has a time where, R = radius of the earth.
period T, then on increasing the temperature by ∆θ° C its l
The graphs l -T and l -T 2 intersect at T = 1 s.
time period changes to ∆T,
∆T 1 y
where, = α ∆θ T = 1s
T 2
l–T
where, α is the temperature coefficient of expansion of the
string.
l
A second’s pendulum is a pendulum whose time period is
2s. At a place where g = 9.8 ms −2 , the length of a second’s l–T2
O x
pendulum is 0.9929 m (or 1 m approx).
l
If the bob of a pendulum (having density ρ) is made to l
The graph between T 2 and 1/g is a straight line.
oscillate in a non-viscous fluid of density σ, then it can be y
shown that the new period is
l
T = 2π
 σ T2
g 1 − 
 ρ
l
If a pendulum is in a lift or in some other carriage moving
vertically with an acceleration a, then the effective value of O x
the acceleration due to gravity becomes (g ± a) and hence, 1/g

T = 2π
l l
The graph between T 2 and g is a rectangular hyperbola.
(g ± a)
y
Here, positive sign is taken for an upward accelerated
motion and negative sign for a downward accelerated T2
motion.
l
If a pendulum is made to oscillate in a freely falling lift or
an orbiting satellite then the effective value of g is zero and
hence, the time period of the pendulum will be infinity and O x
g
therefore pendulum will not oscillate at all.
l
If the pendulum bob of mass m has a charge q and is
oscillating in an electrical field E, then Free, Damped, Forced and
l
T = 2π
 qE 
Resonant Vibrations
g ±  Some of the vibrations are described below.
 m
The positive sign is to be used if the electrical force is
acting vertically downwards and negative sign if the Free Vibrations
electrical force is acting vertically upwards. If a body, capable of oscillating, is slightly displaced from its
l
If pendulum of charge q is oscillating in an electric field E position of equilibrium and then released, it starts oscillating
acting horizontally, then with a frequency of its own.
l Such oscillations are called free vibrations.The frequency
T = 2π
2
q E 2 with which a body oscillates is called the natural frequency
g2 + and is given by
m2
1 k
l
If the length of a simple pendulum is increased to such an ν0 =
2π m
extent that l → ∞, then its time period is
R Here, a body continues to oscillate with a constant amplitude
T = 2π = 84.6 min and a fixed frequency.
g
DAY ELEVEN OSCILLATIONS 121

Damped Vibrations Forced Vibrations


The oscillations in which the amplitude decreases gradually The vibrations in which a body oscillates under the effect of
with the passage of time are called damped vibrations. an external periodic force, whose frequency is different from
Damping force, Fd = − bv
the natural frequency of the oscillating body, are called forced
where, v is the velocity of the oscillator and b is a damping
vibrations.
constant. The displacement of the oscillator is given by
x(t ) = Ae −bt /2 m sin(ω′t + φ) In forced vibrations the oscillating body vibrates with the
frequency of the external force and amplitude of oscillations
k b2
where, ω′ = the angular frequency = − is generally small.
m 4 m2
The mechanical energy E of the oscillator is given
1
Resonant Vibrations
by E (t ) = kA2e −bt / m It is a special case of forced vibrations in which the
2
A frequency of external force is exactly same as the natural
ing
amp

x frequency of the oscillator.


all d

As a result, the oscillating body begins to vibrate with a large


t
Sm

amplitude leading to the phenomenon of resonance to occur.


Larger Resonant vibrations play a very important role in music and
damping
w0 w in tuning of station/channel in a radio/TV etc.
(a) (b)

DAY PRACTICE SESSION 1

FOUNDATION QUESTIONS EXERCISE


1 The displacement of a particle is represented by the 4 The relation between acceleration and displacement of
π  four particles are given below. Which one of the particle
equation y = 3 cos  − 2ω t  . The motion of the
4  is exempting simple harmonic motion?
particle is (a) ax = +2 x (b) ax = +2 x 2
(a) simple harmonic with period 2 π / ω (c) ax = −2 x 2
(d) ax = −2 x
(b) simple harmonic with period π / ω 5 A wave travelling along the x-axis is described by the
(c) periodic but not simple harmonic equation y ( x , t ) = 0.005 cos (α x − β t ). If the wavelength
(d) non-periodic and the time period of the wave are 0.08 m and 2.0 s,
2 The displacement of a particle is represented by the respectively, then α and β in appropriate units are
0.08 2.0
equation y = sin 3ωt . The motion is (a) α = 25.00 π, β = π (b) α = ,β =
π π
(a) non-periodic 0.04 10
. π
(c) α = ,β = (d) α = 12.50 π, β =
(b) periodic but not simple harmonic π π 2.0
(c) simple harmonic with period 2 π / ω
6 The maximum velocity of a particle executing simple
(d) simple harmonic with period π / ω
harmonic motion with an amplitude 7 mm, is 4.4 ms −1.
3 Motion of an oscillating liquid column in a U-tube is The period of oscillation is
(a) periodic but not simple harmonic (a) 0.01 s (b) 10 s (c) 0.1 s (d) 100 s
(b) non-periodic 7 A point mass oscillates along the x-axis according to the
(c) simple harmonic and time period is independent of the law x = x 0 cos (ω t − π /4). If the acceleration of the
density of the liquid particle is written as a = A cos (ω t + δ ), then
π π
(d) simple harmonic and time period is directly proportional (a) A = x 0 ,δ = − (b) A = x 0ω2 ,δ =
to the density of the liquid 4 4
π 3π
(c) A = x 0ω2 ,δ = − (d) A = x 0ω2 ,δ =
4 4
122 40 DAYS ~ JEE MAIN PHYSICS DAY ELEVEN

8 A body is executing SHM when its displacement from the 15 If a spring of stiffness k is cut into two parts A and B of
mean position are 4 cm and 5 cm and it has velocity length lA : lB = 2 : 3, then the stiffness of spring A is given
10 cms − 1 and 8 cms − 1, respectively. Its periodic time t by ª AIEEE 2011
2π 3π 5 3k 2k
(a) s (b) π s (c) s (d) 2 π s (a) k (b) (c) (d) k
2 2 2 5 5
9 A block rests on a horizontal table, which is executing 16 Two springs of force constants k1 and k 2, are connected
SHM in the horizontal direction with an amplitude a. If the to a mass m as shown. The frequency of oscillation of the
coefficient of friction is µ, then the block just starts to slip mass is ν. If both k1 and k 2 are made four times their
when the frequency of oscillation is original values, the frequency of oscillation becomes
1 µg a 1 a a k1 k2
(a) (b) 2 π (c) (d) m
2π a µg 2 π µg µg

10 A coin is placed on a horizontal platform, which ν ν


(a) (b) (c) 4ν (d) 2 ν
undergoes horizontal SHM about a mean position O. The 2 4
coin placed on the platform does not slip, when angular
17 Two springs of force constant k and 2k are connected to
frequency of the SHM is ω. The coefficient of friction
a mass as shown below. The frequency of oscillation of
between the coin and platform is µ. The amplitude of
the mass is
oscillation is gradually increased. The coin will be begin
to slip on the platform for the first time
(a) at the mean position
(b) at the extreme position of the oscillation
(c) for an amplitude of µ g /ω2 2k
m
k
(d) for an amplitude of g /µω2
11 Two particles A and B are oscillating about a point O
1 k 1 2k 1 3k 1 m
along a common line such that equation of A is given as (a) (b) (c) (d)
2π m 2π m 2π m 2π k
x1 = a cos ω t and equation of B is given as
 π 18 A block P of mass m is placed on a horizontal frictionless
x 2 = b sin ω t +  .
 2 plane. A second block Q of the same mass m is placed
on it and is connected to a spring of spring constant k,
Then, the motion of A w.r. t. B is
the two blocks are pulled by a distance A. Block Q
(a) a simple harmonic motion with amplitude (a − b)
oscillates without slipping. What is the maximum value of
(b) a simple harmonic motion with amplitude (a + b)
frictional force between the two blocks?
(c) a simple harmonic motion with amplitude a 2 + b 2
(d) not a simple harmonic motion but oscillatory motion
12 Two particles execute simple harmonic motion on same k µs
straight line with same mean position, same time period A Q
6 s and same amplitude 5 cm. Both the particles start
P
SHM from their mean position (in same direction) with a
time gap of 1 s. Find the maximum separation between
(a) k A/ 2 (b) k A (c) µ − mg (d) Zero
the two particles during their motion.
(a) 2 cm (b) 3 cm (c) 4 cm (d) 5 cm 19 A particle of mass M is attached to three springs A, B
and C having equal force constant k. If the particle is
13 A particle is acted simultaneously by mutually
pushed a little towards any one of the springs and then
perpendicular simple harmonic motion x = a cos ωt and
left on its own, find the time period of its oscillation.
y = a sin ωt . The trajectory of motion of the particle will be
(a) an ellipse (b) a parabola
A 120° B
(c) a circle (d) a straight line
k k
14 A silver atom in a solid oscillates in simple harmonic M
motion in some direction with a frequency of1012 per
C k
second. What is the force constant of the bonds
connecting one atom with the other? (Take, molecular
weight of silver = 108 and Avogadro number = 6.02 × 1023
g mol −1) ª JEE Main 2018 (a) 2 π (M /k) (b) 2 π (2M /k)
(a) 6.4 N/m (b) 7.1 N/m (c) 2.2 N/m (d) 5.5 N/m (c) 2 π (M / 2k) (d) 2 π (M / 3 k)
DAY ELEVEN OSCILLATIONS 123

20 A body performs SHM. Its kinetic energy K varies with 24 For a particle executing SHM, the displacement x is
time T as indicated in the graph given by x = A cos ω t . Identify the graph which
K K represents the variation of potential energy (PE) as a
function of time t and displacement x.
PE PE
(a) (b)
T t T t
I II III
IV
t
K K x
(a) I and III (b) II and IV
(c) II and III (d) I and IV
(c) (d) 25 A simple pendulum performs simple harmonic motion
T t T t
about x = 0 with an amplitude a, and time period T . The
speed of the pendulum at x = a / 2 will be
πa 3 πa 3 3 π2 a πa
21 A particle is executing simple harmonic motion with a (a) (b) (c) (d)
T 2T T T
time period T . At time t = 0, it is at its position of
equilibrium. The kinetic energy-time graph of the particle 26 The value of g decrease by 0.1% on a mountain as
will look, like ª JEE Main 2017 (Offline) compared to sea level. If a simple pendulum is used to
KE KE record the time, then the length must be
(a) increased by 0.1% (b) decreased by 0.1%
(c) increased by 0.2% (d) decreased by 0.2%
(a) (b)
O T t O T/2 T t 5T
27 Two pendulums have time periodsT and . They start
4
KE KE SHM at the same time from the mean position. What will
be the phase difference between them after the bigger
(c) (d) pendulum completes one oscillation?
O T/4 T/2 3T/4 T O T/2 T 2T
t t (a) 45° (b) 90° (c) 60° (d) 30°
28 A simple pendulum of length l is suspended from the roof
22 For a simple pendulum, a graph is plotted between its of a train which is moving in a horizontal direction with an
Kinetic Energy (KE) and Potential Energy (PE) against its acceleration a. Then, the time period T is given by
displacement d. Which one of the following represents
(a) 2 π l/ g (b) 2 π l/ (a 2 + g 2 )1/ 2
these correctly? (graphs are schematic and not drawn to
scale) ª JEE Main 2015 (c) 2 π l/ (a + g) (d) 2 π l/(g − a)
E E
PE 29 Two simple pendulums of length1 m and 4 m respectively
KE
(a) (b) are both given small displacement in the same direction.
PE KE The shorter pendulum has completed number of
d d
oscillations equal to ª JEE Main (Online) 2013
E (a) 2 (b) 7
E PE
KE (c) 5 (d) 3
(c) (d)
KE 30 A pendulum of length 2m lift at P. When it reaches Q, it
d losses 10% of its total energy due to air resistance. The
velocity of Q is
PE
P
23 The total energy of a particle, executing simple harmonic
motion is 2m
(a) ∝ x
(b) ∝ x 2
(c) independent of x Q
(d) ∝ x1/ 2 (a) 2 m/s (b) 1 m/s
where, x is the displacement from the mean position. (c) 6 m/s (d) 8 m/s
124 40 DAYS ~ JEE MAIN PHYSICS DAY ELEVEN

31 Four pendulums A,B,C and D are (c) Statement I is true; Statement II is false
hung from the same elastic support (d) Statement I is false; Statement II is true
as shown alongside. A and C are of B 34 If two springs S1 and S 2 of force constants k1 and k 2,
the same length while B is smaller C A
D respectively are stretched by the same force, it is found
than A and D is larger than A. A is that more work is done on spring S1 than on spring S 2.
given a displacement then in steady state
Statement I If stretched by the same amount, work done
(a) D will vibrate with maximum amplitude on S1, will be more than that on S 2.
(b) C will vibrate with maximum amplitude
(c) B will vibrate with maximum amplitude Statement II k1 < k2
(d) All the four will oscillate with equal amplitude 35 Statement I A particle performing SHM at certain instant
32 Bob of a simple pendulum of length l is made of iron. The is having velocity v. It again acquires a velocity v for the
pendulum is oscillating over a horizontal coil carrying first time after a time interval of T second, then the time
direct current. If the time period of the pendulum is T , period of oscillation isT second.
then ª JEE Main (Online) 2013 Statement II A particle performing SHM can have the
l same velocity at two instants in one cycle.
(a) T < 2 π and damping is smaller than in air alone
g 36 Statement I A particle performing SHM while crossing
l the mean position is having a minimum potential energy,
(b) T = 2 π and damping is larger than in air alone
g this minimum potential energy could be non-zero.
l Statement II In the equilibrium position, the net force
(c)T > 2 and damping is smaller than in air alone
g experienced by the particle is zero, hence potential
l energy would be zero at the mean position.
(d)T < 2 π and damping is larger than in air alone
g 37 Statement I A circular metal hoop is suspended on the
33 The amplitude of a damped oscillator decreases to 0.9 edge by a hook. The hoop can oscillate from one side to
times its original magnitude is 5s. In another 10 s it will the other in the plane of the hoop, or it can oscillate back
decreases to α times its original magnitude, where α and forth in a direction perpendicular to the plane of the
equals ª JEE Main 2013 hoop.
(a) 0.7 (b) 0.81 The time period of oscillation would be more when
oscillations are carried out in the plane of the hoop.
(c) 0.729 (d) 0.6
Statement II Time period of physical pendulum is more if
Direction (Q. Nos. 34-38) Each of these questions the moment of inertia of the rigid body about the
contains two statements : Statement I and Statement II. corresponding axis, passing through the pivoted point is
Each of these questions also has four alternative choices, only more.
one of which is the correct answer. You have to select one of
the codes (a), (b), (c) and (d ) given below. 38 Statement I The time period of a pendulum, in a satellite
(a) Statement I is true, Statement II is true; Statement II is
orbiting around the earth, is infinity.
the correct explanation for Statement I Statement II Time period of a pendulum is inversely
(b) Statement I is true, Statement II is true; Statement II is proportional to the square root of acceleration due to
not the correct explanation for Statement I gravity.
DAY ELEVEN OSCILLATIONS 125

DAY PRACTICE SESSION 2

PROGRESSIVE QUESTIONS EXERCISE


1 A 15 g ball is shot from a spring gun whose spring has a 7 If x, v and a denote the displacement, the velocity and
force constant of 600 Nm −1. The spring is compressed the acceleration of a particle executing simple harmonic
by 5 cm. The greatest possible horizontal range of the motion of time period T, then which of the following does
ball for this compression is (g = 10 ms −2). not change with time?
aT
(a) 10.0 m (b) 6.0 m (c) 12.0 m (d) 8.0 m (a) a 2T 2 + 4 π 2v 2 (b)
x
aT
2 Two simple harmonic motions are represented by the (c) aT + 2 πv (d)
 π v
equations y1 = 01
. sin 100π t +  and y 2 = 01
. cos π t .
 3 8 A simple pendulum has time period T1. The point of
The phase difference of the velocity of particle 1, with suspension is now moved upward according to the
respect to the velocity of particle 2 is (at t = 0) relation y = k t 2,(k = 1ms − 2 ), where y is the vertical
−π π −π π displacement. The time period now becomesT2. The ratio
(a) (b) (c) (d)
6 3 3 6 T2
of 12 is ( take, g = 10 ms − 2 )
3 A piece of wood has dimension a × b × c. It is floating in T2
a liquid of density ρ such that side a is vertical. It is now (a)
6
(b)
5
(c) 1 (d)
4
pushed down gently and released. The time period is 5 6 5
(a) 2 π ρa /g (b) 2 π abc /g 9 A pendulum made of a uniform wire of cross-sectional area
(c) 2 π g / ρa (d) 2 π bc /ρg A has time period T. When an additional mass M is added
4 The length of a spring is α when a force of 4N is applied to its bob, the time period changesTM . If the Young’s
on it. The length of a spring is β when a force of 5N is modulus of the material of the wire isY , then 1/Y is equal to
( g = gravitational acceleration) ª JEE Main 2015
applied on it. Then find the length of the spring when a
 T 2  A  T 2  Mg
(a)   M  − 1 (b)   M  − 1
force of 9 N is applied on the spring.
(a) 5 β − 4 α (b) β − α   T   Mg   T   A
(c) 5 α − 4 β (d) 9(β − α)
 2  T   A2
(c) 1 −  M  
T A
5 A simple pendulum of length l has a bob of mass m with (d) 1 −   
  T   Mg   TM   Mg
a charge q on it. A vertical sheet of charge having  
surface charge density σ passes through the point of 10 The bob of a simple pendulum is a spherical hollow ball
suspension. At equilibrium, the string makes an angle θ filled with water. A plugged hole near the bottom of the
with the vertical. If the tension in the string is T then, oscillating bob gets suddenly unplugged. During
σq σq
(a) tan θ = (b) tan θ = observation, till the water is coming out, the time period of
2 ε0 mg ε0 mg
oscillation would
l l
(c)T > 2 π (d)T = 2 π (a) first increase and then decrease to the original value
g g
(b) first decrease and then increase to the original value
6 A mass m is suspended from a (c) remain unchanged
massless pulley which itself is (d) increase towards a saturation value
suspended with the help of a 11 A pendulum of length l = 1 m is released from θ 0 = 60°.
massless extensible spring as The rate of change of speed of the bob at θ = 30° is
shown alongside. ( take, g = 10 m /s 2 ).
What will be the time period of
oscillation of the mass? The force 60°
constant of the spring is k.
(a) π m / k
(b) 2 π m / k
(c) 4 π m / k (a) 5 3 m/s 2 (b) 5 m/s 2
(d) 2 π m / 2 k (c) 10 m/s 2 (d) 2.5 m/s 2
126 40 DAYS ~ JEE MAIN PHYSICS DAY ELEVEN

12 A particle at the end of a spring executes simple 16 A particle moves with simple harmonic motion in a
harmonic motion with a period t1, while the corresponding straight line. In first τ sec, after starting from rest it travels
period for another spring is t 2. If the period of oscillation a distance a and in next τ sec,it travels 2a, in same
with the two springs in series is T , Then, direction, then ª JEE Main 2014
(a)T = t1 + t 2 (b)T 2 = t12 + t 22 (a) amplitude of motion is 3a
(c)T −1 = t1−1 + t 2−1 (d)T −2 = t1−2 + t 2−2 (b) time period of oscillations is 8τ
(c) amplitude of motion is 4a
13 A particle performs simple harmonic motion with (d) time period of oscillations is 6τ
amplitude A . Its speed is tripled at the instant that it is at
2 17 An ideal gas enclosed in a vertical cylindrical container
a distance A from equilibrium position. The new
3 supports a freely moving piston of mass M. The piston
amplitude of the motion is ª JEE Main 2016 (Offline) and the cylinder have equal cross sectional area A. When
A the piston is in equilibrium, the volume of the gas isV0
(a) 41 (b) 3A
3 and its pressure is P. The piston is slightly displaced from
7
(c) A 3 (d) A the equilibrium position and released. Assuming that the
3
system is completely, isolated from its surrounding, the
14 A wooden cube (density of wood d ) of side l floats in a piston executes a simple harmonic motion with frequency
liquid of density ρ with its upper and lower surfaces ª JEE Main 2013
horizontal. If the cube is pushed slightly down and 1 A γ P0 1 V0MP0
(a) (b)
released, it performs simple harmonic motion of periodT . 2 π V0M 2 π A2 γ
Then, T is equal to
1 A 2 γ P0 1 MV0
lρ ld (c) (d)
(a) 2 π (b) 2 π 2 π MV0 2 π A γ P0
( ρ − d) g ρg
lρ ld 18 If a simple pendulum has significant amplitude (up to a
(c) 2 π (d) 2 π
dg (ρ − d) g factor of 1/e of original) only in the period between t = 0 s
to t = τ s, then τ may be called the average life of the
15 Two particles are executing simple harmonic motion of pendulum. When the spherical bob of the pendulum
the same amplitude A and frequency ω along the x-axis. suffers a retardation (due to viscous drag) proportional to
Their mean position is separated by distance X 0( X 0 > A ). its velocity with b as the constant of proportionality, the
If the maximum separation between them is ( X 0 + A ), the average life time of the pendulum is (assuming damping
phase difference between their motion is is small) in seconds
π π 0.693
(a) (b) (a) (b) b
3 4 b
π π 1 2
(c) (d) (c) (d)
6 2 b b

ANSWERS
SESSION 1 1 (b) 2 (b) 3 (c) 4 (d) 5 (a) 6 (a) 7 (d) 8 (b) 9 (a) 10 (c)
11 (a) 12 (d) 13 (c) 14 (b) 15 (a) 16 (d) 17 (c) 18 (a) 19 (c) 20 (a)
21 (c) 22 (b) 23 (c) 24 (a) 25 (a) 26 (b) 27 (b) 28 (b) 29 (a) 30 (c)
31 (b) 32 (d) 33 (c) 34 (d) 35 (d) 36 (c) 37 (a) 38 (a)
SESSION 2
1 (a) 2 (a) 3 (a) 4 (a) 5 (a) 6 (a) 7 (b) 8 (a) 9 (a) 10 (a)
11 (b) 12 (b) 13 (d) 14 (b) 15 (a) 16 (d) 17 (c) 18 (d)
DAY ELEVEN OSCILLATIONS 127

Hints and Explanations


SESSION 1 5 Given, y = 0.005 cos (αx − β t ) 11 The displacement of A relative to B is
x = x1 − x2
1 Given, y = 3 cos  π − 2ω t  ...(i) Comparing the equation with the π
4  standard form, x = a cos ω t − b sin  ω t + 
dy  2
Velocity, v = y = A cos  −  2 π
x t
dt λ T  = a cos ω t − b cos ω t
π
= 3 × 2 ω sin  − 2ω t  we have, 2π / λ = α = (a − b ) cos ω t
4 
and 2π / T = β Which is a simple harmonic motion with
dv
Acceleration, a = ⇒ α = 2 π / 0.08 = 25.00 π amplitude (a − b ) .
dt
and β= π 12 Phase difference,
π
= − 4ω × 3 cos  − 2 ω t 
2
4  6 Maximum velocity v = Aω, 2π π
φ = ωt = × 1 = rad
= − 4 ω2 y (where, A is the amplitude and ω is the 6 3
angular frequency of oscillation).
As a ∝ y and negative sign shows that,
it is directed towards equilibrium (or ∴ 4.4 = ( 7 × 10−3 ) × 2 π / T P
P′
mean position), hence particle will A
7 × 10−3 2 × 22
execute SHM. or T = × = 0.01 s Smax p/6
Comparing Eq. (i) with equation 4.4 7
p/6
y = r cos(φ − ω ′t )
7 Given, x = x 0 cos  ω t − π 
We have, ω ′ = 2ω  4
2π π Q′ A Q
or = 2ω or T ′ = d2 x
T′ ω Acceleration, a = 2
dt
2 Given equation of motion is π
= − ω2 x 0 cos  ωt −  The maximum separation between the
y = sin 3ω t  4 two particles is
= (3sin ω t − sin 3ω t ) / 4 3π  π
= ω2 x 0 cos  ωt +  S max = 2 A sin
[Qsin 3θ = 3sin θ − 4sin3 θ]  4 6
1
dy  d 3π or S max = 2 × 5 × = 5cm
(3sin ω t ) − (sin 3ω t ) / 4
d
⇒ = So, A = ω2 x 0 and δ = 2
dt  dt dt  4
dy 13 Given, x = acos ωt …(i)
⇒4 = 3ω cos ω t − [3ω cos 3ω t ] 8 Using v 2 = ω2 (a2 − y 2 ), we have
dt 102 = ω2 (a2 − 42 ) Y = asinωt …(ii)
d2 y Squaring and adding Eqs.(i) and (ii), we
⇒4× = −3ω sin ω t + 9ω sin 3ω t
2 2
and 8 = ω (a − 5 )
2 2 2 2

d t2 get
So, 102 − 82 = ω2 (52 − 42 ) = (3ω )2
d2 y  3ω2 sin ω t − 9ω2 sin 3ω t  x2 + y 2 = a2 (cos 2ωt + sin2 ωt )
⇒ = −  ⇒ 6 = 3ω or ω = 2
dt 2  4  ∴ T = 2 π /ω = a2
= 2π /2 = π s [Q cos 2 ωt + sin 2ωt = 1]
d2 y
⇒ is not proportional to y. 9 Force of friction = µ m g = m ω a 2 This is the equation of a circle.
dt 2
= m (2 πν) a 2 Clearly, the locus is a circle of constant
Hence, motion is not SHM. radius a.
1 µg
As the expression is involving sine ⇒ ν=
function, hence it will be periodic. 2π a 14 For a harmonic oscillator,
m
3 The motion of an oscillating liquid 10 Let O be the mean position and x be the T = 2π
k
column in a U-tube is simple harmonic distance of the coin from O. The coin
1
and the time period is independent of will slip, if centrifugal force on the coin where, k = force constant and T =
the density of the liquid. just becomes equal to the force of ν
h friction i.e. ∴ k = 4 π2 ν2 m
T =2 π
m x ω2 = µ m g 108 × 10−3
2
= 4 ×   × (1012 )2 ×
g 22
where, h = height of liquid in each R  7 6.02 × 1023
column. ⇒ k = 71
. N/m
O F Fc
4 For motion to be SHM acceleration of
the particle must be proportional to
Mean position 15 For spring, k ∝ 1
mg l
negative of displacement. x kA lB l + lB 5
i.e. a ∝ − ( y or x ) ∴ = ⇒ kA = A k = k
From the diagram, kB lA lA 2
We should be clear that y has to be
linear. m Aω2 = µ mg or A = µg / ω2
128 40 DAYS ~ JEE MAIN PHYSICS DAY ELEVEN

k1 + k2 24 Potential energy is minimum (in this


16 We know that, ν = 1 case zero) at the mean position ( x = 0)
33 Amplitude of damped oscillator,
2π m −
bt
and maximum at the extreme positions A = A 0e 2m
When k1 and k2 are made four times ( x = ± A ). b( 5)

their original value. At time t = 0, x = A, the potential energy After 5 s, 0.9 A 0 = A 0 e 2m

Then, and should be maximum. Therefore, graph I b ( 5)


is correct. Further in graph III, potential −
1 k + k2 energy is minimum at x = 0. Hence, this ⇒ 0.9 = e 2m
…(i)
ν′ = ⋅2 1 = 2ν
2π m is also correct. After 10 more second,
3
17 The effective spring constant is 25 Since, v = ω a2 − y 2 , −b
(15)  − 5b 
A = A 0e 2m = A0e 2m  …(ii)
K = k + 2k = 3 k . At, x or y = a / 2  
 
The time period of oscillation is given by
1 a2 3 a2 From Eqs. (i) and (ii), we get
T = 2π
m
and ν = ⇒ v = ω a2 − =ω
3k T 4 4 A = 0729
. A0
2π 3a π 3a α = 0729
so, we get = × = Hence, .
1 3k T 2 T
ν= 34 As no relation between k1 and k2 is given
2π m 26 As, T = 2π l / g in the question, that is why, nothing can
be predicted about Statement I. But as in
18 Angular frequency of the system, Taking log and differentiating the Statement II, k1 < k2
k k expression, keeping T constant we have
ω= = dl dg 01
. Then, for same force
m+ m 2m = =− F F2
l g 100 W = F⋅x= F⋅ =
Maximum acceleration of the system K K
∴ (d l / l ) × 100 = − 01
. / 100 × 100 1
kA ⇒ W ∝
will be, ω2 A or . This acceleration of = − 01
.%
2m k
the lower block, is provided by friction. 27 When bigger pendulum of time period i.e. W1 > W2
Hence, f max = ma max = m ω2 A (5T /4) completes one oscillation, the But for same displacement,
smaller pendulum will complete (5/ 4) 1 1
W = F ⋅ x = k x ⋅ x = k x2
= m 
k A k A oscillation. It means, the smaller
 = 2 2
 2m  2 pendulum will be leading the bigger
⇒ W ∝ k , i.e. W1 < W2
pendulum by a phase of T /4 = π /2 rad
19 When the mass m is pushed in a = 90°. Thus, in the light of Statement II,
downward direction through a distance Statement I is false.
x, the effective restoring force, in 28 Effective acceleration = a2 + g 2
magnitude is 35 Consider the situation as shown in the
l adjoint figure. Let us say at any instant t 1 ,
F = k x + k x cos 60° + k x cos 60° ∴Time period, T = 2 π
(a2 + g 2 )1 /2 the particle crosses A as shown, the
= 2k x particle again acquires the same velocity,
∴ Spring factor, k ′ = 2k 29 Let T1 and T2 be the time period of when it crosses B let us say at instant t 2 .
and Inertia factor = M shorter length and larger length According to statement I, (t 2 − t 1 ) is the
M pendulums respectively. According to time period of SHM which is wrong.
So time period, T = 2 π
2k question,
x1 x1
nT1 = (n − 1) T2
20 The frequency of kinetic energy is twice B Equilibrium position A
that of a particle executive SHM. 1 4
So, n 2 π = (n − 1) 2 π
8 8 36 At the mean position,
21 KE is maximum at mean position and
or n = (n − 1) 2 = 2n − 2 ⇒ n = 2 dU
minimum at extreme position F = 0= − =0
 at t = T  . dx
  30 By applying conservation of energy ⇒ U = constant which can be zero or
 4
between P and Q non-zero.
22 During oscillation, motion of a simple 1
mv 2 = 0.9(mgh )
pendulum KE is maximum at the mean 2 37 When the hoop oscillates in its plane,
position where PE is minimum. At ⇒ v 2 = 2 × 0.9 × 10 × 2 = 36 ⇒ v = 6 m/s moment of inertia is
extreme position, KE is minimum and I1 = mR2 + mR2 i.e. I1 = 2mR2
31 As A and C are of same length, so they
PE is maximum. Thus, correct graph is While when the hoop oscillates in a
will be in resonance, hence C will
depicted in option (b). vibrate with the maximum amplitude. direction perpendicular to the plane of
23 In a simple harmonic motion, when a l the hoop, moment of inertia is
particle is displaced to a position from 32 T < 2 π mR2 3 mR2
g I2 = + mR2 =
its mean position, its kinetic energy is 2 2
converted into potential energy. Hence, As, current passed through in the coil Time period of physical pendulum is,
total energy of a particle remains which attracts the molecules of air I
constant or the total energy in simple closer to it, thus density of air increases T = 2π ; d is same in both the
mgd
harmonic motion does not depend on which produces larger damping than
the displacement x. that in air alone. cases.
DAY ELEVEN OSCILLATIONS 129

38 From the relation of the time period, 4 4 = k (α − l ) l


and T2 = 2 π
l 1 5 = k (β − l ) g + ay
T = 2π ⇒ T ∝
4 α −l g + ay
g g 9 = k (γ − l ) ⇒ = T12
5 β−l ∴ =
When the satellite is orbiting around the T22 g
earth, the value of g inside it is zero. or 4β − 4 l = 5α − 5l
10 + 2 6
Hence, the time period of pendulum in l = 5α − 4β = =
10 5
a satellite will be infinity and it is also Now, 9α − 9l = 4γ − 4l
clear that time period of pendulum is 4 γ = 9α − 5l = 9α − 5(5α − 4β ) 9 We know that time period,
inversely proportional to square root of = 9α − 25α + 20β T = 2π
L
acceleration due to gravity g. = 20β − 16α g
γ = 5β − 4α
SESSION 2 When additional mass M is added to its
5 In the figure, we represent the electric bob
1 For getting horizontal range, there must intensity at B due to the sheet of charge,
L + ∆L
be some inclination of spring with
O T M = 2π ,
ground to project ball. g

q T where, ∆L is increase in length.


l We know that Young modulus of the
u
material
B
45° C F Mg / A MgL
Y = =
Rmax ∆L / L A∆L
mg
MgL
2 Sheet of charge ⇒ ∆L =
u AY
R max = 1σ
g E = MgL
2 ε0 L+
But KE acquired by ball AY
Force on bob due to the sheet of charge, T M = 2π
= PE of spring gun g
1 σq
1 1 kx2 F = qE = 2
⇒ mu = kx2 ⇒ u2 =
2
2 ε0  T M  = 1 + Mg
2 2 m ⇒  
As the bob is in equilibrium,  T  AY
kx2 600 × (5 × 10−2 )2
⇒ R max = = mg F T 2
mg 15 × 10−3 × 10 so = = Mg  T M 
OC CB BO or =  −1
AY  T 
= 10 m CB F σq
or = = = tanθ
π mg 2ε0 m g A  T M  
. sin  100 π t + 
OC 2
2 Given, y 1 = 01 1
 or =   − 1
3 Y Mg   T 
6 If mass m moves down a distance y, 
dy 1
⇒ = v 1 = 01
. × 100 π cos then the spring is pulled by 2y and the
dt force with which the spring is pulled 10
 100 π t + π  will be F = R = mg / 2.
 
 3 Hence, mg / 2 = k ( 2 y )
π π
or v 1 = 10 π sin  100 π t + +  ⇒ y / g = m / 4k l l + Dl
 3 2 ⇒ T = 2π y /g G
5π 
or v 1 = 10 π sin  100 π t + = 2 π m /4 k = π m / k G
 G¢
 6
7 As, aT = ω xT
2
Spherical hollow ball Spherical hollow ball
and y 2 = 0.1 cos π t
dy 2 x x filled with water half filled with water
⇒ = v 2 = − 01 . sin π t
dt 4 π2 4 π2
= ×T = l l + ∆l
or v 2 = 01 . sin ( π t + π ) T 2
T T = 2π T1 = 2 π
Hence, the phase difference g g
= constant.
∆φ = φ 1 − φ 2
d2 y
5π  8 Given, y = k t 2 ⇒ a = = 2k
=  100 π t +  − ( πt + π ) dt 2
 6
l
5π π a
= − π=− (at t = 0)
6 6
3 Force of buoyancy = b × c × ρw × g G

= bc g (Q ρw = 1) g axt
Spherical hollow ball
and mass of piece of wood = ab c ρ
or a y = 2 m/s 2
(as, k = 1 m/s ) 2
T2 = 2 π
l
and T1 > T2
So, acceleration
= − bc g / ab c ρ = − (g /aρ ) g
l
ρa ∴ T1 = 2 π Hence, time period first increases and
Hence, time period, T = 2π g
g then decreases to the original value.
130 40 DAYS ~ JEE MAIN PHYSICS DAY ELEVEN

11 Initial velocity of a particle performs i.e. x = A cos ωt , at t = 0, x = A


SHM, When t = τ, then x = A − a …(i)
 2A  
2
30°

v 2 = ω2  A2 − 
T When t = 2τ, then x = A − 3a
60 °   ... (i) …(ii)
  3  
1m  On comparing Eqs. (i) and (ii), we get
where, A is initial amplitude and ω is
A − a = A cos ωτ
angular frequency.
A − 3 a = A cos 2ωτ
m

Final velocity,
g
co

 2A  
2
As cos 2ωτ = 2cos 2ωτ − 1
(3v )2 = ω2  A ′2 − 
s3

mg sin 30° mg   ...(ii)


  3   A − 3 a 2 A2 + 2a2 − 4 Aa − A2
 =
dv From Eqs. (i) and (ii), we get A A2
Rate of change of speed
dt 4 A2
A2 − A2 − 3 aA = A2 + 2a2 − 4 Aa
= tangential acceleration 1 9 7A
tangential force mg sin30° = ⇒ A′ =
= = 9 4 A2 3 a2 = 2aA, A = 2a
mass m A′ −2
9 Now, A − a = A cos ωτ ⇒ cos ωτ = 1 / 2
 1
= g sin30° = 10   m/s = 5m/s2
2
 2 14 Let at any instant, cube is at a depth x 2π π
⇒ τ = ⇒ T = 6τ
from the equilibrium position, then net T 3
12 Time period of the spring, force acting on the cube = upthrust on Mg
the portion of length x 17 = P0 ⇒ Mg = P0 A
T = 2π  
m A
k ∴ F = − p l2 xg = − p l2g x ...(i)
P0V 0γ = (P0 + ∆V 0 ) (V 0 − ∆V 0 )γ
Negative sign shows that, force is γ
Here, k be the force constant of spring.  ∆V 0 
For the first spring, opposite to x. ⇒ P0 = ( P0 + ∆ P0 )  1 − 
 V0 
 m Hence, equation of SHM
t1 = 2 π   …(i)  ∆V 0 
 k1  = ( P0 + ∆ P0 )  1 − r 
 V0 
For the second spring,
 ∆V 0 
 m =  P0 − VP0 + ∆ P0 
t2 = 2 π   …(ii)  V0 
 k2  I X
∆V 0
or ∆P0 = VP0
The effective force constant in the series V0
I
combination is But ∆V = Ax,
k1 k2 where, A = area at cross section of piston
k =
k1 + k2 γ P0 A
F = −k x …(ii) ∴ ∆P0 = x
Time period of combination V0
Comparing Eqs.(i) and (ii), we get
 m (k1 + k2 ) γP0 A2
T = 2π   k = ρl2g Restoring force F = − ∆P0 × A = − x
 k1 k2  V0
m l 3d ld
4 π2 m (k1 + k2 ) ∴ T = 2π = 2π = 2π Comparing it with, F res = − kx
⇒ T2 = …(iii) k ρl 2 g ρg
k1 k2 γ P0 A 2
k =
From Eqs. (i) and (ii), we get 15 Let x1 = A sin (ω t + φ1 ) V0
m m and x2 = A sin(ω t + φ2 ) 1 k 1 γ P0 A 2
t 12 + t 22 = 4 π2  +  ∴ f = =
 k1 k2  x2 − x1 = A 2π M 2π MV 0
 1 [sin(ω t + φ2 ) − sin(ωt + φ1 )]
1
or t 12 + t 22 = 4 π2 m  +  2ω t + φ1 + φ2   φ2 − φ1  18 For damped harmonic motion,
 k1 k2  = 2 A cos   sin  
 2   2  m a = − kx − mb v
4 π2 m (k1 + k2 ) or m a + mb v + k x = 0
or t 12 + t 22 = The resultant motion can be treated as a
k1 k2 Solution to above equation is
simple harmonic motion with bt
φ − φ1  − b2
⇒ t 12 + t 22 = T 2 amplitude 2 A sin  2
[from Eq. (iii)] k
 x = A 0 e 2 sin ω t; with ω2 = −
 2  m 4m
13 The velocity of a particle executing
SHM at any instant, is defined as the Given, maximum distance between the where, amplitude drops exponentially

time rate of change of its displacement particles = X 0 + A with time. −
A τ = A0 e 2
at that instant. ∴ Amplitude of resultant SHM
= X0 + A − X0 = A Average time τ is that duration when
v = ω A2 − x2
 φ2 − φ1  amplitude drops by 63%, i.e. becomes
where, ω is angular frequency, A is ∴ 2 A sin   = A ⇒
 2  A 0 /e bτ
amplitude and x is displacement of a A −
particle. φ2 − φ1 = π /3 Thus, A τ = 0 A 0e 2
e
Suppose that the new amplitude of the
16 In SHM, a particle starts from rest, we bτ 2
motion be A′. or = 1 or τ =
have 2 b
DAY TWELVE

Waves
Learning & Revision for the Day
u Wave Motion u Principle of u Standing or Stationary
u Speed of Waves Superposition of Waves Waves
u Sound Waves u Reflection and Transmission u Beats
of Waves u Doppler’s Effect
u Displacement Relation for a
Progressive or Harmonic Wave

Wave Motion
Wave motion involves transfer of disturbance (energy) from one point to the other with
particles of medium oscillating about their mean positions i.e. the particles of the
medium do not travel themselves along with the wave. Instead, they oscillate back and
forth about the same equilibrium position as the wave passes by. Only the disturbance is
propagated.
1. Longitudinal Waves When particles of the medium vibrate parallel to the direction
of propagation of wave, then wave is called longitudinal wave. These waves
propagate in the form of compressions and rarefactions. They involve changes in
pressure and volume. The medium of propagation must possess elasticity of volume.
They are set up in solids, liquids and gases.
2. Transverse Waves When the particles of the medium vibrate in a direction
perpendicular to the direction of propagation of wave, then wave is called transverse
waves. These wave propagtes in the form of crests and troughs. These waves can be
set up in solids, on surface of liquids but never in gases.

Terms Used in Wave Motion


l
Angular Wave Number Number of wavelength in the distance 2π is called the wave
PREP
number or propagation constant. MIRROR
2π Your Personal Preparation Indicator
K = rad/m
λ u No. of Questions in Exercises (x)—
l
Particle velocity It is the velocity of the particle executing simple harmonic motion.
dy u No. of Questions Attempted (y)—
i.e. v= u No. of Correct Questions (z)—
dt
(Without referring Explanations)
where, y denotes displacement at any instant.
l
Wave Velocity The velocity of transverse wave motion is given by u Accuracy Level (z / y × 100)—
Distance travelled by wave u Prep Level (z / x × 100)—
v=
Time taken In order to expect good rank in JEE,
your Accuracy Level should be above
λ  1 ω
i.e. v= =  λ= or v = νλ 85 & Prep Level should be above 75.
T T  x
132 40 DAYS ~ JEE MAIN PHYSICS DAY TWELVE

Differential Equation of Wave Motion


Factors Affecting Speed of Sound
l

d2 y 1 d2 y
= l
Effect of Temperature on Velocity With rise in
dx2 v2 dt 2 temperature, the velocity of sound increases as
γ RT v T
v= ; i.e. v ∝ T ; 2 = 2
Speed of Waves M v1 T1
Speed of waves is divided in two types as per the nature of Speed of sound in air increases by 0.61 m/s for every 1°C
wave, these are given below rise in temperature.
l
Effect of Pressure for Gase Medium Pressure has no effect
1. Speed of Transverse Wave on the velocity of sound, provided temperature remains
constant.
The expression for speed of transverse waves in a solid and in
case of a stretched string can be obtained theoretically
l
Effect of Humidity When humidity in air increases, its
density decreases and so velocity of sound increases.
η
l
In solids, v = Y B
d For solids, v = . For liquids, v =
D D
where, η is the modulus of rigidity and d is the density of
the medium. where, Y = Young’s modulus of elasticity
B = bulk modulus of elasticity.
T Mg
l
In a stretched string, v = =
m πr 2 d
where, T = the tension in the string, Sound Waves
m = the mass per unit length of the string, The longitudinal waves which can be heard are called sound
M = mass suspended from the string, waves.
r = radius of the string and They are classified into following categories
d = density of the material of the string. l
Infrasonics The longitudinal waves having frequencies
below 20 Hz are called infrasonics. These waves cannot be
heard. These waves can be heard by snakes.
2. Speed of Longitudinal Wave l
Audible waves The longitudinal waves having the
(or Sound Wave) frequency between 20 Hz and 20000 Hz are called audible
Following are the expressions for the speed of longitudinal waves. Human can hear these waves.
waves in the different types of media l
Ultrasonics The longitudinal waves having the
l
If the medium is solid, frequencies above 20000 Hz are called ultrasonics. These
4 waves are also called supersonic waves or supersonics.
B+ η
v= 3
ρ
Displacement Relation for a
where B, η and ρ are values of bulk modulus, modulus of
rigidity and density of the solid respectively. Progressive or Harmonic Wave
If the solid is in the form of a long rod, then The equation of a plane progressive or simple harmonic wave
travelling along positive direction of x-axis is
Y
v= 2π  T
ρ y = a sin (ωt − kx) ⇒ y = a sin t − x  .
T  λ
where, Y is the Young’s modulus of the solid material.
2π  x
l
In a liquid, ⇒ y = a sin (vt − x) ⇒ y = a sin ω t − 
λ  v
B
v= t x
ρ ⇒ y = a sin 2π  −  .
T λ
where B is the bulk modulus of the liquid.
l
If maximum value of y = a, i.e. a is amplitude, then
l
According to Newton’s formula, speed of sound in a gas
is obtained by replacing B with initial pressure p of the gas dy / dt = velocity of particle
i.e. B = p. dy 2π
v= = aω cos ⋅ (vt − x)
p dt λ
v=
ρ  dy  2πva
  = = 2πna = ωa [where, n = frequency]
 dt  max λ
DAY TWELVE WAVES 133

l
Acceleration of particle Constructive and Destructive Interference
d2 y 2π
= − ω2 a sin (vt − x) l
When the wave meet a point with some phase,
dt 2
λ constructuve interference is obtained at that point.
l
For a wave, velocity of sound wave (i) Phase difference between the waves at the point of
v = frequency (n) × wavelength (λ ) observation φ = 0 ° or 2πn.
⇒ v = nλ (ii) Resultant amplitude at the point of observation will
2π 2 πv be maximum, Amax = A1 + A2 .
l
Angular speed, ω = 2 πn = ⇒ω =
T λ l
When the waves meet a point with opposite phase,
destructive interference is obtained at that point.
(i) Phase difference between the waves at the point of
Relation between Phase Difference, observation φ = 180 ° or (2 n − 1)π.
Path Difference and Time Difference (ii) Resultant amplitude at the point of observation will
2π be minimum, Amin = A1 − A2 .
l
Phase difference (φ) = × path difference ( x)
λ Intensity

2πx φλ The intensity of waves is the average amount of energy
φ= ⇒x= transported by the wave per unit area per second normally
λ 2π across a surface at the given point.

l
Phase difference (φ) = × time difference (t ) Intensity (I 1) ∝ (Amplitude A)2
T
2
2πt Tφ I 1  A1 
⇒ φ= ⇒t = ∴ = 
T 2π I2  A2 
T
l
Time difference (t ) = × path difference ( x) If I 1 and I2 are intensities of the interfering waves and φ is
λ the phase difference, then resultant intensity is given by
Tx λt
⇒ t = ⇒x= I = I 1 + I2 + 2 I 1I2 cos φ
λ T
I max = I 1 + I2 + 2 I 1I2 = ( I 1 + I2 )2 , for φ = 2 πn
and I min = I 1 + I2 − 2 I 1I2
Principle of Superposition
I min = ( I 1 − I2 )2, for φ = (2 n + 1) π
of Waves
Two or more waves can traverse the same space Power
independently of one another. The resultant displacement of
each particle of the medium at any instant is equal to the If P is power of a sound source, then intensity (I ) follows
vector sum of displacements produced by the two waves inverse square law of distance (d).
separately. This principle is called principle of superposition P
I =
of waves. 4πd2
y = y1 + y2 + y3 + .....
Reflection and Transmission
of Waves
Interference of Waves When sound waves are incident on a boundary separating two
When two waves of same frequency (or A2 A media, a part of it is reflected back into the initial medium while
same wavelength) travelling along same the remaining is partly absorbed and partly transmitted into the
path superimpose each other, there second medium.
occurs redistribution of energy in the φ
medium. At a given position (x being Standing or Stationary Waves
constant) displacement due to two θ
A1 Standing or stationary wave is formed due to superposition of
waves be two progressive waves of same nature, same frequency (or
y1 = A1 sin ωt and y2 = A2 sin (ωt + φ) same wavelength), same amplitude travelling with same
Then, resultant displacement speed in a bounded medium in mutually opposite directions.
y = y1 + y2 = A sin (ωt + φ) If the incident wave be represented as y1 = A sin(ω t − kx)
and the reflected wave as y2 = A sin (ω t + kx),
where, A= A21 + A22 + 2 A1 A2 cos φ
then y = y1 + y2 = A sin(ω t − kx) + A sin(ω t + kx)
A2 sin φ ⇒ y = 2 A cos kx sin ωt
and tan θ =
A1 + A2 cos φ The resultant wave does not represent a progressive wave.
134 40 DAYS ~ JEE MAIN PHYSICS DAY TWELVE

λ1 v
Standing Waves in String l
First harmonic l =
2
⇒ f1 =
2l
Consider a string of length L stretched under tension T
between two fixed points (i.e. clamped at its ends). Transverse A1 A2
wave is set up on the string whose speed is given by v = T /µ, N
λ
where µ is the mass per unit length of the string. l= 1
2
Different modes of vibration of stretched string are discussed (a)
below
2v
l
Let only one anti-node A is A l
Second harmonic or first overtone l = λ 2 ; f =
2l
formed at the centre and
string vibrates in one A A1 A2 A3
Fundamental or first harmonic N1 N2
segment only, it is called
fundamental mode, then l= λ2
λ (b)
L = 1 or λ 1 = 2 L
2 3λ3 3v
l
Third harmonic or second overtone l = ;f =
Frequency of vibration in fundamental mode 2 2l
v 1 T
ν1 = =
λ 1 2L µ A1 A2 A3 A4
N1 N2 N3
It is known as the fundamental frequency or first 3λ3
harmonic. l=
2
A A (c)
l
If string vibrates in two
segments, then N
A A l
All harmonics are present in open pipe with their
L = λ2 Second harmonic
frequencies in the ratio 1 : 2 : 3 : 4 . . .. and
v 1 T ratio of overtones = 2 : 3 : 4 : 5 K
and ν2 = = = 2ν 1 λ 3λ 5λ
λ2 L µ Position of nodes from one end x = , , K
It is known as first overtone or second harmonic. 4 4 4
Position of anti-nodes from one end
l
If the string vibrates in three segments, λ 3λ
x = 0, , λ , K
3λ A A A 2 2
then L = 3
2 N N 2. Closed Organ Pipe
A A A
v
and ν3 = = 3ν 1 Third harmonic In a chosed organ pipe, always node is formed at the closed
λ3 end. Various mode of vibration of air column in a closed
It is called second overtone or third harmonic. organ pipe are shown below
l
In general, if a string vibrates in p segments [i.e. have
l
First harmonic
A N
( p + 1) nodes and p antinodes], λ
l= 1
4 λ1
p T v l=
then ν pth = = pν 1 f = 4
2L µ 4l (a)
and it is known as pth harmonic or ( p − 1)th overtone. l
Third harmonic
(first overtone) A1 A2 N2
Standing Waves in Organ Pipes 3λ N1
l= 3
4 3λ2
l=
(Air Columns) 3v 4
f3 = (b)
Organ pipes are those cylindrical pipes which are used for 4l
producing musical (longitudinal) sounds. The standing waves l
Fifth harmonic
in both organ pipes (i.e. open organ pipe and closed organ (second overtone) A1 A2 A3 N3
pipe) are described below. 5λ 5 N1 N2
l= ;
4 5λ3
1. Open Organ Pipe 5v
l=
4
f5 = (c)
In an open organ pipe, always anti-node is formed at both 4l
open ends. Various modes of vibration of air column in an l
In closed organ pipe only odd harmonics are present. Ratio
open organ pipe are shown below of harmonic is n1 : n3 : n5 = 1 : 3 : 5.
DAY TWELVE WAVES 135

Ratio of overtones = 3 : 5 : 7
Doppler’s Effect
l

λ 3λ
l
Position of nodes from closed end x = 0, , λ, , ... The phenomena of apparent change in frequency of source
2 2
due to a relative motion between the source and observer is
λ 3λ 5λ
l
Position of antinodes from closed end x = , , , ... called Doppler’s effect.
4 4 4 l
When Source is Moving and Observer is at Rest When
source is moving with velocity vs , towards an observer at
rest, then apparent frequency
Beats  v  vs
When two sound waves of nearly equal (but never equal) or n′ = n  
 v − vs  S v O
slightly different frequencies and equal or nearly equal
amplitudes travelling along the same direction superimpose If source is moving away from observer, then
at a given point, the resultant sound intensity alternately rises  v 
n′ = n  
and falls. This alternate rise and fall of sound at a given  v + vs 
position is called beats. l
When Source is at Rest and Observer is Moving When
l
Number of beats formed per second is called the frequency observer is moving with velocity vo , towards a source at
of beats. If two sound waves of frequencies ν 1 and ν2 rest, then apparent frequency.
superimpose, then frequency of beats = (ν 1 ~ ν2 ), i.e. either  v + vo  vo
(ν 1 − ν2 ) or (ν2 − ν 1). n′ = n  
 v  S O
v
l
For formation of distinct beats, the difference between the When observer is moving away from source, then
frequencies of two superimposing notes should be less than
 v − vo  vo
10 Hz. n′ = n  
 v  S O
l
Our perception of loudness is better co-related with the v
second level measured in decibel (dB) and defined as l
When Source and Observer Both are Moving
follows (a) When both are moving in same direction along the
 I direction of propagation of sound, then
β = 10 log 10   , where I 0 = 10 −12 Wm2 at 1kHz.
 I0  v − vo 
n′ = n  
 v − vs 
Tuning Fork S
v
O
The tuning fork is a metallic device that produces sound of a vs vo
single frequency.
Suppose, a tuning fork of known frequency nA is sounded (b) When both are moving in same direction opposite to
together with another tuning fork of unknown frequency (nB ) the direction of propagation of sound, then
and x beats heard per second.  v + vo 
There are two possibilities to know frequency of unknown n′ = n  
 v + vs 
tuning fork
nA − nB = x …(i) vs vo
nB − nA = x …(ii) S
v
O
We can find true frequency of tuning fork B from a pair of
tuning forks A and B, in which frequency of A is known and (c) When both are moving towards each other, then
where x is the beats per second.  v + vo 
n′ = n  
When B is loaded When B is filled  v − vs 
(its frequency decreases) (its frequency increases) v
S O
(i) If x increases, then (i) If x increases, then vs vo
nB = nA − x nB = nA + x
(d) When both are moving in opposite direction, away
(ii) If x decreases, (ii) If x decreases, then from each other, then
then nB = nA + x nB = nA − x
 v − vo 
(iii) If x remains same, (iii) If x remains same, then n′ = n  
then nB = nA + x nB = nA − x  v + vs 
v
(iv) If x becomes zero, (iv) If x becomes zero, then vo
vs
then nB = nA + x nB = nA − x O S
136 40 DAYS ~ JEE MAIN PHYSICS DAY TWELVE

Similarly, if the source is moving away from the observer as


Transverse Doppler’s Effect shown above, with velocity component vs cos θ, then
l
The Doppler’s effect in sound does not take place in the v
ν′ = ×ν
transverse direction. v + vs cos θ
l
As shown in figure, the position of a source is S and of l
If θ = 90 °, the vs cos θ = 0 and there is no shift in the
observer is O. The component of velocity of source towards frequency. Thus, at point P, Doppler’s effect does not occur.
the observer is v cos θ. For this situation, the approach
frequency is Effect of Wind
θ
If wind is also blowing with a velocity w in the direction of
sound, then its velocity is added to the velocity of sound.
os
vc

v P
Hence, in this condition the apparent frequency is given by
S v  v + w − vo 
S
θ θ ν′ = ν  
 v + w − vs 
vc

T
Applications of Doppler’s Effect
os
θ

The measurement of Doppler shift has been used


O
l
by police to check over speeding of vehicles.
l
at airports to guide the aircraft.
v
ν′ = ×ν l
to study heart beats and blood flow in different parts of the
v − vs cos θ
body.
ν′ will now be a function of θ. So, it will no more be l
by astrophysicist to measure the velocities of plants and
constant. stars.

DAY PRACTICE SESSION 1

FOUNDATION QUESTIONS EXERCISE


1 Which of the following statements are true for wave 4 When tension of a string is increased by 2.5 N, the initial
motion? frequency is altered in the ratio of 3 : 2. The initial tension
(a) Mechanical transverse waves can propagate through in the string is
all mediums (a) 6 N (b) 5 N (c) 4 N (d) 2 N
(b) Longitudinal waves can propagate through solids only 5 The speed of sound in oxygen (O2 ) at a certain
(c) Mechanical transverse waves can propagate through solids temperature is 460 ms −1. The speed of sound in helium
only (He) at the same temperature will be (assume both
(d) Longitudinal waves can propagate through vacuum gases to be ideal)
2 A sound wave is passing through air column in the form (a) 460 ms−1 (b) 500 ms−1 (c) 650 ms−1 (d) 1420 ms−1
of compression and rarefaction. In consecutive 6 It takes 2.0 seconds for a sound wave to travel between
compression and rarefactions, two fixed points, when the day temperature is 10°C. If
(a) density remains constant the temperature rise to 30°C, the sound wave travel
(b) Boyle’s law is obeyed between the same fixed points in
(c) bulk modulus of air oscillates (a) 1.9 s (b) 2.0 s
(d) there is no transfer of heat (c) 2.1 s (d) 2.2 s
 x 1 
3 A sound wave of wavelength λ is travelling in a medium 7 A wave equation is given by y = 4 sin π 
t
− + 
with a speed of v m/s enters into another medium where   5 9 6 
its speed is 2v m/s. Wavelength of sound waves in the where, x is in cm and t is in sec. Which of the following is
second medium is true?
λ (a) λ = 18 cm (b) v = 4 ms −1
(a) λ (b)
2 (c) a = 0.4 m (d) f = 50 Hz
(c) 2λ (d) 4λ
DAY TWELVE WAVES 137

8 A wave equation which gives the displacement along of it is in water. The fundamental frequency of the air
y-direction is given by y = 0.001 sin [100 t + x ], where, column is now ª JEE Main 2016 (Offline)
x and y are in metre and t is time in second. This f 3f
(a) (b) (c) 2f (d) f
represents a wave 2 4
100
(a) of frequency Hz 15 Third overtone of a closed organ pipe is in unison with
π fourth harmonic of an open organ pipe. The ratio of the
(b) of wavelength 1 m
50 lengths of the pipes are
(c) travelling with a velocity of ms −1 in the positive 7 3 5 8
π (a) (b) (c) (d)
x-direction 8 4 7 7

(d) travelling with a velocity of 100 ms −1 in the negative 16 Motion of two particles is given by
x-direction y1 = 0.25 sin ( 310 t ), y 2 = 0.25 sin ( 316 t )
9 Which of the following is not true for progressive wave Find beat frequency.
 t x  (a) 3 (b) 3/π c) 6/π (d) 6
y = 4 sin 2π  −
 0.02 100  17 Three sound waves of equal amplitudes have frequencies
where, y and x are in cm and t in second. ( ν − 1), ν,( ν + 1). They superimpose to give beat. The
(a) Its amplitude is 4 cm number of beats produced per second will be
(b) Its wavelength is 100 cm (a) 4 (b) 3 (c) 2 (d) 1
(c) Its frequency is 50 Hz
18 Two tuning forks P and Q when set vibrating, give
(d) Its propagation speed is 50 × 10−2 cms −1
4 beat/s. If a prong of the fork P is filled, the beats are
10 The equation of a wave on a string of linear mass density reduced to 2 s −1. What is the frequency of P, if Q is
0.04 kg m −1 is given by 250 Hz?
(a) 246 Hz (b) 250 Hz (c) 254 Hz (d) 252 Hz
  t x 
y = 0.02 (m ) sin 2π  −  .
  0.04 (s ) 0.50 (m ) 19 16 tuning forks are arranged in the order of increasing
frequencies. Any two successive forks give 8 beat/s,
The tension in the string is
(a) 4.0 N (b) 12.5 N (c) 0.5 N (d) 6.25 N when sounded together. If the frequency of the last fork is
twice the first, then the frequency of the first fork is
11 When two sound waves travel in the same direction in a
(a) 120 Hz (b) 160 Hz
medium the displacement of a particle located at X at (c) 180 Hz (d) 220 Hz
time t is given by ª JEE Main (Online) 2013
20 A vehicle with a horn of frequency n is moving with a
y1 = 0.05 cos( 0.50 π x − 100πt )
velocity of 30 ms −1 in a direction perpendicular to the
y 2 = 0.05 cos( 0.46 π x − 92 πt )
straight line joining the observer and the vehicle. The
where y1, y 2 and x are in metres and t in seconds. The observer perceives the sound to have a frequency
speed of sound in the medium is
(n + n1). If velocity of sound in air be 330 ms −1, then
(a) 92 m/s (b) 200 m/s (c) 100 m/s (d) 332 m/s n n
(a) n1 = 10n (b) n1 = 0 (c) n1 = (d) n1 = −
12 In order to double the frequency of the fundamental note 11 11
emitted by a stretched string, the length is reduced to 21 An observer is moving with half the speed of light
3
th of the original length and the tension is changed. towards a stationary microwave source emitting waves at
4 frequency 10 GHz. What is the frequency of the
The factor, by which the tension is to be changed, is microwave measured by the observer? (speed of light
3 2 8 9
(a) (b) (c) (d) = 3 × 108 ms −1) ª JEE Main (Offline) 2017
8 3 9 4
(a) 12.1 GHz (b) 17.3 GHz
13 A granite rod of 60 cm length is clamped at its middle (c) 15.3 GHz (d) 10.1 GHz
point and is set into longitudinal vibrations. The density of
22 Two trains are moving towards each other at speeds of
granite is 2.7 × 103 kg/m 3 and its Young’s modulus is
20 ms −1 and 15 ms −1 relative to the ground. The first train
9.27 × 1010 Pa. What will be the fundamental frequency of
sounds a whistle of frequency 600 Hz, the frequency of
the longitudinal vibrations? ª JEE Main 2018
the whistle heard by a passenger in the second train
(a) 5 kHz (b) 2.5 kHz (c) 10 kHz (d) 7.5 kHz
before the train meet is
14 A pipe open at both ends has a fundamental frequency (Speed of sound in air = 340 ms −1)
f in air. The pipe is dipped vertically in water, so that half (a) 600 Hz (b) 585 Hz (c) 645 Hz (d) 666 Hz
138 40 DAYS ~ JEE MAIN PHYSICS DAY TWELVE

23 A fixed source of sound emitting a certain frequency Statement II Amplitude of vibration at antinodes is
appears as νa when the observer is approaching the maximum and at nodes, the amplitude is zero and all
source with speed vo and vr when the observer recedes particles between two successive nodes cross the mean
from the source with the same speed. The frequency of position together.
the source is
νr + νa νr − νa 2νr ⋅ νa
27 Statement I We can recognise our friends by listening
(a) (b) (c) νa ⋅ νb (d) their voices.
2 2 νr + νa
Statement II The quality of sound produced by different
Direction (Q. Nos. 24-31) Each of these questions contains persons are different.
two statements : Statement I and Statement II. Each of these 28 Statement I The basic of Laplace correction was that,
questions also has four alternative choices, only one of which
is the correct answer. You have to select one of the codes (a), exchange of heat between the region of compression and
(b), (c), (d) given below: rarefaction in air is not possible.
(a) Statement I is true, Statement II is true; Statement II is Statement II Air is a bad conduction of heat and velocity
the correct explanation for Statement I of sound in air is large.
(b) Statement I is true, Statement II is true; Statement II is 29 Statement I If two waves of same amplitude, produce a
not the correct explanation for Statement I
resultant wave of same amplitude, then the phase
(c) Statement I is true; Statement II is false
difference between them will be 120°.
(d) Statement I is false; Statement II is true
Statement II The resultant amplitude of two waves is
24 Statement I A tuning fork is in resonance with a closed equal to sum of amplitude of two waves.
pipe. But the same tuning fork cannot be in resonance
30 Statement I Two longitudinal waves given by equation
with an open pipe of the same length.
y1( x , t ) = 2a sin (ω t − kx ) and
Statement II The same tuning fork will not be in
y 2 ( x , t ) = a sin ( 2ω t − 2kx ) will have equal intensity.
resonance with open pipe of same length due to end
correction of pipe. Statement II Intensity of waves of given frequency in
25 Statement I In a sound wave, a displacement node is a same medium is proportional to square of amplitude
pressure antinode and vice-versa. only.
Statement II Displacement node is a point of minimum 31. Statement I If we see the oscillations of a stretched wire
displacement.
at higher overtone mode, frequency of oscillations
26 Statement I Velocity of particles while crossing mean increases, but wavelength decreases.
position (in stationary waves) varies from maximum at 1
Statement II From v = ν ⋅ λ , λ ∝ as v = constant.
antinodes to zero at nodes. ν

DAY PRACTICE SESSION 2

PROGRESSIVE QUESTIONS EXERCISE


1 Sound of frequency f passes through a Quinck’s tube, 3 A train of sound waves is propagated along an organ
adjusted for intensity Im . What should be the length to pipe and gets reflected from an open end. If the
which the tube should be moved to reduce intensity to displacement amplitude of the waves (incident and
50% (speed of sound is v )? reflected) are 0.002 cm, the frequency is 1000 Hz and
v v v v wavelength is 40 cm. Then, the displacement amplitude
(a) (b) (c) (d)
2f 4f 8f 16f of vibration at a point at distance 10 cm from the open
end, inside the pipe is
2 A racing car moving towards a cliff sounds its horn. The (a) 0.002 cm (b) 0.003 cm (c) 0.001 cm (d) 0.000 cm
driver observes that the sound reflected from the cliff has
4 An engine approaches a hill with a constant speed.
a pitch one octave higher than the actual sound of the
When it is at a distance of 0.9 km, it blows a whistle
horn. If v is the velocity of sound, the velocity of the car is
v v whose echo is heard by the driver after 5 s. If the speed
(a) (b) of sound in air is 330 m/s, then the speed of the engine is
2 2
v v ª JEE Main (Online) 2013
(c) (d)
3 4 (a) 32 m/s (b) 27.5 m/s (c) 60 m/s (d) 30 m/s
DAY TWELVE WAVES 139

5 A travelling wave represented by y = A sin (ω t − kx ) is 9 A sonometer wire of length 114 cm is fixed at both the
superimposed on another wave represented by ends. Where should the two bridges be placed so as to
y = A sin (ωt + kx ). The resultant is divide the wire into three segments whose fundamental
(a)a standing wave having nodes at frequencies are in the ratio 1:3:4? ª JEE Main (Online) 2013
1 λ
x =  n +  , n = 0, 1, 2 (a) At 36 cm and 84 cm from one end
 2 2 (b) At 24 cm and 72 cm from one end
(b)a wave travelling along + x direction (c) At 48 cm and 96 cm from one end
(c)a wave travelling along −x direction (d) At 72 cm and 96 cm from one end

(d)a standing wave having nodes at x = , n = 0, 1, 2. 10 The transverse displacement y ( x , t ) of a wave on a string
2 2
+ bt 2 + 2 ab xt )
is given by y ( x , t ) = e − (ax . This represents
6 A piston fitted in cylindrical pipe is pulled as shown in the
figure. A tuning fork is sounded at open end and loudest a
b
sound is heard at open length 13 cm, 41 cm and 69 cm. (a) wave moving in − x direction with speed
a
The frequency of tuning fork if velocity of sound is (b) standing wave of frequency b
350 ms −1, is (c) standing wave of frequency
1
b
(d) wave moving in + x direction with speed a / b
11 A train is moving on a straight track with speed 20 ms −1. It
is blowing its whistle at the frequency of 1000 Hz. The
(a) 1250 Hz (b) 625 Hz
(c) 417 Hz (d) 715 Hz percentage change in the frequency heard by a person
standing near the track as the train passes him is close to
7 A and B are two sources generating sound waves. A (speed of sound = 320 ms −1) ª JEE Main 2015
listener is situated at C. The frequency of the source at A (a) 6% (b) 12% (c) 18% (d) 24%
is 500 Hz. A now, moves towards C with a speed 4 m/s.
The number of beats heard at C is 6. When A moves 12 A uniform string of length 20 m is suspended from a rigid
away from C with speed 4 m/s, the number of beats support. A short wave pulse is introduced at its lowest
heard at C is 18. The speed of sound is 340 m/s. The end. It starts moving up the string. The time taken to
frequency of the source at B is ª JEE Main (Online) 2013 reach the support is (Take, g = 10 ms − 2)
ª JEE Main (Offline) 2016
A C B
(a) 2 π 2 s (b) 2 s (c) 2 2 s (d) 2 s
(a) 500 Hz (b) 506 Hz 13 A motor cycle starts from rest and accelerates along a
(c) 512 Hz (d) 494 Hz
straight path at 2 ms −2. At the starting point of the motor
8 A pipe of length 85 cm is closed from one end.Find the cycle, there is a stationary electric siren. How far has the
number of possible natural oscillations of air column in motor cycle gone when the driver hears the frequency of
the pipe whose frequencies lie below 1250 Hz. The the siren at 94% of its value when the motor cycle was at
velocity of sound in air is 340 m/s. ª JEE Main 2014 rest? (Speed of sound = 330 ms −1)
(a) 12 (b) 8 (c) 6 (d) 4 (a) 49 m (b) 98 m (c) 147 m (d) 196 m

ANSWERS

SESSION 1 1 (c) 2 (d) 3 (c) 4 (d) 5 (d) 6 (a) 7 (a) 8 (d) 9 (d) 10 (d)
11 (b) 12 (d) 13 (a) 14 (d) 15 (a) 16 (b) 17 (c) 18 (a) 19 (a) 20 (b)
21 (b) 22 (d) 23 (a) 24 (c) 25 (b) 26 (a) 27 (a) 28 (a) 29 (c) 30 (c)
31 (a)
SESSION 2
1 (c) 2 (c) 3 (d) 4 (d) 5 (a) 6 (b) 7 (c) 8 (c) 9 (b) 10 (a)
11 (b) 12 (c) 13 (b)
140 40 DAYS ~ JEE MAIN PHYSICS DAY TWELVE

Hints and Explanations


SESSION 1 Wavelength, λ = 2 × 9 = 18 cm 12 From law of string, other factors
1 Mechanical transverse wave can be set Velocity, v = fλ remaining unchanged,
up in solids but never in liquids and . × 18 = 1.8 cms −1
= 01 ν1 l T1
= 2
gases. ν2 l1 T2
8 Comparing given equation with
2 There is no transfer of heat from standard form of wave equation, 3
Now, ν2 = 2 ν1 , l 2 = l 1 ,
compression to rarefaction as air is a bad we get 4
Hence, we have
conductor of heat and time of A = 0.001 m, ω = 100 s −1
compression and rarefaction is too small. 1 3 T1 2 T1
and k = 1 m −1 = × ⇒ =
3 In the first medium, frequency 100 50 2 4 T2 3 T2
c v ∴ ν= = Hz
= f = 2π π T2 9 9
λ λ 2π ⇒ = or T2 = T1
λ= = 2π m T1 4 4
It remains the same in second medium, k
l
i.e. f′= f ω 100 s −1 13
and v = = = 100 ms −1 4
v ′ 2v v 1 m −1
= = ⇒ λ′ = 2λ k
λ′ λ′ λ
Moreover, as the wave equations of the
4 Since, frequency, ν ∝ T form y = A sin (ωt + kx ) , the wave is
ν1 T1 3 T + 2.5 travelling along negative x-direction.
∴ = or = L
ν2 T2 2 T 9 From the given wave equation, we find From vibration mode,
9 2.5 2.5 5 that λ
or = 1+ or = or T = 2 N =L
4 T T 4 2 π −1
A = 4 cm, ω = s = 100 πs −1 2
5 Speed of sound is given by 0.02 ⇒ λ = 2L

γ RT and k = cm −1 ∴ Wave speed, v =
Y
v =
M
100 ρ

7 5 ∴ λ= = 100 cm So, frequency
RT RT k v
v O2 = 5 and v He = 3
ω 100 π f =
32 4 ν= = = 50 Hz λ
vO 2π 2π 1 Y
7×3× 4 ⇒ f =
∴ 2
= ω 100 π
v He 5 × 32 × 5 and v = = = 50 × 10 2 cms–1 2L ρ
k 2π
5 × 32 × 5 1 . × 1010
927
or v He = 460 × ≈ 1420 ms −1 100 =
7×3× 4 2 × 60 × 10−2 2.7 × 103
Hence, answer of wave speed v is
wrong, i.e. option (d) is correct. ≈ 5000 Hz
6 Let distance between two fixed points
be d, then f = 5 kHz
10 Tension in the string is given by
d ω2 14 For open ends, fundamental frequency f
t = also v ∝ T T = µv 2 = µ
v 2 in air, we have
k

t1 v
= 1 =
T2
0.04 (2 π / 0.04)2 λ
= = 6.25 N =l
t2 v2 T1 2
(2 π / 0.50)2
2 303 ⇒ λ = 2l
⇒ = ⇒ t 2 = 1.9 s 11 y 1 = 0.05 cos (0.50 π x − 100 π t ) l
t2 283 v= f λ
and y 2 = 0.05 cos (0.46 π x − 92 πt ) ⇒
v
f = =
v
…(i)
7 The given equation be written as Comparing these two equations with λ 2l
 x 1 
y = 4 sin  π  − +   y = A sin (k x − ω t )
t When a pipe is dipped
…(i)
 5 9 6  We have, ω1 = 100 π and ω2 = 92 π vertically in water, so that
half of it is in water, we l/2
The standard wave equation can be A 0.05
Now, speeds, v 1 = = have
written as 100 π 100 π λ l
l
y = a sin (ωt − kx + φ) =
A 0.05 4 2
and v2 = =
2π 2π
y = a sin  t − x + φ …(ii) 92 π 92 π ⇒ λ = 2l ⇒ v = f ′ λ
T λ  v v
Now, the resultant speed, ⇒ f′= = = f …(ii)
Equating Eqs. (i) and (ii), we get 2 2 λ 2l
 0.05   0.05 
Amplitude, a = 4 cm v =   +   [ π = 3.14] Thus, the fundamental frequency of the
1 1  92 π   100 π  air column is now,
Frequency, f = = Hz = 0.1 Hz
T 10 = 200 m/s f = f′
DAY TWELVE WAVES 141

15 We know that, third overtone of closed by the source will be given by the difference is maximum i.e. at the same
organ pipe means seventh harmonic formula, time, it is a pressure antinode.
1 /2
∴ (ν7 )closed = (ν 4 )open 1 + v /c  On the other hand, at the mid-point of a
fobserved = factual ⋅  
 v   v  1 − v /c  compression or a rarefaction, the
or 7  = 4  v 1 displacement variation is maximum i.e.
 4l c   2l o  Here, frequency, = such a point is displacement antinode.
lo 8 c 2
or = 1 /2 However such a point is pressure node,
So, fobserved = factual 
lc 7 3 / 2 as pressure variation is minimum at such

lc 7  1 / 2 a point.
or =
lo 8 ∴ fobserved = 10 × 3 = 173
. GHz 26 Stationary wave is represented as shown
in figure.
16 y 1 = 0.25 sin (310t ) …(i) 22 Here, v = 340 ms −1 ,
A
and y 2 = 0.25 sin (316t ) …(ii) v s = velocity of 1st train = 20 ms −1 ,
We have, ω1 = 310 v o = velocity of 2nd train = 15ms −1 and
A
310 N N
⇒ f1 = unit ν 0 = 600 Hz. N

and ω2 = 316 As S and O are approaching each other,
316 hence It is quite clear from figure that at nodes
⇒ f2 = unit  v + vo  the amplitude is zero and velocity of
2π ν=  ν0 particle is also zero and at antinodes the
Hence, beat frequency  v − vs 
amplitude is maximum. So that the
316 310  340 + 15 velocity of particle is also maximum and
= f2 − f1 = − =  × 600 = 666 Hz
2π 2π  340 − 20  all particles cross mean position between
3 two successive nodes.
= unit
π 23 Here, ν a = ν  v + v o 
 v  27 Sounds coming from the different
17 Maximum number of beats νa vo sources can be recognised by virtue of
or = 1+ their quality which is characteristics of
= ν + 1 − (ν − 1) = 2 ν v
vo ν sound. That is why we recognise the
18 There are four beats between P and Q , or = a −1 …(i) voices of our friends.
v ν
therefore the possible frequencies of P
v − vo 
are 246 or 254 Again, ν r = ν  
28 According to Laplace, the changes in
 v  pressure and volume of a gas, when
(i.e. 250 ± 4) Hz.
vo ν sound waves propagated through it, are
When the prong of P is filled, its ∴ = 1− r …(ii) not isothermal but adiabatic. A gas is a
v ν
frequency becomes greater than the bad conductor of heat. It does not allow
From Eqs. (i) and (ii), we get
original frequency. the free exchange of heat between
νa ν
If we assume that the original frequency −1 = 1− r compressed layer, rarified layer and its
ν ν surrounding.
of P is 254, then on filling its frequency νa νr
will be greater than 254. The beats or + =2
ν ν 29 The resultant amplitude of two waves is
between P and Q will be more than 4. νa + νr given by
But it is given that the beats are reduced or =2
ν A= a12 + a22 + 2a1 a2 cos θ
to 2, therefore, 254 is not possible.
ν + νr
Therefore, the required frequency must or ν= a Here, a1 = a2 = A = a
be 246 Hz. 2
1 / 2 = 1 + cos θ
19 As forks have been arranged in 24 If a closed pipe of length L is in or cos θ = − 1 / 2 or θ = 120°
resonance with a tuning fork of
ascending order of frequencies, hence if
v 30 I = 1 ρω2 A2 v
frequency of Ist fork be n, then frequency ν, then ν = 2
n2 = n + 8 4L
Here, ρ = density of medium,
n3 = n + 2 × 8 = n + 16 An open pipe of same length L produces A = amplitude,
and n16 = n + 15 × 8 vibrations of frequency
v
. Obviously, ω = angular frequency and
= n + 120 = 2n 2L v = velocity of wave
⇒ n = 120 Hz it cannot be in resonance with the given ∴Intensity depend upon amplitude,
 v  frequency as well as velocity of wave
20 Since, vehicle having siren is moving in tuning fork of frequency ν  = .
 4 L Also, I1 = I2
a perpendicular direction, hence there
will be no Doppler shift in frequency
25 At the point where a compression and a 31 ν = η  v  , where n = 1, 2, 3, …
and n1 = 0.  2l 
rarefaction meet, the displacement is
As, η increases, frequency increases.
21 As the observer is moving towards the minimum and it is called displacement
source, so frequency of waves emitted Hence, wavelength decreases.
node. At this point, the pressure
142 40 DAYS ~ JEE MAIN PHYSICS DAY TWELVE

SESSION 2 Clearly, it is equation of standing wave According to question,


for position of nodes y = 0. Let freqency of source B is Z Hz
1 I m = 4 ka2 , I = 2ka λ ∴ Z = 506 ± 6 ⇒ Z = 500 or 512
i.e. x = (2n + 1)
I ′ = k (a2 + a2 + 2a2 cos 2 θ) 4 and Z = 494 ± 18 ⇒ Z = 512 or 476
1 λ
π =  n +  Thus, required frequency = 512 Hz
⇒ φ=  2 2
2 8 For closed organ pipe = (2n + 1)v
λ 6 In a closed organ pipe in which length 4l
Path difference = = 2x
4 of air-column can be increased or [where, n=0,1,2.....]
v decreased, the first resonance occurs at
⇒ = 2x
4f λ / 4 and second resonance occurs at

85 cm
v 3λ / 4.
⇒ x=
8f
l
l1 =
4
2 Let n be the actual frequency of sound (2n + 1) v
of horn. If v s be the velocity of car, then < 1250
N A 4l
frequency of sound striking the cliff
(source is moving towards listener) 4 × 0. 85
y (2n + 1) < 1250 ×
v ×n 340
n′ = …(i)
v − vs l2 = 3l (2n + 1) < 12. 52
4
The frequency of sound heard on n < 5.25
N
reflection A A So, n = 0,1,2,3,...,5
(v + v s ) n ′ (v + v s ) v ×n y So, we have 6 possibilities.
n ′′ = = ×
(v − v s ) Thus, at first resonance, f
v v
λ 9 ν1 = 1
= 13 …(i) 2L1 µ
n ′′ v + v s
or = =2 4
n v − vs
and at second resonance,
v + v s = 2v − 2v s 3λ L1 L2 L3
v = 41 …(ii)
or vs = 4
3 Let length of three segments be
Subtracting Eq. (i) from Eq. (ii), L1 , L2 and L3 ,
3 The equation of stationary wave for
open organ pipe can be written as, we have 1
ν2 = f /µ
2 πx   2 πft  3λ λ
y = 2 A cos 
2L2
 sin  , − = 41 − 13
 λ   v  4 4 1
ν3 = f /µ
where x is the open end from where 2L3
∴ λ = 56 cm
wave gets reflected. So, that ν1 L1 = ν2 L2 = ν3 L3
Hence, frequency of tuning fork,
Amplitude of stationary wave is,
v 350 As, ν1 : ν2 : ν3 = 1 : 3 : 4
2 πx 
A s = 2 A cos  
n= =
λ 56 × 10−2
= 625Hz ν L
 λ  ν1 = 2 2
L1
For x = 01
. m, 7 Here, frequency of source = 500 Hz and ν2 = 3 ν1 , ν3 = 4ν1
 2 π × 01
. 
A s = 2 × 0.002 cos  =0 Speed of source A = 4 m/s = u ν1 1 L1
 0.4  Then, source is moving towards L2 = L1 = L1 =
ν2 3 3
4 We have, 900 + 900 − x stationary observer,
v ν1 ν L
= 330 × 5 = 1650 ν′ = ν0 and L3 = L1 = 1 L1 = 1
v −u ν3 4ν1 4
(900 – x) m (where, v = speed of sound) L 1 + L2 + L3 = 114
340
= × 500
Now, L 1  1 + +  = 114
340 − 4 1 1
x 900 m 340  3 4
⇒ ν′ = × 500 Hz
 12 + 4 + 3 
336 ⇒ L1   = 114
∴ x = 150 m = 506 Hz  12 
∴ Speed =
150
= 30 m/s Now, when source is reciding from the  19 
L 1   = 114
5 observer  12 
5 y = y1 + y2 ν′ =
v
ν0 (114 × 12)
⇒ L1 = = 72 cm
v +u 19
= A sin (ωt − kx ) + A sin(ωt + kx ) 340
= × 500 Hz L 1 72
y = 2 A sin ω t cos kx L2 = = = 24 cm
344 3 3
∴ ν′ = 494 Hz
DAY TWELVE WAVES 143

10 y ( x, t ) = e − (ax
2
+ bt2 + 2 abxt ) ∆f  2cv s  20 dx t

− ( a x + b t )2
or
f0
× 100 =  2
c − vs 
2
 × 100 ⇒ ∫0 x
= ∫0 g dt
=e
It is a function of type 2 × 320 × 20 0 = 10 t
[2 x ]20
= × 100
y = f (ω t + kx ) 300 × 340 ⇒ 2 20 = 10 t
∴ y ( x, t ) represents wave travelling 2 × 32 × 20
= t =2 2s
along − x direction. 3 × 34
ω b b = 12. 55% = 12% 13 For motor cycle, u = 0, a = 2 ms −2
Speed of wave = = =
k a a 12 A uniform string of length 20 m is Observer is in motion and source is at
11 Apparent frequency heard by the person suspended from a rigid support. Such rest, then apparent frequency,
before crossing the train. that the time taken to reach the support, v − vo
ν′ = ν
 c   320  1000 v + vs
f1 =   fo =   mgx
c − vs   320 − 20  T = 94 330 − v o
l ⇒ ν=ν
100 330
Similarly, apparent frequency
330 × 94
heard,after crossing the trains ⇒ 330 − v o =
100
 c   320  1000 94 × 33
f2 =   fo =   (l , m ) P dx ⇒ v o = 330 −
c + vs   320 + 20  10
[c= speed of sound in air] 33 × 6
x = ms −1
∆f = f1 − f2 10
 2cv  mgx v 2 − u2
=  2 s 2  f0 So, velocity at point P = s=
l 2a
c − vs 
m/l 9 × 33 × 33
=
Percentage change in frequency heard i.e. v = gx 100
by the person standing near the track as 9 × 1089
dx
= gx = ≈ 98 m
the train passes him is dt 100
144 40 DAYS ~ JEE MAIN PHYSICS DAY THIRTEEN

DAY THIRTEEN

Unit Test 2
(Waves and Oscillations)

1 A mass m attached to a spring of spring constant k is 4 A block is kept on a table which performs simple
stretched a distance x 0 from its equilibrium position and harmonic motion with frequency 5Hz in horizontal plane.
released with no initial velocity. The maximum speed The maximum amplitude of the table at which block does
attained by the mass in its subsequent motion and the not slip on the surface of table is, (if coefficient of friction
time at which this speed would be attained are between the block and surface of table is 0.6.) (Given,
respectively, g = 10 m/s 2 ).
k m k x0 π m (a) 0.06 m (b) 0.006 m
(a) x 0, π (b) ,
m k m 2 2 k (c) 0.02 m (d) 0.002 m
k π m k x0 m 5 For a particle executing SHM, determine the ratio of
(c) x 0, (d) ,π
m 2 k m 2 k average acceleration of the particle between extreme
2 The acceleration-displacement graph of a particle position and the equilibrium position w.r.t. the maximum
executing SHM is shown in the figure. The time period of acceleration
SHM is 4 2
(a) (b)
–2
a(ms ) π π
1 1
(c) (d)
π 2π
6 Two springs are made to oscillate simple harmonically
37°
x (m) when the same mass is suspended, individually. The time
periods obtained are T1 and T2 . If both the springs are
connected in series and then made to oscillate
4π when suspended by the same mass, the resulting time
(a) s
3 will be
2π T1T2
(b) s (a)T1 + T2 (b)
3 T1 + T2
(c) The given graph doesn’t represent SHM T + T2
(c) T12 + T22 (d) 1
(d) Information is insufficient 2
3 A spring balance has a scale that can read from 0 to 7 Find the time period of oscillations of a torsional
50 kg. The length of the scale is 20 cm. A body pendulum, if the torsional constant of wire is10 π 2 in
suspended from this balance, when displaced and SI units. The moment of inertia of the rigid body is 10
released, oscillates harmonically with a time period of kg - m 2 about the axis of rotation
0.6 s. The mass of the body is (Take, g = 10 ms −2 ) (a) 1 s (b) 2 s
(a) 10 kg (b) 25 kg 1
(c) 4 s (d) s
(c) 18 kg (d) 22.8 kg 2
DAY THIRTEEN UNIT TEST 2 (WAVES AND OSCILLATIONS) 145

8 Acceleration-displacement graph for four particles are 13 A spring of negligible mass having a force constant k
shown, identify the one which represents SHM for all the extends by an amount y when a mass m is hung from it.
values of displacements The mass is pulled down a little and then released. The
system begins to execute SHM of amplitude A and
acceleration (a) acceleration (a) angular frequency ω. The total energy of the mass-spring
system will be

37
°
37° mω 2 A 2 mω2 A 2 ky 2
(a) (b) (a) (b) −
displacement (x) displacement (x) 2 2 2
ky 2 mω2 A 2 ky 2
(c) (d) +
2 2 2
acceleration (a)
14 Total number of independent harmonic waves in the
20° resultant displacement equation given by
°
37

(c) (d) None of these t


displacement (x)
y = 3 sin2 sin 800t
2
(a) 2 (b) 1 (c) 4 (d) 3
15 A load of mass m falls from a height h on the
9 In case of simple harmonic motion, if the velocity is scale pan hung from a spring as shown. If the k
plotted along the X-axis and the displacement from the spring constant is k, mass of the scale pan is zero
equilibrium position is plotted along the Y-axis, the and the mass m does not bounce relative to the m
h
resultant curve happens to be an ellipse with the ratio pan, then the amplitude of vibration is
major axis (along X ) mg mg 2hk
= 20π. (a) (b) 1+
minor axis (along Y ) k k mg
mg mg 1 + 2hk
What is the frequency of the simple harmonic motion? (c) + (d) None of these
k k mg
(a) 100 Hz (b) 20 Hz
1 16 A sound absorber attenuates the sound level by 30dB.
(c) 10 Hz (d) Hz
10 The intensity of sound is decreased by a factor of
10 A simple pendulum is suspended from the ceiling of a (a) 100 (b) 1000
stationary tram car. Now, the car starts accelerating, the (c) 10000 (d) 10
time period of a simple pendulum is the least when [Take 17 The velocity of a particle executing a simple harmonic
magnitude of acceleration to be same in all the cases] motion is 13 ms − 1 , when its distance from the equilibrium
(a) car is accelerating up position is 3 m and its velocity is 12 ms − 1, when it is 5 m
(b) car is accelerating down away from equilibrium position. The frequency of the SHM
(c) car is accelerating horizontally is
(d) car is stationary 5π 5
(a) (b)
8 8π
11 A particle executes SHM about O with an amplitude A 8π 8
and time period T . The magnitude of its acceleration, at (c) (d)
5 5π
T
s after the particle reaches the extreme position, would 18 Two identical strings A and B, have nearly the same
8
be tension. When they both vibrate in their fundamental
resonant modes, there is a beat frequency of 3 Hz. When
4 π 2A 4 π 2A
(a) 2
(b) string B is tightened slightly, to increase the tension, the
2T T2
beat frequency becomes 6 Hz. This means
2 π 2A
(c) (d) None of these (a) that before tightening A had a higher frequency than B,
2T 2 but after tightening, B has a higher frequency than A
12 A string of length 1.5 m with its two ends clamped, is (b) that before tightening B has higher frequency than A, but
vibrating in the fundamental mode. The amplitude at the after tightening A has higher frequency than B
centre of the string is 4 mm. The minimum distance (c) that before and after tightening A has higher frequency
than B
between two points having amplitude 2 mm, is
(d) that before and after tightening B has higher frequency
(a) 1 m (b) 75 cm than A
(c) 60 cm (d) 50 cm
146 40 DAYS ~ JEE MAIN PHYSICS DAY THIRTEEN

19 The ratio of densities of oxygen and nitrogen is 16 : 14. 26 A transverse wave on a string travelling along positive
At what temperature, the speed of sound in oxygen will x-axis has been shown in the figure below
be equal to its speed in nitrogen at 14°C?
(a) 50°C (b) 52°C
(c) 48°C (d) 55°C
1
20 A train is passing by a platform at a constant speed of
40 ms −1. The horn of the train has a frequency of 320 Hz.
The mathematical form of the wave is shown
Find the overall change in frequency detected by a
 2π 
person standing on the platform, i.e. when the train y = (3.0 cm) sin 2 π × 0.1 t − x
 100 
approaching and then precedes from him. (Take,
velocity of sound in air as 320 ms −1) where t is in seconds and x is in cm. Find total distance
travelled by the particle at the (1), in 10 min 15 s,
(a) 216.4 Hz (b) 81.3 Hz measured from the instant shown in the figure and
(c) 365.7 Hz (d) 284.4 Hz direction of the motion of the particle at the end of this
21 A string of length 0.4 m and mass 10−2 kg is clamped at time.
one end. The tension in the string is1.6 N. Identical wave (a) 6 cm, in upward direction
(b) 6 cm, in downward direction
pulses are generated at the free end, after a time interval
(c) 738 cm, in upward direction
∆t. The minimum value of ∆t, so that a constructive
(d) 732 cm, in upward direction
interference takes place between successive pulses is
(a) 01
. s 27 A wire having a linear mass density of 5 × 10− 3 kg m − 1
(b) 0.05 s is stretched between two rigid supports with a tension of
(c) 0.2 s 450 N. The wire resonates at a frequency of 420 Hz. The
(d) Constructive interference cannot take place next higher frequency at which the wire resonates is
22 A string vibrates according to the equation 490Hz. The length of the wire is
 2 πx  (a) 2.5 m (b) 2.14 m
Y = 5 sin   × cos 20πt , where x and y are in cm
 3  (c) 2.25 m (d) 2.0 m
and t in second. The distance between two adjacent 28 A man stands on a weighing machine placed on a
nodes is horizontal plateform. The machine reads 50 kg. By means
(a) 3 cm (b) 4.5 cm (c) 6 cm (d) 1.5 cm of a suitable mechanism, the plateform is made to
23 A point source of sound is placed in a non-absorbing execute harmonic vibration up and down with a
medium. Two points A and B are at the distance of 1 m frequency of 2 vibrations per second. What will be the
and 2 m, respectively from the source. The ratio of effect on the reading of the weighing machine? The
amplitudes of waves at A to B is amplitude of vibration of plateform is 5 cm.
(a) 1 : 1 (b) 1 : 4 (c) 1 : 2 (d) 2 : 1 (Take, g = 10 ms − 2 )
24 The mathematical form of three travelling waves are (a) 11 kgf to 93 kgf
given by (b) 10.5 kgf to 89.5 kgf
(c) 10 kgf to 15.5 kgf
y1 = ( 2 cm ) sin ( 3x − 6 t ),
(d) 25.6 kgf to 100.5 kgf
y 2 = ( 3 cm ) sin ( 4x − 12 t ) ,
29 A small trolley of mass 2.0 kg resting on a horizontal turn
and y 3 = ( 4 cm ) sin ( 5x − 11 t )
table is connected by a light spring to the centre of the
of these waves,
table. When the turn table is set into rotation at speed of
(a) wave 1 has greatest wave speed and wave has
360 rpm, the length of the stretched spring is 43 cm. If
maximum transverse string speed
(b) wave 2 has greatest wave speed and wave 1 has
the original length of the spring is 36 cm, the force
greatest maximum transverse string speed constant is
(c) wave 3 has greatest wave speed and wave 1 has (a) 17025 Nm− 1 (b) 16225 Nm− 1
maximum transverse string speed (c) 17475 Nm− 1 (d) 17555 Nm− 1
(d) wave 2 has greatest wave speed and wave 3 has 30 At 16°C, two open end organ pipes, when sounded
maximum transverse string speed together give 34 beats in 2 s. How many beats per
25 If the maximum speed of a particle carrying a travelling second will be produced, if the temperature is raised to
wave is v 0, then find the speed of a particle when the 51°C?
displacement is half that of the maximum value (Neglect increase in length of the pipes)
v0 3 v0 3 v0 (a) 18 s− 1 (b) 15 s− 1
(a) (b) (c) (d) v0
2 4 2 (c) 20 s− 1 (d) 10 s− 1
DAY THIRTEEN UNIT TEST 2 (WAVES AND OSCILLATIONS) 147

31 During earthquake, both longitudinal and transverse 36 Statement I When a wave goes from one medium to
waves are produced having speeds 4.0 km/s and other, average power transmitted by the wave may
8.0 km/s, respectively. If the first transverse wave change.
reaches the seismograph 8 minutes after the arrival of
first longitudinal wave, then the distance of the position, Statement II Due to a change in the medium, amplitude,
where the earthquake occurred is speed, wavelength and frequency of the wave may
change.
(a) 3440 km (b) 3880 km
(c) 3840 km (d) 3500 km 37 Statement I A particle performs a simple harmonic
32 A pendulum has time period T in air. When it is made to motion with amplitude A and angular frequency ω. To
oscillate in water, it acquired a time periodT ′ = 2T . The change the angular frequency of simple harmonic motion
density of the pendulum bob is equal to to 3 ω, and amplitude to A /2, we have to supply an extra
(Take, density of water = 1) 5
energy of mω 2 A 2 , where m is the mass of the particle
(a) 2 (b) 2 4
(c) 2 2 (d) None of these executing simple harmonic motion.
33 On a planet a freely falling body takes 2 s when it is Statement II Angular frequency of the simple harmonic
dropped from a height of 8 m. The time period of simple motion is independent of the amplitude of oscillation.
pendulum of length 1 m on that planet is
38 Statement I Time period of spring pendulum is the same
(a) 3.14 s (b) 16.28 s
(c) 1.57 s (d) None of these whether in an accelerated or in an inertial frame of
reference.
Direction (Q. Nos. 34-40) Each of these questions contains
two statements Statement I and Statement II. Each of these Statement II Mass of the bob of the spring pendulum and
questions also has four alternative choices, only one of which the spring constant of spring are independent of the
is the correct answer. You have to select one of the codes (a), acceleration of the frame of reference.
(b), (c), (d) given below
(a) Statement I is true, Statement II is true; Statement II is the 39 Statement I The total energy of a particle executing
correct explanation for Statement I simple harmonic motion, can be negative.
(b) Statement I is true, Statement II is true; Statement II is not
Statement II Potential energy of a system can be
the correct explanation for Statement I
negative.
(c) Statement I is true, Statement II is false
(d) Statement I is false, Statement II is true 40 Statement I A circular metal hoop is suspended on the
34 Statement I Waves on a string can be longitudinal in edge, by a hook. The hoop can oscillate from one side to
nature. the other in the plane of the hoop, or it can oscillate back
and forth in a direction perpendicular to the plane of the
Statement II The string cannot be compressed or
hoop. The time period of oscillation would be more when
rarified.
oscillations are carried out in the plane of hoop.
35 Statement I A wave of frequency 500 Hz is propagating
Statement II Time period of physical pendulum is more, if
with a velocity of 350 m/s. Distance between two particles
with 60° phase difference is 12 cm. the moment of inertia of the rigid body about the
corresponding axis passing through the pivoted point, is
λ more.
Statement II x = φ

ANSWERS
1. (c) 2. (a) 3. (d) 4. (b) 5. (b) 6. (c) 7. (b) 8. (b) 9. (c) 10. (c)
11. (a) 12. (a) 13. (d) 14. (d) 15. (b) 16. (b) 17. (b) 18. (d) 19. (d) 20. (b)
21. (c) 22. (a) 23. (d) 24. (d) 25. (c) 26. (c) 27. (b) 28. (b) 29. (c) 30. (a)
31. (c) 32. (b) 33. (a) 34. (d) 35. (a) 36. (c) 37. (d) 38. (a) 39. (a) 40. (a)
148 40 DAYS ~ JEE MAIN PHYSICS DAY THIRTEEN

Hints and Explanations


Maximum acceleration (amax ) = ω2 A. 2
2π 
Acceleration = − ω2 x = − 
1 At the mean position, the speed will be A
 ×
maximum. Then, the required ratio is, T  2
kx20 mv 2 T /4

2
=
2 =
∫0 ω2 A sin ωt dt
=
2 Magnitude of acceleration =
4 π2 A
T / 4 × ω2 A π 2 T2
k
⇒ v = v max = x0
m 6 Let the spring constants of the two 12 As = 2 A sin kx
T 2 mm = 4 mm sin kx
and this is attained at t = . springs be k1 and k2 respectively, then,
4 m 4 π2 m π 5π
T = 2π and k = ⇒ kx = ,
Time period of motion is, 6 6
K T2
5π π  1 λ
x2 − x1 = 
m
T = 2π When the two springs are connected in ⇒ − × =
k  6 6  k 3
series, then
So, the required time is, As the string is vibrating in fundamental
m
T π m T = 2π mode
t = = keq
4 2 k λ
L = ⇒ λ = 2L = 3 m
k1 k2 2
2 da = − tan 37° = − 3 ⇒ a = − 3 x where, keq =
k1 + k2
dx 4 4 So, required separation between two
On comparing with a = − ω2 x, we get points,
⇒ T = T12 + T22
3 2π 3 4π x2 − x1 = 1 m
ω2 = ⇒ = ⇒ T = s
4 T 2 3 7 For torisional pendulum, τ = − k θ 13 From initial equilibrium position,
k ky = mg
3 The scale can read a maximum of 50 kg, α =− θ
for a length of 20 cm. Let spring I
When block is at distance x below mean
constant be k then, k
⇒ ω =
2 position
kx 0 = mg I Kinetic energy of the block,
[for m = 50 kg, x 0 = 20 cm] I 10
T = 2π = 2π = 2s mω2 A2
k 10 π2 K = cos 2 (ωt − φ)
⇒ k × 0. 2 = 50 × 10 ⇒ k = 2500 Nm 2
Let mass of the body be m 0 , then from 8 For a particle to execute SHM, [From SHM theory]
m0 m0 a = − ω2 x
T = 2π ⇒ 0.6 = 2 π Elastic potential energy of
k 2500 da spring-block-earth system,
So, = − ω2 , where ω2 is positive
⇒ m 0 = 22.8 kg dx k ( y + x )2
Ue =
quantity. This means for a particle to 2
4 Maximum force on the block on the
surface of table due to simple harmonic
execute SHM, the acceleration- where, x = A sin(ωt + φ)
displacement curve should be a straight Gravitational potential energy of
motion, F = mω2 A, where A →
line having a negative slope, which is spring-block-earth system is, U g = − mgx
amplitude. shown in option (b).
Friction force on the block, Fs = µmg
9 In representation of SHM in ellipse as Taking, mean position as reference
It will not slip on the surface of the
velocity along x-axis and displacement position for gravitation potential energy.
table, if
along y-axis. Total energy,
F = Fs
Major axis (along X - axis)
mω2 A = µmg Q = 20 π mω2 A2 ky 2
Minor axis (along Y - axis) E = K + Ue + U g = +
µg 2 2
A= ωA
ω2 ⇒ = 20 π ⇒ ω = 20 π
A 14 y = 3sin2 t sin 800 t
0 ⋅ 6 × 10 2
= ⇒ 2 πf = 20 π
1 − cos t 
(2 × 3 ⋅ 14 × 5)2 The frequency of SHM, f = 10 Hz = 3  sin 800 t
= 0.006 m  2 
10 When car is accelerating horizontally 3 3
5 Let the equation of SHM be, = sin 800 t − sin 800 t cos t
g eff = g 2 + a2 2 2
x = A sin ωt 3 3
= sin 800 t − [2sin 800 t cos t ]
Average acceleration between extreme 11 Let at t = 0,the particle be at the extreme 2 4
position and the equilibrium position, position, then the equation of SHM can 3 3
T = sin 800 t − [sin 801 t + sin 799 t ]
i.e. from time t = 0 to t = be written as 2 4
4 2π
T /4 x = A cos (ωt ) = A cos  × t  3 3 3
= sin 800 t − sin 801 t − sin 799 t
Im =
∫0 ω2 A sin ω t dt T  2 4 4
T π A ∴Number of independent harmonic wave
T /4 At t = , x = A cos =
8 4 2 = 3.
DAY THIRTEEN UNIT TEST 2 (WAVES AND OSCILLATIONS) 149

15 From the conservation principle, If f1 − f2 = 3, then according to given The pulse gets inverted after reflection
1 2 condition, when f2 increases f1 − f2 the from the fixed end, so for constructive
mgh = kx 0 − mgx 0 interference to take place between
2 decreases so the frequencies of strings
where, x 0 is maximum elongation in are related by f2 − f1 = 3. successive pulses, the first pulse has to
i.e. before tightening, f2 > f1 undergo two reflections from the fixed
spring. end.
1 After tightening,
⇒ kx20 − mgx 0 − mgh = 0 2 × 0.4 + 2 × 0.4
2 f2′ − f1 = 6, So, ∆t = = 0.2 s
8
2mg 2mg
⇒ x20 − x0 − h=0 i.e. f2′ > f1 22 The node and antinodes are formed in a
k k
γP γRT standing wave pattern as a result of the
mg  2mg  2mg
2
19 Speed of sound, v = = interference of two waves.
2 ±
  + 4× h ρ M
2  k  k Distance between two nodes is half of
⇒ x0 = Speed of sound in oxygen at t°C, wavelength (λ ).
2
γR(t + 273)
Amplitude = Elongation for lowest v oxy = So, standard equation of standing wave is
extreme position − elongation for M oxy
2 πx 2 πvt
equilibrium position. Speed of nitrogen at 14°C, y = 2asin cos …(i)
mg 2hk  mg  λ λ
= x 0 − x1 = 1+ Q x1 = . γR(14 + 273)
k mg  k  vN = where, a is amplitude, λ is wavelength, v
MN is velocity and t is time.
16 If I1 be initial intensity of sound and I2 As v oxy = v N Given equation
be the final intensity of sound, MN 14 + 273 2 πx
⇒ = y = 5 sin cos 20 πt …(ii)
I I M oxy t + 273 3
then S 1 = 10 log 1 and S 2 = 10 log 2
I0 I0 ρN 14 + 273 Comparing Eqs. (i) and (ii), we get
⇒ = 2 πx 2 πx
S 1 − S 2 = 30 dB ρoxy t + 273 =
I1 I λ 3
10 log − 10 log 2 = 30 14 14 + 273
I0 I0 ⇒ = ⇒ λ = 3 cm
16 t + 273
I I  2296 − 1911 23 Let the power of source be P and let it be
10 log  1 / 2  = 30 ⇒ t = = 55° C. placed at Q.
 I0 I0  7
I Then, intensity at A and at B would be
10 log 1 = 30 20 For situation 1,
I2 given by
v −0 P
I1 fap1 = ×f IA =
log =3 v − vs 4 π × 12
I2
320 P
I1 = × 320 and IB =
= 103 320 − 40 4 π × 22
I2
1 = 3657
. Hz (Amp. ) A IA 22
I2 = I1 ⇒ = = = 2:1
1000 (Amp. )B IB 12
Situation -1 Situation -2
17 Speed in SHM, v = ω a2 − x2 24 For the wave, y = A sin (kx − ωt ),
⇒ v 2 = ω2 (a2 − x2 ) Train Train ω
the wave speed is and the maximum
According to question, k
transverse string speed is Aω.
v 12 = ω2 (a2 − x12 )
Hence option (d) is correct.
v 22 = ω2 (a2 − x22 )
For situation 2, 25 For the wave y = A sin (ωt − kx ),
⇒ v 12 − v 22 = ω2 ( x22 − x12 )
v v 0 = Aω
v 12 − v 22 (13)2 − (12)2 fap2 = × f where A is the maximum displacement.
⇒ω = = v − (− v s )
x22 − x12 (5)2 − (3)2 For the given condition,
= 284.4 Hz A
169 − 144 25 5 = A sin (ωt − kx )
= = = ∆f = fap 1 − fap2 2
25 − 9 16 4 1
5 5 = 81.3 Hz ⇒ sin (ωt − kx ) =
Also, ω = 2 πf = ⇒ f = . 2
4 8π ∂y
21 Velocity of wave on the string and = Aω cos (ωt − kx )
18 Let the fundamental frequencies of A ∂t
T
and B, before tightening of B are f1 and = = 8 ms −1 = Aω
3
=
3 v0
µ 2 2
f2 , respectively.
Then either f1 − f2 = 3 or f2 − f1 = 3. 26 We have, T = 1
s = 10 s
As tension in B increases (due to 0.1
tightening), its frequency increases and 0.4 m In one complete cycle, particle
the beat frequency also increases. travels a distance, 4 times the
amplitude.
150 40 DAYS ~ JEE MAIN PHYSICS DAY THIRTEEN

So, in a time interval of 10 min 15 s i.e., ν16 ν16 34 For longitudinal wave, the medium has
m = n1 − n2 = −
615 s i.e., 61 full + 1 half- cycles, the 2l 1 2l 2 to compress and rarify while the string
distance travelled  1 1  cannot be compressed or rarified.
= (4 × 3) × 61 + (2 × 3) × 1 ⇒ ν16  −  = 17 … (i)
= 732 + 6 = 738 cm
 1
2l 2l2  35 λ = v = 350 = 07
. m
 1 n 500
1 
At this instant, the particle is moving in and ν 51  −  =m … (ii) π π
 2l 1 2l 2  φ = 60° = 60 × = radian
an upward direction. 180° 3
Divide Eq. (ii) by Eq. (i), we get λ
27 The frequency of vibration is As, x = φ
v 51 m 273 + 51 324 2π
ρ T = = =
ν= 273 + 16 π
×   = 012
v 16 17 289 07
.
2l m ⇒ x= . m = 12 cm
18 2 π  3 
ρ T ⇒ m= × 17 = 18 s− 1
As 420 = 17
36 Pav = ρvω A
2 2
2l m
ρ+ 1 T 31 Let d be the distance, then time taken 2
and 490 = by longitudinal wave,
2l m 37 Total initial energy of particle in SHM,
ρ d d

420
= =
6 t1 = = s 1
v1 4 E1 =
mω2 A2
490 ρ + 1 7 2
⇒ 7ρ = 6ρ + 6 ⇒ ρ = 6 Time taken by transverse wave, A
d d Energy when amplitude is and angular
6 450 t2 = = s 2
∴ 420 = v2 8 frequency is 3ω.
2l 5 × 10− 3
As, t 1 − t 2 = 8 min = 8 × 60 s 2
m(3ω )2   = mω2 A2
900 1 A 9
⇒ l = = 2.14 m d d d E2 =
420 ∴ − = 480 ⇒ = 480 2 2 8
4 8 8
∴ Extra energy = E2 – E1
28 Maximum acceleration, ⇒ d = 480 × 8 = 3840 km. 9 1
A max = ω a = (2 πν) a = 4 π ν a
2 2 2 2 = mω2 A2 – mω2 A2
2
32 The effective acceleration of bob in 8 2
= 4 ×   × (2)2 + 0 ⋅ 05
22 water 5 5
= mω A = E1 (gain )
2 2
 7  σ 8 4
g ′ = g 1 − 
= 7.9 ms − 2  ρ
38 The time period of a spring pendulum is
Maximum force on the man where, σ = density of water and m
= m(g + A max ) ρ = density of bob given by, T = 2π and hence is not
k
= 50(10 + 7.9) = 895N The time period of bob in air, T = 2π
l
affected by the acceleration of the frame
= 89.5 kgf g
Minimum force on the man of reference.
l
= m(g − A max ) The time period in water, T ′ = 2π
g′ 39 Total energy of the particle performing
= 50(10 − 7.9) = 105N = 10.5 kgf
 σ simple harmonic motion is,
Hence, the reading of weighing machine g 1 − 
varies between 10.5 kgf and 89.5 kgf. T g′  ρ E = K + U = K max + U min . K is always
∴ = =
T′ positive, while U could be positive,
29 Here m = 2 kg; v = 360 = 6 rps g g
60 negative or zero. If U min is negative and
σ 1
Extension produced in spring, = 1− = 1− its value is greater than K max , then E
ρ ρ
y = 43 − 36 = 7 cm would be negative.
T 1 1 1
= 7 × 10− 2 m Given that, = ⇒ =1−
T′ 2 2 ρ 40 When the hoop oscillates in its plane,
On rotation the required centripetal
or ρ=2 moment of inertia is
force is provided by tension in spring
I1 = mR 2 + mR 2 i .e ., I1 = 2mR 2 .
i.e. ky = mr (2 πν)2 = 4 π2 ν2 mr 33 The body is dropped from a height h
and takes time t, then While when the hoop oscillates in a
4 π ν mr
2 2
⇒ k = 1
y h = ut + g pt 2 direction perpendicular to the plane of
2
where, r is length of stretched spring. 1 the hoop, moment of inertia is
h = g pt 2 (Q u = 0)
4 × (22 / 7)2 × 62 × 2 × (43 × 10− 2 ) 2 mR2 3mR2
⇒k = I2 = + mR2 =
7 × 10− 2 2h 2 × 8
gp = = = 4 ms− 2 2 2
= 17475 Nm − 1 . t2 4
The time period of physical pendulum,
l 1 2π
Time period, T = 2 π = 2π = I
30 Let l 1 and l 2 be the lengths of the two g 4 2 T = 2π .
mgd
pipes, then
= π = 314
. s Here, d is a same in both the cases.
EXAM BITES

This Pdf Is
Downloaded From
www.exambites.in

Visit www.exambites.in for


More Premium Stuffs,Latest
Books,Test Papers,Lectures etc.
jeeneetadda
jeeneetadda_official
jeeneetadda

VISIT NOW !!
DAY FOURTEEN

Properties
of Matter
Learning & Revision for the Day
u Elastic Behaviour u Thermal Stresses and Strains u Equation of Continuity
u Stress u Fluid Statics u Bernoulli’s Principle
u Strain u Pascal’s Law u Surface Tension
u Hooke’s Law u Laws of Floatation u Surface Energy
u Work Done (or Potential u Viscosity u Angle of Contact
Energy Stored) in a Stretched u Streamline and Turbulent u Excess Pressure Over a Liquid Film
Line Flow u Capillary Rise or Capillarity

Elastic Behaviour
Elasticity is the property of body by virtue of which a body regains or tends to regain its
original configuration (shape as well as size), when the external deforming forces acting
on it, is removed.

Stress
The internal restoring force per unit area of cross-section of the deformed body is called
stress.
PREP
Thus, Stress, σ =
Restoring force F
=
MIRROR
Area A Your Personal Preparation Indicator
−2
SI unit of stress is Nm or pascal (Pa). u No. of Questions in Exercises (x)—
Different types of stress are given below u No. of Questions Attempted (y)—
u No. of Correct Questions (z)—

1. Normal or Longitudinal Stress (Without referring Explanations)

If area of cross-section of a rod is A and a deforming force F is applied along the length of u Accuracy Level (z / y × 100)—
the rod and perpendicular to its cross-section, then stress produced in the rod is called u Prep Level (z / x × 100)—
normal or longitudinal stress.
F In order to expect good rank in JEE,
Longitudinal stress = n your Accuracy Level should be above
A 85 & Prep Level should be above 75.
152 40 DAYS ~ JEE MAIN PHYSICS DAY FOURTEEN

Longitudinal stress is of two types


Stress-Strain Relationship
(i) Tensile stress When length of the rod is increased on
application of deforming force over it, then stress For a solid, the graph between stress (either tensile or
produced in rod is called tensile stress. compressive) and normal strain is shown in figure.
(ii) Compressive stress When length of the rod is decreased Plastic region
Breaking D
on application of deforming force, then the stress
produced is called compressive stress. strength C
E
B (Fracture point)
2. Volumetric Stress Elastic limit
A
Proportional limit
When a force is applied on a body such that it produces a
change in volume and density, shape remaining same
(i) at any point, the force is perpendicular to its surface. Stress
O' Strain
(ii) at any small area, the magnitude of force is directly A typical stress-strain curve
proportional to its area.
l
In the above graph, point A is called proportional limit.
Then, force per unit area is called volumetric stress.
Till this point, stress and strain are proportional to each
F
∴ Volumetric stress = v other.
A
l
From point A to B, stress and strain are not proportional, B
3. Shearing or Tangential Stress is called elastic limit and OB is elastic region.

When the force is applied tangentially to a surface, then it is


l
Beyond point B, strain increases without increase in stress,
called tangential or shearing stress. it is called plastic behaviour. Region between point C and
F D is called plastic region.
Tangential stress = t
A
l
Finally, at point D, wire may break, maximum stress
corresponding to point D is called breaking stress.
It produces a change in shape, volume remaining same.
The materials of the wire, which break as soon as stress is
F increased beyond the elastic limit are called brittle.
Graphically, for such materials the portion of graph between B
and E is almost zero. While the materials of the wire, which
have a good plastic range (portion between B and E) are called
ductile.

Fixed face
An object under tangential deforming forces Hooke’s Law
According to the Hooke’s law, for any body, within the elastic
Strain limit, stress developed is directly proportional to the strain
produced.
Strain is the ratio of change in configuration to the original
configuration of the body. Stress ∝ Strain
Being the ratio of two similar quantities, strain is a unitless Stress = E × Strain
and dimensionless quantity. The ratio of stress to strain, within the elastic limit, is called
(i) When the deforming force causes a change in length, it is the coefficient (or modulus) of elasticity for the given
called longitudinal strain. For a wire or rod, longitudinal material.
strain is defined as the ratio of change in length to the Depending on the type of stress applied and resulting strain,
original length. we have the following three of elasticity given as,
Change in length (∆L) Stress
∴ Longitudinal strain = E =
Original length (L) Strain
There are three modulus of elasticity.
(ii) When the deforming force causes a change in volume, the
strain is called volumetric strain.
Change in volume (∆V ) 1. Young’s Modulus
Volumetric strain =
Original volume (V ) Young’s modulus of elasticity (Y ) is defined as the ratio of
normal stress (either tensile or compressive stress) to the
(iii) When the deforming force, applied tangentially to a
longitudinal strain within a elastic limit.
surface, produces a change in shape of the body, the strain
developed is called shearing strain or shear. Young’s modules,
Normal stress F/A FL
Shearing strain, φ =
x Y = = =
L Longitudinal strain ∆ L / L A∆ L
DAY FOURTEEN PROPERTIES OF MATTER 153

Energy stored per unit volume (or energy density)


2. Bulk Modulus
l

U 1 F∆ L 1 Y
It is defined as the ratio of the normal stress to the volumetric = = = stress × strain = (strain)2
V 2 AL 2 2
strain. Coefficient of volume elasticity,
F/A pV
B=
∆V / V
=−
∆V
Thermal Stresses and Strains
F When a body is allowed to expand or contract with increase or
where, p = = the pressure or stress negative sign signifies decrease in temperature, no stresses are induced in the body.
A
But if the deformation of the body is prevented, some stresses
that for an increase in pressure, the volume will decrease.
are induced in the body. Such stresses are called thermal
Reciprocal of bulk modulus is called compressibility. stresses or temperature stresses. The corresponding strains are
called thermal strains or temperature strains.
3. Modulus of Rigidity (Shear modulus)
l, a , Y, A
It is defined as the ratio of tangential stress to shearing stress. B A
Tangential stress F / A F FL ∆l
η= = = =
Shearing strain φ Aφ Ax A rod under thermal stress and strain
l
Breaking force depends upon the area of cross-section of ∆l
the wire. By definition, coefficient of linear expansion α =

∴ Breaking force ∝ A ∆l
Breaking force = P × A thermal strain = α ∆θ
l
Here, P is a constant of proportionality and known as So thermal stress = Yα ∆θ
breaking stress. Tensile or compressive force produced in the body
F = YAα ∆θ
Poisson’s Ratio
For a long bar, the Poisson’s ratio is defined as the ratio of
lateral strain to longitudinal strain. Fluid Statics
Lateral strain ∆ D/ D ∆ r / r The substances which flow are called fluids, that includes
∴ Poisson’s ratio, σ = = =
Longitudinal strain ∆ L / L ∆ L / L both liquid and gas. The science of fluids at rest is called fluid
statics where fluid mass is stationary w.r.t. container,
Poisson’s ratio is a unitless and dimensionless term. Its value containing the fluid.
depends on the nature of the material. Theoretically, value of
σ must lie between −1 and + 0.5 but for most metallic solids
0 < σ < 0.5. Thrust and Pressure
Normal force exerted by fluid at surface in contact is called
Inter-Relations Between Elastic Constants thrust of fluid.
Y = Young’s modulus, η = Rigidity modulus, The thrust exerted by a fluid at rest per unit surface area of
B = Bulk modulus, σ = Poisson’s ratio contact surface is called the fluid pressure.
The inter relation between elastic constants are Normal force (thrust) F
∴ Pressure p =
Y = 2 η (1 + σ), Y = 3 B (1 − 2 σ) Surface area A
9 3 1 9 Bη 3B − 2 η Pressure is a scalar and its SI unit is Nm−2 or pascal (Pa),
= + or Y = or σ =
Y η B η + 3B 6B + 2 η where, 1 Pa = 1 N m−2 .

Pressure due to Fluid Column


Work Done (or Potential Energy Pressure p exerted by the fluids at the bottom of container
having height h
Stored) in a Stretched Wire
p = hρ g
Work is done against the internal restoring forces, while
stretching a wire. This work is stored as elastic potential where, ρ = density of fluid.
energy. The work done is given by
1
Gauge Pressure
l
Work done W = × stretching force × elongation The pressure difference between the real hydrostatic pressure
2
1 1 YA and the atmospheric pressure is known as the gauge pressure.
= F∆ L = (∆ L)2
2 2 L ∴ Gauge pressure = Real pressure (p)
= Energy stored in the wire (U) − Atmospheric pressure ( p0)
154 40 DAYS ~ JEE MAIN PHYSICS DAY FOURTEEN

Case II The density of body is equal to the density of


Pascal’s Law liquid (i.e. ρ B = σ). In this case, W = F . so, the body
According to Pascal’s law of transmission of pressure, the will float fully submerged in neutral equilibrium
increase in pressure at any one point of the enclosed liquid in
anywhere in the liquid.
equilibrium or at rest, is transmitted equally to all other points
of the liquid and also to the walls of the container. Hydraulic Case III The density of body is less than that of liquid (i.e.
lift, hydraulic press, hydraulic brakes etc. are based on the ρ B < σ). In this case, W < F , so the body will move
Pascal’s law.
upwards and in equilibrium will float partially
immersed in the liquid such that
Archimedes’ Principle and Buoyancy
W = Vinσg
When a body is immersed in a fluid, it experiences an upthrust
due to the fluid and as a result the apparent weight of the body [Vin is the volume of body in the liquid]
is reduced. or Vρ B g = Vinρg [as, W = mg = ρ BVg]
∴Apparent weight of the body
or Vρ B = Vinσ …(i)
= weight of the body – upthrust due to fluid
= weight of the body – weight of the fluid displaced
e.g. For a floating body, the volume of a body (V − Vs ) remaining Viscosity
outside the liquid will be given by Viscosity is the property of a fluid due to which it opposes
ρ  ρ the relative motion between its different layers.
V0 = V − Vs = V − V = V 1 − 
σ  σ Force between the layers opposing the relative motion is
where, ρ = density of liquid called viscous force.
and σ = density of body immersed in liquid If there are two fluid layers having surface area A and
velocity gradient dv/dr , then the viscous force is given by
Buoyant Force or Buoyancy dv
Buoyant force, F = hρgA = mg F = −ηA . where, constant η is called the coefficient of
dr
where, h = height of body immersed in liquid, viscosity of the given fluid.
m = mass of body and A = area SI unit of coefficient of viscosity is N m–2s or Pa-s or
l
It is an upward force acting on the body immersed in a poiseuille (Pl).
liquid.
l
It is equal to the weight of liquid displaced by the immersed Terminal Velocity
part of the body.
If a small spherical body is dropped in a fluid, then initially it
l
The buoyant force acts at the centre of buoyancy which is is accelerated under the action of gravity. However, with an
the centre of gravity of the liquid displaced by the body increase in speed, the viscous force increases and soon it
when immersed in the liquid. balances the weight of the body.
l
The line joining the centre of gravity and centre of buoyancy Now, the body moves with a constant velocity, called the
is called central line. terminal velocity.
l
Metacentre, is a point where the vertical line passing through Terminal velocity vt is given by
the centre of buoyancy intersects the central line.
2 r 2 (ρ − σ)g
vt =
9 η
Laws of Floatation where, r = radius of the falling body,
When a body of density ρ B and volume V is immersed in a ρ = density of the falling body
liquid of density σ, the forces acting on the body are
and σ = density of the fluid.
l
The weight of body W = mg = Vρ B g acting vertically
downwards through the centre of gravity of the body.
l
The upthrust F = Vσg acting vertically upwards through the
Stokes’ Law
centre of gravity of the displaced liquid i.e., centre of Stokes proved that for a small spherical body of radius r
buoyancy. moving with a constant speed v called terminal velocity
So, the following three cases are possible. through a fluid having coefficient of viscosity η, the viscous
force F is given by
Case I The density of body is greater than that of liquid (i.e.
ρ B > σ). In this case, as weight will be more than F = 6πηrv
upthrust, the body will sink. (W > F ) It is known as Stokes’ law.
DAY FOURTEEN PROPERTIES OF MATTER 155

Streamline and Turbulent Flow Energy of a Flowing Liquid


Flow of a fluid is said to be streamlined if each element of the There are three types of energies in a flowing liquid.
fluid passing through a particular point travels along the same l
Pressure Energy If p is the pressure on the area A of a
path, with exactly the same velocity as that of the preceding
fluid and the liquid moves through a distance l due to
element. A special case of streamline flow is laminar flow.
this pressure, then
A turbulent flow is the one in which the motion of the fluid
particles is disordered or irregular. Pressure energy of liquid = work done
= force × displacement = pAl
Critical Velocity The volume of the liquid is Al. Hence, pressure energy
pAl
For a fluid, the critical velocity is that limiting velocity of the per unit volume of liquid = =p
Al
fluid flow upto which the flow is streamlined and beyond which
the flow becomes turbulent. Value of critical velocity for the l
Kinetic Energy If a liquid of mass m and volume V is
flow of liquid of density ρ and coefficient of viscosity η, flowing 1
flowing with velocity v, then the kinetic energy = mv2
through a horizontal tube of radius r is given by 2
η ∴ Kinetic energy per unit volume of liquid
vc ∝
ρr 1  m 1
=   v2 = ρv2
where, r = radius of tube. 2 V  2
Here, ρ is the density of liquid and V = volume.
Reynold’s Number (N R ) l
Potential Energy If a liquid of mass m is at a height h
Reynold’s number as the ratio of the inertial force per unit area from the reference line (h = 0), then its potential energy is
to the viscous force per unit area for a fluid. mgh.
v2ρ ρvr ∴ Potential energy per unit volume of the liquid
NR = =
ηv / r η  m
=   gh = ρgh
A smaller value of Reynold’s number (generally N R ≤ 1000) V 
indicates a streamline flow but a higher value (N R ≥ 1500)
indicates that the flow is turbulent and between 1000 to 1500, Bernoulli’s Principle
the flow is unstable. According to the Bernoulli’s principle for steady flow of an
incompressible, non-viscous fluid through a tube/pipe, the
total energy (i.e. the sum of kinetic energy, potential energy
Equation of Continuity and pressure energy) per unit volume (or per unit mass too)
Let us consider the streamline flow of an ideal, non-viscous fluid remains constant at all points of flow provided that there is
through a tube of variable cross-section. no source or sink of the fluid along the flow.
Let at the two sections, the cross-sectional areas be A1 and A2 , Mathematically, we have
respectively and the fluid flow velocities are v1 and v2 , then 1 p v2
according to the equation of continuity p + ρgh + ρv2 = constant or + h+ = constant
2 ρg 2g
A1 v1 ρ1 = A 2 v2 ρ2
v2 p
where, ρ1 and ρ2 are the respective densities of the fluid. In this expression, is velocity head and is pressure
2g ρg
Equation of continuity is based on the conservation of mass.
head.
Q
P
v2 Velocity of Efflux
v1 l
If a liquid is filled in a vessel up to a height H and a small
A2
orifice O is made at a height h, then from Bernoulli’s
theorem it can be shown that velocity of efflux v of the
A1
liquid from the vessel is
A liquid is flowing through a tube of non-uniform
cross-section v = 2 g (H − h)
l
The flowing fluid describes a parabolic path and hits the
If the fluid which is flowing, is incompressible, then
base level at a horizontal distance (called the range)
ρ1 = ρ2 R = 2 h (H − h).
So, equation of continuity is simplified as
H
A1 v1 = A2 v2 or Av = constant The range is maximum, when h = and in that case
2
Rmax = H .
156 40 DAYS ~ JEE MAIN PHYSICS DAY FOURTEEN

Coalescence of Drops If n small liquid drops of radius r


Applications Based on the Bernoulli’s
l

each, combine together so as to form a single bigger drop of


Principle radius R = n1 /3 r then in the process, energy is released.
Release of energy is given by
l
The action of carburetor, paintgun, scent sprayer, atomiser
and insect sprayer is based on the Bernoulli’s principle. ∆U = S × 4π (nr 2 − R2 ) = S × 4πr 2 n (1 − n−1 /3 )
l
The action of the Bunsen’s burner, gas burner, oil stove and
exhaust pump is also based on the Bernoulli’s principle. Angle of Contact
l
Motion of a spinning ball (Magnus effect) is based on Angle of contact for a given liquid-solid combination is
Bernoulli’s theorem. defined as the angle subtended between the tangents to the
liquid surface and the solid surface, inside the liquid, the
l
Blowing of roofs by wind storms, attraction between two tangents are drawn at the point of contact.
parallel moving boats moving close to each other, fluttering Angle of Angle
of a flag etc., are also based on Bernoulli’s theorem. contact of
contact θ
θ
Surface Tension
Beaker Beaker
Surface tension is the property of a liquid due to which its free
surface behaves like a stretched elastic membrane and tends
to have the least possible surface area. Shape of meniscus for different media
Force F
Surface tension S = = l
Value of the angle of contact depends on the nature of
Length l
liquid and solid both.
Here, F is force acting on the unit length of an imaginary line
drawn of the surface of the liquid.
l
For a liquid having concave meniscus when adhesive
force > cohesive force angle of contact θ is acute (θ < 90 ° )
SI unit of surface tension is Nm−1 or Jm−2 . It is a scalar and its but for a convex meniscus (when cohesive force >
dimensional formula is [MT−2 ]. adhesive force) the angle of contact is obtuse (θ > 90 ° ).
l
Value of angle of contact θ decreases with an increase in
temperature.
Surface Energy
Surface energy of a liquid is the potential energy of the Excess Pressure Over a Liquid Film
molecules of a surface film of the liquid by virtue of its
position. If a free liquid surface film is plane, then pressure on the
liquid and the vapour sides of the film are the same, otherwise
When the surface area of a liquid is increased, work is done there is always some pressure difference. Following cases
against the cohesive force of molecules and this work is stored arise.
in the form of additional surface energy.
l
For a spherical liquid drop of radius r, the excess pressure
Increase in surface potential energy 2S
inside the drop p =
∆U = Work done (∆W ) = S∆ A r
where, ∆ A is the increase in surface area of the liquid. where, S = surface tension of the liquid.
Work done in Blowing a Liquid Drop If a liquid drop is 2S
l
l
For an air bubble in a liquid, excess pressure p =
blown up from a radius r1 to r2 , then work done in the r
process, 4S
l
For a soap bubble in air, excess pressure p =
W = S ( A2 − A1 ) = S × 4π (r22 − r12 ) r
Work done in Blowing a Soap Bubble As a soap bubble
l

has two free surfaces, hence, work done in blowing a soap


Capillary Rise or Capillarity
bubble, so as to increase its radius from r1 to r2 , is given by Capillarity is the phenomenon of rise or fall of a liquid in a
capillary tube as compared to that in a surrounding liquid.
W = S × 8π (r22 − r12 )
The height h up to which a liquid will rise in a capillary tube
l
Work done in Splitting a Bigger Drop into n Smaller is given by
Droplets If a liquid drop of radius R is split up into n 2S cos θ 2S
smaller droplets, all of the same size, then radius of each h= =
rρg Rρg
droplet
where, r = radius of the capillary tube
r = R(n)−1 /3 r
and R = = radius of liquid meniscus.
and work done cosθ
W = S × 4π (nr 2 − R2 ) = S × 4πR2 (n1 /3 − 1) 1
The rise in capillary tube h ∝ (Jurin’s law).
r
DAY FOURTEEN PROPERTIES OF MATTER 157

l
If a capillary tube, dipped in a liquid is tilted at an angle α
h
from the vertical, the vertical height h of the liquid column l= .
remains the same. However, the length of the liquid cos α
column (l ) in the capillary tube increases to l
If the capillary tube is of insufficient length, the liquid rises
up to the upper end of the tube and then the radius of its
meniscus changes from R to R′ such that hR = h ′ R ′, where
h h′ = insufficient length of the tube.
α l
l
After connection due to the weight of liquid contained in
the meniscus, the formula for the height is given by
2s r
h= −
Beaker ρrg 3
This is known as ascent formula.
Effect of tilting capillary tube in a liquid

DAY PRACTICE SESSION 1

FOUNDATION QUESTIONS EXERCISE


1 A man grows into a giant such that his linear dimensions 5 The length of a metal wire is l1 when the tension in it isT1
increase by a factor of 9. Assuming that his density and is l2 when the tension isT2. The original length of
remains same, the stress in the leg will change by a the wire is
factor of ª JEE Main 2017 (Offline) l1 + l 2 l1T2 + l 2T1
(a) (b)
(a)
1
(b) 81 2 T1 + T2
9 l1T2 − l 2T1
(c) (d) T1T2l1l 2
1 T2 − T1
(c) (d) 9
81
6 Two wires are made of the same material and have the
2 The Young’s modulus of brass and steel are respectively same volume. However, wire 1 has cross-sectional area A
1.0 × 1011 Nm −2 and 2.0 × 1011 Nm −2 . A brass wire and a and wire 2 has cross-sectional area 3A. If the length of
steel wire of the same length extend by 1 mm, each wire 1 increases by ∆x on applying force F, how much
under the same force. If radii of brass and steel wires are
force is needed to stretch wire 2 by the same amount?
RB and RS respectively, then
ª AIEEE 2009
RB
(a) RS = 2 RB (b) RS = (a) F (b) 4F (c) 6F (d) 9 F
2
R 7 If the ratio of lengths, radii and Young’s modulii of steel
(c) RS = 4RB (d) RS = B
2 and brass wires in the figure are a, b and c respectively,
3 One end of a horizontal thick copper wire of length 2L then the corresponding ratio of increase in their lengths is
and radius 2R is welded to an end of another horizontal ª JEE Main (Online) 2013
thin copper wire of length L and radius R. When the
arrangement is stretched by applying forces at two ends, Steel
the ratio of the elongation in the thin wire to that in the thick
wire is M
(a) 0.25 (b) 0.50
(c) 2.00 (d) 4.00 Brass
4 The following four wires are made of the same material.
Which of these will have the largest extension when the
same tension is applied ? 2M

(a) Length = 50 cm, diameter = 0.5 mm


(b) Length = 100 cm, diameter = 1mm
3c 2a 2c 3a 2ac
(c) Length = 200 cm, diameter = 2 mm (a) 2
(b) (c) (d)
2ab b 2b 2c b2
(d) Length = 300 cm, diameter = 3 mm
158 40 DAYS ~ JEE MAIN PHYSICS DAY FOURTEEN

8 The pressure of a medium is changed from1.01 × 105 Pa 14 A metal rod of Young’s modulus Y and coefficient of
to 1.165 × 105 Pa and change in volume is 10% keeping thermal expansion α is held at its two ends such that its
temperature constant. The bulk modulus of the medium length remains invariant. If its temperature is raised by
is t °C, the linear stress developed in it is ª AIEEE 2011
(a) 204.8 × 105 Pa (b) 102.4 × 105 Pa αt Y 1
(a) (b) (c)Yα t (d)
(c) 51.2 × 105 Pa (d) 1.55 × 105 Pa Y αt Yα t
9 A solid sphere of radius r made of a soft material of bulk 15 A wooden block of mass m and density ρ is tied to a
modulus K is surrounded by a liquid in a cylindrical string, the other end of the string is fixed to the bottom of
container. A massless piston of area a floats on the a tank. The tank is filled with a liquid of density σ with
surface of the liquid, covering entire cross-section of σ > ρ. The tension in the string will be
cylindrical container. When a mass m is placed on the σ − ρ  σ − ρ
(a)   mg (b)   mg
surface of the piston to compress the liquid, the  σ   ρ 
 dr  ρ mg σ mg
fractional decrement in the radius of the sphere,   is (c) (d)
 r  σ ρ
ª JEE Main 2018
16 A ball is made of a material of density ρ where
Ka Ka
(a) (b) ρoil < ρ < ρ water with ρoil and ρ water representing the densities
mg 3mg
mg mg of oil and water, respectively. The oil and water are
(c) (d) immiscible. If the above ball is in equilibrium in a mixture of
3Ka Ka
this oil and water, which of the following pictures
10 The graph shows the behaviour of a Q
represents its equilibrium position? ª AIEEE 2010
length of wire in the region for which
the substance obeys Hooke’s law.
P and Q represent Water Oil

(a) P = applied force, Q = extension P (a) (b)


(b) P = extension, Q = applied force Oil Water
(c) P = extension, Q = stored elastic energy
(d) P = stored elastic energy, Q = extension
11 If work done in stretching a wire by 1mm is 2 J. The work
necessary for stretching another wire of same material Water Oil
but with double the radius and half the length, by 1 mm
(c) (d)
distance, is
(a) 16 J (b) 4 J Oil Water
(c) 1/4 J (d) 8 J
12 Two rods of different materials having coefficients of 17 Two mercury drops (each of radius r ) merge to form a
thermal expansion α 1, α 2 and Young’s moduliiY1, Y2 bigger drop. The surface energy of the bigger drop, if T is
respectively are fixed between two rigid massive walls. the surface tension, is ª AIEEE 2011
The rods are heated such that they undergo the same (a) 2 5/ 3 π 2T (b) 4 π 2T
increase in temperature. There is no bending of the rods. (c) 2 πr 2T (d) 2 8 / 3 πr 2T
If α 1 : α 2 = 2 : 3, the thermal stresses developed in the two
18 A raindrop of radius 0.2 cm is falling through air with a
rods are equal providedY1 : Y2 is equal to
terminal velocity of 8.7 m/s. The viscosity of air in SI units
(a) 2 : 3 (b) 1 : 1
is (take, ρ water = 1000 kg/m 3 and ρ air = 1 kg/m 3).
(c) 3 : 2 (d) 4 : 9
(a) 10−4 poise (b) 1 × 10−3 poise
13 The pressure that has to be applied to the ends of a steel (c) 8.6 × 10−3 poise (d) 1.02 × 10−3 poise
wire of length 10 cm to keep its length constant when its
temperature is raised by 100°C is 19 If a ball of steel (density ρ = 7.8 g cm −3 ) attains a terminal
(For steel, Young’s modulus is 2 × 1011Nm −2 and velocity of 10 cms −1 when falling in a tank of water
coefficient of thermal expansion is 1.1 × 10 −5 K −1) (coefficient of viscosity ηwater = 8.5 × 10−4 Pa-s) then its
ª JEE Main 2014 terminal velocity in glycerine (ρ = 1.2 gcm −3,
η = 13.2 Pa-s) would be nearly ª AIEEE 2011
(a) 2 . 2 × 10 Pa
8
(b) 2 . 2 × 10 Pa
9

(c) 2 . 2 × 107 Pa (d) 2 . 2 × 106 Pa . × 10−5 cms −1


(a) 16 (b) 6.25 × 10−4 cms −1
(c) 6.45 × 10−4 cms −1 . × 10−5 cms −1
(d) 15
DAY FOURTEEN PROPERTIES OF MATTER 159

20 Water is flowing continuously from a tap having an tube is found to be 0.5 cm. Then, compute tension of
internal diameter 8 × 10−3 m. The water velocity as it water using this data (take, contact angle between glass
leaves the tap is 0.4 ms −1. The diameter of the water and water as 0 and g = 9.81 m/s 2).
stream at a distance 2 × 10−1 m below the tap is close to (a) 0.72 N/m
ª AIEEE 2011 (b) 0.77 N/m
(a) 7.5 × 10−3 m (b) 9.6 × 10−3 m (c) 1.67 N/m
(c) 3.6 × 10−3 m (d) 5.0 × 10−3 m (d) None of the above

21 At what speed, the velocity head of water is equal to 26 Match the following columns.
pressure head of 40 cm of Hg? Column I Column II
(a) 10.3 ms −1 (b) 2.8 ms −1 A. Magnus energy 1. Pascal’s law
(c) 5.6 ms −1 (d) 8.4 ms −1 B. Loss of energy 2. Bernoulli’s principle
22 A liquid X of density 3.36 g/cm 3 is poured in a U-tube, C. Pressure is same at the 3. Viscous force
same level in a liquid
which contains Hg. Another liquid Y is poured in the left
D. Gas burner 4. Spinning ball
arm with height 8 cm and upper levels of X and Y are
same. What is the density of Y ? Codes
Y X A B C D A B C D
8 cm (a) 1 4 2 3 (b) 1 2 3 4
10 cm
2 cm (c) 2 2 4 3 (d) 4 3 1 2
P P¢

Direction (Q. Nos. 27-30) Each of these questions


contains two statements : Statement I and Statement II. Each
of these questions also has four alternative choices, only one of
Hg
which is the correct answer. You have to select one of the codes
(a), (b), (c), (d) given below
(a) 0.8 g /cc (b) 1.2 g /cc (c) 1.4 g /cc (d) 1.6 g /cc (a) Statement I is true, Statement II is true; Statement II is
the correct explanation for Statement I
23 A thin liquid film formed between a U-shaped wire and a
(b) Statement I is true, Statement II is true; Statement II is
light slider supports a weight of 1.5 × 10−2N (see figure).
not the correct explanation for Statement I
The length of the slider is 30 cm and its weight
(c) Statement I is true; Statement II is false
negligible. The surface tension of the liquid film is
(d) Statement I is false; Statement II is true
27 Statement I Aeroplanes are made to run on the runway
before take off, so that they acquire the necessary lift.
Film
Statement II This is as per the Bernoulli’s theorem.
28 Statement I Finer the capillary, greater is the height to
which the liquid rises in the tube.
w ª AIEEE 2012 Statement II This is in accordance with the ascent
(a) 0.0125 Nm −1 (b) 0.1 Nm −1 formula.
(c) 0.05 Nm −1 (d) 0.025 Nm −1
29 Statement I The bridges are declared unsafe after a long
24 Work done in increasing the size of a soap bubble from use.
radius of 3 cm to 5 cm is nearly (surface tension of soap
Statement II Elastic strength of bridges decreases with
solution = 0.03 Nm −1) ª AIEEE 2011 time.
(a) 0. 2 π mJ (b) 2 π mJ (c) 0.4 π mJ (d) 4 π mJ
30 Statement I A small drop of mercury is spherical but
25 While measuring surface tension of water using capillary bigger drops are oval in shape.
rise method, height of the lower meniscus from free
Statement II Surface tension of liquid decreases with an
surface of water is 3 cm while inner radius of capillary
increase in temperature.
160 40 DAYS ~ JEE MAIN PHYSICS DAY FOURTEEN

DAY PRACTICE SESSION 2

PROGRESSIVE QUESTIONS EXERCISE


1 A light rod of length L is A B 6 The free surface of oil in a tanker, at rest, is horizontal. If
suspended from a support the tanker starts accelerating the free surface will be
horizontally by means of two tilted by an angle θ. If the acceleration is a ms −2, then the
vertical wires A and B of equal slope of the free surface is
lengths as shown in the figure. L (a) g/a (b) a/g (c) ag (d) a 2 /g
Cross-section area of A is half 7 There is a circular tube in a vertical
that of B and Young’s modulus x plane. Two liquids which do not mix
of A is double than that of B. A and of densities d1 and d 2 are filled in
w
weight w is hung on the rod as the tube. Each liquid subtends 90° a d2
shown. The value of x, so that the angle at centre. Radius joining their
stress in A is same as that in B, is interface makes an angle α with d1
L L 2L 3L vertical. Ratio d1 / d 2 is ª JEE Main 2014
(a) (b) (c) (d)
3 2 3 4 1 + sinα 1 + cosα 1 + tanα 1 + sinα
(a) (b) (c) (d)
2 A pendulum made of a uniform wire of cross-sectional 1 − sinα 1 − cosα 1 − tanα 1 − cosα
area A has time period T. When an additional mass M is 8 On heating water, bubbles being formed at
added to its bob, the time period changes to TM . If the the bottom of the vessel detach and rise.
1 Take the bubbles to be spheres of radius R
Young’s modulus of the material of the wire is Y, then
Y and making a circular contact of radius r R
is equal to (take, g = gravitational acceleration) with the bottom of the vessel. If r <<R and
ª JEE Main 2015 the surface tension of water is T, value of r 2r
 T 2   T  2  Mg
(a)   M  − 1
A just before bubbles detach is (density of
(b)   M  − 1
  T   Mg   T   A water is ρ) ª JEE Main 2014

 2  T   A
2
2ρw g ρ g ρw g
(c) 1 −  M  
T A 3ρw g
(d) 1 −    (a) R 2 (b) R 2 w (c) R 2 (d) R 2
  T   Mg   TM   Mg 3T 6T T T
 
9 An open glass tube is immersed in mercury in such a way
3 The length of an elastic string is 1m, when the
that a length of 8 cm extends above the mercury level. The
longitudinal tension is 4 N and the length is b metres, when
open end of the tube is then closed and sealed and the
the tension is 5 N. The length of the string (in metre) tube is raised vertically up by additional 46 cm. What will be
when the longitudinal tension is 9 N is length of the air column above mercury in the tube now?
a (Atmospheric pressure = 76 cm of Hg]
(a) 2 b − (b) 5 b − 4a (c) 4 a − 3 b (d) a − b ª JEE Main 2014
2
(a) 16 cm (b) 22 cm (c) 38 cm (d) 6 cm
4 A rectangular frame is to be suspended symmetrically by 10 Two narrow bores of diameter 3.0 mm and 6.0 mm are
two strings of equal length on two supports (figure). It
joined together to form a U-tube open at both ends. If the
can be done in one of the following three ways:
U-tube contains water, what is the difference in its levels
in the two limbs of the tube? Surface tension of water at
the temperature of the experiment is 7.3 × 10−2 N/m. Take
the angle of contact to be zero and density of water to be
1.0 × 103 kg/m 3.( take, g = 9.8 m/s 2).
(a) (b) (c)
(a) 6 mm (b) 2 mm (c) 5 mm (d) 3 mm
The tension in the strings will be
11 A long metal rod of length l and relative density σ, is
(a) the same in all cases (b) least in (a)
held vertically with its lower end just touching the surface
(c) least in (b) (d) least in (c)
of water. The speed of the rod when it just sinks in water,
5 To what depth must a rubber ball be taken in deep sea, is given by
so that its volume is decreased by 0.1%. (The bulk
(a) 2glσ (b) 2gl (2 σ − 1)
modulus of rubber is 9.8 × 108 N/m 2; and the density of
(c) 2gl  1 −
1 
sea water is 103 kg/m 3.)  (d) 2gl
 2σ 
(a) 100 m (b) 60 m (c) 75 m (d) 65 m
DAY FOURTEEN PROPERTIES OF MATTER 161

12 A wooden wheel of radius R is made of fractional decrease in the radius of the wire is close to
two semi-circular parts (see figure). The ª JEE Main (Online) 2013
two parts are held together by a ring R (a) 1.0 × 10 −4
(b) 1.5 × 10−4
(c) 0.17 × 10−4 (d) 5 × 10−4
made of a metal strip of cross-sectional
area S and length L. L is slightly less 17 A copper wire of length 1.0 m and a steel wire of length
than 2πR. To fit the ring on the wheel, it 0.5 m having equal cross-sectional areas are joined end
is heated so that its temperature rises by to end. The composite wire is stretched by a certain load
∆T and it just steps over the wheel. As it cools down to which stretches the copper wire by 1 mm.
surrounding temperature, it presses the semi-circular If the Young’s modulii of copper and steel are respectively
parts together. If the coefficient of linear expansion of the 1.0 × 1011 Nm −2 and 2.0 × 1011 Nm −2, the total extension of
metal is α and its Young’s modulus is Y, the force that
the composite wire is ª JEE Main (Online) 2013
one part of the wheel applies on the other part is
ª AIEEE 2012 (a) 1.75 mm (b) 2.0 mm (c) 1.50 mm (d) 1.25 mm
(a) 2 πSYα∆T (b) SYα∆T (c) πSYα∆T (d) 2SYα∆T 18 A wire of mass m, and length l is suspended from a
13 A sonometer wire of length 1.5 m is made of steel. The ceiling. Due to its own weight it elongates, consider
tension in it produces an elastic strain of 1%. What is the cross-section area of wire as A and Young’s modulus of
fundamental frequency of steel, if density and elasticity material of wire as Y . The elongation in the wire is
of steel are 7.7 × 103 kg/m 3 and 2 . 2 × 1011 N /m 2, (a)
2mg
(b)
mgl
(c)
mgl
(d)
3mg
respectively? ª JEE Main 2013 3YA YA 2YA YAl
(a) 188.5 Hz (b) 178.2 Hz (c) 200.5 Hz (d) 770 Hz
Direction (Q. Nos. 19-20) Each of these questions
14 A wire of length 2L and radius 2L contains two statements : Statement I and Statement II.
A C B
r is stretched and clamped Each of these questions also has four alternative choices, only
between A and B. If the L L l one of which is the correct answer. You have to select one of
Young’s modulus of the the codes (a), (b), (c), (d) given below
L2 + l 2
material of the wire be Y , (a) Statement I is true, Statement II is true; Statement II is
D
tension in the wire, when the correct explanation for Statement I
stretched in the position ADB will be (b) Statement I is true, Statement II is true; Statement II is
(a) πr 2YLt (b) πr 2Yl 2 / 2L2 not the correct explanation for Statement I
(c) 2 πr 2YL 2 /l 2 (d) None of these (c) Statement I is true; Statement II is false
15 A uniform cylinder of length L and mass M having (d) Statement I is false; Statement II is true
cross-sectional area A is suspended, with its length 19 Statement I In taking into account the fact that any
veritcal from a fixed point by a massless spring such that object which floats must have an average density less
it is half submerged in a liquid of density σ at equilibrium than that of water, during world war-I, a number of cargo
position. The extenstion x 0 of the spring when it is in vessels were made of concrete.
equilibrium is ª JEE Main 2013 Statement II Concrete cargo vessels were filled with air.
Mg Mg  1 − LAσ 
(a) (b)   20 Statement I The stream of water flowing at high speed
k k  M 
from a garden hosepipe tends to spread like a fountain
Mg  1 − LAσ  Mg  1 + LAσ 
(c)   (d)   when held vertically up, but tends to narrow down when
k  2M  k  M 
held vertically down.
16 A uniform wire (Young's modulus 2 × 1011 Nm −2 ) is Statement II In any steady flow of an incompressible
subjected to longitudinal tensile stress of 5 × 107 Nm −2. If fluid, the volume flow rate of the fluid remains constant.
the overall volume change in the wire is 0.02%, the

ANSWERS

SESSION 1 1 (d) 2 (b) 3 (c) 4 (a) 5 (c) 6 (d) 7 (c) 8 (d) 9 (c) 10 (c)
11 (a) 12 (c) 13 (a) 14 (c) 15 (b) 16 (b) 17 (d) 18 (b) 19 (b) 20 (c)
21 (b) 22 (a) 23 (d) 24 (c) 25 (b) 26 (d) 27 (a) 28 (a) 29 (a) 30 (b)

SESSION 2 1 (c) 2 (a) 3 (b) 4 (c) 5 (a) 6 (b) 7 (c) 8 (a) 9 (a) 10 (c)
11 (c) 12 (d) 13 (b) 14 (b) 15 (c) 16 (c) 17 (d) 18 (c) 19 (b) 20 (a)
162 40 DAYS ~ JEE MAIN PHYSICS DAY FOURTEEN

Hints and Explanations


SESSION 1 ⇒
l1
=3 11 As work done
Weight l2 1
1 Q Stress = F = Y × (strain)2 × volume
Area As, ∆x1 = 1 × l1 …(i) 2
Aγ 2
Volume will become (93 ) times. ∆L 
× Y × 
1
F ⇒ 2=  × AL
So weight = volume × density × g will and ∆x2 = 2 l2 …(ii) 2  L 
3 Aγ
also become (9)3 times. YA(∆L )2
2 Here, ∆x1 = ∆x2 =
Area of cross-section will become (9) F2 F 2L
times ∴ l2 = 1 l1 Now, work done,
3 Aγ Aγ
9 × W0
3
W  Y (4 A )(∆L )2
= = 9 0 l W′ =
9 × A0
2
 A 0 ⇒ F2 = 3F1 × 1 = 3F1 × 3 = 9F 2(L / 2)
l2
Hence, the stress increases by a factor  YA (∆L )2 
of 9. 7 For steel wire, = 8 
F × Ls 3 MgL s  2L 
FL FL ∆ ls = =
2 ∆L = = Ys × A s Ys × πr s2 = 8 × 2 = 16 J
Y A Yπ R2
[Q Mass (steel) = 2M + M = 3M ]
As F , L and ∆L are the same, hence 12 Thermal stress σ = Y α ∆θ
For brass wire,
YR 2 = constant F × lb 2MgLb Given, σ1 = σ2
∆ lb = =
∴ 2.0 × 1011 R S2 = 1.0 × 1011 R B2 Yb × Ab Yb × πrb2 ∴ Y1α1 ∆θ = Y2α2 ∆θ
R [Q Mass (brass) = 2M ] Y1 α2 3
⇒ RS = B or = =
2 ∆ ls 3Mg L s Y × πrb2 Y2 α1 2
⇒ = × b
3 ∆l = FL = FL ∆ lb Ys × π r s
2
2MgLb
AY ( πr 2 )Y 2 13 If the deformation is small, then the
3 L s Yb  rb  stress in a body is directly proportional
L = × × × 
∴ ∆l ∝ 2 2 Lb Ys  r s  to the corresponding strain.
r
3a According to Hooke’s law, i.e.
∆l 1 L /R 2 =
∴ = =2 2b 2c Young’s modulus
∆l 2 2L /(2R )2 Tensile stress
(Y ) =
8 From the definition of bulk modulus, Tensile strain
4 ∆l = Fl = Fl
or (∆l ) ∝
l
B =
− dp F/A
AY  πd  2
d 2
So, Y =
 Y (dV /V ) ∆ L/L
 4  Substituting the values, we have FL
l =
Now, is maximum in option (a). (1.165 − 1.01) × 105 A∆L
B = = 1.55 × 105 Pa
d2 (10 / 100) If the rod is compressed, then
5 Let l = original length of the wire, 9 Q Bulk modulus, compressive stress and strain appear.
∆l 1 = l 1 − l Volumetric stress ∆p Their ratio Y is same as that for tensile
K = = case.
Similarly, the change in length of the Volumetric strain ∆V
second wire is Given, length of a steel wire (L)
V
∆l 2 = l 2 − l = 10 cm
mg
T l T l ⇒ K = Temperature (θ ) = 100° C
3∆r 
Now, Y = 1 × = 2 × a  As length is constant.
A ∆l 1 A ∆l 2  r 
∆L
T1 T Q V = 4 πr 3 , so ∆V = 3∆r  ∴ Strain = = α ∆θ
⇒ = 2 L
∆l 1 ∆l 2  3 V r  Now, pressure = stress = Y × strain
T1 T2 ∆r mg
⇒ = ⇒ = [Given, Y = 2 × 1011 N / m2 and
l1 − l l2 − l r 3aK
α = 1.1 × 10−5 K −1 ]
⇒ T1 l 2 − T1 l = T2 l 1 − T2 l
10 If Hooke’s law is being obeyed, then = 2 × 10 × 1.1 × 10
11 −5
× 100
T2 l 1 − T1 l 2 force extension graph is a straight line.
⇒ l = = 2.2 × 108 Pa
T2 − T1 Stored elastic energy extension
 U = 1 Fx = 1 YA x2  should be a 14 Change in length ∆L = αL ∆T = FL
6 As volume is same.  
 2 2 L  AY
∴ A1 l1 = A2 l2 parabolic curve symmetric about the U F
A l A × l1 l ⇒ Stress = = Yα∆T = Y α t
⇒ l2 = 1 1 = = 1 axis. Hence, in the graph P represents A
A2 3A 3 extension and Q the stored elastic (as, ∆T = t )
energy.
DAY FOURTEEN PROPERTIES OF MATTER 163

r  h +  ρg
4 r
15 From free body diagram of the wooden  D1  2gh
⇒   =1+  3 
block,  D2  v 12 25 As, T =
Vσg = mg + T 2 cos θ
D1
[ V is the volume of block] ⇒ D2 =
0.5 × 10−2 3 +
1/4 0.5
 2gh  × 10−2
1 + 2   3 
 v1 
× 103 × 9.81
8 × 10−3 =
T = 1/4 2
 . 
2 × 10 × 02
vs g 1 +  = 077
. N/m
s  (0.4)2 
mg 26 (A) When a spinning ball is thrown it
= 3.6 × 10−3 m
deviates from its ususal path in flight.
2 This is due to magnus effect.
T = Vσg − mg 21 Velocity head = v
2g (B) Viscous forces tends to reduce the
m  σ − ρ speed of flowing fluid by virtue of
T = σg − mg = mg   and pressure head =
p
ρ  ρ  ρg
internal frictional force. Hence, the
energy of liquid flow reduced.
16 Because ρoil < ρ < ρwater As velocity of water is equal to the
(C) Pascal’s law states that if gravity
pressure head of 40 cm of Hg column,
effect is neglected, the pressure of every
So, it will sink through oil but will not hence
point of liquid in equilibrium of rest is
sink in water. So, it will stay at the v2 hρ g
oil-water interface. = the same.
2g ρg (D) Gas burners are based on Bernoulli’s
17 Let R be the radius of the bigger drop, ⇒ v 2 = 2 hg principle.
then volume of bigger drop ⇒ v = 2 gh
= 2 × volume of small drop 27 The shape of the aeroplane wings is
4 4 = 2 × 9.8 × 0.4 peculier i.e. its upper surface is more
πR3 = 2 × πr 3 ⇒ R = 21 /3 r
3 3 = 2.8 ms −1 curved than its lower surface. Due to
this, when the aeroplane runs on runway
Surface energy of bigger drop, 22 As shown in the figure, in the two arms the speed of air above the wings is larger
E = 4 πR2T = 4 × 22 /3 πr 2T of the tube the pressure remains the
than the speed below the wings. Thus,
= 28/3 πr 2T same on the surface PP ′.
according to Bernoulli’s theorem, the
Hence,
18 We have, pressure above wings becomes lesser
8 × ρ Y × g + 2 × ρ Hg × g = 10 × ρx × g than the pressure below the wings. Due
4 3 4
6 πηrv = πr gρ − πr 3 σg
3 3 ∴ 8ρ Y + 2 × 13.6 = 10 × 3.36 to this difference of pressure a vertical
lift acts on aeroplane.
where, ρ → ρwater and σ → ρair Y X
2gr 2 (ρ − σ ) 8 cm 28 The height of column is given by ascent
⇒ η= 10 cm
9v 2 cm formula,

2 × 9.81 × (02 . × 10−2 )2 × 999 P 2S cos θ r
= h= −
9 × 87. r ρg 3
= 1 × 10−3 poise If the tube is very narrow, r/3 can be
neglected in comparison with h.
ρ − ρ0 Hg 2S cos θ
19 v ∝ Hence, h = .
η r ρg
v 2 ρ − ρ02 η 33.6 − 27.2
∴ = × 1 or ρY = = 0. 8 g / cc Thus, as the value of r (radius of tube)
v 1 ρ − ρ01 η2 8 decreases, the height increases.
7.8 − 1.2 8.5 × 10−4 × 10 23 At equilibrium, weight of the given Q h ∝ 1 
v2 = ×  
7.8 − 1 13.2 block is balanced by force due to  r
= 6.25 × 10−4 cms −1 surface tension, i.e. 2L S = w
w 29 A bridge during its use undergoes
or S = alternating stress and strain for a large
20 From Bernoulli’s theorem, 2L
1 number of times each day, depending on
ρgh = ρ (v 22 − v 12 ) 1.5 × 10−2 N
2 = movement on it. With time it loses its
2 × 0.3 m
1 2   v2  
2 elastic strength and the amount of strain
⇒ gh = v 1    − 1 = 0.025 Nm −1 in the bridge for a given stress becomes
2 
 1v  
24 Work done = Change in surface large and ultimately it may collapse.
1  A 
2  energy
= v 12   1  − 1 2 30 In case of small drop of mercury the
⇒ W = 2T × 4 π (R22 − R1 )
2   A2   force of gravity is small and the surface
= 2 × 0.03 tension plays a vital role. Therefore,
(Q A1 v 1 = A2 v 2 )
2 × 4 π [(5)2 − (3)2 ] × 10−4 surface tends to have minimum surface
 A1  2hg
⇒   =1+ 2 W = 0.4 π mJ area and sphere has minimum area.
 A2  v1
164 40 DAYS ~ JEE MAIN PHYSICS DAY FOURTEEN

In case of large mercury drop, the MgL sin θ max = 1 ⇒ θ = 90°


L +
gravitational pull becomes more AY Matches with option (b).
∴ T M = 2π
effective than the surface tension and g Hence, tension is least for the case (b).
exerts downwards pull on the drop. 2 Note We should be careful when
⇒  T M  = 1 + Mg
Hence, the large drop of mercury   measuring the angle, it must be in the
becomes elliptical or oval.  T  AY
2
direction as given in the diagram.
Also, surface tension of a liquid Mg  T M 
or =  −1 5 Bulk modulus of rubber (K ) = 9.8 × 108
decreases with rise of temperature, AY  T 
S t = S o (1 − αt ) . N/m2
A  T M  
2
1 Density of sea water ( ρ) = 103 kg/m3
or =   − 1
Y Mg   T  
SESSION 2 Percentage decrease in volume,
3 If L is the initial length, then the  ∆V × 100 = 01
1 Let tension in wire A and B be   .
increase in length by a tension  V 
T A and T B , respectively,
∆V 01
.
T A + TB = W F is given by l =
FL or =
πr 2 Y V 100
and T A × x = T B (L − x )
∆V 1
Solving the above equations, 4L or =
Hence, a = L + l = L + = L + 4C V 1000
πr 2 Y Let the rubber ball be taken up to depth h.
5L
and b = L + = L + 5C ∴ Change in pressure ( p ) = hρ g
πr 2 Y ∴ Bulk modulus
TA TB L
where, C = (K ) =
p
πr 2 Y ( ∆V / V )
Thus, on solving for L and C, we get hρ g
=
x L = 5a − 4b and C = b − a ( ∆V / V )
Hence, for F = 9 N, we get K × ( ∆V / V )
or h=
9L ρg
W (L − x ) x=L + 1
TA = , πr 2 Y 9.8 × 108 ×
L = 1000 = 100 m
Wx = L + 9C 103 × 9.8
TB =
L = (5a − 4b ) + 9 (b − a)
TA = 5b − 4 a
6 As the tanker starts accelerating free
Stress in A = , where A A is surface of the tanker will not be
AA 4 Consider the FBD diagram of the horizontal because pseudo force acts.
cross-section area of wire A. rectangular frame
T Consider the diagram where a tanker is
Stress in B = B , where A B is T sin q Tsin q accelerating with acceleration a.
AB
dma cos q
cross-section area of wire B. q q
It is given, T cos q T cos q dm
A m q
AA = B , dmg sin q
2 dma 90-q
TA T
= B q
AA AB
2L mg a
which gives x = dmg
3 Balancing vertical forces
2T sinθ − mg = 0
2 We know that time period,
[T is tension in the string]
L ⇒ 2T sinθ = mg
T = 2π …(i)
g Total horizontal force Consider an elementary particle of the
When additional mass M is added to its = T cos θ − T cos θ = 0
fluid of mass dm
bob Now from Eq. (i), we get The acting forces on the particle with
L + ∆L mg
T M = 2π , T = respect to the tanker are shown above.
g 2sinθ Now, balancing forces (as the particles is
As, mg is constant in equilibrium) along the inclined
where, ∆L is increase in length.
1 mg direction, component of weight =
We know that Young modulus of the ⇒T ∝ ⇒ T max =
sinθ 2sin θ min component of pseudo force, i.e.
material,
Mg / A MgL sin θ min = 0 ⇒ θ min = 0 dmg sinθ = dma cos θ (we have assumed
Y = = that the surface is inclined at an angle θ)
∆L / L A∆L No option matches with θ = 0°
mg where, dma is pseudo force
MgL T min = ⇒ g sin θ = acos θ ⇒ a = g tanθ
⇒ ∆L = 2sin θ max
AY a
(since, sin θ max = 1) ⇒ tanθ = = slope
g
DAY FOURTEEN PROPERTIES OF MATTER 165

7 Equating pressure at A, we get p 2 = patm − ρg (54 − x ) = ρg (22 + x ) 12 Elongation due to change in


V2 = A ⋅ x temperature,
R sin α d 2 + R cos α d 2 + R(1 − cos α ) d 1
= R(1 − sin α ) d 1 ρg 76 × 8 A = ρg (22 + x )A x
x2 + 22 x − 76 × 8 = 0 ⇒ x = 16 cm

10 Given, surface tension of water F


. × 10−2 N/m
(S ) = 73
R R sin a
R a Density of water ( ρ) = 1.0 × 103 kg/m3
T T
90 –a a R d2 Acceleration due to gravity,
g = 9.8 m/s2 ∆l = Lα∆T
d1 which is compensated by elastic strain,
Angle of contact θ = 0°
when temperature becomes normal, i.e.,
A Diameter of one side, 2r1 = 3.0 mm
TL
(sin α + cos α ) d 2 = d 1 (cos α − sin α ) ∴ r1 = 1.5mm = 1.5 × 10−3 m ∆l =
YS
d 1 1 + tan α Diameter of other side TL
⇒ = Thus, = Lα∆T
d2 1 − tan α 2r2 = 6.0 mm YS
r2 = 3.0 mm ⇒ T = YSα∆T
8 The bubble will detach if, Buoyant force = 3.0 × 10−3 m At equilibrium, force exerted by one half
≥ Surface tension force
Height of water column rises in first and on other,
second tubes F = 2T = 2YSα∆T
2S cos θ
h1 = 13 Frequency, f = v = 1 T
=
1 T
r1 ρg 2l 2l µ 2l Ad
2S cos θ
h2 = where, T is tension in the wire and µ is
qR r2 ρg the mass per unit length of wire.
r ∴ Difference in levels of water rises in Also, Young’s modulus,
both tubes, Tl
(q ) Y =
∆h = h1 − h2 A∆l
ò sin q T × dl = T(2pr) sin q 2S cos θ  1 1 T Y∆l
=  −  ⇒ =
4 3 ρg  r1 r2  A l
πR ρ w g ≥ ∫ T × dl sin θ
3 . × 10−2 × cos 0°
2 × 73 Putting this value in expression of
= frequency, we have
( ρw )  πR3  g ≥ (T ) (2 πr )sin θ
4 1.0 × 103 × 9.8
3  1 y∆ l
  f =
1 1 2l ld
r  − −3 
As, sinθ =  1.5 × 10
−3
3.0 × 10  ∆l
R Given, l = 1.5m, = 0.01
14.6 −2  2 − 1  l
2ρw R 4 g 2ρw g = × 10 
Solving, r = = R2  3  d = 7.7 × 10 kg / m3 ,
3
9.8
3T 3T
= 4.9 mm ≈ 5mm Y = 2.2 × 1011 N / m2
9 In this question, the system is 11 Let the densities of metal and water be ρ Putting these values we, have
accelerating horizontally i.e. no 1 2.2 × 1011 × 0.01
and ρ0 respectively and let x be the f =
component of acceleration in vertical
length of the rod immersed in water at 2l 7.7 × 103
direction. Hence, the pressure in the
an instant of time t. Then, acceleration 2 103
vertical direction will remain f = ×
at that instant =apparent weight
unaffected. 7 3
divided by the mass of the rod, i.e.
p1 = p 0 + ρgh f ≈ 178.2 Hz
i.e. dv πr 2 lρg − πr 2 xρ0g
= Hence, option (b) is true.
Again, we have to use the concept that dt πr 2 lρ
the pressure in the same level will be
gxρ0 14 As, Y = Stress = F
= g  1 −
x
same. =g −  Strain πr 2 × strain
lρ  σl 
p2 ⇒ F = T = Yπr 2 × strain
= g  1 −
x dv dx x
8 cm 54 or 
54–x dx dt  σl  Now, strain

= g  1 −
dv x 
1 /2
or v  L2 + l 2 l2 
dx  σl  = − L = 1 + 2  −1
L  L 
On integrating, we get
1 l2 l2
For air trapped in tube,
v2 x2
l
≈ 1+ −1= 2
= g  l −
l  2
p1 V1 = p 2V2 =gx−  2 L 2L
2 2σl  2σ 
p1 = patm = ρg 76 0 (by binomial expansion)
V1 = A ⋅ 8 Yπr 2 ⋅ l 2
v = 2gl  1 −
1  ∴T =
⇒  2L2
[where, A = area of cross-section]  2σ 
166 40 DAYS ~ JEE MAIN PHYSICS DAY FOURTEEN

15 In equilibrium, 17 Here, Yc = 1 × 1011 N/m2 Total elongation,


upward force = downward force ∆l = ∫ ∆(dx )
Ys = 2 × 1011 N/m2
kx 0 + F B = mg mg × xdx
l
lc = 1.0 m, l2 = 0.5m = ∫0
and ∆lc = 1 × 10−3 m YAl
kx0 mgl
stress = .
As, (strain)c = 2YA
Yc
FB −3 stress 19 The density of concrete of course, is
⇒ 1 × 10 =
1 × 1011 more than that of water and a block of
concrete will sink like a stone, if dropped
⇒ stress = 10 N/m
8 2
into water. Concrete cargo were filled
stress
Mg Now, Ys = with air and as such, average density of
strain
Here, kx 0 is restoring force of spring and cargo vessels
108
F B is buoyancy force. ⇒ strain = = 0.5 × 10−3 [Mass of concrete
L 2 × 1011 + Mass of air]
kx 0 + σ Ag = Mg =
∆ls
2 or = 0.5 × 10−3 [Volume of concrete
σLAg 1/2 + Volume of air]
Mg −
x0 = 2 = Mg  1 − σLA  ⇒ ∆ls = 0.25 × 10−3
  It follows that the average density of
k k  2M 
∴ ∆l = ∆lc + ∆ls = 1 + 0.25 cargo vessels must be less than that of
16 Given, Y = 2 × 1011 N/m2 = 1.25 mm water. As a result, the concrete cargo
18 Consider an element of wire of width dx vessels did not sink.
Stress = 5 × 107 N / m2
stress at a distance x from bottom end of wire. 20 From the continuity equation,
As, =Y The force experienced by this element
strain Av = constant
is due to the gravitational force of
5 × 107
⇒ Strain = = 2.5 × 10−4 portion of wire lower to it. where, Av is the volume of liquid
2 × 1011 flowing per second.
T /A
So, Y = , where ∆(dx ) is the 1
It is symmetric strain. ∆(dx ) or A∝
v
Now, strain of 2.5 × 10−4 is equivalent. dx
elongation in this element. As the stream falls, its speed v increases
∆V ∆r
As, = 3   Now, ∆ (dx ) =
T
dx =
mg
× xdx
and hence its area of cross-section a will
V  r  YA YAl decrease. That is why the stream will
become narrow.
2.5 × 10−4
∴ When the stream will go up, its speed
3
decreases, hence its area of cross-section
∆r
= = 0.83 × 10−4 will increase and it will become broader
r
and spreads out like a fountain. Hence,
dx
∴ Fraction decrease in radius dx option (a) is true.
−4
= (1.00 − 0.83) 10
x
= 0.17 × 10−4
DAY FIFTEEN

Heat and
Thermodynamics
Learning & Revision for the Day
u Heat u Zeroth Law of u Carnot Engine and its Efficiency
u Thermometry Thermodynamics u Refrigerator
u Thermal Expansion u First Law of Thermodynamics u Equation of State of Perfect Gas
u Specific Heat Capacity u Thermodynamic Processes u Kinetic Theory of Gases
u Calorimetry u Second Law of u Degree of Freedom (f )
Thermodynamics
u Change of State u Specific Heat Capacities of Gases
u Reversible and Irreversible
u Heating Curve u Mean Free Path
Processes
u Avogadro’s Number

Heat
Heat is a form of energy which characterises the thermal state of matter. It is transferred
from one body to the other due to temperature difference between them.
Heat is a scalar quantity with dimensions [ML2 T−2 ] and its SI unit is joule (J) while
practical unit is calorie (cal); 1 cal = 4.18 J
If mechanical energy (work) is converted into heat then, the ratio of work done (W ) to
heat produced (Q) always remains the same and constant.
W
i.e. = constant = J or W = JQ
Q
The constant J is called mechanical equivalent of heat. PREP
MIRROR
Temperature Your Personal Preparation Indicator
The factor that determines the flow of heat from one body to another when they are in
u No. of Questions in Exercises (x)—
contact with each other, is called temperature. Its SI unit is kelvin.
u No. of Questions Attempted (y)—
u No. of Correct Questions (z)—
Thermometer (Without referring Explanations)
An instrument used to measure the temperature of a body is called a thermometer. For
construction of thermometer, two fixed reference point ice point and steam point are u Accuracy Level (z / y × 100)—
taken. Some common types of thermometers are as follows: u Prep Level (z / x × 100)—

1. Liquid (mercury) thermometer Range of temperature: −50°C to 350°C In order to expect good rank in JEE,
2. Gas thermometer (Nitrogen gas) Range of temperature: −200°C to 1600°C your Accuracy Level should be above
85 & Prep Level should be above 75.
3. Pyrometers Range of temperature: −800°C to 6000°C
168 40 DAYS ~ JEE MAIN PHYSICS DAY FIFTEEN

2. Superficial Expansion or Areal Expansion Expansion of


Scales of Temperature solids along two dimensions of solid objects is defined as
Three most common scales are Celsius scale or Centigrade superficial expansion.
scale, Fahrenheit scale and Kelvin scale (Absolute scale).
Coefficient of superficial expansion,
Ice point / lower Steam point / Upper ∆A
Scale Unit β=
reference point reference point A 0 × ∆T
Celsius 0 100 °C Final area, A f = A0 (1 + β∆T )
where, A0 is the area of the body at temperature T.
Fahrenheit 32 212 °F
3. Volume or Cubical Expansion Expansion of solids along
Kelvin 273.15 373.15 K three dimensions of solids objects is defined as cubical
expansion.
Relation between C, F and K scales is
Coefficient of volume or cubical expansion,
C F − 32 K − 273.15
= = ∆V
5 9 5 γ=
V0 × ∆ T
Temperature of X − Ice point of X
In general,
Steam point of X − Ice point of X Final volume, V = V0 (1 + γ∆ T ).
Temperature of Y − Ice point of Y where, V0 is the volume of the body at temperature T.
=
Steam point of Y − Ice point of Y NOTE • The coefficients α , β and γ for a solid are related to each
other.
β γ
α= =
Thermometry 2 3
The branch dealing with measurement of temperature is • As temperature increases, density decreases according to
called thermometry. relation,
ρ0
Let thermometric properties at temperatures 0°C (ice point), ρ=
1 + γ∆T
100°C (steam point) and t °C (unknown temperature) are X 0 ,
X 100 , and X t , respectively. Then, or ρ = ρ0 ( 1 − γ ∆T ) [valid for small ∆T]

X t − X 0 X 100 − X 0 Xt − X0 t
t
=
100
or =
X 100 − X 0 100
Thermal Expansion of Liquid
Liquids do not have linear and superficial expansion but these
 X − X0  only have volume expansion.
Thus, t = t  × 100 ° C
 X 100 − X 0  Liquids have two coefficients of volume expansion
1. Coefficient of apparent expansion,
Apparent expansion in volume (∆V )a
Thermal Expansion γa = =
Initial volume × ∆T V × ∆T
Almost all substances (solid, liquid and gas) expand on
heating and contract on cooling. The expansion of a substance 2. Coefficient of real expansion,
on heating is called thermal expansion of substance. Real expansion in volume (∆V )r
γr = =
Initial volume × ∆T V × ∆T
Thermal Expansion of Solids
Thermal expansion in solids is of three types:
Anomalous/Exceptional Behaviour of Water
Generally, density of liquids decreases with increase in
1. Linear Expansion Thermal expansion along a single
temperature but for water as the temperature increases from 0
dimension of a solid body is defined as the linear
to 4°C, its density increases and as temperature increases
expansion. beyond 4°C, the density decreases.
If a rod is having length l0 at temperature T, then The variation in the density of water with temperature is
elongation in length of rod due to rise in temperature by shown in the figure given below.
∆T is,
∆l
∆l = l0 α ∆T or α = Density
l0 × ∆ T
where, α is the coefficient of linear expansion whose
value depends on the nature of the material.
Final length, l f = l0 + l0α ∆T 4°C Temperature

= l0 (1 + α ∆T ) Anomalous behaviour of water


DAY FIFTEEN HEAT AND THERMODYNAMICS 169

Thermal Expansion of Gases Calorimetry


Gases have no definite shape, therefore, gases have only Calorimetry means measurement of heat. When a body at
volume expansion. higher temperature is brought in contact with another body at
1. The coefficient of volume expansion at constant pressure, lower temperature, the heat lost by the hot body is equal to the
∆V 1 heat gained by the colder body and provided no heat is
α= × allowed to escape to the surrounding.
V0 ∆ T
A device in which heat measurement can be made is called a
Final volume, V ′ = V (1 + α ∆T ) calorimeter.
2. The coefficient of pressure expansion at constant volume, If temperature changes, heat exchanged is given by
∆p 1 Q = ms∆T
β= ×
p ∆T As temperature of the body increases, it means heat is taken
Final pressure, p′ = p (1 + β ∆T ). by the body, otherwise given out by the body.

Specific Heat Capacity Change of State


The quantity of heat required to raise the temperature of unit When we supply heat (energy) to a body and its temperature
mass of a substance by 1°C is called specific heat. does not change, then the energy consumed by the body is
used up in changing its phase and the process is called change
Q
Specific heat, s = of state.
m × ∆T
The SI unit of specific heat is J kg −1 k −1 .
Latent Heat
l
Specific heat capacity can have any value from 0 to ∞. For
In case of phase change, heat is consumed during melting and
some substances under particular situations, it can have
boiling while released during freezing and condensation.
negative values also.
The heat required to change the phase of a system
l
The product of mass of the body and specific heat is is proportional to the mass of the system i.e.
termed as heat capacity, C = m × s.
Q∝m
Q = mL
Molar Heat Capacity where, L is the latent heat, which is defined as the amount of
The amount of heat required to change the temperature of a heat required to change the phase of the unit mass of a
unit mole of substance by 1°C is termed as its molar heat substance at given temperature.
capacity, l
In case of ice, the latent heat of fusion of ice is 80 cal/gm.
Q
Cm = l
In case of water, the latent heat of vapourisation is
µ∆T 536 cal/gm.
Generally, for gases, two molar heat capacities are very
common—molar heat capacity at constant pressure (C p) and
molar heat capacity at constant volume (CV ).
Sublimation
l
A substance can sometimes change directly from solid to
gaseous phase, this process is termed as sublimation.
Water Equivalent of a Substance Corresponding latent heat is termed as latent heat of
Water equivalent of certain amount of substance is defined as sublimation Ls . The reverse process can also occur.
the amount of water, which when replaced by the substance l
Very pure water can be cooled several degrees below the
requires the same amount of heat for the same rise in freezing temperature without freezing, the resulting
temperature. unstable state is described as supercooled. When
mS
mw = , this supercooled water is disturbed (either by
Sw dropping dust particles etc.), it crystallises within a second
where, mw = water equivalent of substance whose mass is m, or less.
S = specific heat capacity of substance l
A liquid can sometimes be superheated above its normal
boiling temperature. Any small disturbance such as
and S w = specific heat capacity of water
agitation causes local boiling with bubble formation.
170 40 DAYS ~ JEE MAIN PHYSICS DAY FIFTEEN

The internal energy of an ideal gas is totally kinetic and it is


Heating Curve given by
If we supply energy to a body in solid state 3
U = µRT
(temperature<melting point) at a constant rate, then the curve 2
drawn between temperature and time is termed as the heating and change in internal energy
curve. 3
∆U = µR∆T .
Temperature 2
E
For non-ideal gases, internal energy depends not only on the
temperature but also on the pressure.
C
BP D
Work
A B
MP Consider a system in a cylinder with movable piston, whose
volume can be changed (a gas, liquid or solid). Suppose, the
O t1 t2 t3 t4 Time cylinder has a cross-sectional area A and pressure exerted by
system on the piston face is p. The work done by the system
Heating curve
on the surroundings for small displacement dx is dW = pAdx.
OA represents heating of the solid, Vf
W = ∫ dW = ∫ pdV
1 Vi
Ssolid ∝
Slope of OA
i.e. work done in a finite change of volume from Vi to V f .
AB represents melting of the solid, l
Work done by the system depends on the initial and final
length of AB ∝ L f states.
BC represents heating of the liquid, l
If volume of the system increases, then work is done by the
1 system and it is taken as positive work done.
S liquid ∝
Slope of BC l
If volume of the system decreases, then work is done on
the system and it is taken as negative work done.
CD represents boiling (vaporisation) of the liquid, length of
CD ∝ LV , DE represents heating of the gaseous phase,

S gas ∝
1 First Law of Thermodynamics
Slope of DE According to this law, the heat given to a system (∆Q) is equal
to the sum of increase in its internal energy (∆U) and the work
done (∆W ) by the system against the surroundings.
Zeroth Law of Thermodynamics Mathematically, ∆Q = ∆U + ∆W
When there is no exchange of heat between two objects placed
in contact, then both are called in thermal equilibrium. Sign Convention
According to this law, if two systems A and B , separated by ∆Q = + ve when heat supplied
an adiabatic wall, are separately and independently in thermal = − ve when heat is ejected
equilibrium with a third system C, then the systems A and B
∆U = + ve when temperature increases
are also in a state of thermal equilibrium with each other.
= − ve when temperature decreases
System
C ∆W = + ve when work is done by the system (expansion)
C
System System = − ve when work is done on system (compression)
A B First law of thermodynamics is based on the energy
A B
conservation.
Adiabatic wall
Three system of thermal equilibrium
Thermodynamic Processes
A thermodynamic process is the process of change of state of a
Basic Terms Used Thermodynamics system involving change of thermodynamic variables, e.g.
Internal Energy p, V , T etc. When a system undergo a thermodynamic change,
then work done either by system on surrounding or by
Internal energy of a system is defined as the sum of the total surroundings on system is called external work.
kinetic energy of all its constituent particles and sum of all V2
the potential energies of interaction among these particles. Wext = ∫ p dV = area under p-V curve.
V1
DAY FIFTEEN HEAT AND THERMODYNAMICS 171

1. Isothermal Process l
Molar specific heat of a gas under adiabatic condition
It is that process in which temperature remains constant. ∆Q 0
C= = =0
Here, exchange of heat with the surroundings is allowed. m ⋅ ∆T m ⋅ ∆T
dp p
l
Slope of an adiabatic curve at a point is =−γ .
dV V
T1>T2 l
Work done in an adiabatic process
Vf µR
p ∆W = ∫ p dV = (Ti − T f )
T1 Vi (γ − 1)
T2
During an adiabatic expansion ∆W = + ve, hence,
temperature of gas falls, i.e. an adiabatic expansion is
V always accompanied with cooling.
p-V graph of an isothermal expansion process
As per first law of thermodynamics, since, ∆Q = 0 in an
As temperature T remains constant in an isothermal process, adiabatic process hence,
hence as per Boyle’s law
1 ∆U = − ∆W
p ∝ or pV = constant
V l
Free expansion is an adiabatic process in which ∆W = 0.
Molar specific heat of a gas under isothermal condition Hence, in accordance with first law of thermodynamics
∆Q ∆Q ∆U = 0 i.e. the final and initial values of the internal energy
C= = =∞ are equal in free expansion.
m∆T m (0)
dp
Slope of p-V curve at any point is
p
=− ⋅ 3. Isochoric Process
dV V It is that thermodynamic process in which volume remains
Work done in an isothermal process constant.
Vf  Vf  In an isochoric process for a given mass of gas
∆W = ∫ pdV = nRT ln  
Vi  Vi  p ∝ T or
p
= constant
where, n = number of moles, R = gas constant T
and T = temperature.
l
Indicator diagram for an isochoric process is a straight line
parallel to p-axis.
V f and Vi are final and initial volume of the gas respectively.
As per first law of thermodynamics, since, ∆T = 0 , hence,
∆U = 0 for an ideal gas and we have ∆Q = ∆W .
Thus, heat supplied to the system in an isothermal process is
entirely used to do work against external surroundings. p p

2. Adiabatic Process V V
(a) (b)
It is that process in which there is no exchange of heat of the
system with its surroundings. Thus, in an adiabatic process Graph (a) shows isometric heating graph in which pressure
p, V and T change but ∆Q = 0 or entropy remains constant increases, temperature increases, ∆Q is positive and ∆U is
 ∆Q 
 ∆S = = 0 . positive.
 T 
Similarly, Graph (b) shows isometric cooling graph in
which pressure decreases, temperature decreases, ∆Q is
Q2 Q1 negative and ∆U is negative.
l
Molar specific heat of a gas under isochoric condition
p Q1 > Q2 f
CV = R, where f is the number of degrees of freedom per
2
molecule.
V
l
Work done in an isochoric process
p-V graph for adiabatic process ∆W = ∫ p dV = 0
The equation of state for an adiabatic process is
As ∆W = 0 hence, according to first law of thermodynamics,
pV γ = constant we have
or T V γ − 1 = constant µR
(∆Q)V = ∆U = µCV ∆T = ∆T
or T γ p1 − γ = constant (γ − 1)
172 40 DAYS ~ JEE MAIN PHYSICS DAY FIFTEEN

4. Isobaric Process Cyclic Process


It is that process in which pressure remains constant. In cyclic process, if the process takes the path AxB, it returns via
As in an isobaric process for a given mass of gas ByA, the initial and final points are same.
p
V ∝T
B
V
or = constant x
T
y
l
Indicator diagram for an isobaric process is a straight A
line parallel to X -axis.
V
p-V graph of cyclic process

p p Carnot Engine and its Efficiency


Carnot engine is a theoretical, ideal heat engine working in a
V V reversible cyclic process operating between two temperatures T1
(a) (b) (heat source) and T2 (heat sink). The Carnot’s cycle consists of two
isothermal processes connected by two adiabatic processes as
Graph (a) represent isobaric expansion, graph (b) shown in the figure.
represent isobaric compression.
Isothermal
l
Work done in an isobaric process A expansion
Vf T1
∆W = ∫ p dV = p ∫ dV p Q1
Vi B
Adiabatic
= p(V f − Vi ) = p∆V Adiabatic D expansion
compression T2
l
Molar specific heat of a gas under isobaric condition Isothermal Q2 C
compression
f 
C p =  + 1 R = CV + R V
2 
Various process in Carnot cycle
The efficiency of a Carnot’s cycle is given by
Second Law of η=
W Q
=1− 2 =1− 2
T
Q1 Q1 T1
Thermodynamics
The efficiency does not depend on the nature or quantity of the
Two most common statements of second law of working substance.
thermodynamics are given below

Clausius Statement Refrigerator


It is impossible for a self-acting machine, working in a A refrigerator or heat pump is basically a heat engine running in
reverse direction. It takes heat from colder body (sink) and after
cyclic process to transfer heat from a colder body to a
doing some work gives the rest heat to the hotter body (source).
hotter body without the aid of an external agency.
An ideal refrigerator can be regarded as Carnot’s ideal heat engine
working in the reverse direction.
Kelvin-Planck’s Statement
It is impossible to design an engine which extracts heat Source Q1 Working Q2 Sink
from a reservoir and fully converts it into work without T1 substance T2
producing any other effect.
W

Reversible and Irreversible Working of refrigerator


Processes
A reversible process is one which can be reversed in Coefficient of Performance of a
such a way that all changes taking place in the direct Refrigerator
process are exactly repeated in inverse order and in
opposite sense, and no changes are left in any of the It is defined as the ratio of quantity of heat removed per cycle (Q2 ) to
bodies taking part in the process or in the surroundings. the work done (W ) on the working substance per cycle to remove
Any process which is not reversible exactly is an this heat.
irreversible process. Q Q2 T2 1−η
β= 2 = or β = =
W Q1 − Q2 T1 − T2 η
DAY FIFTEEN HEAT AND THERMODYNAMICS 173

Equation of State of a Perfect Gas Work Done on Compressing a Gas


The equation which relates the pressure (P), volume (V ) and Work done W = p ⋅ ∆V , where p = pressure of the gas and
temperature (T ) of the given state of an ideal gas is known as ∆V = change in volume of the gas.
ideal or perfect gas equation. When two ideal gases having molar masses M1 and M2 are
pV mixed, then thermodynamic variables/parameters for mixture
For 1 mole of gas, = R (constant)
T would be given by
pV = RT n1 m1 + n2 m2
l
M (molar mass) =
where, R is universal gas constant. The SI unit of gas constant n1 + n2
is J/mol-K. Its value is 8.314 J/mol-K or 8.314 × 107 erg/mol-K
n1CV1 + n2CV2
or 2 cal/mol-K. The dimensions of R are [ML2 T−1θ −1 ]. l
CV (of the mixture) =
n1 + n2
Work
Moreover, gas constant R = n1C p1 + n2C p2
Moles × Temperature l
C p (of the mixture) =
l
The perfect gas equation for 1 molecule of gas is n1 + n2
pV = kT n1C p + n2C p
Boltzmann’s constant is represented by per mole gas
l
γ (of the mixture) = 1 2
or is given by
n1CV1 + n2CV2
l

constant
R 8.31 n1 + n2 n n2
i.e. k = = = 1.38 × 10 −23 J K −1 = 1 +
N 6.023 × 1023 γ −1 γ1 − 1 γ2 − 1
Its dimensions are [ ML2 T −2θ −1 ]. where, symbols have their usual meanings.

Kinetic Energy and Temperature


Kinetic Theory of Gases In ideal gases, the point particles can have only translational
Kinetic theory of gases relates the macroscopic properties of motion and thus only translational energy.
gases (such as pressure, temperature etc.) to the microscopic 1 3
properties of gas molecules (such as speed, momentum,
l
Translational KE of a molecule = mc2 = kT
2 2
kinetic energy of molecules etc).
3
l
Mean KE per molecule = kT
2
Assumptions of Kinetic l
Mean kinetic energy per gram mole is given by
Theory of Gases 1  3
KEmole =  mc2  N = kTN = RT
3
l
Every gas is composed of tiny particles known as 2  2 2
molecules. The size of molecules is much smaller than the 3
intermolecular spacing.
l
Average kinetic energy of gas = pV
2
l
The molecules of a gas are identical, spherical, rigid and 3 pV 3 RT
perfectly elastic point masses.
l
KE per molecule = =
2N 2N
l
Molecules are in a state of random rapid motion. They 3
collide with each other. There is no loss of energy during
l
KE per mole = kT
2
collision. Only the direction of motion is changed.
3
l
The time spent in collision between two molecules is l
KE per volume = p
2
negligible in comparison to time between two successive
collisions.
l
The number of collisions per unit volume in a gas remains Concept of Pressure
constant. No attractive or repulsive force acts between gas Pressure p exerted by a perfect gas on the walls of container is
molecules. given by
Gravitational attraction among the molecules is ineffective 1 mN c2 1 M 2
p= =
l
c
due to extremely small masses and very high speed of 3 V 3 V
molecules. 1 1 2  2 1  2
pV = (ρc2 ) =  ρc2  =  ρc2  = E
l
Molecules constantly collide with the walls of container 3 
3 2  
3 2  3
due to which their momentum changes. The change in Here, m = mass of each molecule, c = root mean square
momentum is transferred to the walls of the container. velocity of molecules, ρ = density of gas, M = mass of gas
Consequently, on the walls of container pressure is exerted enclosed in volume V of container, and E = Total KE of the
by gas molecules. The density of gas is constant at all
ideal gas.
points of the container.
174 40 DAYS ~ JEE MAIN PHYSICS DAY FIFTEEN

For one mole of gas,


Various Speeds of Gas Molecules M (∆Q)V 1 (∆Q)V  m
l
Root mean square speed It is defined as the square root CV = Mc V = = Q n = M 
m∆T n ∆T
of mean of squares of the speed of different molecules

i.e. vrms =
v12 + v22 + v32 + K
= v2
Specific Heat at Constant Pressure
N The specific heat of a gas at constant pressure is defined as the
1 2 quantity of heat required to raise the temperature of unit mass
From the expression of pressure, p = ρvrms of gas through 1K, when its pressure is kept constant i.e.
3
(∆Q) p
3p 3 pV 3 RT 3 kT cp = .
vrms = = = = m∆T
ρ Mass of gas M m For one mole of gas,
M (∆Q) p
l
Most probable speed It is defined as the speed which is ∴ C p = Mc p = …(i)
m∆T
possessed by maximum fraction of total number of
1 (∆Q) p
molecules of the gas. = [Q n = m / M ]
n ∆T
2p 2 RT 2 kT
vmp = = = Specific heat of a gas at constant pressure is greater than the
ρ M m specific heat at constant volume i.e. C p > CV .
l
Average speed It is the arithmetic mean of the speeds of C p and CV are related to each other according to relation,
R
molecules in a gas at given temperature. C p − CV = …(ii)
v + v2 + v3 + v4 + K J
vav = 1 Eq. (ii) is called Mayer’s relation. If C p and CV are measured in
N
8p 8 RT 8 kT the units of work and R is also in the units of work (or energy),
Average speed, vav = = = then Eq. (ii) becomes C p − CV = R.
πρ π M πm
Specific Heat in Terms of Degree of
Degree of Freedom (f ) Freedom
The term degree of freedom of a system refers to the possible f
independent motions a system can have For a gas at temperature T, the internal energy U = nRT .
2
l
for monoatomic gas, ( f ) = 3 f 1
Change in energy, ∆U = n R∆T ⇒ CV = f R
l
for diatomic gas, ( f ) = 5 2 2
for triatomic gas, ( f ) = 6(non-linear) f 
Specific heat at constant pressure, C p =  + 1 R.
l

l
for triatomic (linear) gas, ( f ) = 7 2 

for N-atomic molecule ( f ) = 6 N – 3 f 


 + 1 R
l

Cp 2  2
l
for N-atomic linear molecule ( f ) = 6 N – 5 Ratio of C p and CV , γ = = =1 + .
CV f f
R
Law of Equipartition of Energy 2
According to law of equipartition of energy for any system in
thermal equilibrium, the total energy is equally distributed
Mean Free Path
among its various degree of freedom and each degree of The distance travelled by a gas molecule between two
1 successive collision is known as free path.
freedom is associated with energy kT
2 Total distance covered
(where, k = 1.38 × 10 −23 J/K and T = absolute temperature of Mean free path =
Number of collisions
the system). The mean free path of a gas molecule is the average distance
between two successive collisions. It is represented by λ.
Specific Heat Capacities of Gases λ=
1
The specific heat of gas can have many values, but out of them 2 π σ2 n
following two values are important Here, σ = diameter of the molecule and n = number of
molecules per unit volume.
Specific Heat at Constant Volume
The specific heat of a gas at constant volume is defined as the Avogadro’s Number
quantity of heat required to raise the temperature of unit mass According to Avogadro’s hypothesis, gram atomic masses of
of gas through 1°C or 1 K when its volume is kept constant i.e. all elements contain the same number of atoms and this
(∆Q)V number is called Avogadro’s number (N A ) and its value is
cV =
m∆T 6.02 × 1023 .
DAY FIFTEEN HEAT AND THERMODYNAMICS 175

DAY PRACTICE SESSION 1

FOUNDATION QUESTIONS EXERCISE


1 An aluminium sphere of 20 cm diameter is heated from T T
0°C to 100°C. Its volume changes by (given that
coefficient of linear expansion for aluminium
(c) (d)
α Al = 23 × 10− 6 /°C) ª AIEEE 2011 2 1 1 2
(a) 28.9 cc (b) 2.89 cc
(c) 9.28 cc (d) 49.8 cc
p p
2 Cp and CV are specific heats at constant pressure and
constant volume, respectively. It is observed that 6 An ideal gas undergoes four different processes from
Cp − CV = a for hydrogen gas Cp − CV = b for nitrogen the same initial state. Four processes are adiabatic,
gas. The correct relation between a and b is isothermal, isobaric and isochoric. Out of 1, 2, 3 and 4
ª JEE Main 2017 which one is adiabatic.
(a) a = b (b) a = 14 b p
1
(c) a = 28 b (d) a = b
14 4
3
3 A copper ball of mass 100 g is at a temperature T . It is
dropped in a copper calorimeter of mass 100 g, filled 2
with 170 g of water at room temperature. Subsequently, 1
the temperature of the system is found to be 75°C. T is V
(Take, room temperature = 30°C, specific heat of copper (a) 4 (b) 3 (c) 2 (d) 1
= 01
. cal/g°C) 7 Equal masses of two liquids A and B contained in vessels
(a) 885°C (b) 1250°C
of negligible heat capacity are supplied heat at the same
(c) 825°C (d) 800°C rate. The temperature-time graphs for the two liquids are
4 100 g of water is heated from 30°C to 50°C. Ignoring the shown in the figure. If S represents specific heat and L
slight expansion of the water, the change in its internal represents latent heat of liquid, then
energy is (Take, specific heat of water is 4184 J/kg/K) Y
Temperature

ª AIEEE 2011 B A
(a) 8.4 kJ (b) 84 kJ
(c) 2.1 kJ (d) 4.2 kJ
5 Consider p-V diagram for an ideal gas shown in figure.
X
p O Time
1
(a) SA > SB ; LA < LB
p = Constant (b) SA > SB ; LA > LB
V
(c) SA < SB ; LA < LB
2
V (d) SA < SB ; LA > LB

Out of the following diagrams which represents the T-p 8 p-V plots for two gases during adiabatic processes are
diagram? shown in the figure. Plots 1 and 2 should correspond
respectively to
T T
2 p
2
1
(a) (b)
2
1 1 V
p p
(a) He and O 2 (b) O 2 and He (c) He and Ar (d) O 2 and N2
176 40 DAYS ~ JEE MAIN PHYSICS DAY FIFTEEN

9 Figure shows the variation in temperature ( ∆T ) with the 14 A gas expands with temperature according to the relation
amount of heat supplied (Q) in an isobaric process V = kT 2/ 3. Calculate the work done when the temperature
corresponding to a monoatomic (M), diatomic (D) and a changes by 60 K ?
polyatomic (P) gas. The initial state of all the gases are (a) 10 R (b) 30 R
the same and the scale for the axes coincide, ignoring (c) 40 R (d) 20 R
vibrational degrees of freedom, the lines a, b and c 15 A Carnot engine takes 3 × 106cal of heat from a reservoir
respectively, correspond to at 627°C and gives it to a sink at 27°C. The work done by
the engine is
a
(a) 4.2 × 106 J (b) 8.4 × 106 J (c) 16.8 × 106 J (d) 3 × 106 J
Q b
16 A Carnot engine operating between temperaturesT1 and T2
c 1
has efficiency . WhenT2 is lowered by 62 K, its efficiency
6
∆T 1
ª JEE Main (Online) 2013 increases to . Then,T1 and T2 are respectively
3 ª AIEEE 2011
(a) P, M and D (b) M, D and P
(c) P, D and M (d) D, M and P (a) 372 K and 330 K (b) 330 K and 268 K
(c) 310 K and 248 K (d) 372 K and 310 K
10 A certain amount of gas is taken through a cyclic process
17 A Carnot engine, whose efficiency is 40%, takes in heat
( A B C D A ) that has two isobars, one isochoric and one
from a source maintained at a temperature of 500 K. It is
isothermal. The cycle can be represented on a p -V
desired to have an engine of efficiency 60%. Then, the
indicator diagram as ª JEE Main (Online) 2013
intake temperature for the same exhaust (sink)
B temperature must be ª AIEEE 2012
p B
C p (a) efficiency of Carnot engine cannot be made larger
(a) (b) C
than 50%
A D A (b) 1200 K
D
(c) 750 K
V
V (d) 600 K
B C
p 18 The temperature-entropy diagram of a reversible engine
(c) (d) p B C
is given in the figure. Its efficiency is
T
A D D
A
V 2T0
V

11 An ideal Carnot engine whose efficiency is 40%, receives


heat at 500 K. If the efficiency is to be 50%, the intake T0
temperature for the same exhaust temperature is
(a) 600 K (b) 900 K S
(c) 700 K (d) 800 K S0 2S0

12 The pressure inside a tyre is 4 atm at 27°C. If the tyre (a) 1/4 (b) 1/2
(c) 1/3 (d) 2/3
bursts suddenly, its final temperature will be
 7 19 The expansion on of unit mass of a perfect gas at
Given, r = 
 5 constant pressure is shown below.
(a) 300 (4)7 / 2 (b) 300 (4) 2 / 7 a
(c) 300 (2)7 / 2 (d) 300 (4)−2 / 7
O b
13 A refrigerator works between the temperature of melting
ice and room temperature (17°C). The amount of energy
(in kWh) that must be supplied to freeze 1kg of water at (a) a = volume, b = ° C temperature
0°C is (b) a = volume, b =K temperature
(a) 1.4 (b) 1.8 (c) 0.058 (d) 2.5 (c) a = ° C temperature, b = volume
(d) a = K temperature, b = volume
DAY FIFTEEN HEAT AND THERMODYNAMICS 177

20 The temperature of an open room of volume 30 m 3 25 p-V diagram of a diatomic gas is a straight line passing
increases from 17°C to 27°C due to the sunshine. The through origin. The molar heat capacity of the gas in the
process will be
atmospheric pressure in the room remains1 × 105 Pa. If ni
(a) 4 R (b) 3 R (c) 4 R/3 (d) 2.5 R
and nf are the number of molecules in the room before
and after heating, then nf − ni will be ª JEE Main 2017
Direction (Q. Nos. 26-30) Each of these questions contains
. × 1023
(a) 138 (b) 2.5 × 1025 two statements : Statement I and Statement II. Each of these
(c) −2.5 × 1025 (d) −161
. × 1023 questions also has four alternative choices, only one of which is
21 Consider a spherical shell of radius R at temperature T. the correct answer. You have to select one of the codes (a), (b),
The black body radiation inside it can be considered as (c), (d) given below:
an ideal gas of photons with internal energy per unit (a) Statement I is true, Statement II is true; Statement II is
U 1 U  the correct explanation for Statement I
volume u = ∝ T 4 and pressure p =   . If the shell (b) Statement I is true, Statement II is true; Statement II is
V 3 V 
not the correct explanation for Statement I
now undergoes an adiabatic expansion, the relation (c) Statement I is true; Statement II is false
between T and R is ª JEE Main 2015 (d) Statement I is false; Statement II is true
(a)T ∝ e − R
26 Statement I Work done by a gas in isothermal expansion
(b)T ∝ e − 3R is more than the work done by the gas in the same
1 expansion adiabatically.
(c)T ∝
R Statement II Temperature remains constant in isothermal
1
(d) T ∝ 3 expansion, but not in adiabatic expansion.
R
27 Statement I When 1 g of water at 100°C is converted to
22 A thermally insulated vessel contains an ideal gas of steam at 100°C, the internal energy of the system does
molecular mass M and ratio of specific heats γ. It is not change.
moving with speed v and suddenly brought to rest. Statement II From dU = nCV dT , if temperature of the
Assuming no heat is lost to the surroundings, its
system remains constant, then dU = 0, i.e. internal energy
temperature increases by ª AIEEE 2011
remains constant.
(γ − 1) γMv 2
(a) Mv 2K (b) K
2γ R 2R 28 Statement I In an isothermal process (quasistatic), the
(γ − 1) (γ − 1) heat exchange between the system and surroundings
(c) Mv 2K (d) Mv 2K
2R 2 (γ + 1) R takes place even though the gas has the same
temperature as that of the surrounding.
23 Three perfect gases at absolute temperaturesT1, T2 and
T3 are mixed. The masses of molecules are m1, m2 and m3 Statement II There is an infinitesimal difference in
and the number of molecules are n1, n 2 and n 3, temperature between the system and the surroundings.
respectively. Assuming no loss of energy , the final 29 Statement I A special type of thermometer (used to
temperature of the mixture is ª AIEEE 2011
measure very high temperatures and calibrated for an
1 1 + n2T 2 + n3T 3 1 1 + n2T 2 + n3T 3
2 2 2
nT nT ideal black body) measures a value lower than the
(a) (b)
n1 + n2 + n3 1 1 + n2T 2 + n3T 3
nT actual value of the temperature of a red hot iron piece
n12T12 + n22T22 + n32T32 (T1 + T2 + T3 ) kept in open.
(c) (d)
1 1 + n2T 2 + n3T 3
nT 3 Statement II As the iron piece is kept in open, it loses its
24 The value of molar specific heat at constant volume for heat.
1 mole of polyatomic gas having n number of degrees of 30 Statement I The internal energy of a perfect gas is
freedom at temperature T K is entirely kinetic and depends only on absolute
(here, R = universal gas constant) temperature of the gas and not on its pressure or volume.
nR nR Statement II A perfect gas is heated keeping pressure
(a) (b)
2T 2 constant and later at constant volume. For the same
nRT amount of heat the temperature of the gas at constant
(c) (d) 2nRT pressure is lower than that at constant volume.
2
ª JEE Main (Online) 2013
178 40 DAYS ~ JEE MAIN PHYSICS DAY FIFTEEN

DAY PRACTICE SESSION 2

PROGRESSIVE QUESTIONS EXERCISE


1 A solid body of constant heat capacity 1 J/°C is being Codes
heated by keeping it in contact with reservoirs in two A B C D A B C D
ways. (a) 3 4 1 2 (b) 2 3 4 1
(i) Sequentially keeping in contact with 2 reservoirs such (c) 1 2 3 4 (d) 4 3 1 2
that, each reservoir supplies same amount of heat. 4 Diatomic molecules like hydrogen have energies due to
(ii) Sequentially keeping in contact with 8 reservoirs such both translational as well as rotational motion. From the
that each reservoir supplies same amount of heat. 2
equation in kinetic theory pV = E , E is
In both the cases, body is brought from initial 3
temperature 100°C to final temperature 200°C. Entropy (a) the total energy per unit volume
change of the body in the two cases respectively, is (b) only the translational part of energy, because rotational
energy is very small compared to the translational
ª JEE Main 2015
energy
(a) In2, 4In2
(b) In2, In2 (c) only the translational part of the energy, because during
(c) In2, 2In2 collisions with the wall, pressure related to change in
(d) 2In2, 8In2 linear momentum
(d) the translational part of the energy, because rotational
2 Consider a collection of a large number of particles each energies of molecules can be of either sign and its
with speed v. The direction of velocity is randomly average over all the molecules is zero
distributed in the collection. What is the magnitude of the
relative velocity between a pairs in the collection? 5 An ideal monoatomic gas is confined in a cylinder by a
2v v spring-loaded piston of cross-section 8 × 10−3m 2. Initially,
(a)
π
(b)
π the gas is at 300 K and occupies a volume of 2.4 × 10−3m 3
8v 4v and the spring is in a relaxed state. The gas is heated by
(c) (d)
π π a small heater coil H. The force constant of the spring is
8000 Nm −1 and the atmospheric pressure is1 × 105 Pa.
3 An ideal gas (molar specific heat CV = 5R /2) is taken The cylinder and piston are thermally insulated. The
along paths acb, adb and ab, p2 = 2p1,V2 = 2V1. Along piston and the spring are massless and there is no
ab, p = kV , where k is a constant. The various parameters friction between the piston and cylinder. There is no heat
are shown in the figure. Match the Column I with the loss through heater coil wire and thermal capacity of the
corresponding options of Column II and mark the correct heater coil is negligible. With all the above assumptions, if
option from the codes given below. the gas is heated by the heater until the piston moves out
p
slowly by 0.1 m, then the final temperature is
p2 c b
T2 Gas Spring
p1 d T1 H
a

V
V1 V2 (a) 400 K (b) 800 K
(c) 1200 K (d) 300 K
Column I Column II
6 A diatomic ideal gas is used in a car engine as the
A. W acb 1. 15RT1/2 working substance. If during the adiabatic expansion part
B. W adb 2. −15RT1/ 2 of the cycle, volume of the gas increases fromV to 32V,
the efficiency of the engine is ª AIEEE 2010
C. ∆U ab 3. RT1
(a) 0.5 (b) 0.75
D. ∆U bca 4. 2 RT1 (c) 0.99 (d) 0.25
DAY FIFTEEN HEAT AND THERMODYNAMICS 179

7 The specific heat capacity of a metal at low temperature coefficient of linear expansion. Suppose we want to bring
3 the cube to its original size by heating. The temperature
 T 
(T ) is given as Cp (kJK −1 kg −1) = 32   . A 100 g should be raised by ª JEE Main 2017
 400 p 3α p
(a) (b) (c) 3 pKα (d)
vessel of this metal is to be cooled from 20 K to 4 K by a αK pK 3α K
special refrigerator operating at room temperature
12 Consider an ideal gas confined in an isolated closed
(27°C). The amount of work required to cool the vessel is chamber. As the gas undergoes an adiabatic expansion,
ª AIEEE 2011 the average time of collision between molecules
(a) equal to 0.002 kJ increases asV q , where V is the volume of the gas. The
(b) greater than 0.148 kJ  Cp 
(c) between 0.148 kJ and 0.028 kJ value of q is  γ = 
 CV  ª JEE Main 2015
(d) less than 0.028 kJ
3γ + 5 3γ − 5 γ+1 γ −1
(a) (b) (c) (d)
8 A horizontal cylinder with adiabatic walls is closed at 6 6 2 2
both ends and is divided into two parts by a frictionless
13 A pendulum clock loses 12 s a day, if the temperature is
piston that is also insulating. Initially, the value of 40°C and gains 4 s in a day, if the temperature is 20°C.
pressure and temperature of the ideal gas on each side The temperature at which the clock will show correct time
of the cylinder are V0, p0 and T0, respectively. A heating and the coefficient of linear expansion (α) of the metal of
coil in the right-hand part is used to slowly heat the gas the pendulum shaft are respectively ª JEE Main 2016
on that side until the pressure in both parts reaches (a) 25°C, α = 1.85 × 10−5 / ° C (b) 60°C, α = 1.85 × 10−4 / ° C
64p0/27. The heat capacity CV of the gas is independent (c) 30°C, α = 1.85 × 10−3 / ° C (d) 55°C, α = 1.85 × 10−2 / ° C
of temperature and Cp /CV = γ = 1.5.
14 An ideal gas undergoes a quasistatic, reversible process
Take, V0 = 16 m 3, T0 = 324 K, p0 = 3 × 105 Pa in which its molar heat capacity C remains constant. If
Column I represents the physical parameters of the gas, during this process, the relation of pressure p and volume
Column II gives their corresponding values, match the V is given by pV n = constant, then n is given by (Here, Cp
Column I with Column II and mark the correct option from and CV are molar specific heat at constant pressure and
the codes given below. constant volume, respectively)
ª JEE Main 2016
Column I Column II Cp C − Cp Cp − C C − CV
(a) n = (b) n = (c) n = (d) n =
A. Final left-hand volume (in m3) 1. 432 CV C − CV C − CV C − Cp
B. Final left-hand temperature (in K) 2. 9
15 Helium gas goes through a cycle ABCDA (consisting of
C. Final right-hand temperature (in K) 3. 1104 two isochoric and isobaric lines) as shown in figure.
D. Work done (in kJ) on the left-hand gas 4. 3200 Efficiency of this cycle is nearly (assume the gas to be
close to ideal gas) ª AIEEE 2012
Codes
2p0 B C
A B C D A B C D
(a) 2 1 3 4 (b) 1 2 3 4
(c) 4 1 2 3 (d) 3 4 1 2
p0 D
9 The mass of a hydrogen molecule is 3.32 × 10−27 kg. If A

1023 hydrogen molecules strike per second, a fixed wall


of area 2 cm 2 at an angle of 45° to the normal and V0 2V0
rebound classically with a speed of103 m/s, then the
(a) 15.4% (b) 9.1% (c) 10.5%2 (d) 12.5%
pressure on the wall is nearly ª JEE Main 2018
(a) 2.35 × 103 N/m 2 (b) 4.70 × 103 N/m 2 16 An ideal gas is taken from the state A (pressure p,
(c) 2.35 × 102 N/m 2 (d) 4.70 × 102 N/m 2 volume V ) to the state B (pressure p /2, volume 2V ), a
long straight line path in the p-V diagram. Select the
10 Two moles of an ideal monoatomic gas occupies a correct statement from the following.
volume V at 27°C. The gas expands adiabatically to a (a) The work done by the gas in the process A to B,
volume 2 V . Calculate (i) the final temperature of the gas exceeds the work that would be done by it, if system
and (ii) change in its internal energy. ª JEE Main 2018 were taken along the isothermal
(a) (i) 189 K (ii) 2.7 kJ (b) (i) 195 K (ii) −2.7 kJ (b) In the T -V diagram, the path AB becomes a part of a
hyperbola
(c) (i) 189 K (ii) −2.7 kJ (d) (i) 195 K (ii) 2.7 kJ
(c) In the p-T diagram, the path AB becomes a part of a
11 An external pressure p is applied on a cube at 0°C, so hyperbola
that it is equally compressed from all sides. K is the bulk (d) In going from A to B, the temperature T of the gas
modulus of the material of the cube and α is its first decreases to a minimum value and then increases
180 40 DAYS ~ JEE MAIN PHYSICS DAY FIFTEEN

17 One mole of diatomic ideal gas undergoes a cyclic 18 Two moles of helium are mixed with n moles of hydrogen.
process ABC as shown in the figure. The process BC is The root mean square speed of the gas molecules in the
adiabatic. The temperatures at A, B and C are 400 K, mixture is 2 times the speed of sound in the mixture.
800 K and 600 K, respectively. Choose the correct Then, value of n is
statement. ª JEE Main 2014 (a) 1 (b) 3/2 (c) 2 (d) 3
p 19 n moles of an ideal gas undergoes a process A and B as
B
800 K shown in the figure. The maximum temperature of the gas
during the process will be ª JEE Main 2016
600 K p
A C
400 K
V
2p0 A
(a) The change in internal energy in whole cyclic process is
250 R p0 B
(b) The change in internal energy in the process CA is
700 R
V
(c) The change in internal energy in the process AB is V0 2V0
− 350 R
(d) The change in internal energy in the process BC is 9 p0V0 3 p0V0 9 p0V0 9p0V0
(a) (b) (c) (d)
−500 R 4 nR 2 nR 2 nR nR

ANSWERS
SESSION 1 1 (a) 2 (b) 3 (a) 4 (a) 5 (c) 6 (c) 7 (d) 8 (b) 9 (c) 10 (c)
11 (a) 12 (d) 13 (c) 14 (c) 15 (b) 16 (d) 17 (c) 18 (c) 19 (c) 20 (c)
21 (c) 22 (c) 23 (a) 24 (b) 25 (b) 26 (a) 27 (a) 28 (a) 29 (c) 30 (b)
SESSION 2
1 (b) 2 (b) 3 (d) 4 (c) 5 (b) 6 (b) 7 (c) 8 (a) 9 (a) 10 (c)
11 (d) 12 (c) 13 (a) 14 (b) 15 (a) 16 (a) 17 (d) 18 (c) 19 (a)

Hints and Explanations


SESSION 1 b = C p − CV =
R
; for N2 K (ii) 5 In the diagram, T is constant and p 1 > p2 .
28 This situation is represented by curve (c).
1 Cubical expansion, we get
∆V = γ V∆T = 3αV∆T
From Eqs. (i) and (ii), we get In the solution figure, in which p 1 > p 2 and
a = 14b straight line parallel to pressure axis
= 3 × 23 × 10−6 ×  π (10)3 
4
3  represents constant temperature.
3 Heat gained (water + calorimeter)

× 100 Q r = = 10cm 
d 6 p
= Heat lost by copper ball
 2 
⇒ m w sw ∆T + mc sc ∆T = m B sB ∆T
= 28.9 cc
⇒ 170 × 1 × (75 − 30) + 100 Isothermal
2 By Mayer’s relation, for 1 g mole of a
gas, × 01
. × (75 − 30) Adiabatic
C p − CV = R = 100 × 01
. × (T − 75) V
So, when n gram moles are given, ∴ T = 885° C Slope of p-V curve
R
C p − CV = 4 As, work done = 0 Isothermal process,
dp
=−
p
n dV V
As per given question, ∴∆U = mC∆T dp p
= 100 × 10−3 × 4184 × (50 − 30) Adiabatic process, = −γ
R dV V
a = C p − C V = ; for H2 K (i)
2 = 8.4 kJ Thus, (c) is correct.
DAY FIFTEEN HEAT AND THERMODYNAMICS 181

7 As temperature of A rises faster than the 80 × 1000 × 4.2 273 273 18 We have,
= =
temperature of B, therefore specific heat W 290 − 273 17 1 3
Q1 = T 0 S 0 + T0S 0 = T0S 0
of A is less than that of B, i.e. S A < S B . 80 × 1000 × 4.2 × 17 2 2
Horizontal portions of graphs represent ∴ W = J T
conversion of liquid into vapours. The 273
horizontal portion is larger for liquid A, 33.6 × 17 × 104
or W = kWh 2T0
therefore L A > L B . 273 × 3.6 × 105 Q3
8 As it is clear from the figure, = 0.058 kWh Q1
Slope of curve 2 > Slope of curve 1
14 dW = pdV = RT dV …(i) T0
(γp ) 2 > (γp ) 1 V
⇒ γ2 > γ1 As, V = kT , 2 /3 Q2
S
⇒ γ He > γ O2 2 S0 2S0
dV = k T −1 /3 dT
Adiabatic curve 2 corresponds to 3
2 Q 2 = T 0 S 0, Q 3 = 0
helium and adiabatic curve 1 k T −1 /3 dT
dV
= 3 2 /3 =
2 dT W Q − Q2
corresponds to oxygen. Q ⇒ η= = 1
V kT 3 T Q1 Q1
9 We know that, From Eq. (i), we get Q2 2 1
dV 2 dT =1− =1− =
Q = C p ∆T T2
W = ∫ RT
T2
= ∫ RT Q1 3 3
T1 T1 3 T
Q Degree of freedom ∝ C p V
2 2 19 In the given graph, line has positive
So, slope is higher for higher degree of W = R (T2 − T1 ) = R × 60 = 40R
freedom. 3 3 slope with the X-axis and negative
intercept on the Y- axis.
10 From given figure, in processes BC and 15 Here, T1 = 627°C So, we can write the equation of lines as
DA, pressure of gas is constant, hence = 627 + 273 = 900 K y = mx − C …(i)
these represent isobaric process. T2 = 27° C = 27 + 273 = 300 K, According to Charles’ law,
In process CD, volume is constant,
Q1 = 3 × 106 cal V
Vt = 0 t + V0
therefore it represents isochoric process. 273
Q 2 T2 300 1 Q
In process AB, temperature is constant, Q = = = ⇒ Q2 = 1 By rewriting this equation, we get
so it represent isothermal process. Q 1 T1 900 3 3
 273 
T Q1 t =  V t − 273 …(ii)
11 As, η=1− 2 ∴ W = Q1 − Q2 = Q1 −  V0 
T1 3
2 2 By comparing Eqs. (i) and (ii), we can
T2 = Q 1 = × 3 × 106 cal say that temperature is represented on
⇒ =1− η 3 3
T1 the Y-axis and volume on the X-axis.
T2 40 3 W = 2 × 106 cal
⇒ =1− = 20 From pV = nRT = N RT
500 100 5 = 8.4 × 106 J NA
⇒ T2 = 300 K We have,
T 16 Q η1 = 1 − T2 pVN A pVN A
Now, 2 = 1 − η′ T1 n f − ni = −
T1 ′ RT f RT i
1 T
50 1 ⇒ =1− 2
=1− = 6 T1 105 × 30
100 2 ⇒ n f − ni = × 6.02 × 1023
T2 5 83
.
⇒ T1′ = 2T2 = 2 × 300 ⇒ = …(i)
. 
1 1 
T1 6 − 
= 600 K  300 290 
T − 62
∴ η2 = 1 − 2
12 In an adiabatic process, T1 = − 2. 5 × 1025
p2(1 − γ ) T2γ = p1(1 − γ ) T1γ 1 T2 − 62 ∴ ∆n = − 2. 5 × 1025
(1 − γ )/ γ ⇒ =1− …(ii)
p  3 T1
T2 = T1  1  21 According to question,
 p2  On solving Eqs. (i) and (ii), we get 1 U 
T1 = 372 K and T2 = 310 K p=  
(1 − 7 / 5)
3 V 
= 300  
4
= 300 (4)−2 /7
7/5
 1 17 Efficiency, η = 1 − Tsink ⇒
nRT 1  U 
=   [Q pV = nRT ]
Tsource V 3 V 
13 T2 = 0° C = 273 K, T1 = 17° C = 17 + 273 T nRT 1
Now, 0.4 = 1 − sink or ∝ T4
500 K V 3
= 290 K
Q2 ⇒ Tsink = 0.6 × 500 K = 300 K or VT 3 = constant
Coefficient of performance = 300 K
W 0.6 = 1 − 4
Thus, or π R3T 3 = constant
T2 T ′source 3
=
T1 − T2 300 K 1
⇒ T ′source = = 750 K or TR = constant ⇒ T ∝
0.4 R
182 40 DAYS ~ JEE MAIN PHYSICS DAY FIFTEEN

22 As no heat is lost. 27 Since, 1 g of water is converted into For an ideal gas,


Loss of kinetic energy steam at constant temperature of 100°C, p1V1 pV
i.e. dT = 0. = 2 2 ⇒ T2 = 4T1
= Gain of internal energy of gas T1 T2
1 ∴ Change in internal energy,
mv 2 = n C V ∆T 5R 15RT1
2 dU = nC V dT = 0 i.e. U = constant ∆U ab = (4T1 − T1 ) =
2 2
1 m R
⇒ mv 2 = ⋅ ∆T 28 In isothermal process, the heat For the process,
2 M γ −1 exchange between system and −15RT1
Mv 2 (γ − 1) surrounding at constant temperature, ∆U bca = − ∆U ab =
⇒ ∆T = K 2
2R i.e. there is an infinitesimal difference
in temperature between the system and 2
4 In the relation pV = E , E is only the
23 F n 1 kT1 + F n 2 kT2 + F n 3 kT3 the surrounding. 3
2 2 2 translational part of energy of molecules.
F 29 Since, the thermometer is caliberated
= (n1 + n2 + n3 ) kT This is because during collision of
2 with an ideal black body, the body that
molecules with the walls, pressure
n 1T1 + n 2T2 + n 3T3 emits or absorbs all the radiations
exerted relates to change in linear
⇒ T = falling on it, shows a lower value of
n1 + n2 + n3 temperature. This is because iron is not
momentum of gas molecules.
a black body, i.e. does not absorb/emit 5 V1 = 2.4 × 10−3 m3 , p1 = p 0 = 105 N m −2
24 According to law of equipartition of all radiation falling on it.
energy, average KE per molecule per and T1 = 300 K (given)
degree of freedom at temperature T is 30 The external energy depends upon If area of cross-section of piston is A and
1 absolute temperature of gas. Also, it moves through distance x, then
kT . The average KE per molecule of Statement II is correct, but both the
2 increment in volume of the gas = Ax.
polyatomic gas statements are independently true. If force constant of a spring is k, then
n force F = kx and
molecule = kT
2 SESSION 2 pressure = F / A = kx/ A.
The average KE per molecule of 1 Since, entropy is a state function, V2 = V1 + Ax
polyatomic gas therefore change in entropy in both the = 2.4 × 10−3 + 8 × 10−3 × 01.
n n
(E ) = kT × N = RT
processes must be same. = 3.2 × 10−3
2 2 kx
2 Consider any two particles having and p2 = p 0 +
d n  n angle θ between directions of their A
CV =  RT  = R
dt  2  2 8000 × 01.
velocities. = 10 +
5
= 2 × 105
8 × 10−3
25 As p-V diagram is a straight line passing v From ideal gas equation,
through origin, therefore p ∝ V or
p1V1 pV
pV −1 = constant. B = 2 2
θ T1 T2
In the process, pV x = constant, molar v
A 105 × 2.4 × 10−3

heat capacity is given by → → → 300
R R Then, v rel = v B − v A
C = + 2 × 105 × 3.2 × 10−3
γ −1 1− x i.e. v rel = v + v 2 − 2v 2 cos θ
2 =
T2
where, x = − 1 and γ = 1.4 for diatomic θ
= 2v 2 (1 − cos θ) = 2v sin ⇒ T2 = 800 K
gas 2
R R 6 The efficiency of cycle,
C = + So, average vrel over all pairs
1.4 − 1 1 − (−1) 2π 2π θ η=1−
T2
∫ v rel dθ ∫0 2v sin dθ T1
5 R v = 0 = 2
= R+ rel 2π 2π
2
∴ C = 3R
2
∫0 dθ ∫0 dθ For adiabatic process,
T V γ − 1 = constant
2v × 2[− cos (θ / 2)]20π 4v
= = >v 7
26 2π π For diatomic gas, γ =
5
3 A → 4.; B → 3.; C → 1.; D → 2. T1V1γ − 1 = T2V2γ − 1
p Isothermal W acb = W ac + Wcb = 0 + p2 (V2 − V1 ) γ −1
V 
T1 = T2  2 
Adiabatic = p2V1 = 2 p1V1 = 2RT1  V1 
W adb = W ad + Wdb 7
−1
V T1 = T2 (32)5
= p1 (V2 − V1 ) + 0 = p1V1 = RT1
= T2 (25 )2 / 5 = T2 × 4
The slope of adiabatic curve is several ∆U ab = U ac + U cb
times the slope of an isothermal curve T1 = 4 T2
= (Q ac − W ac ) + (Q cb − Wcb )
η =  1 − 
and slope of both is negative. Thus, area 1
5R ∴
under adiabatic curve is smaller than C V (Tc − T1 ) + C V (T2 − Tc ) = (T2 − T1 )  4
that under isothermal curve. 2
3
5R = = 075 .
CV = (given) 4
2
DAY FIFTEEN HEAT AND THERMODYNAMICS 183

7 Heat required to change the temperature Work done on the left-hand side gas is 12 For an adiabatic process,
of vessel by a small amount dT p V − p 0V 0
W = 1 1 TV γ −1
= constant.
− dQ = mC pdT γ −1
Total heat required,  64 × 9 − 1 p V We know that, average time of collision
  0 0
 T  dT
3  27 16  2 between molecules,
4
= = p 0V 0
−Q = m ∫2032  400  3
−1 3 τ =
1
4 2 nπ 2 v rms d 2
100 × 10−3 × 32  T 4  = 3200 kJ
=   where, n = number of molecules per
(400)3  4 20 unit volume
9
⇒ Q = 0.001996 kJ mv and v rms = rms velocity of molecules.
Work done required to maintain the 1
45° As, n∝
temperature of sink to T2 , V
Q − Q2
W = Q1 − Q2 = 1 Q2 and v rms ∝ T
Q2 V
T  ⇒ τ ∝
=  1 − 1 Q 2 T
 T2  Momentum imparted due to first
Thus, we can write
collision = 2mv sin 45° = 2mv
 T − T2  n = K 1V −1
⇒ W = 1  Q2 Q sin 45° = 1 
 T2   and v rms = K 2 T 1 /2
2 
For T2 = 20 K, where, K 1 and K 2 are constants.
300 − 20 n 2mv
W1 = × 0.001996 ∴ Pressure on surface = For adiabatic process,
20 Area
. × 10−27 × 103
1023 × 2 × 332 TV γ − 1 = constant.
= 0.028 kJ =
For T2 = 4 K, (2 × 10−2 )2 Thus, we can write
300 − 4 p = 235
. × 103 N/m2 τ ∝ VT −1 /2 ∝ V (V 1 − γ )−1 /2
W2 = × 0.001996
4 γ +1

= 0148
. kJ 10 For adiabatic process, relation of or τ ∝ V 2
temperature and volume is,
As temperature is changing from 20 K to
T2V2γ − 1 = T1V1γ − 1 13 Time period of a pendulum,
4 K, work done required will be more l
than W1 , but less than W2 . ⇒ T2 (2V ) = 300(V )
2 /3 2 /3
T = 2π
g
5
8 A → 2; B → 1; C → 3; D → 4 . [γ = for monoatomic gases]
3 where, l is length of pendulum and g is
The compression in the left-hand side is 300 acceleration due to gravity.
adiabatic ⇒ T2 = 2 /3 ≈ 189 K
2 Such as change in time period of a
p 0V 0γ = p1V1γ Also, in adiabatic process, pendulum,
∆Q = 0, ∆U = − ∆W ∆T 1 ∆l
=
p1 p2 − nR(∆T )
or ∆U = T 2 l
V1 V2 γ −1
When clock losses 12 s, we get
3 25
= − 2 × × (300 − 189) 12 1
T1 T2 2 3 = α (40 − θ) …(i)
T 2
≈ −2.7 kJ
2/3 2/3
p  When clock gains 4 s, we get
V1 = V 0  0  = V 0  
27 p
11 K =
 p1   64  ( − ∆V / V ) 4 1
= α (θ − 20) …(ii)
9V 0 T 2
= = 9 m3 ∆V p
⇒ − = Comparing Eqs. (i) and (ii), we get
16 V K
pV
Also, 0 0 = 1 1
pV 40 − θ
⇒ − ∆V =
pV 3=
T0 T1
K θ − 20
pV 4T 0
⇒ T1 = 1 1 = = 432 K Change in volume, ∆V = γ V ∆T ⇒ θ = 25°C
p 0V 0 3 where, γ = coefficient of volume
64 p 0 Substituting the value of θ in Eq. (i), we
∴ p2 = expansion.
have
27 Again, γ = 3α
12 1
and
p 0V 0
=
p2V2 where, α is coefficient of linear = α (40 − 25)
T0 T2 expansion. T 2
∴ ∆V = V (3α ) ∆T 12 1
⇒ V2 = 2V 0 − V1 ⇒ = α (15)
pV 24 × 3600 2
= 23 m3 ∴ = V (3α ) ∆T
K 24
⇒ T2 =
92T 0
= 1104 K α =
p 24 × 3600 × 15
27 ∴ ∆T =
3αK
α = 1.85 × 10−5/°C
184 40 DAYS ~ JEE MAIN PHYSICS DAY FIFTEEN

log 10 
14 For polytropic process, specific heat for 2V  19 As, T will be maximum temperature,
= RT × 23026
. 
an ideal gas,  V  where product of pV is maximum.
R = 0.693 pV
C = + CV p
1− n Thus, statement (a) is correct.
R
∴ + CV = C 17 According to first law of
1− n thermodynamics, we get 2p0 A
R
⇒ = C − CV (i) Change in internal energy from A
1− n to B, i.e. p0 B
R ∆U AB = nC V (T B − T A )
⇒ =1− n
C − CV 5R
=1× (800 − 400) V
(where, R = C p − C V ) 2 V0 2V0
C p − CV = 1000 R
⇒ =1− n Equation of line AB, we have
C − CV (ii) Change in internal energy from
y − y1
C p − CV B to C, y − y1 = 2 ( x − x1 )
⇒ n=1− ∆U BC = nC V (TC − T B ) x2 − x1
C − CV 5R
=1× (600 − 800) 2 p0 − p0
C − Cp 2 ⇒ p − p0 = (V − 2V 0 )
⇒ n= V 0 − 2V 0
C − CV = − 500 R
(iii) ∆U isothermal = 0 − p0
Thus, number of moles n is given by ⇒ p − p0 = (V − 2V 0 )
C − Cp (iv) Change in internal energy from
V0
n=
C − CV C to A, i.e. ∆U CA − p0 2
nRT = V + 3 p 0V
∆U CA = nC V (T A − TC ) V0
15 Efficiency of a process is defined as the
ratio of work done to energy supplied. =1×
5R
(400 − 600) 1  − p0 2 
T =  V + 3 p 0V 
2 nR  V 0 
Here,
= − 500 R
∆W Area under p - V diagram For maximum temperature,
η= =
∆Q ∆Q AB + ∆Q BC 18 v rms = 3RT , ∂T
=0
M
p 0V 0 ∂V
∴ η=
nC V ∆T1 + nC p∆T2 γ RT − p0
v sound = , (2V ) + 3 p 0 = 0
p 0V 0 M V0
=
3 5 − p0
nR(T B − T A ) + nR(TC − T B ) v rms = 2 v sound (2V ) = − 3 p 0
2 2 V0
p 0V 0 Solving it, we get
= 3
 3 (2 p V − p V )  ⇒ V = V0
3 = 2γ 2
2 0 0 0 0 
 5  3 (condition for maximum temperature)
∴ γ = for the mixture.
 + (4 p 0V 0 − 2 p 0V 0 ) 2 Thus, the maximum temperature of the
 4 
p 0V 0 Cp n1C p1 + n2C p2 gas during the process will be
= As, γ = =
3 5 CV n1 + n2 1
p 0V 0 + ⋅ 2 p 0V 0 T max =
2 2 n1 + n2 nR
×
1 n1C V1 + n2C V2
= = 154. %  − p0 9 2 3 
6.5 n1C p1 + n2C p2  × V0 + 3 p 0 × V0 
γ =  V0 4 2 
n1C V1 + n2C V2
16 Work done = 1  p +  V
p
1  9
p 0V 0 
2 2 9
For helium, =  − p 0V 0 +
5 3 nR  4 2 
3 Cp = R,C V1 = R
=
pV = 0.75 pV 1 9 p 0V 0
2 2
4 =
Work done during isothermal process For hydrogen, 4 nR
7 5
p Cp = R, C V2 = R
p0
2 2 2
2  R  + n  R 
5 7
p 3 2  2 
∴ =
2  R  + n  R 
2 3 5
p/2 2  2 
10 + 7n
=
6 + 5n
V
V 2V V0 ⇒ n=2
DAY SIXTEEN

Transfer of
Heat
Learning & Revision for the Day
u Modes of Heat Transfer u Perfectly Black Body u Newtons Law of Cooling
u Some Common Terms and u Kirchhoff’s Law of Radiation u Wien’s Displacement Law
Points u Stefan’s Law

Heat is a form of energy which characterises the thermal state of matter. It is transferred
from one body to the other due to temperature difference between them.
Heat is a scalar quantity with dimensions [ML2 T−2 ] and its SI unit is joule (J) while
practical unit is calorie (cal); 1 cal = 4.18 J.
The heat can be transferred from one body to the another body, through the following
modes
(i) Conduction (ii) Convection (iii) Radiation

Conduction
The process of heat-transmission in which the particles of the body do not leave their
position is called conduction.

Thermal Conductivity PREP


KA∆T t
MIRROR
The amount of heat transmitted through a conductor is given by Q = Your Personal Preparation Indicator
l
where, A = area of cross-section, u No. of Questions in Exercises (x)—
No. of Questions Attempted (y)—
∆T = temperature difference = T2 − T1 ,
u

u No. of Correct Questions (z)—


t = time elapsed, (Without referring Explanations)
K = thermal conductivity
and l = length of conductor u Accuracy Level (z / y × 100)—
u Prep Level (z / x × 100)—
The rate of transmission of heat by conduction is given by
∆Q KA∆T
H= = In order to expect good rank in JEE,
∆t l your Accuracy Level should be above
85 & Prep Level should be above 75.
The unit of thermal conductivity is Wm−1 K −1 .
186 40 DAYS ~ JEE MAIN PHYSICS DAY SIXTEEN

Thermal Resistance Widemann-Franz Law


∆Q KA ∆T According to the Widemann-Franz law, the ratio of thermal
|H|= = ⋅ ∆T =
∆t l l / KA and electrical conductivities is same for the metals at a
l particular temperature and is proportional to the absolute
The term is generally called the thermal resistance (R). temperature of the metal.
KA
K
Equation for rate of heat conduction can be written as i.e. ∝T
σ
l

Q ∆T
H = = K
t Rthermal or = constant
σT
It is equivalent/analysis to ohm’s law which states that
V
I =
R(electrical) Convection
Q The process of heat-transmission in which the particles of the
where, H = is equivalent of electric current and called as fluid move is called convenction.
t
heat, ∆T is equivalent of voltage (PD) and Rthermal is
equivalent of Relectrical . Natural Convection
In natural convection gravity plays an important role. When a
Combination of Metallic Rods fluid is heated, the hot part expands and becomes less dense.
Consequently it rises and the upper colder part is replaced.
1. Series Combination In a series combination of two
This again gets hot, rises up and is replaced by the colder part
metal rods, equivalent thermal conductivity is given of the fluid.
by
l +l
Ks = 1 2
l1 l
K1 K2
Forced Convection
+ 2 In a forced convection the material is forced to move up by a
K 1 K2 l l
1 2 pump or by some other physical means. Common examples of
2K 1 K 2 Series combination forced convection are human circulatory system, cooling
or Ks = [if l1 = l2 ] of rods system of an automobile engine and forced air heating system
K 1 + K2
in offices, etc.
If temperature of the interface of the series
combination be T, then
K T + K2T2 Radiation
T = 1 1
K 1 + K2 The process of the transfer of heat from one place to another
A1 A2 place without heating the intervening medium is called
2. Parallel Combinations In a radiation.
parallel combination of two
metal rods, thermal conductivity
is given by
K1 K2 Interaction of Radiation with Matter
K A + K2 A2 When radiant energy Q is incident on a body, a part of it Qa is
Kp = 1 1 absorbed, another part Qr is reflected back and yet another
A1 + A2 Parallel combination part Qt is transmitted such that
K 1 + K2 of rods Q = Qa + Qr + Qt
or K p = [if A1 = A2 ]
2 Qa Qr Qt
or + + =1
Q Q Q
Formation and Growth of Ice on a Lake or a + r + t =1
Time required for the thickness of the layer of ice to increase Qa
from d1 to d2 will be where, a = = absorbing power or absorptance,
Q
ρL f 2
t = (d2 − d12 ) Q
2 KT r = r = reflecting power or reflectance
Q
where, ρ = density of ice,
Qt
L f = latent heat of fusion of ice and t = = transmitting power or transmittance
Q
and K = thermal conductivity of ice
DAY SIXTEEN TRANSFER OF HEAT 187

Kirchhoff’s law implies that ‘a good absorber is a good


Some Common Terms and Points
l

emitter (or radiator) too’.


l
Absorptive power (α) It is defined as the ratio of the l
Fraunhoffer’s lines (dark lines observed in solar
radiant energy absorbed by it in a given time to the total spectrum) can be easily explained on the basis of
radiant energy incident on it in the same interval of time. Kirchhoff’s laws.
Energy absorbed
α=
Energy incident
As a perfectly black body absorbs all radiations incident on
Stefan’s Law
it, the absorptive power of a perfectly black body is According to the Stefan’s law, the emissive power of a
maximum and unity. perfectly black body (energy emitted by black body per unit
surface area per unit time) is directly proportional to the
l
Spectral absorptive power (aλ ) It is the ratio of radiant fourth power of its absolute temperature.
energy absorbed by a surface to the radiant energy incident
on it for a particular wavelength λ. The spectral absorptive Mathematically, E ∝ T 4
power aλ is related to absorptive power a through the or E = σT 4
relation ∞ or E = σ (T 4 − T04 )
a = ∫ aλ dλ
0
where, σ is a constant known as the Stefan’s constant and its
l
Emissive power (e) It is the total amount of energy radiated
by a body per second per unit area of surface value is 5.67 × 10 −8 Wm−2 K −4 and T0 is the temperature of
1 ∆Q surrounding of black body.
e= For a body, whose emissivity is ε, Stefan’s law is modified
A ∆t
l

as, e = εσT 4
l
Spectral emissive power (e λ ) It is emissive power for a
particular wavelength λ. Thus,
l
The total radiant energy Q emitted by a body of surface
∞ area A in time t, is given by
e = ∫ e λ dλ
0 Q = Ate = AtεσT 4
l
Emissivity (ε) Emissivity of a body at a given temperature l
The radiant power (P), i.e. energy radiated by a body per
is defined as the ratio of the total emissive power of the unit time is given by
body (e) to the total emissive power of a perfect black body Q
(E) at that temperature, P = = AεσT 4
t
e
i.e. ε= l
If a body at temperature T is surrounded by another body at
E temperature T0 (where, T0 <T ), then according to Stefan’s
law of power
Perfectly Black Body P = εσ A(T 4 − To4 )
A perfectly black body is the one which completely absorbs l
If a body at temperature T is surrounded by another body at
the radiations of all the wavelengths that are incident on it. temperature T0 (where, T0 < T ), then Stefan’s law is
Thus, absorbing power of a perfectly black body is 1 (i.e a = 1). modified as,
When perfectly black body is heated to a suitable high E = σ (T 4 − T04 ) [black body]
temperature, it emits radiation of all possible wavelengths.
and e = εσ (T 4
− T04 ) [any body]
e.g. temperature of the sun is very high (6000 k approx.) it
emits all possible radiations. So, it is an example of black
body.
l
For perfectly black body, a = 1, r = t = 0
Newton’s Law of Cooling
l
For a perfect reflector, a = t = 0, r = 1 According to the Newton’s law of cooling, rate of cooling of a
body is directly proportional to the temperature difference
l
For a perfect transmitter, a = r = 0, t = 1. between the body and the surroundings, provided the
temperature difference is small.
dT dT
Kirchhoff’s Law of Radiation Mathematically, ∝ (T − T0 ) or − = k (T − T0 )
dt dt
Kirchhoff’s law of radiation states that the ratio of emissive where, k is a constant.
power to absorptive power of a body, is same for all surfaces at If a body cools by radiation through a small temperature
the same temperature and is equal to the emissive power of a difference from T1 to T2 in a short time t when the surrounding
perfectly black body at that temperature. temperature is T0 , then
e e
Mathematically, 1 = 2 = K = E (Black body) dT ~ T1 − T2 T1 + T2
a1 a2 − and T =
dt t 2
188 40 DAYS ~ JEE MAIN PHYSICS DAY SIXTEEN

The Newton‘s law of cooling becomes Variation of intensity of thermal radiation with wavelength is
T1 − T2  T + T  shown in fig. The total area under E λ -λ curve gives the total
= k 1 2 − T0 .
 t   2  intensity of radiation at that temperature. The area, in
accordance with the Stefan’s law of radiation, is directly
proportional to the fourth power of the temperature.
Black Body Spectrum
The black body Eλ
Wien’s Displacement Law
spectrum is a T4 > T3
According to Wien‘s law, the product of wavelength
continuous spectrum as corresponding to maximum intensity of radiation and
shown in the figure. At a T3 > T2 temperature of body is constant i.e. λ mT = constant = b, where b
given temperature, is known as the Wien’s constant and its value is 2.89 × 10 −3 mK.
initially the intensity of T2 > T1
thermal radiation Solar Constant
increases with an
The amount of heat received from the sun by one square
increase in wavelength T1
centimeter area of a surface placed normally to the sun rays at
and reaches a maximum λ mean distance of the earth from the sun is known as solar
λm4 λ λ
value at a particular m3 m 2 λm 1 constant. It is denoted by S.
wavelength λ m. On Graph between intensity E λ and λ 2
 r
increasing the S =   σT4
 R
wavelength beyond λ m, the intensity of radiation E λ starts
decreasing. where, r is the radius of sun and R is the mean earth’s distance
from sun value of solar constant S = 1.937 cal/cm2 /min.

DAY PRACTICE SESSION 1

FOUNDATION QUESTIONS EXERCISE


1 A cylindrical rod is having temperaturesT1 and T2 at its 4 The coefficient of thermal conductivity of copper is
ends. The rate of flow of heat is Q1. If all the linear 9 times that of steel. In the composite cylindrical bar
dimensions are doubled keeping the temperature shown in the figure, what will be the temperature at the
constant, then rate of flow of heat Q 2 will be junction of copper and steel?
Q1 Q1
(a) 4 Q1 (b) 2 Q1 (c) (d) 100°C 0°C
4 2
Copper Steel
2 A uniform metallic rod rotates about its perpendicular
bisector with constant angular speed. If it is heated 18 cm 6 cm
uniformly to raise its temperature slightly
(a) 75°C (b) 67°C (c) 25°C (d) 33°C
(a) Its speed of rotation increases
(b) Its speed of rotation decreases 5 Three objects coloured black, grey and white can
(c) Its speed of rotation remains same withstand hostile conditions at 2800°C. These objects are
(d) Its speed increases because its moment of inertia thrown into furnace where each of them attains a
increases temperature of 2000°C. Which object will have the
brightest glow?
3 Two slabs A and B of different materials but with the
same thickness are joined as shown in the figure. The (a) The white object
(b) The black object
thermal conductivities of A and B are K1 and K 2,
(c) All glow with equal brightness
respectively. The thermal conductivity of the composite
(d) Grey object
slab will be
l 6 A black body maintained at a certain temperature
radiates heat energy at the rate Q Watt. If its surface is
A K1 B K2 smoothened, so as to lower its emissivity by 10%, what
will be the increase in its rate of radiation at double the
l/2 l/2
initial temperature?
1
(a) (K1 + K 2 ) (b) K1K 2 (c) (K1 + K 2 ) (d)
2K1K 2 (a) (0.9 × 2 4 − 1) Q W (b) 0.9 × 2 4 Q W
2 (K1 + K 2 ) (c) (0.9 × 2)4 Q W (d) (0.9)4 × 2Q W
DAY SIXTEEN TRANSFER OF HEAT 189

7 We consider the radiation emitted by the human body.


T T
Which of the following statement is true?
(a) The radiation is emitted during the summers and (c) q (d) q
0 0
absorbed during the winters.
(b) The radiation emitted lies in the ultraviolet region and
O t O t
hence is not visible.
(c) The radiation emitted is in the infrared region. 13 A sphere, a cube and a thin circular plate, all of same
(d) The radiation is emitted only during the day. material and same mass are initially heated to same high
8 Parallel rays of light of intensity I = 912 Wm are incident −2 temperature. Then
on a spherical black body kept in surroundings of (a) plate will cool fastest and cube the slowest
temperature 300 K. Take, Stefan constant (b) sphere will cool fastest and cube the slowest
σ = 5.7 × 10−8 Wm −2 K −4 and assume that the energy (c) plate will cool fastest and sphere the slowest
exchange with the surroundings is only through radiation. (d) cube will cool fastest and plate the slowest
The final steady state temperature of the black body is 14 Temperatures of two stars are in the ratio 3 : 2. If
close to ª 2014 Main wavelength for the maximum intensity of the first body is
(a) 330 K (b) 660 K (c) 990 K (d) 1550 4000 Å, what is the corresponding wavelength of the
9 The spectral energy distribution of a star is maximum at second body?
twice temperature as that of the sun. The total energy (a) 9000 Å (b) 6000 Å
radiated by the star is (c) 2000 Å (d) 8000 Å
(a) twice as that of the sun 15 The energy spectrum of a black body exhibits a
(b) same as that of the sun maximum around a wavelength λ 0. The temperature of
(c) sixteen times as that of the sun the black body is now changed such that the energy is
(d) one-sixteenth of the sun 3λ 0
maximum around a wavelength . The power radiated
10 Newton’s law of cooling holds good only, if the 4
temperature difference between the body and the by the two black bodies will now increase by a factor of
surroundings is (a) 64/27 (b) 256/81 (c) 4/3 (d) 16/9
(a) less than 10°C (b) more than 10°C
(c) less than 100°C (d) more than 100°C 16 Three discs, A, B and C having radii 2 m, 4 m and 6 m
respectively, are coated with carbon black on their outer
11 A liquid in a beaker has temperature θ(t ) at time t and θ 0
surfaces. The wavelengths corresponding to maximum
is temperature of surroundings, then according to Newton’s
intensity are 300 nm, 400 nm and 500 nm, respectively.
law of cooling, the correct graph between loge (θ − θ 0 ) and
The power radiated by them are Q A , QB and QC ,
t is
respectively
loge (θ – θ0)

loge (θ – θ0)

(a) QA is maximum
(b) QB is maximum
(a) (b) (c) Q C is maximum
(d) QA = QB = QC
O t O t 17 Variation of radiant energy emitted by sun, filament of
tungsten lamp and welding arc as a function of its
loge (θ – θ0)

loge (θ – θ0)

wavelength is shown in figure. Which of the following


(c) (d)
options is the correct match?

E1
O t O t

12 If a piece of metal is heated to temperature θ and then


T3
allowed to cool in a room which is at temperature θ 0 , the
graph between the temperature T of the metal and time T2
will be closed to ª JEE Main 2013 T1
l
T
(a) Sun-T1, tungsten filament-T2 , welding arc-T3
(a) T (b) q (b) Sun-T2 , tungsten filament-T1, welding arc-T3
0
(c) Sun-T3 , tungsten filament-T2 , welding arc-T1
O t O t (d) Sun-T1, tungsten filament-T3 , welding arc-T2
190 40 DAYS ~ JEE MAIN PHYSICS DAY SIXTEEN

Direction (Q. Nos. 18-20) Each of these questions allowed to cool down in the same environment. The
contains two statements : Statement I and Statement II. hollow sphere cools faster.
Each of these questions also has four alternative choices, only Statement II Rate of cooling follows the Stefan’s law
one of which is the correct answer. You have to select one of which is E ∝ T 4.
the codes (a), (b), (c), (d) given below.
19 Statement I A body that is a good radiator is also a good
(a) Statement I is true, Statement II is true; Statement II is
absorber of radiation at a given wavelength.
the correct explanation for Statement I
(b) Statement I is true, Statement II is true; Statement II is
Statement II According to Kirchhoff’s law, the
absorptivity of a body is equal to its emissivity at a given
not the correct explanation for Statement I
wavelength.
(c) Statement I is true; Statement II is false
(d) Statement I is false; Statement II is true 20 Statement I For higher temperatures, the peak emission
wavelength of a black body shifts towards the lower
18 Statement I A solid sphere of copper of radius R and a wavelength side.
hollow sphere of the same material of inner radius r and
Statement II Peak emission wavelength of a black body
outer radius R are heated to the same temperature and
is proportional to the fourth-power of the temperature.

DAY PRACTICE SESSION 2

PROGRESSIVE QUESTIONS EXERCISE


1 A metallic sphere cools from 50° C to 40°C in 300 s. If the 1 2 4 1
(a) (b) (c) (d)
room temperature is 20°C, then its temperature in the 2 1 1 4
next 5 min will be 6 Two circular discs A and B with equal radii are
(a) 38°C (b) 33.3°C (c) 30°C (d) 36°C blackened. They are heated to same temperature and are
2 A pan filled with hot food cools from 94° C to 86 °C in cooled under identical conditions. What inference do you
2 min, when the room temperature is at 20 °C, how long draw from their cooling curves?
will it take to cool from 71 °C to 69 °C?
A
(a) 14 s (b) 3 s (c) 42 s (d) 13 s
3 Two slabs A and B of equal surface area are placed one R
B
over the other such that their surfaces are completely in
contact. The thickness of slab A is twice that of B. The
coefficient of thermal conductivity of slab A is twice that
of B. The first surface of slab A is maintained at 100°C,
while the second surface of slab B is maintained at 25°C. (q – q 0 )
The temperature at the contact of their surfaces is (a) A and B have same specific heats
(a) 62.5°C (b) 45°C (c) 55°C (d) 85°C (b) Specific heat of A is less
4 Assuming the sun to be a spherical body of radius R at a (c) Specific heat of B is less
(d) None of the above
temperature of T K, evaluate the total radiant power,
incident on the earth, at a distance r from the sun. 7 Three rods of copper, brass and steel are welded
(a) 4 πr02R 2 σT 4 / r 2 (b) πr02R 2 σT 4 /r 2 together to form a Y-shaped structure. Area of
cross-section of each rod is 4 cm 2. End of copper rod is
(c) r02R 2 σT 4 / 4 πr 2 (d) R σT / r
2 4 2
maintained at 100°C whereas ends of brass and steel are
where, r0 is the radius of the earth and σ is the Stefan’s kept at 0°C. Lengths of the copper, brass and steel rods
constant. are 46, 13 and 12 cm respectively.
5 Two identical conducting rods are first connected The rods are thermally insulated from surroundings
independently to two vessels, one containing water at except at ends. Thermal conductivities of copper, brass
100°C and the other containing ice at 0°C. In the second and steel are 0.92, 0.26 and 0.12 in CGS units,
case, the rods are joined end to end and connected to respectively. Rate of heat flow through copper rod is
the same vessels. Let q1 and q 2 g/s be the rate of ª JEE Main 2014
−1 −1
melting of ice in two cases respectively. The ratio of (a) 1.2 cals (b) 2.4 cals
q1 / q 2 is (c) 4.8 cals −1 (d) 6.0 cals −1
DAY SIXTEEN TRANSFER OF HEAT 191

8 A mass of 50 g of water in a closed vessel, with 14 Chamber I Chamber II


surroundings at a constant temperature takes 2 min to
ideal real
cool from 30°C to 25°C. A mass of 100g of another liquid
gas gas
in an identical vessel with identical surroundings takes
the same time to cool from 30°C to 25°C. The specific
heat of the liquid is (The water equivalent of the vessel is 1 2 3 4
30 g.) ª JEE Main (Online) 2013
(a) 2.0 kcal/kg (b) 7 kcal/kg
There are two identical chambers, completely thermally
(c) 3 kcal/kg (d) 0.5 kcal/kg insulated from surrounding. Both chambers have a
partition wall dividing the chambers in two
9 The figure shows a system of two
compartments. Compartment 1 is filled with an ideal gas
concentric spheres of radii r1 and r2
and compartment 3 is filled with a real gas.
and kept at temperatures T1 and T2 r1 Compartments 2 and 4 are vacuum. A small hole (orifice)
respectively. The radial rate of flow T1 is made in the partition walls and the gases are allowed
of heat in a substance between the
r2 to expand in vacuum.
two concentric spheres, is T2
Statement I No change in the temperature of the gas
proportional to
takes place when ideal gas expands in vacuum.
(r2 − r1) r  r1r2
(a) (b) ln  2  (c) (d) (r2 − r1) However, the temperature of real gas goes down
(r1r2 )  r1  (r2 − r1) (cooling) when it expands in vacuum.
10 A slab of stone of area 3600 cm 2 and thickness 10 cm is Statement II The internal energy of an ideal gas is only
exposed on the lower surface to steam at 100°C. A block kinetic. The internal energy of a real gas is kinetic as well
of ice at 0°C rests on the upper surface of the slab. If in as potential. ª JEE Main (Online) 2013
1 h 4.8 kg of ice melted the thermal conductivity of the (a) Statement I is false and Statement II is true
stone is (b) Statement I and Statement II both are true. Statement II
(a) 1.24 W/m/k (b) 2.24 W/m/k is the correct explanation of Statement I
(c) 0.24 W/m/k (d) 1.54 W/m/k (c) Statement I is true and Statement II is false
(d) Statement I and Statement II both are true, but
11 Two spherical stars A and B emit black body radiation.
Statement II is not the correct explanation of Statement I
The radius of A is 400 times that of B and A emits
λ  15 A rod AB of uniform cross-section consists of four section
104 times the power emitted from B. The ratio  A  of AC, CD, DE and EB of different metals with thermal
 λB 
conductivities K, ( 0.8) K, (1.2) K and (1.50) K,
their wavelengths λ A and λ B at which the peaks occur in
respectively. Their lengths are respectively L,
their respective radiation curves is
(1.2) L, (1.5) L and (0.6) L. They are joined rigidly in
(a) 1 (b) 2
succession at C, D and E to form the rod AB. The end A
(c) 3 (d) 5
is maintained at 100 °C and the end B is maintained at
12 A metal is heated in a furnace where a sensor is kept 0°C. The steady state temperatures of the joints C, D and
above the metal surface to read the power radiated (P ) E are respectively TC ,TD and TE . Column I lists the
by the metal. The sensor has a scale that displays temperature differences (TA − TC ),(TC − TD ), (TD − TE ) and
log2(P /P0 ), where P0 is a constant. When the metal (TE − TB ) in the four sections and column II their values
surface is at a temperature of 487°C, the sensor shows a jumbled up. Match each item in column I with its correct
value 1. Assume that the emissivity of the metallic value in column II.
surface remains constant. What is the value displayed by
the sensor when the temperature of the metal surface is A C D E B
raised to 2767°C?
(a) 1 (b) 4 (c) 7 (d) 9 Column I Column II

13 Two spherical bodies A (radius 6 cm ) and B (radius A. (TA − TC ) 1. 9.6


18 cm) are at temperatures T1 and T2, respectively. The B. (TC − TD ) 2. 30.1
maximum intensity in the emission spectrum of A is at C. (TD − TE ) 3. 24.1
500 nm and in that of B is at 1500 nm. Considering them D. (TE − TB ) 4. 36.2
to be black bodies, what will be the ratio of the rate of A B C D A B C D
total energy radiated by A to that of B? (a) 3 4 2 1 (b) 1 2 4 3
(a) 9 (b) 7 (c) 5 (d) 1 (c) 3 4 1 2 (d) 3 2 1 4
192 40 DAYS ~ JEE MAIN PHYSICS DAY SIXTEEN

ANSWERS

SESSION 1 1 (b) 2 (b) 3 (d) 4 (a) 5 (b) 6 (a) 7 (c) 8 (a) 9 (c) 10 (a)
11 (a) 12 (c) 13 (c) 14 (b) 15 (b) 16 (b) 17 (c) 18 (c) 19 (a) 20 (c)

SESSION 2 1 (b) 2 (c) 3 (a) 4 (b) 5 (c) 6 (b) 7 (c) 8 (d) 9 (c) 10 (a)
11 (b) 12 (d) 13 (a) 14 (c) 15 (a)

Hints and Explanations


SESSION 1 5 An ideal black body absorbs all the 9 From Stefan’s law of radiation, E = sT 4
KA (T - T2 ) radiations incident upon it and has an where, s is Stefan’s constant.
1 Initially, Q 1 = 1 1 but on
emissivity equal to 1. If a black body
l1 Given, T = 2TS
and an identical body are kept at the
doubling all dimensions l 2 = 2l 1 and \ E ¢ = s(2TS )4 = 16 sTS4 = 16ES
same temperature, then the black body
A 2 = 4 A 1.
will radiate the maximum power. Hence, total energy radiated by star is
K A 2 (T1 - T2 ) sixteen times as that of the sun.
Hence, Q 2 = Hence, the black object at a temperature
l2 of 2000°C will have the brightest glow. 10 Newton’s law of cooling states, that,‘‘ the
K 4 A 1 (T1 - T2 ) K A 1 (T1 - T2 ) rate of cooling of a body is directly
= =2 6 For black body,
2l 1 l1 4
proportional to temperature difference
Rate of radiation Q = sT between the body and the surroundings,
= 2Q 1
After smoothing and doubling the provided the temperature difference is
2 When a metallic rod is heated it temperature = Rate Q small, (less than 10°C)’’ and Newton’s
expands. Its moment of inertia (I ) about = 0.9 s (2T )4 law of cooling is given by
a perpendicular bisector increases. dT
According to law of conservation of = 0.9 ´ 24 Q µ (q - q 0 )
dt
angular momentum, its angular speed Change = (0.9 ´ 24 - 1) Q W
(w ) decreases, since w µ1 / I. (According 11 According to Newton’s law of cooling,
to law of conservation of angular 7 The heat radiation emitted by the rate of fall in temperature is proportional
momentum). human body have wavelength of the to the difference in temperature of the
body with surrounding, i.e.
3 The thermal resistance of a slab of order of 7.9 ´ 10-7 m to 10-3 m, which is dq
length l 1 , area of cross-section A and ofcourse the range of infrared region. - = k (q - q 0 )
thermal conductivity K is given by dt
l Hence, human body emits radiation in dq
R=
infrared region.
Þ ò q - q0 = ò - k dt
KA
8 In steady state Þ ln (q - q 0 ) = kt + C
Since, the slabs are joined in series, the
which is a straight line with negative
thermal resistance of the composite slab
is slope.
\ RC = R1 + R2 12 According to Newton’s cooling law,
l l /2 l /2 S =4πR2 T2 = T1 + Ce - Kt
or = + I
KC A K1 A K2 A where, C = T i - T1
2K 1 K 2
or KC = πR2 (difference in temperature of body and
(K 1 + K 2 ) Incident Radiation surrounding)
Energy incident per second = Energy Þ T2 µ e - Kt
4 From temperature of interface,
radiated per second Thus, the graph decays exponentially.
K 1 q1 l 2 + K 2 q2 l 1
q= \ IpR2 = s (T 4 - T 04 ) 4pR2 This is shown in fig. (c).
K 1 l2 + K 2 l1
It is given that K Cu = 9K s . ⇒ I = s (T 4 - T 04 ) 4 13 We know that, the rate of loss of heat
from a body is directly proportional to
So, if K s = K 1 = K , then ⇒ T 4 - T 04 = 40 ´ 108 the surface area of the body. For a given
K Cu = K 2 = 9K mass of a material, the surface area of a
9K ´ 100 ´ 6 + K ´ 0 ´ 18 Þ T 4 - 81 ´ 108 = 40 ´ 108
Þ q= circular plate is maximum and of sphere
9K ´ 6 + K ´ 18 ⇒ T 4 = 121 ´ 108 is least. Hence, plate will cool fastest and
5400 K sphere the slowest.
= = 75° C ⇒ T » 330 K
72 K
DAY SIXTEEN TRANSFER OF HEAT 193

14 According to Wien’s displacement law, 20 As the temperature of the black body 3 According to question, the temperature
l m T = constant increases, two distinct behaviours are at the contact of the surface is given by
(l m )1 T2 observed. The first effect is that the K d q + K 2d 1 q2
\ = = 1 2 1
(l m )2 T1 peak of the distribution shifts towards K 1d 2 + K 2d 1
T1 3 the shorter wavelength side. This shift 2K 2d 2 ´ 100 + 2d 2 ´ K 2 ´ 25
Here, = is found to obey the following =
T2 2 2K 2d 2 + K 2 2d 2
relationship called the Wien’s
(l m )1 = 4000 Å displacement law, which is given by 200 + 50
= = 62.5° C
4000 ´ 3 l mT = constant. 4
\ (l m ) 2 =
2
The second effect is that the total 4 From Stefan’s law, the rate at which
= 6000 Å amount of energy, the black body emits energy is radiated by sun from its surface
15 We know that, l mT = constant and the per unit area per unit time increases is
with fourth power of the absolute
power radiated by a black body is
temperature T. r0
proportional to T 4 i.e. P µ T 4 , Hence, TK
-4 r
P µ (l m ) SESSION 2 Earth
4 4 4 R
P2 æ l m1 ö æ l ö 4 1 According to the Newton’s law of
Þ =ç ÷ = ç 0 ÷ = æç ö÷
P1 çè l m2 ÷
ø è 3l 0 4 ø è 3 ø cooling,
256 50 - 40 é 50 + 40 ù Sun
= =K ê - 20ú
81 300 ë 2 û P = s ´ 4pR2 ´ T 4
10 é 90 ù [Sun is a perfect black body as it emits
16 Q µ AT 4 and l mT = constant. Þ =K
êë 2
- 20 = K ´ 25
úû
300 radiations of all wavelengths and so for
A r2 10 it, e = 1]
Hence, Q µ 4
or Q µ Þ K =
(l m ) (l m )4 300 ´ 25 The intensity of this power at the surface
(2) (4) (6)2
2 2
1 of the earth
Q A : QB : Q C = : : = [under the assumption r>> r 0] is
(3)4 (4)4 (5)4 30 ´ 25
4 1 36 P s ´ 4pR2T 4 sR2T 4
= : : Similarly, I = 2
= 2
=
81 16 625 40 - q é 40 + q ù 4pr 4pr r2
=K ê - 20ú
= 0.05 : 0.0625 : 0.0576 300 ë 2 û The area of the earth which receives this
i.e. Q B is maximum. q Kq energy is only one-half of the total
é
= K 20 + - 20 = ù
êë úû surface area of earth, whose projection
17 l mT = constant 2 2 would be pr 02 .
From the graph T3 > T2 > T1 q q
= = \ Total radiant power as received by the
Temperature of sun will be maximum. 2 ´ 30 ´ 25 1500 pr 2 R2 sT 4
earth = pr 02 ´ I = 0 2
Therefore, (c) is the correct option. Þ 300 q = 1500(40 - q) r
= 60000 - 1500 q
18 As external radii of both the spheres are 5 When the rods are placed in vessels
equal, the surface areas of the two are Þ 1800 q = 60000
60000 q (T1 - T2 )
also equal. Therefore, when the two Þ q= = 33.3 ° C =
1800 t R
spheres are heated to the same
æ qö mL (100 - 0)
temperature, both radiate heat at the 2 The average temperature of 94 °C and =ç ÷ = = q1 L = …(i)
è t ø1 t R /2
same rate. 86 °C is 90 °C, which is 70 °C above the
Now, rate of loss of heat from a sphere When the rods are joined end to end
room temperature, under these
dq conditions the pan cools 8°C in 2 min, æ q ö = mL = q L = (100 - 0) …(ii)
= Mc ç ÷ 2
dt we have è t ø2 t 2R
Therefore, rate of cooling Change in temperature From Eqs. (i) and (ii), we get
dq rate of loss of heat = kDT q1 4
= Time = .
dt Mc 8°C q2 1
dq 1 = k (70° C ) …(i)
or µ 2 min
dt M 6 According to Newton’s law of cooling,
The average of 69 °C and 71°C is 70 °C, rate of cooling is given by,
Since, mass of a hollow sphere is less,
which is 50 °C above room temperature. æ -dT ö = eAs (T 4 - T 4 )
its rate of cooling will be fast. ç ÷ 0
K is same for this situation as for the è dt ø mc
19 According to Kirchhoff’s law of original where, c is specific heat of material.
radiation, the ratio of emissive power to æ -dT ö µ 1
2° C or
absorptive power of a body, is same for = k(50°C ) …(ii) ç ÷
Time è dt ø c
all surfaces at the same temperature and
at a particular wavelength. On dividing Eq. (i) by Eq. (ii), we get i.e. rate of cooling varies inversely as
8° C/2 min k (70° C ) specific heat. From the graph, for A,rate
Thus, Kirchhoff‘s law implies that a = ,
2° C/ time k (50° C ) of cooling is larger. Therefore, specific
good absorber is a good emitter (or heat of A is smaller.
radiator) too or vice-versa. T = 07
. min = 42s
194 40 DAYS ~ JEE MAIN PHYSICS DAY SIXTEEN

7 In thermal conduction, it is found that Let us consider an element (spherical 12 Given, log 2 P1 = 1
in steady state the heat current is shell) of thickness dx and radius x as P0
directly proportional to the area of shown in figure. Let us first find the P1
equivalent thermal resistance between Therefore, =2
cross-section A, which is proportional to P0
the change in temperature (T1 - T2 ). inner and outer sphere.
dx According to Stefan’s law,
DQ KA(T1 - T2 ) Resistance of shell = dR =
Then, = P µT2
Dt x K ´ 4px2 4 4
According to thermal conductivity, we æ From R = 1 , ö P2 æ T2 ö æ 2767 + 273 ö 4
ç ÷
Þ = ç ÷ =ç ÷ =4
get KA P1 è T1 ø è 487 + 273 ø
ç ÷
è where, K = thermal conductivity ø P2 P
100°C r 2 dx = 2 = 44
Þ ò dR = R = ò P1 2P0
l1 r 1 4pKx2
dQ1 P2
= 2 ´ 44
K1 dt 1 é1 1ù r2 - r1 P0
= ê - ú=
4pK ë r1 r2 û 4pK (r1 r2 ) P
log 2 2 = log 2 [2 ´ 44 ] = log 2 2 + log 2 44
T - T2 P0
dQ2
K2 K3 dQ3 Rate of heat flow = H = 1
R = 1 + log 2 28= 9
dt l2 l3 dt T - T2
= 1 ´ 4pK (r1 r2 )
0° C 0°C r2 - r1 13 We know that, l m µ 1
T
dQ 1 dQ 2 dQ 3 r r l A TB 500 1
i.e. = + µ 1 2 \ = = =
dt dt dt r2 - r1 l B T A 1500 3
0.92(100 - T ) 0.26(T - 0)
= 10 Assuming that heat loss from the sides E µT4 A
46 13 of the slab is negligible, the amount of
012
. (T - 0) (where, A = surface area = 4pR2 )
+ heat flowing through the slab is
12 kA (T1 - T2 )t \ E µ T 4 R2
Q = …(i)
Þ T = 40° C d 4 2 2
E A æT A ö æ R A ö 4 æ 6 ö
dQ 1 0.92 ´ 4(100 - 40) If m is the mass of ice and L the latent =ç ÷ ç ÷ = (3) ç ÷ = 9
\ = E B è T B ø è RB ø è 18 ø
dt 46 heat of fusion, then
= 4.8 cals -1 Q = mL …(ii) 14 Intermolecular distance in ideal gases is
From Eqs. (i) and (ii), we have assume to be large as compared to real
8 As, DQ = msDq (for water) one. Hence, the internal energy of an
kA (T1 - T2 )t
= 50 ´ s ´ 5 mL = ideal gas and a real gas is kinetic as well
d
Þ æ dQ ö = rate of cooling mLd as potential.
ç ÷ or k = …(iii)
è dt ø s According to Newton’s cooling law,
A (T1 - T2 )t
250 25 option (c) is correct answer.
= = Given, m = 4.8 kg, d = 10 cm = 01
. m,
2 ´ 60 2 ´ 6 15 A ® 3 ; B ® 4 ; C ® 2; D ® 1
A = 3600cm2 = 036
. m2 ,
(Q S w = 1 cal/g) We have four sections, AB, BC, CD and
dQ ö T1 = 100°C, T2 = 0°C DE with (dQ/dt) as the steady state
Now, other liquid æç ÷
è dt ø l and t = 1 h = (60 ´ 60)s thermal energy transmitted per second (A
We know that, being the areas of cross-section)
= rate of cooling
dQ KA (100 - Tc )
100 ´ s ´ 5 50 L = 80cal/g = 80000cal/ kg =
= = s dt L
2 ´ 60 2´ 6 = 80000 ´ 42 . ´ 105 J /kg
. J /kg = 336
A (0.8) K (TC - T D )
dQ ö æ dQ ö Substituting these values in Eq. (iii) and =
Now, æç ÷ =ç ÷ (1.2) L
è dt ø l è dt ø s solving, we get
(1.2) KA (T D - T E ) (1.5) KAT E
Þ s = 0.5cal/g = 0.5 kcal/kg k = 1.24J/s /m /° C or 1.24 W/m/k = =
(1.5) L (0.6)L
9 To measure the radial rate of heat flow, 11 Power, P = (sT 4 A ) = sT 4 (4pR2 )
These give
we have to go for integration technique or, P µ T 4 R2 …(i) 0.8
as here the area of the surface through (100 - TC ) = æç ö÷(TC - T D )
According to Wien’s law, è 12
. ø
which heat will flow is not constant. 1
lµ æ 12
. ö æ 1.5 öT
T = ç ÷(T D - T E ) = ç ÷ E
è 1.5ø è 0.6 ø
(l is the wavelength at which
dx peak occurs) 6(100 - TC ) = 4(TC - T D )
x \ Eq. (i) will become, = (4.8)(T D - T E ) = 15T E
1/4
R2 é R2 ù Solving for the differences
P µ 4 or l µ ê ú (100 - TC ),(TC - T D ),(T D - T E ) and T E
r1
T1 l ëP û
1 /2 1/4 remaining that the sum of these
l A éRAù é PB ù differences is 100, we obtain
Þ =ê ú ê ú
l B ë RB û ë PA û
r2 (T A - TC ) = 241. ,(TC - T D ) = 362
.
1/4
T2 1 /2 é 1 ù (T D - T E ) = 301
.
= [400] =2
êë 104 úû and (T E - T B ) = 9.6
DAY SEVENTEEN UNIT TEST 3 (GENERAL PROPERTIES OF MATTER) 195

DAY SEVENTEEN

Unit Test 3
(General Properties
of Matter)

1 A jar is filled with two non-mixing liquids 1 and 2 having 3 A hot metallic sphere of radius r radiates heat. Its rate of
densities d1 and d 2 respectively. A solid ball, made of a cooling is
material of density d 3 , is dropped in the jar. It comes to (a) independent of r
equilibrium in the position as shown in the figure. Which (b) proportional to r
of the following is true for d1, d 2 and d 3 ? (c) proportional to r 2
(d) proportional to 1/r
4 A certain ideal gas undergoes a polytropic process
Liquid 1 d1
pV n = constant such that the molar specific heat during
d3
the process is negative. If the ratio of the specific heat of
Liquid 2 d2
the gas be γ, then the range of values of n will be
(a) 0 < n < γ
(a) d1 > d3 > d2 (b) d1 < d2 < d3 (b) 1< n < γ
(c) d1 < d3 < d2 (d) d3 < d1 < d2 (c) n = γ
2 A spherical body of volume V and density σ is (d) n > γ
suspended from a string, the other end of the string is 5 Pressure p, volume V and temperature T for a certain
connected to the roof of a sealed container filled with an material are related by
ideal fluid of density ρ. AT − BT 2
p=
V
where, A and B are constants. Find an expression for the
a
work done by the material if the temperature changes
from T1 to T2 reduce while the pressure remains constant.
(a)W = A (T2 − T1) − B (T23 − T13 )
If the container accelerates towards right with a constant
(b)W = A (T22 − T12 ) − B (T2 − T1)
acceleration a, then the force exerted by the liquid on the
(c)W = A (T2 − T1) − B T2 − T1 
1
body when it is in equilibrium w.r.t. fluid, is  2 
(a)V ρ a 2 + g 2 + Vσ a (b)V σ a (d)W = A (T2 − T1) − B (T22 − T12 )

(c) [Vρ (g + a)]2 + [Vσ a] 2 (d)Vρ g 2 + a 2


196 40 DAYS ~ JEE MAIN PHYSICS DAY SEVENTEEN

6 A soap bubble is very slowly blown on the end of a glass 11 The coefficients of thermal conductivity of copper,
tube by a mechanical pump which supplies a fixed mercury and glass are Kc , Km and K g , respectively, such
volume of air every minute whatever be the pressure that Kc > km > K g . If the same quantity of heat is to flow
against which it is pumping. The excess pressure ∆p per second per unit area of each and corresponding
inside the bubble varies with time is shown by which of temperature gradients are, X c , X m and X g , respectively,
then
the graph?
(a) Xc = Xm = X g (b) Xc > Xm > X g
Dp Dp (c) Xc < Xm < X g (d) Xm < Xc < X g
12 The temperature of the source of a Carnot’s heat engine
(a) (b) is 1000°C. Its efficiency could be 100% only if the
temperature of the sink is
t t
(a) 1000°C (b) 0°C
(c) equal to triple of water (d) − 273.16°C
Dp Dp
13 A steel rod is 3.00 cm in diameter at 25°C. A brass ring
has an interior diameter of 2.992 cm at 25°C. At what
(c) (d)
common temperature will the ring just slide onto the rod?
(take, α s = 11 × 10−6 ° C−1, α b = 19 × 10−6 ° C−1 )
t t
(a) 460°C (b) 260°C (c) 500°C (d) 360°C
7 A small electric immersion heater is used to heat 100 g of 14 A diver is hunting for a fish with a water gun. He
water for a cup of instant coffee. The heater is labelled accidentally fires the gun, so that bullet punctures
“200 W,” which means that it converts electrical energy the side of the ship. The hole is located at a depth of
to thermal energy at this rate. Calculate the time required 10 m below the water surface. The speed with which
to bring all this water from 23°C to 100°C, ignoring any water enter in the ship is
heat losses. [c = 4190 J kg −1 K −1 ]
(a) 18 ms −1 (b) 14 ms −1
(a) 100 s (b) 200 s (c) 190 s (d) 161 s
(c) 25 ms −1 (d) Cannot be determined
8 The average depth of Indian ocean is about 3000 m.
15 A material has a Poisson’s ratio 0.3. If a uniform rod of it
Calculate the fractional compression, ∆V / V , of water at
suffers longitudinal strain 4 ⋅ 5 × 10− 3 , then calculate the
the bottom of the ocean. Given that the bulk modulus of
percentage change in its volume.
water is 2.2 × 109 Nm −2 .
(a) 0.15% (b) 0.25% (c) 0.18% (d) 0.5%
(Take, g = 10 ms −2 )
16 Compute the number of moles and in 1.00 cm 3 of an ideal
(a) 1. 36 × 10− 2 (b) 3 × 10− 3
gas at a pressure of 100 Pa and at a temperature of 220 K.
. × 10− 2
(c) 15 (d) 1. 36 × 10− 6
(a) 3.35 × 10−8 mol (b) 4.57 × 10−7mol
9 The temperature of the two outer surfaces of a composite (c) 5.47 × 10−8 mol (d) 2.75 × 10−8 mol
slab, consisting of two materials having coefficients of
17 A slab consists of two parallel layers of copper and brass
thermal conductivity K, 2K and thickness x, 4x,
of the same thickness same area of cross-section and
respectively are T2 and T1 (T2 > T1 ). The rate of heat having thermal conductivities in the ratio 1 : 4. If the free
transfer through the slab in a steady state is face of brass is at 100°C and that of copper is at 0°C, the
 A(T2 − T1 )K  temperature of the interface is
 x  f with f which is equal to (a) 80°C (b) 20°C (c) 60°C (d) 40°C
−2
(a) 1 (b) 1/2 18 On applying a stress of x Nm , the length of wire of
(c) 2/3 (d) 1/3 some material becomes double. Value of the Young’s
10 An aeroplane has a mass of 1.60 × 10 kg and each wing
4 modulus for the material of the wire in Nm −2 , is [Assume
has an area of 40 m 2 . During level flight, the pressure on Hooke’s law to be valid] “Go for approx results”
the wings’s lower surface is 7 × 104 Pa. The pressure on (a) x (b) 2x
the upper surface of the wing is (c) x / 2 (d) Insufficient information

(Take, p0 = 10 5 Pa and assume the pressure difference 19 743 J of heat energy is added to raise the temperature of
is only on wings and not on body) 5 mole of an ideal gas by 2 K at constant pressure. How
5
(a) 10 Pa much heat energy is required to raise the temperature of
(b) 6.8 × 104 Pa the same mass of the gas by 2K at constant volume?
(c) 7 × 104 Pa (Take, R = 8.3 J/K-mol)
(d) 6.6 × 104 Pa (a) 826 J (b) 743 J (c) 660 J (d) 620 J
DAY SEVENTEEN UNIT TEST 3 (GENERAL PROPERTIES OF MATTER) 197

20 Six identical conducting rods are joined as shown in The water which comes out from the hole at the instant
figure given below. Points A and D are maintained at when the hole is at height H above the ground, strikes
temperatures 200 °C and 20°C, respectively. The the ground at a distance x from P. Which of the following
temperature of junction B will be is correct for the situation described?
2 hH 4 hH
(a) The value of x is 2 (b) The value of x is
20°C
3 3
200°C
(c) The value of x cannot be computed from the information
A B C D
provided
(d) The question is irrelevant as no water comes out from
(a) 120°C (b) 100°C (c) 140 °C (d) 80°C the hole

21 One mole of an ideal gas is taken along the process in 26 Water flows through a horizontal pipe of varying
which pV x = constant. The graph shown represent the cross-section at the rate of 20 litres per minute. Then the
variation of molar heat capacity of such a gas with respect velocity of water at a point where diameter is 4 cm, is
to x. The values of c′ and x′, respectively, are given by (a) 0 . 25 ms− 1 (b) 0 . 26 ms− 1 (c) 0 . 22 ms− 1 (d) 0 . 4 ms− 1
c
27 Oxygen gas having a volume of 1000 cm 3 at 40.0°C and
1.01 × 105 Pa expands until its volume is 1500 cm 3 and its

pressure is 1.06 × 105 Pa. Find the final temperature of
(3/2)R the sample.
x
x¢ (a) 197°C (b) 220 K (c) 300°C (d) 300 K
28 A mercury drop of radius 1.0 cm is sprayed into 10 6
droplets of equal sizes. The energy spent in this process is
5 5 5 5 7 7 5 7
(a) R, (b) R, (c) R, (d) R, [Surface tension of mercury is equal to 32 × 10 −2 Nm −1 ]
2 2 2 3 2 2 2 5
(a) 3.98 × 10−4 J (b) 8.46 × 10−4 J
22 A wire of length L and radius r is fixed at one end. When (c) 3.98 × 10−2 J (d) 8.46 × 10−2 J
a stretching force F is applied at the free end, the
29 A chef, on finding his stove out of order, decides to boil
elongation in the wire is l. When another wire of the same
material but of length 2L and radius 2r, also fixed at one the water for his wife’s coffee by shaking it in a thermos
end is stretched by a force 2F applied at the free end, flask. Suppose that he uses tap water at 15°C and that
then elongation in the second wire will be the water falls 30 cm in each shake, the chef making 30
(a) l/2 (b) l (c) 2l (d) l/4
shakes each minute. Neglecting any loss of thermal
energy by the flask, how long must he shake the flask
23 A Carnot engine has an efficiency of 22.0%. It operates until the water reaches 100°C ?
between constant-temperature reservoirs differing in
(a) 2.25 × 103 min (b) 3.97 × 103 min
temperature by 75.0°C. What are the temperatures of the
(c) 4.03 × 103 min (d) 5.25 × 103 min
two reservoirs?
(a) 58°C, 10°C (b) 78°C, −5 ° C 30 The maximum amount of heat which may be lost per
(c) 68°C, −7 ° C (d) 50°C, 0°C second by radiation by a sphere 14 cm in diameter at a
temperature of 227°C, when placed in an enclosure at
24 A material has a Poisson’s ratio 0.50. If a uniform rod of it
27°C. Given, Stefan's constant = 5.7 × 10− 8 Wm − 2K − 4
suffers a longitudinal strain of 2 × 10−3 , then the
percentage change in volume is (a) 45.48 cal/s (b) 40 cal/s
(c) 42.5 cal/s (d) 40.5 cal/s
(a) 0.6 (b) 0.4 (c) 0.2 (d) zero
31 Four moles of an ideal gas undergo a reversible isothermal
25 An open vessel full of water is falling freely under gravity.
expansion from volume V1 to volume V 2 = 2V1 at temperature
There is a small hole in one face of the vessel, as shown T = 400 K. Find the entropy change of the gas.
in the figure.
(a) 8.22 × 103 J K −1 (b) 8.22 × 102 J K −1
−1
(c) 23.1 J K (d) 10.00 × 103 J K −1
2h/3
32 A swimmer of mass m rests on top of a styrofoam slab,
h
Hole
which has a thickness h and density ρs . The area of the
g slab if it floats in water with its upper surface just awash
H
is (take, density of water to be ρw )
m m m m
(a) (b) (c) (d)
Ground P h (ρs + ρw ) hρw h (ρs − ρw ) h (ρw − ρs )
198 40 DAYS ~ JEE MAIN PHYSICS DAY SEVENTEEN

33 An ice-berg of density 900 kg -m − 3 is floating in water of Direction (Q. Nos. 37-40) Each of these questions
density 1000 kg -m − 3 . The percentage of volume of contains two statements : Statement I and Statement II.
ice-berg outside the water is Each of these questions also has four alternative choices,
only one of which is the correct answer. You have to select
(a) 20% (b) 35% (c) 10% (d) 11%
one of the codes (a), (b), (c) and (d) given below
34 A uniform capillary tube of length l and inner radius r with (a) Statement I is true, Statement II is true; Statement II is
its upper end sealed is submerged vertically into water. the correct explanation for Statement I
The outside pressure is p0 and surface tension of water (b) Statement I is true, Statement II is true; Statement II is
is γ. When a length x of the capillary is submerged into not the correct explanation for Statement I
water, it is found that the water level inside and outside (c) Statement I is true; Statement II is false
the capillary coincide. The value of x is (d) Statement I is false; Statement II is true
l  p r  p r l 37 Statement I A ship floats higher in water on a high
(a) (b) l  1 − 0  (c) l  1 − 0  (d)
 p0r   4γ   2γ   p0r  pressure day than on a low pressure day.
1 +  1 + 
 4γ   2γ  Statement II Floating of ship in the water is possible
35 2 moles of an ideal monoatomic gas is carried from a because of the buoyant force which is present due to the
state ( p0, V0 ) to state ( 2p0, 2V0 ) along a straight line path pressure difference.
in a p-V diagram. The amount of heat absorbed by the 38 Statement I More is the cohesive force, more is the
gas in the process is given by surface tension.
9
(a) 3 p0V0 (b) p0V0 Statement II More cohesive force leads to more shrinking
2
3 of the liquid surface.
(c) 6p0V0 (d) p0V0
2 39 Statement I Water expands both when heated or cooled
36 The stress along the length of a rod (with rectangular from 4°C.
cross-section) is 1% of the Young’s modulus of its materials. Statement II Density of water is minimum at 4°C.
What is the approximate percentage of change of its
volume? (Poisson’s ratio of the material of the rod is 0.3)
40 Statement I If the temperature of a star is doubled, then
the rate of loss of heat from it becomes 16 times.
(a) 3% (b) 1%
(c) 0.7% (d) 0.4% Statement II Specific heat varies with temperature.

ANSWERS
1. (c) 2. (d) 3. (d) 4. (b) 5. (d) 6. (b) 7. (d) 8. (a) 9. (d) 10. (b)
11. (c) 12. (d) 13. (d) 14. (b) 15. (c) 16. (c) 17. (a) 18. (a) 19. (c) 20. (c)
21. (b) 22. (b) 23. (c) 24. (b) 25. (d) 26. (b) 27. (a) 28. (c) 29. (c) 30. (a)
31. (c) 32. (d) 33. (c) 34. (d) 35. (c) 36. (d) 37. (d) 38. (b) 39. (c) 40. (b)
DAY SEVENTEEN UNIT TEST 3 (GENERAL PROPERTIES OF MATTER) 199

Hints and Explanations


1 d 3 floats in d 2 and sinks in 8 Here, h = 3000 m, T1 − T2
× 100 = 100
d1 ⇒ d1 < d3 < d2 . T1
. × 109 Nm − 2
B = 22
2 The forces exerted by liquid on body is ⇒ T1 − T2 = T1
p = hρg = 3000 × 103 × 10
shown in figure, when body is in ⇒ T2 = 0 K = − 27316
. °C
= 3 × 107 Nm − 2
equilibrium w.r.t. fluid. ∆V p 13 If Ds = Db ,
∴Compressional strain = =
V B then Ds 0 + αs Ds 0 ∆T = Db 0 + αb Db 0 ∆T
Vρa V
3 × 107 −2 Ds 0 − Db 0
= = 136
. × 10 So, ∆T =
. × 109
22 αb Db 0 − αs Ds 0
Vρg 3.000 − 2.992
9 Equation of thermal conductivity of the =
l + l2 (19 × 10−6 )(2.992) − (11 × 10−6 )(3.00)
Forces exerted by liquid given combination K eq = 1
l1
+ 2
l = 335° C
So, required force, F = ρV a2 + g 2 K1 K2
The temperature is
x + 4x 5 T = 25° C + 335° C = 360° C
3 Rate of cooling = = K
x 4x 3
dθ Aεσ (T 4 − T 04 ) +
Rc = = K 2K 14 Applying the Bernoulli’s theorem for
dt mc any two convenient points, let us say
Hence, rate of flow of heat through the we are applying just at the water surface
dθ A r2 dθ 1
⇒ ∝ ∝ ⇒ ∝ given combination is and just inside the hole.
dt V r3 dt r θ K eq A(T2 − T1 ) 5/ 3KA(T2 − T1 )
= = ρv 2
( x + 4 x) p0 + 0 + 0 = p0 + + ρg ( − h )
4 Since, pV n = constant and also t 5x 2
pV = RT , taking 1 mol of the gas for 1 / 3KA(T2 − T1 )
= where, v is required speed and water
simplicity dU = C V dt x surface is taken as the reference level.
where, C V → molar specific heat at On comparing it with given equations, ⇒ v = 2 gh = 2 × 9.8 × 10
constant volume 1
we get f = .
Now, the molar specific heat in a 3 = 14 ms −1
polytropic process ∆l
10 Let p1 be the 2p1A 15 Here, σ = 03
., = 4.5 × 10− 3
pV n = constant is given by l
pressure on the 2pA
 R   R  (n − γ )R ∆R / R ∆R ∆l
CV = −  = upper wing σ =− ⇒ =−σ
 γ − 1   n − 1  (n − 1)(γ − 1) surface, then for ∆l / l R l
∆R
…(i) the vertical ⇒ = − 03. × (4.5 × 10− 3
From this equation, we see that C V will equilibrium of the R
mg
be negative when n < γ and n > 1, plane . × 10− 3
= − 135
simultaneously, i.e. 1 < n < γ. Since, γ Volume, V = πR2 l
for all ideal gases is greater than 1, if 2 p1 A + mg = 2 pA
∆V 2∆R ∆l
n > γ or n < 1, then C V will be positive. mg = +
⇒ p1 = p − V R l
2A
5 Work W = p( V2 − V1 ) at constant p ∆V
1.6 × 104 × 10 Percentage change in volume = × 100
= 7 × 104 − V
Initial volume, V1 = ( AT1 − BT12 ) / p 2 × 40 2∆R ∆l 
=  +  × 100
= 6.8 × 104 Pa  R l 
Final volume is V2 = ( AT2 − BT22 ) / p
∆θ . ) + 4.5] × 10− 3 × 100
= [2 × (− 135
= K  
dQ /dt
⇒ W = A(T2 − T1 ) − B (T22 − T12 ) 11
A  ∆x  = 1.8 × 10− 3 × 100 = 018
. %
6 ∆p = 2T Rate of flow of heat per unit area
R = Thermal conductivity × Temperature 16 From the ideal gas law, pV = nRT
As, the number of moles of air gradient pV (100)(1.0 × 10−6 )
n= =
increases. Radius increases and ∆p Temperature gradient (X) RT (8.31)(220)
1
decreases. ∝ = 5.47 × 10−8 mol
Thermal conductivity (K)
7 Heat required to raise the temperature 17
 As , dQ /dt = constant Temperature of the interface,
of water must be equal to the power   K θ + K 2 θ2
output of the heater P, multiplied by  A  θ= 1 1
K1 + K2
time t. As, K c > K m > K g , therefore
 K1 1 
Q cm(T f − T i ) Xc < X m < X g Q = ⇒ If K 1 = K , then K 2 = 4 K 
t = =  K2 4 
P P 12 Efficiency of Carnot’s heat engine
(4190) (0.100) (100° − 23° ) = 100% K × 0 + 4 K × 100
= = 161 s ⇒ θ= = 80°C
200 5K
200 40 DAYS ~ JEE MAIN PHYSICS DAY SEVENTEEN

18 Y = Stress = x x
= = x ε = (T H − TC )/ T H . Thus, 28 Let r be the radius of one droplet.
Strain 2l − l 1 T H = (T H − TC )/ ε 4 3 4 R
Now, πR = 106 × πr 3 , r =
l = (75 K) / (0.22) = 341 K 3 3 100
In actual, the above expression is not 1
= 68° C . = cm = 10−4 m
exact for this much elongation. 100
The temperature of the cold A i = 4 πR 2
19 For constant pressure process, reservoir is
Q 1 = nC p ∆T = 743 J TC = T H − 75 = 341 K − 75K A = 106 × 4 πr 2
f
For constant volume process,
= 266 K = − 7° C. Change in area,
Q 2 = nC V dT = n (C p − R ) dT
= nC pdT − nRdT dV dl ∆A = A f − A i = 4 π × 99 × 10−4 m2
24 = (1 + 2σ )
Q 2 = 743 − 5 × 8.3 × 2 = 660 J V l Increase in surface energy
= 2 × 2 × 10−3 = 4 × 10−3 = S∆ A = 32 × 10−2 × 4 π × 99 × 10− 4 J
20 Let the thermal resistance of each rod
be R. Q σ = 0.5 = 1  = 3.98 × 10−2 J
 2 
The two resistances connected along The increase in surface energy is at the
two paths from B to C are equivalent to ∴Percentage change in volume expense of internal energy, so energy
2 R each and their parallel combination = 4 × 10−1 = 0.4% spent = 3.98 × 10−2 J
is R.
Effective thermal resistance between B 25 As vessel is falling freely under 29 Time taken,
and D = 2R gravity, the pressure at all points cm(T f − T i ) c (T f − T i )
Q
within the liquid remains the same as t = = =
the atmospheric pressure. If we apply Rmgh Rmgh Rgh
R R
Bernoulli’s theorem just inside and (4190) (100 − 15)
=
R R outside the hole, then (30)(9.8)(0.30)
A B C D ρv inside
2
= 4.03 × 103 min
p inside + + ρg H
R R 2
30 Temperature of sphere
ρv outside
2

R q 2R = poutside + + ρgH T = 227° C = 500 K


200°C 20°C 2
A B D Temperature of surroundings
v inside = 0, p inside = poutside = p
T 0 = 27° C = 300 K
Temperature of interface, [atmospheric pressure]
Radius, r = 7 cm = 0.07 m
R θ + R2θ1 Therefore, v outside = 0
θ= 1 2 Area of sphere,
R1 + R2 i.e. no water comes out from the hole. 22
R × 20 + 2R × 200 420 A = 4 πr 2 = 4 × × (0.07)2
θ= = = 140° C 20 × 1000 7
R + 2R 3 26 V = 20 litres/min = m 3 s− 1 . × 10− 2 m2
= 616
60 × (100)3
21 At x = ∞,C = 3 R, from pV x = constant 1 The energy lost from A,
2 = × 10− 3 m 3 s− 1 AE = Aσ(T 4 − T 04 )
3
⇒ p1 / x V = another constant, so at
4
Radius, r = = 2 cm = 0.02 m . × 10−2 × 5.7 × 10− 8 × [(500)4 − ( 300)4 ]
616
x = ∞, V = constant
3
2 . × 5.7 × 10− 10 × 1004 (54 − 34 )
= 616
hence C = C v = R and then Area of cross-section,
2 22 . × 5.7 × 10− 2 × 544 Js− 1
= 616
5 a = πr 2 = × (0.02)2 m 2
C p = CV + R = R 7 The heat lost per sec.
2 Let v be the velocity of the flow of AE . × 5.7 × 10− 2 × 544
616
at x = 0, P = constant and C = C ′ H = =
water at the given point, then J 42
.
5 V = av
hence, C ′ = C p = R = 45⋅ 48 cal / s
2 1 −3 22
⇒ × 10 = × (0.02)2 × v 31 We have p = nR T / V . The work done by
at x = x ′ ,C = 0, so the process is 3 7
adiabatic,hence 7 × 10− 3 the gas during the isothermal expansion
Cp 5 ⇒ v = is
x′ = = 3 × 22 × (0.02)2 V2 V2 dV V
CV 3 W = ∫ pdV = nRT ∫ = nRT ln 2
= 02639
. . ms− 1
− 026
~ V1 V1 V V1
F/A
22 Y = Substituting V2 = 2V1 to obtain W = nRT
∆ l /l 1.06 × 105 × 1000 × 10–6
27 n = PV = = (4.00)(8.314)(400) ln 2
F ×L RT . × 313
831
For the first wire, Y = = 9.22 × 103 J
πr 2 × l = 4.07 × 10–2
2F × 2L Since, the expansion is isothermal,
For the second wire, Y = Using pV = nRT ∆E int = 0 and Q = W
π(2r )2 × l ′
pV (1.06 × 105 ) (1500 × 10−6 ) W
From the above two equations, l ′ = l T = = Thus, ∆S =
nR (4.07 × 10−2 )(8.31) T
23 For an ideal engine, the efficiency is = 470 K 9.22 × 103 J
related to the reservoir temperatures by = = 23.1 J K −1
= 197° C 400 K
DAY SEVENTEEN UNIT TEST 3 (GENERAL PROPERTIES OF MATTER) 201

− 0.3
=  2 ×
32 From equilibrium, ⇒ x=
l 1 
+  × 100 = 0.4%
mg + Ahρs × g = Ahρw × g p r
1+ 0  100 100 
where, A is the required cross-sectional 2γ
area 37 A body of weight w = mg = Vρg float in
m 35 The internal energy, ∆U = nC V ∆T a liquid as a upthrust F = Vσg acts
⇒ A=
h ( ρw − ρs ) C V = specific heat of gas at constant vertically upwards through the centre of
volume gravity of displaced liquid also called
33 Let volume of ice-berg is V and its 3R  4 p 0V 0 the centre of buoyancy. It is
− 0 0 
p V
density is ρ. If V in is volume inside the ⇒ ∆U = n ⋅  independence of atmospheric pressure.
2  nR nR 
water, then
V in σg = Vρg = n⋅
3R 3 p 0V 0

9
= p 0V 0 38 Surface tension, S = Force = F
2 nR 2 Length l
where, σ = density of water
Work done by the gas, ∴S ∝ F
ρ
⇒ V in =   V V 3 p 0V 0 Thus, more the force, the surface
σ W = (2 p 0 + p 0 ) 0 =
2 2 tension is more. Also, this force tends to
σ − ρ
⇒ V out = V − V in =  V From first law of thermodynamics, have the least possible surface area.
 σ 
∆Q = dW + dU 39 At 4°C, the volume of water is
1000 − 900 
=  V =
3 p 0V 0 9
+ p 0V 0 minimum. When it is cooled below 4°C
 1000  2 2 or heated above 4°C, then it expands or
V 12 p 0V 0 its volume increases. As volume at 4°C
= = = 6 p 0V 0
10 2 is minimum, thus its density

V out
= 0.1 = 10%.  = mass  will be maximum.
V 36 Stress = F = 1% of Y = Y 
 volume 

∆A 100
34 The pressure inside tube changes when Y 40 From Stefan’s law, E = σT 4 or E ∝ T 4
it is submerged in water. Thus Stress F / ∆A 100
Also, Y = = = 4 4
p1V1 = p2V2 ∆l / l ∆l / l E 1  T1 
= 
Strain T 
∴ =  
p 0 (lA ) = p ′ (l − x )A ∆l 1 E 2  T2   2T 
⇒ =
p0 l l 100
∴ p′ = E1 1
l − x − ∆r / r or =
Poisson’s ratio, σ = E2 16
As level of water is same inside and ∆l / l
outside of capillary tube ∆r ∆l 0.3 E2 = 16E1 = 16 times
⇒ =−σ =− or
2γ r l 100
∴ p′ − p0 = Specific heat too varies with
r ∆V  2∆r ∆l 
∴ × 100 =  +  × 100 temperature. As a matter of fact, specific
p0 l 2γ V  r l 
or − p0 = heat is zero at 0K for all the materials.
l − x r
DAY EIGHTEEN

Electrostatics
Learning & Revision for the Day
u Electric Charge u Electric Field due to u Electric Potential
u Coulomb’s Law of Forces a Point Charge u Electric Potential Energy
between Two Point Charges u Continuous Charge Distribution u Equipotential Surface
u Superposition Principle u Electric Dipole u Conductors and Insulators
u Electric Field u Electric Flux (φE ) u Electrical Capacitance
u Motion of A Charged Particle u Gauss Law u Capacitor
in An Electric Field

If the charge in a body does not move, then the fricitional electricity is known as static
electricity. The branch of physics which deals with static electricity is called
electrostatics.

Electric Charge
Electric charge is the property associated with matter due to which it produces and
experiences electric and magnetic effects.

Conservation of Charge
We can neither create nor destroy electric charge. The charge can simply be transferred
from one body to another. There are three modes of charge transfer:
(a) By friction (b) By conduction (c) By induction

Quantisation of Charge PREP


Electric charge is quantised. The minimum amount of charge, which may reside
independently is the electronic charge e having a value of 1.6 × 10 –19 C, i.e. Q = ± ne,
MIRROR
Your Personal Preparation Indicator
where, n is any integer.
Important properties of charges are listed below u No. of Questions in Exercises (x)—
u No. of Questions Attempted (y)—
l
Like charges repel while opposite charges attract each other.
u No. of Correct Questions (z)—
l
Charge is invariant i.e. charge does not change with change in velocity. (Without referring Explanations)
l
According to theory of relativity, the mass, time and length change with a change in
velocity but charge does not change. u Accuracy Level (z / y × 100)—
u Prep Level (z / x × 100)—
l
A charged body attracts a lighter neutral body.
l
Electronic charge is additive, i.e. the total charge on a body is the algebraic sum of all In order to expect good rank in JEE,
the charges present in different parts of the body. For example, if a body has different your Accuracy Level should be above
85 & Prep Level should be above 75.
charges as + 2q, + 4q, − 3 q, − q, then the total charge on the body is + 2 q.
DAY EIGHTEEN ELECTROSTATICS 203

Electric field intensity (E) is a vector quantity.


Coulomb’s Law of Forces between The direction of electric field is same as that of force acting on
Two Point Charges the positive test charge. Unit of E is NC−1 or Vm −1.
l
If q 1 and q2 be two stationary point charges in free space
separated by a distance r, then the force of attraction / Electric Field Lines
repulsion between them is
An electric field line in an electric field is a smooth curve,
K |q 1||q2| 1 |q 1||q2|  1 
F = = ⋅ K = 4πε 
tangent to which, at any point, gives the direction of the
r 2
4π ε0 r2  0 electric field at that point.
9 × 10 9 × | q 1|| q2| Properties of electric field lines are given below
= [K = 9 × 10 9 N-m2 /c2 ]
r2 l
Electric field lines come out of a positive charge and go into
l
If some dielectric medium is completely filled between the the negative charge.
given charges, then the Coulomb’s force between them l
No two electric field lines intersect each other.
becomes l
Electric field lines are continuous but they never form a
1 q 1q2 1 q 1q2  ε  closed loop.
Fm = = Q ε = ε r or k 
4πε r 2 4πε 0ε r r 2  0  l
Electric field lines cannot exist inside a conductor. Electric
1 q q
= ⋅ 1 2 shielding is based on this property.
4πkε 0 r 2

Forces between Multiple Charges Motion of a Charged Particle in


When a number of point charges are present in a region then an Electric Field
force acting between any two point charges remains Let a charged particle of mass m and charge q, enters the
unaffected by the presence of other charges and remains same electric field along X -axis with speed u. The electric field E
as according to Coulomb’s law. If four identical charges of is along Y-axis is given by
magnitude q each are placed at the four corners of square of
side a, then the force on any one charge due to the rest of the F y = qE
three charges is and force along X -axis remains zero, i.e.
1 q2 Fx = 0
F = (2 2 + 1)
4πε 0 a2 Y E

P (x, y)
Superposition Principle
It states that, the net force on any one charge is equal to the O X
vector sum of the forces exerted on it by all other charges. If u
there are four charges q 1, q2 , q3 and q 4, then the force on q 1
∴Acceleration of the particle along Y-axis is given by
(say) due to q2 , q3 and q 4 is given by F1 = F12 + F13 + F14,
where F12 is the force on q 1 due to q2 , F13 that due to q3 and Fy qE
ay = =
F14 that due to q 4. m m
The initial velocity is zero along Y-axis (u y = 0).
Electric Field ∴The deflection of charged particle along Y-axis after time t
1
The space surrounding an electric charge q in which another is given by y = u y t + a y t 2
charge q 0 experiences a force of attraction or repulsion, is 2
called the electric field of charge q. The charge q is called the qE 2
= t
source charge and the charge q 0 is called the test charge. The 2m
test charge must be negligibly small so that it does not modify Along X -axis there is no acceleration, so the distance
the electric field of the source charge. covered by particle in time t along X -axis is given by x = ut
Eliminating t , we have
Intensity (or Strength) of Electric Field (E) y=
 qE 
x2 
 2 mu2 
The intensity of electric field at a point in an electric field is
the ratio of the forces acting on the test charge placed at that y ∝ x2
point to the magnitude of the test charge. This shows that the path of charged particle in
F perpendicular field is a parabola.
E= , where, F is the force acting on q 0.
q0
204 40 DAYS ~ JEE MAIN PHYSICS DAY EIGHTEEN

Electric Field due to 4. Electric Field due to a Uniformly Charged


Infinite Plane Sheet
a Point Charge
Electric field near a uniformly charged infinite plane sheet
1. Electric Field due to a Point Charge at a having surface charge density σ is given by
Distance σ
E=
2 ε0
Electric field at a distance r from a point charge q is
1 q
E = ⋅ 5. Electric Field due to a Uniformly Charged
4πε 0 r 2
Thin Spherical Shell
l
If q 1 and q2 are two like point charges, separated by a distance
For a charged conducting sphere/ E r<R
r, a neutral point between them is obtained at a point distant shell of radius R and total charge
r1 from q 1, such that Q, the electric field is given by
r
r1 = Case I E = 0, for r < R
 q2  Q
1 +  Case II E = , for r = R O
R
 q1  4πε 0R 2 Variation of electric field
l
If q 1 and q2 are two charges of opposite nature separated by Q with distance for uniformly
a distance r, a neutral point is obtained in the extended line Case III E = , for r > R charge spherical shell
4πε 0r 2
joining them, at a distance r1 from q 1, such that,
r
r1 =
 q2  Continuous Charge Distribution
 − 1
 q1  The continuous charge distribution may be one dimensional,
2. Electric Field due to Infinitely Long Uniformly two dimensional and three-dimensional.
Charged Straight Wire 1. Linear charge density (λ ) If charge is distributed along a
Electric field at a point situated at a normal distance r, line, i. e., straight or curve is called linear charge
from an infinitely long uniformly charged straight wire distribution. The uniform charge distribution q over a
having a linear charge density λ, is length L of the straight rod.
λ Then, the linear charge density, λ =
q
E =
2 πε 0 r L
Its unit is coulomb metre −1 (Cm–1 ).
3. Electric Field due to a Charged Cylinder
2. Surface charge density (σ) If charge is distributed over a
l
For a conducting charged cylinder of linear charge density
surface is called surface charge density, i. e.,
λ and radius R, the electric field is given by
λ σ =q/A
E = , for r > R,
2 πε 0 r Its unit is coulomb m–2 (Cm–2 )
λ 3. Volume charge density (ρ) If charge is distributed over
E = , for r = R
2 πε 0 R the volume of an object, is called volume charge density,
q
and E = 0, for r < R i. e., p = . Its unit is coulomb metre −3 (Cm–3 ).
V
l
For a non-conducting charged cylinder, for r ≤ R,
λr λ
E = and E = , for r > R
2 πε 0R2 2 πε 0 r Electric Dipole
An electric dipole consists of two equal and opposite charges
E Emax E Emax separated by a small distance.
A p B
r>R
R
r<

r>R –q +q
r

E ∝ 12

2a
r
E

O
The dipole moment of a dipole is defined as the product
O r=R r r=R r of the magnitude of either charges and the distance
(a) Variation of electric field (b) Variation of electric field between them. Therefore, dipole moment
with distance for conducting with distance for
cylinder non-conducting cylinder p = q(2a)
DAY EIGHTEEN ELECTROSTATICS 205

Electric Field due to a Dipole Work Done in Rotating a Dipole


l
At a point distant r from the centre of a dipole, along its If an electric dipole initially kept in an uniform electric field E,
axial line E =
1
⋅ 2
2 pr making an angle θ 1, is rotated so as to finally subtend an angle
4πε 0 (r − a2 )2 θ2 , then the work done for rotating the dipole is,
[direction of E is the same as that of p] W = pE (cos θ 1 − cos θ2 )
For a short dipole,
E=
1 2p
⋅ [r > > a]
Potential Energy of a Dipole
4πε 0 r 3 It is the amount of work done in rotating an electric dipole
from a direction perpendicular to electric field to a particular
l
At a point distant r from the centre of a dipole, along its
direction.
equatorial line
1 p Hence, U = − pE cos θ or U = − p ⋅ E
E= ⋅
4πε 0 (r 2 + a2 )3 / 2
[direction of E is opposite to that of p] Electric Flux (φE )
1 p E
For a short dipole E = ⋅ [r > > a] It is a measure of the flow of
4πε 0 r 3 electric field through a surface. It
can be defined as the total ∆s n
l
At a point distant r from the centre of a short dipole, along
a line inclined at an angle θ with the dipole axis number of lines of electric field
1 p passes through a surface placed
E = ⋅ 3 3 cos2 θ + 1 perpendicular to direction of
4πε 0 r
field.
E Er i.e. φ E = ∫ EdS cos θ = ∫ E ⋅ dS = ∫ E ⋅ n$ dS
β

EQ P (r, θ)
Gauss’s Law
ds
The total electric flux linked with a E
z
1
closed surface is equal to times, the
θ ε0
θ
–q O +q net charge enclosed by that surface.
2d Thus,
E subtends an angle β from r such that 1
φ E = ∫ E ⋅ dS =
l
[Qenclosed ]
1 S ε0
tan β = tan θ
2 i= n
where, Q enclosed Σ q is the algebraic sum of all the charges
i
i= 1
Torque on a Dipole in a Uniform inside the closed surface.
Electric Field
When a dipole is placed in an external electric field, making Electric Potential
an angle θ with the direction of the uniform electric field E, it The amount of work done in bringing a unit positive charge,
experiences a torque given by without any acceleration, from infinity to that point, along
τ = qE × AC any arbitrary path.
τ=p×E V =
W
τ = pE sin θ q0
or qE × 2 d sin θ = (q × 2 d) E sin θ Electric potential is a state function and does not depend on
the path followed.
A
qE
+q 1. Electric Potential Due to a Point Charge
E
2d θ Potential due to a point charge Q, at a distance r is given
1 Q
by V = ⋅
4πε 0 r
qE – C
q B
206 40 DAYS ~ JEE MAIN PHYSICS DAY EIGHTEEN

2. Electric Potential Due to a System of Charges Electric Potential Energy


If a number of charges q 1, q2 , q3 ,… are present in space, The electric energy of a system of charges is the work that has
then the electric potential at any point will be been done in bringing those charges from infinity to near each
V = V1 + V2 + V3 +… other to form the system. For two point charges q 1 and q2
n separated by distance r12 , the potential energy is given by
1 q 1 q2 q3  1  qi  1 q 1q2
= 
4πε 0  r1
+
r2
+
r3
+… =
 4πε 0
∑  
 ri 
U=
4πε 0 r12
.
i= 1

In general, for a system of n charges, the electric potential


3. Electric Potential Due to an Electric Dipole energy is given by
1 p cos θ q iq j
At any general point, V = U= Σ
1
,i ≠ j
4πε 0 r2 2 4πε 0rij
1 p
On the dipole axis, θ = 0 ° and V = ⋅ 1 
4πε 0 r 2
2 is used as each term in summation will appear twice 
On the equitorial axis, θ = 90 ° and V = 0

4. Electric Potential due to Some Common Relation between E and V


Charge Distributions Because E is force per unit charge and V is work per unit
Potential at a point distant r V r < R charge. E and V are related in the same way as work and force.
from an infinitely long wire
having linear charge density λ, Work done against the field to take a unit positive charge from
P
is r>R infinity (reference point) to the given point VP = − ∫ E ⋅ dr volt

λ
V = ⋅ ln r where, the negative sign indicates that the work is done
2 πε 0
O against the field.
r=R r
For a charged conducting
sphere/shell having total charge Q and radius R, the
potential at a point distant r from the centre of the Equipotential Surface
sphere/shell is
Equipotential surface is an imaginary
Q 1 Q
(i) V = , for r > R (ii) V = , for r = R surface joining the points of same
4πε 0r 4πε 0 R potential in an electric field. So, we
Q can say that the potential difference +q
(iii) V = , for r ≤ R
4πε 0R between any two points on an
For a charged non-conducting (dielectric) sphere of radius R, equipotential surface is zero.
the charge Q is uniformly distributed over the entire volume. The electric lines of force at each
Q point of an equipotential surface are
Hence, (i) V = , for r > R normal to the surface. Figure shows
4πε 0r
Q the electric lines of force due to point charge +q. The spherical
(ii) V = , for r = R surface will be the equipotential surface and the electrical
4πε 0R
lines of force emanating from the point charge will be radial
Q 3 R2 − r 2  and normal to the spherical surface.
and (iii) V =  , for r < R
4πε 0  2 R3  Regarding equipotential surface, following points are worth
At the centre of the sphere (r = 0) noting

3Q 3  Q  (i) Equipotential surface may be planar, solid etc. But


V = = Vs Vs = 4πε R  equipotential surface can never be point size.
8 πε 0 R 2  0 
(ii) Equipotential surface is single valued. So, equipotential
V surfaces never cross each other.
r<
R
(iii) Electric field is always perpendicular to equipotential
surface.
r>R (iv) Work done to move a point charge q between two points
on equipotential surface is zero.
O (v) The surface of a conductor in equilibrium is an
r=R r
equipotential.
DAY EIGHTEEN ELECTROSTATICS 207

Conductors and Insulators C1 C2 C3


+Q + – –Q +Q + – –Q +Q + – –Q
Conductors are those materials through which electricity + – + – + –
+ – + – + –
can pass through easily. e.g. metals like copper, silver, iron + – + – + –
etc. Insulators are those materials through which electricity V1 V2 V3
cannot pass through, e.g. rubber, ebonite, mica etc.
+ –

Dielectrics and Polarisation V

Dielectrics are insulating materials which transmit electric The equivalent capacitance Cs is given by
effect without actually conducting electricity. i= n
1 1 1 1 1
e.g. mica, glass, water etc. = + + +… = Σ
Cs C1 C2 C3 i = 1 Ci
When a dielectric is placed in an external electric field, so
the molecules of dielectric gain a permanent electric dipole 2. Parallel Grouping
moment. This process is called polarisation.
In a parallel arrangement, Q = Q1 + Q2 + Q3 +…
and Q1 : Q2 : Q3… = C1 : C2 : C3…
Electrical Capacitance C1
Capacitance of a conductor is the amount of charge needed in +Q1 + – –Q1
+ –
order to raise the potential of the conductor by unity. + –
+ –
Q
Mathematically, Capacitance C = C2
V +Q2 + – –Q2
+ –
+ –
+ –
Sharing of Charges +Q3 C3 –Q3
+ –
l
Let us have two charged conductors having charges + –
Q1 and Q2 (or potentials V1, V2 and capacitances C1, C2 + –

+
respectively). If these are joined together. In such a cases
Q + Q2 C1V1 + C2V2 + –
Common potential, V = 1 =
C1 + C2 C1 + C2 V
l
During sharing of charges, there is some loss of The equivalent capacitance is given by
electrostatic energy, which in turn reappears as heat or i= n
light. The loss of electrostatic energy C p = C1 + C2 + C3 + K = Σ Ci
i= 1
C1C2
∆U = Ui − U f = (V1 − V2 )2
2(C1 + C2 )
l
When charges are shared between any two bodies, their Capacitance of a Parallel Plate
potential become equal. The charges acquired are in the
ratio of their capacitances.
Capacitor
1. Capacitor without Dielectric Medium
Capacitor between the Plates
A B If the magnitude of charge on each plate of a parallel plate
A capacitor is a device which stores +Q –Q capacitor be Q and the overlapping area of plates be A,
electrostatic energy. It consists of
then
conductors of any shape and size
carrying charges of equal l
Electric field between the plates,
magnitudes and opposite signs and σ Q
E = =
separated by an insulating ε0 ε0 A
medium. d l
Potential difference between the plates
There are two types of combination σd Qd
of capacitors: V = E ⋅d = = , where d = separation between the two
ε0 ε0 A
1. Series Grouping plates.
In a series arrangement, V = V1 + V2 + V3 +… Q ε0 A
l
Capacitance, C = =
1 1 1 V d
and V1 : V2 : V3… = : : :…
C1 C2 C3
208 40 DAYS ~ JEE MAIN PHYSICS DAY EIGHTEEN

2. Capacitor with Dielectric Medium between Energy Stored in a Capacitor


the Plates If a capacitor of capacity C is charged to a potential V , the
l
If a dielectric medium of dielectric constant K is electrostatic energy stored in it is,
completely filled between the plates of a capacitor, then its 1
capacitance becomes, U = CV 2
2
Kε A ′  ε A′ 
C′ = 0 = KC0  where, C0 = 0  1
= QV
d  d  2
l
If a dielectric slab/sheet of thickness t (where, t < d) is 1 Q2
introduced between the plates of the capacitor, then =
2 C
ε0 A
C′ =
 t 
d − t + 
 K
Energy Loss During Parallel
l
Magnitude of the attractive force between the plates of a
Combination
parallel plate capacitor is given by When two capacitor of C1 capacitance charge to potential V1 ,
whereas another of C2 charge to potential of V2 , then after
σ A
2
Q 2
CV 2
F = = = parallel combination.
2 ε 0 2 Aε 0 2d
1 C1C2
Loss in energy = (V1 − V2 )2
l
The energy density between the plates of a capacitor 2 C1 + C2
U 1
u= = ε0 E2
Volume 2

DAY PRACTICE SESSION 1

FOUNDATION QUESTIONS EXERCISE


1 Two balls of same mass and carrying equal charge are look like figure given in (figures are schematic and not
hung from a fixed support of length l. At electrostatic drawn to scale) ª JEE Main 2015
equilibrium, assuming that angles made by each thread
is small, the separation, X between the balls is
++++ +++
proportional ª JEE Main (Online) 2013 (a) +
–– ––
+
(b) ++ +
–– –
–– –– –– –––
(a) l (b) l 2 (c) l 2 / 3 (d) l1/ 3
2 In the basic CsCl crystal structure, Cs and − Cl ions are
+

arranged in a bcc configuration as shown below. The ++++


++++
net electrostatic force exerted by the 8 Cs + on the the –– –+
+
(c) –– ––+
+
(d) –
–– –– –– ––
Cl − ion is

Cs+ Cs+ 4 A positively charged particle P enters the region between


two parallel plates with a –
Cs+ Cs+
velocity u, in a direction
Cl –
parallel to the plates. There is
Cs+ a uniform electric field in this u
P
Cs+ E
region. P emerges from this
Cs+ Cs+ region with a velocity v.
Taking C as a constant, v will +
1 4e2 1 16e 2 1 32a 2 depend on u as
(a) (b) (c) (d) zero
4π ε0 3 a 2
4π ε0 3 a 2
4π ε0 3 a 2
(a) v = Cu (b) v = u 2 + Cu
3 A long cylindrical shell carries positive surface charge σ (c) v = u 2 +
C
(d) v = u 2 +
C
in the upper half and negative surface charge − σ in the u u2
lower half. The electric field lines around the cylinder will
DAY EIGHTEEN ELECTROSTATICS 209

5 An infinite line charge produces a field of 9 × 104 N / C at 12 Consider a finite insulated, uncharged conductor placed
a distance of 2 cm. Calculate the linear charge density. near a finite positively charged conductor. The
(a) 10−3 C / m (b) 10−4 C / m (c) 10−5 C / m (d) 10−7 C / m uncharged body must have a potential
ª JEE Main (Online) 2013
6 In a uniformly charged sphere of total charge Q and
radius R, the electric field E is plotted as function of (a) less than the charged conductor and more than at
distance from the centre. The graph which would infinity
correspond to the above will be ª AIEEE 2012 (b) more than the charged conductor and less than at
infinity
E E
(c) more than the charged conductor and more than at
infinity
(a) (b) (d) less than the charged conductor and less than at infinity
13 The electrostatic potential inside a charged spherical ball
R r R r
is given by φ = ar 2 + b, where r is the distance from the
E E centre where a, b are constants. Then the charge density
inside the ball is ª AIEEE 2011

(c) (d) (a) − 6a ε0r (b) − 24 πaε0


(c) − 6a ε0 (d) − 24 πaε0r

R r R r 14 Two points P and Q are maintained at the potentials of


10 V and − 4 V, respectively. The work done in moving
Q 100 electrons from P to Q is ª AIEEE 2009
7 Let ρ(r ) = r be the charge density distribution for a
πR 4 (a) −19 × 10−17 J (b) 9.60 × 10−17 J
solid sphere of radius R and total charge Q. For a point P (c) −2.24 × 10−16 J (d) 2.24 × 10−16 J
inside the sphere at distance r1 from the centre of the 15 The variation of electric potential with distance from a
sphere, the magnitude of electric field is ª AIEEE 2009 fixed point is shown in the figure. What is the value of
Q Qr12 Qr12 electric field at x = 2 m?
(a) zero (b) (c) (d)
4 πε0r12 4 πε0R 4 3 πε0R 4
Y
8 Two points dipoles of dipole moment P1 and P2 are at a 6
distance x from each other an P1 | | P2. The force between
V (volt)

the dipoles is ª JEE Main (Online) 2013


4
1 4P3 P2 1 3 P3 P2
(a) (b)
4 πε0 x 3 4 πε0 x 3
2
1 6P1 P2 1 3 P1 P2
(c) (d)
4 πε0 x 3 4 πε0 x 3 O
1 2 3 4
X
x (m)
9 If the electric flux entering and leaving an enclosed
surface respectively are φ1 and φ 2, the electric charge (a) Zero (b) 6/2
inside the surface will be (c) 6/1 (d) 6/3
φ 2 − φ1 φ1 + φ 2 φ1 − φ 2 16 The electric potentialV at any point ( x , y , z ) in space is
(a) (b) (c) (d) ε0 (φ1 + φ 2 )
ε0 ε0 ε0 given by V = 4x 2 volt. The electric field at (1, 0, 2) m in
Vm −1 is
10 A cylinder of radius R and length L is placed in a uniform
electric field E parallel to the axis of the cylinder, the total (a) 8, along the positive x-axis
(b) 8, along the negative x-axis
electric flux for the surface of the cylinder is
(c) 16, along the x-axis
πR 2 πR 2 + πR 2 (d) 16, along the z-axis
(a) 2 πR 2E (b) (c) (d) zero
E E
17 Assume that an electric field E = 30x 2 i exists in space.
11 A large insulated sphere of radius r, charged with Q units Then, the potential differenceVA − VO , where VO is the
of electricity, is placed in contact with a small insulated potential at the origin andVA the potential at x = 2 m/s
uncharged sphere of radius r′ and is then separated. The ª JEE Main 2014
charge on the smaller sphere will now be (a) 120 J/C (b) −120 J/C
(a) Q (r + r ′) (b) Q (r − r ′) (c) −80 J/C (d) 80 J/C
Q Qr ′
(c) (d)
r′ + r r′ + r
210 40 DAYS ~ JEE MAIN PHYSICS DAY EIGHTEEN

18 Figure shows some equipotential lines distributed in space. 23 A parallel plate capacitor is made of two circular plates
A charged object is move from point A to point B. separated by a distance of 5 mm and with a dielectric of
dielectric constant 2.2 between them. When the electric
field in the dielectric is 3 × 104 V/m, the charge density of
the positive plate will be close to ª JEE Main 2014
A B A B A B (a) 6 × 10−7 C/m 2 (b) 3 × 10−7 C/m 2
(c) 3 × 104 C/m 2 (d) 6 × 104 C/m 2
24 A parallel plate capacitor of capacitance 90 pF is
10V 20V 30V 40V 50V 10V 30V 50V 10V 20V 40V 50V
connected to a battery of emf 20 V. If a dielectric material
Fig. (i) Fig. (ii) Fig. (iii) 5
(a) The work done in Fig. (i) is the greatest of dielectric constant K = is inserted between the
3
(b) The work done in Fig. (ii) is least plates, the magnitude of the induced charge will be
(c) The work done is the same in Fig.(i), Fig. (ii) and Fig. (iii)
ª JEE Main 2018
(d) The work done in Fig. (iii) is greater that Fig. (ii) but
(a) 1.2 nC (b) 0.3 nC (c) 2.4 nC (d) 0.9 nC
equal to that in
25 A parallel plate capacitor having a separation between
19 Two condensers C1 and C2 in a circuit are joined as
the plates d, plate area A and material with dielectric
shown in the figure. The potential of point A is V1 and that
constant K has capacitance C0. Now one-third of the
of B is V2. The potential of point D will be
material is replaced by another material with dielectric
A C1 D C2 B constant 2K, so that effectively there are two capacitors
V1 V2 1
one with area A , dielectric constant 2K and another
(a)
1
(V1 + V2 ) (b) 1 2 + C2V1
CV 3
2 C1 + C2 2
with area A and dielectric constant K. If the
CV + C2V2 C2V1 − CV 3
(c) 1 1 (d) 1 2
C1 + C2 C1 + C2 capacitance of this new capacitor is C then C/C0 is
ª JEE Main (Online) 2013
20 Two capacitors C1 and C2 are charged to 120 V and
4 2 1
200 V respectively. It is found that by connecting them (a) 1 (b) (c) (d)
3 3 3
together the potential on each one can be made zero.
Then ª JEE Main 2013 26 A parallel plate capacitor of area 60 cm 2 and separation
(a) 5 C1 = 3 C2 (b) 3 C1 = 5 C2 3 mm is charged initially to 90 µC. If the medium between
(c) 3 C1 + 5 C2 = 0 (d) 9C1 = 4 C2 the plate gets slightly conducting and the plate loses the
21 A combination of capacitors is set-up as shown in the charge initially at the rate of 2.5 × 10−8 C/s, then what is
figure. The magnitude of the electric field, due to a point the magnetic field between the plates?
charge Q (having a charge equal to the sum of the ª JEE Main (Online) 2013
charges on the 4 µF and 9 µF capacitors), at a point (a) 2.5 × 10−8 T (b) 2.0 × 10−7 T
distance 30 m from it, would equal to (c) 1.63 × 10−11 T (d) Zero
3 µF 27 Case I Identical point charges of magnitude Q are kept
4 µF
9 µF at the corners of a regular pentagon of side a.
Case II One charge is now removed. Match the following
2 µF for above two cases.

Column I Column II
+ –
A. Electric field as the centre of pentagon 1. 1 Q×5
8V
ª JEE Main 2016 (Offline) in case I 4 πε0 a
(a) 240 N/C (b) 360 N/C (c) 420 N/C (d) 480 N/C
B. Electric potential at the centre of 2. Zero
22 An uncharged parallel plate capacitor having a dielectric pentagon in case I
of constant K is connected to a similar air cored parallel 1 Q
C. Electric field at the centre of pentagon 3.
capacitor charged to a potentialV . The two share the 4 πε0 a 2
in case II
charge and the common potential isV ′. The dielectric
constant K is D. Electric potential at the centre of 4. 1 Q
×4
V′ −V V′ −V V ′ −V V −V′ pentagon in case II 4 πε0 a
(a) (b) (c) (d)
V′ + V V′ V V′
DAY EIGHTEEN ELECTROSTATICS 211

Codes (d) Statement I is false, Statement II is true


A B C D A B C D 28 Statement I For a charged particle moving from point P
(a) 2 1 3 4 (b) 1 2 3 4 to point Q, the net work done by an electrostatic field on
(c) 2 3 4 2 (d) 4 1 2 4 the particle is independent of the path connecting point
Direction (Q. Nos. 28-29) Each of these questions P to point Q.
contains two statements : Statement I and Statement II. Statement II The net work done by a conservative force
Each of these questions also has four alternative choices, only on an object moving along a closed loop is zero.
one of which is the correct answer. You have to select one of
the codes (a), (b), (c), (d) given below: 29 Statement I No work is required to be done to move a
(a) Statement I is true, Statement II is false test charge between any two points on an equipotential
(b) Statement I is true, Statement II is true; Statement II is not surface.
the correct explanation of Statement I Statement II Electric lines of force at the equipotential
(c) Statement I is true, Statement II is true; Statement II is the surfaces are mutually perpendicular to each other.
correct explanation of Statement I ª JEE Main (Online) 2013

DAY PRACTICE SESSION 2

PROGRESSIVE QUESTIONS EXERCISE


1 A capacitance of 2 µF is required in an electrical circuit charge Q. The value of A such that the electric field in the
across a potential difference of 1kV. A large number of region between the spheres will be constant is
1µF capacitors are available which can withstand a ª JEE Main 2016 (Offline)
potential difference of not more than 300 V. The minimum Q Q
(a) (b)
number of capacitors required to achieve this is 2 πa 2 2 π(b 2 − a 2 )
ª JEE Main 2017 (Offline) 2Q 2Q
(c) (d)
(a) 16 (b) 24 (c) 32 (d) 2 π(a 2 − b 2 ) πa 2

2 Three concentric metal shells A, B and C of respective 5 A charge Q is placed at each of the opposite corners of a
radii a, b and c (a < b < c ) have surface charge densities square. A charge q is placed at each of the other two
+ σ, − σ and + σ, respectively. The potential of shell B is corners. If the net electrical force on Q is zero, then
Q
ª JEE Main 2018 q
σ a 2 − b 2  σ a 2 − b 2  equals ª AIEEE 2009
(a)  + c (b)  + c
ε0 ε0 1
 a   b  (a) −2 2 (b) –1 (c) 1 (d) −
b 2 − c 2  b 2 − c 2  2
σ σ
(c)  + a (d)  + a
ε0  b  ε0  c 
6 In the given circuit, charge Q 2 on the 2 µF capacitor
changes as C is varied from 1 µF to 3 µF.Q 2 as a function
3 An electric dipole has a fixed dipole moment p, which of C is given properly by (figures are drawn schematically
makes angle θ with respect to X-axis. When subjected to and are not to scale) ª JEE Main 2015
an electric field E1 = E $i , it experiences a torque E1 = τ k$ .
When subjected to another electric field E = 3E $j, it2 1
1mF
experiences a torque T2 = − T1. The angle θ is C
ª JEE Main 2017 (Offline) 2mF

(a) 45° (b) 60° (c) 90° (d) 30°


4 The region between two concentric spheres of radii a
and b, respectively (see the figure), E
A Charge Charge
has volume charge density ρ = ,
r a Q2 Q2
Q (a) (b)
where A is a constant and r is the
distance from the centre. At the C C
1m F 3 mF 1m F 3 mF
b
centre of the spheres is a point
212 40 DAYS ~ JEE MAIN PHYSICS DAY EIGHTEEN

Charge Charge electrostatic potential at the centre of line joining them will
 1 
(c) Q2 (d) Q2
be  = k .
 4πε 0  ª JEE Main (Online) 2013
C C
1 mF 3 mF 1 mF 3 mF
(a) 2 k mg tanθ (b) k mg tanθ

7 A uniform electric field E exists between the plates of a (c) 4 k mg / tanθ (d) 4 k mg tanθ
charged condenser. A charged particle enters the space 13 A point charge of magnitude +1 µC is fixed at ( 0, 0, 0). An
between the plates and perpendicular to E . The path of isolated uncharged spherical conductor, is fixed with its
the particle between the plates is a centre at ( 4, 0, 0). The potential and the induced electric
ª JEE Main (Online) 2013 field at the centre of the sphere is ª JEE Main (Online) 2013
(a) straight line (b) hyperbola (a) 1. 8 × 105 V and −5.625 × 106 V /m
(c) parabola (d) circle
(b) 0 V and 0 V / m
8 The surface charge density of a thin charged disc of (c) 2.25 × 103 V and 5.625 × 102 V /m
radius R is σ. The value of the electric field at the centre (d) 2.25 × 105 V and 0 V / m
σ
of the disc is ⋅ With respect to the field at the centre, 14 Two identical charged spheres are suspended by strings
2ε 0 of equal lengths. The strings make an angle of 30° with
the electric field along the axis at a distance R from the each other. When suspended in a liquid of density
centre of the disc ª JEE Main (Online) 2013 0.8 g cm –3 , the angle remains the same. If density of the
(a) reduces by 71% (b) reduces by 29.3% material of the sphere is 16 gcm –3 , the dielectric constant
(c) reduces by 9.7% (d) reduces by 14.6% of the liquid is ª AIEEE 2010
9 A uniformly charged solid sphere of radius R has (a) 4 (b) 3 (c) 2 (d) 1
potential V0 (measured with respect to ∞) on its surface. 15 A circuit is shown in figure for which C1 = ( 3 ± 0.011) µF,
For this sphere, the equipotential surfaces with C2 = ( 5 ± 0.01) µF and C3 = (1 ± 0.01) µF. If C is the
3V 5V 3V V equivalent capacitance across AB, then C is given by
potentials 0 , 0 , 0 and 0 have radius R1, R 2, R 3,
2 4 4 4
and R 4 respectively. Then, ª JEE Main 2015 C2 C3
(a) R1 = 0 and R 2 > (R 4 − R 3) A B
C1 C1 C1
(b) R1 ≠ 0 and (R 2 − R 1) > (R 4 − R 3)
(c) R1 = 0 and R 2 < (R 4 − R 3)
(d) 2R < R 4 (a) (0.9 ± 0114
. ) µF (b) (0.9 ± 0.01) µF
(c) (0.9 ± 0.023) µF (d) (0.9 ± 0.09) µF
10 A charge Q is uniformly distributed over a long rod AB of
length L as shown in the figure. The electric potential at 16 A thin semi-circular ring of radius r has a positive charge
the point O lying at distance ª JEE Main 2013 q distributed uniformly over it. The net field E at the centre
j
O is ª AIEEE 2010
O A B
L L
Q 3Q i
(a) (b) O
8 π ε0L 4 π ε0L
Q Q In 2 q q
(c) (d) (a) j (b) − j
4 π ε0L In 2 4 π ε0L 4 π 2 ε0r 2 4 π 2 ε0r 2
q q
(c) − j (d) j
11 Two charges, each equal to q, are kept at x = − a and 2 π 2 ε0r 2 2 π 2 ε0r 2
x = a on the x-axis. A particle of mass m and charge
17 Two positive charges of magnitude q are placed at the
q 0 = q /2 is placed at the origin. If charge q 0 is given a
ends of a side 1 of a square of side 2a. Two negative
small displacement ( y < a ) along the y-axis, the net force charges of the same magnitude are kept at the other
acting on the particle is proportional to ª JEE Main 2013 corners. Starting from rest, if a charge Q moves from the
(a) y (b) − y (c) 1/ y (d) − 1/ y middle of side 1 to the centre of square, its kinetic energy
12 Two small equal point charges of magnitude q are at the centre of square is ª AIEEE 2011
suspended from a common point on the ceiling by 1 2qQ  1 
(a) 1 −  (b) zero
insulating massless strings of equal lengths. They come 4 πε0 a  5
to equilibrium with each string making angle θ from the 1 2qQ  1  1 2qQ  2 
(c) 1 +  (d) 1 − 
vertical. If the mass of each charge is m, then the 4 πε0 a  5 4 πε0 a  5
DAY EIGHTEEN ELECTROSTATICS 213

18 Two identical charged spheres suspended from a 4 ρ0r  5 r  ρ0r  5 r 


(c)  −  (d)  − 
common point by two massless strings of length l are 3 ε0  4 R  3 ε0  4 R 
initially a distance d (d < < l ) apart because of their 22 Let C be the capacitance of a capacitor discharging
mutual repulsion. The charge begins to leak from both through a resistor R. Suppose t1 is the time taken for the
the spheres at a constant rate. As a result charges energy stored in the capacitor to reduce to half its initial
approach each other with a velocity v. Then as a function value and t 2 is the time taken for the charge to reduce to
of distance x between them, ª AIEEE 2011
one-fourth its initial value. Then, the ratio t1 / t 2 will be
(a) v ∝ x −1 (b) v ∝ x1/ 2 (c) v ∝ x (d) v ∝ x −1/ 2 ª AIEEE 2010
19 Combination of two identical capacitors, a resistor R and (a) 1 (b) 1 / 2
a DC voltage source of voltage 6 V is used in an (c) 1 / 4 (d) 2
experiment on C-R circuit. It is found that for a parallel 23 The question has statement I and statement II. Of the four
combination of the capacitor, the time in which the choices given after the statements, choose the one that
voltage of the fully charged combination reduces to half best describes the two statements.
its original voltage is 10 s. For series combination, the An insulating solid sphere of radius R has a uniform
time needed for reducing the voltage of the fully charged positive charge density ρ. As a result of this uniform
series combination by half is ª AIEEE 2011 charge distribution, there is a finite value of electric
(a) 20 s (b) 10 s (c) 5 s (d) 2.5 s potential, at the surface of the sphere, at the surface of
20 A resistor R and 2 µF capacitor in series is connected the sphere and also at a point outside the sphere. The
through a switch to 200 V direct supply. Across the electric potential at infinite is zero.
capacitor is a neon bulb that lights up at 120 V. Calculate Statement I When a charge q is taken from the centre of
the value of R to make the bulb light up 5 s after the switch the surface of the sphere its potential energy changes by

has been closed (take, log10 2.5 = 0.4) ª AIEEE 2011 .
3 ε0
(a) 1.7 × 105 Ω (b) 2.7 × 106 Ω
(c) 3.3 × 107 Ω (d) 1.3 × 104 Ω Statement II The electric field at a distance r ( r < R ) from
ρr
21 Let there be a spherically symmetric charge distribution the centre of the sphere is .
3ε 0 ª AIEEE 2012
5 r 
with charge density varying as ρ(r ) = ρ 0  −  upto
4 R (a) Statement I is false, Statement II is true
r = R , and ρ(r ) = 0 for r > R , where, r is the distance from (b) Statement I is true, Statement II is false
(c) Statement I is true, Statement II is true, Statement II is
the origin. The electric field at a distance r (r < R ) from
the correct explanation for Statement II
the origin is given by ª AIEEE 2010
(d) Statement I is true, Statement II is true, Statement II is
4 πρ0r  5 r  ρ0r  5 r 
(a)  −  (b)  −  not the correct explanation of Statement I
3 ε0  3 R  4 ε0  3 R 

ANSWERS

SESSION 1 1 (d) 2 (d) 3 (a) 4 (d) 5 (d) 6 (c) 7 (c) 8 (c) 9 (d) 10 (d)
11 (d) 12 (a) 13 (c) 14 (d) 15 (a) 16 (b) 17 (c) 18 (c) 19 (b) 20 (b)
21 (c) 22 (d) 23 (a) 24 (a) 25 (b) 26 (d) 27 (a) 28 (b) 29 (b)

SESSION 2 1 (c) 2 (b) 3 (b) 4 (a) 5 (a) 6 (d) 7 (c) 8 (a) 9 (c,d) 10 (d)
11 (a) 12 (c) 13 (c) 14 (c) 15 (c) 16 (c) 17 (a) 18 (d) 19 (d) 20 (b)
21 (b) 22 (c) 23 (a)
214 40 DAYS ~ JEE MAIN PHYSICS DAY EIGHTEEN

Hints and Explanations


SESSION 1 5 Let l be the linear charge density. 9 Let - q1 be the charge, due to which flux
q2 Given, distance r = 2 cm = 2 ´ 10-2 m f1 is entering the surface.
1 F = 1 = T sin q
- q1
4pe0 x2 Electric field E = 9 ´ 10 N/C 4
f1 = or q1 = - f1 e0
e0
Support Using the formula of electric field due
to an infinite line charge. Let + q2 be the charge, due to which flux
f2 , is leaving the surface.
q q l
E = q
2pe0r f2 = 2 or q2 = e0f2
T T cos q e0
Dividing and multiplying by 2 to get
q Electric charge inside the surface
1 1
T sin q because, we have the value of = q2 - q1 = e0f2 + e0f1 = e0(f2 + f1 )
F F 4pe0 4pe0
X
2 l 2l 10 As uniform electric field is parallel to the
mg cosq mg E = ´ = cylindrical axis
2 2pe0r 4pe0r
The resultant of components mg cos q Putting the values, we get
ò E × dS = ò E dS cos 90° = 0
and force of repulsion balances, the 2 ´ 9 ´ 109 ´ l Further flux entering the cylinder at one
tension in the string for the equilibrium 9 ´ 104 = -2 end = flux leaving the cylinder at other
massive change. 2 ´ 10 end. Therefore, total electric flux is zero.
T cos q = mg 9 ´ 104 ´ 2 ´ 10-2
l= = 10-7 C/m 11 Common potential,
1 q2 2 ´ 9 ´ 109 Q + 0
tan q = × 2 V =
4pe0 x mg Thus, the linear charge density is 4pe0(r + r ¢ )
x q2 10-7 C/m. Charge on smaller sphere,
=
2l 4pe0 x2 mg 6 Electric field inside the uniformly Qr ¢
4pe0r ¢ ´ V =
x ql charged sphere varies linearly, as r + r¢
Þ µ 2 or x3 µ l or x µ l 1 /3
2l x kQ
E = 3 × r , (r £ R ), while outside the 12 The uncharged body must have a
Thus, we find the separation between R potential less than the charged conductor
the balls is proportional to l1 /3 , where, l sphere, it varies as inverse square of and more than at infinity.
kQ
is length of string. distance, E = 2 ; (r ³ R) which is i.e. V ¥ < V or V > V ¥
r
2. By symmetry resultant force applied by d f
correctly represented in option (c). 13 Electric field, E = = - 2ar
eight charges on corners is zero. dr
7 According to Gauss’ theorem, By Gauss’ theorem,
3 Field lines should originate from q
positive charge and terminate to dr E (4pr 2 ) =
P e0
negative charge. Thus, (b) and (c) are
not possible. Electric field lines cannot r Þ q = - 8pe0ar 3
form corners as shown in (d). Thus, dq dq dr
r= = ´
correct option is (a). R dV dr dV
Eq 1 ö
4 v x = u, t = l , a y = =a = (-24 pe0ar 2 ) æç ÷
u m r1 Q è 4pr 2 ø
ò0 r 4pr 2 dr
l E 4pr12 = pR 4 = - 6 e0 a
e0
14 W = QdV = Q (Vq - V p )
Qr 12
Þ E = = -100 ´ (1.6 ´ 10-19 ) ´ (-4 - 10)
P u 4pe0R 4
= +100 ´ 1.6 ´ 10-19 ´ 14
8 We know that, F = 1 2P13P2 = +2.24 ´ 10-16 J
al 4pe0 r
v y = at =
u P2 15 As, E = dV and around x = 2 m,
Pole P1 dr
As, v 2 = v 2x + v 2y
V = constant
a2 l 2 \ dV = 0 and E = 0
v 2 = u2 +
u2 Pole P2 16 As E and V are related as,
2C P1
= u + 2, -dv -d
u With the help of this relation, we find E = = (4 x2 ) = -8 x
1 6 P1 P2 dx dx
2 C the force between dipole is .
v = u + 2 4pe0 x3 At (1, 0, 2), E = -8(1) = -8
u
DAY EIGHTEEN ELECTROSTATICS 215

17 As we know that, potential difference 22 Initial charge = CV and æ Q ö


Electric potential V = 5 ´ ç ÷
V A - VO is è 4pe0a ø
Final charge = CV ¢ + KCV ¢
VA 2
Since, initial charge = final charge \ B ®1
dV = - Edx, ò dV = - ò 30 x2dx
V - V¢ V - V¢ In case II, the electric field of three
Vo 0 K = K =
V¢ V¢ identically placed charges is zero. Net
2
3 field at the centre O is due to a single
éx ù
V A - VO = - 30 ´ ê ú 23 When free space between parallel plates point charge and is given as
ë 3 û0 of capacitor, the Q
= - 10 ´ [23 - (0)3 ] E =
s 4pe0a2
eletric field, E =
= - 10 ´ 8 = - 80 J/C e0 \ C ®3
18 The work done by a electrostatics force When dielectric is introduced between In case II, the electric potential, being
is given by W12 = q(V2 - V1 ). Here, initial parallel plates of capacitor, scalar, becomes
and final potentials are same in all three s Q
E¢= V = 4´
cases and same charge is moved, so 4pe0a
Ke 0
work done is same in all three cases.
\ D ®4
Electric field inside dielectric
19 Let the potential of point D be V . 28 Work done by conservative force does
s
If q is charge on each condenser, then = 3 ´ 104 not depend on the path. Electrostatic
V1 - V = qC1 Ke 0
force is a conservative force.
Þ V - V2 = qC2 where, K = dielectric constant of
V1 - V C medium = 22 . 29 As, W = q(V A - V B ). At equipotential
Divide = 1 e0 = permittivity of free space
V - V2 C2 surface V A = V B so, W = 0
= 8.85 ´ 10-12
VC1 - V2C1 = V1C2 - VC2 Now, we know that field lines makes
Þ s = 22 . ´ 8.85 ´ 10-12 ´ 3 ´ 104
an angle of 90° with the equipotential
V (C1 + C2 ) = C1V2 + C2V1 ; = 6.6 ´ 8.85 ´ 10-8 surfaces but these are parallel to
C V + C2V1
V = 1 2 = 5841
. ´ 10-7 = 6 ´ 10-7 C/m2 one-another.
C1 + C2
24 Magnitude of induced charge is SESSION 2
20 C1 given by
+ –
Q ¢ = (K - 1) CV 0 1 As each capacitors cannot withstand
120 V 5 more than 300 V, so there should be four
= æç - 1ö÷ 90 ´ 10-12 ´ 20
è3 ø capacitors in each row became in this
C2 condition 1 kV i.e. 1000 V will be
– + = 1.2 ´ 10-9C
divided by 4 (i.e. 250 not more than
Þ Q ¢ = 1.2 nC
200 V 300 V).
For potential to be made zero after 25 As capacitance, Now, equivalent capacitance of one row
connection, the charge on both the e0 A 1
C0 = k = ´ 1 mF = 0.25mF
capacitors are equal. d 4
i.e. q1 = q2 Now equivalent capacitance, [Q in series combination, Ceq = C / n]
\ C1V1 = C2V2 , 120C1 = 200C2 2 K e0 A 2 k e0 A 4 k e0 A Now, we need equivalent of 2mF, so let
C = + =
Þ 3 C1 = 5 C2 3 d 3 d 3 d we need n such rows
21 Resultant circuit, 4k e0 A \ n ´ 0.25 = 2mF
C 4 [Q in parallel combination Ceq = nC ]
3m F \ Ratio = = 3 d =
e0 e0 A 3 2
k n= =8
d 0.25
9m F
4mF 4mF 12mF 26 + \ Total number of capacitors

+ – = number of rows ´ number
+ – 60 cm of capacitors in each row
+ – = 8 ´ 4 = 32
3 mF Q=90 mC
+ –
3 mm 2 Potential of B = Potential due to charge
As, charge on 3mF = 3mF ´ 8V = 24 mC
on A + Potential due to charge on B
\ Charge on 4m F = Charge on 12 mF dQ
Given, = 2.5 ´ 10-8 C/s + Potential due to charge on C.
= 24 mC dt
Charge on 3m F = 3m F ´ 2V = 6mC But in case of capacitor, there is no C
Charge on 9m F = 9 m F ´ 2V = 18 m C magnetic field inside the capacitor i.e. B A
Charge on 4 m F + Charge on 9mF zero.
= (24 + 18)mC = 42mC 27 Point charges are uniformly distributed
\ Electric field at a point distant 30 m around the centre O. Hence, electric +σ
9 ´ 103 ´ 42 ´ 10- 6 field E is zero. –σ
= = 420N/C
30 ´ 30 \ A ®2 +σ
216 40 DAYS ~ JEE MAIN PHYSICS DAY EIGHTEEN

k (Q A + Q B ) kQ C 1 éQ æ r 2 - a2 ö ù 7 + –
\ VB = +
b c E = ê 2 + A 2p ç 2
÷ú
4p e0
ër è r øû
1 é s 4pa2 s 4pb 2 s 4pc 2 ù –
= - + 1 æQ 2
A 2p a ö + – v
4pe0 êë b b c úû E = ç + A 2p - ÷
4p e0 è r 2 r2 ø –
se æ a2 - b 2 c2 ö
= ç + ÷ 1 + E –
e0 è b c ø E = ´ A ´ 2p
4p e0
2 2
s æa - b ö
At the centre of the spheres is a point As the speed of particle is far to the
= ç + c÷
e0 è b ø charge Q. The value of A such that the intensity vector, and there is no
electric field in the region between the acceleration in the direction for to E, but
s æ a2 - b 2 ö
spheres will be constant is there is a electric force exerting on the
VB = ç + c÷
e0 è b ø Q particle (charge) whenever, it motion is
As, Q = 2pAa2 i.e. A =
2pa2 in electric field.
3 Y
Hence, continuously a force exerting on
5 Three forces F41 , F42 and F43 acting on
the particle for to its velocity or speed.
p Q as shown. Hence, path of particle must be parabola.
90° – q Resultant of F 41 + F 43 = 2 Feach As in the projectile motion.
1 Qq
q
X
= 2
4pe0 d 2
8 Given, E 0 = s
2 e0
F43
A
Torque applied on a dipole t = pE sin q
F41 r
where, q = angle between axis of dipole q Q
and electric field. R
For electric field E1 = E$i T
F43 O R
it means field is directed along positive
X direction, so angle between dipole
and field will remain q, therefore torque
Q q Also, we know that
in this direction
1 qR
E1 = pE1 sin q Resultant on Q becomes zero only when Ep =
q charges are of negative nature. 4pe0 (R2 + R2 )3 /2
In electric field E2 = 3 E$j,
it means field is directed along positive \ F =
1 Q ´Q but q = s A = spR2
Y-axis, so angle between dipole and 4pe0 ( 2 d )2 1 spR3 s
Þ Ep = =
field will be 90° - q. dQ Q ´ Q 4pe0 2 2 R3 8 2 e0
Þ 2 2 =
Torque in this direction d 2d 2 \ Reduction in electric field
t2 = pE sin (90° - q) Q ´Q
Þ 2´q = s é 1 ù
= p 3 E1 cos q 2 = 1-
2e0 êë 4 2 úû
According to question, Q Q
\ q = or = -2 2 Clearly, reduction is approx 71%.
t2 = - t1 Þ |t2| = | t1| 2 2 q
9 Potential at the surface of the charged
\ pE1 sin q = p 3 E1 cos q 6 Assume negative terminal of the battery sphere
tan q = 3 as grounded (0 V). Suppose, potential
of point x is V . Q
Þ tan q = tan 60° 1 mF
\ q = 60°
x
4 As, Gaussian surface at distance r from R
centre, C
2 mF
Charged sphere
KQ KQ
E V0 = , V = ,r ³ R
a R r
Q KQ
From the circuit diagram, we can write = 3 (3R2 - r 2 ); r £ R
QC = Q 1 + Q2 2R
KQ
b or C (E - V ) = 1 ´ V + 2 ´ V Vcentre = Vc = ´ 3R2
CE 2R3
or V [C + 3] = CE or V = 3 KQ 3V 0
3+ C = = Þ R1 = 0
r
A 2 CE 2E 2R 2
Q + ò 4pr 2dr \ Q 2 = C2 (V ) = =
r As potential decreases for outside points.
3 + C 1 + 3/C
a = E 4pr 2 Thus, according to the question, we can
e0 As, C varied from 1mF to 3 mF, charge write
4p æ r 2 - a2 ö increases with decreasing slope. 5V KQ
E 4p e0 r 2 = Q + A ç ÷ V R2 = 0 = (3R2 - R22 )
r2 è 2 ø Note As C ® ¥, Q 2 ® 2E = constant 4 2R3
DAY EIGHTEEN ELECTROSTATICS 217

5V 0 V 12 F
Þ = 02 (3R2 - R22 ) In liquid F ¢ =
4 2R K
2 q q where, K = dielectric constant of liquid
5 æ R2 ö
or =3- ç ÷ W ¢ = W - upthrust = Vrg - Vsg
2 èRø
2 q A O B q where, r = density of material,
R 5 1 R
or æç 2 ö÷ = 3 - = or R2 = x Fq
èRø 2 2 2 q s = density of liquid
mg cos q
Similarly, mg mg In air, using Lami’s theorem,
W F
3V 0 KQ 3 KQ 1 q2 =
V R3 = Þ = ´ Force of attraction = sin(90°- q) sin(180°- q)
4 R3 4 R 4pe0 x2
W F
4 Potential at the mid-point O or = …(i)
or R3 = R 1 q 1 q cos q sin q
3 = + In liquid,
4pe0 x 4pe0 x
KQ V 0 W¢ F¢
V R4 = = 1 q =
R4 4 = …(i) cos q sin q
2pe0 x
KQ 1 KQ (As angles are same)…(ii)
Þ = ´ or R 4 = 4R From the figure, On dividing Eq. (i) by Eq. (ii), we get
R4 4 R
1 q2 W F
=
Thus, both options (d) and (c) are 4pe0 x2 W¢ F¢
tan q =
correct. mg W
or K =
W - upthrust
10 L L q2 kq 2
\ mg tan q = k 2
, x= Vrg r 1.6
x mg tan q Þ K = = = =2
O A B Vrg - Vsg r - s 1.6 - 0.8
x dx k
Þ x=q …(ii)
mg tan q 15 The capacitor C2 is shorted, so it is not
æ Q ö dx
2 L kd Q
ç ÷ playing any role in circuit and can be
2L è L ø From Eqs. (i) and (ii), we get
V =ò =ò removed. The 3 capacitors each of C1 are
L x L x Potential = 4 K mg / tan q
2L æ 1 ö
connected in parallel and this is
Q
= dx connected to C3 in series.
4pe0L òL è x ø
ç ÷ y
13
3C1C3 3´3´1
Q Ceq = =C=
= [log e x]2LL 3C 1 + C 3 3´3+ 1
4pe0 L
+1mC = 0.9 mF
Q
= [log e 2L - log e L] x DC 3 DC 1 DC 3 3 DC 1 + D C 3
4pe0L (0,0,0) (4,0,0) So, = + +
C C1 C3 3C 1 + C 3
Q é 2L ù
= log e 4 [For computation of errors
4pe0L ëê L ûú z
Potential at the centre of the sphere. worst has to be taken]
Q
= In(2) DC 3 ´ 0.011 0.01
4pe0L 1 Q 1 1 ´ 10-6 Þ = +
V = = 0.9 3 1
4pe0 r 4pe0 4
11 Fnet = 2F cos q (0.033 + 0.01)
9 ´ 109 ´ 10-6 +
y = = 2.25 ´ 103 V 10
4
Now, electric field at the centre of sphere Þ DC = ± 0.023 mF
– q/2 1 Q 1 ´ 10-6 q
E = 2
= 9 ´ 109 16 Linear charge density l = æç ö÷
q q F 4pe0 r 42 è pr ø
F
= 0.5625 ´ 103 y
x
q

= 5.625 ´ 102 V/m


rd

q a a q
=
dl

dq
14 Each ball in equilibrium have following
F sin q F sin q three forms
q x
(i) tension (ii) repulsive force q O charge elementary length dl
(iii) weight dq = ldl
2F cos q By applying Lami’s theorem,
Net field at O,
q E = ò dE sin q (- j )
2kq æç ö÷
è2ø y T q q K × dq
F net = × = sin q (- j )
2
( y + a ) 2 2 2
y + a 2 q q r¢ ò r2
F F¢ K qr
q
2kq æç ö÷ y 2
E =
r2
ò pr dqsin q (- j ) (Q dl = rdq)
è2ø kq y
F net = Þ µ y W W¢ K q p q
( y 2 + a2 )3 /2 a3 = sin q (- j ) = 2 2 (- j )
p ò0
In air In liquid
r2 2p e0r
218 40 DAYS ~ JEE MAIN PHYSICS DAY EIGHTEEN

2kqQ 2k (-q ) Q Þ vx1 /2 = constant t


17 U i = + Þ R=
a 5a -1 /2 C ln 2 .5
Þ v µ x
t
1 2qQ é 1 ù =
= × 1- 19 If Ce is the effective capacitance, then 2303
. C log 2.5
4pe0 a êë 5 ûú
1 q q = 2 .7 ´ 106W
Uf = 0 VC = V 0, = 0
By conservation of energy, 2 Ce 2 Ce
-t
21 Apply shell theorem, the total charge
q 1Q q upto distance r can be calculated as
As q = q 0 (1 - e RCe ) followed
q dq = 4pr 2 × dr × r
Þ q 0 (1 - e - t / RCe ) = 0
2 5 r
Þ t = RCe ln2 = 4pr 2 × dr × r 0 é - ù
Ö5a ëê 4 R ûú
For parallel grouping,
2a é5 r3 ù
= 4pr 0 ê r 2dr - dr
C ë4 R úû
ræ 5 2 r3 ö
C ò dq = q = 4pr 0 ò0 çè 4 r dr - R dr ÷ø
–q –q é 5 r3 1 r4 ù
2a R = 4pr 0 ê -
ë4 3 R 4 úû
Gain in KE = loss in PE,
Ce = 2 C kq
1 2qQ é1 - 1 ù E =
K = ×
êë \ t 2 = 2 RC ln 2 r2
4pe0 a 5 úû
For series grouping, 1 1 é 5 æ r3 ö r 4 ù
= 2
× 4pr 0 ê ç ÷ - ú
18 At any instant, 4pe0 r ë 4 è 3 ø 4R û
C C
T cos q = mg …(i) r r 5 r
2 E = 0 é - ù
kq 4e0 ëê 3 R ûú
T sin q = Fe = …(ii)
x2
R 22 Energy stored in capacitor,
1 q2 1
U = = (q 0 e - t / t )2
C 2C 2C
q Ce =
2 q 20 - 2 t / t
RC = e [where, t = CR]
l l \ t1 = ln 2 2C
T 2
t2 1 U = U i e -2 t/ t
\ =
t1 4 1
U i = U i e - 2 t1 / t
t 1 10 2
Fe Þ t2 = = 1
x 4 4 = e - 2 t1 / t
Þ t 2 = 2 .5 s 2
mg mg t
Þ t 1 = ln 2
From Eqs. (i) and (ii), we get 20 Neon bulb is filled with gas, so its 2
2
resistance is infinite, hence no current Now, q = q0e - t/ t
kq flows through it.
= mg tan q
x2 1
q 0 = q 0e - t2 / t
(q = magnitude of charge) B 4
mg x x t 2 = t ln 4 = 2t ln 2
Þ 2
q = × × x2 æçQ tan q = ö÷ R t1 1
k 2l è 2l ø \ =
mg 3 C t2 4
2
Þ q = x …(iii)
2kl 23 Statement I is dimensionally wrong
dq 3 mg 2 dx while from Gauss’s law,
Þ 2q = x
dt 2kl dt 4
E S r × pr 3
mg 3 ö
1 /2
dq 3mg 2 E (4pr 2 ) = 3
Þ 2 æç x ÷ = x v Now, Vc = E (1 - e - t / RC ) e0
è 2kl ø dt 2kl
Þ 120 = 200 (1 - e - t / RC ) Þ E =
rr
1 /2
é æ mg x3 ö ù 2 3e0
êQ q = ç ÷ ú Þ e - t / RC = Þ t = RC ln 2 .5
êë è 2kl ø ú 5 Given Statement II is correct.
û
EXAM BITES

This Pdf Is
Downloaded From
www.exambites.in

Visit www.exambites.in for


More Premium Stuffs,Latest
Books,Test Papers,Lectures etc.
jeeneetadda
jeeneetadda_official
jeeneetadda

VISIT NOW !!
DAY NINETEEN

Current
Electricity
Learning & Revision for the Day
u Electric Current u Series and Parallel Combinations of u Kirchhoff’s Laws and their
u Ohm’s Law Resistors Applications
u Electrical Resistivity u Electric Energy and Power u Wheatstone’s Bridge
u Electrical Resistance u Electric Cell u Meter Bridge (Special Case of
Potential Difference and emf of a Cell Wheatstone Bridge)
u Resistance of Different Materials u

Electric Current
Electric current is defined as the amount of charge flowing across any section of wire per
unit time. If charge ∆q passes through the area in time interval ∆t at uniform rate, then
current i is defined as
∆q
i= …(i)
∆t
SI unit of electric current is ampere (A).
l
Conventional direction of flow of current is taken to be the direction of flow of positive
charge or opposite to the direction of flow of negative charge.
l
Electric current is a scalar as it does not follow the law vector of addition.

Current Density PREP


Current per unit area is termed as current density. MIRROR
I Your Personal Preparation Indicator
J = (Am −2 )
A
It is a vector quantity. u No. of Questions in Exercises (x)—
u No. of Questions Attempted (y)—
No. of Correct Questions (z)—
Drift Velocity
u

(Without referring Explanations)


l
Drift velocity is the average uniform velocity acquired by conduction electrons inside
a metallic conductor on application of an external electric field. u Accuracy Level (z / y × 100)—
u Prep Level (z / x × 100)—
The drift velocity is given by the relation
eE
vd = − τ In order to expect good rank in JEE,
m your Accuracy Level should be above
85 & Prep Level should be above 75.
where, τ known as relaxation time.
220 40 DAYS ~ JEE MAIN PHYSICS DAY NINETEEN

Drift velocity per unit electric field is called the mobility of


Resistance of Different Materials
l

the electrons. Thus, mobility,


A perfect conductor would have zero resistivity and a perfect
µ =vd 
= e τ
insulator would have infinite resistivity. Though these are ideal
E m
limits, the electrical resistivity of substances has a very wide
l
In terms of drift speed, electric current flowing through a range. Metals have low resistivity of 10 −8 Ωm to 10 −6 Ωm,
conductor is expressed as I = nAevd
while insulators like glass or rubber have resistivity, some 10 18
where, A = cross-section area of conductor, times (or even more) greater, Generally, good electrical
n = number of conduction electrons per unit volume, conductors like metals are also good conductors of heat, while
vd = drift velocity of electrons insulators like ceramic or plastic materials are also poor
and e = charge of one electron. thermal conductors.

Ohm’s Law V-I Characteristics of Ohmic and


Ohm’s law states that the physical conditions such as Non-ohmic Conductors
temperature, mechanical strain, etc., are kept constant, then
current (i) flowing through a conductor is directly Substances obeying Ohm’s law are called Ohmic resistors, e.g.
proportional to the potential difference across its two ends. metals and their alloys. Substances which do not obey Ohm’s
V law are called non-ohmic resistors, e.g. electrolytes, gases,
i.e. i ∝ V or V ∝ i or V = Ri or = R = a constant, thermionic tubes, transistors, rectifiers, etc.
i
where R depends on the nature of material and it given
dimension.
I I
Electrical Resistivity
The resistance of a resistor (an element in a circuit with some V V
resistance R) depends on its geometrical factors (length, Non-ohmic resistors Ohmic resistors
cross-sectional area) as also on the nature of the substance of
which the resistor is made. Electrical resistance of a
rectangular slab depends on its length (l ) and its Colour Code for Resistors
cross-sectional area ( A). The value of resistance used in electrical and electronic
i. e., R∝l …(i) circuits vary over a wide range. These resistances are usually
1 carbon resistances and a colour code is used to indicate the
and R∝ …(ii) value of resistance.
A
Combining the two relations, we get Their value ranges from kilo-ohm to mega-ohm. Their
percentage accuracy is indicated by a colour code printed on
l
R∝ …(iii) them. Carbon resistors are compact, inexpensive and are used
A in electronic circuits.
ρl
or R= …(iv) Colour Code for Carbon Resistors
A
where, ρ is a constant of proportionality called resistivity. Colour Letter of Number Multiplie Tolerance
m remember r
ρ= 2
ne τ Black B 0 10 0 —
1
Brown B 1 10 —
Resistance of Different Materials Red R 2 10 2 —
Resistance offered by the conductors is minimum while Orange O 3 10 3 —
resistance offered by an insulator is maximum. Yellow Y 4 10 4 —
Semiconductors have resistance which is intermediate to
conductor and insulator. Green G 5 10 5 —
6
Blue B 6 10 —
Electrical Resistance Violet V 7 10 7 —
Grey G 8 10 8 —
Electrical resistance is defined as the ratio in the potential
difference (v) across the ends of the conductor to the current (i) White W 9 10 9 —
V Gold — — 10 −1 5%
flowing through it, i.e. R =
i Silver — — 10 −2 10%
The SI unit of electrical restristance is Ω (ohm) and its No — — — 20%
dimension is [ML2 T −3 A −2 ]. colour
DAY NINETEEN CURRENT ELECTRICITY 221

Now to find the colour coding of carbon resistor, we must The equivalent resistance of resistors in parallel,
remember the bold capital letters of the following sentences: 1 1 1 1
= + + +K
Black Brown ROY Great Britain Very Good Wife Wearing R R1 R2 R3
Gold Silver Necklace
Or
Temperature Dependence
Black Brown Rods of Your Gate Became Very Good When
Given Silver Colour of Resistance
The colours of first two bands A and B correspond to figures Resistance and resistivity of metallic conductors increases
and the colour of the third band C represents multipliers, with increase in temperature. The relation is written as
respectively. The fourth band represents the tolerance.
Rθ = R0 (1 + αθ + βθ2 ) and ρ θ = ρ 0 (1 + αθ + βθ2 )
where, R0 and ρ 0 are values of resistance and resistivity at 0° C
and Rθ and ρ θ at θ° C. α and β are two constants whose value
vary from metal to metal.
A B C D

e.g. Consider a carbon resistor of bands A and B of black and Electric Energy and Power
red colour having figures 0 and 2. The third band C of green
Whenever the electric current is passed through a conductor,
colour having multiplier 10 5.
it becomes hot after short time. This effect is known as
∴ Resistance of the value is given by heating effect of current or Joule heating effect.
R = 02 × 10 5 Ω I 2 Rt
H = W = I 2 Rt joule = cal
But the fourth band D having gold colour, which represents a 418
.
tolerance of ± 5%.
The rate at which work is done by the source of emf in
Hence, the value of carbon resistance is maintaining the effect of current in a circuit is called electric
R = 02 × 10 5 Ω ± 5% power of the circuit,
P = VI watt
Other expressions for power,
Series and Parallel Combinations V2
P = I 2 R watt ⇒ P =
of Resistors R

Series Combination Electric Cell


A series circuit is a circuit in
R1 R2 R3 An electric cell is a device which maintains a continuous flow
which resistors are arranged in a V
chain, so the current has only one of charge (or electric current) in a circuit by a chemical
path to take. The current is the reaction. In an electric cell, there are two rods of different
I
same through each resistor. The metals called electrodes.
total resistance of the circuit is found by simply adding up the
resistance values of the individual resistors. Equivalent Internal Resistance of a Cell
resistance of resistors in series
Thus, when a current is drawn through a source, the potential
R = R1 + R2 + R3 + K difference between the terminal of the source is
V = E − ir
Parallel Combination This can also be shown as below
A parallel circuit is a circuit in I1 R1 E r
which the resistors are arranged I I2 I A B
I
with their heads connected R1 R2
together and their tails connected V I3 R3 V A − E + Ir = VB
together. The current in a parallel or V A − VB = E − Ir
circuit breaks up, with some
flowing along each parallel branch and recombining, when Following three special cases are possible
the branches meet again. The voltage across each resistor is (i) If the current flows in opposite direction (as in case of
parallel is the same. charging of battery), then V = E + Ir
The total resistance of a set of resistors in parallel is found by (ii) V = E , if the current through the cell is zero.
adding up the reciprocals of the resistance values, and then,
taking the reciprocal of the total. (iii) V = 0, if the cell is short circuited.
222 40 DAYS ~ JEE MAIN PHYSICS DAY NINETEEN

This is because current in the circuit,


E
Combination of Cells in Series and
I = or E = Ir
r in Parallel
∴ E − Ir = 0 or V = 0 A group of cells is called a battery. Two common grouping of
cells are

Short circuited 1. Series Grouping


In series grouping, if all the cells are joined so as to supply
E r current in the same direction, then resultant
emf,
Thus, we can summarise, it was follows
E Eeq = E 1 + E2 + E3 + …
r
⇒ However, if one or more cells are joined so as to supply
I
current in reverse direction, then emf of that/those cells is
V = E − ir or V < E
taken as negative, while calculating the equivalent emf.
E r
⇒ E1 E2 E3
–ve
I +ve
V = E + Ir or V > E r1 r2 r3
E r
⇒ The equivalent internal resistance of the cell,
V = E , if I = 0 req = r1 + r2 + r3 + …
If n cells, each of emf E and internal resistance r, are joined
E in series, then
i
r
E eq = nE and req = nr
r
E 2. Parallel Grouping
V = 0 is short circuited
In parallel grouping, if positive terminals of all cells have
been joined at one point and all negative terminals at
Potential Difference and another point, then
1
=
1
+
1
+
1
+…
req r1 r2 r3
emf of a Cell
Electromotive force (emf) of a cell is the terminal potential
difference of cell when it is in an open circuit, i.e. it is not The equivalent emf of the parallel grouping is given by
supplying any current to the external circuit. However, when E eq E 1 E2 E
it is supplying a current to an external resistance, the voltage = + + 3 +…
req r1 r2 r3
across the terminals of cell is called the terminal voltage or
terminal potential difference. E1
+
If E be the emf and r the internal resistance of a cell and a r1
resistance R is joined with it, then current in the circuit,
E E
I = and terminal potential difference,
+ve + 2 –ve
R+r r2
ER
V = IR = E3
(R + r ) +
r3
or V = E − Ir
If n cells, each of emf E and internal resistance r, all joined
Internal resistance of cell, in parallel, then
E −V E 
r =  R = R  − 1 req =
r
 V  V  n
Terminal voltage is more than emf of cell when cell is charged But E eq = E
and it is given by V = E + Ir .
DAY NINETEEN CURRENT ELECTRICITY 223

Kirchhoff’s Laws and their Wheatstone’s Bridge


Applications For measuring accurately any resistance Wheatstone bridge is
widely used. It is an arrangement of four resistances used to
Many electric circuits cannot be reduced to simple series measure one of them in terms of the other three.
parallel combinations. For example, two circuits that cannot
Consider four resistances P, Q, R and S are connected in the
be so broken down are shown in figure
four arms of a quadrilateral according to figure, the bridge is
A B
R1 E1 said to be balanced when galvanometer gives zero deflection,
i.e. potential at point B and D is same (VB = VD ).
D C
R2 E2 In this condition,
P R
=
F E Q S
R3
T1 B
(a)
A
B P Q
C D
R1 P I1
A C
R2 R3 R4 G
I2 I2
S2
R S
I I
E1 E2 E3
D
F E
E G
H S1
I
R5
(b)
However, it is always possible to analyze such circuits by
Meter Bridge
applying two rules, derived by Kirchhoff. (Special Case of Wheatstone Bridge)
This is the simplest form of Wheastone bridge and is specially
Junction Rule useful for comparing resistances more accurately. The
The algebraic sum of the currents at any junction is zero. construction of the meter bridge is shown in the figure. It
consists of one metre resistance wire clamped between two
i.e. Σ i =0 metallic strips bent at right angles and it has two points for
junction
connection.
i2 There are two gaps; in one of whose value is to be determined
i1
is connected. The galvanometer is connected with the help of
i3 jockey across BD and the cell is connected across AC. After
i4 making connections, the jockey is moved along the wire and
the null point is found. Wheatstone bridge, wire used is of
uniform material and cross-section. the resistance can be
This law can also be written as, ‘‘the sum of all the currents found with the help of the following relation
directed towards a point in circuit is equal to the sum of all R S
the currents directed away from that point.’’
Thus, in figure, i 1 + i2 = i3 + i 4 B
The junction rule is bases on conservation of electric charge.
G

Loop Rule A D C
The algebraic sum of the potential difference in any loop I1 100 I1
including those associated emf’s and those of resistive
elements, must be equal to zero. Metre sale
That is, Σ ∆V = 0
closed loop
This law represent conservation of energy. V K1
Applying Kirchhoff’s law for the following circuit, we have
R l1 l1
Resulting equation is = or R =S
S (100 − l1) 100 − l1
Vr1 + Vr2 + Vr3 − 10 = 0.
224 40 DAYS ~ JEE MAIN PHYSICS DAY NINETEEN

Potentiometer Applications of Potentiometer


Principle Potentiometer is an ideal device to measure the (i) To find emf of an unknown battery
potential difference between two points. It consists of a long
resistance wire AB of uniform cross-section in which a steady E1 E1
direct current is set up by means of a battery.
E1 i i i i

l1 l1
A B A B
i i i C1 i C2
L i2 =0 i2 =0
A B G G
i C Ek EU
i2 =0
G We calibrate the device by replacing E2 by a source of known
E2 , r
emf E k and then by unknown emf Eu . Let the null points are
Potential gradient, obtained at lengths l1 and l2 .
Potential difference across AB Then, E K = i( ρl1) and EU = i ( ρl2 )
k =
Total length Here, ρ = resistance of wire AB per unit length
V l 
= AB ∴
E K l1
= or EU =  2  E K
L EU l2  l1 
iR AB
= = iλ So, by measuring the lengths l1 and l2 , we can find the emf of
L
R AB an unknown battery.
where, λ = = resistance per unit length of
L (ii) To find the internal resistance of a cell
potentiometer wire.
Firstly, the emf E of the cell is balanced against a length
The emf of source balanced between points B and C AD = l 1. For this, the switch S′ is left opened and S is closed. A
E2 = kl known resistance R is then connected to the cell as shown.
R The terminal voltage V is now balanced against a smaller
= i AB × l = iRCB length AD′ = l2 . Here, now switch is opened and S′ is closed.
l
l1
Here, AB is a long uniform resistance wire (length AB may be
ranging from 1 m to 10 m). E 0 is a battery whose emf is known l2
S
supplying a constant current I for flow through the
E
potentiometer wire. If R be the total resistance of
potentiometer wire and L its total length, then potential
gradient, i.e. fall in potential per unit length along the D′ D
A B
potentiometer will be
V IR (E, r)
k = = G
L L S′
E 0R
=
(R0 + R) L R

where, E 0 = emf of battery, E l1


Then, =
R0 = resistance inserted by means of rheostat Rh V l2
E R+r
k = potential gradient. Since, = {Q E = i(R + r ) and V = iR}
V R
L → balancing length
R + r l1 l 
J → jockey. or = ⇒ r =  1 − 1 R
R l2  l2 
DAY NINETEEN CURRENT ELECTRICITY 225

DAY PRACTICE SESSION 1

FOUNDATION QUESTIONS EXERCISE


1 A wire of length L and 3 identical cells of negligible
internal resistance are connected in series.Due to the (a) P and R (b) P and Q
current, the temperature of the wire is raised by ∆T in a (c) Q and R (d) Any two points
time t. A number N of similar cells is now connected in 7 An infinite ladder network of resistances is constructed
series with a wire of the same material and cross-section with 1 Ω and 2 Ω resistance as shown in figure. The
but of length 2L. The temperature of the wire is raised by 6 V battery between A and B has negligible
the same amount ∆T in the same time t. The value of N is internal resistance, then effective resistance between
(a) 4 (b) 6 (c) 8 (d) 9 A and B is
2 In a large building, there are 15 bulbs of 40W, 5 bulbs of 1W 1W 1W
100W, 5 fans of 80 W and 1 heater of 1kW. The voltage A
of the electric mains is 220 V. The minimum capacity of
the main fuse is ª JEE Main 2014 6V 2W 2W 2W

(a) 8A (b)10A (c) 12A (d) 14A


B
3 Two electric bulbs marked 25 W-220 V and 100 W-220 V
are connected in series to a 440 V supply. Which of the (a) 3 Ω (b) 2 Ω
bulbs will fuse? ª AIEEE 2012 (c) 6/5 Ω (d) 5/6 Ω
(a) Both (b) 100 W (c) 25 W (d) Neither 8 In the circuit in the figure below, the potential difference
across P and Q will be nearest to
4 A given resistor has the following colour scheme of the
various strips on it brown, black, green and silver. Its
value in ohm is 100 W
(a) 1.0 × 104 ± 10% (b) 1.0 × 105 ± 10% 48 V
80 W Q
(c) 1.0 × 106 ± 10% (d) 1.0 × 107 ± 10%
100 W 20 W P
5 Correct set up to verify Ohm’s law is
ª JEE Main (Online) 2013
A
V V (a) 9.6 V (b) 6.6 V
(a) (b) A
(c) 4 V (d) 3.2 V
9 The charge on the 4 µF capacitor in the steady state, is
5W 2 mF
A V
(c) (d)
10 V 5W
V A
4 mF
6 Six equal resistances are connected between points P , Q
and R as shown in figure. Then net resistance will be 20 10
(a) µC (b) µC
maximum between ª JEE Main (Online) 2013 3 3
5
P (c) µC (d) 10µC
6

10 The temperature dependence of resistances of Cu and


undoped Si in the temperature range 300-400 K, is best
r
r r described by ª JEE Main 2016 (Offline)
r
(a) linear increase for Cu, linear increase for Si
r (b) linear increase for Cu, exponential increase for Si
(c) linear increase for Cu, exponential decrease for Si
Q R (d) linear decrease for Cu, linear decrease for Si
r
226 40 DAYS ~ JEE MAIN PHYSICS DAY NINETEEN

11 When an electric heater is switched ON, the current (a) 5.5 Ω (b) 3.5 Ω
flowing through it is plotted against time (t). Taking (c) 4.5 Ω (d) 2.5 Ω
into account the variation of resistance with 15 Two batteries of emf E 1 and E 2(E 2 > E 1) and internal
temperature, which of the following best represents resistances r1 and r2, respectively are connected in parallel
the variations as shown in figure.
I I E1
r1

A B
(a) (b)
r2
E2

t t (a) Two equivalent emf Eeq of the two cells is between E1 and
I I E 2 , i.e. E 1 < E eq < E 2
(b) The equivalent emf Eeq is smaller than E1
(c) The Eeq is given by Eeq = E1 + E 2 always
(c) (d)
(d) Eeq is independent of internal resistance r1 and r2
16 Match the following columns.
t t
Column I Column II
12 Two bars of radii r and 2r are kept in contact as A. Smaller the resistance 1. If the same voltage is
shown. An electric current i is passed through the greater the current applied
bars. Which one of the following is correct? B. Greater the resistance 2. If the same current is
smaller the current passed
l/2 C. Greater the resistance 3. When resistances are
l/2 smaller the power connected in series
2r r
i D. Greater the resistance 4. When resistances are
C greater the power connected in parallel
A B
Codes
(a) Heat produced in bar BC is 4 times the heat A B C D A B C D
produced in bar AB (a) 1 2 3 4 (b) 4 1 2 3
(b) Electric field in both halves is equal (c) 4 3 1 2 (d) 3 4 2 1
(c) Current density across AB is double that of across
BC
17 In the circuit shown, battery, ammeter and voltmeter are
ideal and the switch S is initially closed as shown in figure.
(d) Potential difference across AB is 4 times that of
across BC When switch S is opened, match the parameter of column I
with the effects in column II.
13 Which of the four resistances P , Q , R and S generate R R
the greatest amount of heat when a current flows from
A to B ? ª JEE Main (Online) 2013
V
S
P=2W Q=4W
A
E
A B
R=1W S=2W Column I Column II

(a) Q (b) S (c) P (d) R A. Equivalent resistance across the battery 1. Remains same
14 A DC source of emf E 1 = 100 V E1
r=0.5 Ω B. Power dissipated by left resistance R 2. Increases
and internal resistance r = 0.5 Ω,
C. Voltmeter reading 3. Decreases
a storage battery of emf
E 2 = 90 V and an external E2
D. Ammeter reading 4. Becomes zero
resistance R are connected as
Codes
shown in figure. For what value R
of R no current will pass through the battery? A B C D A B C D
ª JEE Main (Online) 2013 (a) 2 3 3 3 (b) 3 2 2 2
(c) 4 1 1 1 (d) 1 4 4 4
DAY NINETEEN CURRENT ELECTRICITY 227

18 In the circuit shown in figure. The point F is grounded. 22 In the below circuit, the current in each resistance is
Which of the following is wrong statement? ª JEE Main 2017 (Offline)
5W 2V 2V 2V
B C

2W 3W
1Ω 1Ω 1Ω
A D
10 V 3V
4W
F E 2V 2V 2V
(a) 0.25 A (b) 0.5 A (c) 0 A (d) 1 A
(a) D is at 5 V 23 Which of the following statements is false?
(b) E is at zero potential ª JEE Main 2017 (Offline)
(c) The current in the circuit will be 0.5 A
(d) None of the above (a) In a balanced Wheatstone bridge, if the cell and the
galvanometer are exchanged, the null point is disturbed
19 The reading of ammeter as shown in figure is, (b) A rheostat can be used as a potential divider
2W (c) Kirchhoff’s second law represents energy conservation
(d) Wheatstone bridge is the most sensitive when all the four
6W 3W
resistances are of the same order of magnitude
A 24 In a metre bridge experiment, null point is obtained at
3W 6W 40 cm from one end of the wire when resistance X is
balanced against another resistanceY . If X < Y , then the
new position of the null points from the same end, if one
(14 V, 0.8 W)
decides to balance a resistance of 3X against Y , will be
(a) 6.56 A (b) 3.28 A close to ª JEE Main (Online) 2013
(c) 2.18 A (d) 1.09 A (a) 80 cm (b) 75 cm (c) 67 cm (d) 50 cm
20 What is the potential difference between points A and D 25 On interchanging the resistances, the balance point of a
of circuit as shown in figure? meter bridge shifts to the left by 10 cm. The resistance of
their series combination is1 kΩ. How much was the
2W 4V resistance on the left slot before interchanging the
A C
resistances? ª JEE Main 2018
5V
(a) 990 Ω (b) 505 Ω (c) 550 Ω (d) 910 Ω
6V 3W 6W
26 In an experiment to measure the internal resistance of a
1W 4W
D
cell by potentiometer, it is found that the balance point is
B at a length of 2 m when the cell is shunted by a 4 Ω
resistance; and is at a length of 3 m when the cell is
(a) 5 V (b) 9 V
(c) 10.4 V (d) 11.4 V shunted by a 8 Ω resistance. The internal resistance of
the cell is, then
21 In the circuit shown below, the current in the1 Ω resistor (a) 12 Ω (b) 8 Ω (c) 16 Ω (d) 1 Ω
is ª JEE Main 2014
27 A 6 V battery is connected to the terminals of a 3 m long
6V
P 2Ω
wire of uniform thickness and resistance of 100 Ω. The
difference of potential between two points on the wire
separated by a distance of 50 cm will be
1Ω 9V (a) 2 V (b) 3 V (c) 1 V (d) 15 V
28 The current in the primary circuit of potentiometer is 0.2A.
3Ω Q 3Ω The specific resistance and cross-section of the
(a) 1.3A, from P to Q potentiometer wire are 4 × 10−7 Ω m and 8 × 10−7 m 2
(b) 0A respectively. The potential gradient will be equal to
(c) 0.13 A, from Q to P ª AIEEE 2011
(d) 0.13 A, from P to Q (a) 0.2 V/m (b) 1 V/m (c) 0.3 V/m (d) 0.1 V/m
228 40 DAYS ~ JEE MAIN PHYSICS DAY NINETEEN

Direction (Q. Nos. 29-32) Each of these questions contains higher temperature. The null point can be obtained at the
two statements : Statement I and Statement II. Each of these same point as before by decreasing the value of the
question also has four alternative choices, only one of which standard resistance.
is the correct answer. You have to select one of the codes (a),
(b), (c), (d) given below Statement II Resistance of a metal increases with
(a) Statement I is true, Statement II is true; Statement II is increase in temperature.
the correct explanation for Statement I 32 Statement I In the potentiometer circuit shown in figure,
(b) Statement I is true, Statement II is true; Statement II is E 1 and E 2 are the emfs of cells C1 and C2 respectively
not the correct explanation for Statement I with E 1 > E 2. Cell C1 has negligible internal resistance. For
(c) Statement I is true; Statement II is false a given resistor R, the balance length is x. If the diameter
(d) Statement I is false; Statement II is true of the potentiometer wire AB is increased, the balance
length x will decrease.
29 Statement I As temperature decreases, the relaxation C1 R
time of a conducting material increases.
Statement II Number of collisions per unit time of
electrons with lattice ions increases as the x D
A B
temperature increases.
30 Statement I Potential difference across the terminals of a
battery can be greater than its emf.
C2 G
Statement II When current is taken from battery,
V = E − Ir (Symbols have their usual meaning). Statement II At the balance point, the potential
31 Statement I In a meter bridge experiment, null point for difference between AD due to cell C1 = E 2, the emf of
an unknown resistance is measured. Now, the unknown cell C 2 .
resistance is put inside an enclosure maintained at a

DAY PRACTICE SESSION 2

PROGRESSIVE QUESTIONS EXERCISE


1 If a wire is stretched to make it 0.1% longer, its 4 A letter A is constructed of a uniform wire with resistance
resistance will ª AIEEE 2012 1.0 Ω per cm. The sides of the letter are 20 cm and the
(a) increase by 0.2% (b) decrease by 0.2% cross piece in the middle is 10 cm long. The apex angle
(c) decrease by 0.05 % (d) increase by 0.05% is 60°. The resistance between the ends of the legs is
2 In the given circuit diagram, when the current reaches close to ª JEE Main (Online) 2013
steady state in the circuit, the charge on the capacitor of (a) 50.0 Ω (b) 10 Ω
capacitance C will be (c) 36.7 Ω (d) 26.7 Ω
E r 5 Two batteries with emf 12 V and 13 V are connected in
r1 parallel across a load resistor of10 Ω. The internal
resistances of the two batteries are1 Ω and 2 Ω,
C r2 respectively. The voltage across the load lies between
ª JEE Main 2018
ª JEE Main 2017 (Offline)]
r1 r2 (a) 11.6 V and 11.7 V (b) 11.5 V and 11.6 V
(a) CE (b) CE (c) 11.4 V and 11.5 V (d) 11.7 V and 11.8 V
(r2 + r ) (r + r2 )
r1 6 In a potentiometer experiment, it is found that no current
(c) CE (d) CE
(r1 + r ) passes through the galvanometer when the terminals of
the cell are connected across 52 cm of the potentiometer
3 When 5V potential difference is applied across a wire of wire. If the cell is shunted by a resistance of 5 Ω, a
length 01 . m, the drift speed of electrons is 2 . 5 × 10−4 ms −1.
balance is found when the cell is connected across
If the electron density in the wire is 8 × 1028m −3 the
40 cm of the wire. Find the internal resistance of the cell.
resistivity of the material is close to ª JEE Main 2015
ª JEE Main 2018
(a) 1. 6 × 10−8 Ωm (b) 1. 6 × 10−7 Ωm
(a) 1 Ω . Ω
(b) 15 (c) 2 Ω (d) 2.5 Ω
(c)1. 6 × 10−6 Ωm (d) 1. 6 × 10−5 Ωm
DAY NINETEEN CURRENT ELECTRICITY 229

7 Two conductors have the same resistance at 0°C but (a) 40 loge 4 (b) 30 loge 3
their temperature coefficients of resistance are α 1 and α 2. (c) 20 loge 2 (d) 10 loge 2
The respective temperature coefficients of their series 11 The potential difference across 8Ω resistance is 48V as
and parallel combinations are nearly ª AIEEE 2010
shown in figure. The value of potential difference across
α1 + α 2 α1 + α 2
(a) , α1 + α 2 (b) α1 + α 2 , points A and B will be
2 2
α1α 2 α + α 2 α1 + α 2
(c) α1 + α 2 , (d) 1 , A 3Ω
α1 + α 2 2 2
8 There are two concentric spheres of radius a and b 20 Ω 30 Ω 60 Ω
respectively. If the space between them is filled with
medium of resistivity ρ, then the resistance of the
intergap between the two spheres will be
24 Ω 8Ω 48 V
ρ ρ  1 − 1
(a) (b)  
4 π (b + a) 4π b a
B 1Ω
ρ  1 1 ρ  1 1
(c)  −  (d)  − 
4 π  a2 b 2  4π a b  (a) 62 V (b) 80 V (c) 128 V (d) 160 V
9 To find the resistance of a 12 The V - I graph for a conductor at temperatures T1 and T2
galvanometer by the half i are as shown in the figure, The termT2 − T1 is proportional
deflection method the following to
circuit is used with resistances R1 R2
T2

Voltage (V )
R1 = 9970 Ω, R 2 = 30 Ω and
R 3 = 0. The deflection in the
galvanometer is d. With T1
G R3
R 3 = 107 Ω the deflection
d q
changed to ⋅ The galvanometer q
2 Current (I)
resistance is approximately ª JEE Main (Online) 2013
(a) cos 2θ (b) sin 2θ (c) cot 2θ (d) tan 2θ
(a) 107 Ω (b) 137 Ω (c) 107 / 2 Ω (d) 77 Ω
13 The charge supplied by source varies with time t as
10 A source of emf E = 10 V and having negligible internal
Q = at − bt 2. The total heat produced in resistor 2R is
resistance is connected to a variable resistance. The
resistance varies as shown in figure. The total charge R
that has passed through the resistor R during the time
interval from t1 to t 2 is +
R Source R 2R

40 Ω
a 3R a 3R
(a) (b)
20 Ω 6b 27b
a 3R
(c) (d) None of these
t 3b
t1 = 10 s t2 = 30 s

ANSWERS

SESSION 1 1 (b) 2 (c) 3 (c) 4 (c) 5 (a) 6 (b) 7 (b) 8 (d) 9 (a) 10 (c)
11 (b) 12 (a) 13 (b) 14 (c) 15 (a) 16 (a) 17 (a) 18 (b) 19 (c) 20 (c)
21 (c) 22 (c) 23 (a) 24 (c) 25 (c) 26 (b) 27 (c) 28 (d) 29 (a) 30 (b)
31 (d) 32 (d)

SESSION 2 1 (a) 2 (b) 3 (d) 4 (d) 5 (b) 6 (b) 7 (d) 8 (d) 9 (d) 10 (d)
11 (d) 12 (c) 13 (b)
230 40 DAYS ~ JEE MAIN PHYSICS DAY NINETEEN

Hints and Explanations


SESSION 1 Thus, in order to study Ohm’s law 10 As, we know copper is a conductor, so
ρ2L experimentally, voltmeter (V) should be
1 Here, R = ρL ; R1 =
increase in temperature, increases the
= 2R connected parallel to the resistor. resistance. Then, silicon (Si) is
A A However, ammeter (A) should be semiconductor, so with increase in
Mass connected in series with the resistor. temperature, resistance will decrease.
As, Density =
Volume 6 By solving this, we get net resistance as, 11 The filament of the heater reaches its
m 5 4
i.e. d = R PQ = r , RQR = r steady resistance when the heater
AL 11 11 reaches the steady temperature, which is
3 much higher than room temperature. The
∴ m = ALd ; m1 = A 2Ld = 2m and R PR = r resistance at room temperature is then
11
Now, heat produced in first case much lower than the resistance of its
R PQ > RQR > R PR steady state.
(3E )2
H1 = × t = ms∆T …(i) Therefore, R PQ is maximum.
R When the heater is switched ON, it
7 Let R be the resistance of infinite ladder. draws a larger current than its steady
In the second case, state current as the filament heats up, its
The addition or subtraction of one step
(NE )2 resistance increases and current falls to
H2 = × t = 2ms∆T …(ii) in the ladder will not affect the total
2R resistance of network. Therefore, steady state value.
equivalent circuit will be as shown in 12 Current flowing through both the bars is
On solving Eqs. (i) and (ii), we get
figure. equal. Now, the heat produced is given by
N =6
1W H = I 2 Rt
2 Total power (P ) consumed
or H ∝R
= (15 × 40) + (5 × 100) + (5 × 80) 6V 2W R H AB R (1 / 2r )2
or = AB =
+ (1 × 1000) H BC R BC (1 / r )2
= 2500 W
Q R ∝ 1 ∝ 1 
2× R  
As we know that, Total resistance = 1 + =R  A r2 
R+2 1
Power, i.e. P = VI =
2500 125 or R + 2 + 2R = R2 + 2R 4
⇒ I = A=
220 11 ⇒ R2 − R − 2 = 0 or H BC = 4 H AB
= 11.3 A On solving, we get 13 We know that, I ∝ 1 ,
Minimum capacity should be 12 A. R =2Ω R
2 I1 2W 4W
3 Resistance of bulb is given by R = V . 8 Total resistance of circuit
P = 100 + 100 + 80 + 20 = 300 Ω
As the rated power of bulb 25 W is less 48 I2
Current I = = 016
. A A B
than 100 W, it implies that 25 W bulb 300
1W 2W
has higher resistance. As in series Potential difference across P
6 2
combination, current through both the and Q = 20 × 016 . = 3.2 V Here, I2 = I = I
3+ 6 3
bulbs is same, so heating in 25 W bulb 9 In the steady state, A
is more than that of 100 W bulb. So, 25 the capacitors are 3 1
5W or I1 = I = I
W bulb will get fused. fully charged and 6+ 3 3
acts as open circuit,
4 Numbers attached for brown, black, so the equivalent 10 V 5W Power rate in 2 Ω of upper series
green and silver are 1, 0, 5, ± 10 %. circuit in steady 2
= 2 ×  I  = I 2
1 2
Therefore, the resistance of given state would be as
shown alongside 3  9
resistor
figure. B Power rate in 4 Ω of upper series
= 10 × 105 Ω ± 10% 10
Steady state current I = = 1A 2
= 4 ×  I  = I 2
= 1.0 × 10 Ω ± 10%
6 1 4
5+ 5
3  9
5 Ohm’s law states that the current (I ) So, potential drop across AB is
V = 5V Power rate of 1 Ω in lower series
flowing through a conductor is directly 2
= 1 ×  I  = I 2
Sum of potential difference across 2µF 2 4
proportional to the potential difference
and 4 µF capacitors is 5V. As capacitors 3  9
(V ) across its ends provided its physical
are in series, charges on them would be
conditions such as temperature, same, let us say it is q. Power rate of 2 Ω in lower series
2
mechanical strain, etc. kept constant,
= 2 ×  I  = I 2
q q 2 8
From KVL, + =5
i.e. I ∝ V or V ∝ I 2 4 3  9
or V = RI (where, R is constant) 20 ∴ Greatest amount of heat is generated
⇒ q = µC
3 by S.
DAY NINETEEN CURRENT ELECTRICITY 231

−1
V  =
14 Given, E1 = 100 V, 2W 23
or
 15 5
r = 0.5Ω, −3
E2 = 90 V 2W 2W or V = = −0.13 V
A 23
External resistance = R ∴ Current = −1 × 013.
For no current pass through the battery = − 013. A
100 90
= 22 Each resistance is connected with two
R+ r R 14 V, 0.8 W
cells combined in opposite direction, so
10 9 2W
⇒ = potential drop across each resistor is
1 R
R+ Ω zero. Hence, the current through each of
2 4W resistor is zero.
⇒ 10R = 9R + 4.5 Ω A
∴ R = 4.5 Ω 23 In a balanced Wheatstone bridge, there is
no effect on position of null point, if we
15 The equivalent emf of this combination exchange the battery and galvanometer.
is given by 14 V, 0.8 W
So, option (a) is incorrect.
E2 r1 + E1 r2 Total resistance of the circuit 40 40
E eq = 24 As, x = =
r1 + r2 2× 4 8 12.8
= + 0.8 = + 0.8 = Ω y 100 − 40 60
2+ 4 6 6
= 3  =
This suggest that the equivalent emf 3x 40 120 2
Main current in the circuit =
E eq of the two cells is given by y  60  60 1
14 84
E 1 < E eq < E2 . = = A
(12.8 / 6) 12.8 Now, the total length = 100
100
16 Ohm’s law, I = V Reading of ammeter ∴Required length = × 2 = 67 cm
R 84 2 3
= × = 2.18 A
V2 12.8 6 25 We have, X + Y = 1000 Ω
and power = I 2 R =
R 20 Let I1 and I2 be the currents drawn X Y=1000 – X
When the resistors are connected in from cells of emf 6 V and 4 V in the
series, the effective resistance is more circuits, respectively. Then, G
than that as when they are connected in 6
I1 = =1A 100 – l
parallel. 2+ 3+ 1 l
17 When switch S is opened, then right 4 X 1000 − X
and I2 = = 0.4 A Initially, = …(i)
side resistance R which was short 6+ 4 l 100 − l
circuited earlier contributes to V A − VB = 1 × 3 = 3 V;
equivalent resistance. Hence, equivalent When X and Y are interchanged, then
V B − VC = 5V
resistance across the battery increases, Y=1000 – X X
and VC − V D = 0.4 × 6 = 2.4 V
power dissipated by left resistance R
decreases, voltmeter reading decreases ∴ V A − VD = (V A − V B ) + ( V B − VC )
and ammeter reading decreases. + (VC − V D ) G
= 3 + 5 + 2.4 = 10.4 V
18 Effective emf of circuit = 10 − 3 = 7 V
Total resistance of circuit 21 Connect point Q to ground and apply (l – 10) (110 – l)
= 2 + 5+ 3 + 4 KCL. Consider the grounded circuit as 1000 − X X
shown below. =
= 14 Ω l − 10 100 − (l − 10)
Current, I = 7 / 14 = 0.5 A 6V
P 1000 − X X
Potential difference between or = …(ii)
A and D = 0.5 × 10 = 5 V l − 10 110 − l
Potential at D = 10 − 5 = 5V 1Ω 9V
From Eqs. (i) and (ii), we get
Potential at E = 5 − 3 = 2 V 100 − l l − 10
Q =
Hence, E cannot be at zero potential, as
3Ω 5Ω l 110 − l
there is potential drop at E.
(100 − l) (110 − l) = (l − 10) l
19 The equivalent circuit of the given
circuit will be reduced to as shown in 11000 − 100 l − 110 l + l2 = l2 − 10 l
Applying KCL at point Q, we can write
figure. ⇒ 11000 = 200 l
Incoming current at Q = outgoing
2W
current from Q ∴ l = 55cm
Substituting the value of l in Eq. (i), we
6W 6W V +6 V 9−V
⇒ + = get
3 1 5 X 1000 − X
A =
V  + + 1 = − 2 55 100 − 55
1 1 9
3W 3W or
 3 5  5
⇒ 20 X = 11000
5 + 3 + 15 9 − 10
14V, 0.8 W or V  = ∴ X = 550 Ω
 15  5
232 40 DAYS ~ JEE MAIN PHYSICS DAY NINETEEN

l − 2  l − 3 8
26 As,  Q Potential drop across capacitor 5 10 W
 4 = 
 2   3  = Potential drop across r2
⇒ l =6 Er2 + 12V
= Ir2 =
l − 2 r + r2 –
Therefore, r =   4 = 8Ω 1W
 2 
∴ Stored charge of capacitor,

27 Potential gradient along the wire, Q = CV + 2W
6 r2 13V
K = V/cm = CE
300 r + r2 For parallel combination of cells,
Potential difference across 50 cm length E1 E
+ 2
is 3 According to the question, Eeq =
r1 r2
6 0.1m 1 1
V = k × 50 = × 50 = 1 V +
300 r1 r2
12 13
28 Potential gradient of a potentiometer, +
∴ Eeq = 1 2 = 37 V
. × 4 × 10−7
Iρ 02 1 1
K = = = 01
. V/m + 3
A 8 × 10−7 5V 1 2
v d = 2. 5 × 10−4 m/s Potential drop across 10 Ω resistance,
30 When the battery is undergoing
charging processes, then  E  37 / 3
⇒ n = 8 × 1028 / m3 V =  × 10 = × 10
V = E + Ir > E R   10 + 2 
 total   
So, Statement I is correct. We know that,  3
J = nev d = 11.56 V
Statement II is also correct but not
or I = ne v d A ∴ V = 11.56 V
explaining Statement I.
where, symbols have their usual
31 With increase in temperature, the value 6 With only the cell,
meaning.
of unknown resistance will increase. E¢
V
In balanced Wheatstone bridge ⇒ = nev d A
condition, R
52 cm
R l VA
= 1 or = nev d A
X l2 ρL
Here, G
V
or = nev d E, r
R = value of standard resistance, ρL
X = value of unknown resistance. On balancing,
V E = 52 × x
l1 or ρ= …(i)
To take null point at same point or to nev d L where, x is the potential gradient of the
l2
5 wire.
remain unchanged,
R
should also = When the cell is shunted,
X 8 × 1028 × 1. 6 × 10−19 × 2. 5

remain unchanged. × 10−4 × 0.1
Therefore, if X is increasing R, should
also increase. ρ = 1.6 × 10−5 Ω m 40 cm

32 If the diameter of wire AB is increased, 4 We have, equivalent resistance in series


its resistance will decrease. Hence, the R1 + R2 = 10 + 10 = 20 Ω G
potential difference between A and B E, r
A
due to cell C1 will decrease. Therefore,
the null point will be obtained at a
10W 60° 10W R=5 Ω
higher value of x.
Similarly, on balancing,
B C Er
SESSION 2 10W 10W V =E− = 40 × x …(ii)
10W (R + r )
1 R = ρl = ρl (V = volume)
2
D E Solving Eqs. (i) and (ii), we get
A V
E 1 52
∆R ∆l 1 1 1 3 = =
∴ =2 = + 0.2% and in parallel = = + = V r 40
R l RP 20 10 20 1−
R+r
20
2 In steady state, no current flows ⇒ RP = E R+ r 52
through the capacitor. So, resistance r1 3 ⇒ = =
20 V R 40
becomes ineffective. Therefore, Req = + 10 + 10 5+ r 52
So, the current in circuit, 3 ⇒ =
20 + 30 + 30 5 40
E =
I = 3 ⇒
3
r = Ω ⇒ r = 1.5 Ω
r + r2 (Total Resistance) 2
80
= = 26.66 = 26.7 Ω
3
DAY NINETEEN CURRENT ELECTRICITY 233

7 Let R 0 be the initial resistance of both Now, when R3 = 107 Ω As potential difference across R2 (= 6 Ω )
conductors and R2 = 30 Ω is 48 V, hence
d R
∴ At temperature θ their resistances Then, if the deflection is , so V AB = 48 ×
will be, 2 R2
R1 = R 0(1 + α1θ) equivalent resistance should be 48 × 20
= = 160 V
and R2 = R 0(1 + α2θ) 30
= 15 Ω
6
For series combination, 2 12 R1 = tan θ = R 0(1 + αT1 )
R s = R1 + R2 It is only when equivalent resistance and R 2 = cot θ = R 0(1 + αT2 )
R s 0(1 + α sθ) = R 0(1 + α1θ) + R 0(1 + α2 θ) and R3 and R g will be parallel to R2 cot θ − tan θ
where, R s 0 = R 0 + R 0 = 2R 0 giving resistance 15 Ω. = R 0(1 + αT2 ) − R 0(1 + αT1 )
∴ 2R 0(1 + α sθ) = 2R 0 + R 0 θ (α1 + α2 ) Let R3 − R g = equivalent = R 0α(T2 − T1 )
α + α2 = 30 Ω = R 1
or αs = 1 or T2 − T1 = (cot θ − tan θ)
2 1 1 1 1 1 α R0
∴ + = + =
For parallel combination, R2 R 30 30 15
1  cos θ sin θ 
R p0(1 + α pθ) ∴ Req = 15 Ω =  − 
R 0(1 + α1θ)R 0(1 + α2θ) α R 0  sin θ cos θ 
= Thus, R g must will be 77 Ω in order to
R 0(1 + α1θ) + R 0(1 + α2θ) 2cos 2θ
maintain =
R 0R 0 R R3 − R g = 30 α R 0 sin 2θ
where, R p0 = = 0
R0 + R0 2 ⇒ 107 − R g = 30 =
2
cot 2θ
R0 = R g = 77 Ω α R0
∴ (1 + α pθ)
2 10 Let R = at + b T2 − T1 ∝ cot 2θ
R2 (1 + α1θ + α2θ + α1α2θ2 )
= 0 At t = 10 s, R = 20 Ω 13 Q = at − bt 2
R 0(2 + α1θ + α2θ)
∴ 20 = 10 a + b …(i)
R
as α1 and α2 are small quantities. At t = 30 a + b …(ii) A B
C
∴ α1 , α2 is negligible. Solving Eqs. (i) and (ii), we get (I – I1)
So, neglect α1 , α2 , θ2 a = 1.0 Ω / s
α1 + α2 and b = 10 Ω 2R
or α p = R
2 + (α1 + α2 )θ ∴ R = (t + 10)
E 10 I I1
α1 + α2   α1 + α2  θ I = =
= 1 −    R t + 10 F D
2  2   E
30
[Binomial expansion] ∆q = ∫10 Idt ∴
dQ
I = = a − 2bt
as (α1 + α2 )2 is negligible 30  10  dt
∴ αp = 1
α + α2 = ∫10  t  dt
+ 10  when, t = t 0, I = 0,
2 i .e ., a − 2bt = 0 …(i)
= 10 log e (2)
In loop BCDEB,
8 Consider a concentric spherical shell of
radius x and thickness dx as shown in
11 Effective value of resistance of parallel I1 (2R ) − (I − I1 )R = 0 or 3I1 = I
combination of 20 Ω,30 Ω, 60 Ω is R1 , i a − 2bt
figure. Its resistance, dR is I1 = =
where 3 3
t0

dx
1
=
1
+
1
+
1 H = ∫0 (I12 (2h )
R1 20 30 60
2R t 0
9 ∫0
3+ 2+ 1 6 1 = (a − 2bt )2 dt
x = = =
a
60 60 10 2R  t 0 2
(a − 4b 2t 2 − 4dt )dt 
9  ∫0
R1 = 10 Ω =

Similarly, effective value of parallel
2R   2 4b 2t 2 4bat 2  
t0
b combination of 24 Ω and 8 Ω resistance =  a t + −  
is given by 9  3 2 0 
 
ρ dx Q R = ρl  24 × 8
dR =   2R  2
 A
R2 = = 6Ω 4b 2t 30 2
4 πx2 24 + 8 = a t 0 + − 2bat 0 
9  3 
∴Total resistance,
ρ b dx 48 V a
4 π ∫ a x2
R= t0 = [from Eq. (i)]
2b
ρ 1 1 3Ω 10 Ω 6Ω 1Ω 2R  2 a 4b 2 a3 a2 
= − H = a × + − 2ab 2 
4 π  a b 
3
A B 9  2b 3 8b 4b 
Hence, the circuit may be redrawn as 2R  a3 a3 a3 
9 As at initial condition the deflection is d =  + − 
shown in the adjacent figure, where 9  2b 6b 2b 
while R3 = 0,
total resistance across A and B,
then equivalent resistance of R2 and R3
R = 3 + 10 + 6 + 1 = 20 Ω. a3 R
= R2 + R3 = R2 = 30 Ω =
27b
234 40 DAYS ~ JEE MAIN PHYSICS DAY TWENTY

DAY TWENTY

Unit Test 4
(Electrostatics and
Current Electricity)
1 Two concentric spheres of radii r1 and r2 carry charges 4 In the given circuit, it is observed that the current I is
q1 and q 2 , respectively. If the surface charge density ( σ ) independent of the value of resistance R 6. Then, the
is the same for both the spheres, the electric potential at resistance value must satisfy
the common centre will be R6
σ r σ r2
(a) × 1 (b) ×
ε0 r 2 ε0 r1 I R1 R3
σ σ
(c) (r1 − r2 ) (d) (r1 + r2 ) R5
ε0 ε0

R2 R4
2 Point charge q moves from
point P to point S along the E 1 1 1 1
path PQRS (figure shown) in a (a) + = ×
P R 5 R 6 R1 + R 2 R 3 + R 4
uniform electric field E pointing
Q
X (b) R1R 4 = R 2R 3
coparallel to the positive S
(c) R1R 2R 5 = R 3R 4R 6
direction of the X-axis. The (d) R1R 3 = R 2R 4 = R 5R 6
R
coordinates of the points
P , Q , R and S (a,b,0) ( 2a,0, 0) (a,−b,0) and ( 0, 0, 0)
5 Two cells of internal resistance r1 and r2 ; and at same emf
are connected in series, across a resistor of resistance R.
respectively.The work done by the field in the above
If the terminal potential difference across the cells of
process is given by the expression
internal resistance r1 is zero, then the value of R is
(a) qEa (b) −qEa
(a) R = 2 (r1 + r2 ) (b) R = r2 − r1
(c) qEa 2 (d) qE [(2a)2 + b 2 ]
(c) R = r1 − r2 (d) R = 2 (r1 – r2 )
3 The variation of potential with distance R from a fixed 6 The electric dipole moment of an electron and proton
point is as shown below.The electric field at R = 5 m is 4.30 nm apart is
(a) 6.88 × 10−28 C-m (b) 5.88 × 10−28 C-m
Potential in volts

5
4 (c) 6.88 × 1028 C-m (d) 5.88 × 1028 C-m
3
2 7 At what distance along the central axis of a uniformly
1 charged plastic disk of radius R is the magnitude of the
0 electric field equal to one-half the magnitude of the field
1 2 3 4 5 6
Distance R in metre at the centre of the surface of the disk?
R R
(a) 2 .5 V/m (b) −2 .5 V/m (a) (b) (c) 2 R (d) 3 R
2 3
(c) 2 / 5 V/m (d) −2 / 5 V/m
DAY TWENTY UNIT TEST 4 (ELECTROSTATICS AND CURRENT ELECTRICITY) 235

8 Work done in placing a charge of 8 × 10−18 C on a 16 In figure, the net potential at point P due to the four point
condenser of capacity 100 µF is charges, if V = 0 at infinity is
(a) 16 × 10−32 J (b) 3.1 × 10−28 J +5q –5q
(c) 64 × 10−32 J (d) 32 × 10−32 J
d
9 A drop, having a mass of 4.8 × 10 −10 g and a charge of –5q
−18 d
2.4 × 10 C is suspended between two charged d
horizontal plates at a distance 1.0 cm apart. Find the P
potential difference between the plates. If polarity of the d
plates be changed, then calculate the instantaneous
acceleration of the drop. +5q

(a) 1.96 × 106 V, 18.6 ms –2 (b) 1.86 × 104 V, 18.6 ms – 2 3q 5q 7q 9q


(a) (b) (c) (d)
(c) 1.96 × 104 V, 19.6 ms –2 (d) 2.96 × 104 V, 17.6 ms –2 8 πε0 8 π ε0 8 πε0 8 πε0
10 What is the direction of the electric field at the centre of 17 V -I graphs for parallel and series combination of two
the square of figure, if q = 1.0 × 10 −8 C and a = 5.0 cm? identical resistors are as shown in figure. Which graph
Y X represent parallel combination?
+q –2.0q V B
a

a a A

–q a
+2.0q I
(a) Graph-A (b) Graph-B
(a) 30° with X-axis (b) 45° with X-axis
(c) Both graph A and B (d) None of these
(c) 60° with X-axis (d) 90° with X-axis
18 The equivalent resistance between points A and B in the
11 In a potentiometer experiment, the balancing with a cell is following diagram is
at length 240 cm. On shunting the cell with a resistance of
2W 2W 2W 2W 2W 2W
2 Ω, the balancing length becomes 120 cm. The internal A
resistance of the cell is
6W 1 6W 2 4W
(a) 4 Ω (b) 2 Ω (c) 1 Ω (d) 0.5 Ω
12 The resistance of a wire at 20°C is 20 Ω and at 500°C B
2W 2W 2W 2W 2W 2W
is 60 Ω. At which temperature its resistance will be
25Ω? (a) 2 Ω (b) 8 Ω (c) 9 Ω (d) 10 Ω
(a) 50°C (b) 60°C 19 Two cells connected in series have electromotive force of
(c) 70°C (d) 80°C 1.5 V each. Their internal resistance are 0.5 Ω and 0.25 Ω
13 A charged cloud system produces an electric field in the respectively. This combination is connected to a
air near earth’s surface. A particle of charge −2 × 10−9 C resistance of 2.25 Ω . Potential difference across the
is acted on by a downward electrostatic force of terminals of each cell
3 × 10−6N, when placed in this field. The ratio of the
magnitude of the electrostatic force to the magnitude of 1.5 V 1.5 V
0.5 W 0.25 W
the gravitational force in the case of proton is
(a) 1.6 × 10−19 (b) 1.5 ×10−10 (c) 1.6 × 1019 (d) 1.4 × 1010 r1 r2
I I
14 An infinite non-conducting sheet has a surface charge
2.25 W
density σ = 0.10 µC m on one side. How far apart are
–2

equipotential surfaces whose potentials differ by 50 V?


(a) 1 V, 0.25 V (b) 1 V, 1.25 V
(a) 5.8 × 10−3 m (b) 6.8 × 10−3 m (c) 1.5 V, 2.25 V (d) 1.5 V, 2.56 V
(c) 7.8 × 10−3 m (d) 8.8 × 10−3 m
20 A charge of 0.8 C is divided into two charges Q1 and Q 2 .
15 An electron is released from rest in a uniform electric
−1 These are kept at a separation of 30 cm. The force on Q
field of magnitude 2.00 × 10 NC 4
. Acceleration of the
is maximum, when
electron is (ignore gravitation)
(a) Q1 = Q2 = 0.4 C (b) Q ≈ 0.8 C,Q2 is negligible
(a) 2.51 × 1015 ms –2 (b) 2.51 × 10−15 ms –2 (c) Q1 is negligible, Q2 ≈ 0.8 C (d) Q1 = 0.2 C,Q2 = 0.6 C
(c) 3.51 × 1015 ms –2 (d) 3.51 × 10−15 ms – 2
236 40 DAYS ~ JEE MAIN PHYSICS DAY TWENTY

21 A particular 12 V car battery can send a total charge of (a) both bulbs work properly
84 A-h through a circuit, from one terminal to other. If this (b) both bulbs will fuse
(c) Only 60 W bulb will fuse
entire charge undergoes a potential difference of 12 V,
(d) Only 90 W bulb will fuse
how much energy is involved?
27 Masses of the three wires of same material are in the ratio
(a) 1.6 × 106 J (b) 2.6 × 106 J
(c) 3.6 × 106 J (d) 4.6 × 106 J of 1 : 2 : 3 and their lengths in the ratio of 3 : 2 : 1.
Electrical resistance of these wires will be in the ratio of
22 AB is uniform resistance wire of length 1 m. A 2 V
(a) 1 : 1 : 1 (b) 1 : 2 : 3 (c) 9 : 4 : 1 (d) 27 : 6 : 1
accumulator, a Daniell cell of 1.08 V and a galvanometer
G are connected as shown. If the sliding contact is 28 Six wires, each of resistance r1 are connected so as to
adjusted for null deflection then the potential gradient in form a tetrahedron. The equivalent resistance of the
AB and the balancing length, measured from end A are combination when current enters through one corner and
respectively. leaves through other corner is
r r
2V
(a) r (b) 2r (c) (d)
3 2
29 A unit negative charge with mass M resides at the
A B
mid-point of the straight line of length 2a adjoining two
fixed charges of magnitude + Q each. If it is given a very
1.08 V
a small displacement x ( x < < a ) in a direction
perpendicular to the straight line, it will
(a) 0.02 V/cm, 54 cm (b) 0.0308 V/cm, 46 cm (a) came back to its original position and stay there
(c) 0.0092 V/cm, 49.6 cm (d) 0.02 V/cm, 50.4 cm 1 Q
(b) execute oscillations with frequency
23 A potentiometer wire of length 200 cm has a resistance 2 π 4 πε0Ma 3
of 20 Ω. It is connected in series with a resistance of10 Ω (c) execute oscillations with frequency
1 Q
and an accumulator of emf 6 V having negligible internal 2 π 4 πε0Ma 2
resistance. A source of 2.4 V is balanced against a 1 Q
(d) execute oscillations with frequency
length L of the potentiometer wire. Find the value of L. 2 π 2 πε0Ma 3
10 W 6V
30 A 28 µF capacitor is charged to 100 V and another 2 µ F
capacitor to 200 V, they are connected in parallel. Then,
L the total final energy is
A B
(a) 01537
. J (b) 0.0155 J (c) 01865
. J (d) 0123
. J
31 A hollow copper tube of 1m length has got external
diameter equal to 10cm and its walls are 5mm thick. Then
2.4 V G
the resistance of tube, if its specific resistance is
(a) 100 cm (b) 120 cm (c) 110 cm (d) 140 cm . × 10−8 Ω m, is
17
24 A charged ball A hangs from a silk thread, (a) 1139
. × 10−5 Ω (b) 1327
. × 10−6 Ω
which makes an angle φ with a large (c) 1150
. × 10−5 Ω (d) 1125
. × 10−4 Ω
B
charged conducting sheet B as shown in φ
32 Manjeet’s room heater is marked as 1000 W-200V. If the
the figure. The surface charge density of voltage drops to 160 V, the percentage change in the
the sheet is proportional to A power of the heater is
(a) sin φ (b) cot φ
(a) 40% (b) 42% (c) 36% (d) 50%
(c) cos φ (d) tan φ
Direction (Q. Nos. 33-40) Each of these questions contains
25 Two concentric conducting thin shells of radius r and 2r two statements : Statement I and Statement II. Each of these
carry charges +2q and +6q, respectively. The magnitude questions also has four alternative choices, only one of which
of electric field at a distance x outside and inside from is the correct answer. You have to select one of the codes (a),
the surface of outer sphere is same, then the volume of x (b), (c), (d) given below
is (a) Statement I is true, Statement II is true; Statement II is
r 2r r r the correct explanation for Statement I
(a) (b) (c) (d)
2 3 3 6 (b) Statement I is true, Statement II is true; Statement II is
not the correct explanation for Statement I
26 There are two electric bulbs rated 60 W, 120 V and 90 W,
(c) Statement I is true; Statement II is false
120 V. They are connected in parallel with 240 V supply,
(d) Statement I is false; Statement II is true
then
DAY TWENTY UNIT TEST 4 (ELECTROSTATICS AND CURRENT ELECTRICITY) 237

33 Statement I A and B are two conducting spheres of V


same radius. A being solid and B hollow. Both are E = 2V
charged to the same potential. Then,
Charge on A = charge on B. r = 1W
A
Statement II Potentials on both are same. R = 1W
34 Statement I A thin metallic wire is bent into semicircular
shape, then its resistivity decreases. 37 Statement I In the following circuit, emf is 2V and internal
Statement II On bending, the drift of electron in the wire resistance of the cell is1 Ω and R = 1 Ω, then reading of
the voltmeter is 1V.
remains same.
2
Statement II V = E − Ir , where E = 2 V, I = = 1 A and
35 Statement I The circuits containing capacitor be handed 2
cautiously, even when there is no current. R = 1 Ω.
Statement II A dielectric differs from an insulator. 38 Statement I The power delivered to a light bulb is more
36 Statement I In the following circuit, the net resistance just after it is switched ON and the glow of the filament is
increasing, as compared to when the bulb is glowing
between points A and B is R
steadily, i.e. after sometime of switching ON.
Statement II As temperature increases, resistance of
R conductor increases.
A B 39 Statement I When a wire is stretched, so that its diameter
R R R is halved, its resistance becomes 16 times.
Statement II Resistance of wire decreases with increase
in length.
R
40 Statement I A potentiometer is preferred over that of a
Statement II All the resistances are in parallel to each voltmeter for measurement of emf of a cell.
other. Statement II Potentiometer is preferred as it does not
draw any current from the cell.

ANSWERS
1. (d) 2. (b) 3. (a) 4. (b) 5. (c) 6. (a) 7. (b) 8. (d) 9. (c) 10. (b)
11. (b) 12. (d) 13. (d) 14. (d) 15. (c) 16. (b) 17. (a) 18. (b) 19. (b) 20. (a)
21. (c) 22. (a) 23. (b) 24. (d) 25. (b) 26. (b) 27. (d) 28. (d) 29. (d) 30. (c)
31. (a) 32. (c) 33. (a) 34. (d) 35. (b) 36. (c) 37. (a) 38. (a) 39. (c) 40. (a)

Hints and Explanations


1 Electric potential of the common 3 Intensity at 5m is same as at any point
Y
centre, is between B and C because the slope of
q1 q2 E BC is same throughout (i.e. electric
V = +
4 πε0 r1 4 πε0 r2 (a,b,0) P field between B and C is uniform).
σ σ
V = × r1 + × r2 b Ö a2 + b2
ε0 ε0 5 A B
a O q
σ X
Potential in volt

= (r1 + r2 ) q q 4
ε0 S b Q
3
√ a2 + b2 √ a2 + b2
Q q1 = 4 πr12 × σ  2
  R
 q2 = 4 πr2 × σ 
2
1
C
O
2 As electric field is a conservative field ∴ W PQRS = W PQS = W PO + WOS 1 2 3 4 5 6
Distance R in metre
Hence, the work done does not depend = Fb cos 90°+ Facos 180°
on path = 0 + q Ea(−1) = − q Ea
238 40 DAYS ~ JEE MAIN PHYSICS DAY TWENTY

Therefore electric field at R = 5m is Energy stored =


1
qV It is upward in the diagram, from the
equal to the slope of line BC hence by 2 centre of the square towards the centre of
− dV 1 the upper side.
E = ; = × 8 × 10−18 × 8 × 10−14
dr 2 l −l
11 Here, r = 1 2 × 2 Ω
(0 − 5) = 32 × 10−32 l2
E =− = 2. 5V/m
6− 4 9 Let m and q be the mass and the charge where, l 1 = 240 cm, l 2 = 120 cm
240 − 120 120
4 From figure, it can be seen that I is of the drop and E the intensity of = × 2= × 2 = 2Ω
electric field between the plates. 120 120
independent of resistance R 5, so no
current flow through it. This require Since, the drop is in equilibrium, the 12 Use R t = R 0 (1 + αt )
that the R1 and R2 junction is at same electric force qE acting on it balances
Here, 20 = R 0 (1 + 20 α )
potential of the junction at R3 and R 4 . its weight mg, i.e.
60 = R 0 (1 + 500 α )
So, according to Wheatstone bridge qE = mg
Here, R t = 25 Ω
condition, If the potential difference between the
R1 R Solving, we find t = 80°C
= 3 ⇒ R1 R 4 = R2 R3 plates is V and the distance between
R2 R4 them is d, then E = V /d . 13 We have, F = qE
∴ q (V /d ) = mg or V = mgd /q F 3 × 10−6
5 Given, V1 = 0 Thus, E = =
Here, m = 4.8 × 10−10 g = 4.8 × 10−13 kg, q 2 × 10−9
V1 + V2 = IR
⇒ V2 = IR = 1.5 × 103 NC –1
g = 9.8 N kg −1 ,
Magnitude of the electrostatic force on a
− E − Ir2 = IR d = 1.0 cm = 1.0 × 10−2 m proton is
But V1 = E − Ir1 ⇒ E = Ir1 [QV1 = 0] Fe = eE = (1.60 × 10−19 )(1.5 × 103 )
and q = 2.4 × 10−18C
⇒ Ir1 − Ir2 = IR
or R = r1 − r2 (4.8 × 10−13 ) × 9.8 × (1.0 × 10−2 ) = 2.4 × 10−16 N
∴ V =
−18
2.4 × 10 Magnitude of the gravitational force on
6 Magnitude of a dipole moment is the proton is
p = qd = 1.96 × 10 V 4
F g = mg = (1.67 × 10−27 )(9.8)
= (1.60 × 10−19 )(430
. × 10−9 )
On changing the polarity of the plates,
= 6.88 × 10−28 C-m = 1 . 63 × 10−26 N
the electric force q E will also be
directed downwards. Then, the The ratio of the force is
7 At a point on the axis of a uniformaly acceleration of the drop is Fe 2.4 × 10−16
charged disk at a distance x above the qE + mg =
centre of the disk, the magnitude of the a= Fg 1. 63 × 10−26
m
electric field is But qE = mg = 1. 4 × 1010
 
E =
σ
1 −
x

∴ a = 2 g = 19.6 ms –2 14 Electric field, E = σ
2ε0 2ε0
 x2 + R2  10 Since, each charge distance from centre x
2a a and electric potential, V = Vs − ∫ E dx
σ E 1 d = = 0
But Ec = such that =
2 ε0 Ec 2 2 2 = Vs − Ex
Net field due to these two charges is Here, two surfaces are separated by ∆x,
x 1
Then, 1 − = 1  2q q  1 q then their potentials difference in
2 Ex =  − =
x + R
2 2
4πε 0  a2 /2 a2 /2  4πε 0 a2 /2 magnitude by
x 1 (9 × 109 )(1.0 × 10−8 )  σ 
or = = ∆V = E∆x =   ∆x
x +R
2 2 2 (0.050) 2  2ε0 
2 2ε0 ∆V
Squaring both side, we get Thus, ∆x =
2
= 7.19 × 104 NC –1 σ
x 1
= 1  2q q  2(8.85 × 10−12 )(50)
x2 + R2 and E y = − =
4 πε0  a2 /2 a2 /2 
4
. × 10−6
010
x2 R2 1 q
or x2 = + = = 7.19 × 104 N / C = 8.8 × 10−3 m
4 4 4 πε0 a2 /2
R2 15 We know that, F = eE
Thus, x =
2
The magnitude of the field is
3 By Newton’s second law,
R E = E 2x + E 2y = 2 (7.19 × 10 ) 4 2
x= F eE
a= =
3 = 1.02 × 105 NC –1 m m
8 Here, q = 8 × 10−18C, C = 100µF = 10−4 F Angle made with the x-axis is (1.60 × 10−19 )(2.00 × 104 )
Ey =
θ = tan −1 . × 10−31
911
q 8 × 10−18
V = = = 8 × 10−14 V Ex
C 10−4 = 3.51 × 1015 ms – 2
= tan −1 (1) = 45°
DAY TWENTY UNIT TEST 4 (ELECTROSTATICS AND CURRENT ELECTRICITY) 239

16 Net potential at point P, The force of Q 1 due to Q 2 , For equilibrium of forces at A,


q  5 5 5 5 5q Q 1Q 2 Q Q × 100 T sin φ = Q E
V = F = kP =K 1 2
4 πε0  − 2d − d + d + d  = 8 πε 30 × 10−2 30
σ
T sin φ = Q ⋅ , [from Eq. (i)]
  0
ε0
17 R Parallel < RSeries . From graph A it is 0.4 × 0.4 × 100 8 Qσ
=K × = K T sin φ = …(ii)
clear that slope of the line A is lower 30 15 ε0
than the slope of the line B. (b) When Q 1 = 0.8 C,Q 2 ≈ 0 and T cos φ = mg …(iii)
Also, slope = resistance, so line A 0.8 × 0
F =k × =0 On dividing Eq. (ii) by Eq. (iii), we get
represents the graph for parallel 30 × 10−2 Qσ
combination. tan φ =
(c) When Q 1 ≈ 01 , Q 2 = 0.8 C ε0 mg
18 The resistances 2 Ω and 2 Ω at the last 0 × 0.8 ε mg
F =k × =0 or σ = 0 tan φ
terminals are outside the circuit and so 30 × 10−2 Q
they may be ignored. Now, in loop 2,
(d) When Q 1 = 02. C, Q 2 = 0.6 C ⇒ σ ∝ tan φ
the resistances (2 Ω + 2 Ω ), 4 Ω and
. × 0.6 100 2
02
(2 Ω + 2 Ω ) are in series. Their F =k × = k 25 Electric field at a distance x outside from
equivalent resistance is 12 Ω, which is in 30 5
Hence, for Q 1 = Q 2 = 0.4C, the force will surface of outer shell
parallel with 6 Ω. The equivalent
resistance is be maximum. = Electric field at a distance x inside from
6 × 12 surface of outer shell
R′ = = 4Ω 21 An ampere is coulomb per second, so 1 2q + 6q 1 2q
6 + 12 84 A - h = 84 × 3600 = 3.0 × 105C i.e. ⋅ = ⋅
4 πε0 (2r + x )2 4 πε0 (2r − x )2
The change in potential energy is 4 1
2W 2W 2W 2W 2W 2W ∆U = q∆V = 3.0 × 105 × 12 = 3.6 × 106 J ⇒ =
A (2r + x )2
(2r − x )2
22 Potential difference per cm 2 1
6W 1 6W 2 4W ⇒ =
2V
== 0.02 V/cm 2r + x 2r − x
B 100 cm ⇒ 4r − 2 x = 2r + x
2W 2W 2W 2W 2W 2W Balancing length =
100
× 1.08 = 54 cm ⇒ 2r = 3 x
2 2r
Similarly, in loop 1, the resistances ⇒ x=
3
(2 Ω + 2 Ω), R ′ (= 4 Ω ) and (2 Ω + 2 Ω ) are 23 The current in the potentiometer wire
AB is v 12 1202
in series and these are in parallel with 26 R1 = = = 240 Ω
6 P1 60
6 Ω. Hence, their equivalent resistance I = = 0. 2 A
20 + 10
is v 22 1202
R2 = = = 160 Ω
R ′′ = 4 Ω The potential difference across the P2 90
potentiometer wire is P 60
Lastly, between the points A and B, the V = current × resistance ∴ I1 = 1 = = 0.5 A
resistances 2 Ω, R ′ ′ = (4 Ω ) and 2 Ω are in v 1 120
= 0.2 × 20 = 4 V
series. Hence, their equivalent P 90
The length of the wire is l = 200 cm. So, I2 = 2 = = 075
. A
resistance is v 2 120
2Ω + 4Ω + 2Ω = 8Ω the potential gradient along the wire is
V 4 When both bulbs are connected in
k = = parallel, then equivalent resistance
19 The arrangement is shown in the figure. l 200
The effective emf in the circuit is R1 R2
= 0.02 Vcm –1 RP =
E = 1.5 + 1.5 = 3.0 V R1 + R2
The emf 2.4 V is balanced against a
and the total resistance is 240 × 160
length L of the potentiometer wire. = = 96 Ω
R = 0.5 + 025. + 225. = 3.0 Ω 2.4 240 + 160
i.e. 2. 4 = kL or L =
Hence, the current in the circuit is k ∴ When they are connected with 240 V
E 3.0 2.4 supply, then
I = = = 1.0 A = = 120 cm
R 3.0 240
0.02 I = = 2.5 A
Potential difference across the terminals 96
24 Electric field at point A Now, current in 60 W bulb,
of the first cell is
σ 160
V1 = E − Ir1 = 1.5 − (1.0) × (0.5) = 1.0 V E= …(i) I1′ = I ⋅
ε0 400
Potential difference across the terminals 160
of the second cell is = 2.5 × = 1A
400
V2 = E − Ir2 = 1.5 −(1.0) ×(0.25) = 1.25 V B φ T cos φ Current in 90 W bulb,
20 Given, T φ 240
I2′ = 2.5 × = 1.5 A
Q 1 ←  → Q 2 F=QE 400
30 cm
T sin φ A
(a) Q 1 = Q 2 = 0.4 C Since, I1′ > I and I2′ > I2
Hence, both bulbs will fuse.
240 40 DAYS ~ JEE MAIN PHYSICS DAY TWENTY

27 Mass, M = Volume × Density 1 2KQ 1 2×1×Q drift velocity of electron does not depend
= =
2 π Ma3 2 π 4 π ε0 Ma3 on area of cross-section, so it remains
M
= Al × d or A= same.
ld 1 Q
= 35 A capacitor does not discharge itself. In
ρl ρl ρl2d 2 π 2 π ε0 Ma3
Resistance, R = = = case the capacitor is connected in a
A M M circuit containing a source of high
  30 Here C1 = 28 µ F = 28 × 10 F −6
 ld  voltage, the capacitor charges itself to a
2 C2 = 3 µ F = 3 × 10−6 F very high potential. So, if a person
l
So, R∝ V1 = 100V and V2 = 200V handles it without discharging, he may
M get a severe shock.Dielectrics and
⇒ Charge of C1 ,
l 21 l2 l2 insulators cannot conduct electricity but
Thus, R1 : R2 : R3 = : 2 : 3 q1 = C1V1 = 28 × 10−6 × 100 in case of a dielectric, when an external
M1 M2 M3
= 28 × 10−4 C field is applied, induced charges appear
32 22 12 on the faces of the dielectric. In other
= : : = 27 : 6 : 1 Charge of C2
1 2 3 words, the dielectric have the property of
q2 = C2V2 = 3 × 10−6 × 200 transmitting electric effects without
28 Six wires each of resistance r from a = 6 × 10−4 C conducting.
tetrahedron as shown in the following
Potential, 36 The equivalent circuit is represented as,
figure.
q + q2 28 × 10−4 + 6 × 10−4 This is balanced Wheatstone bridge
3 V = 1 = hence, resistance in branch MN is not
C1 + C2 28 × 10−6 + 3 × 10−6
taken into consideration. Hence, the
1 r r = 109.68 V equivalent resistance between points A
1 2 1 and B is given by
r Total final energy, U = (C1 + C2 )V 2
2 M
1
3
r r = (28 × 10−6 + 3 × 10−6 )(109.68)2
2
4 R R
2 4 = 01865
. J
r A R B
In Wheatstone circuit, the equivalent 31 External radius, R R
10
resistance of upper circuit r2 = = 5cm = 0.05m
I I 1 2 1 2
= + = =
R 2r 2r 2r r Internal radius r1 = r2 − thickness of tube N
⇒ R=r = 0.05 − 0.005 1 1 1
= +
It will be in parallel with outer = 0.045 m R AB (R + R ) (R + R )
resistance, Area of cross-section = π (r22 − r12 ) 1 2 1
1 1 1 2 or = =
= + = = π [(0.05)2 − (0.045)2 ] R AB 2R R
R eq r r R
= 1.492 × 10−3 m2 ∴ R AB = R
r
⇒ R eq = . Resistance of copper tube, 39 The resistance of a wire is
2
l 17. × 10−8 × 1 l
R =ρ = R =ρ ,ρ being specific resistance
29 From figure the net force, a 1.492 × 10−3 A
Al
+Q = 1139
. × 10−5 Ω or R∝
F A2
32 Resistance of heater, 1
or R∝ (Q A = πr 2 )
q V 2 (200)2 r4
–1C R= = = 40Ω
q P 1000 Hence, when diameter is halved the
Power of heater at V ′ = 160V resistance of the wire is
F
V ′2 (160)2 R∝
1
= 16 R
–Q P′= = = 640 W 4
…(i)
R 40 r
F net = − F cos θ + (− F cos θ)  
Percentage fall in the power of the  2
= − 2F cos θ heater,
kQ (− 1) x P − P′ 1000 − 640 Hence, its resistance will become
= −2 × × 100 = × 100 = 36%
x2 + a2 16 times.
x + a2
2
P 1000
2kQ Again from Eq. (i), we get
= ⋅x l l2
(x + a )
2 2 3 /2
34 Resistivity of metallic wire does not R∝ or R ∝ or R ∝ l 2
A Al
= + 
2 kQ  depend on shape of wire because it is a
Or F net ⋅ x [Q x << a]
material property. On bending, the
 a3  Therefore, on increasing the length
cross-sectional area of wire changes but resistance increases.
Frequency of oscillation,
DAY TWENTY ONE

Magnetic Effect
of Current
Learning & Revision for the Day
u Concept of Magnetic Field u Force on a Moving Charge in u Magnetic Force on a Current Carrying
u Biot-Savart’s Law and its Uniform Magnetic Field Conductor
Applications u Cyclotron u Moving Coil Galvanometer
u Ampere’s Circuital Law

Concept of Magnetic Field


If a magnet is placed in a magnetic field, then it experiences a force on it, Also, when a
magnet is placed near a current carrying conductor, then it experiences the similar force,
it means that current carrying conductor produces a magnetic field around it. This effect
of current is called magnetic effect of current.

Biot-Savart’s Law and its Applications


The magnetic field d B at a point P, due to a current element I dl is given by
µ I (d l × r)
dB = 0
4π r3
where, θ is the angle between d l and r.

I PREP
MIRROR
θ Your Personal Preparation Indicator
dl
u No. of Questions in Exercises (x)—
r u No. of Questions Attempted (y)—
P u No. of Correct Questions (z)—
X (Without referring Explanations)
inside

Direction of magnetic field produced due to a current carrying straight wire can be u Accuracy Level (z / y × 100)—
obtained by the right hand thumb rule. u Prep Level (z / x × 100)—

In order to expect good rank in JEE,


your Accuracy Level should be above
85 & Prep Level should be above 75.
242 40 DAYS ~ JEE MAIN PHYSICS DAY TWENTY ONE

For a wire of infinite length φ1 = φ2 = 90 ° and hence


Magnetic Field due to Circular l

µ I
B= 0
Current Loop 2 πr
l
If there is a circular coil of radius R and N number of turns, l
When the wire XY is of infinite length, but the point P
carrying a current I through the turns, then magnetic field lies near the end X or Y, then φ1 = 0 ° and φ2 = 90 ° and
at the centre of coil is given by µ I
hence, B= 0
µ NI 4πr
B= 0
2R l
When point P lies on axial position of current carrying
l
If there is a circular arc of wire R conductor, then magnetic field at P,
subtending an angle θ at the centre B = 0.
of arc, then the magnetic field at O θ I l
When wire is of infinite length, then magnetic field
the centre point near the end will be half, that of at the perpendicular
R
µ I  θ bisector.
B= 0  
2R  2π 
2. Magnetic Field due to a Thick (Cylindrical) Wire
l
At a point P situated at a distance
r from centre of a current Magnetic field at a point
I R outside the wire
carrying circular coil along its
axial line. µ I
O I P B = 0 , where r is the B
The magnetic field is 2 πr r P
µ 0 NIR2 distance of given point
B = r from centre of wire and I
2(R2 + r 2 )3 /2
r > R. R
µ 0NIR2 Thick cylindrical wire
If r >> R, then at a point along the axial line, B = l
Magnetic field at a point
2r3 inside the wire at a
distance r from centre of wire (r < R) is
µ I r
Ampere’s Circuital Law B= 0 ⋅ 2
2π R
The line integral of the magnetic
i5
l
Magnetic field inside a hollow current carrying
field B around any closed path is
conductor is zero.
equal to µ 0 times the net current I
B
threading through the area
enclosed by the closed path.
dI i2 3. Magnetic Field due to a Solenoid
Mathematically, ∫ B ⋅ dl = µ 0 ΣI i1
i3 A current carrying
i4 solenoid behaves as a bar
Now, consider the diagram above. magnet. The face, where
Here, ∑ I = i 1 + i2 − i3 current is flowing
Hence, ∫ B ⋅ dl = µ 0 ⋅ (i 1 + i2 − i3 ) clockwise behaves as
South pole and the face, (a) (b)
where current is seen
Applications of Ampere’s law flowing anti-clockwise, behaves as North pole.
For such a solenoid, the magnetic field inside it is uniform
1. Magnetic field due to Straight Current and directed axially.
Carrying Wire l
For a solenoid coil of infinite length at a point on its
The magnetic field due to a current
X axial line, the magnetic field, B = µ 0nI
carrying wire of finite length at a I where, n is number of turns per unit length.
point P situated at a normal distance φ2 1
r φ1
l
At the end of solenoid, B = µ 0nI
r is P 2
µ I At the end field is half of at the centre, this is called
B = 0 (sin φ1 + sin φ2 ) l

4πr end effect.


Y
l
If point P lies symmetrically on
the perpendicular bisector of wire XY, then φ1 = φ2 = φ 4. Toroidal Solenoids
(say) and hence For a toroid (i.e. a ring shaped closed solenoid) magnetic
µ I
B = 0 ⋅ 2 sin φ = µ 0I sin φ field at any point within the core of toroid B = µ 0nI ,
4πr 2 πr N
where n = , R = radius of toroid.
2πR
DAY TWENTY ONE MAGNETIC EFFECT OF CURRENT 243

If the direction of a v is parallel or anti-parallel to B, θ = 0


Force on a Moving Charge in
l

or θ = 180 ° and therefore F = 0. Hence, the trajectory of the


Uniform Magnetic Field and particle is a straight line.
Electric Field If the velocity of the charged particle is not perpendicular to
the field, we will break the velocity in parallel ( v|| ) and
l
If a charge q is moving with velocity v enters in a region in perpendicular (v⊥ ) components.
which electric field E and magnetic field B both are mv⊥
present, it experiences force due to both fields r=
qB
simultaneously. The force experienced by the charged
particle is given by the expression Pitch, p = (v||)T
F = q (v × B) + qE
Here, magnetic force Fm = q (v × B) = Bqv sin θ and electric Cyclotron
force Fe = qE.
l
The direction of magnetic force is same as v × B if charge is It is a device used to accelerate positively charged particles,
positive and opposite to v × B, if charge q is negative. e.g. proton, deuteron, α-particle and other heavy ions to
high energy of 100 MeV or more.
Motion of a Charged Particle in a Cyclotron frequency, ν =
Bq
.
2 πm
Uniform Magnetic Field
Maximum energy gained by the charged particle
l
(i) If a charge particle enters a uniform magnetic field B
with a velocity v in a direction perpendicular to that of q 2 B2  2
E max =  r
B (i.e.θ = 90 ° ), then the charged particle experiences a  2m 
force Fm = qvB. Under its influence, the particle
describes a circular path, such that where, r = maximum radius of the circular path followed by
mv the positive ion.
Radius of circular path, r =
qB Maximum energy obtained by the particle is in the form of
In general, kinetic energy.
mv p 2mK
r= = =
qB qB qB Magnetic Force on a Current
=
2 mqV
=
1 2 mV Carrying Conductor
qB B q If a current carrying conductor is placed in a magnetic field B,
where, p = mv = momentum of charged then a small current element I dl experiences a force given by
particle, K = kinetic energy of charged particle dFm = Idl × B
and V = accelerating potential difference. and the total force experienced by whole current carrying
2πm conductor will be
(ii) The period of revolution of charged particle T = ,
qB Fm = ∫ dFm = ∫ I(dl × B)
qB
the frequency of revolution ν = The direction of force can also be determined by applying
2 πm Fleming’s left hand rule or right hand thumb rule.
qB
or angular frequency ω = ⋅ Force between Two Parallel Current
m
Carrying Conductors
l
(i) If a charged particle is moving at an angle θ, to the 1 2 1 2
magnetic field (where θ, is other than 0°, 90° or 180°), it
l
Two parallel current
describes a helical path, where carrying conductors
radius of helical path, exert magnetic force on I2 I2
one another. I1 I1
mv sin θ
r= . l
Magnetic force
qB
2πm experienced by length l
(ii) Revolution period, T = F12 F21 F12 F21
of any one conductor
qB
due to the other
qB r
or Frequency, ν = current carrying r
2 πm conductor is (a) (b)
(iii) Moreover, pitch (the linear distance travelled during µ 0 2I1 I2
F = ⋅ l
one complete revolution) of helical path is given by 4π r
2 πmv cos θ F µ 2I I
p = v cos θ ⋅ T = Force per unit length, = F0 = 0 ⋅ 1 2
qB I 4π r
244 40 DAYS ~ JEE MAIN PHYSICS DAY TWENTY ONE

NOTE — If the conductors carries current in same direction, then pivoted type moving coil galvanometer G. S is known as
force between them will be attractive. shunt.
— If the conductor carries current in opposite direction, then
force will be repulsive. G
I Ig I

Torque I – Ig
When a current carrying loop placed in a uniform magnetic
field, it experience torque, S
τ = NIAB sin θ
Ammeter
where, NiA is defined as the magnitude of the dipole moment
of the coil Then, from circuit,
(pm ) ⋅ τ = pm B sin θ I g × G = (I − I g ) × S
⇒ τ = pm × B  Ig 
⇒ S = G
I − I g 
NOTE • A current carrying loop (of any shape) behaves as a
magnetic dipole whose magnetic moment is given by So, S << G, only a small fraction of current goes through the
( pm ) = IA galvanometer.
• If we have a current carrying coil having N turns, then l
Conversion of Galvanometer into Voltmeter
magnetic moment Pm of dipole will be A voltmeter is made by connecting a resistor of high
( pm ) = NIA resistance R in series with a pivoted type moving coil
• Magnetic moment of a current carrying coil is a vector and galvanometer G.
its direction is given by right hand thumb rule. V
a b

Moving Coil Ig
Galvanometer (MCG)
MCG is used to measure the current upto nanoampere. The G R
deflecting torque of MCG,
Voltmeter
τdef = NBIA
A restoring torque is set up in the suspension fibre. If α is the V V
From the circuit, Ig = ⇒R = −G
angle of trust, the restoring torque is G+ R Ig
τ restoring = KI l
Current Sensitivity The current sensitivity of a
where, K is galvanometer constant. galvanometer is defined as the deflection produced in the
galvanometer per unit current flowing through it.
α NBA
Some Important Concepts Related to Sl = =
I C
Moving Coil Galvanometer l
Voltage Sensitivity Voltage sensitivity of a galvanometer is
Some of the important concepts related to galvanometer, i.e. defined as the deflection produced in the galvanometer per
current sensitivity, voltage sensitivity and some of
unit voltage applied to it.
conversions used in galvanometer are given below.
α α S I NBA
l
Conversion of Galvanometer into Ammeter An ammeter is SV = = = =
V IR R RC
made by connecting a low resistance S in parallel with a
DAY TWENTY ONE MAGNETIC EFFECT OF CURRENT 245

DAY PRACTICE SESSION 1

FOUNDATION QUESTIONS EXERCISE


1 A loosely wound helix made of stiff wire is mounted 5 The magnitude of the magnetic field (B) due to loop
vertically with the lower end just touching a dish of ABCD at the origin (O) is ª AIEEE 2009
mercury. When a current from a battery is started in the B
coil through the mercury
a A
(a) the wire oscillates
(b) the wire continues making contact I1
30° I
(c) the wire breaks contact just as current is passed O
(d) the mercury will expand by heating due to passes of
D
current b
2 A current I flows through a closed loop as shown in C
figure. µ 0 I (b − a)
(a) zero (b)
Q I 24ab
µ 0I b − a  µ 0I  π
(c) (d) 2 (b − a) + (a + b)
4 π  ab  4 π  3 
O
I 6 A current I flows in an infinity long wire with cross-section
R 2q in the form of a semicircular ring of radius R. The
I magnitude of the magnetic induction along its axis is
P S ª AIEEE 2012
µ I µ I µ I µ0I
The magnetic field induction at the centre O is (a) 02 (b) 0 (c) 0 (d)
µ I µ I 2π R 2 πR 4 πR π2 R
(a) 0 θ (b) 0 (θ + sin θ)
4 πR 4 πR 7 Two coaxial solenoids of different radii carry current I in
µ I µ I the same direction. Let F1 be the magnetic force on the
(c) 0 (π − θ + sin θ) (d) 0 (π − θ + tan θ)
4 πR 2 πR inner solenoid due to the outer one and F2 be the
magnetic force on the outer solenoid due to the inner
3 The magnetic induction at the centre O in the figure as one. Then, ª JEE Main 2015
shown is
(a) F1 = F2 = 0
R2 (b) F1 is radially inwards and F2 is radially outwards
(c) F1 is radially inwards and F2 = 0
R1
(d) F1 is radially outwards and F2 = 0

I O 8 An infinitely long conductor PQR is bent to form a right


angle as shown. A current I flows through PQR. The
µ 0I  1 1 µ 0I  1 1
(a)  −  (b)  +  magnetic field due to this current at the point M is H1. Now
4  1
R R 2  4  1
R R 2  another infinitely long straight conductor QS is connected
µ 0I µ 0I
(c) (R1 − R 2 ) (d) (R1 + R 2 ) at Q, so that the current is I/2 in OR as well as in QS, the
4 4 current in PQ remaining unchanged. The magnetic field at
4 An electron moves in a circular orbit with a uniform M is now H2 . The ratio H1 : H2 is given by
speed v. It produces a magnetic field B at the centre of M
the circle. The radius of the circle is proportional to
v

I 90°
e –¥ +¥
O P Q 90° S

r
R
B v v B –¥
(a) (b) (c) (d) 1 2
v R B v (a) (b) 1 (c) (d) 2
2 3
246 40 DAYS ~ JEE MAIN PHYSICS DAY TWENTY ONE

9 For a positively charged particle moving in a xy-plane 16 A thin flexible wire of length L is connected to two
initially along the x-axis,there is a sudden change in its adjacent fixed points and carries a current I in the
path due to the presence of electric and/or magnetic clockwise direction, as shown in the figure. When the
fields beyond P. The curved path is shown in the system is put in a uniform magnetic field of strength B
xy-plane and is found to be non-circular. Which one of going into the plane of the paper, the wire takes the
the following combinations is possible? shape of a circle. The tension in the wire is ª AIEEE 2011
y × × × × ×× × ×
× × × × × × ×
× × × × ×× × ×
× × × × ×× × ×
× × × × × × ×
P × × × × ×× × ×
x
IBL IBL IBL
(a) IBL (b) (c) (d)
π 2π 4π
17 Two parallel long wires A and B carry currents I1 and
(a) E = 0, B = b $i + ck$ (b) E = a $i ; B = ck$ + a $i
I 2 ( < I1 ). When I1 and I 2 are in the same direction, the
(c) E = 0; B = c$j + bk$ (d) E = a $i ; B = ck$ + b$j
magnetic field at a point mid-way between the wires is
10 A magnetic field 4 × 10−3 kT exerts a force 10 µT. If I 2 is reversed, the field becomes 30 µT. The ratio
( 4 $i + 3 $j ) × 10−10 N on a particle having a charge10−9 C I1 / I 2 is
and going on the xy-plane. The velocity of the particle is (a) 1 (b) 3 (c) 2 (d) 4
(a) −75 $i + 100 $j (b) −100 $i + 75 $j 18 A loop carrying current I lies in the xy- plane as shown in
(c) 25 $i + 2 $j (d) 2 $i + 25 $i the figure. The unit vector k$ is coming out of the plane of
11 An electron, a proton and an alpha particle having the the paper. The magnetic moment of the current loop is
same kinetic energy are moving in circular orbits of radii ª JEE Main (Online) 2013
y
re , rp , rα respectively, in a uniform magnetic field B. The
relation between re , rp , rα is ª JEE Main 2018
(a) re > rp = rα (b) re < rp = rα (c) re < rp < rα (d) re < rα < rp
I
12 The cyclotron frequency of an electron gyrating in a a x
magnetic field of 1 T is approximately
a
(a) 28 MHz (b) 280 MHz (c) 2.8 GHz (d) 28 GHz
13 A proton and an α-particle enters a uniform magnetic π
field perpendicularly with the same speed. If proton (a) a 2 I k$ (b)  + 1 a 2 I k$
2 
takes 25 µs to make 5 revolutions, then the periodic time π
(c) −  + 1 a 2 I k$ (d) (2 π + 1) a 2 I k$
for the α-particle would be 2 
(a) 50 µs (b) 25 µs (c) 10 µs (d) 5 µs
19 Magnetic field at the centre of a circular loop of area A is B.
14 Two long conductors separated by a distance d carry The magnetic moment of the loop will be
current I1 and I 2 in the same direction. They exert a force BA 2 BA 3 / 2 BA 3 / 2 2 BA 3 / 2
(a) (b) (c) (d)
F on each other. Now the current in one of them is µ0 π µ0 π µ 0 π1 / 2 µ 0 π1 / 2
increased to two times and its direction is reversed. The
distance is also increased to 3d. The new value of the 20 A rectangular loop of sides 10 cm and 5 cm carrying a
force between them is current I of 12 A is placed in different orientations as
F 2F F shown in the figures below. ª JEE Main 2015
(a) − 2 F (b) (c) (d) − z
3 3 3 z
I
15 Two very thin metallic wires placed Y B
C I I B
along X and Y -axes carry equal B (i) y (ii)
I I I
currents as shown in figure. AB and I y
O x I
CD are lines at 45° with the axes with X z x
I I
origin of axes at O. The magnetic field I B z
will be zero on the line A D I
I B
(a) AB (iii) y (iv)
I I
(b) CD I y
x
(c) segment OB only of line AB I
x I
(d) segment OC only of line CD
DAY TWENTY ONE MAGNETIC EFFECT OF CURRENT 247

If there is a uniform magnetic field of 0.3T in the positive Statement II The particle is performing uniform circular
z-direction, then in which orientations the loop would be mv 2
motion and net force acting on it is .
in 1. stable equilibrium and 2. unstable equilibrium? r
(a) (i) and (ii) respectively (b) (i) and (iii) respectively 24 Statement I A uniformly moving charged particle in a
(c) (ii) and (iv) respectively (d) (ii) and (iii) respectively magnetic field, may follow a path along magnetic field
21 When a current of 5 mA is passed through a lines.
galvanometer having a coil of resistance 15 Ω, it shows Statement II The direction of magnetic force
full scale deflection. The value of the resistance to be put experienced by a charged particle is perpendicular to
in series with the galvanometer to convert it into a its velocity and B.
voltmeter of range 0-10 V is ª JEE Main 2017 (Offline)
25 Statement I The magnetic force experienced by a
(a) 2.045 × 103 Ω (b) 2.535 × 103 Ω moving charged particle in a magnetic field is invariant in
(c) 4.005 × 103 Ω (d) 1985
. × 103 Ω
nature just like any other force.
22 A galvanometer having a coil resistance of 100 Ω gives a Statement II Magnetic force experienced by a charged
full scale deflection when a current of 1 mA is passed particle is given by F = q( v × B ), where v is the velocity of
through it. The value of the resistance which can convert charge particle w.r.t. frame of reference in which we are
this galvanometer into ammeter giving a full scale taking F.
deflection for a current of 10 A, is ª JEE Main 2016 (Offline)
26 Statement I Cyclotron is a device which is used to
(a) 0.01 Ω (b) 2 Ω (c) 0.1 Ω (d) 3 Ω accelerate the positive ion.
Direction (Q. Nos. 23-28) Each of these questions contains Statement II Cyclotron frequency depends upon the
two statements : Statement I and Statement II. Each of these velocity.
questions also has four alternative choice, only one of which 27 Statement I Magnetic field due to a infinite straight
is the correct answer. you have to select one of the codes (a),
conductor varies inversely as the distance from it.
(b), (c), (d) given below
(a) Statement I is true, Statement II is true; Statement II is Statement II The lines of electric force due to a straight
the correct explanation for Statement I current carrying conductor are concentric circles.
(b) Statement I is true, Statement II is true; Statement II is 28 Statement I If a proton and α-particle enter a uniform
not the correct explanation magnetic field perpendicularly with the same speed, the
(c) Statement I is true; Statement II is false time period of revolution of α-particle will be double than
(d) Statement I is false; Statement II is true that of proton.
23 Statement I If a charged particle is projected in a region Statement II Time period of charged particle is given by
where B is perpendicular to velocity of projection, then 2πm
T = .
the net force acting on the particle is independent of its Bq
mass.

DAY PRACTICE SESSION 2

PROGRESSIVE QUESTIONS EXERCISE


1 A cell is connected between two points of a uniformly µ 0 NI 2 µ 0 NI
(a) (b)
thick circular conductor. I1 and I 2 are the currents flowing b a
µ 0 NI b µ 0I N b
in two parts of the circular conductor of radius a . The (c) loge (d) loge
2 (b − a) a 2 (b − a) a
magnetic field at the centre of the loop will be
µ0 3 A particle of mass m and charge q moves with a constant
(a) zero (b) (I1 − I 2 )
4π velocity v along the positive x-direction. It enters a region
µ µ containing a uniform magnetic field B directed along the
(c) 0 (I1 + I 2 ) (d) 0 (I1 + I 2 )
2a a negative z-direction, extending from x = a to x = b. The
2 A coil having N turns is wound tightly in the form of a minimum value of v required, so that the particle can just
spiral with inner and outer radii a and b respectively. enter the region x > b is
When a current I passes through the coil, the magnetic (a) qbB /m (b) q (b − a)B /m
field at the centre is (c) qaB /m (d) q (b + a)B /2m
248 40 DAYS ~ JEE MAIN PHYSICS DAY TWENTY ONE

4 The magnetic field normal to the plane of a wire of n turns


and radius r which carries a current I is measured on the
(c) |B | (d) |B |
axis of the coil at a small distance is from the centre of
the coil. This is smaller than the magnetic field at the
centre by the fraction R/2 R r
R/2 R r
(a) (2 / 3)r 2 /h 2 (b) (3 / 2)r 2 /h 2
(c) (2 / 3)h 2 /r 2 (d) (3 / 2)h 2 /r 2 9 Electrons emitted with negligible speed from an electron
5 A coil having N turns is wound tightly in the form of a gun are accelerated through a potential differenceV0
spiral with inner and outer radii a and b respectively. along the x-axis. These electrons emerge from a narrow
When a current I passes through the coil, the magnetic hole into a uniform magnetic field of strength B directed
field at the centre is ª AIEEE 2012 along x-axis.
µ 0 NI 2µ 0 NI Some electrons emerging at slightly divergent angles as
(a) (b)
b a shown. These paraxial electrons are refocussed on the
µ NI b µ0I b x-axis at a distance.
(c) 0 In (d) In
2 (b − a) a 2 (b − a) a

6 Two long current carrying thin wires, both with current I, x-axis
are held by insulating threads of length L and are in B
equilibrium as shown in the figure, with threads making
an angle θ with the vertical. If wires have mass λ per unit
V0
length, then the value of I is
(g = gravitational acceleration)
8 π 2 mV0 8 π 2 mV0 2 π 2 mV0 4 π 2 mV0
(a) (b) (c) (d)
3 eB eB 2 eB 2 eB 2

L 10 A conductor lies along the z-axis at −1. 5 ≤ z < 1.5 m and


q carries a fixed current of 10.0 A in −a z direction (see
figure). For a field B = 3.0 × 10−4 e −0. 2 x a y T, find the power
I I required to move the conductor at constant speed to
ª JEE Main 2015
x = 2.0 m, y = 0 in 5 × 10−3 s. Assume parallel motion
πλgL πλgL
(a) sinθ (b) 2 sinθ along the x-axis. ª JEE Main 2014
µ 0 cos θ µ 0 cos θ
z
πgL πλgL 1.5
(c) 2 tan θ (d) tan θ
µ0 µ0
l
7 Two identical wires A and B, each of length l, carry the B y
2.0
same current I. Wire A is bent into a circle of radius R x –1.5
and wire B is bent to form a square of side a. If BA and BB
are the values of magnetic field at the centres of the
B (a) 1.57 W (b) 2.97 W (c) 14.85 W (d) 29.7 W
circle and square respectively, then the ratio A is
BB 11 A long straight wire carries a current I. A particle of
ª JEE Main 2016 (Offline) charge + q and mass m is projected with a speed v from
π 2
π 2
π2
π 2 a distance x 0 as shown. The minimum separation
(a) (b) (c) (d) between the wire and particle is
8 16 2 16 8 2
8 An infinitely long hollow conducting cylinder with inner
radius R/2 and outer radius R carries a uniform current
x0
density along its length. The magnitude of the magnetic I
I X
field, | B | as a function of the radial distance r from the
axis is best represented by ª JEE Main (Online) 2013

(a) |B | (b) |B | Y
− 2 πmv − mv 2 πx 0 − 2 πmv
µ 0 qI µ 0 qI qI
(a) x 0 e (b) x 0 e (c) x 0 e (d) zero
R/2 R r R/2 R r
DAY TWENTY ONE MAGNETIC EFFECT OF CURRENT 249

12 A current carrying circular loop of radius R is placed in Region containing


the xy-plane with centre at the origin. Half of the loop with Magnetic field
x > 0 is now bent, so that it now lies in the yz-plane.
(a) The magnitude of magnetic moment now diminishes
1 2 3 4
(b) The magnetic moment does not change
r2 r1
(c) The magnitude of B at (0, 0, z), z>>R increases
(d) The magnitude of B at(0, 0, z), z>>R is unchanged r4 r3

13 A 100 turn rectangular coil ABCD (in xy-plane) is hung


The table below gives the masses and charges of the ions
from one arm of a balance (shown in figure). A mass
500 g is added to the other arm to balance the weight of ION MASS CHARGE
the coil. A current of 4.9 A passes through the coil and a A 2m +e
constant magnetic field of 0.2 T acting inward (in B 4m −e
xz-plane) is switched on such that only arm CD of length C 2m −e
1cm lies in the field. How much additional mass m must D m +e
be added to regain the balance?
The ions fall at different positions 1,2,3 and 4 as shown.
Correctly match the ions with respective falling positions
Column I Column II
A 1
B 2
C 3
A B D 4

A B C D A B C D
D C (a) 4 3 2 1 (b) 1 2 3 4
(c) 4 1 2 3 (d) 3 4 1 2
(a) 2 g (b) 1 g (c) 0.75 g (d) 1.5 g
16 Two long parallel wires are at a distance 2d apart. They
14 A charge Q is uniformly distributed over the surface of carry steady equal current flowing out of the plane of the
non-conducting disc of radius R. The disc rotates about
paper as shown. The variation of the magnetic field along
an axis perpendicular to its plane and passing through its
the line XX ′ is given by ª AIEEE 2010
centre with an angular velocity ω. As a result of this
rotation, a magnetic field of induction B is obtained at the B
centre of the disc. If we keep both the amount of charge
placed on the disc and its angular velocity to be constant
(a) X X′
and vary the radius of the disc then the variation of the
magnetic induction at the centre of the disc will be
represented by the figure. ª JEE Main (Online) 2013 d d
B

B B
(a) (b) (b) X X′

d d
R R B

B B (c) X X′
(c) (d)
d d
B
R R

15 A beam consisting of four types of ions a,b,c and d (d) X X′


enters a region that contains a uniform magnetic field as
shown in figure. The field is perpendicular to the plane of d d
the paper, but its precise direction is not given. All ions in
the beam travel with the same speed.
250 40 DAYS ~ JEE MAIN PHYSICS DAY TWENTY ONE

ANSWERS

SESSION 1 1 (a) 2 (d) 3 (a) 4 (c) 5 (b) 6 (d) 7 (a) 8 (c) 9 (b) 10 (a)
11 (b) 12 (d) 13 (c) 14 (c) 15 (a) 16 (c) 17 (c) 18 (b) 19 (d) 20 (c)
21 (d) 22 (a) 23 (c) 24 (d) 25 (d) 26 (c) 27 (b) 28 (a)

SESSION 2 1 (a) 2 (c) 3 (b) 4 (d) 5 (c) 6 (b) 7 (d) 8 (d) 9 (b) 10 (b)
11 (a) 12 (a) 13 (b) 14 (a) 15 (c) 16 (a)

Hints and Explanations


SESSION 1 =
m 0I m 0I
-
m I
= 0 (b - a) H 2 = Magnetic field at M due to QR
24a 24b 24ab + magnetic field at M due to QS
1 Loosely wound helix get compressed
when possess current. So, connection of 6 Consider the wire to be made up of + magnetic field at M due to PQ
H 3
mercury lost and demagnetisation takes large number of thin wires of infinite = 0 + 1 + H1 = H1
place. So, result a oscillatory motion. length. Consider such wire of thickness 2 2
H1 2
dl subtending an angle d q at centre. Þ =
2 B 0 = B PQS + B SP H2 3
m0 I m 1
= (2p - 2q) + 0 9 Electric field can deviate the path of the
4p R 4p R cos q
particle in the shown direction only
dq

(sin q + sin q) q when it is along negative y-direction. In


m 0I q dB the given option E is either zero or along
= (p - q + tan q) q
2pR x-direction. Hence, it is the magnetic
q
field which is really responsible for its
3 Magnetic field due to straight parts of curved path. Options (a) and (c) cannot
dB
wire at point O = 0. Field due to a
dq be accepted as the path will be helix in
semicircular current loop of radius Current through this wire, dI = I that case (when the velocity vector
m I p m I
R1 , B1 = 0 æç ö÷ = 0 into the plane p makes an angle other than 0°,180 or 90°
2R1 è 2p ø 4R1 \ Magnetic field at centre due to this with the magnetic field, path is a helix).
of paper. portion,
Option (d) is wrong because in that case
m 2dI m I
Field due to semicircular current loop of dB = 0 × = 02 d q component of net force on the particle
m I 4p R 2p R also comes in k direction which is not
radius R 2 , B2 = 0 outside the plane Net magnetic field at the centre
4R 2 acceptable as the particle is moving in
p/2
xy-plane. Only in option (b) the particle
of paper.
B = ò- p/2dB cos q can move in xy-plane.
\ Net field B = B1 - B2 m 0I p/2
In option (d) : Fnet = qE + q(v ´ B)
m 0I æ 1 1 ö
= 2
2p R
ò - p/2 cos q d q
Initial velocity is along x-direction. So let
= ç - ÷ m 0I
4 çR R ÷ = v = vi$
è 1 2ø
p2 R \ Fnet = qa $i + q[(v$i ) ´ (ck$ + b$j )]
4 Equivalent current, I = ev 7 Consider the two coaxial solenoids. Due = qa$i - qvc$j + qvbk$
2pr
to one of the solenoids magnetic field at In option (b),
Hence, magnetic field at centre of circle the centre of the other can be assumed
m ev Fnet = q(a$i ) + q[(v$i ) ´ (ck$ + a$i )
m I m ev to be constant. Due to symmetry, forces
B = 0 = 0× = 0 2
2r 2r 2pr 4pr on upper and lower part of a solenoid = qa$i + qv$j
m 0 ev v will be equal and opposite and hence
Þ r = Þr µ 10 From Lorentz force, F = q(v ´ B )
4pB B resultant is zero.
Given, F = (4 i$ + 3 $j ) ´ 10-10 N,
Therefore, F1 = F2 = 0
5 Net magnetic field due to loop ABCD at q = 10-9C, B = 4 ´ 10-3 kT $
O is 8 Magnetic field at any point lying on the $ $
\ (4 i + 3 j ) ´ 10 -10
B = B AB + B BC + BCD + BOA current carrying straight conductor is
m I p m I p zero. = 10-9(a i$ + b $j ) ´ (4 ´ 10-3 )
= 0+ 0 ´ + 0- 0 ´ Here, H 1 = Magnetic field at M due to
4pa 6 4pb 6 Solving, we get
current in PQ a = - 75, b = 100 Þ v = - 75 $i + 100 $j
DAY TWENTY ONE MAGNETIC EFFECT OF CURRENT 251

2
11 From Bqv = mv , we have 18 As, M = I ´ Area of loop 24 Statement I is false, as B is perpendicular
r é pa2 ù to F, so particle cannot follow magnetic
= I ´ ê a2 + ´ 4ú k
mv 2mK 4´2 field lines (tangent to which gives the
r = = ë û
Bq Bq direction of magnetic field).
2ép p
where, K is the kinetic energy. = I´a ù
+ 1 k = ç + 1ö÷ a2 I k
æ
êë 2 úû è2 ø 25 Statement I is false and Statement II is
As, kinetic energies of particles are true. Magnetic force may have different
same; 19 As, B = m 0 2pI = m 0I value in different frames of reference,
m 4p r 2r that’s why it is not invariant in nature.
r µ
q 2Br
Þ I =
m0 26 Cyclotron frequency is given by
me mp 4m p 1 /2 1 Bq
Þ re : r p : r a = : : A n= =
e e 2e Also A = pr 2 or r = æç ö÷ T 2pm
èpø
Clearly, r p = r a and re is least It is obvious that cyclotron frequency
2Br
[Q me < m p] Magnetic moment, M = IA = A does not depend upon velocity of
m0 charged particle.
So, re < r p = r a 1 /2
2BA æ A ö 2BA3 /2
= ´ç ÷ = 27 The magnetic field at a point due to
12 Cyclotron frequency, m0 èpø m 0p1 /2
Bq 1 ´ 1.6 ´ 10-19 current flowing through an infinitely
n= = 20 Since, B is uniform only torque acts on long conductor is given by
2pm 2p ´ 91. ´ 10-31 a current carrying loop. m 2I
B = 0× .
= 2.8 ´ 1010 Hz = 28 GHz As, t = M ´ B Þ |t| = |M||B| sin q 4p a
For orientation shown in figure (ii), where, a is the distance of that point
13 Time taken by to make one revolution
25 q = 00, t = 0 (stable equilibrium) from conductor. Now, according to right
= = 5 ms hand thumb rule it follows that magnetic
5 and for figure (iv), q = p, t = 0
(unstable equilibrium) field is in the form of concentric circles,
2pm T m q whose centres lie on the straight
As T = ; so 2 = 2 ´ 1
qB T1 m1 q2 21 Suppose a resistance R s is connected in conductor.
series with galvanometer to convert it
m q 5 ´ 4 m1 q 28 We know that,
or T2 = T1 2 1 = = into voltmeter.
m1q2 m1 2q Ig RS m é 2pm ù
T µ êQ T = ú
= 10 ms G q Bq û
ë
14 Initial force F = m 0 × 2I1 I2 (attractive) For a-particle,
4p d 4m
V Ta µ
m 0 2 (2I1 )I2 2q
and final force F ¢ = × V
4p (3d ) I g (G + R s ) = V Þ R = -G For proton,
2 Ig
(repulsive) Þ F ¢ = F m
T p+ µ
3 Þ R = 1985 = 1.985 kW q
or R = 1.985 ´ 103 W
15 Along the line AB, magnetic field due to So, 2T p = T a
two wires is equal but in mutually 22 Maximum voltage that can be applied
opposite directions (as per right hand across the galvanometer coil SESSION 2
thumb rule). Hence, net magnetic field = 100 W ´ 10-3 A = 01
. V
will be zero along the line AB. 1 The resistance of the portion PRQ will be
Ammeter R1 = l 1 r
16 IdIB 1mA 100 W
I2
Q
dq S O
T T RS P
I1

dq If R s is the shunt resistance, then


2T sin æç ö÷ = IdlB
è2ø R s ´ 10 A = 01 . V
dq Þ R s = 0.01 W Resistance of the portion PSQ will be
Þ 2T = IRdqB [Q dl = Rdq] R 2 = l2 r
2 23 In this case, the charged particle
BIL Potential difference across P and Q
Þ T = BIR = [Q L = 2pR] performs uniform circular motion and
2p = I1 R1 = I2 R2
magnetic force is providing the
I1 l 1 r = I2 l 2 r or I1 l 1 = I2 l 2 ...(i)
m 0 2I1 m 0 2I2 necessary centripetal force,
17 - = 10 mT
4p r 4p r mv 2 Magnetic field induction at the centre O
i.e. = qvB due to currents through circular
m 02I1 m 0 2I2 r
+ = 30 mT conductors PRQ and PSQ will be
4pr 4p r mv 2
is not the force acting on charged = B1 - B 2
On solving, I1 = 20 A and I2 = 10 A r
m I l sin 90° m 0 I2 l 2 sin 90°
So, I1 / I2 = 2 particle it is simply equal to net force = 0 1 1 2 - =0
acting on the particle. 4p r 4p r2
252 40 DAYS ~ JEE MAIN PHYSICS DAY TWENTY ONE

2 Refer to the figure, number of turns in Number of turns in the ring As, 2pR = l (l is length of a wire)
N × dx Ndx l
dx, n = . = dN = R=
b-a b-a 2p
m0 I
Þ BA =
l
dx 2´
x dx 2p
b m Ip
x = 0 ...(i)
a l

b
a
BA O I
The magnetic field induction at the Magnetic field at the centre due to the
centre O due to current I through the ring element R
entire spiral is m (dN )I m 0I Ndx 1
dB = 0 = × ×
b m 2pnIdx 2x 2 (b - a) x
B =ò 0
a 4p x \ Field at the centre Magnetic field in case of square of side a,
m0 b æ N ö dx m 0NI bdx we get
= ò dB =
2(b - a) òa x
= 2p ç ÷ I
4p ò a è b - a ø x
m I æ 1 1 ö
BB = 4 ´ 0 ´ ç + ÷
m 0NI b 4p æ a ö è 2 2ø
m 0 NI m NI b = In ç ÷
= (log e x )ab = 0 log e 2(b - a) a è2ø
4p (b - a) 2 (b - a) a 4Im 0 m 02 2I
6 Consider free body diagram of the wire. Þ BB = =
3 In the figure, the z-axis points out of the pa 2 ap
As the wires are in equilibrium, they
paper and the magnetic field is directed must carry current in opposite
into the paper, existing in the region direction. BB
between PQ and RS. The particle moves a
in a circular path of radius r in the 45° 45°
magnetic field. It can just enter the
L q T cos q
region x > b for r ³ (b - a) . q
T 8 2 m0 I
l
q As, 4a = l, a = Þ BB = ...(ii)
Q S 4 pl
y d
FB
T sin q Dividing Eq. (i) by Eq. (ii), we get
ÄB BA p2
mg=lLg =
x>B BB 8 2
v 2
m 0I L
O x=a x=b
Here, F B = , where, L is length of 8 Case I x < R ,
x 2pd 2
P R each wire are d is separation between |B|= 0 R/2
wires. x
mv R x
Now, r = ³ (b - a) Case II £ x < R
qB 2 R
q(b - a)B q(b - a)B ò B × d l = m 0I
or v ³ Þ v min = r P
m m |B|2px = m 0
2
2 pnI 2 pnIr 2 m Jæ R2 ö é 2 æRö ù
4 B1 = m 0 m
and B2 = 0 2 |B|= 0 ç x2 - ÷ ê px - p ç ÷ ú J
4p r 4 p (r + h2 )3 /2 2x è 4 ø êë è2ø ú
û
3 From figure, d = 2L sin q
- Case III x ³ R
B æ h2 ö 2 T cos q = mg = lLg
So, 2 = ç1 + 2 ÷
B1 è r ø (in vertical direction) …(i) ò B × d l = m 0I
m 0I 2 L é R ù
2
Fractional decrease in the magnetic T sin q = F B = |B|2px = m 0 ê pR2 - p æç ö÷ ú J
B - B2 æ B ö 4pL sin q è2ø ú
field will be = 1 = ç1 - 2 ÷ êë û
(In horizontal direction) …(ii)
B1 è B1 ø m0J 3 2
From Eqs. (i) and (ii), we get |B|= R ,
é -3 /2 ù 2x 2
æ h2 ö T sin q m 0I 2 L
= ê1 - ç1 + 2 ÷ ú = 3m 0 JR2
ê è r ø ú T cos q 4pL sin q ´ l Lg |B|=
ë û 8x
æ 3 h 2ö
3 h2 4plLg sin2 q plLg So,
= 1 - ç1 - × 2 ÷ = \I = = 2sin q
è 2 r ø 2 r2 m 0 cos q m 0 cos q
|B |
5 Number of turns per unit width = N 7 Magnetic field in case of circle of radius
b-a R, we have
Consider an elemental ring of radius x m0 I r
BA = R/2 R
and thickness dx. 2R
DAY TWENTY ONE MAGNETIC EFFECT OF CURRENT 253

9 The electrons will be refocussed after x 2p m v Magnetic field B = 0.2 T (xz-plane)


Þ log =-
distance equal to pitch. x0 m 0 qI Length of arm CD = 1 cm,
-2 p m v
2pm Mass added to balance = m
Pitch = V x T = V m 0 qI
eB Þ x = x0 e Let F be the force due to magnetic field.
2eV 0 2pm 12 For a circular loop of radius R, carrying The direction of magnetic field is inward
= ×
m eB current I in xy-plane, the magnetic in xz-plane, the length vector is left, so
2 moment M = I ´ pR2 . It acts by using the Fleming’s left hand rule, the
8p mV 0 éQ eV = 1 mv 2 ù direction of force is downwards in the
= 0 perpendicular to the loop along
eB 2 êë 2 úû plane of paper
z -direction. When half of the current
loop is bent in y z- plane, then magnetic F = I (l ´ B ) = 4.9 (0.01 ´ 0.2 sin 90° )
10 When force exerted on a current F = 4.9 ´ 0.01 ´ 0.2 ...(i)
carrying conductor moment due to half current loop in
Fext = B IL x y-plane, M1 = I (pR2 / 2) acting along For balancing,
Work done z-direction. Magnetic moment due to mass of coil ´ g + force due to
Average power = half current loop in yz-plane, magnetic field
Time taken
2 2 M2 = I (pR2 / 2) along x-direction. = 500 ´ 10- 3 g + m ´ g
1 1
P = ò Fext. . dx = ò B ( x )IL dx
t 0 t 0 Effective magnetic moment due to 0.5 ´ 9.8 + 4.9 ´ 0.01 ´ 0.2
2 entire bent current loop, = 500 ´ 10-3 ´ 9.8 + m ´ 9.8
1 . x
-4 -02
= ò 3 ´ 10 e ´ 10 ´ 3 dx M ¢ = M12 + M22 9.8 (0.5 + 0.001) = 9.8 (0.5 + m )
5 ´ 10-3 0
-0. 4 = (I p R2 / 2)2 + (I p R 2 / 2)2 [From Eq. (i)]
= 9 [1 - e ]
m = 0.001 kg = 1 g
1 IpR2
= 9 é1 - 0. 4 ù = 2.967 » 2.97 W = 2<M Thus, 1 g of mass must be added to
êë e úû 2
regain the balance.
i.e. magnetic moment diminishes
11 F = q($i v x + $j v y ) ´ éê m 0I k$ ùú 14 dB =
m 0(dq ) æ w ö
ë 2px û The magnitude of B at a point on the ç ÷
axis of loop, distance z from the centre 2r è 2p ø
$ m 0I $ m I
= jq v x - iq v y 0 of current loop in x y-plane is m w Q R rdr
B = ò dB = 0 × 2p
4p pR2 ò0 r
2px 2px
Y m0 2p / R2
B = m wQ m wQ 1
4p (R + z2 )3 /2
2 B = 0 2 × R, B = 0 Þ B µ
x0 2pR 2pR R
I The magnitude of B at a point distance z
I X 15 A ® 4, B ® 1, C ® 2, D ® 3
from the centre of bent current loop,
whose half part is in R = mv /qB
xy-plane and half part is in y x- plane, is RB > R A
Fx m 0Iqv y 2 and R A = R C (in opposite sense)
\ ax = =- é m0 p / R2 ù
m 2p xm ê and R D is smallest.
2 2 3 /2 ú
ë 4p ( R + z ) û
dv x dv x dx v x dv x B =
Also, ax = = × = 2 16 The magnetic field in between because of
dt dx dt dx é m0 p / R2 ù
+ ê 2 2 3./2 ú each will be in opposite direction
Since, v 2x + v 2y = v 2 ë 4p (R + z ) û m I m 0I
B in between = 0 $j - (- $j )
or 2v x dv x + 2v ydv y = 0 m0 p / R2 2px 2p (2d - x )
= 2<B
Þ v x dv x = - v y dv y 4p (R + z2 )3 /2
2
m 0I é1 1 ù $
Hence,
= ê - ú(j )
2p ë x 2d - xû
v x dv x v ydv y m 0Iq v y 13 Let the mass of coil = M
=- =- Mass added other arm = 500 g at x = d , B in =0
dx dx 2p x m between
= 500 ´ 10-3 kg For x < d , B in = ( $j ) and
dx dv y2p m between
Þ = Mass of coil = Mass in other arm
x m 0 qI For x > d , B in between = (- $j )
(for balancing)
Initially, x = x 0 and v y = 0 M = 500 ´ 10- 3 kg Towards x, net magnetic field will add
At minimum separation v x = 0, v y = v
M = 0.5 kg up and direction will be (- j ) and
x dx 2pm - v
Thus, ò = dv y
m 0 qI ò 0
x0 x
When the current is switched on, towards x¢, net magnetic field will add
Current, I = 4.9 A up and direction will be ( j ).
DAY TWENTY TWO

Magnetism
Learning & Revision for the Day

u Current Loop as a Magnetic u Magnetic Field Lines u Hysteresis Curve


Dipole u The Earth’s Magnetism u Electromagnet
u Bar Magnet u Magnetic Behaviour of Materials

Current Loop as a Magnetic Dipole


A current loop is equivalent to a magnetic dipole. If A(= πa2 ) be the area of the loop, then
the magnitude of its dipole moment is
pm = iA = iπa2
where, a is radius of coil, i is current flowing through it
p
i = m2 …(i)
πa
Magnetic field at the centre of a circular current loop is given by
µ i
B= 0 …(ii)
2a
Putting the value of i from Eq. (i) in Eq. (ii), we get
µ p
B= 0 m
2 π a3
This is the expression for the magnetic field at the centre of the current loop in terms of
its dipole moment. Instead of circular loop, if there is a circular coil having n turns, its
dipole moment would be pm = ni A = ni πa2 .

Bar Magnet PREP


A bar magnet may be viewed as a combination of two magnetic poles, North pole and
MIRROR
Your Personal Preparation Indicator
South pole, separated by some distance. The distance is known as the magnetic length
of the given bar magnet. u No. of Questions in Exercises (x)—
A bar magnet exhibits two important properties, namely u No. of Questions Attempted (y)—
(i) the attractive property and (ii) the directive property. u No. of Correct Questions (z)—
(Without referring Explanations)
l
If m is the pole strength and 2l the magnetic length of the bar magnet,then its
magnetic moment is M = m (2 l ). u Accuracy Level (z / y × 100)—
l
Magnetic moment is a vector quantity whose direction is from S pole towards N pole. u Prep Level (z / x × 100)—
l
SI unit of magnetic pole strength (m), is ampere metre (Am) and of magnetic dipole
moment (M) is ampere metre2 (Am2 ). In order to expect good rank in JEE,
your Accuracy Level should be above
l
If a bar magnet is broken, the fragments are independent magnetic dipoles and not 85 & Prep Level should be above 75.
isolated magnetic poles.
DAY TWENTY TWO MAGNETISM 255

Magnetic Field due to a Bar Magnet Bar Magnet as an Equivalent Solenoid


The magnetic field in free space, at a point having distance r The magnetic field (axial) at a point at a distance r and
from the given bar magnet (or magnetic dipole) is calculated µ 0 nlIa2
in two conditions, along axial line and along equatorial line. radius a of solenoid is given by B =
r3
µ 2 Mr
l
Along axial line B = 0 2 and magnetic moment of solenoid is M = n (2 l) Iπa2
4π (r − l 2 )2
and the direction of B is the same as the direction of M. For NOTE • Even a single electron moving in its orbit behaves as a
a short dipole (or for a far away point on the axis) when magnetic dipole and has a definite magnetic moment.
r > > l , the above relation is simplified as • Bohr magnetron is the magnetic moment due to the orbital
µ 2M motion of an electron revolving in the inner most orbit
B= 0 3 eh
4π r ( n = 1). Its value is m B = = 9.27 × 10 −24 A-m 2
N 4 πme
S

P Torque on a Magnetic Dipole in a


l
r Magnetic Field
l
Along the equatorial line A magnetic dipole when placed in an uniform magnetic field,
µ M does not experience any net force. However, it experiences a
B= 0 2 torque given by τ = M × B or τ = MB sin θ
4π (r + l 2 )3 /2
where, θ is the angle from the magnetic field, along which the
µ0 M
If r > > l , the relation is modified as, B = dipole has been placed.
4π r 3 l
Work done in rotating a magnetic dipole in a uniform
However, along the equatorial line, magnetic field B from an initial orientation θ1 to the final
the direction of B is opposite to that of M. orientation θ2 , is given by W = MB(cos θ1 − cos θ2 ).
q m1
qm1
Normal
l I Normal
r

I
r r θ1 θ2
l

I I
S N qm2
qm2
l
Initial position Final position
fig. (i) fig. (ii)
l
In general, in a direction making an angle θ from with the
magnetic axis, the magnetic field is given by l
Potential energy of a magnetic dipole placed in a uniform
µ M magnetic field, is given by UB = − M ⋅ B = − MB cos θ
B = 0 3 (3 cos2 θ + 1)
4π r where, θ is the angle from the direction of magnetic field
In these relations, µ 0 is a constant having a value of and the axis of dipole.
4π × 10 −7 T mA −1 and it is known as the magnetic l
The magnetic compass (needle) of magnetic moment M
permeability of free space. and moment of inertia I and allowing it to oscillate in the
magnetic field. Then, its time-period is T = 2 π I / MB
For solenoid B = µ 0 ni
where, n is number of turns per unit length of solenoid and l
Behaviour of a magnetic dipole in a magnetic field, is
i the current through it. similar to the behaviour of an electric dipole in an electric
1 µ
B Br field. However, the constant is replaced by 0 .
Bθ α 4πε 0 4π

P
l
If a magnetic dipole is in the form of a wire or a thin rod,
when bent, its magnetic dipole moment M changes because
the separation between its poles has changed.
r

Magnetic Field Lines


S M
N The magnetic field lines is defined as the path along which
O the compass needles are aligned. They are used to represent
magnetic field in a region.
256 40 DAYS ~ JEE MAIN PHYSICS DAY TWENTY TWO

l
Magnetic field lines are closed continuous curves. Similarly, at the magnetic equator, BV = BE sin 0 ° = 0
l
Tangent drawn at any point on magnetic field lines gives and at the poles, BV = BE sin 90 ° = BE .
the direction of magnetic field at that point. Magnetic elements of the earth at a place change with time also.
l
Two magnetic field lines cannot intersect each other.
l
Outside a magnet, they are directed from north to south pole
Neutral Points
and inside a magnet they are directed from south to north. A neutral point is a point at which the resultant magnetic field
is zero. Following two cases are of special interest.
The Earth’s Magnetism 1. When a bar magnet is placed along the magnetic meridian
The earth is a natural source of magnetic field, thus a with its North pole pointing towards geographical North,
magnetic field is always present everywhere near the surface of two neutral points are obtained on either side of the
the earth. A freely suspended magnet always points in the magnet along its equatorial line. If r be the distance of the
µ M
north-south direction even in the absence of any other neutral point, then 0 3 = BH .
magnet. This suggests that the earth itself behaves as a magnet 4π r
which causes a freely suspended magnet (or magnetic needle) 2. When a bar magnet is placed along the magnetic
to point always in a particular direction : north and south. The meridian, with its North pole pointing towards the
shape of earth’s magnetic field resembles that of a bar magnet geographical South, two neutral points are obtained on
of length one-fifth of earth’s diameter buried at its centre. either side of the magnet along its axial line.
Magnetic field of earth is shown in the figure given below. µ 2M
Hence, we have 0 = BH .
Geographical L 4π r 3
North
Magnetic P α O Tangent Galvanometer
North H δ
It is an instrument to measure electric current. The essential
Geographical S parts are a vertical coil of conducting wire and a small
meridian compass needle pivoted at centre of coil. The deflection, θ of
Magnetic N needle is given by,
meridian M V B µ IN
tan θ = ⇒ BH tan θ = 0
BE BH 2r
Q R
2 r BH
or i= tan θ = K tan θ
µ 0N
Magnetic Elements of Earth
l
Angle of Declination (α) At a given place, the acute angle Magnetisation of Materials
between the magnetic meridian and the geographical There are some substances/materials which acquire magnetic
meridian is called the angle of declination (or magnetic properties on placing them in magnetic field. This phenomena
declination) α at that place. is called magnetisation of materials.
l
Angle of Inclination or Dip (δ ) The angle of dip δ at a place To describe the magnetic properties of material, we have to
is the angle which the direction of the earth’s total understand the following terms:
magnetic field BE subtends with the horizontal direction. (i) Magnetic Induction or Magnetic Flux Density (B)
Whenever a piece of magnetic substance is placed in an
l
Horizontal Component of the Earth’s Magnetic Field
external magnetising field, the substance becomes
( B H ) As earth’s magnetic field, in general, is inclined at an magnetised. If B0 is the magnetic field in free space, then
angle δ with the horizontal direction, it may be resolved B = µ r B0 .
into horizontal component BH and a vertical component BV ,
where BH = BE cos δ and BV = BE sin δ ∫ B ⋅ dS is magnetic flux which is equal to µ 0 m inside , where
m inside is the net pole strength inside a close surface.
B
⇒ BE = B2H + B2V and tan δ = V (ii) Magnetic Permeability (µ ) It is the degree or extent to
BH
which the magnetic lines of induction may pass through a
given distance.
Variation of Magnetic Elements Magnetic permeability of free space µ 0 has a value of
4π × 10 −7 TmA −1 . However, for a magnetic material,
of the Earth
absolute permeability ( µ) has a value, different than µ 0 .
At the magnetic equator, angle of dip is zero. Value of the
angle of dip gradually increases, on going from equator to µ B
For any magnetic substance, = = µ r = relative
magnetic poles. At the magnetic poles, value of the dip angle µ 0 B0
is 90°. magnetic permeability of that substance. Relative
At the magnetic equator, BH = BE cos 0 ° = BE magnetic permeability µ r is a unitless and dimensionless
and at poles, BH = BE cos 90 ° = 0. term.
DAY TWENTY TWO MAGNETISM 257

(iii) Intensity of Magnetisation (I) Intensity of magnetisation of a Oxygen, air, platinum, aluminium, etc., are examples of
substance is defined as the magnetic moment induced in the paramagnetic materials.
substance per unit volume, when placed in the magnetising l
In a paramagnetic material, the net magnetic moment
M
field. Thus, I = of every atom is non-zero.
V l
Paramagnetic materials are feebly attracted in an
It is a vector quantity and its SI unit is Am−1 .
external magnetic field and thus, have a tendency to
(iv) Intensity of Magnetising Field or Magnetic Intensity (H) It is a shift from the weaker to the stronger regions of
measure of the capability of external magnetising field to magnetic field.
magnetise the given substance and is mathematically defined l
The relative permeability µ r of a paramagnetic
as
material is slightly greater than one (µ r > 1). Magnetic
B B susceptibility χ m of paramagnetic materials is positive.
H = 0 or H =
µ0 µ l
Paramagnetism is temperature dependent. According
Magnetic intensity H is a vector quantity and its SI unit is Am −1 . to the Curie’s law, the magnetic susceptibility of a
(v) Magnetic Susceptibility ( χ m ) Magnetic susceptibility of a paramagnetic substance is inversely proportional to its
temperature T.
substance is the ratio of the intensity of magnetisation I induced
I C
in the substance to the magnetic intensity H.Thus, χ m = .It is Mathematically, χ m = , where C is the Curie constant.
H T
a scalar quantity and it has no units or dimensions.
Relation between µ r and χ m we have, B = µ 0 (I + H ) Ferromagnetic Materials
 I  B These are the materials which are strongly attracted by a
or B = µ 0 H  + 1 or B = B0 (χ m + 1) or = χm + 1
H  B0 magnetic field and can themselves be magnetised even in
a weak magnetising field. Iron, steel,nickel and cobalt are
B µ
But = = µ r = relative permeability ferromagnetic.
B0 µ 0 l
These materials show a large increase in the magnetic
∴ µr = χm + 1 flux, when placed in a magnetic field. Thus, for them
µ r > > 1. Accordingly, χ m is positive and large.
Magnetic Materials l
Ferromagnetic materials exhibit all properties
According to behaviour of magnetic substances, they are classified exhibited by paramagnetic substances and by a much
into three cases: larger measure.
l
Magnetic susceptibility of ferromagnetic materials
Diamagnetic Materials decreases steadily with a rise in temperature. Above a
These are materials which show a very small decrease in magnetic certain temperature Tc (known as Curie temperature),
flux, when placed in a strong magnetising field. Hydrogen, water, the substance loses its ferromagnetic character and
copper, zinc, antimony, bismuth, etc., are examples of diamagnetic begins to behave as a paramagnetic substance.
materials. l
Above the Curie temperature Tc , the magnetic
l
In a diamagnetic material, the net magnetic moment (sum of susceptibility of a ferromagnetic material varies as
that due to orbital motion and spin motion of electrons) of an 1 C
atom is zero. The external magnetic field B distorts the electron χm ∝ or χ m =
(T − Tc ) (T − Tc )
orbit and thus induces a small magnetic moment in the opposite
direction. where, C is a constant. It is known as the Curie-Weiss
l
Diamagnetic materials are feebly repelled in an external law.
magnetic field and thus have a tendency to shift from the
stronger to weaker regions of the magnetic field. Hysteresis Curve
l
The relative permeability of any diamagnetic substance is The lag of intensity of I
slightly less than 1 (i.e. µ r < 1) and susceptibility has a small magnetisation behind the
A
negative value. magnetising field during B
the process of Retentivity
l
Diamagnetism is an intrinsic property and does not vary with
magnetisation and C
H
magnetic field B or temperature. demagnetisation of a O F
ferromagnetic material is
Paramagnetic Materials called hysteresis. The D
E

These are the materials which show a small increase in the whole graph ABCDEFA is a
magnetic flux when placed in a magnetising field. closed loop and known as Coercivity
hysteresis loop.
258 40 DAYS ~ JEE MAIN PHYSICS DAY TWENTY TWO

The effect is the multiplication of the magnetic field by


Electromagnet factors of ten to eleven thousands. The solenoid field
Electromagnets are usually in the form of iron core solenoids. relationship is B = k µ 0 nI , where µ = k µ 0 and k is the
The ferromagnetic property of the iron core causes the internal relative permeability of the iron, the figure shows the
magnetic domains of the iron to line up with the smaller magnetic effect of the iron core.
driving magnetic field produced by the current in the
solenoid.
Permanent Magnet
N = North pole Substances which at room temperature retain their
N
ferromagnetic property for a long period of time are called
permanent magnets. Permanent magnets can be made in a
Iron core variety of ways.
An efficient way to make a permanent magnet is to place a
S ferromagnetic rod in a solenoid and pass a current. The
S = South pole magnetic field of the solenoid magnetises the rod.

DAY PRACTICE SESSION 1

FOUNDATION QUESTIONS EXERCISE


1 A short bar magnet placed with its axis at 30° with a (a) 0.2 T (b) 0.3 T (c) 0.5 T (d) 0.4 T
uniform external magnetic field of 0.16 T, experiences a 5 A magnetic needle of magnetic moment 6.7 × 10−2 Am 2
torque of magnitude 0.032 J. The magnetic moment of and moment of inertia 7.5 × 10−6 kg m 2 is performing
the bar magnet will be simple harmonic oscillations in a magnetic field of 0.01 T.
(a) 0.23 JT −1 (b) 0.40 JT −1 Time taken for 10 complete oscillations is
(c) 0.80 JT −1 (d) zero ª JEE Main 2017 (Offline)
2 A bar magnet of length 10 cm and having pole strength (a) 8.89 s (b) 6.98 s
equal to 10−3 Wb, is kept in a magnetic field having (c) 8.76 s (d) 6.65 s
magnetic induction B equal to 4π × 10−3 T. It makes an 6 Hysteresis loops for two magnetic materials A and B are
angle of 30° with the direction of magnetic induction. The as given below
value of the torque acting on the magnet is
B B
(a) 0.5 N-m (b) 2 π × 10−5 N-m
(c) π × 10−5 N-m (d) 0.5 × 10 2 N-m
3 M and M 3 are the M H H
magnetic dipole moments
of the two magnets, which
q (A) (B)
are joined to form a cross B
figure. The inclination of These materials are used to make magnets for electric
the system with the field, if generators, transformer core and electromagnet core.
their combination is MÖ3
Then, it is proper to use ª JEE Main 2016 (Offline)
suspended freely in a
(a)A for electric generators and transformers
uniform external magnetic field B is (b)A for electromagnets and B for electric generators
(a) θ = 30° (b) θ = 45 ° (c) θ = 60° (d) θ = 15 ° (c)A for transformers and B for electric generators
−3 (d)B for electromagnets and transformers
4 A coil of 50 turns and area1.25 × 10 2
m is pivoted
about a vertical diameter in an uniform horizontal 7 The coercivity of a small magnet where the ferromagnet
magnetic field and carries a current of 2 A. When the coil gets demagnetised is 3 × 103 Am −1. The current required
is held with its plane in the N-S direction, it experiences a to be passed in a solenoid of length 10 cm and number
couple of 0.04 Nm, and when its plane is along the of turns 100, so that the magnet gets demagnetised when
East-West direction, it experiences a couple of 0.03 Nm. inside the solenoid is ª JEE Main 2014
The magnetic induction is (a) 30 mA (b) 60 mA (c) 3 A (d) 6 A
DAY TWENTY TWO MAGNETISM 259

8 The figure illustrates how B, the cross-section. In (i) E is ideally treated as a constant
B
flux density inside a sample of between plates and zero outside. In (ii) magnetic field is
unmagnetised ferromagnetic P constant inside the solenoid and zero outside. These
material, varies with B0, the Q idealised assumptions, however contradict fundamental
magnetic flux density in which R
B0
laws as below
the sample is kept. For the O
(a) case (i) contradicts Gauss’ law for electrostatic fields
sample to be suitable for (b) case (ii) contradicts Gauss’ law for magnetic fields
S
making a permanent magnet. (c) case (i) agrees with ∫ E ⋅ d I = 0
(d) case (ii) contradicts ∫ H ⋅ d I = I en
(a) OQ should be large, OR should be small
(b) OQ and OR should both be large 14 A paramagnetic sample shows a net magnetisation of
(c) OQ should be small and OR should be large 8Am −1 when placed in an external magnetic field of 0.6 T
(d) OQ and OR should both be small at a temperature of 4 K. When the same sample is placed
9 Match the following columns. in an external magnetic field of 0.2 T at a temperature of
16 K, the magnetisation will be
Column I Column II 32 2
(a) Am−1 (b) Am−1 (c) 6 Am−1 (d) 2.4 Am−1
A. Magnetic moment 1. [ML0T −2A −1 ] 3 3
B. Permeability 2. Vector 15 If the areas under the I-H hysteresis loop and B-H
C. Intensity of magnetisation 3. Nm3 / Wb hysteresis loop are denoted by A 1 and A2, then
D. Magnetic induction 4. Scalar (a) A 2 = µ 0 A1 (b) A 2 = A1
A
Codes (c) A 2 = 1 (d) A 2 = µ 02 A 1
µ0
A B C D A B C D
(a) 1 2 3 4 (b) 3 4 2 1 16 A short magnet oscillates with a time period 0.1 s at a place,
(c) 4 3 1 2 (d) 2 1 3 4 where horizontal magnetic field is 24 µT. A downward
current of 18 A is established in a vertical wire 20 cm East
10 A current carrying coil is placed with its axis
of the magnet. The new time period of oscillator
perpendicular to N-S direction. Let horizontal component
of earth’s magnetic field be H0 and magnetic field inside (a) 0.1 s (b) 0.089 s (c) 0.076 s (d) 0.057 s
the loop be H. If a magnet is suspended inside the loop, 17 Two bar magnets of the same mass, same length and
it makes angle θ with H. Then, θ equal to breadth but having magnetic moments M and 2M,
H  respectively are joined together pole to pole and
(a) tan−1  0 
H
(b) tan−1  
H   H0  suspended by a string. The time period of the assembly
H  in a magnetic field of strength H is 3 s. If now the polarity
(d) cot −1  0 
H
(c) cosec −1   of one of the magnets is reversed and the combination is
 H0  H 
again made to oscillate in the same field, the time of
11 Two short bar magnets of length 1 cm each have oscillation is
magnetic moments 1.20 Am 2 and 1.00 Am 2 , (a) 3 s (b) 3 3 s (c) 3 / 3 s (d) 6 s
respectively. They are placed on a horizontal table
18 Two magnets A and B are identical and these are
parallel to each other with their N poles pointing towards
arranged as shown in the figure. Their lengths are
the South. They have a common magnetic equator and
are separated by a distance of 20.0 cm. The value of negligible in comparision
the resultant horizontal magnetic induction at the to the separation
between them. A A
mid-point O of the line joining their centres is close to B
(Horizontal component of the earth’s magnetic induction magnetic needle is P
q
is 3.6 × 10−5 Wb/m 2 ) ª JEE Main 2013 placed between the
magnets at point P and it
(a) 3.6 × 10–5 Wb / m2 (b) 2.56 × 10–4 Wb / m2 d1 d2
gets deflected by an
(c) 3.50 × 10–4 Wb / m2 (d) 5.80 × 10–4 Wb / m2
angle θ. The ratio of
12 The magnetic susceptibility of a paramagnetic substance distances d1 and d 2, will be
at –73°C is 0.0060, then its value at –173°C will be
(a) (2 cot θ)1/ 3 (b) (2 tan θ)1/ 3
(a) 0.0030 (b) 0.0120 (c) 0.0180 (d) 0.0045 (c) (2 cot θ)−1/ 3 (d) (2 tan θ)−1/ 3
13 Consider the two idealised systems (i) a parallel plate 19 The plane of a dip circle is set in the geographic meridian
capacitor with large plates and small separation and and the apparent dip is δ1. It is then set in a vertical plane
(ii) a long solenoid of length L >> R , radius of perpendicular to the geographic meridian.
260 40 DAYS ~ JEE MAIN PHYSICS DAY TWENTY TWO

The apparent dip angle is δ 2. The declination θ at the Statement I A current carrying loop is free to rotate. It is
place is placed in a uniform magnetic field. It attains equilibrium
(a) θ = tan−1 (tan δ1 tan δ 2 ) (b) θ = tan−1 (tan δ1 + tan δ 2 )
when its plane is perpendicular to the magnetic field.
 tan δ1  Statement II The torque on the coil is zero when its
(c) θ = tan−1   (d) θ = tan−1 (tan δ1 − tan δ 2 ) plane is perpendicular to the magnetic field.
 tan δ 2 
(a) Statement I is true, Statement II is true; Statement II is the
20 This question contains two statements : Statement I correct explanation for Statement I
(Assertion) and Statement II (Reason). The question also (b) Statement I is true, Statement II is true; Statement II is not
has four alternative choices, only one of which is the the correct explanation for Statement I
correct answer. You have to select one of the codes (a), (c) Statement I is true; Statement II is false
(b), (c), (d). (d) Statement I is false; Statement II is true

DAY PRACTICE SESSION 2

PROGRESSIVE QUESTIONS EXERCISE


1 The dipole moment of a circular loop carrying a current I, geographical and magnetic equators. The declination
is m and the magnetic field at the centre of the loop is B1. and dip angles at P and Q are
When the dipole moment is doubled by keeping the (a) 0° and 11.3° (b) 0° and 0°
current constant, the magnetic field at the centre of the (c) 11.3° and 6.5° (d) 11.3° and 11.3°
B
loop is B2. The ratio 1 is 6 A bar magnet has pole strength 3.6 A-m and length
B2 ª JEE Main 2018 12 cm. Its area of cross-section is 0.9 cm 2. The magnetic
(a) 2 (b) 3 field B at the centre of the bar magnet is
(c) 2 (d)
1 (a) 6 × 10−3 T (b) 5 × 10−2 T
2 (c) 2.5 × 10−2 T (d) 2.5 × 10−8 T
2 The current on the winding of a toroid is 2A. It has 7 A bar magnet suspended by a suspension fibre, is
400 turns and mean circumferential length is 40 cm. With placed in the magnetic meridian with no twist in the
the help of search coil and charge measuring instrument suspension fibre. On turning the upper end of the
the magnetic field is found to be 1T. The susceptibility is suspension fibre by an angle of 120° from the meridian,
(a) 100 (b) 290 the magnet is deflected by an angle of 30° from the
(c) 398 (d) 397 meridian. Then, the angle by which the upper end of the
suspension fibre has to be twisted, so as to deflect the
3 The length of a magnet is large compared to its width and
magnet through 90° from the meridian is
breadth. The time period of its oscillation in a vibration
(a) 270° (b) 240°
magnetometer is 2 s. The magnet is cut along its length
(c) 330° (d) 180°
into three equal parts and three parts are then placed on
each other with their like poles together. The time period 8 A bar magnet 8 cm long, is placed in the magnetic
of this combination will be meridian with the N pole, pointing towards the
2 geographical North. Two neutral points, separated by a
(a) 2 s (b) s distance of 6 cm are obtained on the equatorial axis of
3
2 the magnet. If BH = 3. 2 × 10−5 T, then the pole strength of
(c) 2 3 s (d) s
3 the magnet is
4 A thin circular disc of radius R is uniformly charged with (a) 5 A -cm2 (b) 10 A -cm2
2
density σ > 0 per unit area. The disc rotates about its axis (c) 2.5 A -cm (d) 20 A -cm2
with a uniform angular speed ω. The magnetic moment of 9 There are two current carrying planer coils made each
the disc is ª AIEEE 2011 from wire of length L. C1 is circular coil (radius R) and C2
(a) 2 πR 4 σω (b) πR 4 σω is square (side a). These are so constructed that they
πR 4 πR 4 have same frequency of oscillation when they are placed
(c) σω (d) σω in the same uniform magnetic field B and carry the same
2 4
current. The value of a in terms of R is
5 Consider the plane S formed by the dipole axis and the
(a) 3 R (b) 3R
axis of the earth. Let P be point on the magnetic equator
(c) 2R (d) 2R
and in S. Let Q be the point of intersection of the
DAY TWENTY TWO MAGNETISM 261

ANSWERS

SESSION 1 1 (b) 2 (a) 3 (c) 4 (d) 5 (d) 6 (d) 7 (c) 8 (b) 9 (b) 10 (a)
11 (b) 12 (b) 13 (b) 14 (b) 15 (a) 16 (c) 17 (b) 18 (a) 19 (c) 20 (c)

SESSION 2 1 (c) 2 (d) 3 (b) 4 (d) 5 (a) 6 (b) 7 (a) 8 (a) 9 (a)

Hints and Explanations


SESSION 1 5 Time period of oscillation is 10 In given case H and H 0 are
1 As, M = τ I perpendicular to each other.
B sin θ T = 2π
MB From figure,
0.032 H
= = 0.40 JT −1 tanθ = 0
. × sin 30°
016 7.5 × 10−6 H
⇒ T = 2π = 0.665 s
. × 10−2 × 0.01
67 H0
2 As, µ 0m = 10−3 Wb
10−3 Time taken to complete 10 oscillations
m=
µ0 = 10 × 0.665 = 6.65 s q
H
Magnetic moment of the magnet, Hence, time for 10 oscillations
10−3 10−4 is t = 6.65 s. θ = tan −1  0 
H
M = m × 2l = . )=
(01 ⇒
µ0 µ0  H 
6 Area of hysteresis loop is proportional
Now, τ = MB sin θ to the net energy absorbed per unit 11 B net = B1 + B2 + B H
 10−4  volume by the material, as it is taken
µ 0 ( M1 + M2 )
=  × 4 π × 10−3 sin 30° over a complete cycle of magnetisation. B net = + BH
 µ0  4π r3
For electromagnets and transformers,
= 0.5 N-m 10−7 (1.2 + 1)
energy loss should be low. = + 3.6 × 10−5
3 Torque (τ) acting on the magnet (1) is i.e. thin hysteresis curves. (0.1)3
τ1 = MB sin θ Also,|B | → 0 when H = 0 and|H | = 2.56 × 10−4 Wb/m2
τ2 = 3 MB sin θ should be small when B → 0. 1
12 As, χm ∝
For equilibrium, τ1 = τ2 7 For solenoid, the magnetic field needed T
∴ MB sin θ = 3 MB cos θ χ 2 T1
to be magnetised the magnet. ⇒ =
χ 1 T2
⇒ tan θ = 3 = tan 60° ⇒ θ = 60° H = nI
 where, N = 100,  χ2 273 − 73 200
or = = =2
4 Here, N = 50, A = 1.25 × 10−3 m2 ,  10  0.0060 273 − 173 100
 l = 10 cm = m = .
01 m 
I = 2A 100
M = NIA N 100 or χ2 = 0.0120
H = I ⇒ 3 × 10 = 3
×I
= 50 × 2 × 1.25 × 10−3 l .
01 13 As Gauss’ law states,
= 0.125 A-m2 I = 3A
q
If the normal to the face of the coil 8 For making permanent magnet, the ∫SE ⋅ dS = ε0
for electrostatic field. It does
makes an angle θ with the magnetic material should have high residual
induction B, the torque not contradict for electrostatic fields as
magnetism and high coercivity i.e. OQ the electric field lines do not form
τ = MB cos θ = 0.04 …(i) and OR should be large. continuous closed path. According to
Now, when the plane of the coil is
9 A - SI unit of magnetic moment is Gauss’s law in magnetic field,
turned through 90°, the torque
Nm3 / Wb.
becomes, ∫ E⋅d S = 0
τ = MB sin θ = 0.03 …(ii) B - Permeability (µ ) is a scalar. S

Squaring and adding Eqs. (i) and (ii), we M A It contradicts for magnetic field, because
get C-I = = −1 , it is a vector. there is a magnetic field inside the
V m
τ = 0.05 ⇒ MB = 0.05 solenoid and no field outside the
0.05 0.05 D - Magnetic induction solenoid carrying current but the
⇒ B = = = 0.4 T
M 0125
. B = ML0T −2 A −1 magnetic field lines form the closed path.
262 40 DAYS ~ JEE MAIN PHYSICS DAY TWENTY TWO

14 As Curie’s law explains, we can deduce 2 tan θ B = µ 0µ r µ


=
a formula for the relation between d 13 d 23 ⇒ µ 0µ
1
= 5 × 10−4
r =
magnetic field induction, temperature 3 2000
 d1  2
and magnetisation.   = = 2 cot θ 5 × 10−4 5 × 10−4
 d2  tan θ ⇒ µr = = = 398
i.e. I (magnetisation) µ0 4 π × 10−7
d1

B (magnetic field induction) = (2 cot θ)1 /3 ⇒ χ = µ r − 1 = 397
t (temperature in kelvin) d2
V 3 The time period of oscillations of

I2 B t
= 2 × 1 19 tan δ1 = magnet,
I1 B1 t 2 H cos θ
T = 2π  I 
Let us suppose, here I1 = 8 Am −1   …(i)

q
 MH 

s
co
B1 = 0.6T,t 1 = 4 K

H
q where, I = moment of inertia of magnet
B2 = 0.2T,t 2 = 16 K
H mL2

0.2 4
×
I
= 2 =
0.6 16 8 12
V [m, being the mass of magnet]
2
⇒ I2 = Am −1 H sin q
3 M = pole strength × L
V V and H = horizontal component of the
15 As, B = µ 0(H + l ) tan δ2 = =
H cos (90° − θ) H sin θ earth’s magnetic field.
⇒ dB = µ 0 dH + µ 0 dI
or ∫ HdB = µ 0 ∫ HdH + µ 0 tan δ1 sin θ
∫ H dI = = tan θ
When the three equal parts of magnet are
tan δ2 cos θ placed on one another with their like
Q ∫ HdH = 0 poles together, then
 tan δ1 
∫ HdB = µ 0 ∫ H ⋅ dI or θ = tan −1   1  m  L 
2
 tan δ2  I′=   ×   ×3
Area of the B - H loop 12  3   3 
= µ 0 × area of I-H loop 20 The torque experienced by a coil in a 1 mL2 I
= =
i.e. A2 = µ 0 A 1 magnetic field is, 12 9 9
16 Given, B H = 24µT τ = M × B = MB sin θ L
and M ′ = pole strength × ×3= M
−7 when plane is perpendicular, 3
µ 0I 2 × 10 × 18
Bcurrent = =
2 πd 02
. θ = 90°  I /9 
Hence, T ′ = 2 π  
= 18 µ T τ = MB sin 90° = MB [maximum]  MH 
24 01. ×2 1 2
Now, T ′ = T = = 0.076 s Hence, Statement I is true, while or T′= ×T or T ′ = s
42 7 statement II is false. 3 3
4 Let us consider the disc to be made up of
SESSION 2 large number of concentric elementary
18 A 1 As m = IA, so to change dipole moment rings. Consider one such ring of radius x
and thickness dx. Charge on this
(current is kept constant), we have to
elementary ring,
change radius of loop.
µ I
20 cm Initially, m = IπR2 and B1 = 0
2R1 x

17 At the pole, for the combination Finally, m ′ = 2m = IπR22


M1 = 2M + M = 3 M , T1 = 3 s ⇒ 2IπR12 = IπR22 dx
When the polarity of one is reversed,
then or R2 = 2R1 dq = σ × 2 πxdx = 2 πσxdx
M2 = 2M − M = M Current associated with this elementary
µ 0I µ 0I
So, B2 = = ring,
Thus, we have, from 2(R2 ) 2 2R1 dq
T22 M  I  dI = = dq × f = σωxdx
= 1 Q T = 2π   µ 0I  dt
T12 M2  MB    [Q f is frequency and ω = 2 πf ]
B1  2R1 
T22 3M Hence, ratio = = 2 Magnetic moment of this elementary
⇒ = =3 B2  µ 0I 
T12 M   ring,
 2 2R1  dM = d /πx2 = πσωx 3dx
∴ T22 = 3T12 = 3 × 9 = 27
∴ T2 = 27 = 3 3 s ∴ Ratio
B1
= 2 ∴ Magnetic moment of the entire disc,
B2 R
18 B = B H tan θ
M = ∫ 0 dM
2 As, n = 400 = 400
= 1000
µ 0  2M   µ 0 M  2 πR 40 × 10−2 = πσω ∫ x3dx =
R 1
πR 4 σω
  =  tan θ
4 π  d 13   4 π d 23  µ = ni = 1000 × 2 = 2000
0 4
DAY TWENTY TWO MAGNETISM 263

5 P is in the plane S, needle is in North, 1


= =2 a
so the declination is zero. 1 /2

w or φ − 90° = 180°
N
S or φ = 180° + 90° = 270°
11.3° ME P
8 At the neutral point,
B = BH
GE C2 = Square of side a and perimeter L,
µ0 M
⇒ × = BH number of turns per unit length n2 =
L
Q 4 π (r 2 + l 2 )3 /2 4a
S
In CGS system, Magnetic moment of C2
M ⇒ m2 = n2 I A 2
= BH
P is also on the magnetic equator, so the (r 2 + l 2 )3 /2 L
m2 = ⋅ I ⋅ a2
angle of dip = 0, because the value of m × 2l 4a
⇒ = BH LIa
angle of dip at equator is zero Q is also (r 2 + l 2 )3 /2 m2 = …(iii)
4
on the magnetic equator, thus the angle m × 2× 4
⇒ = 032
. Moment of inertia of C2
of dip is zero. As the earth tilted on its (32 + 42 )3 /2
Ma2
axis by 11.3°, thus the declination at Q 8m ⇒ I2 = …(iv)
⇒ = 032
. 12
is 11.3°. (25)3 / 2 Frequency of C1
125 × 032
.
3.6 ⇒ m= I1
6 As, I = m = −4 8 ⇒ f1 = 2 π
A 0.9 × 10 m1 B
= 5 A-cm2
= 4 × 104 Am −1 Frequency of C2
9 I2
HN =
m ⇒ f2 = 2 π
4 πd 2 R m2 B
3.6 According to question,
=
4 π × (6 × 10−2 )2 f1 = f2
= 79.6 Am −1 I1 I2
2π = 2π
H = H N + HS C1 = Circular coil of radius R, m1 B m2 B
⇒ H = H N + H S = 1592 . A / m, length L, number of turns per unit I1 I
towards S pole L = 2
length n1 = m1 m2
B = µ 0(H + I ) = 4 π × 10−7 2 πR m2 I
or = 2
(4 × 104 − 1592
. ) Magnetic moment of C1 m1 I1
−2
= 5 × 10 T, towards N pole ⇒ m1 = n1 I A 1 Putting the values by Eqs. (i), (ii), (iii)
7 As, τ = Cφ = MH sin θ L ⋅ I ⋅ π R2 and (iv), we get
m1 = ,
2π R LIa ⋅ 2 Ma2 ⋅ 2
Case I =
θ = 30° , φ° = 120° − 30° = 90° …(i) LIR 4 × LIR 12 ⋅ MR2
m1 = …(i)
Case II C (φ − 90° ) = MH sin 90° …(ii) 2 3R = a
On dividing Eq. (ii) by Eq. (i), we get Moment of inertia of C1 Thus, the value of a is 3R.
φ − 90° MH sin 90° MR2
= ⇒ I1 = …(ii)
90° MH sin 30° 2
EXAM BITES

This Pdf Is
Downloaded From
www.exambites.in

Visit www.exambites.in for


More Premium Stuffs,Latest
Books,Test Papers,Lectures etc.
jeeneetadda
jeeneetadda_official
jeeneetadda

VISIT NOW !!
DAY TWENTY THREE

Electromagnetic
Induction
Learning & Revision for the Day
u Magnetic Flux ( φB ) u Motional Emf u Mutual Induction
u Faraday’s Law of u Rotational Emf u Combination of Inductors
Electromagnetic Induction u Self-Induction u Eddy Currents

Magnetic Flux ( φB )
ds
The flux associated with a magnetic field is defined in a similar B
manner to that used to define electric flux. Consider an element
θ
of area ds on an arbitrary shaped surface as shown in figure. If
the magnetic field at this element is B, the magnetic flux
through the element is,
dφB = B ⋅ ds = Bds cos θ
Here, ds is a vector that is perpendicular to the surface and has a
magnitude equal to the area ds and θ is the angle between B and ds at that element.
Magnetic flux is a scalar quantity. Outward magnetic flux is taken as positive
(i.e. θ < 90 °) and inward flux is taken as negative (i.e. θ > 90 °).
SI unit of magnetic flux is 1 weber (1 Wb).
where, 1 Wb = 1 T × 1 m2 = 1 T-m2
Dimensional formula of magnetic flux is [ML2 T−2 A−1 ] . PREP
MIRROR
Your Personal Preparation Indicator
Faraday’s Law of Electromagnetic Induction
u No. of Questions in Exercises (x)—
This law states that, the induced emf in a closed loop equals the negative of the time rate
of change of magnetic flux through the loop.
u No. of Questions Attempted (y)—
No. of Correct Questions (z)—
dφ B
u

Induced emf, |e | = (Without referring Explanations)


dt
dφ B Accuracy Level (z / y × 100)—
|e | = N
u
l
For N turns,
dt u Prep Level (z / x × 100)—
However, if we consider the direction of induced emf, then
In order to expect good rank in JEE,
dφ B Nd(BA cos θ) − NBA(cos θ2 − cos θ 1) your Accuracy Level should be above
e=−N =− = 85 & Prep Level should be above 75.
dt dt ∆t
DAY TWENTY THREE ELECTROMAGNETIC INDUCTION 265

If the given electric circuit is a closed circuit having a total


Rotational Emf
l

resistance R, then the induced current,


e N dφ B Let a conducting rod of length l rotate about an axis passing
I = =− through one of its ends (that end may be fixed), with an angular
R R dt
velocity ω in a plane perpendicular to the magnetic field B, then
N
Induced charge, dq = Idt = − dφ B an induced emf is set up between the ends of the rod, whose
R magnitude is given by
2
e2 N 2  dφ B 
and induced power, P = =   |e | =
1 2
Bl ω
R R  dt  2

Lenz’s Law Self-Induction


The negative sign in Faraday’s equations of electromagnetic Self-induction is the phenomenon due to which an induced
induction describes the direction in which the induced emf emf is set up in a coil or a circuit whenever the current
drives current around a circuit. However, that direction is passing through it changes. The induced emf opposes the
most easily determined with the help of Lenz’s law. This law change that causes it and is thus known as back emf.
states that the direction of any magnetic induction effect is Induced current
such as to oppose the cause of the effect. (make)
Later, we will see that Lenz’s law is directly related to energy
Induced current +
conservation. A
(break) –

Motional Emf + –
Let a conducting rod of length l be moving with a uniform
K Rh
velocity v perpendicular to a uniform magnetic field B, an
induced emf is set up. l
Inductance is the inherent property of electrical circuits
X X X X + X X X X and is known as the electrical inertia.
l X X X X v X X X X
l
An inductor is said to be an ideal inductor if its resistance
is zero.
X X X X – X X X X
l
An inductor does not oppose current but opposes changes
The magnitude of the induced emf will be (growth or decay of current) in the circuit.
|e | = B l v
l
If the rod is moving such that it makes an angle θ with the Self-Inductance
direction of the magnetic field, then
Flux linked with the coil is
| e| = B l v sin θ
Nφ B ∝ I or Nφ B = LI,
Hence, for the motion parallel to B, the induced emf is zero.
where the constant L is known as the coefficient of
l
When a conducting rod moves horizontally, then an
self-induction or self-inductance of the given coil.
induced emf is set up between its ends due to the vertical
component of the earth’s magnetic field. However, at the It may be defined as the magnetic flux linked with the coil,
magnetic equator, induced emf will be zero, because when a constant current of 1 A is passed through it.
B V = 0. Induced emf due to self-induction,
If during landing or taking off, the wings of an aeroplane dφ dI
e=–N =–L
l

are along the East-West direction, an induced emf is set up dt dt


across the wings (due to the effect of BH ). SI unit of inductance is henry.

Motional Emf in a Loop Magnetic Potential Energy of an Inductor


l
In building, a steady current in an electric circuit, some
If a conducting rod moves on two parallel conducting rails,
work is done by the emf of the source, against the
then an emf is induced whose magnitude is|e | = B l v
self-inductance of the coil.
and the direction is given by the Fleming’s right hand rule.
1
|e | B l v The work done, W = LI 2
l
Induced current,| I | = = 2
R R l
The work done is stored as the magnetic potential energy of
B2 l2 v that inductor.
l
Magnetic force, Fm = BIl =
R 1
Thus, U = L I2
2
266 40 DAYS ~ JEE MAIN PHYSICS DAY TWENTY THREE

Formulae for Self-Inductance For a pair of two magnetically coupled coils of self-inductances
L1 and L2 respectively, the mutual inductance,
l
For a circular coil of radius R and N turns, the
self-inductance, M 12 = M21 = M = k L1L2
1 where, k is the coupling coefficient.
L = µ0 π N2 R
2
l
For a solenoid coil having length l, total number of turns N Formulae for Mutual Inductance
and cross-sectional area A, l
Assuming the coupling coefficient k = 1 and medium to be
µ0 N2 A  N a free space or air. Mutual inductance of a pair of concentric
L= = µ 0 n Al
2
where, n = l 
l circular coils is
For a toroid of radius R and number of turns N, µ 0N 1N2 πr 2
M=
l

1 2R
L = µ0 N2 R
2 where, r = radius of the coil (of small radius)
l
For a square coil of side a and number of turns N, and R = radius of the coil (of larger radius).
2 2 l
For a pair of two solenoid coils, wound one over the
L= µ0 N2 a µ N N A
π other, M= 0 1 2
l
For a pair of concentric coplanar square coils,
Mutual Induction 2 2 µ 0N 1N2 a2
Mutual induction is the phenomenon due to which an emf is M=
πb
induced in a coil when the current flowing through a
neighbouring coil changes. where, a = side of the smaller coil and b = side of the larger
coil.
For a given pair of coils, mutually coupled, then according
Mutual Inductance
l

to theorem of reciprocity,
Mutual inductance of a pair of coils is defined as the magnetic M 12 = M21 = M
flux linked with one coil, when a constant current of unit
magnitude, flows through the other coil.
Mathematically, NφB2 = MI 1 Combination of Inductors
where, M is known as the mutual inductance for the given l
If two coils of self-inductances L 1 and L 2 are placed quite
pair of coils. far apart and are arranged in series, then their equivalent
Induced emf due to mutual inductance, inductance,
dφB2 dI L s = L1 + L 2
e2 = – N =–M 1
dt dt l
If the coils are placed quite close to each other, so as to
Hence, mutual inductance for a pair of coils is numerically mutually affect each other, then their equivalent inductance,
equal to the magnitude of induced emf in one coil when L s = L1 + L2 ± 2 M
current in the other coil changes at a rate of 1 As −1. Here, M has been written with ± sign depending on the fact
SI unit of mutual inductance M, is henry. whether currents in the two coils are flowing in same sense
or opposite sense.
Mutual inductance of a pair of coils is maximum, when the
two coils are wound on the same frame. However, mutual
l
If two coils of self-inductances L1 and L 2 are connected in
inductance is negligible when the two coils are oriented parallel, then equivalent inductance L p is given by
mutually perpendicular to each other (see figure). In this 1 1 1 L1L 2
= + ⇒ Lp =
context, we define a term coupling coefficient k. L p L1 L 2 L1 + L 2

S
S
Eddy Currents
P S Currents induced in the body of bulk of the conductors due to
P
change in magnetic flux linked to them, are called the eddy
P currents. The production of eddy currents in a metallic
(a) (b) (c)
conductor leads to a loss of electric energy in the form of
Coupling coefficient is given by heat energy.
Magnetic flux linked with secondary coil Eddy currents can be minimised by taking the metal
k =
Magnetic flux developed in primary coil (generally soft iron) core in the form of a combination of thin
laminated sheets or by slotting process.
It is observed that 0 ≤ k ≤ 1.
DAY TWENTY THREE ELECTROMAGNETIC INDUCTION 267

DAY PRACTICE SESSION 1

FOUNDATION QUESTIONS EXERCISE


ω
1 The magnetic field in a certain region is given by 7 Coil A is made to rotate about a A B
B = ( 40 $i − 18 k$ ) gauss. How much flux passes through a vertical axis (figure). No current
loop of area 5 cm 2, in this region, if the loop lies flat on flows in B, if A is at rest. The
the xy-plane? current in coil A, when the current
(a) – 900 × 10−9 Wb (b) 900 × 10−9 Wb in B (at t = 0) is counter clockwise
(c) Zero (d) 9 Wb and the coil A is as shown at this instant, (t = 0), is
(a) constant current clockwise
2 The flux linked with a coil at any instant t is given by
(b) varying current clockwise
φ = 10 t − 50 t + 250. The induced emf at t = 3 s is
2
(c) varying current counter clockwise
(a) –190 V (b) –10 V (c) 10 V (d) 190 V (d) constant current counter clockwise
3 A coil having n turns and resistance R Ω is connected 8 One conducting U-tube can slide A B
with a galvanometer of resistance 4R Ω. This inside another as shown in figure, v
combination is moved for time t seconds from a maintaining electrical contacts v
magnetic field W1 weber to W2 weber. The induced between the tubes. The magnetic D C
current in the circuit is field B is perpendicular to the
W2 − W1 n (W2 − W1) plane of the figure. If each tube moves towards the other
(a) (b) −
5 Rnt 5 Rt at a constant speed v, then the emf induced in the circuit
(W − W1) n (W2 − W1)
(c) − 2 (d) − in terms of B, l and v , where l is the width of each tube, will
Rnt Rt be
4 In a coil of resistance 100 Ω, a current is induced by (a) Blv (b) −Blv
changing the magnetic flux through it as shown in the (c) zero (d) 2Blv
figure. The magnitude of change in flux through the coil is 9 A boat is moving due to East in a region where the earth’s
ª JEE Main 2017 (Offline) magnetic field is 5 . 0 × 10−5 NA −1m −1 due to North and
horizontal. The boat carries a vertical aerial 2 m long.If the
10 speed of the boat is 1.50 ms −1, the magnitude of the
Current induced emf in the wire of aerial is ª AIEEE 2011
(A) (a) 1mV (b) 0.75 mV
(c) 0.50 mV (d) 0.15 mV
10 A helicopter rises vertically with a speed of 10 ms −1.
Time (s) 0.5 If helicopter has a length of 10 m and the
horizontal component of the earth’s magnetic field is
(a) 225 Wb (b) 250 Wb (c) 275 Wb (d) 200 Wb
1.5 × 10 −3 Wbm −2 , the emf induced between the tip of the
2
5 A coil has an area of 0.05 m and has 800 turns. After nose and the tail of the helicopter, is
placing the coil in a magnetic field of strength (a) 0.15 V (b) 125 V
4 × 10−5 Wb m −2, perpendicular to the field, the coil is (c) 130 V (d) 5 V
rotated by 90° in 0.1 s. The average emf induced is A
11 The magnetic field as shown in the
(a) zero (b) 0.016 V (c) 0.01 V (d) 0.032 V w
figure is directed into the plane of
6 A cylindrical bar magnet is rotated about its axis. A wire B
paper. A X C A is a semi-circular X O
is connected from the axis and is made to touch the conducting loop of radius a with
cylindrical suface through a contact. Then, centre O. The loop rotates clockwise
(a) a direct current flows in the ammeter A with velocity ω about an axis fixed at C
(b) no current flows through the ammeter A O and perpendicular to the plane of
(c) an alternating sinusoidal current flows through the the paper. The resistance of the loop is R. The induced

ammeter A with a time periodT = current is
ω
ωr 2 B ωr 2 − 2R 2R
(d) a time varying non- sinusoidal current flows through the (a) (b) − (c) (d)
2R 2R B ωr ωr 2
ammeter A
268 40 DAYS ~ JEE MAIN PHYSICS DAY TWENTY THREE

12 A metal rod of resistance 20 Ω is fixed along the 16 An inductor (L = 0.03 H) and a resistor (R = 015
. kΩ ) are
diameter of a conducting ring of radius 0.1 m and lies connected in series to a battery of 15V emf in a circuit
on the xy-plane. There is a magnetic field B = | 50 k|. The shown below. The key K1 has been kept closed for a long
time. Then at t = 0, K1 is opened and key K 2 is closed
ring rotates with an angular velocity ω = 20 rads −1 about
simultaneously. At t = 1ms, the current in the circuit will be
its axis. An external resistance of 10 Ω is connected
(e 5 ~
= 150) ª JEE Main 2015
across the centre of the ring and the rim. The current
0.03H 0.15 kW
through the external resistance is
1 1 1
(a) A (b) A (c) A (d) zero K2
2 3 4
13 Two coils, x and y are kept in close vicinity of each other. K1
When a varying current, I(t) flows through coil x, the
induced emf [ V(t)] in coil Y, varies in the manner shown 15V
here. The variation if I(t ), with time can then be (a) 100 mA (b) 67 mA (c) 6.7 mA (d) 0.67mA
represented by the graph labelled as graph. 17 An inductor coil stores 32J of magnetic energy and
ª JEE Main (Online) 2013 dissipates it as heat at the rate of 320 W when a current
of 4 A is passed through it. The time constant of the
circuit is
(a) I(t) (b) (a) 0.2 s (b) 0.1 s (c) 0.3 s (d) 0.4 s
I(t)

t t
18 In series R -L circuit, switch is closed at t = 0. The charge
which passes through the battery in one time constant is
R L

(d) V0
(c)
I(t) I(t)
t t
V0 e V0 e Rτ V0 τ
(a) (b) (c) (d)
Rτ Rt V0 e Re
14 In the circuit shown here, the point C is kept connected to 19 An uniformly wound solenoid of inductance1.8 × 10−4H
point A till the current flowing through the circuit becomes and resistance 6 Ω is broken into two identical parts.
constant. Afterward, suddenly point C is disconnected
These identical coils are then connected in parallel
from point A and connected to point B at time t = 0. Ratio
of the voltage across resistance and the inductor at across a 15 V battery of negligible resistance. The time
t = L / R will be equal to ª JEE Main 2014 constant of the circuit is
(a) 3 × 10−5 s (b) 6 × 10−5 s . × 10−5 s (d) 1.8 × 10−5 s
(c) 15
A C R
20 A uniformly wound solenoidal coil of self-inductance
1.8 × 10−4 H and a resistance of 6 Ω is broken up into two
L identical coils. These identical coils are then connected in
B parallel across a 120 V battery of negligible resistance.
The time constant of the current in the circuit and the
e 1− e
(a) (b) 1 (c) − 1 (d) steady state current through the battery is
1− e e
(a) 3 × 10−5 s, 8 A (b) 1.5 × 10−5 s, 8 A
15 An inductor (L = 100 mH), a resistor (R = 100 Ω ) and a (c) 0.75 × 10−4 s, 4 A (d) 6 × 10−5 s, 2 A
battery (E = 100 V ) are initially connected in series as 21 A coil is suspended in a uniform magnetic field with the
shown in the figure. After a long time, the battery is plane of the coil parallel to the magnetic lines of force. When
disconnected after short circuiting the points A and B. The
a current is passed through the coil, it starts oscillating; it is
current in the circuit 1 millisecond after the short circuit is
L very difficult to stop. But if an aluminium plate is placed near
to the coil, it stops. This is due to ª AIEEE 2011

R (a) development of air current when the plate is placed


A B (b) induction of electrical charge on the plate
(c) shielding of magnetic lines of force as aluminium is a
E paramagnetic material
(a) 1/e A (b) e A (c) 0.1 A (d) 1 A
(d) electromagnetic induction in the aluminium plate giving
rise to electromagnetic damping
DAY TWENTY THREE ELECTROMAGNETIC INDUCTION 269

Direction (Q. Nos. 22-26) Each of these questions 24 Statement I An artificial satellite with a metal surface, is
contains two statements : Statement I and Statement II. moving about the earth in a circular orbit. A current is
Each of these questions also has four alternative choices, only induced when the plane of the orbit is inclined to the
one of which is the correct answer. You have to select one of plane of the equator.
the codes (a), (b), (c), (d) given below
Statement II The satellite cuts the magnetic field of earth.
(a) Statement I is true, Statement II is true; Statement II is the
correct explanation for Statement I 25 Statement I A coil A is connected to a voltmeterV and
(b) Statement I is true, Statement II is true; Statement II is not the other coil B is connected to an alternating current
the correct explanation for Statement I source. If a large copper sheet C is placed between the
(c) Statement I is true; Statement II is false two coils, the induced emf in the coil A is reduced.
C
(d) Statement I is false; Statement II is true A B
22 Statement I The mutual inductance of two coils is
doubled if the self-inductance of the primary and the
V
secondary coil is doubled.
Statement II Mutual inductance is proportional to the Statement II Copper sheet between the coils, has no
square root of self-inductance of primary and secondary effect on the induced emf in coil A.
coils. 26 Statement I When a DC current is made to flow in a soft
23 Statement I The energy stored in the inductor of 2 H, wire loop of arbitrary shape, it tend to acquire a circular
when a current of 10 A flows through it, is 100 J. shape.
Statement II Energy stored in an inductor is directly Statement II Flux linked with a wire loop is maximum
proportional to its inductance. when loop is a circle.

DAY PRACTICE SESSION 2

PROGRESSIVE QUESTIONS EXERCISE


1 The inductance per unit length of a double tape line as 4 A circular coil of radius 8.0 cm and 20 turns is rotated
shown in the figure. about its vertical diameter with an angular speed of
50 rad/s in a uniform horizontal magnetic field of
h magnitude 3.0 ×10−2 T. Obtain the maximum and
average emf induced in the coil. If the coil forms a
d
I closed-loop of resistance 10 Ω, calculate the maximum
b value of current in the coil. Calculate the average power
loss due to Joule heating.
I
(a) 0.012 W (b) 0.1 W
µ h b µ0b hb
(a) 0 (b) (c) (d) (c) 0.018 W (d) 0.42 W
b µ0h h µ0
5 A long straight solenoid with cross-sectional radius a and
2 An air-cored solenoid with length 30 cm, area of number of turns per unit length n has a current varying
cross-section 25 cm 2 and number of turns 500, carries a with time as I As −1. The magnitude of the eddy current
current of 2.5 A. The current is suddenly switched off in a as a function of distance r from the solenoid axis is
brief time of 10−3 s. How much is the average back emf − nµ 0a 2 I µ In − na 2 I µ 0I a
induced across the ends of the open switch in the (a) (b) 0 (c) (d)
2r 2a 2µ 0 r 2n
circuit? Ignore the variation in magnetic field near the
ends of the solenoid. 6 A circular loop of radius 0.3 cm lies parallel to a much
(a) 6.5 V (b) 7.4 V (c) 8.2 V (d) 9.3 V bigger circular loop of radius 20 cm. The centre of the
3 A short circuited coil is placed in a time varying magnetic small loop is on the axis of the bigger loop. The distance
field. Electrical power is dissipated due to the current between their centres is 15 cm. If a current of 2.0 A flows
induced in the coil. If the number of turns are quadrupled through the smaller loop, then the flux linked with bigger
and the wire radius is halved, the electrical power loop is
dissipated in the coil, would be (a) 9.2 × 10−11 Wb (b) 6 × 10−11 Wb
(a) halved (b) the same (c) doubled (d) quadrupled (c) 3.3 × 10−11 Wb (d) 6.6 × 10−9 Wb
270 40 DAYS ~ JEE MAIN PHYSICS DAY TWENTY THREE

7 A metallic rod of length l is tied to a string of length 2l and


A
made to rotate with angular speed ω on a horizontal table
L
with one end of the string fixed.If there is a vertical 12 V R1
magnetic field B in the region, the emf induced across
the ends of the rod is
R2
S

12 −3t
(a) 6e −5t V (b) e V
t
(c) 6(1 − e −t / 0. 2 )V (d) 12 e −5t V
11 In a uniform magnetic field of induction B, a wire in the
form of semi-circle of radius r rotates about the diameter
of the circle with angular frequency ω. If the total
resistance of the circuit is R, the mean power generated
2Bωl3 3Bωl3 4Bωl2 5Bωl2 per period of rotation is
(a) (b) (c) (d)
2 2 2 2 B πr 2ω (B πr 2ω)2
(a) (b)
2R 8R
8 A rectangular loop has a sliding connector PQ of length l
(B πrω)2 (B πrω2 )2
and resistance R Ω and it is moving with a speed v as (c) (d)
2R 8R
shown. The setup is placed in a uniform magnetic field
going into the plane of the paper. The three currents I1, I 2 12 The inductance between A and D is
and I are
P l
A D
3H 3H 3H

v
RW RW RW (a) 3.66 H (b) 9 H
(c) 0.66 H (d) 1 H
I I2
13 A loop, made of straight edges has six corners at
I1 Q A ( 0, 0, 0), B (L, 0, 0), C (L, L, 0), D ( 0, L, 0), E ( 0, L, L ) and
B lv 2B l v F ( 0,0, L ). A magnetic field B = B0( $i + k$ ) T is present in the
(a) I1 = − I 2 = ,I =
R R region. The flux passing through the loop ABCDEFA (in
B lv 2B l v that order) is
(b) I1 = I 2 = ,I =
3R 3R (a) B 0L2 Wb (b) 2B 0L2 Wb
B lv
(c) I1 = I 2 = I = (c) 2B 0L2 Wb (d) 4B 0L2 Wb
R
B lv Blv 14 Magnetic flux in a circular coil of resistance10 Ω changes
(d) I1 = I 2 = ,I =
6R 3R with time as shown in figure. Cross indicates a direction
perpendicular to paper inwards. Match the following.
9 An ideal coil of 10 H is connected in series with a
f
resistance of 5 Ω and a battery of 5 V. 2s after the
connection is made, the current flowing (in ampere) in the 10
circuit is
(a) (1− e) (b) e (c) e −1 (d) (1 − e −1)
2 6 8 10 14 16 t(s)
10 An inductor of inductance L = 400 mH and resistors of
resistances R1 = 4 Ω and R 2 = 2 Ω are connected to –10
battery of emf 12 V as shown in the figure. The internal
resistance of the battery is negligible. The switch S is
closed at t = 0. The potential drop across L as a function
of time is ª AIEEE 2009
DAY TWENTY THREE ELECTROMAGNETIC INDUCTION 271

Column I Column II Column I Column II


A. at t = 1 s 1. emf induced is zero A. Loop is moved towards 1. Induced current in the loop is
B. at t = 5 s 2. emf induced in anti-clockwise direction right clockwise
C. at t = 9 s 3. emf induced in clockwise direction B. Loop is moved towards 2. Induced current in the loop is
D. at t = 15 s 4. 2 A left anti-clockwise
C. Wire-1 is moved towards 3. Induced current in the loop is
Codes left zero
A B C D A B C D D. Wire-2 is moved towards 4. Induced current in the loop is
(a) 2 1 1 2 (b) 1 2 3 4 right non-zero
(c) 4 3 2 1 (d) 2 4 1 3
Codes
15 A square loop is symmetrically placed 1 2
A B C D A B C D
between two infinitely long current carrying
(a) 1 2 1 2 (b) 1 3 2 4
wires in the same direction. Magnitude of
(c) 1 2 3 4 (d) 4 2 2 4
currents in both the wires are same.Now
match the following two columns.

ANSWERS

SESSION 1 1 (a) 2 (b) 3 (b) 4 (b) 5 (b) 6 (b) 7 (b) 8 (d) 9 (d) 10 (a)
11 (b) 12 (b) 13 (c) 14 (c) 15 (a) 16 (d) 17 (a) 18 (d) 19 (a) 20 (a)
21 (d) 22 (a) 23 (a) 24 (a) 25 (c) 26 (b)

SESSION 2 1 (a) 2 (a) 3 (b) 4 (c) 5 (a) 6 (a) 7 (d) 8 (b) 9 (d) 10 (d)
11 (b) 12 (d) 13 (b) 14 (a) 15 (a)

Hints and Explanations


SESSION 1 ∴ Induced current, ∴ Here, R is constant
I =
e
=−
n dφ ∴ φ = R ∫ Idt
1 As loop is in the xy -plane, only the …(i)
R′ R ′ dt
z -component of the magnetic field, is
Given, R ′ = R + 4R = 5R,
∫ I ⋅ dt = Area under I - t graph
effective. 1
B = − 18 gauss = −18 × 10−4 T dφ = W2 − W1 ,dt = t =× 10 × 0.5 = 2.5
2
A = 5 × 10−4 m2 [here, W1 and W2 are flux ∴ φ = R × 2.5 = 100 × 2.5
−4 −4 associated with one turn] = 250 Wb
φ = B A cos 0° = −18 × 10 × 5 × 10
Putting the given values in Eq. (i), we
= −90 × 10−8 Wb 5 As from Faraday’s rule,
get
−9
= − 900 × 10 Wb n (W2 − W1 ) dφ NAdB
I =− e =N =
5R t dt dt
2 φ = 10t 2 − 50t + 250 4 × 10−5
= 800 × 0.05 ×
From Faraday’s law of electromagnetic 4 Induced constant, I = e .
01
induction, R = 0.016 V
e = − dφ/ dt dφ
Here, e = induced emf = 6 When cylindrical bar magnet is rotated
∴ e = − [10 × 2t − 50] dt
about its axis,no change in flux linked
∴e | t = 3 s = − [10 × 6 − 50] = − 10 V e  dφ  1
I = =  ⋅ with the circuit takes place, consequently
R  dt  R
3 The rate of change of flux or emf no emf induces and hence,no current flows
dφ dφ = IRdt through the ammeter A.
induced in the coil is e = − n . ⇒ φ = ∫ IRdt
dt
272 40 DAYS ~ JEE MAIN PHYSICS DAY TWENTY THREE

1 E 100
V = Bωr 2 I0 = = =1A
Axis 2 R 100
A 1 [as inductor would be shorted in steady
N = × 50 × 20 × (01
. )2 = 5 V
2 state]
After this decay of current starts in the
Bar The equivalent circuit of the
w circuit, according to the equation,
magnet arrangement is shown in figures.
I = I 0e − t / τ
A
5V 10 W where, τ = L /R
10 W 10W L
5V 10 W

S R
10 W 10 W

7 When the current in B(at t = 0) is B 5V


=   A
counter clockwise and the coil A is 5W −3
)/(100 × 10−3 /100) 1
I = 1 × e − (1 × 10
considered above to it. The counter e 
clockwise flow of the current in B is [Q t = 1 millisecond = 1 × 10−3 s
equivalent to North pole of magnet and
10 W and L = 100 × 10−3 H]
magnetic field lines are emerging
1 1 1 2 1
upward to coil A. When coil A start = + ⇒ = ⇒ Rp = 5Ω 16 After long time inductor behaves as
rotating at t = 0, the current in A is time R p 10 10 10 5
short-circuit.
varying along clockwise direction by Current through the external resistance, At t = 0, the inductor behaves as
Lenz’s rule. E 5 1 short-circuited. The current
I = = = A E 15V
8 Relative velocity = v − (− v ) = 2v = dl R + r 10 + 5 3 I0 = 0 = = 100 mA
dt R . kΩ
015
dφ Bl dl 13 Firstly, the current decreases due to As K 2 is closed, current through the
Now, e = ⇒e = [Q φ = BA]
dt dt electrical inertia goes to zero, but due to inductor starts decay, which is given at
Q dl = 2 v  back emf induced in the easily, the any time t as
Induced emf, e = 2 B l v
 dt  induced current in the coil decreases off − tR − t × 15000
a point when back emf is equal to the I = I0 e L = (100 mA) e 3

9 E ind = B × v × l = 5.0 × 10−5 × 1. 50 × 2 applied emf induced in the another coil.


At t = 1 ms
The value of emf of two current is 1 ×10−3 ×15×103
= 10. 0 × 10−5 × 1. 5 zero.Then, current is regularly I = (100 mA ) e

3
= 15 × 10−5 = 015
. mV increased, after that time became it is
I = (100 mA ) e − 5 = 0.6737 mA
continuously by supplied by the source
10 e = Bl v = B H l v (variable). or I = 0.67 mA
= 1.5 × 10−3 × 10 × 10 = 015
. V 14 After connecting C to B hanging the LI2 L(4)2
17 U = or 32 = = L = 4H
π r 2 θ r 2 ωt r 2 switch, the circuit will act like L-R 2 2
11 A = θ = = discharging circuit. 320
π 2 2 2 P = I 2 R, R = 2 = 20 Ω
(4)
R
L 4
θ ∴ τ = = = 0.2 s
B R 20
L
18 I = V 0 (1 − e − t / τ )
R
τV
Q = ∫ I dt = ∫ 0 (1 − e − t / τ ) dt
ωt r 2 Applying Kirchhoff’s loop equation, 0 R
Flux, φ = BA = B
VR + VL = 0 ⇒ VR = − VL V V V τ
2 ⇒ Q = 0 τ + τ (e −1 − 1) 0 = 0
dφ B ω r2 VR R R Re
Emf, e = − =− ∴ = −1
dt 2 VL
19 Inductance of each part,
− B ω r2 15 This is a combined example of growth L1 = L2 =
L
= 0.9 × 10−4 H
I =
2R and decay of current in an L-R circuit. 2
ω tr 2 L=100 mH Resistance of each part,
After half rotation A(t ) = πr 2 − R
2 R1 = R2 = = 3Ω
will give same current but in opposite 2
direction. R=100W Time constant,
L ||L L1 L2 R + R2
A B τ = 1 2 = × 1
12 Here, resistance of rod = 20 Ω, R1||R2 L1 + L2 R1 R2
r = 01
. m, B = 50 T, acting along the E=100 V 1.8 × 10−4 × 0.9 × 10−4 6+ 3
z-axis and ω = 20 rad s −1 . τ = ×
The current through circuit just before 1.8 × 10−4 + 0.9 × 10−4 6×3
Potential difference between the centre shorting the battery,
1.62 × 10−4
of the ring and the rim is = . × 10−4 = 3 × 10−5s
= 03
54
.
DAY TWENTY THREE ELECTROMAGNETIC INDUCTION 273

2 2 4
20 Since, the self-inductance in parallel is due to the alternating current through l1  r2  P n  r 
B. Thus, induced current will be or =   ⇒ 2 =  2 × 2
given by l2  r1  P1  n1   r1 
1 1 1 2 L reduced.
= + = ⇒ L p =  n2 r 1
Lp L L L 2 26 Each section of wire repels Given, = 4 and 2 = 
diametrically opposite section as  n1 r1 2
1.8 × 10−14
= 0.9 × 10−4 H
4
and L =
= (4)2 ×   = 16 ×
current flows in opposite direction. P2 1 1
2 So, =1
P1  2 16
∴ L p = 0.45 × 10−4 H SESSION 2
4 Average induced emf
Resistance of each part, r = 6 / 2 = 3 Ω 1 Neglecting end effects of magnetic field, 1 2π 1 π

Now,
1 1 1 2
= + =
we have e av =
T ∫0 e dt =
T ∫0 NBA ω sin ω t dt
µ I
rp 3 3 3 B = 0 2π
b 1  cos ω t 
e av = ⋅ NAB ω 
 ω  0
3
∴ rp = Ω Flux φ per unit length of the plates is T
2 µ 0I µ hI
× h×1= 0 NBA
So, the time constant of the circuit is = [cos 2 π − cos 0° ]
b b T
given by µ h
L p 0.45 × 10−4 Also, φ = L I ⇒ L = 0 NBA
e av = [1 − 1] = 0
τ = = = 3 × 10−5 s b T
rp 3/2
2 Given, length of solenoid l = 30 cm For full cycle average emf, e av = 0
and the steady current is, = 30 × 10−2 m Average power loss due to heating
V 12
I = = = 8A Area of cross-section A = 25cm2 E I 0.603 × 0.0603
rp 3/2 = 0 0 = = 0.018 W
= 25× 10−4 m2 2 2
21 According to Lenz’s law, Number of turns N = 500 5 B = n µ 0 I and ∫ E ⋅ dl = − dφ
electromagnetic induction takes place Current I1 = 2.5 A, I2 = 0 dt
in the aluminium plate for which eddy Brief time dt = 10−3 s For r < a, E (2 π r ) = − π r 2 nµ 0I
current is developed. This causes loss in Induced emf in the solenoid nµ 0 I r
energy which results in damping of dφ d or E = − (for r < a)
e = = (BA) (Q φ = BA ) 2
oscillatory motion of the coil. dt dt dI
where, I = . (for r > a)
22 If two coils of inductances L 1 and L 2 Magnetic field induction B at a point dt
well inside the long solenoid carrying
are joined together, then their mutual − n µ 0 a2 I
current I is E (2 πr ) = − π a2 n µ 0 I ⇒ E =
inductance is given by B = µ 0nI 2r
M = k L1 L2  where, n = Number of turns  6 Mutual inductance of two coils,
 N 
It is clear from the relation, if the  per unit length =  µ 0R12 πR22
 l  M =
self-inductance of the primary and the 2(R12 × x2 )3 /2
dB d  N 
secondary coil is doubled, the mutual e = NA = A µ
 0 I 
inductance of the coils, will also be dt dt  l  Flux through the bigger coil,
doubled. µ N dl µ 0 π2 R12 R22
= A 0 ⋅ M = ⋅
23 The energy stored in the inductor is l dt 4 π (R12 + x2 )
given by e = 500 × 25 × 10−4 × 4 × 314
. × 10−7
1 1 500 2.5 Substituting the values
U = L I 20= × 2 × (10) 2 = 100 J × ×
2 2 30 × 10−2 10−3 µ 0(2)(20 × 10−2 )2
M = × π(0.3 × 10−2 )2
It is obvious that energy stored in the = 6.5 V 2[(0.2)2 + (015
. )2 ]
inductor, is directly proportional to its 3 The magnitude of the induced voltage is On solving,
inductance. proportional to the rate of change of
magnetic flux which, in turns depends M = 9.216 × 10−11
24 It is concept of eddy current losses.
on the number of turns in the coil, i.e. = 9.216 × 10−11 ≈ 9.2 × 10−11 Wb
25 In the absence of the copper sheet, V ∝ n.
induced emf will be produced in the 7 Consider a small element of length dx as
So, resistance of a wire is given by
coil A due to the mutual induction ρl shown below emf induced due to whole
between the coils A and B. As a result, R= [ A = π r2] rod
π r2
voltmeter will show deflection 3l [(3 l)2 − (2l)2 ]
l e = ∫ (ωx )Bdx = Bω
depending on the magnitude of the i.e. R ∝ 2 2l 2
r
induced emf. 5Bl2ω
[ ρ is a resistivity of a wire] =
When the copper sheet is placed 2
V2 n2 (nr )2
between the two coils, eddy currents ∴ P = ∝ ⇒ P =
R l /r 2
l I
will be setup in the coil. Since, the eddy w 2I
2 2
currents have an opposing effect, the P2  n2  r  l 
magnetic flux linked with A due to eddy ∴ =   × 2 × 1
P1  n1   r1   l2  x dx
current will always be opposite to that
274 40 DAYS ~ JEE MAIN PHYSICS DAY TWENTY THREE

8 A moving conductor is equivalent to a Q Potential drop = E − I2 R2 Here, inductors are in parallel.


battery of emf = vBl (motion emf) I2 = I 0(1 − e − T / tc ) ∴
1 1 1 1
= + +
Equivalent circuit, I = I1 + I2 [current as a function of time] L 3 3 3
E 12 or L = 1H
P ⇒ I0 = = = 6A
R2 2
13 Also, the magnetic flux linked with
L 400 × 10−3 uniform surface of area A in uniform
and tc = = = 0.2
R2 2 magnetic field is given by
R R R
I2 = 6(1 − e − t / 02
.
) Y
⇒ I2 = 6(1 − e −5t )
Q Potential drop across D C
I1 I2
L = E − R2 I2
E
Applying Kirchhoff’s law, = 12 − 2 × 6(1 − e −5 t )
I1 R + I R − v B l = 0 X
...(i) = 12e −5 t V A B
I2 R + I R − v B l = 0 ...(ii) F
Adding Eqs. (i) and (ii), we get 11 The flux associated with coil of area A Z
and magnetic induction B is
2 IR + IR = 2v B l φ = B⋅A
1
2v B l φ = BA cos θ = Bπr 2 cos ωt
I = 2 A = A1 + A2
3R = (L2 k$ + L2 i$ )
Q A = 1 πr 2 
Blv  2 
I1 = I2 = and B = B 0($i + k$ ) T
3R dφ
∴ e induced = − Now,
dt
9 Rise of current in L - R circuit is given by d 1  φ = B⋅ A
=−  Bπr cos ωt 
2

I = I 0(1 − e − t / τ ) dt  2  = B ($i + k$ ).(L2 k$ + L2 $i )


0

E 5 1 = 2 B 0 L2 Wb
where, I0 = = = 1A = Bπr ω sin ωt
2

R 5 2
e2 14 A → 2, B → 1, C → 1, D → 2
10 H 5Ω ∴ Power, P = induced [Q P = V 2 / R]
R A. At t = 1 s, flux is increasing in the
B 2 π2 r 4ω2 sin2 ωt inward direction, hence induced emf will
= be in anti-clockwise direction.
4R
B. At t = 5s, there is no change in
Hence, Pmean = < P > flux,so induced emf is zero.
B 2 π2 r 4ω2 1 C. At t = 9 s, flux is constant, hence
= ⋅
4R 2 induced emf will be zero.
5V
Q < sin2 ωt > = 1  D. At t = 15s, flux is decreasing in
L 10  2  upward direction, so induced emf will be
Now, τ = = = 2s
R 5 (Bπr 2ω )2 in anti-clockwise direction.
After 2s, i.e. at t = 2s =
8R 15 A - 1, B - 2, C - 1, D - 2
Rise of current I = (1 − e −1 ) A When loop is moved towards right,
12
upward flux increases, so current in loop
10 I1 = E = 12 = 3 A E F is clockwise. When loop is moved
R1 4 A D
B 3H 3H C 3H towards left, downward flux increases, so
I
current in loop is anti-clockwise. When
I1 I2
A F D wire 1 is moved left, upward flux
L 3W through loop increases, so current is
R1
3W clockwise. When wire 2 is moved right,
E C downward flux increases, so current is
R2 anti-clockwise.
S 3W
A B
DAY TWENTY FOUR

Alternating
Current
Learning & Revision for the Day
u Peak and RMS Values of u Series AC Circuits u AC Generator
Alternating Current/Voltage u Power in an AC Circuit u Transformer
u Different Types of AC Circuits

An alternating current is the current (or voltage) whose magnitude keeps on changing
continuously with time, between zero and a maximum value and its direction also
reverses periodically.

VI
I0
V0 t
0
T T 3T T
4 2 4

Peak and RMS Values of


Alternating Current/ Voltage
1 V
PREP
RMS value of alternating voltage is equal to times of peak value. i.e. Vrms = 0 MIRROR
2 2
I0 Your Personal Preparation Indicator
Similarly, I rms =
2 u No. of Questions in Exercises (x)—

Mean Value or Average Value u

u
No. of Questions Attempted (y)—
No. of Correct Questions (z)—
The steady current, which when passes through a circuit for half the time period of (Without referring Explanations)
alternating current, sends the same amount of charge as done by the alternating current
in the same time through the same circuit, is called mean or average value of alternating u Accuracy Level (z / y × 100)—
current. It is denoted by i m or i av u Prep Level (z / x × 100)—
2i
i m or i av = 0 = 0.636i 0
π In order to expect good rank in JEE,
Mean or average value of alternating current during a half cycle is 0.636 times (or 63.6% your Accuracy Level should be above
85 & Prep Level should be above 75.
of) its peak value (i 0).
276 40 DAYS ~ JEE MAIN PHYSICS DAY TWENTY FOUR

Similarly, mean or average value of alternating emf

Irms
2V R I
Vm or Vav = 0 = 0.636 0 V
π

Peak Value V = V0 sin wt


V
(a) (b) (c)
The maximum value (amplitude) of alternating current and
voltage is called peak value. Current and voltage are in the same phase, i.e. current is
given by I = I 0 sin ωt .
RMS Value 2. Pure Inductive Circuit
The steady current, which when passes through a resistance
Let an alternating voltage V = V0 sin ωt be applied across a
for a given time will produce the same amount of heat as the
alternating current does in the same resistance and in the pure inductance L.
same time, is called rms value of alternating current. It is π
i Then, the average power = Vrms I rms cos = 0
denoted by i rms or i v = 0 = 0.707 i 0 2
2
The inductance offers some opposition to the flow of AC,
where, i 0 = peak value of alternating current known as inductive reactance X L = 2πνL = Lω.
Similarly, rms value of alternating emf Thus, a pure inductance does not oppose the flow of DC
V (ω = 0) but opposes the flow of AC.
V rms = 0 = 0.707 V0
2 V
Current flowing, I =
XL
Reactance and Impedance V XL
l
The opposition offered by a pure inductor or capacitor or
both to the flow of AC, through it, is called reactance (X ). L (b) (c)
(a)
Its unit is ohm (Ω) and dimensional formula is π
[ML 2 T−3 A −2 ]. 2
V = V0 sin ωt 0 I 0 V
l
Reactance is of two types
(i) Inductive reactance, X L = Lω and In pure inductive circuit, current decreases with an
1 π
(ii) Capacitive reactance, X C = increase in frequency, it lags behind the voltage by
Cω 2
π
l
Reciprocal of reactance is known as susceptance. (or voltage leads the current by ) and is thus given by
1 2
Thus, S =  π
X I = I 0 sin ωt − 
 2
l
Total opposition offered by an AC circuit to the flow of
current through it, is called its impedance (Z). Its unit is
ohm and dimensional formula is [ML 2 T −3 A −2 ]. 3. Pure Capacitive Circuit
Let an alternating voltage V = V0 sin ωt be applied across a
For an AC circuit, Z = X 2 + R2 = ( X L − X C )2 + R2
pure capacitance C. Then, the capacitance offers some
opposition to the flow of current, but allows AC to pass
l
Reciprocal of impedance is known as admittance.
1 through it. The opposition offered is known as the
Thus, Y = . Its unit is Siemens (S). capacitive reactance.
Z
1
XC = Ω

Different Types of AC Circuits =
1

C × 2 πν
The circuit consists of only resistor or only capacitor or only
inductor are called pure resistive, pure inductive and pure V
Current flowing, I =
capacitive circuit. XC

I XC
1. Pure Resistive Circuit π
2
Let an alternating voltage V = V0 sin ωt be applied across a (a) C (b) (c)
pure resistance R. Then,
V V
Current, I = or I rms = rms V = V0 sin ωt V
V
R R
DAY TWENTY FOUR ALTERNATING CURRENT 277

In pure capacitive circuit, current increases with an ∴ V = VR2 + VC2


π
increase in frequency and leads the voltage by (or
2 where, VR = voltage across resistor R
π
voltage lags behind the current by ) and is thus, given by and VC = voltage across capacitor.
2
 π
I = I 0 sin ωt + 
 2
3. Series L-C Circuit
The potential difference across a capacitor in AC lags in
phase by 90° and leads in phase by 90° across inductor
Series AC Circuits with the current in the circuit.
Some of the series AC circuits are given below V0
V = V0 sin ωt , I = sin(ωt − φ)
Z
1. Series L-R Circuit X L − XC
where, Z = X L − X C and tan φ = =∞
The potential difference across the resistance in an AC 0
circuit is in phase with current and it leads in phase by L C VL
90° with current across the inductor.
L R VL – VC
VL V
VL VR I
φ V = V0 sin ωt
VR I VC

V = V0 sin ωt π π
For X L > X C , φ = and for X L < X C , φ = −
2 2
V0 1
V = V0 sin ωt and I = sin(ωt − φ) If X L = X C i.e. at ω = , Z = 0 and I 0 becomes infinity.
Z LC
This condition is termed as the resonant condition and
where, Z = R2 + (ωL)2 this frequency is termed as natural frequency of the
circuit.
Current lags behind the voltage by φ. I Z
ωL
and tan φ =
R
∴ V = VR2 + VL2

where, VR = voltage across resistor R ω ω


ω0 ω0
and VL = voltage across inductor. Variation of I with Variation of Z with
ω in L-C circuit ω in L-C circuit

2. Series R-C Circuit


The potential difference across a resistance in AC circuit
4. Series L-C-R Circuit
is in phase with current and it lags in phase by 90° with V0
For L-C-R circuit V = V0 sin ωt , I = sin (ωt − φ)
the current in the capacitor. Z
V 2
V = V0 sin ωt and I = 0 sin (ωt + φ)  1 
Z where, Z = R2 + ωL − 
 ωC 
2
 1  X L − XC
where, Z = R2 +   and tan φ =
 ωC  Z
Current leads the voltage by φ. VL
−1 / ωC L C R
and tan φ = VL – VC
R V
C R φ
VR I
I VR
VC VR φ
V = V0 sin ωt
V
VC
VC Phasor diagram for L-C-R
V = V0 sin ωt series circuit for XL>XC
278 40 DAYS ~ JEE MAIN PHYSICS DAY TWENTY FOUR

For X L > X C , current lags voltage. It is known as resonance frequency.


X L < X C , current leads voltage. At resonance frequency admittance is minimum of the
X L = X C , current and voltage are in phase. impedance is maximum. Thus, the parallel circuit does not
1 allow this frequency from the source to pass in the circuit.
If X L = X C ⇒ ω 0 = , i.e. the natural frequency of the Due to this reason, the circuit with such a frequency is known
LC as rejector circuit.
circuit is equal to the applied frequency, then the circuit is
1 L
said to be in resonance. l
Dynamic resistance, Z max = =
Ymax CR
At resonance, the current in the circuit is maximum and
the impedance is minimum and equal to R. V0 V CR
l
Peak current through the supply = = 0
1 L/CR L
Resonance frequency, ν =
2 π LC V0
l
The peak current through capacitor = = V0ωC
1/ωC
Quality Factor V0ωC ωL
l
Q-factor = =
V0CR/ L R
The Q-factor or quality factor of a resonant L-C-R circuit is
defined as ratio of the voltage drop across inductor This is basically the measure of current magnification.
(or capacitor) to applied voltage. Thus,
voltage across L (or C)
Q= L-C Oscillations
applied voltage
1 L An L-C circuit also called a resonant circuit, tank circuit or
Q= tuned circuit. When connected together, they can act as an
R C
electrical resonator, storing energy oscillating at the circuits
resonant frequency.
Parallel Resonant Circuit I

In this combination, a capacitor is connected in parallel with a


series combination of inductor and resistor. L C
R L
IL
IC The energy oscillates back and forth between the capacitor
C
and inductor until internal resistance makes the oscillations
I
die out. The oscillation frequency is determined by the
capacitance and inductance values,
ω 1
V = V 0 sin ωt f = 0 =
2 π 2 π LC
From the figure,
I = I L + IC Power in an AC Circuit
V V V
or = + Let a voltage V = V0 sin ωt be applied across an AC and
Z R + jωL − j/ωC consequently a current I = I 0 sin(ωt − φ) flows through the
1 1 circuit. Then,
∴ = + jωC
Z R + jωL l
Instantaneous power = V I = V0I 0 sin ωt sin(ωt − φ)
1
is known as admittance (Y ). and its value varies with time. Here, φ is known as phase
Z difference between V and I.
1 R − jωL
Thus, Y = = 2 + jωC l
Average power over a full cycle of AC is
Z R + ω2 L2 1
Pav = Vrms I rms cos φ = V0I 0 cos φ
R2 + (ωCR2 + ω3 L2C − ωL)2 2
∴ Y = The term Vrms I rms is known as the apparent or virtual
R2 + ω2 L2
power, but Vrms I rms cos φ is called the true power.
The admittance will be minimum, when
l
The term cos φ is known as the power factor of the given
1 R2
ωCR + ω L C − ωL = 0 or ω =
2 3 2
− 2 circuit. Thus,
LC L R true power
cos φ = = power factor =
ω 1 1 R2 Z apparent power
∴ f = = − 2
2π 2π LC L
DAY TWENTY FOUR ALTERNATING CURRENT 279

For a pure resistive circuit, V and I are in phase


AC Generator
l

(φ = 0 °), hence cos φ = 1 and average power = Vrms I rms


For a pure inductive or a pure capacitive circuit, current An electric generator or dynamo is a device used to produce
π π electrical energy at the expense of mechanical/thermal energy.
and voltage differ in phase by  i.e. φ =  . It works on the principle of electromagnetic induction, when
2 2
a coil is rotated in a uniform magnetic field, an induced emf
∴ Pavg = 0 is set up between its ends. The induced emf is given by
V2 e = e 0 sin ωt = NBAω sin ωt .
l
Power loss = I 2 R =
R The direction of the induced emf is alternating in nature.
Wattless Current
Average power is given by Pav = E rms I rms cos φ Transformer
The phase difference between E rms and I rms is φ. We can It is a device which works in AC circuits only and is based on
resolve I rms into two components the principle of mutual induction.
I rms cos φ and I rms sin φ Transformer is used to suitably increase or decrease the voltage
Here, the component I rms cos φ contributes towards power in an AC circuit. Transformer which transforms strong AC at
low voltage into a weaker current at high alternating voltage is
dissipation and the component I rms sin φ does not contribute
called a step-up transformer. A step-down transformer
towards power dissipation. Therefore, it is called wattless
transforms weak current at a higher alternating voltage into a
current. strong current at a lower alternating voltage.
e V N Ip
Choke Coil For an ideal transformer s = s = s = =k
ep Vp Np Is
A low resistance inductor coil used to suppress or limit the
where, k is known as the transformation ratio.
flow of alternating current without affecting the flow of direct
current is called choke coil. For a step-up transformer, k > 1 but for a step-down
transformer, k < 1.
Let us consider a choke coil (used in tube lights) of large
inductance L and low resistance R. The power factor for such In a transformer, the input emf and the output emf differ in
a coil is given by phase by π radians.
R R The efficiency of a transformer is given by
cos φ = ≈ [as, R << ωL] output power Vs I s
R +ω L
2 2 2 ω L η= =
input power V p I p
As R << ωL, cos φ is very small. Thus, the power absorbed by
the coil V rms I rms cos φ is very small. On account of its large For an ideal transformer, η = 100% or 1. However, for practical
transformers, η ≈ 85 - 90%.
impedance Z = R2 + ω2 L2 , the current passing through the
Possible causes of energy loss in transformer are
coil is very small. Such a coil is used in AC circuits for the l
Heating due to winding resistance
purpose of adjusting current to any required value without
wastage of energy. l
Eddy current losses
The only loss of energy is due to hysteresis in the iron core, l
Magnetic flux leakage and
which is much less than the loss of energy in the resistance l
Hysteresis loss. To minimise these losses, the transformer
that can also reduce the current, if placed instead of the core is made up of a laminated soft iron strips.
choke coil.
280 40 DAYS ~ JEE MAIN PHYSICS DAY TWENTY FOUR

DAY PRACTICE SESSION 1

FOUNDATION QUESTIONS EXERCISE


1 The alternating current in a circuit is described by the V V (R 1 + R 2 )
(b) at t = 0 and at t = ∞
graph as shown in figure. The rms current obtained from R2 R 1R 2
the graph would be V VR1R 2
(c) at t = 0 and at t = ∞
3 R2 R12 + R 22
2 V (R1 + R 2 ) V
(d) at t = 0 and at t = ∞
1 R1R 2 R2
T
0
2T 5 An L-C circuit is in a state of resonance. If C = 01
. µF and
–1 t
I (A)

L = 0 . 25 H, then neglecting the ohmic resistance of the


–2
circuit, find the frequency of oscillations.
–3
(a) 1007 Hz (b) 100 Hz (c) 109 Hz (d) 500 Hz
(a) 1.414 A (b) 2.2 A 6 An L-C-R circuit as shown in the figure is connected to a
(c) 1.9 A (d) 2.6 A
voltage sourceVAC whose frequency can be varied. The
2 An alternating voltage (in volts) given by frequency, at which the voltage across the resistor is
V = 200 2 sin(100 t ) maximum, is ª JEE Main (Online) 2013
is connected to a 1µF capacitor through an AC ammeter.
V
The reading of the ammeter will be
(a) 10 mA (b) 20 mA
(c) 40 mA (d) 80 mA 24 H 2 mF 15 W

3 The figure shows an experimental plot discharging of a


capacitor in an R-C circuit. The time constant τ of this VAC = V0 sin wt
circuit lies between ª AIEEE 2012
(a) 902 Hz (b) 143 Hz (c) 23 Hz (d) 345 Hz
Potential difference V

25
7 In the circuit shown here, the voltage across L and C are
20 respectively, 300 V and 400 V. The voltage E of the AC
(in volt)

15 source is ª JEE Main Online 2013


L
10

5 C
0
50 100 150 200 250 300
E
Time (in second)
(a) 400 V (b) 500 V (c) 100 V (d) 700 V
(a) 150 s and 200 s (b) 0 and 50 s
(c) 50 s and 100 s (d) 100 s and 150 s 8 A fully charged capacitor C with initial charge q 0 is
connected to a coil of self-inductance L at t = 0. The time
4 In the circuit shown below, the key K is closed at t = 0.
at which the energy is stored equally between the electric
The current through the battery is ª AIEEE 2010
and the magnetic fields is ª AIEEE 2011
V K π
(a) LC (b) 2 π LC (c) LC (d) π LC
4
L R1 9 In an L-C-R circuit, if V is the effective value of the
applied voltage, VR is the voltage across R, VL is the
R2 effective voltage across L, VC is the effective voltage
across C, then
VR 1R 2 V (a) V = VR + VL + VC (b) V 2 = VR2 + VL2 + VC2
(a) at t = 0 and at t = ∞
R 12 + R 2
2
R2 (c) V 2 = VR2 + (VL − VC ) 2 (d)V 2 = VL2 + (VR − VC ) 2
DAY TWENTY FOUR ALTERNATING CURRENT 281

10 An AC circuit of variable frequency f is connected to an 14 The plot given below is of the average power delivered to
L-C-R series circuit. Which one of the graphs in the an L-R-C circuit versus frequency. The quality factor of the
figure, represents the variation of current I in the circuit circuit is ª JEE Main (Online) 2013
with frequency f ?

Average power (microwatts)


I I

1.0
(a) (b)
1.5
f f

I I 0.0

3 4 5 6 7
(c) (d) Frequency (kHz)
(a) 5.0 (b) 2.0
f f (c) 2.5 (d) 0.4

11 In a series L-C-R circuit, C = 10−11 F, L = 10−5 H and 15 For an R-L-C circuit driven with voltage of amplitude v m
1
R = 100 Ω, when a constant DC voltage E is applied to and frequency ω 0 = , the current exhibits resonance.
LC
the circuit, the capacitor acquires a charge10−9 C. The
The quality factor Q is given by
DC source is replaced by a sinusoidal voltage source in
ª JEE Main 2018
which the peak voltage E 0 is equal to the constant DC
ωL ωR
voltage E . At resonance, the peak value of the charge (a) 0 (b) 0
R L
acquired by the capacitor will be ª JEE Main (Online) 2013
R CR
(a) 10−15 C (b) 10−6 C (c) (d)
ω0 C ω0
(c) 10−10 C (d) 10−8 C
12 In a L-C-R circuit as shown below, both switches are 16 In a series L-C-R circuit, R = 200 Ω and the voltage; and
open initially. Now, switch S1 and S 2 , kept open the frequency of the main supply is 220 V and 50 Hz,
respectively. On taking out the capacitance from the
(q is charge on the capacitor and τ = RC is capacitance
circuit, the current lags behind the voltage by 30°. On
time constant). Which of the following statement is
taking out the inductor from the circuit, the current leads
correct ? ª JEE Main 2013
the voltage by 30°. The power dissipated in the L-C-R
circuit is ª AIEEE 2010
V S1
R (a) 305 W (b) 210 W (c) zero (d) 242 W

C 17 In an AC circuit, the voltage applied is E = E 0 sin ωt . The


 π
S2 resulting current in the circuit is I = I 0 sin ωt −  . The
 2
L
power consumption in the circuit is given by
(a) Work done by the battery is half of the energy E0 I0
(a) P = (b) P = zero
dissipated in the resistor 2
E0 I0
(b) At t = τ, q =
CV (c) P = (d) P = 2 E0 I0
2 2
(c) At t = 2 τ, q = CV (1 − e − 2 ) 18 Which of the following components of an L-C-R circuit,
τ
(d) At t = , q = CV (1 − e − 1) with an AC supply, dissipates energy?
2
(a) L (b) R (c) C (d) All of these
13 In a series resonant L-C-R circuit, the voltage across R is
19 An AC circuit consists of a 220 Ω resistance and a 0.7 H
100 V and R = 1 kΩ with C = 2 µF. The resonant
choke. The power absorbed from 220 V and 50 Hz
frequency ω is 200 rad/s. At resonance, the voltage
source connected in this circuit, if the resistance and
across L is
choke are joined in series is
(a) 2.5 × 10−2 V (b) 40 V
(c) 250 V (d) 4 × 10−3 V (a) 110 W (b) 50 W (c) 220 W (d) 440 W
282 40 DAYS ~ JEE MAIN PHYSICS DAY TWENTY FOUR

20 The output of a step-down transformer is measured to be (c) Statement I is true; Statement II is false
24 V when connected to a 12 W light bulb. The value of (d) Statement I is false; Statement II is true
the peak current is 22 Statement I Two identical heaters are connected to two
(a) 1 / 2 A (b) 2A (c) 2A (d) 2 2A different sources one DC and other AC having same
21 A transformer has turn ratio 2 and input power 3600 W. potential difference across their terminals.The heat
Load current is 20 A. Efficiency η = 90%. The internal produced in heater supplied with AC source is greater.
resistance is Statement II The net impedance of an AC source is
(a) 1 Ω (b) 0.9 Ω . Ω
(c) 19 (d) 3 Ω greater than resistance.
23 Statement I In a series L - C - R circuit, the resonance can
Direction (Q. Nos. 22-24) Each of these questions contains take place.
two statements : Statement I and Statement II. Each of these
questions also has four alternative choices, only one of which Statement II Resonance takes place, if the inductive and
capacitive reactances are equal and opposite.
is the correct answer. You have to select one of the codes (a),
(b), (c), (d) given below. 24 Statement I In a series R-L-C circuit, the voltage across
(a) Statement I is true, Statement II is true; Statement II is the resistor, inductor and capacitor are 8V, 16V and 10V,
the correct explanation for Statement I respectively. The resultant emf in the circuit is 10V.
(b) Statement I is true, Statement II is true; Statement II is Statement II Resultant emf of the circuit is given by the
not the correct explanation for Statement I relation E = VR2 + (VL − VC ) 2 .

DAY PRACTICE SESSION 2

PROGRESSIVE QUESTIONS EXERCISE


1 In an AC circuit, the instantaneous emf and current are different values L 1 and L 2 (L 1 > L 2 ) of L, then which of the
given by following represents this graph correctly? (Plots are
π schematic and not drawn to scale) ª JEE Main 2015

e = 100 sin 30 t , i = 20 sin  30 t − 
 4
L1 Q2 L1
2 max
In one cycle of AC, the average power consumed by the (a) Qmax (c) L2
L2
circuit and the wattless current are respectively, t t
ª JEE Main 2018
1000
(a) 50 W, 10 A (b) W, 10 A L2 Q2 Q0 (For both L1and L2)
2
2 (b) Qmax max
50 L1 (d)
(c) W, 0 A (d) 50 W, 0 A
2 t t

2 An arc lamp requires a direct current of 10 A at 80 V to 4 A resistor R and 2 µF capacitor in series is connected
function. If it is connected to a 220 V (rms), 50 Hz AC through a switch to 200 V direct supply. Across the
supply, the series inductor needed for it to work is close
capacitor is a neon bulb that lights up at 120 V. Calculate
to ª JEE Main 2016
the value of R to make the bulb light up 5 s after the switch
(a) 80 H (b) 0.08 H (c) 0.044 H (d) 0.065 H
has been closed ( Take, log10 2 .5 = 0.4) ª AIEEE 2011
3 An L-C-R circuit is equivalent to a damped pendulum. In
an L-C-R circuit, the capacitor is charged to Q 0 and then . × 105 Ω
(a) 17 (b) 2.7 × 106 Ω
connected to the L and R as shown below (c) 3.3 × 107 Ω . × 104 Ω
(d) 13

R L
5 Let C be the capacitance of a capacitor discharging
through a resistor R. Suppose t1 is the time taken for the
energy stored in the capacitor to reduce to half its initial
value and t 2 is the time taken for the charge to reduce to
C t
one-fourth its initial value. Then, the ratio 1 will be
t2 ª AIEEE 2010
If a student plots graphs of the square of maximum 1 1
2
(a) 1 (b) (c) (d) 2
charge (Q max ) on the capacitor with time (t) for two 2 4
DAY TWENTY FOUR ALTERNATING CURRENT 283

6 The average current in terms of I 0 for the waveform as 12 An inductor of reactance1 Ω and a resistor of 2Ω are
shown is connected in series to the terminal of a 6V(rms) AC
V source.The power dissipated in the circuit is
(a) 8 W (b)12 W (c)14.4 W (d)18 W
+ I0 13 You are given many resistances, capacitors and inductors.
t These are connected to a variable DC voltage source (the
T first two circuits) or an AC voltage source of 50Hz
– I0 frequency (the next three circuits)in different ways as
shown in Column II. When a current I (steady state for DC
I0 I0 I0 or rms for AC) flows through the circuit, the corresponding
(a) I 0 (b) (c) (d) voltageV1 andV2 (indicated in circuits) are related as
3 2 4
shown in Column I. Match the Column I with Column II and
7 A bulb is rated 55 W/110 V. It is to be connected to a mark the correct option from the codes given below.
220 V/50 Hz with inductor in series. The value of
inductance, so that bulb gets correct voltage is Column I Column II
(a) 200 Ω (b) 110 Ω (c) 50 Ω (d) 220 Ω V1 V2
8 A coil of 0.01 H inductance and1 Ω resistance is
6 mH 3 mF
connected to 200 V, 50 Hz AC supply. The impedance of A. I ≠ 0, V1 is proportional to I 1.
the circuit and time lag between maximum alternating
voltage and current would be
1 1 V
(a) 3.3 Ω and s (b) 3.9 Ω and s
250 160 V1 V2
1 1
(c) 4.2 Ω and s (d) 2.8 Ω and s
100 120
6 mH 2W
B. I ≠ 0, V2 > V1 2.
9 The bandwidth in a series L-C-R circuit is
LC 2LC
(a) (b)
2 R C + 4 LC
2 2
R C 2 + 4 LC
2
V
R 2 C 2 + 4 LC V1 V2
(c) (d) zero
LC
10 An L - C - R circuit, consists of an inductor, a capacitor and 6 mH 2W
C. V1 = 0, V2 = V 3.
a resistor driven by a battery and connected by two
switches S1 and S 2 , as shown in the figure.
V
V V1 V2
R S1
C
6 mH 3 mF
D. I ≠ 0, V2 is proportional to I 4.
S2
L
V
At time t = 0, when the charge on the capacitor plates is V1 V2
q, switch S1 is opened and S 2 is closed. The maximum
charge the capacitor can hold, is q 0 . Choose the correct 1kW 3 mF
5.
equation
π π
(a) q = q0 cos +  (b) q = q0 cos − 
t t
 LC 2  LC 2  V
d 2q 1 d 2q
(c) q = − LC 2 (d) q = − Codes
dt LC dt 2
A B C D
11 An alternating emf of angular frequency ω is applied (a) (2,3,4,5) (1,2) (2,3,4,5) (3,4,5)
across an inductance. The instantaneous power (b) (3,4,5) (2,3,4,5) (1,2) (2,3,4,5)
developed in the circuit has an angular frequecy (c) (1,2) (3,4,5) (2,3,4,5) (2,3,4,5)
(d) (3,4,5) (1,2) (2,3,4,5) (2,3,4,5)
(a) ω / 4 (b) ω / 2 (c) ω (d) 2ω
284 40 DAYS ~ JEE MAIN PHYSICS DAY TWENTY FOUR

Direction (Q. Nos. 14 and 15) Each of these questions 14 Statement I A sinusoidal AC current flows through a
contains two statements : Statement I and Statement II. resistance R. If the peak current is I 0 , then the power
Each of these questions also has four alternative choices, only RI 2
dissipated is 0 .
one of which is the correct answer. You have to select one of 2
the codes (a), (b), (c), (d) given below.
Statement II For a purely resistive circuit, the power
(a) Statement I is true, Statement II is true; Statement II is factor, cos φ = 1.
the correct explanation for Statement I
(b) Statement I is true, Statement II is true; Statement II is 15 Statement I The nature of the impedance of L - C - R
not the correct explanation for Statement I circuit, at resonance is pure inductive.
(c) Statement I is true; Statement II is false Statement II The phase angle between E and I in a
(d) Statement I is false; Statement II is true R - L - C circuit at resonance, is zero.

ANSWERS

SESSION 1 1 (a) 2 (b) 3 (d) 4 (b) 5 (a) 6 (c) 7 (c) 8 (a) 9 (c) 10 (d)
11 (c) 12 (c) 13 (c) 14 (d) 15 (a) 16 (d) 17 (b) 18 (b) 19 (a) 20 (a)
21 (b) 22 (a) 23 (a) 24 (a)

SESSION 2 1 (b) 2 (d) 3 (a) 4 (b) 5 (c) 6 (c) 7 (d) 8 (a) 9 (c) 10 (c)
11 (d) 12 (c) 13 (a) 14 (b) 15 (d)

Hints and Explanations


SESSION 1 4 At t = 0, inductor behaves like an =
1000
infinite resistance. 2 × 314
. × 6.92
I + I + I
2 2 2
1 I rms = 1 2 3
V =
1000
= 23 Hz
3 So, at t = 0, I = and at t = ∞,
R2 43.45
12 + 22 + 12 6
= = inductor behaves like a conducting 7 Since, reactances produced by inductor and
3 3 V V (R1 + R2 )
wire I = = capacitor in opposite direction. So, voltage
= 2 = 1.414 A R eq R1 R2 in these elements are distributed at 180°, i.e.
out of phase.
2 Given, V = 200 2 sin(100 t ). 5 At the resonance, ν = 1
Comparing this equation with 2 π LC Net voltage = 400 V − 300 V
V = V 0 sin ω t, we have 1 = 100 V
=
V 0 = 200 2 V and ω = 100 rad s −1 2 × 314
. × . × 10−6
0.25 × 01 8 As initially charge is maximum,
The current in the capacitor is = 1007 Hz q = q 0 cos ω t
V
I = rms = V rms × ωC 6 As voltage across resistance is ⇒ I =
dq
= − ω q 0 sin ω t
ZC dt
Q Z = 1  maximum, therefore a power is
  1 q2
 C
ωC  maximum which is at the resonance Given, L I 2 =
V frequency. 2 2C
= 0 × ωC At resonance, 1 (q cos ω t )2
2 ⇒ L (ω q 0 sin ω t )2 = 0
1 1 2 2C
200 2 =
= × 100 × 1 × 10−6 frequency 2 π LC 1
2 But ω=
1 1 LC
= 20 × 10−3 A = 20 mA =
2 π 24 × 2 × 10−6 ⇒ tan ω t = 1
3 Time constant τ is the duration when π
1 1000 ωt =
the value of potential drops by 63% of = 4
2 π 48 π π
its initial maximum value (i.e. V 0 / e ).
1 1000 ⇒ t = = LC
Here, 37% of 25 V = 9.25 V which lies = 4ω 4
between 100 s to 150 s in the graph. 2 π 6.9V
DAY TWENTY FOUR ALTERNATING CURRENT 285

9 Phaser diagram of L-C-R series circuit is 13 At resonance, w L = 1 / w C Now, power absorbed in the circuit is
shown in figure. Current flowing through the circuit, P = V rms I rms cos f
V 100 = (220) (0707
. ) (0707
. )
VL I = R = = 01
. A
R 1000 = 109.96 » 110 W
(VL – VC )
So, voltage across L is given by 20 Secondary voltage, VS = 24 V
V L = I X L = Iw L
But w L = 1 / w C Power associated with secondary
VR I Current axis PS = 12 W
VC I
\ VL = = VC P 12 1
wC \ IS = S = = A = 0.5 A
VS 24 2
01
.
= = 250 V
200 ´ 2 ´ 10-6 Peak value of the current in the
V 2 = V R2 + (V L - VC )2 secondary,
14 As quality factor, Q = w 0 I 0 = IS 2
10 The current in an L-C-R circuit is given B
by where, w 0 = resonant frequency = (0.5)(1.414)
V and B = bandwidth. 1
I = 1/ 2 = 0707
. = A
é 2 æ 2
1 ö ù From the graph, B = 2.5 kHz, 2
ê R + ç wL - ÷ ú Q = 0.4
è wC ø úû
êë (by observing the curve) 21 As, V2 = 3600 = 180 V
20
where, w = 2p f 15 Sharpness of resonance of a resonant \ V 0 = V2 ´ h = 180 ´ 0.9 = 162 V
Thus, I increases with an increase in w L-C-R circuit is determined by the ratio Now, V 0 = V2 - I2 r
upto a value given by of resonant frequency with the
r
w = wc , i.e. at w = w c , we have selectivity of circuit. This ratio is also
1 called ‘‘quality factor’’ or Q-factor.
wL =
wC w0 w L
Q-factor = = 0
1 2Dw R 180 V V0
Þ wc =
LC 16 The given circuit is under resonance as
where, I is maximum. XL = X C
Hence, power dissipated in the circuit is V2 - V 0 18
At w > w c , I again starts decreasing \ r = = = 0.9 W
with an increase in w . V2 I 20
P = = 242 W
R 22 For the case of DC, the frequency is zero
q2
11 As energy stored in capacitor = 1
2 C 17 For given circuit, current is lagging the and the net impedance is equal to the
p resistance.For the case of AC, the
Now, when AC is connected to the voltage by , so circuit is purely
1 2 impedance of the AC circuit is given by
circuit energy speed = LI 2 inductive and there is no power
2 Z = R2 + w2 L2
consumption in the circuit. The work
By equating the energies, we get
done by battery is stored as magnetic where, R = resistance,
1 q2 1 2 energy in the inductor.
= LI
2 C 2 w = angular frequency
18 In an AC circuit, only resistor R
(10-9 )2 1 and L = inductance.
= ´ 10-5 I 2 dissipates energy. L and C do not
10-11 2 dissipate energy, because for both of 23 At a particular value of angular
1 them current is wattless (f = 90° ).
Þ I = A frequency, the inductive reactance and
10
19 In series impedance of circuit is capacitive reactance will becomes just
Now, V = IR
1 Z = R2 + w2 L2 = R2 + (2p f L )2 equal to each other and opposite in
= ´ 100 = 10 V value. So that, the impedance of circuit
10 = (220 )2 + (2 ´ 314 . )2
. ´ 50 ´ 07
Therefore, is minimum, i.e. equal to R.
= 311 W
Q = CV V X L = XC
= 10-11 ´ 10 = 10-10 C \ I rms = rms 1
Z Þ w 0L =
220 w 0C
12 For charging of capacitor, = = 0707
. A = 0707
. A
311 1
q = CV (1 - e - t/ t ) at t = 2t, Þ w=
R 220 LC
q = CV (1 - e -2
) and cos f = = = 0.707
Z 311
286 40 DAYS ~ JEE MAIN PHYSICS DAY TWENTY FOUR

24 In a series R-L-C circuit, the resultant 3 Consider the L-C-R circuit at any time t 2
Þ e - t / RC =
emf is 5
R i + –
E = V R2 + (V L - VC )2 Þ t = RC ln 2.5
= 82 + (16 - 10)2 Þ R=
t
= 64 + 36 = 100 C ln 2.5
+ –
= 10 V t
q C =
2303
. C log 2.5
SESSION 2 Now, applying KVL, we have
q Ldi = 2.7 ´ 106W
- iR - =0
1 Given, e = 100sin 30t C dt
q2
5 As, U = 1 = 1 (q 0e - t / t )
2
p As current is decreasing with time we
and i = 20 sin æç30 t - ö÷
è 4ø dq 2C 2C
can write i = -
\ Average power, dt q 20 -2 t / t
= e (where, t = CR)
Pav = e rms I rms cos f q dq Ld 2q 2C
Þ + R+ =0
100 20 p
C dt dt 2 U = U I e -2 t / t
= ´ ´ cos d 2q R dq q
2 2 4 1 1
or 2
+ + =0 U I = U I e -2 t1 / t , = e -2 t1 / t
dt L dt LC 2 2
1000
= W This equation is equivalent to that of a t
2 Þ t 1 = ln 2
damped oscillator. 2
Wattless current is Thus, we can write the solution as 1
I = I rms sin f Now, q = q 0 e - t / t , q 0 = q 0e - t2 / t ,
Q max (t ) = Q 0 × e - Rt /2 L 4
20 p t 2 = t ln 4 = 2t ln 2
= ´ sin Rt
2 4 or Q 2max = Q 20 e - L t1 1
\ =
As L1 > L2 damping is faster for L2 . t2 4
20
= = 10 A Aliter Inductance is inertia of circuit. It
2
means inductance opposes the flow of 6 As, I = 2I 0 t
1000 T0
\ Pav = W charge, more inductance means decay
2 T0
of charge is slow. where, 0 < t <
and I wattless = 10 A 2
In option (a), in a given time to,
Q 12 > Q 22 . æt ö
2 Given, I = 10 A, V = 80 V and I = 2I 0 ç - 1÷
èT 0 ø
V 80
\ R= = = 8W T0
I 10 L1 where, < t < T0
2 2
and w = 50 Hz Q1
2 T0
\ Iav = I dt
For AC circuit, we have 2
Q2 T ò0
R=8 W L L2 T
2 é 2 2 I 0t ù
10A = ê ò0 dt ú
T 0 êë T0 úû
t0
2
So, L1 > L2 . é
2 2I T ù I
= 2 ê 0 0ú= 0
Hence, option (a) is correct. T0 ë 2 ´ 4 û 2
220 V
V 4 Neon bulb is filled with gas, so its Vapp R
I = 7 We have, 110 =
resistance is infinite, hence no current
2
8 + X 2L R2 + L2 w2
flows through it.
220 220 R
Þ 10 = Þ 110 =
B
64 + X 2L R + L2 w2
2

Þ 64 + X 2L = 22 R Þ 4 R = R2 + L2 w2
2

Squaring on both sides, we get Þ Lw = 3 R


C
64 + X 2L = 484 Þ L( 100p ) = 220 3
Þ X 2L = 484 - 64 = 420 22
. 3
Þ L =
XL = 420 314
.
E S
Þ 2p ´ wL = 420 22
. ´ (1732
. )
= = 1.2 H
Now, VC = E (1 - e - t / RC ) 314
.
Series inductor on an arc lamp,
420 110 ´ 110
L = = 0.065 H Þ 120 = 200 (1 - e - t / RC ) Þ R= = 220W
(2p ´ 50) 55
DAY TWENTY FOUR ALTERNATING CURRENT 287

8 Given, inductance, L = 0.01 H, where, f is not equal to p / 2. 13 In circuit 1,


resistance, R = 1W, voltage, V = 200 V Differentiating Eq. (ii) twice with In steady state, I = 0
and frequency, f = 50 Hz respect to t, we get So, no option matches for circuit 1.
d 2q In circuit 2,
Impedance of the circuit, = - w2q 0 cos(w t + f) = - w2q
2 dt 2 V1 = 0 and V2 = 2I = V
Z = R2 + X L = R2 + ( 2 p f L )2 1 d 2q
q =- 2 2 \ B, C, D ® 2
= 12 + (2 ´ 3.14 ´ 50 ´ 0.01)2 w dt In circuit 3,
or Z = 10.86 = 33 . W d 2q V1 = X L I = 2pf LI
= - LC 2 [using Eq. (i)]
w L 2p f L dt = 2p ´ 50 ´ 6 ´ 10-3 = 1.88 I
\ tan f = =
R R 11 The instantaneous value of emf and and V2 = 2I
2 ´ 3.14 ´ 50 ´ 0.01 current in inductive circuit are given by \ A, B, D ® 3
=
1 p In circuit 4,
E = E 0 sin wt and I = I 0 sin æç wt - ö÷
= 3.14 è 2ø V1 = X L I = 1.88 I
f = tan - 1 (3.14) = 72° respectively. V2 = X C I = 1061 I
72 ´ p \ A, B, D ® 4
Phase difference, f = rad So, Pinst = EI
180 In circuit 5,
p
Time lag between alternating voltage = E 0 sin wt ´ I 0 sin æç wt - ö÷ V1 = IR = 1000 I
è 2ø V2 = X C I = 1016 I
and current,
f 72p 1 = E 0I 0 sin wt \ A, B, D ® 5
Dt = = = s
w 180 ´ 2p ´ 50 250 æsin wt cos p - cos wt sin p ö 14 Power dissipated is given by
ç ÷
è 2 2ø P = E rms I rms cos f
9 At cut-off frequency, Z = 2 R
2 = E 0I 0 sin wt cos wt We know that for a purely resistive
æ 1 ö circuit, the power factor,
R 2 + ç Lw - ÷ = 2R
2
1
è C wø = E 0I 0 sin 2wt R R
2 cos f = = =1
1 (Qsin 2wt = 2sin wt cos wt ) Z R
Þ Lw - =R
Cw Hence, P = E rms ´ I rms
2 Hence, angular frequency of
Þ LCw - RCw - L = 0 = (RI rms ) ´ I rms
instantaneous power is 2w.
= R (I rms ) 2
RC ± R2C 2 + 4LC 12 Given, X L = 1W, R = 2W
Þ w= 2
I R I 20
2LC = R æç 0 ö÷ =
Dw = w 01 - w 02 E rms = 6 V, Pav = ? è 2ø 2
Average power dissipated in the circuit,
2 R2C 2 + 4LC Pav = E rms cos f
15 Since, at resonance,
= 1
2LC I E wL =
2 2
I rms = 0 = rms wC
R C + 4 LC 2 Z
= and impedance,
LC Z = R2 + X 2L = 4 + 1 = 5 2
1 ö
10 When S2 is closed and S1 is open, the 6 Z = R2 + æç wL - ÷
\ I rms = A è wC ø
charge oscillates in the L - C circuit at an 5
angular frequency is given by R 2 or Z = R2 = R
cos f = =
1 Z 5 Hence, nature of impedance at
w= ...(i)
LC 6 2 resonance is resistive.
Pav = 6 ´ ´
5 5
Now, q ¹ 0 at t = 0. Hence, options (a) Also, in a L-C-R circuit, phase angle
and (b) are wrong. The charge q varies [from Eq.(i)]
between the emf and the alternating
72 72
with time t as = = = 14.4 W current I, is zero at resonance.
q = q 0 cos(w t + f) ...(ii) 5 5 5
DAY TWENTY FIVE

Electromagnetic
Waves
Learning & Revision for the Day
u Electromagnetic Waves and u Maxwell’s Equations u Spectrum of Electromagnetic
their Characteristics u Transverse Nature of Electromagnetic Waves Radiation

Electromagnetic Waves and


their Characteristics
Electromagnetic waves are those waves, in which electric and magnetic fields vary
sinusoidally in space with time. The electric and magnetic fields are mutually
perpendicular to each other and each field is perpendicular to the direction of
propagation of the wave.
l
Maxwell’s theory predicted that electromagnetic waves of all frequencies (and hence
all wavelengths) propagate in vacuum, with a speed given by
1
c= .
µ 0ε 0
where, µ 0 is the magnetic permeability and ε 0 is the electric permittivity of vacuum.
Now, for the vacuum, µ 0 = 4π × 10 −7 TmA −1 and ε 0 = 8.85 × 10 −12 C2 N −1m−2 .
Substituting these values in the above relation, we have PREP
c=
1
~ 3.0 × 10 8 ms −1 MIRROR
[(4π × 10 −7 )(8.85 × 10 −12 )]1/ 2 Your Personal Preparation Indicator
l
All the electromagnetic waves are of the transverse nature whose speed depends u No. of Questions in Exercises (x)—
upon the medium, but their frequency does not depend on the medium. u No. of Questions Attempted (y)—
l
Transverse waves can be polarised. u No. of Correct Questions (z)—
l
Energy is being transported with the electromagnetic waves. (Without referring Explanations)

u Accuracy Level (z / y × 100)—


Conduction Current u Prep Level (z / x × 100)—
It is a current in the electric circuit, which arises due to the flow of electrons in the
connecting wires of the circuit, in a definite closed path. In order to expect good rank in JEE,
your Accuracy Level should be above
85 & Prep Level should be above 75.
DAY TWENTY FIVE ELECTROMAGNETIC WAVES 289

In this, E 0 and B0 are the amplitudes of the fields.


Maxwell’s Displacement Current E 1
Further, c = 0 = = speed of light in vacuum
It is that current which comes into play in the region, B0 ε0 µ 0
whenever the electric field and hence the electric flux is
changing with time.
l
The rate of flow of energy in an electromagnetic wave, is
dφ E described by the vector S called the Poynting vector, which
id = ε 0 is defined by the expression,
dt
1
The generalised form of the Ampere’s law is S= E×B
µ0
 dφ E 
∫ B ⋅ d l = µ 0(ic + id ) = µ 0  i c + ε 0 dt 
 l
The time average of S over one cycle is known as the wave
intensity. When the average is taken, we obtain an
where, ic is conduction current. expression involving the time average of cos2 (kx − ωt )
1
which equals .
2
Maxwell’s Equations E B E2 c B02
Maxwell in 1862, gave the basic laws of electricity and Thus, I = Sav = 0 0 = 0 = Wm −2
2 µ 0 2 µ 0c 2 µ 0
magnetism in the form of four fundamental equations, which
are known as Maxwell’s equations. l
The total average energy per unit volume is,
ε E2 B2
l
Gauss’s law for electrostatics This law states that the total u = uE + uB = 0 0 =
electric flux through any closed surface is always equal to 2 2µ0
1 The radiation pressure p exerted on a perfectly absorbing
times the net charged enclosed by that surface. l

ε0 S
surface, p = .
q c
Mathematically, ∫ E ⋅ dS =
S ε0 l
If the surface is a perfect reflector and incidence is normal,
l
Gauss’s law for magnetism This law also predicts that the then the momentum transported to the surface in a time t is
isolated magnetic monopole does not exist. 2u 2S
given by, p = and the radiation pressure will be, p = .
i.e. net magnetic flux through any closed surface is always c c
zero. l
Energy density of electromagnetic wave,
Mathematically, ∫ B ⋅ dS = 0 1 1 B2
S ue = ε 0Eu2B =
2 2 µ0
l
Faraday’s law of electromagnetic induction It states that
u
the induced e.m.f. produced in a circuit is numerically l
Momentum delivered, p = (absorbing surface)
equal to time rate of change of magnetic flux through it. c
dφ 2u
Mathematically, ∫ E ⋅ dl = − B p= (reflecting surface)
dt c
hc
l
Ampere-Maxwell’s law At an instant in a circuit, the l
Energy of wave = = hν
conduction current is equal to displacement current. λ
 dφ 
Mathematically, ∫ E ⋅ dl = µ 0  I c + ε 0 E 
 dt  Transverse Nature of
These equations are collectively called Maxwell’s
equations.
Electromagnetic Waves
According to Maxwell, electromagnetic waves consist of time
varying electric and magnetic fields, which are perpendicular
Properties of to each other, as well as direction of wave propagation.
Electromagnetic Waves Y Wave propagation
E B
l
If the electromagnetic wave is travelling along the positive
direction of the X -axis, the electric field is oscillating
parallel to the Y-axis and the magnetic field is oscillating X
parallel to the Z-axis.
E = E 0 sin(ωt − kx) ⇒ B = B0 sin(ωt − kx) Z B E
290 40 DAYS ~ JEE MAIN PHYSICS DAY TWENTY FIVE

detecting the fracture of bones, hidden bullet, needle,


Spectrum of Electromagnetic costly material etc. inside the body, and also used in the
Radiation study of crystal structure.
The array obtained on arranging all the electromagnetic l
Ultraviolet Rays The major part of the radiations received
waves in an order on the basis of their wavelength is called from sun consists of the ultraviolet radiation. Its other
the electromagnetic spectrum. sources are the electric discharge tube, carbon arc, etc.
The Electromagnetic Spectrum These radiations are mainly used in excitation of
photoelectric effect and to kill the bacteria of many diseases.
Frequency Wavelength l
Visible Light Visible light is obtained from the glowing
Name Source
Range (Hz) Range (m) bodies, while they are white hot. The light obtained from
Radio 104 to 108 0.1 to 600 Oscillating electric the electric bulbs, sodium lamp, fluorescent tube is the
waves circuits visible light.
Microwaves 109 to 1012 10−3 to 0.3 Oscillating current in l
Thermal or Infrared Waves A body on being heated, emits
special vacuum tubes out the infrared waves. These radiations have the
Infrared 1011 to 10−6 to Outer electrons in atoms maximum heating effect. The glass absorbs these
5 × 1014 5 × 10−3 and molecules radiations, therefore for the study of these radiations, rock
salt prism is used instead of a glass prism. These waves are
Visible 4 × 1014 to 4 × 10−7 to Outer electrons in atoms mainly used for therapeutic purpose by the doctors because
light 7 × 1014
8 × 10−7 of their heating effect.
Ultraviolet 1015 to 1017 1.5 × 10−7 to Outer electrons in atoms
l
Microwaves These waves are produced by the spark
discharge or magnetron valve. They are detected by the
3.5 × 10−7
crystal or semiconductor detector. These waves are used
X-rays 1018 to 1020 10−11 to 10−8 Inner electrons in atoms mainly in radar and long distance communication.
and sudden l
Radio waves They can be obtained by the flow of high
deacceleration of high
frequency alternating current in an electric conductor.
energy free electrons
These waves are detected by the tank circuit in a radio
Gamma 1019 to 1024 10−16 to 10−13 Nuclei of atoms and
receiver or transmitter.
rays sudden deacceleration of
high energy free electrons
Applications of
Various Electromagnetic Radiations Electromagnetic Spectrum
l
Gamma rays The main sources of gamma rays are the l
Radio waves are used in radar and radio broadcasting.
natural and artificial radioactive substances. These rays l
Microwaves are used in long distance wireless
affect the photographic plate and mainly used in the communications via satellites.
treatment of cancer disease. l
Infrared, visible and ultraviolet radiations are used to know
l
X-rays X-rays are produced, when highly energetic the structure of molecules.
cathode rays are stopped by a metal target of high melting l
Diffraction of X-rays by crystals, gives the details of the
point. They affect the photographic plate and can penetrate structure of crystals.
through the transparent materials. They are mainly used in
DAY TWENTY FIVE ELECTROMAGNETIC WAVES 291

DAY PRACTICE SESSION 1

FOUNDATION QUESTIONS EXERCISE


1 During the porpagation of electromagnetic waves in a (a) 4 × 10−6 T (b) 6 × 10−8 T
−9
medium, ª JEE Main 2014 (c) 9 × 10 T (d) 11 × 10−11 T
(a) electric energy density is double of the magnetic energy 9 The magnetic field in a travelling electromagnetic wave
density has a peak value of 20 nT. The peak value of electric field
(b) electric energy density is half of the magnetic energy
strength is ª JEE Main 2013
density
(a) 3 V/m (b) 6 V/m (c) 9 V/m (d) 12 V/m
(c) electric energy density is equal to the magnetic energy
density 10 The rms value of the electric field of the light coming from
(d) Both electric and magnetic energy densities are zero the sun is 720 NC −1. The average total energy density of
2 A perfectly reflecting mirror has an area of 1 cm 2. the electromagnetic wave is
Light energy is allowed to fall on it for 1 h at the rate of (a) 4.58 × 10−6 Jm−3 (b) 6.37 × 10−9 Jm−3
10 W cm −2. The force acting on the mirror is (c) 81.35 × 10−12 Jm−3 (d) 3 .3 × 10−3 Jm−3
. × 10−8 N
(a) 67 (b) 2.3 × 10−4 N 11 A radiation of energy E falls normally on a perfectly
(c) 10−3 N (d) zero reflecting surface. The momentum transferred to the
3 The magnetic field between the plates of radius 12 cm, surface is
separated by a distance of 4 mm of a parallel plate E 2E E
(a) (b) (c) E c (d)
capacitor of capacitance 100 pF along the axis of plates c c c2
having conduction current of 0.15 A, is 12 An electromagnetic wave in vacuum has the electric and
(a) zero (b) 1.5 T (c) 15 T (d) 0.15 T magnetic fields E and B, which are always perpendicular
4 Instantaneous displacement current of 1.0 A in the space to each other. The direction of polarisation is given by X
between the parallel plates of a1µF capacitor, can be and that of wave propagation by k.$ Then, ª AIEEE 2012
established by changing potential difference of (a) X || B and k$ || B×E (b) X || E and k$ || E×B
(a) 10−6 Vs–1 (b) 106 Vs–1 (c) X || B and k$ || E×B (d) X || B and k$ || B×E
(c) 10−8 Vs–1 (d) 108 Vs–1
13 An electromagnetic wave travels in vacuum along
5 A large parallel plate capacitor, whose plates have an z-direction E = (E 1 $i + E 2 $j ) cos (kz − ωt ). Choose the
area of 1 m 2 and are separated from each other by correct option from the following
1 mm, is being charged at a rate of 25 Vs −1. If the
(a) The associated magnetic field is given as
dielectric between the plates has the dielectric constant 1
B = (E1 $i − E $j) cos (kz − ωt )
10, then the displacement current at this instant is C
(a) 25 µA (b) 11µA (c) 2.2 µA (d) 1.1 µA (b) The associated magnetic field is given as
1
6 A parallel plate capacitor with plate area A and B = (E1 $i − E $j) cos (kz − ωt )
separation between the plates d, is charged by a C
(c) The given electromagnetic field is circularly polarised
constant current I. Consider a plane surface of area A /2
(d) The given electromagnetic wave is plane polarised
parallel to the plates and drawn simultaneously between
the plates. The displacement current through this area is 14 Match List I (Electromagnetic wave type) with List II (Its
(a) I (b)
I
(c)
I
(d)
I association/application) and select the correct option
2 4 8 from the choices given below the lists. ª JEE Main 2014
7 Select the correct statement from the following
ª JEE Main (Online) 2013 List I List II
A. Infrared waves 1. To treat muscular strain
(a) Electromagnetic waves cannot travel in vacuum
(b) Electromagnetic waves are longitudinal waves B. Radio waves 2. For broadcasting
(c) Electromagnetic waves are produced by charges C. X-rays 3. To detect fracture of bones
moving with uniform velocity D. Ultraviolet waves 4. Absorbed by the ozone layer of
(d) Electromagnetic waves carry both energy and the atmosphere
momentum as they propagate through space Codes
8 In an apparatus, the electric field was found to oscillate A B C D A B C D
with an amplitude of 18 Vm −1. The magnitude of the (a) 4 3 2 1 (b) 1 2 4 3
oscillating magnetic field will be (c) 3 2 1 4 (d) 1 2 3 4
292 40 DAYS ~ JEE MAIN PHYSICS DAY TWENTY FIVE

15 Arrange the following electromagnetic radiations per Statement II Ultraviolet radiations are absorbed by the
quantum in the order of increasing energy. atmosphere.
A. Blue light B. Yellow light
17 Statement I If the earth did not have atmosphere, its
C. X-ray D. Radio wave average surface temperature would be lower than what is
ª JEE Main 2016 (Offline) now.
(a) D, B, A, C (b) A, B, D, C Statement II Greenhouse effect of the atmosphere would
(c) C, A, B, D (d) B, A, D, C
be absent, if the earth did not have atmosphere.
Direction (Q. Nos. 16-20) Each of these questions contains 18 Statement I Electromagnetic waves exert radiation
two statements : Statement I (Assertion) and Statement II pressure.
(Reason). Each of these questions also has four alternative Statement II Electromagnetic waves carry energy.
choices, only one of which is the correct answer. You have to
select one of the codes (a), (b), (c), (d) given below 19 Statement I Light is a transverse wave, but not an
(a) Statement I is true, Statement II is true; Statement II is electromagnetic wave.
the correct explanation for Statement I Statement II Maxwell showed that speed of
(b) Statement I is true, Statement II is true; Statement II is electromagnetic waves is related to the permeability and
not the correct explanation for Statement I the permittivity of the medium through which it travels.
(c) Statement I is true; Statement II is false 20 Statement I Out of radio waves and microwaves, the
(d) Statement I is false; Statement II is true radio waves undergo more diffraction.
16 Statement I Ultraviolet radiation being higher frequency Statement II Radio waves have greater frequency
waves are dangerous to human being. compared to microwaves. ª JEE Main (Online) 2013

DAY PRACTICE SESSION 2

PROGRESSIVE QUESTIONS EXERCISE


1 You are given a 2 µF parallel plate capacitor. How would 4 The ratio of contributions made by the electric field and
you establish an instantaneous displacement current of magnetic field components to the intensity of an
1 mA in the space between its plates? electromagnetic wave is
(a) By applying a varying potential difference of 500 V/s (a) c:1 (b) c 2 :1 (c) 1:1 (d) c :1
(b) By applying a varying potential difference of 400 V/s
(c) By applying a varying potential difference of 100 V/s 5 An FM radio station, antenna radiates a power of 10 kW
(d) By applying a varying potential difference of 300 V/s at a wavelength of 3 m. Assume the radiated power is
2 A uniform but time varying magnetic field B(t ) exists in a confined to and is uniform over a hemisphere with
circular region of radius a and is B(t) antenna at its centre. E max at a distance of 10 km from
directed into the plane of the paper× × × × × × × × × ×
× × × × × × × × × P× antenna is
as shown in the figure. The r
× × × × × × × × × ×
(a) 0.62 NC −1 (b) 0.41 NC −1
magnitude of the induced electric × × × × × × × × × ×
(c) 0.31 NC −1 (d) 0.10 NC −1
field at a point P, a distance r from× × × ×a × × × × × ×
× × × × ×× × × × ×

the centre of the circular region × × ××× × × × × × × 6 Assume that all the energy from a 1000 W lamp is
radiated uniformly, then the amplitude of electric field of
× × × × × × × × ×
(a) increases with r × × × × × × × × × ×
(b) decreases with r × × × × × × × × × × radiation at a distance of 2 m from the lamp is
1
(c) decreases as 2 (a) 245.01 V/m (b) 17 V/m
r (c) 0 (d) 2.96 V/m
(d) zero
7 A red LED emits light at 0.1W uniformly around it. The
3 The magnetic field of a beam emerging from a filter facing amplitude of the electric field of the light at a distance of
a flood light is given by 1m from the diode is ª JEE Main 2015
B = 12 × 10−8 sin (1.20 × 107 z − 3.60 × 1015 t ) T. What is the
(a) 1.73 V/m
average intensity of the beam?
(b) 2.45 V/m
(a) 1.7 W/ m2 (b) 2.3 W/m2 (c) 5.48 V/m
2
(c) 2.7 W/m (d) 3.2 W/m2 (d) 7.75 V/m
DAY TWENTY FIVE ELECTROMAGNETIC WAVES 293

8 In a transverse wave, the distance between a crest and εr1 εr1 εr1 1 εr1 1
(a) =4 (b) =2 (c) = (d) =
neighbouring through at the same instant is 4.0 cm and εr2 εr2 εr2 4 εr2 2
the distance between a crest and trough at the same
10 An electromagnetic wave of frequency ν = 3 .0 MHz
place is 1.0 cm. The next crest appears at the same
passes from vacuum into a dielectric medium with
place after a time interval of 0.4 s. The maximum speed
permittivity ε = 4.0. Then,
of the vibrating particles in the medium is
(a) wavelength is doubled and the frequency remains
ª JEE Main (Online) 2013
unchanged

(a) cm/s (b) 5 π cm/s (b) wavelength is doubled and frequency becomes half
2
π (c) wavelength is halved and frequency remains unchanged
(c) cm/s (d) 2π cm/s
2 (d) wavelength and frequency both remain unchanged
9 An EM wave from air enters a medium. The electric 11 The magnetic field at a point between the plates of a
 z  capacitor at a perpendicular distance R from the axis of
fields are E1 = E 01x$ cos 2πν  − t   in air and
  c  the capacitor plate radius R, having the displacement
E2 = E 02x$ cos[k ( 2z − ct )] in medium, where the wave current ID is given by
number k and frequency ν refer to their values in air. µ IDr µ 0 ID
(a) (b)
The medium is non-magnetic. 2 πR 2 2 πR
µ 0 ID
(c) (d) zero
If εr1 and εr2 refer to relative permittivities of air and πr 2
medium respectively, which of the following options is
correct? ª JEE Main 2018

ANSWERS
SESSION 1 1 (c) 2 (a) 3 (a) 4 (b) 5 (c) 6 (b) 7 (d) 8 (b) 9 (b) 10 (a)
11 (b) 12 (b) 13 (d) 14 (d) 15 (a) 16 (b) 17 (a) 18 (b) 19 (d) 20 (c)

SESSION 2 1 (a) 2 (b) 3 (a) 4 (c) 5 (d) 6 (a) 7 (b) 8 (b) 9 (c) 10 (c)
11 (b)

Hints and Explanations


SESSION 1 3 As B ∝ r , since the point is on the axis, = 8.85 × 10−8 × 25
1 Both the energy densities are equal, where r = 0, so B = 0. = 2.2 × 10−6 = 2.2 µA
i.e. energy is equally divided between
electric and magnetic field. 4 As, Id = C  V  Q I = dQ 
 
6 Charge on the capacitor plates, at time t is,
t   dt  q = It
2 Let E = energy falling on the surface V I
Electric field between the plates at this
or = d instant,
per second = 10 J t C q It
Momentum of photons, E = =
h h =
1.0
= 106 Vs –1 A ε0 A ε0
p= = 10−6
λ c/f Electric flux through the given area,
5 As, C = ε0KA φ E =   E =
A It
hf E
= = d 2 2 ε0
c c
Therefore, displacement current,
On reflection, change in momentum (8.85 × 10−12 ) × 10 × 1
= dφ
per second = force 10 −3 Id = ε 0 E
2E 2 × 10 dt
= 2p = =
c 3 × 108 = 8.85 × 10−8 F d  It  I
= ε0  =
= 6.7 × 10−8 N d dV dt  2 ε0  2
∴ I = (CV ) = C
dt dt
294 40 DAYS ~ JEE MAIN PHYSICS DAY TWENTY FIVE

7 B E Electromagnetic wave 14 (a) Infrared waves are used to treat 19 In free space or vacuum, the speed of
muscular strain. electromagnetic waves is
Direction of
(b) Radio waves are used for 1
Propagation c = …(i)
broadcasting purposes. µ 0ε0
B E (c) X-rays are used to detect fracture Here, µ 0 = 4 π × 10−7 Ns 2 C – 2 is
As electromagnetic waves contains both of bones.
electric field and magnetic field. It carry (d) Ultraviolet waves are absorbed permeability (constant) of free space.
both energy and momentum according by ozone. ε0 = 8.85418 × 10−12 C2 N –1 m – 2 is the
to de-Broglie wave particle duality of
15 As, we know energy liberated, permittivity of free space. On substituting
radiations.
hc the values in Eq. (i), we have
8 Here, E 0 = 18 Vm –1 E =
λ 1
E0 18 c =
∴ B0 = = i.e. E ∝
1 4 π × 10 × 8.85418 × 10−12
−7
c 3 × 108 λ
= 2.99792 × 108 ms –1
= 6 × 10−8 T So, lesser the wavelength, than greater
will be energy liberated by This is same as the speed of light in
9 E = B × c keep value of electric field electromagnetic radiations per vacuum. From this we conclude that
⋅ c | = 20 × 10−9 × 3 × 108
|E| = |B|| quantum. light is an electromagnetic wave.
= 6 V/m As, order of wavelength is given by
20 The frequency of radio waves less than
X-ray, VIBGYOR, Radio waves the frequency of microwaves.
10 Total average energy = ε0 E 2rms (C) (A) (B) (D) Q Frequency of radio waves = 3 × 108 Hz
= 8.85 × 10−12 × (720)2 ∴Order of increasing energy of and frequency of microwaves = 1010 Hz
= 4.58 × 10−6 Jm −3 electromagnetic radiations per
∴ ν radio waves < ν microwaves
quantum.
11 Initial momentum of surface, p i = E /c ⇒ D<B<A<C
SESSION 2
where, c = velocity of light (constant). 16 Ultraviolet radiations are
Since, the surface is perfectly reflecting, 1 Given, capacitance of capacitor, C = 2µF
electromagnetic waves. The wavelength
so the same momentum will be ° Displacement current, Id = 1 mA
of these waves ranges between 4000 A Charge, q = CV
reflected completely. Final momentum, °
p f = E /c [negative value] to 100 A, i.e. of smaller wavelength and Id dt = CdV [Q q = It ]
∴ Change in momentum, higher frequency. They are absorbed by dV
or Id = C
∆ p = p f − pi ozone layer of stratosphere in dt
atmosphere. They cause skin diseases dV
=−
E

E
=−
2E 1 × 10−3 = 2 × 10−6 ×
and they are harmful to eye and may dt
c c c
cause permanent blindness. dV 1 +3
Thus, momentum transferred to the or = × 10 = 500 V/s
surface is 17 Earth is heated by sun’s infrared dt 2
2E radiation.The earth also emits radiation Clearly, by applying a varying potential
∆ p ′ = |∆ p| = most in infrared region. These
c difference of 500 V/s, we would produce
radiations are reflected back by heavy a displacement current of desired value.
12 In electromagnetic wave, the direction gases like CO2 in atmosphere. These
of propagation of wave, electric field back radiation keep the earth’s surface 2 A time varying magnetic field produces
and magnetic field are mutually warm at night. This phenomenon is an electric field. The magnitude of the
perpendicular, i.e. wave propagates called greenhouse effect. When the electric field at a distance r from the
perpendicular to E and B or along E×B. atmosphere were absent, then centre of a circular region of radius a,
While polarisation of wave takes place where a time varying field B exists, is
temperature of the earth falls.
given by
parallel to electric field vector.
18 Electromagnetic waves have linear a2 dB
E =
13 Here, in electromagnetic wave, the momentum as well as energy. From this 2r dt
electric field vector is given as we conclude that, we can exert At r = a,
E = (E1 $i+E2 $j )cos (kz − ωt ) radiation pressure by making a beam of
E =  
a dB
electromagnetic radiation fall on an  2  dt
In electromagnetic wave, the associated object. Let us assume that object is free
magnetic field vector, to move and that the radiation is This is the value of E at the edge of the
E E i$+E2 $j entirely absorbed in the object during circular region. For r > a, E decreases
B= = 1 cos(kz − ωt )
c c time interval ∆t. The object gains an with r.
Also, E and B are perpendicular to each energy ∆U from the radiation. Maxwell 3 Magnetic field, B = B 0 sin ω t .
other and the propagation of showed that the object also gains linear
Given equation,
electromagnetic wave is perpendicular momentum, the magnitude ∆p of the
change in momentum of the object is B = 12 × 10−8
to E as well as B, so the given
related to the energy change ∆U as sin (1.20 × 107 z − 3.60 × 1015t ) T
electromagnetic wave is plane
∆U
polarised. ∆p = (total absorption)
c
DAY TWENTY FIVE ELECTROMAGNETIC WAVES 295

On comparing this equation with 9 Speed of progressive wave is given by,


6 Poynting vector, S = E × H
standard equation, we get ω
v =
B 0 = 12 × 10−8 = EH sin 90° = EH k
1000 W As electric field in air is,
The average intensity of the beam, Energy of lamp =
πr 2 2 πνz
I av =
1 B 20
⋅c E1 = E 01 x$ cos  − 2 πνt 
2 µ0 =
1000
Jm −2 s −1  c 
−8 2 π × 22 2 πν
1 (12 × 10 ) × 3 × 10 8
∴ Speed in air = =c
= × S represents energy flow per unit area  2 πν 
2 4 π × 10−7  
per second, we have  c 
= 1.7 W/m 2
1000 1
EH = = 79.61, Also, c = …(i)
π × 22 µ 0εr1 ε0
4 Intensity in terms of electric field,
1 E
U av = ε0E 20 = 377 In medium,
2 H E2 = E 02 x$ cos (2kz − kct )
E
Intensity in terms of magnetic field, EH × = 79.61 × 377 = 300159. kc c
H ∴ Speed in medium = =
1 B 20 2k 2
U av = ⇒ E = 300159 .
2µ0 c 1
Also, = …(ii)
= 173.25V/m 2 µ 0εr2 ε0
Now, taking the intensity in terms of Amplitude of electric field of radiation
electric field, is As medium is non-magnetic medium,
µ medium = µ air
1 E 0 = E 2 = 245.01 V/m
(U av ) electric field = ε0E 20 On dividing Eq. (i) by Eq. (ii), we have
2
1 7 Consider the LED as a point source of εr2 εr 1
= ε0(cB 0 )2 (Q E 0 = cB 0) 2= ⇒ 1 =
light. Let power of the LED is P. εr1 εr2 4
2
Intensity at r from the source,
1
But, c = P 10 In vacuum, ε0 = 1
µ 0ε0 I = …(i)
4 πr 2 In medium, ε = 4
1 1
∴ (U av ) electric field = ε 0 × B 20 As we know that , ∴ Refractive index,
2 µ 0ε0 1
I = ε0E 20c …(ii) ε 4
1 B 20 2 µ = = =2
= ⋅ ε0 1
2 µ0 From Eqs.(i) and (ii), we can write
P 1 Wavelength,
Thus, the energy in electromagnetic = ε0E 20c
4 πr 2 2 λ λ
wave is divided equally between λ′ = =
electric field vector and magnetic field 2P µ 2
or E 20 =
vector. Therefore, the ratio of 4 πε0r 2c and wave velocity,
contributions by the electric field and c
v = =
c Q µ = c 
magnetic field components to the 2 × 01
. × 9 × 109 µ  v 
= 2
intensity of an electromagnetic wave 1 × 3 × 108 Hence, it is clear that wavelength and
is 1:1.
or E 20 = 6 velocity will become half, but frequency
ρ 104 10−4 remains unchanged when the wave is
5 As, I = = = ⇒ E0 = 6 = 2.45 V /m
2 πr 2
2 π (10 × 10 )
3 2
2π passing through any medium.

1 8 Given, λ = 4 cm 11 According to the Ampere-Maxwell’s law,


and I = ε0 E 20 c 4
2 for a closed surface,
∴ λ = 16 cm and T = 0.4 s
⇒ E0 =
2I
As, f λ × T = 2 π ∫ B⋅dl = µ 0I D
ε0c 2π 5 π −1
⇒ f = = s As, B (2πR ) = µ 0 I D
10−4 16 × 0.4 16 µ I
= ⇒ B= 0 D
π × 8.85 × 10−12 × 3 × 108 5π 2 πR
Now, v = f λ= × 16 = 5π cm/s
16
. NC −1
or E 0 = 010
296 40 DAYS ~ JEE MAIN PHYSICS DAY TWENTY SIX

DAY TWENTY SIX

Unit Test 5
(Magnetostatic, EMI & AC, EM Wave)
1 A solenoid of some length and radius 2 cm has a layer of
winding. A 2 cm long wire of mass 5 g lies inside the 5 A thin circular ring of area A is held perpendicular to a
uniform magnetic field of induction B. A small cut is made
solenoid along the axis of solenoid. The wire is
in the ring and a galvanometer is connected across the
connected to some external circuit, so that a current of
ends such that the total resistance of the circuit is R.
5 A flows through the wire. The value of current to be in When the ring is suddenly squeezed to zero area, the
the winding, so that magnetic force supports the wire charge flowing through the galvanometer is
weight is ( take , g = 10 ms − 2 ) BR AB B 2A
(a) (b) (c) ABR (d)
(a) zero (b) 400A
• A R R2
(c) 32000A
• (d) Not possible
6 Two infinite long current carrying wires A and B are placed
2 A circular coil of 20 turns and radius 10 cm is placed in a as shown in figure. Each wire carries same current I. The
uniform magnetic field of 0.1 T normal to the plane of the resultant magnetic field intensity at point P is
coil. The coil carries a current of 5 A. The coil is made up
P
of copper wire of cross-sectional area10−5 m 2 and the
number of free electrons per unit volume of copper is a
10 29. The average force experienced by an electron in
the coil due to magnetic field is A B
−25
(a) 5 × 10 N (b) zero I a a I
(c) 8 × 10−24 N (d) None of these 2 µ 0I
µ 0I
(a) (b)
3 A wire AB of length 5 m carrying a current of 2A is 2 πa 2 πa
placed in a region of uniform magnetic field B = 0.5 T as µ 0I µ 0I
(c) (d)
shown in figure. The magnetic force experienced by wire 2 2 πa 4 2 πa
is 7 An equilateral triangular loop is kept near to a current
B
B carrying long wire as shown in figure. Under the action of
magnetic force alone, the loop
5m 2A
Y
37°
A I
I a X
(a) 5 N (b) 4 N (c) 3 N (d) 8 N a
4 A coil of metal wire is kept stationary in a non-uniform
magnetic field, then
(a) an emf and current both induced in the coil (a) must move away from wire along X-axis
(b) a current but no emf is induced in the coil (b) must move towards wire along X-axis
(c) an emf but no current is induced in the coil (c) must move along Z-axis
(d) Neither emf nor current is induced in the coil (d) must move alongY-axis
DAY TWENTY SIX UNIT TEST 5 (MAGNETOSTATIC, EMI & AC, EM WAVE) 297

8 A current carrying loop is placed in the non-uniform 13 A conducting loop is placed in a magnetic field (uniform)
magnetic field whose variation in space is shown in as shown in the figure. For this situation mark the correct
figure. Direction of magnetic field is into the plane of statement.
paper. The magnetic force experienced by loop is

I R
×
I

B
(a) The force of compression experienced by loop is IRB
α α x (b) The force of compression experienced by loop is 2π IRB
(c) The force of expansion experienced by loop is IRB
(a) non-zero (b) zero
(d) The force of expansion experienced by loop is 2 π IRB
(c) can’t say anything (d) None of these
14 Mark the correct statement.
9 An electron is launched with velocity v in a uniform
magnetic field B. The angle θ between v and B lies (a) Ideal inductor does not dissipate power in an AC circuit
π (b) Ideal inductor dissipates maximum power in an AC circuit
between 0 and . Its velocity vector v returns to its initial (c) In an inductor, current lags behind the voltage by π
2
value in a time interval of (d) In inductor, current leads voltage by π
2 πm 15 Two very long straight parallel wires carry steady currents
(a)
eB I and − I, respectively. The distance between the wires is
2 × 2 πm
(b) d. At a certain instant of time, a point charge q is at a
eB
πm point equidistant from the wires in the plane of the wires.
(c)
eB Its instantaneous velocity v is perpendicular to this plane.
(d) depends upon angle between v and B The magnitude of the force due to the magnetic field
10 A straight conductor carrying a direct current of 3I is acting on the charge at this instant is
split into 2I and I as shown in the figure. µ 0 Iqv µ 0 Iqv 2 µ 0 Iqv
(a) (b) (c) (d) zero
2 πd πd πd
2I
16 An aeroplane is moving North horizontally, with a speed of
3I 3I 200 ms −1, at a place where the vertical component of the
r
earth’s magnetic field is 0.5 × 10 – 4 T . What is the induced
emf set up between the tips of the wings if they are 10 m
I apart?
Magnetic induction at centre of loop of radius r is (a) 0.01 V (b) 0.1 V (c) 1 V (d) 10 V
µ I 17 A square metal wire loop of side 10 cm and resistance 1
(a) 0 (b) 0
2 πr Ω is moved with a constant velocity v in a uniform
π I µ I magnetic field B = 2 T as shown in the figure. The
(c) 0 (d) 0
2r 4r magnetic field is perpendicular to the plane of the loop
11 A wire in the form of a circular loop of radius r lies with its and directed into the paper. The loop is connected to a
plane normal to a magnetic field B. If the wire is pulled to network of resistors, each equal to 3 Ω. What should be
take a square shape in the same plane in time t, the emf the speed of the loop, so as to have a steady current of
induced in the loop is given by 1 mA in the loop?
πBr 2 1 − π  πBr 2 1 − π  B (into page)
(a)   (b)  
t  10  t  8 × × ×
Q
πBr 2 1 − π  πBr 2 1 − π  × × × –v
(c)   (d)   3Ω 3Ω
t  6 t  4 × × × 3Ω
P R
12 The mutual inductance between two planar concentric × × ×
3Ω 3Ω
rings of radii r1 and r2 (with r1 > r2) placed in air is given by × × ×
S
µ πr 2
µ 0 πr12 × × ×
(a) 0 2 (b)
2 r1 2r2 Metal loop
µ 0 π (r1 + r2 ) 2 µ 0 π (r1 + r2 ) 2
(c) (d) (a) 1 cm s −1 (b) 2 cm s −1 (c) 3 cm s −1 (d) 4 cm s −1
2 r1 2 r2
298 40 DAYS ~ JEE MAIN PHYSICS DAY TWENTY SIX

18 At a certain place, the horizontal component of earth’s 25 The resonant frequency and Q-factor of a series L-C-R
magnetic field is 3 times the vertical component. The circuit with L = 3.0 H, C = 27 µF and R = 7.4 Ω. How will
angle of dip at that place is you improve the sharpness of resonance of the circuit by
(a) 30° (b) 45° (c) 60° (d) 90° reducing its full width at half maximum by a factor of 2?
19 A magnetic dipole is acted upon by two magnetic fields (a) Resistance of circuit should be increased
which are inclined to each other at an angle of 75°. One (b) Resistance of the circuit remain same
of the fields has a magnitude of 2 × 10−2 T. The dipole (c) Resistance of circuit should be increased by 3.7 Ω
(d) Resistance of the circuit should be reduced to 3.7 Ω
attains stable equilibrium at an angle of 30° with this
field. What is the magnitude of the other fields? 26 About 5% of the power of a 100 W light bulb is converted
(a) 0.01 T (b) 0.02 T (c) 0.03 T (d) 0.04 T to visible radiation. What is the average intensity of visible
radiation
20 A charge of 4 µC is placed on a small conducting sphere
(i) at a distance of 1 m from the bulb?
that is located at the end of thin insulating rod of length
0.5 m. The rod rotates in horizontal plane with a constant (ii) at a distance of 10 m?
angular velocity of 100 rads −1 about a vertical axis that Assume that the radiation is emitted isotropically and
passes through its other end. The magnetic moment of the neglect reflection.
rotating charge is (a) 0.2 Wm−2 , 0.002 Wm−2 (b) 0.6 Wm−2 , 0.006 Wm−2
(c) 0.3 Wm−2 , 0.003 Wm−2 (d) 0.4 Wm−2 , 0.004 Wm−2
(a) zero
(b) 0.5 × 10−4 Am2 27 A charged particle oscillates about its mean equilibrium
(c) 1.25 × 10−4 Am2 position with a frequency of109Hz. Frequency of the
(d) magnetic moment is not defined for this case electromagnetic waves produced by the oscillator is
21 A parallel plate capacitor is moving with a velocity of (a) 10 Hz (b) 105 Hz (c) 109 Hz (d) 1010 Hz
25 ms −1 through a uniform magnetic field of 1.5 T as
28 The amplitude of the magnetic field part of a
shown in figure. If the electric field within the capacitor
electromagnetic wave in vacuum is B0 = 510 nT. What is
plates is 175 NC −1 and plate area is 25 × 10−7 m 2, then the
the amplitude of the electric field part of the wave?
magnetic force experienced by positive charge plate is
(a) 140 NC −1 (b) 153 NC −1
(c) 163 NC −1 (d) 133 NC −1
29 Match the following column I with column II.
B
Column I Column I
A. Capacitor 1. increases AC
v
−13
B. Inductor 2. reduces AC
(a) 1.45 × 10 N (b) zero
C. Energy dissipation is 3. is conductor for DC
(c) 8.67 × 10−15 N (d) 3.87 × 10−15 N
due to
22 In an AC circuit, the potential difference V and current I D. A transformer 4. resistance only
are given respectively by V = 100 sin (100 t ) V and
 π Code
I = 100 sin 100 t +  mA. The power dissipated in the
 3 A B C D
circuit will be (a) 2 2, 3 4 1, 2
(a) 104 W (b) 10 W (c) 2.5 W (d) 5 W (b) 4 3,4 2 2,3
(c) 1 2,3 4 2
23 An inductor L, a capacitor of 20 µF and a resistor of 10 Ω
(d) 3 2 4 1
are connected in series with an AC source of frequency
50 Hz. If the current is in phase with the voltage, then the 30 Match the following of column I with column II.
inductance of the inductor is Column I Column II
(a) 2.00 H (b) 0.51 H (c) 1.5 H (d) 0.99 H q
24 An AC circuit having an inductor and a resistor in series
A. Lorentz force 1.
∫ E.dA = ε0
draws a power of 560 W from an AC source marked B. Gauss’s law 2. µ 0 IdI × r
dB =
210 V-60 Hz. The power factor of the circuit is 0.8, the 4π r3
impedance of the circuit and the inductance of the
C. Biot-Savart law 3. F = q [E + ( v × B )]
inductor is
D. Coulomb’s law 4. 1 q 1q 2
(a) 65 Ω, 0.2 H (b) 64 Ω, 1.0 H F =
4 πε0 r 2
(c) 63 Ω, 0.1 H (d) 50 Ω, 1.5 H
DAY TWENTY SIX UNIT TEST 5 (MAGNETOSTATIC, EMI & AC, EM WAVE) 299

Code (b) Statement I is true, Statement II is true; Statement II is


A B C D A B C D not the correct explanation for Statement I
(a) 3 1 2 4 (b) 1 2 3 4 (c) Statement I is true; Statement II is false
(d) Statement I is false; Statement II is true
(c) 4 3 2 1 (d) 2 1 4 3
31 Match the following of column I with column II. 34 Statement I A flexible wire loop of irregular shape
carrying current when placed in a uniform external
Column I Column II magnetic field acquires circular shape.
A. Ultraviolet 1. Radar system Statement II Any current carrying loop when placed in
B. Infrared 2. Roengten external magnetic field tries to acquires minimum energy
C. X-rays 3. Heat radiation and hence maximum magnetic flux and for a given
D. Microwaves 4. Water purification
perimeter circular shape is having greatest area.
35 Statement I A hanging spring is attached to a battery
Code
and switch. On closing the switch a current suddenly
A B C D A B C D flows in the spring, as a result spring compresses.
(a) 4 3 2 1 (b) 1 2 3 4
Statement II When two current carrying coils are placed
(c) 2 1 4 3 (d) 3 2 4 1 close to each other in same plane, then they attract each
32 Assertion (A) Two identical heaters are connected to two other if sense of current is same in both.
different sources one DC and other AC having same 36 Statement I No current is induced in a metal loop if it is
potential difference across their terminals. The heat rotated in an electric field.
produced in heater supplied with DC source is greater. Statement II The electric flux through the loop does not
Reason (R) The net impedance of an AC source is change with time.
greater than resistance.
37 Statement I The power factor of an inductor is zero.
(a) Both Assertion and Reason are true and Reason is the
Statement II In the inductor, the emf and current differ in
correct explanation of Assertion
π
(b) Both Assertion and Reason are false phase by .
2
(c) Assertion is true but Reason is false
(d) Both Assertion and Reason are true but Reason is not 38 Statement I In a series R-L-C circuit the voltage
true explanation of Assertion across-resistor, inductor and capacitor are 8 V, 16 V and
10 V respectively. The resultant emf in the circuit is 10 V.
33 Assertion (A) Induction coils are made of copper.
Statement II Resultant emf of the circuit is given by the
Reason (B) Induced current is more in wire having less
relation E = VR2 + (VL − VC ) 2 .
resistance
(a) Both Assertion and Reason are true and Reason is the 39 Statement I X-rays travel faster than light waves in
correct explanation of Assertion vacuum.
Statement II The energy of X-rays photon is greater than
(b) Both Assertion and Reason are false the light photon.
(c) Assertion is true but Reason is false
40 Statement I An electron moving in the positive
(d) Both Assertion and Reason are true but Reason is not
true explanation of Assertion x-direction enters a region where uniform electric and
magnetic fields exist perpendicular to each other. The
Direction (Q. Nos. 34-40) Each of these questions contains electric field is in the negative y-direction. If the electron
two statements : Statement I and Statement II. Each of these travels undeflected in this region, the direction of the
questions also has four alternative choices, only one of which magnetic field is along the negative z-axis.
is the correct answer. You have to select one of the codes (a),
Statement II If a charged particle moves in a direction
(b), (c), (d) given below
perpendicular to a magnetic field, the direction of the
(a) Statement I is true, Statement II is true; Statement II is
force acting on it is given by Fleming’s left hand rule.
the correct explanation for Statement I

ANSWERS
1. (d) 2. (a) 3. (c) 4. (d) 5. (b) 6. (a) 7. (a) 8. (b) 9. (a) 10. (d)
11. (d) 12. (a) 13. (d) 14. (a) 15. (d) 16. (b) 17. (b) 18. (a) 19. (a) 20. (b)
21. (a) 22. (c) 23. (b) 24. (c) 25. (d) 26. (d) 27. (c) 28. (b) 29. (a) 30. (a)
31. (a) 32. (d) 33. (a) 34. (a) 35. (a) 36. (c) 37. (a) 38. (a) 39. (d) 40. (b)
300 40 DAYS ~ JEE MAIN PHYSICS DAY TWENTY SIX

Hints and Explanations


1 Whatever be the current through 8 Each and every pair of loop elements 1
= − E 0I 0 sin ωt cos ωt = − E 0I 0 sin 2 ωt
solenoid winding, the direction of located symmetrically w.r.t. central line 2
magnetic field is along the axis of experiences zero net force. So, total Average power for one complete cycle is
solenoid and hence the magnetic force magnetic force experienced by loop is
1  1  T
experienced by wire is zero and hence
zero. P =  − E 0i 0 
T  2  ∫ 0 sin 2ωt dt =0
its weight cannot be supported by 9 Time interval in which its velocity v
magnetic force. returns to its initial value is same as 15 Since, B and v are anti-parallel to each
time period of the particle, to execute other, angle between the two is zero or
2 Drift speed of electron, the circle.
I 5 180°.
vd = = Since, it does not depend upon θ. So, F = q v × B = qv B sin θ
neA 1029 × 1.6 × 10−19 × 10−5
2 πm For θ = 0, sin 0 = 0 ⇒ F = 0
−5 −1 time required = ⋅
= 3.125 × 10 ms eB
16 ∴
µ 2I
10 ∴B net = 0 − µ 0I = µ 0I
Average magnetic force experienced by e = Blv = 0.5 × 10– 4 × 10 × 200 = 0.1 V
each electron is, 4r 4r 4r
F = qvB 17 The network PQRS is a balanced
= 1.6 × 10−19 × 3.125 × 10−5 × 0.1 11 Induced emf Wheatstone’s bridge. Hence, the
Magnetic field × change in area
= 0.5 × 10−24 N = 5 × 10−25 N (e ) = resistance of 3 Ω between P and R is
Time
ineffective. The net resistance of the
B∆A
3 Length of wire AB, l = 5 m = network, therefore, is 3 Ω. Total
t resistance R = 3 Ω + 1 Ω = 4 Ω.
Current, i = 2 A, θ = 37° and B = 0.5 T Since, the circumference of the circular Now, induced emf is
Magnetic force on wire, F = iBl sinθ loop = 2πr, e = Blv = 2 × 01. × v = 02 . v.
= 2 × 0.5 × 5 × sin 37° = 3 N 2 πr π r 0.2 v
The side of the square loop = = ∴ Induced current I =
e
=
Change of flux 4 2 R 4
5 ∴Induced charge, q = Therefore,
Resistance Given, I = 1 × 10−3 A
2
φ f − φi  πr 2 π
∆ A = πr − 2
 = πr  1 − 
=  2   4 Hence, 1 × 10−3 =
0.2 v
R 4
B ( πr ) 
2
π
But final area = 0, therefore, φ f = 0. ∴ e = 1 − 
t  4 Which gives, v = 2 × 10−2 ms −1 = 2 cms –1
Numerically, φ i = BA. Therefore,
q = BA / R. 12 Magnetic field due to the larger coil at 18 ∴B H = B cos θ and B V = B sinθ
its centre is BV
µ 0I µ 0I Hence, = tan θ
6 B A = BB = B = BH
2π × 2 a 2r1 BV 1
Given, =
BA where, I is the current in the larger coil. BH 3
45° Flux through the inner coil is 1
Therefore, tan θ = , i.e. θ = 30°
µ I
φ = B × πr22 = 0 × πr22 3
2r1
19 Refer to figure. Let θ1 (= 30° ) be the angle
BB But φ = MI
between the magnetic moment vector m
µ πr 2
Therefore, M = 0 2 and the field vector B1 (= 1.5 × 10– 2 T) .
45° 2r1
Then, as shown in figure, the angle
a a 13 By right hand thumb between m and the other field B2 will be
rule force of expansion dT θ2 = 75° − 30° = 45°.
µ 0I
So, B = B 2A + B 2B = would act on it whose B1
2 πa m
magnitude is given by dθ
dT = I (dl × B ) R
30°
⇒ T = IB ∫ dl
N

7 The net force 45° B2


acting on loop F1 = 2 π IRB
would be along
X -axis 14 The instantaneous
(to determine voltage and current in an AC circuit S
F2
whether it is along containing an ideal inductance only are
π The field B1 exerts a torque τ1 = m × B1
positive or E = E 0 sin ωt , I = I 0 sin  ωt −  on the dipole and the field B2 exerts a
negative, X -axis  2
F torque τ2 = m × B2 , where m is the
π
calculation has to be carried out) as Pins = EI = E 0 I 0 sin ωt sin  ωt −  magnetic moment of the dipole.
shown in figure.  2
DAY TWENTY SIX UNIT TEST 5 (MAGNETOSTATIC, EMI & AC, EM WAVE) 301

Since, the dipole is in stable 1 1 Power P


or ωL = or L = 2 = =
equilibrium, the net torque τ (= τ1 + τ2 ) ωC ωC Area 4 πr 2
must be zero, i.e. the two torques must . × 50 s −1
Here, ω = 2 π f = 2 × 314 (i) Intensity =
5
be equal and opposite. In terms of 4 × 314
. ×1×1
= 314 s −1 and C = 20
magnitudes, we have
µF = 20 × 10−6 F = 0.4 Wm −2
mB1 sin θ1 = mB 2 sin θ2 5
B sin θ1 (ii) Intensity =
or B2 = 1 ∴L =
1
= 0.51 H 4 × 314
. × 10 × 10
sin θ2 (314 s −1 )2 × (20 × 10−6 F )
−2
= 0.004 Wm −2
2 × 10 × sin 30°
= = 0.01 T 24 The average power over a complete
sin 45° 27 Frequency of electromagnetic wave =
cycle is
Frequency of oscillation of charge.
20 A moving charge along a circle is P = E rms × I rms × cos φ
equivalent to a current carrying ring, where, cos φ is the power factor 28 E 0 = c o r E 0 = cB 0
whose current is given by P 560 B0
ω ∴ I rms = =
E rms × cos φ 210 × 0.8 V max I max
32 Heat (AC) = V rms I rms =
10 2
= A
3 Heat (DC) = V max I max
The impedance of the circuit is 33 Since, copper consists of a very small
E 210 V
Z = rms = = 63 Ω ohmic resistance so, inductance coils are
I rms (10 / 3) A made of copper. A large induced current
The power is consumed in R only. is produced in such.
q q
I = = Therefore,
T 2π / ω 560
34 Each section of irregular wire loop
P = (I rms )2 R or R = = 50.4 Ω experiences an outward force.
4 × 10−6 2
⇒ I = × 100 = 0.64 × 10−4 A  10 
2π   35 Opposite polarities are produced in the
 3
forces of spring loops.
Magnetic moment of rotating charge is Now, the impedance of an L - R circuit is
M = IA 36 A current is induced in a loop only if
⇒ M = 0.64 × 10−4 × π × (0.5)2 Z = R2 + (ωL )2 magnetic flux linked with the coil
= 0.5 × 10 −4 2
Am ∴ (ωL )2 = Z 2 − R2 = (63) 2 − (50.4) 2 changes.

21 Electric field in between the capacitor


= 1428.84 Ω 2 37 ∴cos φ = cos π = 0
2
plates is given by or ωL = 1428.84 = 37.8 Ω
q 38 The resultant emf in the L-C-R circuit is
E = 37.8 37.8
ε0 A ∴ L = = = 0.1 H given by
2 πf 2 × 314
. × 60
where, q is the charge on capacitor. E = V R2 + (V L − VC )2
q = ε0 A × E 25 Given, L = 3.0 H,
E = (8)2 + (16 − 10)2 = 64 + 36
= 8.85 × 10−12 × 25 × 10−7 × 175 C = 27 µF = 27 × 10−6 F
E = 10 V
= 3.87 × 10−15 C and R = 7.4 Ω
39 Since, X-rays are electromagnetic waves,
Magnetic force experienced by positive The resonant frequency is given by we know that electromagnetic wave
plate is, 1 1
ω0 = = travels with same velocity of light in
F m = qvB = 1.45 × 10−13 N in a direction LC 3.0 × (27 × 10−6 ) vacuum.
out of plane of paper. = 111 rads −1 Now, from the formula
hc
22 Voltage amplitude V 0 = 100 V, The Q-factor of the circuit is given by E =
ω L 111 × 3.0 λ
current amplitude I 0 = 100 mA Q = 0 = = 45 The wavelength of X-rays are small than
R 7.4
= 100 × 10−3 A light waves and energy is inversely
The “bandwidth” (the difference of proportional to the wavelength. Hence,
and phase difference between I and V is
π half-power frequencies is given by the energy of X-rays photon will be
φ = = 60°. ω2 − ω1 = R/L greater than light waves.
3
Smaller the value of (ω2 − ω1 ), sharper 40 Because electron has a negative charge,
Now, power dissipated is given by
is the resonance. To reduce (ω2 − ω1 ) by an electric field in the negative
V I
P = 0 0 cos φ a factor of 2, the resistance R should be y-direction will deflect it in the positive
2 halved that is, the resistance of the y-direction. It will travel undeflected if
100 × 100 × 10− 3 circuit should be reduced to 37 . Ω.
= × cos 60° = 2.5 W the magnetic field imparts an equal
2 deflection in the negative y-direction.
26 Power converted into visible radiation
23 In an L-C-R circuit, the current and the Since, the magnetic force is
5 perpendicular to the magnetic field and
voltage are in phase (φ = 0), when P = × 100 = 5W
100 the charge of electron is negative, the
1
ωL − Energy direction of the magnetic field (according
ω C =0 Intensity =
tan φ = Area × Time to Fleming’s left hand rule) should be
R along the negative z-direction.
EXAM BITES

This Pdf Is
Downloaded From
www.exambites.in

Visit www.exambites.in for


More Premium Stuffs,Latest
Books,Test Papers,Lectures etc.
jeeneetadda
jeeneetadda_official
jeeneetadda

VISIT NOW !!
DAY TWENTY SEVEN

Ray Optics
Learning & Revision for the Day
u Reflection of Light u Lens u Dispersion by a Prism
u Mirror formula u Total Internal Reflection (TIR) u Refraction Through a Prism
u Refraction of Light u Deviation by a Prism u Scattering of Light

Reflection of Light
The phenomena of bouncing back of light on striking a smooth surface is called
reflection of light.
l
According to the laws of reflection, (i) the incident ray, reflected ray and the normal
drawn on the reflecting surface at the point of incidence lie in the same plane and
(ii) the angle of incidence ∠i = angle of reflection ∠r.
l
Laws of reflection are true for reflection from a polished mirror or from an unpolished
surface or for diffused reflection.
l
Whenever reflection takes place from a denser medium, the reflected rays undergo a
phase change of π.

Reflection from a Plane Mirror


l
If a ray is incident on a plane mirror at an angle of incidence i, then it suffers a
deviation of (π − 2i) and for two inclined plane mirrors deviation is (360 °−2θ).
l
While keeping an object fixed, a plane mirror is rotated in its plane by an angle θ,
then the reflected ray rotates in the same direction by an angle 2θ. PREP
l
Focal length as well as the radius of curvature of a plane mirror is infinity. Power of a
plane mirror is zero.
MIRROR
Your Personal Preparation Indicator
l
If two plane mirrors are inclined to each other at an angle θ, the total number of
No. of Questions in Exercises (x)—

u

or  

images formed of an object kept between them, is n = − 1 , when it is odd. No. of Questions Attempted (y)—
θ
u
 θ 
u No. of Correct Questions (z)—
l
The minimum size of a plane mirror fixed on a wall of a room, so that a person at the (Without referring Explanations)
centre of the room may see the full image of the wall behind him, should be 1/3rd the
size of the wall. u Accuracy Level (z / y × 100)—
u Prep Level (z / x × 100)—

Reflection from a Spherical Mirror In order to expect good rank in JEE,


your Accuracy Level should be above
l
A spherical mirror is a part of a hollow sphere whose one surface is polished, so that 85 & Prep Level should be above 75.
it becomes reflecting. The other surface of the mirror is made opaque.
DAY TWENTY SEVEN RAY OPTICS 303

Image formed by a concave mirroris is virtual and erect,


Refractive Index
l

when the object is placed between the pole and the


principal focus of concave mirror. In all other cases, the For a given pair of media, the ratio of the sine of angle of
image formed is real and inverted one. incidence (i) to the sine of angle of refraction (r ) is a constant,
l
Image formed by a convex mirror is virtual, erect and which is called the refractive index of second medium, w.r.t.
diminished in size irrespective of the position of the object. first medium.
Moreover, image is formed in between the pole and the sin i n
Thus, = constant = n21 = 2
principal focus of the mirror. sin r n1
l
The focal length of a spherical mirror is half of its radius of This is also called Snell’s law.
R Refractive index is a unitless, dimensionless and a scalar
curvature, i.e. f = .
l

2 quantity.
l
The refractive index of a medium w.r.t. vacuum
Mirror Formula (or free space) is known as its absolute refractive index. It is
Let an object be placed at a distance u from the pole of a defined as the ratio of the speed of light in vacuum (c) to
mirror and its image is formed at a distance v from the pole. the speed of light in a given medium (v).
1 1 1 c
Then, according to mirror formula, + = ∴ n=
v u f v
l
The power of a mirror (in dioptre), is given as Value of absolute refractive index of a medium can be 1 or
more than 1, but never less than 1.
1
P= l
When light travels from one material medium to another,
f (in metre)
the ratio of the speed of light in the first medium to that in
l
If a thin object of height h is placed perpendicular to the the second medium is known as the relative refractive
principal axis of a mirror and the height of its image be h′, index of second medium, w.r.t. the first medium. Thus,
then the transverse or lateral magnification produced is v c / v2 n2
n21 = 1 = =
given by v2 c / v1 n1
h′ v f f −v
m= =− = = l
When light undergoing refraction through several media
h u f −u f finally enters the first medium itself, then
Negative sign of magnification means the inverted image n31
and positive sign means an erect image. n21 × n32 × n 13 = 1 or n32 =
n21
l
When a small sized object is placed along the principal
axis, then its longitudinal (or axial) magnification is given
l
When the object is in denser medium and the observer is in
by rarer medium, then real and apparent depth have the
Real depth
2 2 2 relationship, = n21
dv  v   f   f − v Apparent depth
Axial magnification = − =  =  = 
du  u   f − u  f   1 
i.e. real depth > apparent depth shift, y = h − h′ = 1 − h
 n21 

Refraction of Light where, h and h′ are real and apparent depths.


When light passes from one medium, say air, to another
medium, say glass, a part is reflected back into the first Refraction from a Spherical Surface
medium and the rest passes into the second medium. When it Let an object be placed in a medium of refractive index n1 at a
passes into the second medium, it either bends towards the distance u from the pole of a spherical surface of radius of
normal or away from the normal. curvature R and after refraction, its image is formed in a
This phenomenon is known as refraction. medium of refractive index n2 at a distance v, then
n2 n1 n2 − n1
− =
v u R
Rarer Denser i
i medium The relation is true for all surfaces, whether the image formed
medium
is real or virtual.
Denser Rarer
medium medium r
r
Lens
(a) (b) A lens is part of a transparent refracting medium bound by
Fig. (a) and (b) shows refraction of light two surfaces, with atleast one of the two surfaces being a
curved one. The curved surface may be spherical or
cylindrical.
304 40 DAYS ~ JEE MAIN PHYSICS DAY TWENTY SEVEN

1 1 1
The lens formula is given by − =
v u f
Lens Maker’s Formula
For a lens having surfaces with radii of curvature R1 and R2
For a thin object of height h placed perpendicular to the
respectively, its focal length is given by
principal axis at a distance u, if the height of image formed is h′,
then lateral or transverse magnification m is given by 1  1 1
P= = (n21 − 1)  − 
h′ v f f −v f  1
R R2
m= = = = n2
h u f +u f where, n21 = = refractive index of the lens material w.r.t.
n1
For a small sized object placed linearly along the principal axis,
its axial or longitudinal magnification is given by the surroundings.
2 2 2
dv  v   f   f − v
Cutting and Combination of a Lens
Axial magnification = − =  =  = 
du  u   f + u   f 
If a symmetrical convex f 2f 2f f f
lens of focal length f is
Silvering of Lens cut into two parts along
When one surface of a lens is silvered, it behaves as a mirror. its optic axis, then focal
1 2 1 length of each part
The focal length of silvered lens is = +
F fl fm (a plano-convex lens) is
2 f . However, if the two
where, f1 is focal length of plane convex lens and fm is focal (a) (b) (c) (d)
parts are joined as shown
length of corresponding mirror.
in the figure, the focal
In case of plano-convex lens length of the combination is again f .
l
When curved surface is silvered, then focal length of If a symmetrical convex lens of focal length f is cut into two
R parts along the principal axis, then the focal length of each
silvered lens is F =
2µ part remains unchanged, as f [Fig. f]. If these two parts are
joined with the curved ends on one side, the focal length of
f
the combination is [Fig. g]. But on joining the two parts in
+ 2
opposite sense, the net focal length becomes ∞ (or net power
= 0) (Fig. h).
F fl fm
f f f ∞
where, R = 2 fm or R = fl (µ − 1) 2
l
When plane surface is silvered, then
R R
F = and fl = , fm = ∞
2(µ − 1) (µ − 1)
f
+ (e) (f) (g) (h)
l
The equivalent focal length of co-axial combination of two
1 1 1 d
F fl fm lenses is given by = + −
F f1 f2 f1 f2
l
When double convex lens is silvered, then f1 f2
R R R
F = and fl = ⇒ fm = .
2(2µ − 1) 2(µ − 1) 2
d < f1 d < f2

O1 O2
+

d
F fl fm 1 1 1
l
If a number of lenses are in contact, then = + + ...
F f1 f2
Power of a Lens l
If two thin lenses of focal lengths f1 and f2 are in contact,
The power of a lens is mathematically given by the
1 1 1
reciprocal of its focal length, i.e. power P =
1 then their equivalent focal length = +
f (m) feq f 1 f2

SI unit of power is dioptre (D). Power of a converging lens is In terms of power, Peq = P1 + P2
positive and that of a diverging lens is negative.
DAY TWENTY SEVEN RAY OPTICS 305

Deviation produced by a A
Important Results
prism is,
l
Effective diameter of light transmitting area is called n n′
aperture. Intensity of image ∝ (Aperture)2 δ = i + i′ − A
δ
2 ⇒ r + r′ = A
 f  i i′
l
Relation between object and image speed, vi =   vo For grazing incidence r r′
 f + u
i = 90 °
l
Newton’s formula states, f 2 = x 1 x2 where, x 1 and x2 =
and grazing emergence
distance of object; and image from first and second principlal i′ = 90° B C
foci. This formula is also called thin lens formula.
For minimum deviation
f (aµ g − 1) (aµ g − 1) δ + A
If lens immersed in a liquid, then l = = sin  m 
 2 
l

fa (l µ g − 1)  aµ g  (i) i = i′ and r = r ′ (ii) µ =


 − 1 A
 aµ l  sin
2
In case of minimum deviation, ray is passing through prism
Total Internal Reflection (TIR) symmetrically.
When a ray of light goes from a denser to a rarer medium, it For maximum deviation (δ max ), i = 90 ° or i′ = 90 °
bends away from the normal. For a certain angle of incidence For thin prism, δ = (µ − 1) A
iC , the angle of refraction in rarer medium becomes 90°. The
angle iC is called the critical angle. Dispersion by a Prism
r Rarer Dispersion of light is the phenomenon of splitting of white
90° n1 light into its constituent colours on passing light through a
prism. This is because different colours have different
i ic i r wavelength and hence different refractive indices.
Angular dispersion = δ v − δ r = (nv − nr ) A
Denser where, n v and n r represent refractive index for violet and red
n2 lights.
TIR in a medium n − nr n + nr
Dispersive power, ω = v , where n = v is the
n1 1 n−1 2
sin iC = = mean refractive index.
n2 n21 By combining two prisms with angle A and A′ and refractive
For the angle of incidence greater than the critical angle (i > iC ) index n and n′ respectively, we can create conditions of
in the denser medium, the light ray is totally internally (n − 1) A
l
Dispersion without deviation when, A′ = −
reflected back into the denser medium itself. (n′ − 1)
l
Deviation without dispersion when,
Conditions for Total Internal Reflection  n − nr 
A′ = −  v A
The light ray should travel from the denser medium towards
 n′v − n′r 
l

the rarer medium.


l
The angle of incidence should be the greater than the critical
angle. Refraction Through a Prism
Common Examples of Total Internal Reflection A ray of light suffers two refractions at the two surfaces on
l
Looming An optical illusion in cold countries. passing through a prism. Angle of deviation through a prism
l
Mirage An optical illusion in deserts. δ = i + e − A. where, i is the angle of incidence, e is the
l
Brilliance of diamond Due to repeated internal reflections angle of emergence and A is the angle of prism.
diamond sparkles.
l
Optical fibre Each fibre consists of core and cladding. The
refractive index of core material is higher than that of
Scattering of Light
cladding. Light entering at small angle on one end undergoes Molecules of a medium after absorbing incoming light
repeated total internal reflections along the fibre and finally radiations, emit them in all directions. This phenomenon is
comes out. called scattering. According to Rayleigh, intensity of
1
scattered light ∝ 4 .
Deviation by a Prism λ
A prism is a homogeneous, transparent medium bounded by There are some phenomenon based on scattering
two plane surfaces inclined at an angle A with each other. l
Sky looks blue due to scattering.
These surfaces are called as refracting surfaces and the angle l
At the time of sunrise and sunset, sun looks reddish.
between them is called angle of prism A. l
Danger signals are made of red colour.
306 40 DAYS ~ JEE MAIN PHYSICS DAY TWENTY SEVEN

DAY PRACTICE SESSION 1

FOUNDATION QUESTIONS EXERCISE


1 To get three images of a single object, one should have 7 A ray of light is incident on the surface of separation of a
two plane mirrors at an angle of medium at an angle 45° and is refracted in the medium at
(a) 60° (b) 90° (c) 120° (d) 30° an angle 30°. What will be the speed of light in the
2 A source of light lies on the angle bisector of two plane medium?
mirrors inclined at angle θ. The values of θ, so that the . × 108 ms−1
(a) 196 (b) 2.12 × 108 ms−1
(c) 3.18 × 108 ms−1 (d) 3.33 × 108 ms−1
light reflected from one mirror does not reach the other
mirror will be 8 A beaker contains water upto a height h1 and kerosene of
(a) θ ≥ 120° (b) θ ≥ 90° height h2 above water so that the total height of (water +
(c) θ ≤ 120° (d) θ < 30° kerosene) is (h1 + h2 ). Refractive index of water is µ1 and
3 A beam of light composed of red and green rays is that of kerosene is µ 2. The apparent shift in the position of
incident obliquely at a point on the face of a rectangular the bottom of the beaker when viewed from above is
glass slab. When coming out of the opposite parallel ª AIEEE 2011
face, the red and green rays emerge from  1  1  1  1
(a)  1 −  h2 +  1 −  h1 (b)  1 +  h1 +  1 +  h2
(a) two points propagating in two different non-parallel  µ1   µ2   µ1   µ2 
directions  1  1  1  1
(c)  1 −  h1 +  1 −  h2 (d)  1 +  h2 −  1 +  h1
(b) two points propagating in two different parallel directions  µ1   µ2   µ1   µ2 
(c) one point propagating in two different directions
(d) one point propagating in the same direction 9 A printed page is pressed by a glass of water. The
refractive index of the glass and water is 1.5 and 1.33,
4 The optical path of a monochromatic light is same if it
respectively. If the thickness of the bottom of glass is
goes through 4.0 cm of glass or 4.5 cm of water. If the
1 cm and depth of water is 5 cm, how much the page will
refractive index of glass is 1.53, the refractive index of the
appear to be shifted if viewed from the top?
water is
ª JEE Main (Online) 2013
(a) 1.30 (b) 1.36
(c) 1.42 (d) 1.46 (a) 1.033 cm (b) 3.581 cm
(c) 1.3533 cm (d) 1.90 cm
5 On a hot summer night, the refractive index of air is
smallest near the ground and increases with height from 10 A fish looking up through the water sees the outside
world, contained in a circular horizon. If the refractive
the ground. When a light beam is directed horizontally as 4
it travels, the light beam ª JEE Main 2015 index of water is and the fish is 12 cm below the water
3
(a) becomes narrower surface, the radius of this circle in cm, is
(b) goes horizontally without any deflection 36
(c) bends downwards (a) 36 7 (b) (c) 36 5 (d) 4 5
7
(d) bends upwards
11 A green light is incident from the water to the air-water
6 A transparent solid cylinder rod has a refractive index of interface at the critical angle (θ ). Select correct statement.
2
. It is surrounded by air. A light ray is incident at the ª JEE Main 2014
3
(a) The spectrum of visible light whose frequency is more
mid-point of one end of the rod as shown in the figure.
than that of green light will come out to the medium
(b) The entire spectrum of visible light will come out of the
water at various angles to the normal
θ
(c) The entire spectrum of visible light come out of the water
at an angle of 90° to the normal
The incident angle θ for which the light ray grazes along (d) The spectrum of visible light whose frequency is less
the wall of the rod is than that of green light will come out to the medium
 3
(a) sin−1  
1
(b) sin−1   12 Light is incident from a medium into air at two possible
 2  2 
angles of incidence (a) 20° and (b) 40°. In the medium,
(c) sin−1 
2  −1  1 
 (d) sin   light travels 3.0 cm in 0.2 ns. The ray will
 3  3
ª JEE Main (Online) 2013
DAY TWENTY SEVEN RAY OPTICS 307

(a) suffer total internal reflection in both cases (a) and (b) 19 A biconvex lens of focal length f forms a circular image of
(b) suffer total internal reflection in case (b) only radius r of sun in the focal plane. Then, which option is
(c) have partial reflection and partial transmission in case (b) correct?
(d) have 100% transmission in case (a)
(a) πr 2 ∝ f
13 The graph between angle of deviation (δ) and angle of (b) πr 2 ∝ f 2
incidence (i) for a triangular prism is represented by (c) If lower half part is covered by black sheet, then area of
the image is equal to πr 2 / 2
δ δ (d) If f is doubled, intensity will increase
(a) (b)
20 In an optics experiments, with the position of the object
fixed, a student varies the position of a convex lens and
O i O i for each position, the screen is adjusted to get a clear
image of the object. A graph between the object distance
u and the image distance v, from the lens, is plotted
(c) δ (d) δ using the same scale for the two axes. A straight line
passing through the origin and making an angle of 45°
with the x-axis meets the experimental curve at P. The
O i O i coordinates of P will be ª AIEEE 2009

(b)  , 
14 The refractive index of glass is 1.520 for red light and f f
(a) (2f , 2f )
 2 2
1.525 for blue light. Let D1 and D2 be the angles of
(c) (f , f ) (d) (4f , 4f )
minimum deviation for the red and blue light respectively
in a prism of this glass. Then, 21 A student measures the focal length of a convex lens by
(a) D1 < D2 putting an object pin at a distance u from the lens and
(b) D1 = D2 measuring the distance v of the image pin. The graph
(c) D1 can be less than or greater than D2 depending upon between u and v plotted by the student should look like
the angle of prism v (cm) v (cm)
(d) D1 > D2
15 Two beams of red and violet colours are made to pass (a) (b)
separately through a prism (angle of the prism is 60°). In O u (cm) O u (cm)
the position of minimum deviation, the angle of refraction
v (cm) v (cm)
will be
(a) 30° for both the colours
(b) greater for the violet colour (c) (d)
(c) greater for the red colour O u (cm) O u (cm)
(d) equal but not 30° for both the colours
16 The maximum magnification that can be obtained with a 22 An object approaches a convergent lens from the left of
convex lens of focal length 2.5 cm is (The least distance the lens with a uniform speed 5 m/s and stops at the
of distinct vision is 25 cm) focus. The image
(a) 10 (b) 0.1 (c) 62.5 (d) 11 (a) moves away from the lens with a uniform speed 5 m/s
17 A wire mesh consisting of very small squares is viewed (b) moves away from the lens with a uniform acceleration
at a distance of 8 cm through a magnifying lens of focal (c) moves away from the lens with a non-uniform
length 10 cm, kept close to the eye. The magnification acceleration
produced by the lens is (d) moves towards the lens with a non-uniform acceleration
(a) 5 (b) 8 (c) 10 (d) 20 23 In an experiment to determine the focal length (f ) of a
18 When the distance between the object and the screen is concave mirror by the u-v method, a student places the
more than 4F, we can obtain image of an object on the object pin A on the principal axis at a distance x from the
screen for the two positions of a lens. It is called pole P. The student looks at the pin and inverted image
displacement method. In one case, the image is from a distance keeping his/her eye in line with PA. When
magnified and in the other case it is diminished. Then, the student shifts his/her eye towards left, the image
the ratio of the size of image to the diminished image is appears to the right of the object pin. Then,
(D + d )2 D D2 D+d (a) x < f (b) f < x < 2f
(a) (b) (c) (d) (c) x = 2f (d) x > 2f
(D − d ) 2
d d 2
D −d
308 40 DAYS ~ JEE MAIN PHYSICS DAY TWENTY SEVEN

24 The image of an illuminated square is obtained on a 31 An equiconvex lens is cut into two Y
screen with the help of a converging lens. The distance of halves along (i) XOX ′ and
the square from the lens is 40 cm. The area of the image (ii) alongYOY ′ as shown in figure.
is 9 times that of the square. The focal length of the lens Let f , f ′ and f ′′ be the focal lengths X′ X
is ª JEE Main (Online) 2013 O
of the complete lens, of each half in
(a) 36 cm (b) 27 cm case (i), and of each half in case (ii)
(c) 60 cm (d) 30 cm respectively. Choose the correct
Y′
25 Which one of the following spherical lenses does not statement from the following.
exhibit dispersion? The radii of curvature of the surfaces of (a) f ′ = 2f , f ′′ = f (b) f ′ = f , f ′′ = f
the lenses are as given in the diagrams. (c) f ′ = 2f , f ′′ = 2f (d) f ′ = f , f ′′ = 2f

R1 R2 R ∞
Direction (Q. Nos. 32-36) Each of these questions contains
(a) (b) two statements : Statement I and Statement II. Each of these
questions also has four alternative choices, only one of which
is the correct answer. You have to select one of the codes (a),
(b), (c), (d) given below
(c) R R (d) R ∞ (a) Statement I is true, Statement II is true; Statement II is
the correct explanation for Statement I
(b) Statement I is true, Statement II is true; Statement II is
26 A double convex lens made of glass (refractive index not the correct explanation for Statement I
n = 1.5 ) has the radii of curvature of both the surfaces as
(c) Statement I is true; Statement II is false
20 cm. Incident light rays parallel to the axis of the lens
(d) Statement I is false; Statement II is true
will converge at a distance L such that
(a) L = 20 cm (b) L = 10 cm
32 Statement I The formula connecting u, v and f for a
20 spherical mirror is valid only for mirrors whose sizes are
(c) L = 40 cm (d) L = cm
3 very small as compared to their radii of curvature.
27 When monochromatic red light is used instead of blue Statement II Laws of reflection are strictly valid for plane
light in a convex lens, its focal length will ª AIEEE 2011 surfaces, but not for large spherical surfaces.
(a) not depend on colour of light 33 Statement I Endoscopy involves use of optical fibres to
(b) increase study internal organs.
(c) decrease
Statement II Optical fibres are based on the
(d) remain same
phenomenon of total internal reflection.
28 A concave lens and a convex lens have the same focal
length of 20 cm and both are kept in contact. The 34 Statement I The refractive index of diamond is 6 and
combination is used to view an object 5 cm long kept at a that of liquid is 3. If the light travels from diamond to the
distance of 20 cm from the lens combination. As liquid, it will be totally reflected when the angle of
compared to the object, the image will be incidence is 30°.
1
(a) magnified and inverted Statement II n = , where n is the refractive index of
(b) diminished and erect sin C
(c) of the same size and erect diamond with respect to liquid.
(d) of the same size and inverted
35 Statement I A double convex lens (n = 1.5) has a focal
29 To make an achromatic combination, a convex lens of length 10 cm. When the lens is immersed in water
focal length 42 cm having dispersive power of 0.14 is (n = 4 / 3), its focal length becomes 40 cm.
placed in contact with a concave lens of dispersive 1 n − nm  1 1
power 0.21. The focal length of the concave lens should Statement II = l  − .
f nm  R1 R 2 
be
(a) 63 cm (b) 21 cm 36 A thin air film is formed by putting the convex surface of a
(c) 42 cm (d) 14 cm plane-convex lens over a plane glass plate. With
30 A convex lens is in contact with a concave lens. The monochromatic light, this film gives an interference
3 pattern due to light reflected from the top (convex)
magnitude of the ratio of their focal length is . Their surface and the bottom (glass plate) surface of the film.
2
equivalent focal length is 30 cm. What are their individual Statement I When light reflects from the air-glass plate
focal lengths? interface, the reflected wave suffers a phase change of π.
(a) −75 , 50 (b) −10, 15 (c) 75 , 50 (d) −15 , 10 Statement II The centre of the interference pattern is
dark. ª AIEEE 2011
DAY TWENTY SEVEN RAY OPTICS 309

DAY PRACTICE SESSION 2

PROGRESSIVE QUESTIONS EXERCISE


1 Two plane mirrors are inclined at 90°. An object is placed 5 The speed at which the image of the luminous point
between them whose coordinates are (a, b ). The position object is moving, if the luminous point object is moving at
vectors of all the images formed is speed v 0 towards a spherical mirror, along its axis is
(Given, R = radius of curvature, u = object distance)
2
 R 
(a, b) (a) vi = − v 0 (b) vi = − v 0  
 2u − R 
2u − R   R 
(c) vi = − v 0   (d) vi = − v 0  
O  R   2u − R 
(a) − a $i − b $j, a $i + b $j, − a $i + b $j 6 Diameter of a plano-convex lens is 6 cm and thickness at
(b) − a $i + b $j, − a $j − b $j, a $i − b $j the centre is 3 mm. If speed of light in material of lens is
(c) a $i + b $j, − a $i − b $j, a $i − b $j 2 × 108 m/s, the focal length of lens
(d) None of the above (a) 15 cm (b) 20 cm (c) 30 cm (d) 10 cm
2 A small coin is resting on the bottom of a beaker filled 7 A plano-convex lens of refractive index 1.5 and radius of
with a liquid. A ray of light from the coin travels upto the curvature 30 cm is silvered at the curved surface. Now,
surface of the liquid and moves along its surface (see this lens has been used to form the image of an object. At
figure). what distance from this lens, an object be placed in order
3 cm to have a real image of the size of the object?
(a) 20 cm (b) 30 cm (c) 60 cm (d) 80 cm
8 Monochromatic light is incident on
4 cm

a glass prism of angle A. If the


θ A
refractive index of the material of
the prism is µ, a ray incident at an
Coin
angle θ, on the face AB would get
How fast is the light travelling in the liquid? transmitted through the face AC
. × 108 ms−1
(a) 18 (b) 2.4 × 108 ms−1
of the prism provided
(c) 3.0 × 108 ms−1 (d) 1.2 × 108 ms−1 ª JEE Main 2015
−1   1 
3 The reflective surface is given by y = 2 sin x and it is (a) θ > sin µ sin A − sin−1  
  µ 
facing positive axis. What is the least value of coordinate
  1 
of the point where a ray parallel to positive x-axis (b) θ < sin−1 µ sin A + sin−1  
becomes parallel to positive y-axis after reflection?   µ 
π π π π   1 
(a)  , 3  (b)  , 2  (c)  , 2  (d)  , 3  (c) θ > cos−1 µ sin A + sin−1  
3  2  3  4    µ 
  1 
4 A light ray is incident perpendicular to one face of a 90° (d) θ < cos−1 µ sin A − sin−1  
prism and is totally internally reflected at the glass-air   µ 
interface. If the angle of reflection is 45°, we conclude
 3
that for the refractive index n as 9 A thin convex lens made from crown glass µ =  has
 2
focal length f. When it is measured in two different liquid
4 5
having refractive indices and , it has focal lengths f1
3 3
45°
and f2, respectively. The correct relation between focal
length is ª JEE Main 2014
°
45

(a) f2 > f and f1 becomes one


1 1 (b) f1 and f2 both becomes one
(a) n < (b) n > 2 (c) n > (d) n < 2
2 2 (c) f1 = f2 < f
(d) f1 > f and f2 becomes one
310 40 DAYS ~ JEE MAIN PHYSICS DAY TWENTY SEVEN

10 A light ray falls on a square glass slab 45º 15 An object 2.4 m infront of a lens forms a sharp image on
as showin in the diagram. The index of Incident ray a film 12 cm behind the lens. A glass plate 1 cm thick,
refraction of the glass, if total internal of refractive index 1.50 is interposed between lens and
reflection is to occur at the vertical face, film with its plane faces parallel to film. At what distance
is equal to ª JEE Main 2013 Diagram (from lens) should object shifted to be in sharp focus on
film? ª AIEEE 2012
( 2 + 1) 5 3 3
(a) (b) (c) (d) (a) 7.2 m (b) 2.4 m (c) 3.2 m (d) 5.6 m
2 2 2 2
16 Let the zx-plane be the boundary between two
11 A spectrometer gives the following reading when used to
transparent media. Medium 1 in z ≥ 0 has a refractive
measure the angle of a prism.
index of 2 and medium 2 with z < 0 has a refractive index
Main scale reading : 58.5 degree of 3. A ray of light in medium 1 given by the vector
Vernier scale reading : 09 divisions A = 6 3 i + 8 3 j − 10 k is incident on the plane of
Given that 1 division on main scale corresponds to separation. The angle of refraction in medium 2 is
0.5 degree. Total divisions on the vernier scale is 30 and ª AIEEE 2011
match with 29 divisions of the main scale. The angle of the
(a) 45° (b) 60° (c) 75° (d) 30°
prism from the above data is ª AIEEE 2012
17 Two plane mirrors A and 2√3m
(a) 58.59° (b) 58.77°
(c) 58.65° (d) 59° B are aligned parallel to B
each other, as shown in
12 A car is fitted with a convex side-view mirror of focal

30°
the figure. A light ray is 0.2m
length 20 cm. A second car 2.8 m behind the first car is
incident at an angle 30° A
overtaking the first car at a relative speed of 15 m/s. The
at a point just inside one
speed of the image of the second car as seen in the
end of A. The plane of incidence coincides with the plane of
mirror of the first one is ª AIEEE 2011
1 1
the figure.
(a) m/s (b) 10 m/s (c) 15 m/s (d) m/s The maximum number of times the ray undergoes
15 10
reflections (including the first one) before it emerges out is
13 A diverging lens with magnitude of focal length 25 cm is
(a) 28 (b) 30 (c) 32 (d) 34
placed at a distance of 15 cm from a converging lens of
magnitude of focal length 20 cm. A beam of parallel light 18 A light ray is incident on a horizontal plane mirror at an
falls on the diverging lens. The final image formed is angle of 45°. At what angle should a second plane mirror
ª JEE Main 2017 (Offline) be placed in order that the reflected ray finally be
reflected horizontally from the second mirror as shown in
(a) virtual and at a distance of 40 cm from convergent lens
(b) real and at a distance of 40 cm from the divergent lens
figure, is
(c) real and at a distance of 6 cm from the convergent lens Q
G
(d) real and at a distance of 40 cm from convergent lens
C
14 In an experiment for determination of refractive index of A D
α
glass of a prism by i - δ, plot, it was found that a ray S α
incident at an angle 35° suffers a deviation of 40° and 45° 45° N
that it emerges at an angle 79°. In that case, which of the
following is closest to the maximum possible value of the P Q
B
refractive index? ª JEE Main 2016 (Offline)
(a) 1.5 (b) 1.6 (c) 1.7 (d) 1.8 (a) θ = 45 ° (b) θ = 60° (c) θ = 22.5 ° (d) θ = 15.3 °

ANSWERS
SESSION 1 1 (b) 2 (a) 3 (b) 4 (b) 5 (d) 6 (d) 7 (b) 8 (c) 9 (d) 10 (b)
11 (d) 12 (b) 13 (c) 14 (a) 15 (a) 16 (d) 17 (a) 18 (a) 19 (b) 20 (a)
21 (c) 22 (c) 23 (b) 24 (a) 25 (c) 26 (a) 27 (b) 28 (c) 29 (a) 30 (d)
31 (d) 32 (c) 33 (a) 34 (d) 35 (a) 36 (b)

1 (b) 2 (a) 3 (a) 4 (b) 5 (b) 6 (c) 7 (a) 8 (a) 9 (d) 10 (d)
SESSION 2
11 (c) 12 (a) 13 (d) 14 (a) 15 (d) 16 (a) 17 (b) 18 (c)
DAY TWENTY SEVEN RAY OPTICS 311

Hints and Explanations


SESSION 1 7 n=
sin i
=
c 11 m r < m g < m v
sin r v
1 Number of images Þ sin( q r ) > sin(q g ) > sin q v
c sin r
360° Hence, v = q r > q g > q v [q r , q g , q v are critical angles
n= -1 sin i
q of corresponding colours]. Thus, all
where, q is angle between mirrors. 3 ´ 108 ´ sin 30° colours from red to green will emerge out
=
360° sin 45° of water.
\ 3= - 1 or q = 90°
q
3 ´ 108 12 Speed of light in medium
=
2 n = 360 2 3 ´ 10-2
q 8
=
Number of images, N = n, which is odd . ´ 10 ms–1
= 212 0.2 ´ 10-9
= n-1 3
8 We know, apparent shift, = ´ 108 ms -1 = 1. 5 ´ 108 ms -1
2
For the given condition, no successive æ 1ö m2 v
reflection takes place. So, the number Dh = ç1 - ÷ h As, = 1
è mø m1 v2
of images will be N £ 2
\ Apparent shift produced by water, m 3 ´ 108
n-1£2 = Þ m=2
æ 1ö 1 1. 5 ´ 108
Dh1 = ç1 - ÷ h1
n£3 è m1 ø 1
We have, sinC =
360 and apparent shift produced by 2
£3
1
q kerosene, Þ C = sin -1 æç ö÷ = 30°
æ 1ö è2 ø
120 £ q Þ q ³ 120° Dh2 = ç1 - ÷ h2
è m2 ø When the value of incidence angle is
3 In any medium other than air or greater than critical angle than total
vacuum, the velocities of different \ Dh = Dh1 + Dh2
internal reflection take place.
colours are different. Therefore, both æ 1ö æ 1ö
= ç1 - ÷ h1 + ç1 - ÷ h2 In the second case we get total interval
red and green colours are refracted at è m1 ø è m2 ø reflection.
different angles of refractions.
9 Given, m = 1. 5, t 1 = 5 cm, 13 Angle of deviation depends upon the
Hence, after emerging from glass slab
m2 = 1.33 and t 2 = 1 cm angle of incidence. If we determine
through opposite parallel face, they
Change in path = Dt 1 + Dt 2 experimentally the angles of deviation
appear at two different points and move
corresponding to different angles of
in two different parallel directions. æ 1ö æ 1ö
= ç1 - ÷ ´ t 1 + ç1 - ÷ ´ t2 incidence, then the plot between i and d
è m1 ø è m2 ø that we will get is shown below
4 As optical paths are equal, hence
ng × x g = nw × x w æ 1 ö æ 1 ö
xg = ç1 - ÷ ´ 5+ ç1 - ÷´1 δ
Þ nw = ng × è 1. 5ø è 1.33 ø
xw
» 1.90 cm
4.0
= 1.53 ´ = 1.36 10 The situation is shown in figure. δm
4.5
5 A horizontal beam is travelling from a O
denser to a rarer medium, so it bends i
A R B
upwards (away from normal).
14 D = (n - 1) A
6 sin C = 3 θC
12 cm

…(i) θC For blue light n is greater than that for


2 red light, so D2 > D1 .

r
C O 15 At minimum deviation (d = d m )
θ r1 = r2 =
A 60°
= = 30°
1 2 2
sin qC =
1 m (For both colours)
sin r = sin (90° - C ) = cos C =
2 AB
tan qC = 16 Maximum magnification is obtained
sin q m2 OA
= when image is formed at near point of
sin r m1 eye. In that case,
\ AB = OA tan qC
2 1 OA 12 36 D 25
sin q = ´ or AB = = = m = 1+ = 1+ = 11
3 2 n2 - 1 2 7 f 2.5
4
æ ö -1
1 ç ÷
q = sin -1 æç ö÷ è3ø
è 3ø
312 40 DAYS ~ JEE MAIN PHYSICS DAY TWENTY SEVEN

17 1 = 1 - 1 …(i) 21 \ 1 - 1 = 1 = constant, = 0.5 ´


2
=
1
f v u v u f 20 20
which is a rectangular hyperbola. Þ f = 20 cm
Given, f = 10 cm
(as lens is converging) 22 When object would approach the lens Incident rays travelling parallel to the
then image would move away from the axis of lens will converge at its second
u = - 8 cm principal focus.
lens with non-uniform acceleration.
(as object is placed on left This can be seen from the fact that Hence, L = + 20 cm
side of the lens) when object is far away, image would
27 1 = (m - 1) ç 1 - 1 ÷
æ ö
On substituting these values in be formed at focus P. When object
f è R1 R2 ø
Eq. (i), we get approaches the lens then image would
move away such that when object Also, by Cauchy’s formula,
1 1 1 approaches to focus, then image would B C
= - m = A + 2 + 4 + ...
10 v - 8 approach to infinity. l l
1 1 1 1
Þ = - 23 Since, object and image move in Þ mµ
v 10 8 opposite directions, the positioning l
1 8 - 10 should be as shown in the figure. Object As l blue < l red Þ m blue > m red
Þ =
v 80 lies between focus and centre of Hence, f red > f blue
80 curvature f < x < 2 f .
\ v = = - 40 cm 28 When a concave lens is joined in contact
-2
with a convex lens of same focal length,
Hence, magnification produced by the the combination behaves as a glass plate
lens Image of infinite focal length or zero power.
v - 40 Object right Hence, the image of an object will be of
m= = =5
u -8 left the same size and erect.

18 In displacement method, the ratio of 29 For an achromatic combination, the


the diminished image to the object is 24 As magnification = 9 = v condition is
u
I2 D -d w1 w
m2 = = + 2 =0
O D +d f1 f2

and the ratio of image to the object is . , f1 = 42 cm


Here, w1 = 014
D +d 40 cm
Converging thus, we get f2 = - 63 cm
D-d
lens
m1 I O (D + d )2 30 Let focal length of convex lens is + f ,
Hence, = 1 ´ = \ v = 9 u = 9 ´ 40 = 360
m2 O I2 (D - d )2 1 1 1 then focal length of concave lens would
Now, - = 3
v u f be - f .
19 2
1 1 1
- = From the given condition,
f v -40 f 1 1 2 1
= - =
θ r 1 1 1 30 f 3f 3f
+ =
360 40 f
\ f = 10 cm
Þ é 1 + 1ù 1 = 1
r = f tan q or r µ f êë 9 úû 40 f Therefore, focal length of convex lens
= + 10 cm and that of concave lens
\ pr 2 µ f 2 10 1 1
Þ ´ = = - 15 cm.
20 It is possible when object kept at centre 9 40 f
31 (i) Cutting along YOY ¢
of curvature because then only position Þ f = 36 cm Focal length get doubled.
of object and image would be same. i.e.
25 1 = ( m - 1) ç 1 - 1 ÷ f ¢¢ = 2 f
æ ö
u = v [which is the point of intersection
f è R1 R2 ø (ii) Cutting along XOX ¢
between curve and straight line]
1 Focal length unchanged.
For no dispersion, =0
v f 32 Laws of reflection are valid for plane
or R1 = R2 = R. surfaces, irregular surface and curved
So, (c) is correct option. surface.

26 Here, n = 1.5, as per sign convention 33 Optical fibre consists of a very long and
(v) thin fibre of quartz glass. When a light
followed
u ray is incident at one end of the fibre
u (u > f ) R1 = + 20 cm and R2 = - 20 cm
making a small angle of incidence, it
1 æ 1 1 ö suffers refraction from air to quartz and
\ = (n - 1) ç - ÷
u=v f è R1 R2 ø strikes the fibre-layer interface at an
u = 2f , v = 2f angle of incidence greater than the
é 1 1 ù
= (1.5 - 1) ê - critical angle.
ë ( + 20) ( - 20)úû
DAY TWENTY SEVEN RAY OPTICS 313

It therefore, suffers total internal The centre of the circle is lying on m = tan 45° = 1
reflection and strikes its opposite meeting point of mirrors (i.e. O). The
(from law of reflection)
interface. At this interface also, the position of images from diagram is for
angle of incidence is greater than the S 1 , r1 = - a $i + b $j, r2 = - a $i - b $j, y = 2 sin x
critical angle. So, it again suffers total
r = a $i - b $j. dy
internal reflection. Thus, optical fibre is 3 \ m= = 2cos x = 1
based on total internal reflection. Hence, option (b) is true. dx
Endoscopy is a process for viewing 1
cos x =
internal organs of human body. This 2 As shown in figure, a light ray from the 2
process use a device endoscope which coin will not emerge out of liquid, if p
i > C. x=
is based on total internal reflection. 3
The corresponding value of
34 Refraction index of diamond w.r.t. A R B p
liquid y is 2 sin = 3
1 3
lnd =
i>C
sin C C 4 For total internal reflection from glass-air
6 1 h interface, critical angle C must be less
\ = than angle of incidence.
3 sin C
1
or sin C = = sin 45° Coin
2 A C
S
\ C = 45°
Therefore, minimum radius R 45°
For total internal reflection angle should
be greater than critical angle. corresponds to i = C . 45°
But here angle of incidence is lower In DSAB,
than critical angle, so total internal

°
45
reflection does not occur in light.
n B
35 In water, 1
1 æ n - nm ö æ 1 1 ö i.e. C < i
=ç l ÷ç - ÷ ...(i)
f w è n m ø è R1 R2 ø or C < 45° (QÐ i = 45° )
C 1 1
1 æ 1 1 ö
...(ii)
But n = Þ C = sin -1 æç ö÷
= (n l - 1) ç - ÷ sin C ènø
fa è R1 R2 ø n2 −1
fw ( n l - 1) (1.5 - 1)(4 / 3) R 1
= = = tan C or sin -1 æç ö÷ < 45°
fa æ nl - nm ö (1.5 - 4 / 3) h ènø
ç ÷ or R = h tan C
è nm ø 1
h or < sin 45°
0.5 ´ 4 / 3 (2 / 3) 6 or R= n
= = 2
3 4 1 n -1
- é ù 1
2 3 1 sin C or n>
êQ sin C = and tan C = ú sin 45°
\ f w = 4 f a = 4 ´ 10 = 40cm êë n 1 - sin2 C úû
1
Given, R = 3cm , h = 4cm or n>
36 Both statements I and II are correct but (1 / 2 )
statement II does not explain statement I. 3 1
Hence, = or n> 2
4 n2 - 1
SESSION 2 25 5 5 1+ 1=1
or n2 = or n= v u f
1 The images formed by combination of 9 3
two plane mirrors are lying on a circle Differentiating both sides
c 1 dv 1 du
whose radius is equal to OS. But n = - 2 = 2
v v dt u dt
Y c 3 ´ 108 dv v du
2
or v = = = 1.8 ´ 108 ms -1 = v i = - æç ö÷
n 5/ 3 dt è u ø dt
2
(– a, b) S1 S (a, b) 3 Let the incidence point is P ( x, y ) v
= - æç ö÷ v 0
n èu ø
θ θ 45°
90° – θ 90° – θ 1 1 1 2 1 2u - R
45° P Again, = - = - =
X v f u R u Ru
O 90° – θ
90° – θ
θ θ uR
v =
2u - R
S2 S3
(a, – b) 2
(– a, –b) v
2
æ R ö
v i = - æç ö÷ v 0 = - v 0 ç ÷
èu ø è 2u - R ø
314 40 DAYS ~ JEE MAIN PHYSICS DAY TWENTY SEVEN

6 i.e. O =I 8
From Eq. (ii), f1 =
v I æ 1 1 ö
\ m=- = ç - ÷
R u O è R1 R2 ø
v
Þ = -1 -10
R u From Eq. (iii), f2 =
3 mm æ 1 1 ö
or v = -u ç - ÷
Plano-convex lens è R1 R2 ø
Thus, from lens formula,
1 1 1
By Pythagoras theorem,
= - 10 From figure, we get r = sin -1 æç 1 ö÷
F v u è 3ø
R2 = 32 + (R - 3 mm )2 1 1 1
= -
Þ R2 = 32 + R2 - 2 R (3 mm ) 10 - u u 45º
+ (3 mm ) 2 1 2
=-
10 u
Þ R • 15 cm \ u = - 20 cm
c 3 ´ 108 3 r
Also, m= Þ m= = Hence, to get a real image, object must
v 2 ´ 108 2
be placed at a distance 20 cm on the left
1 æ 1 1 ö side of lens.
As, = (m - 1) ç - ÷ r
f è R1 R2 ø 90 – r
8 1 sin q = m sin r1 90 – r
1 æ3 1
Þ = ç - 1ö÷ é - 0ù r
f è2 ø êë 15 úû
Þ f = 30cm 1
A For critical angle, sinC =
m
7 A plano-convex lens behaves as a
concave mirror if its one surface θ Now, by Snell’s law, we have
(curved) is silvered. The rays refracted r2 m sin i sin 45°
= =
from plane surface are reflected from µ 1 sin r æ -1 æ 1 ö ö
sin çsin ç ÷ ÷
curved surface and again refract from è è 3 øø
plane surface. Therefore, in this lens m sin qc = 1sin 90° 1
two refractions and one reflection 1 3
occur.
sin qc =
m = 2, m =
1 2
Let the focal length of silvered lens is F .
r2 < qc 3
1 1 1 1 2 1
= + + = + sin r2 < sin qc
F f f fm f fm
r1 + r2 = A , r1 = A - r2 , r1 = A - qc 11 1 Vernier scale division = 29 main scale
30
where, f = focal length of lens before sin r1 > sin( A - qc ) division
silvering, °
Þ m sin r1 > m sin( A - qc ) 29 æ 29 ö
f m = focal length of spherical 1 VSD = ´ 0.5° = ç ÷
Þ sin q > m sin( A - qc ) 30 è 60 ø
mirror.
1 2 2 é æ 1 öù Thus, least count = 1 MSD - 1VSD
= + ...(i) q > sin -1 êm sin ç A - sin -1 ÷ ú 1
°
29
°
1
°
F f R ë è m øû = æç ö÷ - æç ö÷ = æç ö÷
è 2 ø è 60 ø è 60 ø
(QR = 2 f m )
9 1 = æç 3 - 1ö÷ ç 1 - 1 ÷
æ ö So, reading = main scale reading +
Now,
f è 2 ø è R1 R2 ø vernier scale reading
1 æ 1 1 ö
= (n - 1) ç - ÷ ...(ii) 1æ 1 1 ö = MSR + n ´ LC
f è R1 R2 ø = ç - ÷ …(i)
2 è R1 R2 ø 1 °
Here, R1 = ¥, R2 = 30 cm = 58.5°+9 ´ æç ö÷ = 58.65°
è 60 ø
1 1 1ö 1 3/2 æ 1 1 ö
\ = (1.5 - 1) æç - ÷ = æç - 1ö÷ ç - ÷
f è ¥ 30 ø f1 è 4 / 3 ø è R1 R2 ø 12 1 + 1 = 1 Þ -21 du - 12 dv = 0
u v f u dt v dt
1 0.5 1 1æ 1 1 ö
or = =- = - …(ii) dv - v 2 du
f 30 60
ç ÷
8 è R1 R2 ø = 2 æç ö÷
dt u è dt ø
or f = - 60 cm 1 3/2 æ 1 1 ö v f
= æç - 1ö÷ ç - ÷ But =
f2 è 5 / 3 ø è R1 R2 ø u u- f
Hence, from Eq. (i), we get
2
1 2 2 6 -1 æ 1 1 ö dv æ f ö æ du ö
= + = = ç - ÷ …(iii) \ = -ç ÷ ç ÷
F 60 30 60 10 è R1 R2 ø dt è u - f ø è dt ø
F = 10 cm 2 2
From Eq. (i), f = 0.2 ö ´ 15 = 1 ms -1
æ 1 1 ö = æç ÷
Again given that, ç - ÷ è -2.8 - 0.2 ø 15
è R1 R2 ø
size of object = size of image
DAY TWENTY SEVEN RAY OPTICS 315

A + d min ö
13 Focal length of diverging lens is sin æç ÷ 16 As refractive index for z > 0 and z £ 0 is
25 cm. è 2 ø different, xy-plane should be boundary
1.5 =
As the rays are coming parallel, so the sin A / 2 between two media.
image (I1 ) will be formed at the focus of 74° + d min ö Angle of incidence,
diverging lens i.e. at 25 cm towards left sin æç ÷
è 2 ø ½ ½
of diverging lens. Þ 1.5 = Az 1
sin 37° cos i = ½½ 2 ½=
2 2½ 2
A
½ x + A y + A z½
d
15 cm Þ 0.9 = sin æç37° + min ö÷
è 2 ø \ i = 60°
(Q sin 37° » 0.6 ) From Snell’s law,
æ d min ö sin i m2 3
sin 64° = sin ç37 °+ ÷ = =
è 2 ø sin r m1 2
I1
25 cm (Q sin 64° = 0.9) 2
v d min sin r = ´ sin 60°
°
37 + °
= 64 Þ d min » 54° 3
Now, the image (I1 ) will work as object 2
2 3
for converging lens. This angle is greater than the 40° = ´
3 2
For converging lens, distance of object u deviation angle already given. For
1
(i.e. distance of I1 ) greater m, deviation will be even higher. = = 45°
= - (25 + 15) = - 40 cm 2
Hence, m of the given prism should be
f = 20 cm Þ r = 45°
lesser than 1.5. Hence, the closest
1 1 1 0 .2
\ From len's formula = - option will be 1.5. 17 d = 0.2 tan 30° =
f v u 3
1 1 1 1 1 1 15 Shift in image position due to glass
= - Þ = - l = 2√3 m
20 v - 40 v 20 40 plate,
æ 1ö 1 ö d
1
=
1 S = ç1 - ÷ t = æç1 - ÷ ´ 1 cm
è mø è 1.5ø
v 90

30°
1 0.2 m

30°
Þ v = 40 cm = cm
v is positive so image will be real and 3
will form at right side of converging For focal length of the lens,
lens at 40 cm. 1 1 1 1 1
= - = - l 2 3
f v u 12 -240 = = 30
14 If m is refractive index of material of d 02
. / 3
1 20 + 1 240
prism, then from Snell’s law or = Þ f = cm
f 240 21 Therefore, maximum number of
sin i sin( A + d m )/ 2
m= = …(i) Now, to get back image on the film, lens reflections are 30.
sin r sin A / 2
has to form image at 18 From the figure CD = emergent ray, and
where, A is angle of prism and d m is æ12 - 1 ö cm = 35 cm such that the glass
minimum deviation through prism. ç ÷ CD is parallel to PQ and BC is a line
è 3ø 3 intersecting these parallel lines.
Given, i = 35° , d = 40 °, e = 79 °. plate will shift the image on the film. So, < DCB + < CBQ = 180°
So, angle of deviation by a glass prism, 1 1 1
As, = - Ð DCN + ÐNCB + ÐCBQ = 180°
d = i + e -A f v u a + a + 45° = 180°
Þ 40 ° = 35° + 79° - A 1 1 1 3 21 a = 67.5°
Þ = - = -
i.e. Angle of prism Þ A = 74°. u v f 35 240 But Ð NCS = 90°
Such that, r1 + r2 = A = 74°. 48 ´ 3 - 7 ´ 21 1 So, second mirror is at angle of 22.5°
= =-
Let us put m = 1.5 in Eq. (i), we get 1680 560 with horizontal.
Þ u = - 56. m
DAY TWENTY EIGHT

Optical
Instruments
Learning & Revision for the Day
u Microscope u Telescope u Resolving Power of an Optical Instrument

An optical instrument is used to enhance and analyses the light waves. The light waves are in
the form of photons, hence optical instruments also determine the characteristics properties of
light waves.

Microscope
It is an optical instrument which forms a magnified image of a small nearby object and thus,
increases the visual angle subtended by the image at the eye, so that the object is seen to be
bigger and distinct.

1. Simple Microscope (Magnifying Glass)


It consists of a single convex lens of small focal length and forms a magnified image of an
object placed between the optical centre and the principal focus of the lens.
A'

L
PREP
A''
A MIRROR
Your Personal Preparation Indicator
b
a F
B' No. of Questions in Exercises (x)—
F B C
u

u No. of Questions Attempted (y)—


u No. of Correct Questions (z)—
f
(Without referring Explanations)
 D
If the image is formed at the near point of eye, then m = 1 + 
 f u Accuracy Level (z / y × 100)—
u Prep Level (z / x × 100)—
D
But, if the image is formed at infinity, then m =
f In order to expect good rank in JEE,
where, D = normal viewing distance (25 cm), your Accuracy Level should be above
85 & Prep Level should be above 75.
f = focal length of magnifying lens.
DAY TWENTY EIGHT OPTICAL INSTRUMENTS 317

In normal adjustment, the final image is formed at infinity


2. Compound Microscope and magnifying power of the telescope is
It consists of two lenses of small focal length and small f
m=− o
apertures. Also, the focal length and aperture of objective lens fe
are smaller than that of eyepiece. The image formed by the
In practical adjustment, the final image is formed at the near
objective lens is real, inverted and magnified. This image acts
point of the observer’s eye. In this arrangement,
as the object for the eyepiece and the final image is highly
fo  f 
magnified, virtual and inverted w.r.t. the original object. m=− 1 + e 
fe  D
vo ue Eyepiece
uo fo
Reflecting Telescope
h F
q
It consists of an objective which is a large paraboloid
h'
concave mirror of maximum possible focal length fo and the
eyepiece is a convex lens of small focal length and small
Objective aperture, then
f
Magnifying power, m=− o
fe
Image
Reflecting type telescope is considered superior as it is free
from spherical and chromatic aberrations, is easy to install
If m o and m e be the magnifications produced by the objective and maintain, and can produce image of greater intensity.
and the eyepiece respectively, then total magnification of
microscope m = m o × m e . NOTE • The large aperture of telescope objective, helps in
forming a brighter image.
If final image is formed at the near point (D) of the eye, then
• If diameter of pupil of human eye is d and that of
vo  D telescope be D, then image formed by telescope will be
m =− 1 + 
 fe 
2
uo  D  times brighter than the image of the same object,
 
 d
L  D
or m=− 1 +  (approx) seen directly by the unaided eye.
fo  fe 
If final image is formed at infinity, then
v D L D
Resolving Power of an Optical
m = − o ⋅ = − ⋅ (approx)
uo fe fo fe Instrument
Length of tube of microscope, L = vo + ue . Resolving power of an optical instrument is its ability to
produce distinct images of two points of an object (or two
NOTE • Huygens’ eyepiece is free from chromatic and spherical nearby objects) very close together. Resolving power of an
aberration, but it cannot be used for measurement purposes. optical instrument is inverse of its limit of resolution.
• Ramsden’s eyepiece can be used for precise measurement as Smaller the limit of resolution of a device, higher is its
cross wires can be fixed in this eyepiece. It slightly suffers resolving power. Limit of resolution of a normal human eye
from spherical and chromatic aberrations. is 1′ .
The minimum distance (or angular distance) between two
points of an object whose images can be formed distinctly by
Telescope the lens of an optical instrument, is called its limit of
Telescope is an optical instrument which increases, the visual resolution.
angle at the eye by forming the image of a distant object at the
least distance of distinct vision, so that the object is seen Resolving Power of a Telescope
distinct and bigger.
If the aperture (diameter) of the telescope objective be the D,
then the minimum angular separation (dθ) between two
Refracting Telescope distant objects, whose images are just resolved by the
It consists of an objective lens of large focal length fo and large telescope, is
aperture. The eyepiece consists of a convex lens of small 1. 22 λ
dθ =
aperture and small focal length fe . D
and resolving power of the telescope,
Distance between the two lenses is set as,
1 D
L = fo + fe RP = =
dθ 1. 22 λ
318 40 DAYS ~ JEE MAIN PHYSICS DAY TWENTY EIGHT

θ = semi angle of the cone of light from the point object.


Resolving Power of a Microscope The term nm sin θ is generally called the numerical aperture of
The least distance (d) between two points, whose images are
the microscope.
just seen distinctly by a microscope, is given by
1. 22 λ ∴ Resolving power of the microscope,
d= 1
2 nm sin θ RP =
d
where, λ = wavelength of light used to illuminate the object, 2 nm sin θ nm sin θ
= =
nm = refractive index of the medium between the object and the 1. 22 λ 0.61 λ
objective lens, and

DAY PRACTICE SESSION 1

FOUNDATION QUESTIONS EXERCISE


1 To obtain a magnified image at the distance of distinct 7 An astronomical telescope is set for normal adjustment
vision with simple microscope, where should the object and the distance between its objective and eyepiece is
be placed? 1.05 cm. The magnifying power of the telescope is 20.
(a) Away from the focus What is the focal length of the objective?
(b) At focus (a) 2m (b) 1 m (c) 0.05 m (d) 0.25 m
(c) Between the focus and the optical centre
(d) None of the above 8 The magnifying power of a telescope is 9. When it is
adjusted for parallel rays, the distance between the
2 To obtain the maximum magnification with a simple
objective and the eyepiece is found to be 20 cm . The
microscope, where should the eye be placed?
focal length of lenses are
(a) Close to the lens
(a) 18 cm, 2 cm (b) 11 cm, 9 cm
(b) Half way between the focus and the optical centre
(c) 10 cm, 10 cm (d) 15 cm, 5 cm
(c) Close to the focus
(d) None of the above 9 A simple telescope, consisting of an objective of focal
3 A man can see clearly upto 3 m. To see upto 12 m, he length 60 cm and a single eye lens of focal length 5 cm is
must use a lens of focussed on a distant object in such a way that parallel
3 1 rays emerge from the eye lens. If the object subtends an
(a) − D (b) 3 D (c) −3 D (d) − D angle of 2° at the objective, the angular width of the
4 4
image is
4 A magnifying glass is used as the object to be viewed
(a) 10° (b) 24° (c) 50° (d) (1/6)°
can be brought closer to the eye than the normal near
point. This results in 10 In a laboratory four convex lenses L1, L2, L3 and L4 of focal
(a) the formation of virtual erect image and larger angle to lengths 2 , 4 , 6 and 8 cm respectively are available. Two of
be subtended by the object at the eye and hence these lenses form a telescope of length10 cm and
viewed in greater detail magnifying power 4. The objective and eye lenses are
(b) increase in field of view respectively,
(c) infinite magnification at near point (a) L2 ,L3 (b) L1,L4 (c) L1,L2 (d) L4 ,L1
(d) a diminished but clear image
11 The magnifying power of an astronomical telescope is 8
5 In a compound microscope, the objective produces a and the distance between the two lenses is 54 cm . The
magnification of 10, while the eyepiece produces a focal length of eye lens and objective lens will be
magnification of 5, then the over all magnification respectively
achieved by a compound microscope is (a) 6 cm and 48 cm (b) 48 cm and 6 cm
(a) 2 (b) 50 (c) 0.5 (d) 25.00 (c) 8 cm and 64 cm (d) 6 cm and 60 cm
6 A telescope consists of two thin lenses of focal lengths 12 In a refracting astronomical telescope, the final image is
0.3 m and 3 cm, respectively. It is focussed on moon (a) real, inverted and magnified
which subtends on angle of 0.5° at the objective. Then, (b) real, erect and magnified
the angle subtended at the eye by the final image will be (c) virtual, erect and magnified
(a) 5° (b) 0.25° (c) 0.5° (d) 0.35° (d) virtual, inverted and magnified
DAY TWENTY EIGHT OPTICAL INSTRUMENTS 319

13 The magnifying power of a telescope is high, if Direction (Q. Nos. 17-20) Each of these questions contains
(a) both the objective and eyepiece have short focal two statements Statement I and Statement II. Each of these
lengths
questions also has four alternative choices, only one of which
is the correct answer. You have to select one of the codes (a),
(b) both the objective and the eyepiece have long focal (b), (c), (d ) given below
lengths (a) Statement I is true, Statement II is true; Statement II is
(c) the objective has a short focal length and the eyepiece the correct explanation for Statement I
has a long focal length (b) Statement I is true, Statement II is true; Statement II is
(d) the objective has a long focal length and the eyepiece not the correct explanation for Statement I
has a short focal length (c) Statement I is true; Statement II is false
(d) Statement I is false; Statement II is true
14 An astronomical telescope has a large aperture to
(a) reduce spherical aberration 17 Statement I Very large size telescopes are reflecting
(b) have high resolution telescopes instead of refracting telescopes.
(c) increase span of observation Statement II It is easier to provide mechanical support to
(d) have low dispersion large size mirrors than large size lenses.
15 Two point white dots are 1 mm apart on a black paper. ª JEE Main (Online) 2013
They are viewed by eye of pupil diameter 3 mm.
18 Statement I The resolving power of a telescope is more if
Approximately, what is the maximum distance at which
the diameter of the objective lens is more.
these dots can be resolved by the eye?
Statement II Objective lens of large diameter collects
(take, wavelength of light = 500 nm) more light.
(a) 5 m (b) 1 m
(c) 6 m (d) 3 m 19 Statement I The focal length of the objective of telescope
is larger than that of eyepiece.
16 Wavelength of light used in an optical instrument are
λ 1 = 4000 Å and λ 2 = 5000 Å, then ratio of their Statement II The resolving power of telescope increases
when the aperture of objective is small.
respective resolving powers (corresponding to
λ 1 and λ 2) is 20 Statement I Resolving power of an optical instrument is
reciprocal to its limit of resolution.
(a) 16 : 25
(b) 9:1 Statement II Smaller the distance between two point
(c) 4:5 objects the instrument can resolve, higher is its resolving
(d) 5:4 power.

DAY PRACTICE SESSION 2

PROGRESSIVE QUESTIONS EXERCISE


1 An observer looks at a distance tree of height 10 m with 4 A telescope consists of two lenses of focal length 10 cm
a telescope of magnifying power of 20. To the observer and 1 cm. The length of the telescope when an object is
the tree appears ª JEE Main 2016 (Offline) kept at a distance of 60 cm from the objective and the
(a) 10 times taller (b) 10 times nearer final image is formed at least distant of distinct vision is
(c) 20 times taller (d) 20 times nearer (a) 15.05 cm
(b) 12.96 cm
2 Assuming human pupil to have a radius of 0.25 cm and a
(c) 13.63 cm
comfortable viewing distance of 25 cm, the minimum (d) 14.44 cm
separation between two objects that human eye can
5 A light source is placed at a distance b from a screen.
resolve at 500 nm wavelength is ª JEE Main 2015
The power of the lens required to obtain k-fold
(a) 30 µm (b) 100 µm (c) 300 µm (d) 1µm
magnified image is
3 For compound microscope fo = 1 cm, fe = 2.5 cm . An k+1 (k + 1)2
object is placed at distance 1. 2 cm from objective lens. (a) (b)
kb kb
What should be the length of microscope for normal kb kb
adjustment? (c) (d)
k+1 (k − 1)2
(a) 8.5 cm (b) 8 . 3 cm (c) 6.5 cm (d) 6. 3 cm
320 40 DAYS ~ JEE MAIN PHYSICS DAY TWENTY EIGHT

6 Two white dots are 1 mm apart on a black paper. They and 10 m. Angle subtended by the moon’s image on eye
are viewed by naked eye of pupil size 3 mm diameter. of observer will be
Upto what distance, the dots are clearly and separately (a) 15° (b) 20°
visible? (c) 30° (d) 35°
(a) 3 m (b) 6 m 10 Match the following column I with column II.
(c) 1 m (d) 5 m
Column I Column II
7 Resolution of human eye is about 1 min. From what
A. Magnification of simple microscope 1. − v 0  D
distance a normal human eye can just resolve two 1 + 
objects which are 3m apart. u0  fe 

(a) 10 km (b) 15 km (c) 20 km (d) 30 km B. Magnification of compound microscope 2.  D 


 
8 A student make a compound microscope by using two
when image formed at least distance of vision  1.22 λ 
lenses of focal lengths 1.5 cm and 6.25 cm. She kept an C. Tube length of telescope 3.  1 + D 
 
object at 2 cm from objective and forms its final image at  f
25 cm from eye lens. Distance between two lenses is D. Resolving power of a telescope 4. f0 + fe
(a) 6 cm (b) 7 cm
(c) 9 cm (d) 11 cm Codes
9 Orbital radius of moon is 3.8 × 10 km and its diameter is
5 A B C D A B C D
(a) 1 2 3 4 (b) 2 1 4 3
3.5 × 10 km.It is seen from a telescope with lenses of 4 m
2
(c) 3 1 4 2 (d) 3 2 4 1

ANSWERS

SESSION 1 1 (c) 2 (a) 3 (d) 4 (a) 5 (b) 6 (a) 7 (b) 8 (a) 9 (b) 10 (d)
11 (a) 12 (d) 13 (d) 14 (b) 15 (a) 16 (d) 17 (a) 18 (a) 19 (c) 20 (c)

SESSION 2 1 (c) 2 (a) 3 (b) 4 (b) 5 (b) 6 (d) 7 (a) 8 (d) 9 (b) 10 (c)

Hints and Explanations


SESSION 1 Hence, power of lens required, 8 fo + fe = 20 cm,
1 −1
1 Magnification is obtained in a simple P = = D m = fo / fe = 9.
f 4
microscope when the object is close to This gives
the lens between the focus and the 4 This results in the formation of virtual, fe = 2 cm and fo = 18 cm.
optical centre, as lens is converging. erect image and the object subtends a
2 The magnification of simple larger angle at the eye and the image is 9 It is a case of normal adjustment.
viewed in greater detail.
microscope is given by Hence, m = fo ⁄ fe
D−a 5 m = mo × me = 10 × 5 = 50 Also, m =β ⁄α
m = 1+
f β f
6 For a telescope, β = o
f Therefore, = o
α α fe
where, D = least distance of vision, fe
f = focal length and β 03
. Here, fo = 60 cm, fe = 5 cm,α = 2°
∴ = ⇒ β = 5°
a = distance of lens from eye 0.5° 0.03 Hence, β = 24° .
So, lesser the distance between eye 7. Here, m = fo ⁄ fe 10 Length of tube = 10 cm
and lens, greater is the magnification.
and L = fo + fe = 1.05 fo + fe = 10 cm …(i)
xy f
3 Using, f = or fe = 1.05 − fo Magnification, m = o = 4
x− y fe
and 20 = fo /(1.05 − fo )
3 × 12
⇒ f = = −4 m This gives fo = 1 m and fe = 0.05 m . fo = 4 fe
3 − 12
DAY TWENTY EIGHT OPTICAL INSTRUMENTS 321

On putting in Eq. (i), we get where, a is the diameter of objective 1.22 λ


We have the formula, sin θ =
5 fe = 10 cm lens and λ is the wavelength of light d
or fe = 2 cm used. It is obvious that on increasing a, 1.22 × 5 × 10−7
and fo = 8 cm more light is collected by objective lens ∴ sin θ =
0.5 × 10−2
Hence, L 4 and L1 will be used. and so, the image formed is more bright.
Thus, resolving power of telescope 1.22 × 5 × 10−7
Note In telescope, objective always have =
larger focal length than eyepiece. increases. 5 × 10−3
11 As L = fo + fe = 54 cm 19 The magnifying power of telescope in = 1.22 × 10−4
fo relaxed state is
and |m | = =8 f
The distance of comfortable viewing is
fe m= o D = 25 cm = 0.25 m
fe
On simplification, we get Let Y be the minimum separation
fe = + 6 cm So, for high magnification, the focal between two objects that human eye can
fo = + 48 cm length of objective length should be Y
and resolve i.e. sinθ =
larger than that of eyepiece. D
12 The image formed by the objective lens Resolving power of a telescope ∴ y = D sin θ = 0.25 × 1.22 × 10−4
is real, inverted and larger object and d
=
the eyepiece forms a second image but 1.22 λ = 3 × 10−5 m = 30 µm
virtual, inverted and larger than the
first. For high resolving power, diameter (d ) 3 When final image is formed at normal
of objective should be higher. adjustment, then length of compound
13 The magnifying power of a telescope (if microscope,
the object is at infinity) is given by 20 Resolving power of an optical
instrument is its ability to produce
uo fo fe D
f D + fe distinct image of two points of an object L = v o + ue = +
M = o . very close together. Resolving power of (uo + fo ) fe + D
fe D
an optical instrument is inverse of its − 1.2 × 1 2.5 × 25
where, D = least distance of distinct limit of resolution. Smaller the limit of = +
vision, where the final image is formed. − 1.2 + 1 2.5 + 25
resolution of a device higher is its
14 An astronomical telescopic as a large resolving power. = 6 + 2.27
aperture to have high resolution. = 8.27 ≈ 8.3 cm
SESSION 2
15 We know that, 4 Two lenses used are eyepiece and
y λ 1 Height of image depends upon the objective.
≥ 1.22
D d magnifying power to see a 20 times For eyepiece, fe = 1 cm,
yd taller object, as the angular
⇒ D≤ D = v e = 25cm
1.22 λ magnification should be 20 and we
observe angular magnification. Option 1 1 1
− =
10− 3 × 3 × 10− 3 ve ue fe
∴ D≤ = 5m (c) would not be very correct as the
1.22 × 5 × 10− 7 telescope can be adjusted to form the 1 1
⇒ − − =1
⇒ D max = 5m image anywhere between infinity and 25 ue
least distance for distinct vision. 25
16 Resolving power of an optical ⇒ ue = − cm
Suppose that the image is formed at 26
instrument is inversely proportional to infinity. Then, the observer will have to For objective uo = − 60 cm,
λ, i.e. focus the eyes at infinity to observe the
1 fo = 10 cm
RP ∝ image. Hence, it is incorrect to say that 1 1 1
λ the image will be appear nearer to the − =
v o uo fo
Resolving power at λ1 λ observer.
∴ = 2 1 1 1
Resolving power at λ 2 λ1 2 We can write resolving angle of naked + =
vo 60 10
5000 eye as
= ⇒
1
=
1

1
4000 v o 10 60
Eye lens
= 5: 4 1 5
⇒ =
Y vo 60
17 As very large size telescope needs θ
mechanical support to large size mirror 60
⇒ vo =
than size of lens. So, in order to fulfil 5
25cm
this mechanical support telescope is Retina = 12 cm
reflecting instead of refracting Length of telescope,
telescope. Assuming human pupil to have a radius
L = v o + ue
r = 025
. cm or diameter
18 Resolving power of telescope is 25
d = 2r = 2 × 025
. = 0.5 cm, the = 12 +
a 26
= wavelength of light λ = 500 nm
1.22λ = 5 × 10−7 m = 12.96 cm
322 40 DAYS ~ JEE MAIN PHYSICS DAY TWENTY EIGHT

5 Distance of light source from lens is x 0 . λ


122 x uo fo f D
6 Using, = = + e
and distance of screen from lens is a d uo − fo fe + D
(b − x 0 ). xa 2 × 1.5 6.25 × 25
⇒ d = = + = 11 cm
. λ
122 2 − 1.5 6.25 + 25
1 × 10−3 × 3 × 10−3
= ≈ 5m 9 Angle subtended by moon on the
. × 500 × 10−9
122 objective of telescope is,
Note In case, wavelength of light is not 3.5 × 103
given, we take mean value of visible α =
3.8 × 105
region ≈ 500 nm. 3.5
= × 10−2 rad
7 Using, θ = d 3.8
x0 b – x0 r f β
As, m = o =
b fe α
∴ Image is formed on screen, hence ∴ β=
400
×α
(b − x 0 ) is also the image distance. d 10
Image is formed on the screen, so m will θ = 40α = 40 ×
3.5
× 10−2 ×
180
≈ 20°
be negative. r 3.8 π
v
m= −k = d 10 A - Magnification of simple microscope
u We have, r =
θ is given by,
∴ v = − ku
 where, θ is in radians 
m =  1 + 
Here, u = − x 0, v = b − x 0 D
  1 ° 1 × π  
b  = 1′ =   =  F
∴ b − x0 = k x 0 , x0 =  60 
1+ k  60 180 
B - Magnification of compound
3
From lens formula, = microscope, when image is formed at
 1 × π least distance of vision is,
1 1 1
− =  
 60  180
v u f −v 0  D
3 × 60 × 180 m= 1 + 
1 1 = u0  fe 
− =p π
b − x0 − x0
≈ 10.3 km ≈ 10 km C - Tube length of telescope,
b
⇒ p= 8 As, fo < fe , for a compound microscope. L = f 0 + fe
x 0(b − x 0 )
So, fo = 1.5cm and fe = 6.25cm D - Resolving power of telescope
Putting the value of x 0, we have
Now, length of tube  D 
(k + 1)2 = 
p= = distance between lenses . λ
 122
kb
DAY TWENTY NINE

Wave Optics
Learning & Revision for the Day
u Wavefront u Coherent Sources u Polarisation of Light
u Interference of Light u Interference in Thin Films u Brewster’s Law
u Young’s Double Slit Experiment u Diffraction u Law of Malus
u Polaroids

According to Huygens’, light is a form of energy, which travels in the form of waves
through a hypothetical medium ‘ether’. The medium was supposed to be all pervading,
transparent, extremely light, perfectly elastic and an ideal fluid.
Light waves transmit energy as well as momentum and travel in the free space with a
constant speed of 3 × 10 8 ms −1. However, in a material medium, their speed varies from
medium to medium depending on the refractive index of the medium.

Wavefront
A wavefront is the locus of all those points (either particles) which are vibrating in the
same phase. The shape of the wavefront depends on the nature and dimension of the
source of light.
l
In an isotropic medium, for a point source of light, the wavefront is spherical in
nature.
l
For a line (slit) source of light, the wavefront is cylindrical in shape.
l
For a parallel beam of light, the wavefront is a plane wavefront.

Huygens’ Principle PREP


Every point on a given wavefront, acts as secondary source of light and emits secondary
wavelets which travel in all directions with the speed of light in the medium. A surface
MIRROR
Your Personal Preparation Indicator
touching all these secondary wavelets tangentially in the forward direction, gives the
new wavefront at that instant of time. u No. of Questions in Exercises (x)—
Laws of reflection and refraction can be determined by using Huygens’ principle. u No. of Questions Attempted (y)—
No. of Correct Questions (z)—
Interference of Light
u

(Without referring Explanations)


Interference of light is the phenomenon of redistribution of light energy in space when
two light waves of same frequency (or same wavelength) emitted by two coherent u Accuracy Level (z / y × 100)—
sources, travelling in a given direction, superimpose on each other. If a1 and a2 be the u Prep Level (z / x × 100)—
amplitudes of two light waves of same frequency and φ be the phase difference
between them, then the amplitude of resultant wave is given by In order to expect good rank in JEE,
your Accuracy Level should be above
85 & Prep Level should be above 75.
AR = a21 + a22 + 2 a1a2 cos φ
324 40 DAYS ~ JEE MAIN PHYSICS DAY TWENTY NINE

and in terms of intensity of light, xd λ


Case II If = (2n − 1) , then we get nth dark fringe.
I R = I 1 + I2 + 2 I 1I2 cos φ. D 2
Hence, for nth dark fringe,
Condition for Constructive Interference (2n − 1) Dλ
If at some point in space, the phase difference between x=
2d
two waves, φ = 0 ° or 2nπ or path difference between two
waves, ∆ = 0 or nλ, where n is an integer, then AR = a1 + a2 where, n = 1, 2, 3, …… .
or I R = I 1 + I2 + 2 I 1I2 is maximum. Such an interference is
called constructive interference. Fringe Width
Condition for Destructive Interference The separation between any two consecutive bright or dark
fringes is called fringe width β.
If at some point in space, the phase difference between two
λ Dλ
waves, φ = (2 n − 1)π or path difference, δ∆ = (2 n − 1) , then at Thus, β=
2 d
such points AR = (a1 − a2 ) and I R = I 1 + I2 − 2 I 1I2 is and for a given arrangement, it is constant, i.e. all fringes are
uniformly spaced.
minimum leading to a destructive interference.
Moreover, fringe width β is
Amplitude Ratio 1
2 (i) β ∝ D, (ii) β ∝ λ and (iii) β ∝ .
I max I 1 + I2 + 2 I 1I2  I 1 + I2  d
= = 
I min I 1 + I2 − 2 I 1 I2  I 1 − I2  Angular fringe width of interference pattern,
2 2 β λ
 a + a2  r + 1 α= =
= 1  = r − 1 D d
 1
a − a2    If in a given field of view n1, fringes of light of wavelength
a
where, r = 1 = amplitude ratio. λ 1 are visible and n2 fringes of wavelength λ 2 are visible, then
a2 n1λ 1 = n2 λ 2
NOTE • For identical sources, I1 = I2 = I0
φ
• For constructive interference, Imax = 4 I0 and I = 4 I0 cos 2 NOTE • If whole apparatus of Young’s double slit experiment is
2
• For destructive interference, Imin = 0 immersed in a transparent medium of refractive index nm,

then fringe width in the medium, β m = .
nm d
Young’s Double Slit Experiment
The arrangement is shown in figure monochromatic light of
one wavelength is used. Displacement of Fringe Pattern
P When a thin transparent plate is introduced in the path of one
of the interfering waves trains, it is found that the entire fringe
pattern is shifted through a constant distance. This shift takes
x
place towards the wave train, in the path of which the plate is
S1 d introduced.
O
Light S S2 t P
source D S1
Shift
Young’s experimental arrangement to produce O
interference pattern
S2
Bright and dark fringes are formed on the screen with central
point O behaving as the central bright fringe, because for O, Young’s double slit experiment
the path difference ∆ = 0.
For light waves reaching a point P, situated at a distance x The fringe width of the patterns remains same. Effective
from central point ∆, the path difference, optical path that is equivalently covered in air is
xd S 1P + t (µ − 1).
∆ = S2 P − S 1 P =
D Thus, the path difference = S2 P − S 1P − t (µ − 1)
xd xd
Case I If = nλ, then we get nth bright fringe. Hence, = − t (µ − 1)
D D
position of bright fringes on the screen are given by the
[Optical path = refractive index× width of the material]
relation, x = nDλ ⋅
d ∴ Extra path = µt − t = (µ − 1) t
DAY TWENTY NINE WAVE OPTICS 325

Coherent Sources NOTE • Fresnel’s biprism is a device to produce coherent sources


by division of wavefront,
Two light sources are said to be coherent, if they emit light of d = 2 a ( n − 1) α
exactly same frequency (or wavelength), such that the
The distance between the coherent sources and screen,
originating phase difference between the waves emitted by
D=a+ b
them is either zero or remains constant. For sustained Dλ λ ( a + b)
interference pattern, the interfering light sources must be The fringe width is given by β = =
d 2a ( n − 1) α
coherent one.
There are two possible techniques for obtaining coherent
light sources.
Diffraction
l
In division of wavefront technique, we divide the
wavefront emitted by a narrow source in two parts by Diffraction of light is the phenomenon of bending of light
reflection, refraction or diffraction. around the edges of an aperture or obstacle and entry of light
even in the region of geometrical shadow, when size of
l
In division of amplitude technique, a single extended light
aperture or obstacle is comparable to wavelength of light
beam of large amplitude is splitted into two or more waves
by making use of partial reflection or refraction. used. Diffraction is characteristic of all types of waves.
Greater the wavelength, more pronounced is the diffraction
• Two independent sources of light can never be coherent. effect. It is due to this reason that diffraction effect is very
NOTE
Two light sources can be coherent only, if these have been commonly observed in sound.
derived from a single parental light source.
Diffraction due to a Single Slit
Interference in Thin Films Fraunhofer’s arrangement for studying diffraction at a single
narrow slit (width = a) is shown in adjoining figure. Lenses L1
In white light thin films, whose thickness is comparable to
and L2 are used to render incident light beam parallel and to
wavelength of light, show various colours due to interference
focus parallel light beam.
of light waves reflected from the two surfaces of thin film.
For interference in reflected light condition of constructive W
interference (maximum intensity), A θ P
λ
∆ = 2 nm t cos r = (2 n − 1) a θ
2 O
Condition of destructive interference (minimum intensity), S θ
λ f1 L1 B
∆ = 2 nm t cos r = (2 n) L2
2 f2
Slit Screen
For interference in refracted light condition of constructive
interference (maximum intensity), Fresnel diffraction through a slit
λ
∆ = 2 nmt cos r = (2 n) As a result of diffraction, we obtain a broad, bright maxima at
2
symmetrical centre point O and on either side of it, we get
Condition of destructive interference (minimum intensity), secondary diffraction maxima of successively falling intensity
λ and poor contrast, as shown in figure.
∆ = 2 nmt cos r = (2 n − 1) , where n = 1, 2, 3,… .
2 I

Shift in Interference Pattern


If a transparent thin sheet of thickness t and refractive index
nm is placed in the path of one of the superimposing waves
(say in front of slit S2 of Young’s double slit experiment), then
it causes an additional path difference due to which –ve O +ve
θ (or x)
interference pattern shifts. Intensity curve
l
Additional path difference due to sheet = (nm − 1) t
D β
l
Condition of diffraction minima is given by
l
Fringe shift = (nm − 1)t = (nm − 1)t a sin θ = nλ
d λ
l
If due to presence of thin film, the fringe pattern shifts by n where, n = 1, 2, 3, 4,... .
fringes, then But the condition of secondary diffraction maxima is
(n − 1) t nλ λ
n= m or t = a sin θ = (2 n + 1)
λ (nm − 1) 2
Shift is independent of the order of fringe and wavelength. where, n = 1, 2, 3, 4,... .
326 40 DAYS ~ JEE MAIN PHYSICS DAY TWENTY NINE

l
Angular position of nth secondary minima is given by called plane polarised light. The vibrations in a plane
λ polarised light are perpendicular to the plane of polarisation.
sin θ = θ = n
a
NOTE • Only transverse waves can be polarised. Thus, it proved
nDλ nf2 λ
and linear distance, x n = Dθ = = that light waves are transverse waves.
a a
where, f2 is focal length of lens L2 and D = f2 . Brewster’s Law
l
Similarly, for nth maxima, we have According to this law, when unpolarised light is incident at an
(2 n + 1) λ (2 n + 1) Dλ (2 n + 1) f2 λ angle called polarising angle, i p on an interface separating air
sin θ = θ = and x n = = from a medium of refractive index µ, then the reflected light is
2a 2a 2a
fully polarised (perpendicular to the plane of incidence),
2λ provided
The angular width of central maxima is 2 θ =
a µ = tan i p
2 Dλ 2 f2 λ This relation represents Brewster’s law. Note that the parallel
or linear width of central maxima = =
a 2 components of incident light do not disappear, but refract into
the medium, with the perpendicular components.
NOTE • The angular width of central maxima is double as compared
to angular width of secondary diffraction maxima.
Law of Malus
When a beam of completely plane polarised light is incident
Polarisation of Light on an analyser, the resultant intensity of light (I ) transmitted
l
Light is an electromagnetic wave in which electric and from the analyser varies directly as the square of cosine of
magnetic field vectors very sinusoidally, perpendicular to angle ( θ) between plane of transmission of analyser and
each other as well as perpendicular to the direction of polariser.
propagation of wave of light. i.e. I ∝ cos2 θ
l
The phenomenon of restricting the vibrations of light If intensity of plane polarised light incidenting on analyser is
(electric vector) in a particular direction, perpendicular to I 0, then intensity of emerging light from analyser is I 0 cos2 θ.
the direction of wave motion is called polarisation of light.
The tourmaline crystal acts as a polariser. NOTE • We can prove that when unpolarised light of intensity I0
A gets polarised on passing through a polaroid, its intensity
D 1
E becomes half, i.e. I = I0 .
F 2

H G Polaroids
C Polaroids are thin and large sheets of crystalline polarising
B
material (made artifically) capable of producing plane
Polarisation of Light polarised beams of large cross-section.
Thus, electromagnetic waves are said to be polarised when The important uses are
their electric field vector are all in a single plane, called the l
These reduce excess glare and hence sun glasses are fitted
plane of oscillation/vibration. Light waves from common with polaroid sheets.
sources are upolarised or randomly polarised. l
These are also used to reduce headlight glare of cars.
Plane Polarised Light l
They are used to improve colour contrast in old oil
The plane ABCD in which the vibrations of polarised light are paintings.
confined is called the plane of vibration. It is defined as The l
In wind shields of automobiles.
light, in which vibrations of the light (vibrations of electric l
In window panes.
vector) when restricted to a particular plane the light itself is l
In three dimensional motion pictures.
DAY TWENTY NINE WAVE OPTICS 327

DAY PRACTICE SESSION 1

FOUNDATION QUESTIONS EXERCISE


1 The wavefront of a distant source of light is of shape 7 In a Young’s double slit experiment, two coherent
(a) spherical (b) cylindrical (c) elliptical (d) plane sources are placed 0.90 mm apart and the fringes are
2 Which of the following cannot be explained on the basis observed one metre away. If it produces the second dark
of wave nature of light? fringe at a distance of 1 mm from the central fringe, the
wavelength of monochromatic light used will be
(i) Polarisation (ii) Optical activity (a) 60 × 10−4 cm (b) 10 × 10−4 cm
(iii) Photoelectric effect (iv) Compton effect (c) 10 × 10−5 cm (d) 6 × 10−5 cm
(a) III and IV (b) II and III 8 In Young’s double slit experiment, the two slits act as
(c) I and III (d) II and IV coherent sources of waves of equal amplitude A and
wavelength λ. In another experiment with the same
3 A Young’s double slit experiment uses a monochromatic
arrangement the two slits are made to act as incoherent
source. The shape of the interference fringes formed on
sources of waves of same amplitude and wavelength. If
a screen is ª AIEEE 2005
the intensity at the middle point of the screen in the first
(a) hyperbola (b) circle I
(c) straight line (d) parabola case is I1 and in the second case I 2, then the ratio 1 is
I2
4 Two light rays having the same wavelength λ in vacuum ª AIEEE 2012
are in phase initially. Then, the first ray travels a path L1
(a) 4 (b) 2 (c) 1 (d) 0.5
through a medium of refractive index µ1, while the
second ray travels a path of length L2 through a medium 9 A mixture of light consisting of wavelength 590 nm and an
of refractive index µ 2. The two waves are then combined unknown wavelength, illuminates the Young’s double slit
to observe interference. and gives rise to two overlapping interference patterns on
The phase difference between the two waves is the screen. The central maximum of both lights coincide.
2π 2π Further, it is observed that the third bright fringe of known
(a) (µ 1 L 1 − µ 2 L2 ) (b) (L2 − L 1)
λ λ light coincides with the 4th bright fringe of the
2 π  L 1 L2  2π unknown light. From this data, the wavelength of the
(c)  −  (d) (µ 2 L 1 − µ 1 L 2 )
λ  µ1 µ 2  λ unknown light is ª AIEEE 2003
(a) 885.0 nm (b) 442.5 nm
5 The Young’s double slit experiment is performed with (c) 776.8 nm (d) 393.4 nm
blue and with green light of wavelengths 4360 Å and 10 Two coherent point sources S1 and S 2 are separated by a
5460 Å respectively. If x is the distance of the 4th small distance d as shown. The fringes obtained on the
maxima from the central one, then screen will be ª JEE Main 2013
(a) x (blue ) = x (green )
(b) x (blue ) > x (green ) Screen
(c) x (blue ) < x (green )
(d) x (blue ) / x (green ) = 5460/4360 d

6 In Young’s double slit experiment, the length of band is S1 S2


1mm. The fringe width is 0.021 mm. The number of D
fringes is
(a) points (b) straight lines
A (c) semi-circle (d) concentric circles
0.5 mm 11 The source that illuminates the double-slit in ‘double-slit
C 1 mm interference experiment’ emits two distinct
monochromatic waves of wavelength 500 nm and
600 nm, each of them producing its own pattern on the
B screen. At the central point of the pattern when path
(a) 45 (b) 46
difference is zero, maxima of both the patterns coincide
(c) 49 (d) 48 and the resulting interference pattern is most distinct at
the region of zero path difference.
328 40 DAYS ~ JEE MAIN PHYSICS DAY TWENTY NINE

But as one moves out of this central region, the two fringe The percentage of incident light transmitted by the first
systems are gradually out of step such that maximum polariser will be
due to one wave length coincides with the minimum due (a) 100% (b) 50% (c) 25% (d) 125%
to the other and the combined fringe system becomes
completely indistinct. This may happen when path 19 A beam of ordinary unpolarised light passes through a
difference in nm is tourmaline crystal C1 and then it passes through another
tourmaline crystal C2 which is oriented such that its
ª JEE Main (Online) 2013
principal plane is parallel to that of C2. The intensity of
(a) 2000 (b) 3000 (c) 1000 (d) 1500
emergent light is I 0. Now, C2 is rotated by 60° about the
12 In Young’s double slit experiment, the intensity at a point ray. The emergent ray will have an intensity
λ
where the path difference is (λ being the wavelength of (a) 2 I 0 (b) I 0 / 2 (c) I 0 / 4 (d) I 0 / 2
6
the light used) is I. If I 0 denotes the maximum intensity, I / I 0 20 A beam of light AO is incident on a glass slab (µ = 1.54) in
is equal to ª AIEEE 2007 a direction as shown in the figure. The reflected ray OB is
1 3 1 3 passed through a nicol prism. On viewing through a nicol
(a) (b) (c) (d)
2 2 2 4 prism, we find on rotating the prism that
13 In Young’s double slit experiment, the intensity at a point A N B
is 1/4 of the maximum intensity. Angular position of this
point is ª AIEEE 2005
33° 33°
λ  λ  −1  λ  −1  λ 
(a) sin−1   (b) sin−1   (c) sin   (d) sin   O
d   2d   3d   4d 
(a) the intensity is reduced down to zero and remains zero
14 The first diffraction minimum due to single slit diffraction (b) the intensity reduces down somewhat and rises again
is θ, for a light of wavelength 5000 Å. If the width of slit is (c) there is no change in intensity
1 × 10−4 cm. Then, the value of θ is (d) the intensity gradually reduces to zero and then again
(a) 30° (b) 45° (c) 60° (d) 15° increases
15 A parallel monochromatic beam of light is incident 21 When an unpolarised light of intensity I 0 is incident on a
normally on a narrow slit. A diffraction pattern is formed polarising sheet, the intensity of the light which does not
on a screen placed perpendicular to the direction of get transmitted is ª AIEEE 2005
incident beam. At the first maximum of the diffraction 1 1
(a) I0 (b) I0 (c) zero (d) I 0
pattern, the phase difference between the rays coming 2 4
from the edges of the slit is
22 Two beams, A and B of plane polarised light with
(a) 0 (b) π 2 (c) π (d) 2 π
mutually perpendicular planes of polarisation are seen
16 In Fraunhofer diffraction experiment, L is the distance through a polaroid. From the position when the beam A
between screen and the obstacle, b is the size of has maximum intensity (and beam B has zero intensity), a
obstacle and λ is wavelength of incident light. The rotation of polaroid through 30° makes the two beams
general condition for the applicability of Fraunhofer appear equally bright.If the initial intensities of the two
diffraction is beams are I A and IB respectively, then I A / IB equals
b2 b2 b2 b2 ª JEE Main 2014
(a) >>1 (b) =1 (c) <<1 (d) ≠1
Lλ Lλ Lλ Lλ (a) 3 (b)
3
(c) 1 (d)
1
17 In a Fraunhofer diffraction experiment at a single slit 2 3
using a light of wavelength 400 nm, the first minimum is 23 A ray of light is incident on the surface of a glass plate of
formed at an angle of 30°. The direction θ of the first refractive index 1.732 at the polarising angle. The angle
secondary maximum is given by of refraction of the ray is
(a) sin− 1   (b) sin− 1  
2 3 (a) 45° (b) 60° (c) 15° (d) 30°
 3  4
24 A beam of unpolarised light of intensity I 0 is passed
(c) sin− 1   (d) tan− 1  
1 2 through a polaroid A and then through another polaroid B
 4  3 which is oriented, so that its principal plane makes an
18 An unpolarised beam of light is incident on a group of angle of 45° relative to that of A. The intensity of the
four polarising sheets which are arranged in such a way emergent light is ª JEE Main 2013
that the characteristic direction of each polarising sheet (a) I 0 (b) I 0 / 2
makes an angle of 30° with that of the preceding sheet. (c) I 0 / 4 (d) I 0 / 8
DAY TWENTY NINE WAVE OPTICS 329

Direction (Q. Nos. 25-28) Each of these questions contains 26 Statement I To observe diffraction of light, the size of
two statements : Statement I and Statement II. Each of these obstacle/aperture should be of the order of10−7 m.
questions also has four alternative choices, only one of which
Statement II 10−7m is the order of wavelength of the
is the correct answer. You have to select one of the codes (a),
visible light.
(b), (c) and (d) given below:
(a) Statement I is true, Statement II is true; Statement II is 27 Statement I For a given medium, the polarising angle is
the correct explanation for Statement I 60°. The critical angle for this medium is 35°.
(b) Statement I is true, Statement II is true; Statement II is Statement II µ = tan ip .
not the correct explanation for Statement I
(c) Statement I is true; Statement II is false 28 Statement I In Young’s double slit experiment, the
(d) Statement I is false; Statement II is true number of fringes observed in the field of view is small
with longer wavelength of light and is large with shorter
25 Statement I The thick film shows no interference pattern. wavelength of light.
Take thickness of the order of a few cms.
Statement II In the double slit experiment the fringe
Statement II For interference pattern to be observed path
difference between two waves is of the order of few width depends directly on the wavelength of light.
wavelengths. ª JEE Main (Online) 2013

DAY PRACTICE SESSION 2

PROGRESSIVE QUESTIONS EXERCISE


1 A parallel beam of light of intensity I 0 is incident on a
glass plate, 25% of light is reflected by upper surface S1
and 50% of light in reflected from lower surface. The
ratio of maximum to minimum intensity in interference P
A θ O
region of reflected ray is C
L=
d/4 Screen
S2
Air

AC = CO = D, S1C = S 2 C = d << D
5 5
(a) below point O (b) below point O
2 2
235 231
1 3 1 3 5 5
 +   +  (c) below point O (d) below point O
(a)  2 8  (b)  4 8  (c) 5 (d)
8 220 110
1− 3 1− 3 8 5
    4 n identical waves each of intensity l0 interfere with each
2 8 2 8
other. The ratio of maximum intensities if the interference
2 White light is used to illuminate the two slits in a Young’s is (i) coherent and (ii) incoherent is
double slit experiment. The separation between the slits ª JEE Main (Online) 2013
is b and the screen is at a distance d ( >> b ) from the slits. (a) n 2 (b)
1
(c)
1
(d) n
At a point on the screen directly in front of one of the n n2
slits, certain wavelengths are missing. Some of these 5 In a Young’s double slit experiment, one of the slit is
missing wavelengths are wider than other, so that amplitude of the light from one
2b 2 b2 2b 2 3b 2 slit is double of that from other slit. If Im be the maximum
(a) λ = (b) λ = (c) λ = (d) λ =
3d 3d d d intensity, the resultant intensity I when they interfere at
phase difference φ is given by ª AIEEE 2012
3 A small transparent slab containing material of µ = 1.5 is
Im
(4 + 5 cos φ)
Im  1 + 2 cos2 φ 
placed along AS 2 (figure). What will be the distance from (a) (b)  
9 3  2
O of the principal maxima and of the first minima on
Im  1 + 4 cos2 φ  Im  1 + 8 cos2 φ 
either side of the principal maxima obtained in the (c)   (d)  
5  2 9  2
absence of the glass slab.
330 40 DAYS ~ JEE MAIN PHYSICS DAY TWENTY NINE

6 An object 2.4 m in front of a lens forms a sharp image on (a) 25 µm (b) 50 µm


a film 12 cm behind the lens. A glass plate 1 cm thick, of (c) 75 µm (d) 100 µm
refractive index 1.50 is interposed between lens and film 11 The box of a pin hole camera of length L, has a hole of
with its plane faces parallel to film. At what distance radius a. It is assumed that when the hole is illuminated
(from lens) should object shifted to be in sharp focus on by a parallel beam of light of wavelength λ the spread of
film? ª AIEEE 2012 the spot (obtained on the opposite wall of the camera) is
(a) 7.2 m (b) 2.4 m (c) 3.2 m (d) 5.6 m the sum of its geometrical spread and the spread due to
7 In a YDSE, bichromatic light of wavelength 400 nm and diffraction. The spot would then have its minimum size
560 nm are used. The distance between the slits is (say bmin ) when ª JEE Main 2016 (Offline)
0.1 mm and the distance between the plane of the slits λ2  2 λ2   2 λ2 
and the screen is 1 m. The minimum distance between (a) a = and bmin =   (b) a = λL and bmin =  
L  L   L 
two successive regions of complete darkness is
λ2
ª AIEEE 2004 (c) a = λL and bmin = 4λL (d) a = and bmin = 4λL
L
(a) 4 mm (b) 5.6 mm (c) 14 mm (d) 28 mm
12 In the YDSE apparatus shown in figure ∆x is the path
8 The maximum number of possible interference maxima difference between S1 P and S 2 P.
for slit-separation equal to twice the wavelength in
Young’s double slit experiment, is P
ª AIEEE 2004
S1
(a) infinite (b) five (c) three (d) zero O
9 Unpolarised light of intensity I passes through an ideal S2
polariser A. Another identical polariser B is placed
behind A. The intensity of light beyond B is found to be
I
. Now, another identical polariser C is placed between Now a glass slab is introduced in front of S 2, then match
2
the following columns.
1
A and B. The intensity beyond B is now found to be .
8 Column I Column II
The angle between polariser A and C is ª JEE Main 2018 A ∆x at P will 1. increase
(a) 0° (b) 30° (c) 45° (d) 60° B Fringe width will 2. decrease
10 The angular width of the central maximum in a single slit C Fringe pattern will 3. remain same
diffraction pattern is 60°. The width of the slit is 1 µm. The D Number of fringes 4. shift upward
slit is illuminated by monochromatic plane waves. If between O and P will
another slit of same width is made near it, Young’s fringes 5. shift downward
can be observed on a screen placed at a distance 50 cm
A B C D A B C D
from the slits. If the observed fringe width is 1 cm, what is (a) 1 3 5 3 (b) 2 3 5 3
slit separation distance? (i.e. distance between the (c) 3 4 1 3 (d) 5 5 1 2
centres of each slit.) ª JEE Main 2018

ANSWERS

SESSION 1 1 (d) 2 (a) 3 (a) 4 (a) 5 (c) 6 (c) 7 (d) 8 (b) 9 (b) 10 (d)
11 (d) 12 (d) 13 (c) 14 (a) 15 (d) 16 (c) 17 (b) 18 (b) 19 (c) 20 (d)
21 (a) 22 (d) 23 (d) 24 (c) 25 (a) 26 (a) 27 (a) 28 (a)

SESSION 2 1 (a) 2 (b) 3 (b) 4 (d) 5 (d) 6 (d) 7 (d) 8 (b) 9 (c) 10 (a)
11 (c) 12 (a)
DAY TWENTY NINE WAVE OPTICS 331

Hints and Explanations


SESSION 1 10 It will be concentric circles. 15 The phase difference (f) between the
wavelets from the top edge and the
1 When the point source or linear source 11 Q nl1 = æç n + 1 ö÷ l1 2p
of light is at very large distance, è 2ø bottom edge of the slit is f = (d sin q)
wavefronts are plane wavefronts. l
Þ n ´ 500 ´ 10-9
where, d is the slit width. The first
2 Photoelectric effect and Compton effect 1
= æç n + ö÷ ´ 600 ´ 10-9 minima of the diffraction pattern occurs
cannot be explained on the basis of è 2ø l
wave nature of light while polarisation at sin q = ,
3 d
and optical activity can be explained. Þ n=
4 2p æ lö
So, f = çd ´ ÷ = 2p.
3 Shape of interference fringes formed on Now from the formula, Dx = nl l è dø
a screen is hyperbolic in nature. æn + 1öl
or ç ÷ 16 The general condition for Frounhofer
4 Optical path for 1st ray = m1 L1 è 2ø
b2
we get, Dx = 1500 nm diffraction is << 1.
Optical path for 2nd ray = m2 L2 Ll
\ Path difference = ( m1 L1 - m2 L2 ) 12 Phase difference
Now, phase difference 2p 17 For first diffraction minimum,
= ´ path difference a sin q = l
2p l
= ´ (path difference) l
l 2p l p Þ a=
i.e. f= ´ = sin q
2p l 6 3
= ( m1 L1 - m2 L2 )
l f For first secondary maximum,
As, I = I max cos 2 æç ö÷ 3l
5 Distance of nth maxima, è2ø a sin q¢ =
I 2
D f
x = nl µl or = cos 2 æç ö÷ 3l 1 3l sin q
d I max è2ø or sin q¢ = ´ = ´
2 a 2 l
As, lb < l g 3 3
I p 3
or = cos 2 æç ö÷ = = ´ sin 30° =
\ x blue < x green I0 è 6ø 4 2 4
6 The number of fringes on either side of -1 æ 3 ö
or q¢ = sin ç ÷
C of screen is 13 I = I max cos2 æç f ö÷ è 4ø
è ø 2
é AC ù é 0.5 ù
n1 = ê ú=ê ú
= [23.8] » 24 18 First polariser just polarises the
I f
ë b û ë 0.021 û \ max = I max cos 2 æç ö÷ unpolarised light. Therefore, intensity of
Total number of fringes 4 è2ø polarised light transmitted from first
= 2n1 + fringe at centre = 2n1 + 1 f 1 f p polariser is
cos = or = 1
= 2 ´ 24 + 1 = 48 + 1 = 49 2 2 2 3 I 0 = 50% I 0
2
7 x = ( 2n - 1 ) l D \ f=
2p æ 2p ö
= ç ÷ Dx ...(i)
2 d 3 èlø 19 Intensity of light from C2 = I 0
2 xd On rotating through 60°,
or l = where, Dx = d sin q
( 2n - 1 ) D I = I 0 cos 2 60°
Substituting in Eq. (i), we get
2 ´ 10-3 ´ 0.9 ´ 10-3 l 2
= sin q = 1
(2 ´ 2 - 1) ´ 1 3d = I 0 æç ö÷ = I 0 / 4
è2 ø
= 6 ´ 10-7 m = 6 ´ 10-5 cm æ l ö
or q = sin -1 ç ÷ 20 As, i p = tan -1 (1.54) = 57°
è 3d ø
8 For coherent sources and in the figure given in question
I1 = 4 l 0 cos 2 f / 2 = 4 l 0 14 The distance of first diffraction i = 90° - 33° = 57° = i p
For incoherent sources minimum from the central principal
I2 = I 0 + I 0 = 2l 0 maximum is \ Reflected light along OB is plane
l1 Dl polarised. On rotating the nicol prism,
\ =2 x=
I2 d intensity gradually reduces to zero and
x l then increases again.
3 Dl k 4 Dl m =
9 We have, = D d
d d l 21 I = I 0 cos2 q
Þ d =
where, l k is the known wavelength and sin q I0
Intensity of polarised light =
l m is the unknown wavelength. Thus, l 5000 ´ 10-8 2
Þ sin q = =
we get d 1 ´ 10-4 \ Intensity of untransmitted light
3 lk 3 I I
lm = = ´ 590 = 442.5 nm = 0.5 = sin 30° = I0 - 0 = 0
4 4 Þ q = 30° 2 2
332 40 DAYS ~ JEE MAIN PHYSICS DAY TWENTY NINE

22 By law of Malus i.e. I = I 0 cos2 q ± l / 2 - 0.5 L


SESSION 2 or sin q1 =
2d
Initially Finally 1 The intensity of light reflected from ± l /2 - d / 8
upper surface is =
IA IA 2d
25 I
IB IB I1 = I 0 ´ 25% = I 0 ´ = 0 ± l /2 - l / 8 1 1
Polaroid Polaroid 100 4 = =± -
2l 4 16
Intensity of transmitted light from
upper surface is [Q The diffraction occurs if the
I 3I wavelength of waves is nearly
Transmission axis Transmission axis I = I0 - 0 = 0 equal to the slit width (d)]
Now, I A¢ = I A cos 2 30° 4 4
\ Intensity of reflected light from lower On the positive side
I B¢ = I B cos 2 60° 1 1 3
surface is sin q¢1 = + - =
As, I ¢A = I ¢B 3I 50 3I 4 16 16
I A cos 2 30° = I B cos 2 60° I2 = 0 ´ = 0 On the negative side
4 100 8
3 1 I 1 2 1 1 5
Þ I A = IB Þ A = I max ( I1 + I2 ) sin q¢¢1 = - - =-
4 4 IB 3 = 4 16 16
I min ( I1 - I2 )2 The first principal maxima on the
2
23 From Brewster’s law, æ I0 3I 0 ö positive side is at distance
m = tan q p ç + ÷
I max è 4 8 ø sin q1¢
1.732 = tan q p = D tan q1¢ = D
I min 2 1 - sin2 q¢1
Þ q p = tan -1 (1.732) = 60° æ I0
-
3I 0 ö
ç ÷
è 4 8 ø 3 3D
Since, the angle between i p and r is 90° =D = above point O.
2 162 - 32 247
when the ray is incident at polarising æ1 3ö
ç + ÷ The first principal minima on the
angle, then è2 8ø
= 2 negative side is at distance
q p + r = 90° æ1 3ö
r = 90° - q p = 90° - 60° = 30° ç - ÷ 5 5
D tan q¢¢1 = = below
è2 8ø 2 2 231
16 - 5
24 Relation between intensities is
2 Path difference = S2 P - S1 P . point O.
B From the figure, 4 When interference is coherent
45° (S2 P )2 - (S 1 P )2 = b 2 When two waves of intensities I1 and I2
I0 (I0/2)
or (S 2 P - S 1 P ) (S 2 P + S 1 P ) = b 2 having a phase difference f interfere, the
(Unpolarised) IR resultant intensity is given as
b2
or (S 2 P - S 1 P ) =
2d I = I1 + I2 + 2 I1 I2 cos f …(i)
For dark fringes, The intensity will maximum, then f = 0
A b2 l or cos f = 1
I I 1 I = ( 2n + 1 )
I R = æç 0 ö÷ cos 2 (45° ) = 0 ´ = 0 2d 2 maximum intensity.
è2ø 2 2 4 \ I max = I1 + I2 + 2 I1 I2
b2 l b2
For n = 0, = or l =
25 For interference to occur, the path 2d 2 d = ( I1 + I1 )2
difference between two waves is of the In case, n identical waves each of
b2 3l b2
order of few wavelength. For n = 1, = or l =
2d 2 3d intensities I 0 interfere,
26 For diffraction to occur, the size of an I max = ( I 0 + I0 + I0 + K + n
obstacle/aperture is comparable to the 3 In case of transparent glass slab of
wavelength of light wave. The order of refractive index m, the path difference times)2
2
wavelength of light wave is 10-7 m, so = 2d sin q + ( m - 1 ) L . = (n I0 ) …(i)
diffraction occurs. For the principal maxima, (path \ I max = n2 I 0 …(ii)
27 From the relation, difference is zero) When interference is incoherent
m = tan i p = tan 60° = 3 ...(i) i.e. 2 d sin q 0 + ( m - 1 ) L = 0 Since, the average value of cos f,
L( m - 1 ) - L (0.5) over a complete cycle is zero
1 1 or sin q 0 = - = The Eq. (i), becomes,
and sinC = = [from Eq. (i)] 2d 2d
m 3 I = I1 + I2 + 2 I1 I2 ´ 0
[\ L = d /4 ]
1 ö -1 = I1 + I2 …(iii)
-1 æ or sin q 0 =
\ C = sin ç ÷ = sin -1 (0.577) = 35° 16 In case, n identical waves, each of
è 3ø
-D intensities I 0 interfere,
28 The number of fringe is smaller in case or OP = D tan q 0 = D sin q 0 =
16 Minimum intensity,
of larger wavelength is used while in
For the first minima, the path difference I min = I 0 + I 0 + I 0 + K n times
case of smaller wavelength is used the
number of fringe is larger. l I min = nI 0 …(iv)
is ±
Also, fringe width is given by 2 I max n2 I 0
\ Ratio = =n
lD l I min nI 0
b= Þ bµl \ 2 d sin q1 + 0.5 L = ±
d 2
DAY TWENTY NINE WAVE OPTICS 333

5 Given, a1 = 2a2 Next 11th minima of 400 nm will Þ l = 10-6 ´ sin 30°
Þ I1 = 4 I 2 = 4 I 0 coincide with 8th minima of 560 nm. 10-6
Þ l= m
\ I m = ( I1 + I2 )2 = (3 I2 )2 Location of this minima is, 2
(2 ´ 11 - 1)(1000)(400 ´ 10-6 ) Now, in case of interference caused by
Im Y2 =
= 9 I2 = 9 I 0 = I 0 = 2 ´ 01
. bringing second slit,
9
\ Fringe width,
Now, resultant intensity, = 42 mm lD
b=
I = I1 + I2 + 2 I1 I2 cos f \ Required distance = Y2 - Y1 = 28 mm d

8 For possible interference maxima of the 10-6 1


= 4 I 0 + I 0 + 2 4 I 0 I 0 cos f [here, l = m, b = 1 cm = m,
screen, the condition is 2 100
= 5I 0 + 4 I 0 cos f d sin q = nl ...(i) 50
d = ? and D = 50 cm = m]
I Given, d = slit width = 2l 100
= m (5 + 4 cos f)
9 \ 2l sin q = nl lD 10-6 ´ 50
So, d = =
Im or 2 sin q = n b 1
= [1 + 4(1 + cos f)] 2´ ´ 100
9 The maximum value of sin q is 1, hence, 100
I = 25 ´ 10-6 m
= m (1 + 8 cos 2 f / 2) n=2´1=2
9 or d = 25 mm
Thus, Eq. (i) must be satisfied by 5
é(1 + cos q) = 2cos 2 f ù integer values, i.e. - 2, - 1, 0, 1, 2. Hence, 11 In diffraction, first minima, we have
êë 2 úû l
the maximum number of possible sin q =
interference maxima is 5. a
6 Shift in image position due to glass
plate, 9 A C B
æ 1ö 1 ö 1 L λ/a
S = ç1 - ÷ t = æç1 - ÷ ´ 1 cm = cm I/2 I
è mø è 1.5ø 3 I 8
For focal length of the lens, L a
1 1 1 1 1 20 + 1 α β
= - = - =
f v u 12 -240 240
Using Malus’s law, intensity available
240 I
Þ f = cm after C = ´ cos 2 a
21 2
Now, to get back image on the film, lens and intensity available after So, size of a spot,
has to form image at I 2 Ll
B = cos 2 a ´ cos 2 b b = 2a + …(i)
æ12 - 1 ö cm = 35 cm such that the glass 2 a
ç ÷
è 3ø 3 I Then, minimum size of a spot, we get
= (given)
plate will shift the image on the film. 8 ¶b Ll
= 0 Þ 1- 2 = 0
1 1 1 I I ¶a a
As, = - So, ´ cos 2 a × cos 2 b =
f v u 2 8 Þ a = lL …(ii)
1
1 1 1 Þ cos 2 a × cos 2 b =
Þ = - 4
u v f So, b min = 2 lL + 2 lL
This is satisfied with a = 45°
3 21
= - and b = 45° [by substituting the value of a from
35 240 Eq. (ii) in Eq. (i)]
1 48 ´ 3 - 7 ´ 21 1 So, angle between A and C is 45°.
= =- = 4 lL
u 1680 560 10 Angular width of diffraction pattern = 60°
So, the radius of the spot,
Þ u = - 5.6 m 4
b min = lL = 4lL
7 Let nth minima of 400 nm coincides 2
with mth minima of 560 nm, then 12 From YDSE,
400 ö æ 560 ö
(2n - 1) æç ÷ = (2m - 1) ç ÷ xd
è 2 ø è 2 ø (A) Path difference D x =
30° D
2n - 1 7 14
or = = =K So, as x increases, D x also
2m - 1 5 10
increases.
i.e. 4th minima of 400 nm coincides Dx
(B) Fringe width (b ) =
with 3rd minima of 560 nm. d
Location of this minima is, independent of D x.
(2 ´ 4 - 1)(1000)(400 ´ 10-6 ) For first minima, (C) Fringe pattern will shift
Y1 = a l downward.
2 ´ 01
. sin q = ,
2 2 (D) b is constant, so number of
= 14 mm
[here, a = 10-6 m, q = 30°] fringes unaffected.
334 40 DAYS ~ JEE MAIN PHYSICS DAY THIRTY

DAY THIRTY

Unit Test 6
(Optics)
1 Minimum light intensity that can be perceived by normal length 100 cm. The value of µl should be
human eye is about10−10 W m −2. What is the minimum (a) 3/2 (b) 4/3 (c) 5/3 (d) 2
number of photons of wavelength 660 nm that must enter 8 A thin convex lens of crown glass having refractive index
the pupil in one second, for one to see the object? Area 1.5 has power 1 D. What will be the power of similar
of cross-section of the pupil is10−4 m 2. convex lens of refractive index1. 6 ?
(a) 3.3 × 10 2 (b) 3.3 × 10 3 (c) 3.3 × 10 4 (d) 3.3 × 10 5 (a) 0.6 D (b) 0.8 D (c) 1.2 D (d) 1.6 D
2 A beam of light converges to a point P. A lens is placed 9 A short pulse of white light is incident from air to a glass
in path of light 1.2 cm from P. If focal length of lens is +20 slab at normal incidence. After travelling through the slab,
cm, then image distance from lens is the first colour to emerge is
(a) 4.8 cm (b) 20 cm (c) 7.5 cm (d) 5.2 cm (a) blue (b) green (c) violet (d) red
3 A ray of light incident at an angle θ on a refracting face of 10 An object approaches a converging lens from the left of
a prism emerges from the other face normally. If the the lens with a uniform speed 5 m/s and stops at the
angle of the prism is 5° and the prism is made of a focus. The image
material of refractive index 1.5, the angle of incidence is (a) moves away from the lens with uniform speed 5 m/s
(a) sin−1 (013
. ) (b) sin−1 (052
. ) (c) sin−1 (017
. ) (d) sin−1 (0.86) (b) moves away from the lens with uniform acceleration
(c) moves away from the lens with a non-uniform
4 Two coherent point sources S1 and S 2 vibrating in phase
acceleration
emit light of wavelength λ. The separation between the (d) moves towards the lens with a non-uniform acceleration
sources is 2λ. The smallest distance from S 2 on a line
11 A passenger in an aeroplane shall
passing through S 2 and perpendicular to S1S 2, where a
(a) never see a rainbow
minimum of intensity occurs is
(b) may see a primary and a secondary rainbow as
7λ 15 λ λ 3λ
(a) (b) (c) (d) concentric circles
12 4 2 4
(c) may see a primary and a secondary rainbow as
5 A thin glass prism µ = 1.5 is immersed in water µ = 1.3. If concentric arcs
the angle of deviation in air for particular ray be D, then (d) shall never see a secondary rainbow
that in water will be 12 A narrow slit of width 1 mm is illuminated by
(a) 0.2 D (b) 0.3 D (c) 0.5 D (d) 0.6 D monochromatic light λ = 600 nm. The distance between
6 The refractive index of the material of equilateral prism is first minima on either side of center line of a screen
3. The angle of minimum deviation for the prism is placed 2 m away is
(a) 30° (b) 41° (c) 49° (d) 60° (a) 1.2 cm (b) 1.2 mm (c) 2.4 mm (d) 2.4 cm

7 A thin convergent glass lens µ = 1. 5 has a power of 13 A myopic person having far point 80 cm uses spectacles
of power −1.0 D. How far can he see clearly?
+ 5.0 D. When this lens is immersed in a liquid of
refractive index µl , it acts as a diverging lens of focal (a) 1 m (b) 2 m
(c) 4 m (d) More than 4 m
DAY THIRTY UNIT TEST 6 (OPTICS) 335

14 The radius of curvature of the curved surface of a (a) at infinity


plano-convex lens is 20 cm. If the refractive index of the (b) at the mid-point of distance between two mirrors
material of the lens be 1.5, it will (c) on the pole of concave mirror
(d) on the pole of convex mirror
(a) act as a convex lens only for the objects that lie on its
curved side 20 A cube of side 3 m is placed in front of a concave mirror
(b) act as a concave lens for the objects that lie on its of focal length 2 m with its face P at a distance 4 m and
curved side face Q at a distance 7 m from the mirror. What is distance
(c) act as a convex lens irrespective of the side on which between the images of face P and Q ?
the object lies
3m
(d) act as a concave lens irrespective of the side on which
the object lies
15 Monochromatic light of wavelength 800 nm is used in 4m
double slit experiment. One of the slit is covered with a P Q
transparent slab of thickness 2.4 × 10−5 m. The refractive
index of the material of slab is 1.4. What is the number of (a) 1.2 m (b) 2.4 m (c) 2.1 m (d) 2.2 m
fringes that will shift due to introduction of the sheet? 21 A diminished image of an object is to be obtained on a
(a) 14 (b) 12 (c) 16 (d) 10 screen 1 m from it. This can be achieved by
approximately placing.
16 Two coherent sources are S2
(a) a concave mirror of suitable focal length
4.4 mm apart and 4.4 m from
(b) a convex mirror of suitable focal length
the screen as shown in the
4.4 mm

(c) a concave lens of suitable focal length


figure. If the sources emit (d) a convex lens of focal length less than 0.25 m
light of wavelength 440 nm
which produce an 22 A convex lens and a concave lens are placed in contact.
S1 O The ratio of the magnitude of the power of the convex
interference pattern on the 4.4 m
screen. The pattern of the interference at point O is lens to that of the concave lens is 4 : 3. If the focal length
of the convex lens is 12 cm, the focal length of the
(a) constructive only
(b) destructive only
combination will be
(c) cannot be predicted (a) 16 cm (b) 24 cm (c) 32 cm (d) 48 cm
(d) may be constructive or destructive 23 The radius of curvature of a thin plano-convex lens is
17 A lens behaves as a converging lens in air and a 10 cm and the refractive index of its glass is 1.5. If the
diverging lens in water. The refractive index of the plane surface is silvered, then it will behave like a
material of the lens is (a) concave mirror of focal length 10 cm
(a) equal to unity (b) equal to 1.33 (b) concave mirror of focal length 20 cm
(c) between unity and 1.33 (d) greater than 1.33 (c) convex mirror of focal length 10 cm
(d) convex mirror of focal length 20 cm
18 Angular width of central maximum in the Fraunhoffer’s
diffraction pattern is measured. Slit is illuminated by the 24 When the plane surface of a plano-convex lens of
light of wavelength 6000 Å. If slit is illuminated by light of refractive index 1.5 is silvered, it behaves like a concave
mirror of focal length 30 cm. When its convex surface is
another wavelength, angular width decreases by 30%.
silvered, it will behave like a concave mirror of focal
Wavelength of light used is
length
(a) 3500 Å (b) 4200 Å (c) 4700 Å (d) 6000 Å
(a) 10 cm (b) 20 cm (c) 30 cm (d) 45 cm
19 A convex mirror and a concave mirror of radius 10 m
25 Two stars are situated at a distance of 8 light years from
each are placed 15 m apart facing each other. An object
the earth. Their images are just resolved by a telescope of
is placed mid-way between them. If the reflection first
diameter 0. 25 m. If the wavelength of light from stars is
take place in the concave mirror and then in another
5000 Å, then the distance between the stars is around
mirror, the position of the final image is
(a) 3 × 1010 m (b) 3. 35 × 1011 m
. × 1011 m
(c) 195 (d) 4.32 × 1010 m
26 The refractive index of air is 1.0003. The thickness of air
column which will have one more wavelength of yellow
15 cm
light (λ = 6000 Å), then in the same thickness in vacuum is
(a) 2 mm (b) 2 cm (c) 2 m (d) 2 km
336 40 DAYS ~ JEE MAIN PHYSICS DAY THIRTY

27 A thin symmetric convex lens of refractive index 1.5 and 34 Polarising angle for water is 53°4'. If light is incident at
radius of curvature 0.3 m is immersed in water of this angle on the surface of water and reflected, the angle
refractive index 4/3. Its focal length in water is of refraction is
(a) 0.15 m (b) 0.30 m (c) 0.60 m (d) 1.20 m (a) 53 ° 4 ′ (b) 126° 56′
28 A parallel beam of sodium light of wavelength 6000 Å is (c) 36° 56′ (d) 30° 4 ′
incident on a thin glass plate of µ = 1.5 , such that the 35 The distance between the first and the sixth minima in the
angle of incidence in the plate is 60°. The smallest diffraction pattern of a single slit is 0.5 mm. The screen is
thickness of the plate which will make it appear dark by 0.5 m away from the slit. If the wavelength of light used is
reflected light is 5000 Å, then the slit width will be
(a) 1260 Å (b) 2440 Å (c) 3260 Å (d) 4000 Å (a) 5 mm (b) 2.5 mm
(c) 1.25 mm (d) 1.0 mm
29 Two polaroids are oriented with their principal planes
making an angle of 60°. The percentage of incident Direction (Q. Nos. 36-40) Each of these questions contains
unpolarised light which passes through the system is two statements : Statement I and Statement II. Each of these
(a) 50% (b) 100% (c) 12.5% (d) 37.5% questions also has four alternative choices, only one of which
is the correct answer. You have to select one of the codes (a),
30 In the visible region of the spectrum the rotation of the
(b), (c) and (d) given below
b
plane of polarisation is given by θ = a + . (a) Statement I is true, Statement II is true; Statement II is
λ2
the correct explanation for Statement I
The optical rotation produced by a particular material is (b) Statement I is true, Statement II is true; Statement II is
found to be 30° per mm at λ = 5000 Å and 50° per mm at not the correct explanation for Statement I
λ = 4000 Å. The value of constant a will be (c) Statement I is true; Statement II is false
50° 50° (d) Statement I is false; Statement II is true
(a) + per mm (b) − per mm
9 9 36 Statement I Angle of deviation depends on the angle of
9° 9°
(c) + per mm (d) − per mm prism.
50 50
Statement II For thin prism δ = (µ − 1) A.
31 The refracting angle of a prism is A and the refractive
where, δ = angle of deviation, µ = refractive index,
index of the prism is cot ( A / 2). The angle of minimum
deviation is A = angle of prism.
(a) 180°−3 A (b) 180° + 2 A (c) 90° − A (d) 180°−2 A 37 Statement I Glass is transparent but its powder seems
32 Cross-section of a glass prism is an isosceles triangle. opaque. When water is poured over it, it becomes
One of refracting faces is silvered. A ray of light falls transparent.
normally on the other refracting face. After being Statement II Light gets refracted through water.
reflected twice, it emerges through the base of the prism 38 Statement I If convex lens is kept in water its convergent
perpendicular to it. The angles of prism are power decreases.
(a) 54°, 54°, 72° (b) 72°, 72°, 36° Statement II Focal length of convex lens in water
(c) 45°, 45°, 90° (d) 57°, 57°, 76° increases.
33 A spherical surface of radius of curvature R, separates 39 Statement I Danger signals are made of red colours.
air and glass (n air = 1.0, n glass = 1.5). The centre of Statement II Velocity of red light is maximum and thus,
curvature is in glass. A point object P placed in air is more visibility in dark.
found to have a real image in the glass. The line PQ cuts
40 Statement I The clouds in sky generally appear to be
the surface at a point O such that PO = OQ .
whitish.
Distance PQ is
Statement II Diffraction due to clouds is efficient in equal
(a) 5R (b) 3R (c) 2R (d) R measure for all wavelengths.
DAY THIRTY UNIT TEST 6 (OPTICS) 337

ANSWERS
1. (c) 2. (c) 3. (a) 4. (a) 5. (b) 6. (d) 7. (c) 8. (c) 9. (d) 10. (c)
11. (b) 12. (c) 13. (c) 14. (c) 15. (b) 16. (a) 17. (c) 18. (b) 19. (d) 20. (a)
21. (d) 22. (d) 23. (a) 24. (a) 25. (c) 26. (a) 27. (d) 28. (b) 29. (c) 30. (b)
31. (d) 32. (b) 33. (a) 34. (c) 35. (b) 36. (a) 37. (a) 38. (a) 39. (c) 40. (c)

Hints and Explanations


3λ  1 1 
1 I = 10−10 Wm −2 = 10−10Js −1 m2 . Let the or 4λ2 + x2 − x = 8 P = (µ − 1)  −
2 
number of photons required per second  R1 R2 
be n. P2  µ 2 − 1  P 1.6 − 1
nhv P
Hence, =  i.e. 2 =
Then, = 10−10 P1  µ 1 − 1  1 1.5 − 1
10−4
Hence, P2 = 1.2 D
Hence, n = 10−10 × 10−4 /hv x
λ 9 In air, all the colours of light travel with
= 10−14
hc the same velocity, but in glass, velocities
10−14 × 660 × 10−9 S1 of different colours are different. Velocity
= 2λ S2
of red colour is largest and velocity of
6.6 × 10−34 × 3 × 108 Solving this equation, we get violet colour is smallest. Therefore, after

= 3.3 × 104 x= travelling through the glass slab, red
12 colour will emerge first.
2 1=1+1 5 δ~
− (µ − 1) A
v f u 10 When an object approaches a convergent
For air, D = (1.5 − 1) A lens from the left of the lens with a
Here, f = + 20 and u = +12
For water, δ = ( g µ w − 1) A = 
1.5  uniform speed of 5 m/s, the image moves
∴ v = 7.5 cm − 1 A
 13
.  away from the lens with a non-uniform
3 Here, A = 5°, i1 = ? 02
. D acceleration. For example, f = 20 m and
Hence, δ= × ≅ 0.3 D u = − 50 m; − 45m, − 40 m and − 35m; we
13
. 0.5
get v = 333. m; 36 m; 40 m and 46.7 m.
A + δm 
5° sin  
Clearly, image moves away from the lens
N1 N2  2  with a non-uniform acceleration. Option
6 µ =
(c) is correct.
sin  
90° A
i1 r1
2 11 In an aeroplane, a passenger may observe
A + δm 
sin 
a primary and a secondary rainbow as

 2  concentric circles.
⇒ 3= ,
sin 30° 12 For first dark fringe on either side,
As the ray emerges from the because A = 60° d sinθ = λ
other face of prism normally, A + δm 3 y
or sin = = sin 60° and sinθ =
i2 = 0° 2 2 D
∴ r2 = 0°, dy λD
or A + δ m = 2 × 60 = 120 So, = λ or y =
As r1 + r2 = A This gives δ m = 60°.
D d
r1 = A − r2 = 5 − 0 = 5° Distance between two minima = 2y
where, δ m is minimum deviation.
sin i1 2 × 600 × 10−6 × 2 × 103
From µ = , = mm
sin r1 7 When the lens is in air, we have 1.0
1 µg −µa  1 1  = 2.4 mm
sin i1 = µ sin r1 Pa = = −
 
sin i1 = 1.5 × sin 5° fa µ a  R1 R2  1 1 1
13 Use − =
= 1.5 × 0.087 When the lens is in liquid, we have v u f
sin i1 = 01305
. 1 µg −µl  1 1  Here, v = −80 cm, f = −100 cm
i1 = sin −1 (01305
. ) Pl = =  −  1 1 1
fl µl  R1 R2  Hence, − =−
− 80 u 100
4 Path difference at S2 is 2λ. Therefore, Here, Pa = 5, P l = −1,µ a = 1,µ g = 1.5 1 1 1 − 80 + 100
for minimum intensity at P. On solving, we get or − = − + =
u 100 80 80 × 100
3λ λ µl =
5
S1 P − S2 P = ≠ …(i) This gives u = − 400 cm = − 4 m
2 2 3
338 40 DAYS ~ JEE MAIN PHYSICS DAY THIRTY

14 Here, µ = 1.5 17 The focal length f of the lens in air is 14


or v2 = = 2.8 m
If object lies on plane side; given by 5
R1 = ∞, R2 = −20 cm 1  1 1  Therefore, v 1 − v 2 = (4 − 2.8) = 1.2 m
= (µ g − 1)  − 
1  1 1  f  1
R R 2 21 Convex mirror and concave lens do not
= (µ − 1) − 
f  R1 R2  form real image. For concave mirror
where, µ g is the refractive index of the
v > u, so image will be enlarged, hence
lens. If µ w is the refractive index of
= (1.5 − 1) +
1 1 1
 = water, the focal length in water ( f ′ ) is
only convex lens can be used for the
 ∞ 20  40
purpose.
f = + 40 cm. given by
|P |
The lens behaves as convex. 1 µg − µw   1 1  22 Given, f1 = + 12 cm and 1 = 4
=  −  |P2 | 3
If object lies on its curved side, f ′  µ w   R1 R2  f2 4
R1 = 20 cm, R2 = ∞, Since f2 is negative, =−
 1 Since, the lens placed in air is f1 3
1 1 
= (µ − 1) −  convergent, f is positive. Therefore, 4 4
f′  R1 R2  Hence, f2 = − f1 = − × 12
µ g > 1. For the lens to be divergent 3 3
= (1.5 − 1) −  = when placed in water, f ′ must be
1 1 1
= −16 cm
 20 ∞  40 negative, i.e. µ g < µ w . Now, µ w = 1.33.
Hence, µ g must lies between 1 and The focal length F of the combination is
f ′ = 40 cm given by
1.33.
The lens behaves as convex. 1 1 1 1 1 1
18 The condition for minima is given by = + = + =
15 The total fringe shift is H = β (µ − 1) t d sinθ = n λ
F f1 f2 12 −16 48
λ F = 48 cm
The number of fringes that will shift For n = 1, we have
total fringe shift d sinθ = λ 23 When the plane surface of a
= If angle is small, then sinθ = θ ⇒ dθ = λ plano-convex lens is silvered, it behaves
fringe width
λ like a concave mirror of focal length f
β Half angular width θ =
(µ − 1)t d given by
(µ − 1) t
or n= λ = λ 1 2( µ − 1)
=
β λ Full angular width, 2θ = 2
d f R
(1.4 − 1) × 2.4 × 10−5 2λ ′ R 10
or n= Also, ω ′ = or f = = = 10 cm
800 × 10−9 d 2(µ − 1) 2(1.5 − 1)
0.4 × 2.4 × 10−5 λ′ ω ′
or n= or n = 12 ∴ = 24 When the plane surface is silvered the
8 × 10−7 λ ω
focal length f1 is given by
ω′
or λ ′ = λ = 6000 × 07 . = 4200 Å 1 2( µ − 1)
16 The path difference at point of ω = ... (i)
f1 R
observation is given by
19 For reflection from concave mirror
∆ = S 2O − S 1O But when the convex surface is silvered,
1 1 1
S2 Applying, = + , we get the focal length f2 is given by
f v u 1 2µ
= ... (ii)
1 1 1 f2 R
= −
4.4 mm

−5 v 7.5 Dividing Eq. (i) by Eq. (ii), we have


1 1 1 µ
or =− + or v = −15m f1
= =
1.5
=3
v 5 7.5 f2 µ − 1 1.5 − 1
S1 The image is formed on the pole of the f 30
O
convex mirror, which will be the or f2 = 1 = = 10 cm
4.4 m 3 3
From the figure, position of the object for convex mirror.
S 2O = [(4.4) 2 + (4.4 × 10−3 ) 2 ]1 /2 Therefore, 25 Limit of resolution of the telescope
u = 0 and f = 5m 1.22λ d
or S 2O = 4.4[1 + (10−3 ) 2 ]1 /2 α = =
1 1 1 1 a x
= − = − ∞ =∞
or S 2O = 4.4 1 + (10−3 ) 2  1.22λ x
1 Hence,
  v 5 0 5 or d =
2 a
or v =0
Therefore, ∆ = 4.4 1 + (10 ) − 1
 1 −3 2 1.22 × 5 × 10−7 × 8 × 1016
  Therefore, final image is formed on the =
2 0.25
pole of convex mirror.
4.4 × 10−6 = 1. 95 × 1011 m
= = 2.2 × 10−6 m 20 For surface P, we have
2
1 1 1 26 Let d in cm be the thickness of air column
Interference will be constructive, if path = −
difference is an integral multiple of v1 f u = thickness of vacuum column (given).
wavelength. 1 1 1 Number of waves of wavelength λ = 6000
= − = or v 1 = 4 m Å = 6 × 10−5 cm in a thickness d cm in
∆ 2.2 × 10−6 2 4 4
n= = =5 vacuum is
λ 440 × 10−9 For surface Q, we have
1 1 5 d
Hence, pattern of interference at point O v2 = − = nv =
2 7 14 λ
is constructive.
DAY THIRTY UNIT TEST 6 (OPTICS) 339

Since, the refractive index of air b 35 For the first minimum,


30 θ = a+
µ = 1.0003, the wavelength in air will be λ2 ad 1
a sinθ1 = λ ≈ aθ1 =
λ b
λa = 30 = a + D
µ (5000)2 For the sixth minimum,
ad 6
Therefore, number of waves of
and 50 = a +
b a sinθ 6 = 6λ = aθ 6 =
wavelength λ a in d cm of air is (4000)2 D
a
d dµ Solving for a, we get ∴ (6λ − λ ) = (d 6 − d 1 )
na = = D
λa λ 50° 5 Dλ
a= − per mm or a =
9 (d 6 − d 1 )
Given that, n a − 1 = n v
dµ d  A + δm  5 × 0.5 × 5 × 10−7
Hence, −1= sin   =
 
= cot  
λ λ 2 A 0.5 × 10−3
31 Given, µ =
 A 2
λ 6 ×10−5cm sin   = 2.5 × 10−3 m
d = = 2
µ − 1 1.0003 − 1 = 2.5 mm
= 0.2 cm = 2 mm cos ( A / 2)
= 36 The relation between angle of deviation δ
sin( A / 2)
µ − µw  1 1  for a thin prism an angle of prism and
27 1 = g  −   A + δm   A
f µw  1
R R2 or sin   = cos   refractive index of material of prism is
 2  2 given by δ = ( µ − 1 ) A

= sin  90° − 
For a symmetric lens A
R2 = − R1 = − 0.3 m 
37 We know very well that, glass is
2
transparent. But when the glass is
1 (1.5 − 4 / 3)  1 1  A + δm A
Therefore, = + ∴ = 90° − powdered, the irregular reflections occur
f 4/3  0.3 0.3  2 2 from the surface of powdered glass and
or δ m = 180° − 2 A finally the light returns back into the
which gives f = 1. 20 m a
32 same medium. Because of it the
90 – i
28 1 b powdered glass looks opaque. When we
c pour water over the powdered glass,
i refraction of light takes place and it
E i
becomes transparent.
60° 60° 2
30° D 38 The focal length f w of convex lens in
A C i 90 – 2i
d water of refractive index µ w is given by
r i µg − µw   1
t 1 1 
r =  −  ...(i)
90 – θ f w  µ w   R1 R2 
B θ θ Here, µ g is the refractive index of lens
(glass).
sin 60° 3/2
sin r = = The focal length of lens in air is given by
µ 1.5
In ∆abc, 1  1 1 
∴ r = 30° = (µ g − 1)  −  ...(ii)
A + 90°+90°− i = 180° f  R1 R2 
AB = t sec r = 1.15t
⇒ i = A
AC = 2( AD ) = 2(t tan r ) = 115
. t From Eqs. (i) and (ii), we conclude the
Also, angle θ = 2i = 2 A
AE = AC cos 30° = 0.99 t focal length of lens in water ( f w ) is
This condition is satisfied by option (b) greater than focal length of lens in air
Now, net path difference between 1
only. ( f a ). Therefore, the focal length of lens in
and 2,
∆ X = µ (2 AB ) − AE
33 For given condition, water get increased consequently power
decreases.
= (1.5)(23
. t ) − 0.99t = 2.45 t
For minimum intensity, 39 Red colour consists of longest
O wavelength and it scatteres least.
∆X = λ P Q
Therefore, signals of red colour are being
or (2.45t ) = 6000 seen from the long distances. Hence,
or t = 2448 Å ≈ 2440 Å µ1 µ2 µ2 − µ1 the signals are made of red colour light.
+ =
29 Intensity of polarised light from first −u v R 40 The clouds consist of dust particles and
100 1 1.5 1.5 − 1
polariser = = 50 + = water droplets. The scattering of sun
2 − (− x ) x R light by the big dust particles and water
From law of Malus intensity from ⇒ x = 5R drops is not in accordance with the
second nicol Rayleigh law. But they scatter the light of
34 i p + r = 90° all the colours by the same amount.
I = 50 cos 2 60°
50 or r = 90° − i p = 90°−53° 4′ = 36° 56′ Hence, the clouds are seen generally
= = 12.5 % white.
4
DAY THIRTY ONE

Dual Nature
of Matter
Learning & Revision for the Day
u Photon u Laws of Photoelectric Effect
u Particle Nature of Light u de-Broglie Waves
u Photoelectric Effect u Davisson-Germer Experiment

Photon
A particle of light called a photon has energy E that is related to the frequency f and
wavelength λ of light wave.
hc
By the Einstein equation, E = hf = …(i)
λ
where, c is the speed of light (in vacuum) and h is Planck’s constant.
h = 6.626 × 10 −34 J-s = 4136
. × 10 −15 eV-s
Since, energies are often given in electron volt (1eV = 1.6 × 10 −19 J) and wavelengths are
in Å, it is convenient to the combination hc in eV-Å. We have,
hc = 12375 eV-Å
Hence, Eq. (i), in simpler form can be written as,
12375
E ( in eV) =
λ ( in Å)
…(ii) PREP
The propagation of light is governed by its wave porperties whereas the exchange of
MIRROR
Your Personal Preparation Indicator
energy between, light with matter is governed by its particle properties. The
wave-particle duality is a general property in nature. For example, electrons (and other u No. of Questions in Exercises (x)—
so called particles) also propagate as waves and exchange energy as particles. u No. of Questions Attempted (y)—
u No. of Correct Questions (z)—

Particle Nature of Light (Without referring Explanations)

Photoelectric effect gave evidence to the strange fact that, light in interaction with u Accuracy Level (z / y × 100)—
matter behaved as if it was made of quanta or packets of energy, each of energy hν. u Prep Level (z / x × 100)—
 hν 
Einstein stated that the light quantum can also be associated with momentum   ⋅
 c  In order to expect good rank in JEE,
your Accuracy Level should be above
This particle like behaviour of light was further confirmed, in 1924, by the Compton 85 & Prep Level should be above 75.
experiment of scattering of X-rays from electrons.
DAY THIRTY ONE DUAL NATURE OF MATTER 341

Photoelectric Effect
Photoelectric current
l
Photoelectric effect is the phenomenon of emission of
electrons (known as photoelectrons) from the surface of
ν 3 > ν 2 > ν1
metals when light radiation of suitable frequency are
Saturation
incident on them. ν3 current
l
The minimum energy of incident radiation needed to eject ν2
ν1
the electrons from metal surface is known as work function
(φ0) of that surface.
– V03 – V02 – V01 0 Collector plate
l
The frequency or wavelength corresponding to the work Stopping potential potential
function is called threshold frequency or threshold I-V0, when frequency changed
wavelength. Work function is related to threshold
frequency as, A photon may collide with a material particle. The total
hc energy and the total momentum remain conserved in such a
φ 0 = hν 0 = collision. Photoelectric emission is an instantaneous
λ0
phenomenon.
where, λ 0 = threshold wavelength. V0
hc 12400
l
In electron volt units, φ (eV) = =
eλ 0 λ (Å)
l
For photoemission to take place energy of incident light (E ) O
ν
ν0
is related as, E ≥ p0
l
According to Einstein’s photoelectric equation,
– φ0
hν = φ0 + K max
Cut-off voltage versus frequency of incident light
1
where, K max = mv2max = maximum kinetic energy of
2 Variation of stopping potential V0 with frequency ν of
ejected photoelectron. incident radiation is as shown in above figure.
h φ
As, eV0 = h(ν − ν 0) = hν − φ0 ⇒ V0 = ν − 0
Effect of Intensity on e e
Photoelectric Emission h
Thus, V0- ν graph is a straight line whose slope is and
e
For a light of given frequency ν > ν 0 (or given wavelength intercept is − φ0 eV. The graph meets the ν-axis at ν 0.
λ < λ 0), if the intensity of light incident on photosensitive metal
1
surface is increased, the number of photoelectrons and Photocurrent ∝ ∝ λ
consequently the photoelectric current I increases. However, the ν
stopping potential V0 remain constant.
Energy and Momentum of Photon
Photoelectric

l
From Einstein’s mass-energy relation E = hν = mc2
current

I 1 > I2 > I3

I1 Kinetic mass of photon is m =
I2 c2
I3 c
But ν = , where λ is wavelength of the photon.
Stopping Saturation λ
potential current h c h
∴Kinetic mass of photon, m = 2   =
c  λ  cλ
–V0 O Collector plate
potential hν h
Stopping potential Kinetic mass of photon, m = 2 =
Photoelectric current versus stopping potential curve c cλ
l
Momentum of photon,
Effect of Frequency on p = kinetic mass of photon × velocity of photon
hν hν
Photoelectric Emission = 2 ×c =
c c
c
If keeping the intensity of incident light constant, the Also, ν =
λ
frequency of incident light is increased, then the stopping
hc h
potential V0 (and hence, K max ) increases, but the photoelectric ∴Momentum of photon, p =   =
current I remains unchanged. c  λ λ
342 40 DAYS ~ JEE MAIN PHYSICS DAY THIRTY ONE

Ratio of wavelength of photon and electron The


Laws of Photoelectric Effect l

wavelength of photon of energy E is given by


Lenard and Millikan gave the following laws on the basis of hc
experiments on photoelectric effect. λp = while the wavelength of an electron of kinetic
E
l
The rate of emission of photoelectrons from the surface of a h
energy K is given by λ c = ⋅ Therefore for same
metal varies directly as the intensity of the incident light 2 mK
falling on the surface. energy, the ratio
l
The maximum kinetic energy of the emitted photoelectrons λp c 2 mc2 K
is independent of the intensity of the incident light. = 2 mK =
λe E E2
l
The maximum kinetic energy of the photoelectrons
increases linearly with increase in the frequency of the
incident light. Davisson-Germer Experiment
l
As soon as, the light is incident on the surface of the metal, l
The de-Broglie hypothesis was confirmed by
the photoelectrons are emitted instantly, i.e. there is no Davisson-Germer experiment. It is used to study the
time lag between incidence of light and emission of scattering of electron from a solid or to verify the wave
electrons (≈ 10 −9 s). nature of electron.
l
A beam of electrons emitted by electron gun is made to fall
de-Broglie Waves on nickel crystal cut along cubical axis at a particular
Light is said to have dual character, i.e. it behaves like matter angle. Ni crystal behaves like a three dimensional
(particle) and wave both. Some properties like interference, diffraction grating and it diffracts the electron beam
diffraction can be explained on the basis of wave nature of obtained from electron gun.
light, while the phenomena like photoelectric effect, black l
The diffracted beam of electrons is received by the detector
body radiation, etc. can be explained on the basis of particle which can be positioned at any angle by rotating it about
nature of light. the point of incidence.
In 1942, Louis de-Broglie explained that like light, matter also l
The energy of the incident beam of electrons can also be
show dual behaviour, there is a wave associated with moving
varied by changing the applied voltage to the electron gun.
particle, known as matter waves or de-Broglie waves.
l
According to classical physics, the intensity of scattered
beam of electrons at all scattering angle will be same but
de-Broglie Relation Davisson and Germer found that the intensity of scattered
According to quantum theory, energy of photon beam of electrons was not same but different at different
E = hν …(i) angles of scattering.
If mass of the photon is taken as m, then as per Einstein’s l
It is maximum for diffracting angle 50°
equation E = mc2 …(ii) at 54 V potential difference. Incident beam 54 V
From Eqs. (i) and (ii), we get, hν = mc 2 l
If the de-Broglie waves exist for
c electrons, then these should be
h = mc2 , diffracted as X-rays.
λ 50°
where, λ = wavelength of photon Using the Bragg’s formula 2d sin θ = nλ ,
h we can determine the wavelength of
λ= these waves, Diffraction angle
mc
where d = distance between versus intensity
de-Broglie asserted that the above equation is completely a curve
general function and applies to photon as well as all other diffracting planes,
moving particles. 180 − φ
θ=
h h 2
So, λ= = = glancing angle for incident beam = Bragg’s angle.
mv 2 mE
where, m is mass of particle and v is its velocity. Clearly from figure, we have θ + φ + θ = 180 °
l
de-Broglie wavelength associated with charged particle
h h h
λ= = = θ φ
p 2 mE 2 mqV D
θ d
l
de-Broglie wavelength of a gas molecule
h
λ=
3 mkT
where, T = absolute temperature Reflection of electron (beam) by atoms
and k = Boltzmann’s constant = 1.38 × 10 −23 J / K
DAY THIRTY ONE DUAL NATURE OF MATTER 343

DAY PRACTICE SESSION 1

FOUNDATION QUESTIONS EXERCISE


1 If a source of power 4kW produces1020photons/second, Codes
the radiation belong to a part of the spectrum called A B C A B C
(a) X-rays (b) ultraviolet rays (a) 1 2 3 (b) 3 2 1
(c) microwaves (d) γ-rays
(c) 2 1 3 (d) 2 3 1
2 What will be the number of photons emitted per second 8 When a point source of monochromatic light is at a
by a 10 W sodium vapour lamp assuming that 90% of the distance of 0.2 m form a photoelectric cell the cut-off
consumed energy is converted into light? (Wavelength of
voltage and saturation current are 0.6 V and 18 mA
sodium light is 590 nm and h = 6.63 × 10−34 J - s )
respectively. If the same source is placed 0.6 m away
(a) 0. 267 × 1018 (b) 0. 267 × 1019
from the photoelectric cell, then
(c) 0. 267 × 10 20 (d) 0. 267 × 1017
(a) the stopping potential will be 0.2 V
3 Two monochromatic beams A and B of equal intensity I, (b) the stopping potential will be 0.6 V
hit a screen. The number of photons hitting the screen by (c) the saturation current will be 6 mA
beam A is twice that by beam B. Then, what inference (d) the saturation current will be 18 mA
can you make about their frequencies? 9 The figure shows the variation of photocurrent (I) with
(a) νB = 2 νA (b) νB = νA anode potential (Va) of a photosensitive surface for three
(c) νA = 2 νB (d) νB > νA
different radiations. Let la,lb and lc be the intensities and fa,
4 The eye can detect 5 × 104 photons m −2s −1 of light of fb and fc the frequencies for the waves a,b and c
wavelength 500 nm. The ear can hear intensity upto respectively
10−13 Wm −2. As a power detector, which is more
sensitive?
I
(a) Sensitivity of eye is one-fifth of the ear
c
(b) Sensitivity of eye is five times that of the ear a
(c) Both are equally sensitive b
(d) Eye cannot be used as a power detector
5 The threshold wavelength for photoelectric emission from
° Va
a material is 5200A. Photoelectrons will be emitted when O
this material is illuminated with monochromatic radiation (a) fa = fb and Ia ≠ Ib (b) fa = fc and Ia = Ic
from a (c) fa = fb and Ia = Ic (d) fb = fc and Ib = Ic
(a) 50 W infrared lamp (b) 1 W infrared lamp 10 In a photoelectric effect measurement, the stopping
(c) 50 W ultraviolet lamp (d) 1 W ultraviolet lamp potential for a given metal is found to be V0 volt when
6 The wavelength of the photoelectric threshold for silver is radiation of wavelength λ 0 is used. If radiation of
λ 0. The energy of the electron ejected from the surface of wavelength 2λ 0 is used with the same metal, then the
silver by an incident light of wavelength λ ( λ < λ 0 ) will be stopping potential (in volt) will be
hc V0
(a) hc (λ 0 − λ) (b) (a) (b) 2V0
λ0 − λ 2
hc hc
h 1 1   λ − λ (c)V0 + (d)V0 −
(c) + −  (d) hc  0  2 eλ 0 2 eλ 0
c  λ λ0   λ0 λ 
11 In an experiment on photoelectric effect, a student plots
7 In photoelectric effect match the following column I with stopping potential V0 against reciprocal of the wavelength
column II. λ of the incident light for two different metals A and B.
Column I Column II These are as shown in the figure.
A. If frequency of incident 1. Stopping potential may Metal A
light is increased increase Metal B
B. If intensity of incident light 2. Stopping potential must Vo
is increased increase
C. If work function of metal is 3. Photo effect may stop
1/λ
increased
344 40 DAYS ~ JEE MAIN PHYSICS DAY THIRTY ONE

Looking at the graphs, you can most appropriately say 17 The voltage applied to an electron microscope to
that produce electrons of wavelength 0.50 Å is
(a) Work function of metal B is greater than that of metal A (a) 602 V (b) 50 V (c) 138 V (d) 812 V
(b) Work function of metal A is greater than that of metal B
(c) Students data is not correct
18 The energy of photon is equal to the kinetic energy of a
(d) None of the above proton.The energy of photon is E. Let λ 1 be the
de-Broglie wavelength, of the proton and λ 2 be the
12 A copper ball of radius 1cm and work function 4.47 eV is
wavelength of photon. The ratio λ 1 / λ 2 is proportional to
irradiated with ultraviolet radiation of wavelength 2500 Å. 1
The effect of irradiation results in the emission of (a) E 0 (b) E 2
electrons from the ball. Further the ball will acquire (c) E −1 (d) E −2
charge and due to this there will be a finite value of the 19 An electron is moving with an initial velocity v = v 0i and is
potential on the ball. The charge acquired by the ball is in a magnetic field B = B0j . Then its de-Broglie
(a) 5.5 × 10−13 C (b) 7.5 × 10−13 C wavelength
(c) 4.5 × 10−12 C (d) 2.5 × 10−11 C
(a) remains constant
13 Match List I (fundamental experiment) with List II (its (b) increase with time
conclusion) and select the correct option from the (c) decrease with time
choices given below the list. (d) increases and decreases periodically
20 Orbits of a particle moving in a circle are such that the
List I List II
perimeter of the orbit equals an integer number of
A. Franck-Hertz experiment 1. Particle nature of light de-Broglie wavelengths of the particle. For a charged
B. Photo-electric experiment 2. Discrete energy levels of atom particle moving in a plane perpendicular to a magnetic
C. Davisson-Germer experiment 3. Wave nature of electron field, the radius of the nth orbital will therefore be
4. Structure of atom proportional to
(a) n 2 (b) n
A B C A B C (c) n1/ 2 (d) n1/ 4
(a) 1 4 3 (b) 2 4 3
(c) 2 1 3 (d) 4 3 2 Direction (Q. Nos. 21-27) Each of these questions
14 The anode voltage of a photocells kept fixed. The contains two statements : Statement I and Statement II.
wavelength λ of the light falling on the cathode is Each of these questions also has four alternative choices,
gradually changed. The plate current I of photocell varies only one of which is the correct answer. You have to select
as follows one of the codes (a), (b), (c), (d) given below.
I I (a) Statement I is true, Statement II is true; Statement II is
the correct explanation for Statement I
(a) (b) (b) Statement I is true, Statement II is true; Statement II is
not the correct explanation for Statement I
λ λ (c) Statement I is true; Statement II is false
I I (d) Statement I is false; Statement II is true
21 Statement I As intensity of incident light (in photoelectric
(c) (d) effect) increases, the number of photoelectrons emitted
per unit time increases.
λ λ Statement II More intensity of light means more energy
15 de-Broglie wavelength of an electron accelerated by a per unit area per unit time.
voltage of 50V is close to (e = 1.6 × 10−19 C, 22 Statement I The relative velocity of two photons travelling
. × 10−31kg, h = 6.6 × 10−34 J-s).
me = 91 in opposite directions is the velocity of light.
° °
(a) 0.5 A (b) 1.7A Statement II The rest mass of photon is zero.
° °
(c) 2.4 A (d) 1.2 A 23 Statement I Work function of copper is greater than the
16 Photons of an electromagnetic radiation has an energy work function of sodium but both have same value of
11keV each. To which region of electromagnetic threshold frequency and threshold wavelength.
spectrum does it belong? Statement II The frequency is inversely proportional to
(a) X-ray region (b) Ultraviolet region wavelength.
(c) Infrared region (d) Visible region
DAY THIRTY ONE DUAL NATURE OF MATTER 345

24 Statement I The de-Broglie wavelength of a molecules frequency incident on the surface is doubled, both the
varies inversely as the square root of temperature. Kmax and V0 are also doubled.
Statement II The root mean square velocity of molecule Statement II The maximum kinetic energy and the
depends on the temperature. stopping potential of photoelectrons emitted from a surface
are linearly dependent on the frequency of incident light.
25 Statement I Davisson-Germer experiment established
the wave nature of electrons. 27 Statement I When ultraviolet light is incident on a
Statement II If electrons have wave nature, they can photocell, its stopping potential isV0 and the maximum
interfere and show diffraction. kinetic energy of the photoelectrons is Kmax . When the
ultraviolet light is replaced by X-rays, bothV0 and Kmax
26 Statement I A metallic surface is irradiated by a
increase.
monochromatic light of frequency ν > ν 0 (the threshold
frequency). The maximum kinetic energy and the Statement II Photoelectrons are emitted with speeds
ranging from zero to a maximum value because of the
stopping potential are Kmax and V0, respectively. If the
range of frequencies present in the incident light.

DAY PRACTICE SESSION 2

PROGRESSIVE QUESTIONS EXERCISE


1 An electron of mass m and charge e are initially at rest. It 6 The graph between 1 / λ and stopping potential (V) of
gets accelerated by a constant electric field E. The rate three metals having work functions φ1 , φ 2 and φ 3 in an
of change of de-Broglie wavelength of this electron at experiment of photoelectric effect is plotted as shown in
time t is the figure. Which of the following statement (s) is/are
−h −nh −h −eht correct? (Here, λ is the wavelength of the incident ray)
(a) (b) (c) (d)
eE t 2 eE t 2 eE E
V
2 When a surface 1cm thick is illuminated with light of
Metal 1 Metal 2 Metal 3
wavelength λ, the stopping potential is V0, but when the
same surface is illuminated by light of wavelength 3λ,
the stopping potential is V0 / 6, the threshold wavelength
for metallic surface is θ nm–1
0.001 0.002 0.004 1/λ
(a) 4λ (b) 5λ (c) 3λ (d) 2λ
3 A photocell with a constant potential difference of V volt (a) Ratio of work function φ1: φ2 : φ3 = 1: 2 : 4
across it is illuminated by a point source from a distance (b) Ratio of work function φ1: φ2 : φ3 = 4 : 2 :1
of 25 cm. When the source is moved to a distance of 1m, (c) tan θ is directly proportional to hc / e where, h is Planck’s
constant and c is speed of light
the electrons emitted by the photocell
(d) The violet colour light can eject photoelectrons from
(a) carry 1/4th their previous energy
metal 2 and 3
(b) are 1/16th as numerous as before
(c) are 1/4th as numerous as before 7 Electrons are accelerated through a potential difference
(d) carry 1/4th their previous momentum V0 and protons are accelerated through a potential
4 Consider a metal exposed to light of wavelength 600 nm. difference 4V. The de-Broglie wavelength are λ e and λ p
The maximum energy of the electron doubles when light for electrons and protons respectively.
of wavelength 400 nm is used. Find the work function in eV. λe
The ratio of is given by (Given, me is mass of
(a) 2.83 eV (b) 2 eV (c) 1.02 eV (d) 3.42 eV λp
5 A metallic surface is illuminated with monochromatic light electrons and mp is mass of proton).
of wavelength λ, the stopping potential for photoelectric
λe mp λe me
current is 3V0 and when the same surface is illuminated (a) = (b) =
λp me λp mp
with light of wavelength 2λ, the stopping potential is V0.
The threshold wavelength of this surface for photoelectric λe 1 me λe mp
(c) = (d) =2
effect is λ p 2 mp λp me
(a) 4λ /3 (b) 6λ (c) 8λ (d) 4λ
346 40 DAYS ~ JEE MAIN PHYSICS DAY THIRTY ONE

8 An electron from various excited states of hydrogen atom 10 A particle A of mass m and initial velocity v collides with a
emit radiation to come to the ground state. Let λ n , λ g be particle B of mass
m
which is at rest. The collision is
the de-Broglie wavelength of the electron in the nth state 2
and the ground state, respectively. Let Λn be the head on, and elastic. The ratio of the de-Broglie
wavelength of the emitted photon in the transition from wavelengths λ A to λ B after the collision is
the nth state to the ground state. For large n, (A, B are λA λA 2 λA 1 λA 1
(a) =2 (b) = (c) = (d) =
constants) λB λB 3 λB 2 λB 3
B
(a) Λn ≈ A + (b) Λn ≈ A + Bλ2n 11 Radiation of wavelength λ is incident on a photocell. The
λ2n fastest emitted electron has speed v. If the wavelength is
(c) Λ2n ≈ A + Bλ2n (d) Λ2n ≈ λ changed to 3λ / 4, the speed of the fastest emitted
9 An electron beam is accelerated by a potential difference electron will be
1/ 2 1/ 2 1/ 2 1/ 2
(a) > v   (b) < v   (c) = v   (d) = v  
V to hit a metallic target to produce X-rays. It produces 4 4 4 3
continuous as well as characteristic X-rays. If λ min is the  3  3  3  4
smallest possible wavelength of X-rays in the spectrum, 12 The radiation corresponding to 3 → 2 transition of
the variation of log λ min with logV is correctly represented hydrogen atom falls on a metal surface to produce
in photoelectrons. These electrons are made to enter a
magnetic field of 3 × 10−4 T. If the radius of the largest
circular path followed by these electrons is 10.0 mm, the
logλmin

logλmin

(a) (b)
work function of the metal is close to
(a) 1.8 eV (b) 1.1 eV (c) 0.8 eV (d) 1.6 eV
log V log V
13 The surface of a metal is illuminated with the light of
400 nm. The kinetic energy of the ejected photoelectrons
logλmin

logλmin

(c) (d) was found to be 1.68 eV. The work function of the metal
is (hc = 1240 eV-nm)
log V log V
(a) 3.09 eV (b) 1.42 eV (c) 151 eV (d) 1.68 eV

ANSWERS
SESSION 1 1 (a) 2 (c) 3 (a) 4 (b) 5 (c,d) 6 (d) 7 (c) 8 (b) 9 (a) 10 (d)
11 (c) 12 (a) 13 (c) 14 (d) 15 (b) 16 (b) 17 (a) 18 (b) 19 (a) 20 (c)
21 (a) 22 (b) 23 (d) 24 (b) 25 (a) 26 (d) 27 (d)

SESSION 2 1 (a) 2 (b) 3 (b) 4 (c) 5 (d) 6 (a,c) 7 (d) 8 (a) 9 (d) 10 (a)
11 (a) 12 (b) 13 (b)

Hints and Explanations


SESSION 1 6. 63 × 3 3 Intensity A = Intensity B
= × 10−18
59
1 4 × 10 = 10 × hf
3 20
The number of photons of beam A = n A
Light energy produced per second
4 × 103 90 The number of photons of beam B = n B
f = E = × 10
1020 × 6.023 × 10−34 100 According to question, n A = 2n B
f = 6.64 × 1016 Hz =9W Let ν A be the frequency of beam A and
E ν B be the frequency of beam B.
The obtained frequency lies in the band Number of photons emitted per sec =
E1 ∴ Intensity ∝ Energy of photons
of X-rays. I ∝ (hν) × Number of photons
9 × 59
= I A n Aν A
2 Energy of photon, 6. 63 × 3 × 10−18 ∴ =
hc 6. 63 × 10−34 × 3 × 108 IB nBνB
E1 = = = 2.67 × 1019 = 0.267 × 10 20
λ 590 × 10−9
DAY THIRTY ONE DUAL NATURE OF MATTER 347

According to question, I A = I B 14 As λ is increased, there will be a value 21 From quantum theory of light, as
ν n 1 of λ above which photoelectron will be intensity of light increases means
∴ n Aν A = n B ν B ⇒ A = B =
νB nA 2 cease to come out, so photocurrent will number of photons/area/time increases
So, νB = 2 ν A becomes zero. and hence more photons take part in
ejecting the photoelectron, thus
4 Sensitivity of eye = energy detected per 15 de-Broglie wavelength is, increasing the number of
square meter h h photoelectrons.
λ= = = 1 .7 Å
5 × 104 × 6.6 × 10−34 × 3 × 108 mv 2mqV
= 22 Velocity of first photon = u = c
500 × 10−9 hc
16 As, E = Velocity of second photon = v = − c
= 0.2 × 10−13 Wm −2 λ Now relative velocity of first photon
The sensitivity of ear = 1 × 10−13 Wm −2 hc 6.6 × 10−34 × 3 × 108 with respect to second photon
and λ = = u−v c − (− c )
Thus, the sensitivity of eye is five times E 11 × 1 . 6 × 10−19 = =
uv c × (− c )
more than that of the ear. = 1.125 × 10−7 m 1− 2 1−
c c2
5 For photo emission to take place, Hence, UV region.
=
2c
wavelength of incident light should be c2
less than the threshold wavelength of
17 de-Broglie wavelength is 1+ 2
h h c
ultraviolet light < 5200 Å while that of λ= =
mv 2mE 2c 2c
infrared radiations > 5200Å. = = =c
But E = eV 1+ 1 2
 
6 E k = hc − hc = hc  λ 0 − λ  λ=
h
⇒V =
h2 Also, the rest of mass of photon is zero.
λ λ0 λ
 0  λ 2 meV 2meλ2
23 When work function of copper is greater
7 As we know, (6.62 × 10−34 )2 than the work function of sodium,then
V =
eV 0 = h (ν − ν 0 ) and hν = KE + W 0 (0.5 × 10−10 )2 × 2 × 91
. φCu > φ Na
So, A → 2, B → 1,C → 3 × 10−31 × 1.6 × 10−19 (hνo )Cu > ( hνo ) Na …(i)
⇒ V = 601.98 V ≈ 602 V c
8 By changing distance the intensity But we know that, νo =
changes but frequency remains same, so 18 As, λ1 = h
, λ2 =
hc λc
stopping potential remains same. 2m pE E
Hence, Eq. (i), becomes
9 Threshold voltage for a and b is same λ1 h / 2m pE E  hc   hc 
∴ = =   >  (λo ) Na > ( λo )Cu
and it depends on frequency. So, f a = fb . λ2 hc / E 2m pe 2  λo  Cu  λo  Na
Photo current of a and b are unequal λ1 λ
and photo current depends on intensity. ⇒ ∝ E ⇒ 1 ∝ E 1 /2 24 de-Broglie wavelength associated with
λ2 λ2 gas molecules varies
So, I a ≠ Ib
1
19 Here, v = v o i B = Bo j λ∝
10 eV 0 = hc − W 0 and eV ′ = hc − W 0 T
λ0 2λ 0 Force on moving electron due to
magnetic field is Also, root mean square velocity of gas
Subtracting them, we have
hc  1 hc F = − e (v × B) = − e [v o i × Bo j] molecules is v rms =
3RT
.
e (V 0 − V ′ ) = 1−  = = − e v o Bo k
λ 0  2  2λ 0 M
hc As this force is perpendicular to v and
or V ′ = V0 − 25 Davisson and Germer experimentally
2 eλ 0 B, so the magnitude of v will not change
established wave nature of electron by
i.e. momentum (mv) will remain
observing diffraction pattern while
11 We have, eV 0 = hc − φ ⇒ V 0 = hc − φ constant is magnitude. Hence,
bombarding electrons on Ni crystal.
λ eλ e de-Broglie wavelength λ = h / mv
V 0 = mx + c remains constant. 26 Maximum kinetic energy (KE) max is
20 As, 2 πr = nλ ⇒ r = nλ given by (KE) max = hν − hν 0.
Q Data is not sufficient.
hc 2π
−φ When frequency is increased (KE) max
1 Q λ h
12 As, = Now, de-Broglie equation λ = will increase stopping potential is that
4 πε0 1 × 10−2 Q p negative voltage given to the anode at
h h nh which photocurrent stops, hence
⇒ Q = 5.5 × 10−13 C ⇒ mvn = = =
λ 2 πr n 2 πr n doubling frequency will not effect it, also
13 (A) Franck-Hertz experiments is n If ν′ = 2 ν
associated with discrete energy levels of Also, for charged particle moving in a ∴ K ′ max = eν′ 0 = h (2ν1 − ν 0 )
atom. magnetic field K ′ max = 2K max + hν 0
mv n nh ∴ K ′ max > 2 K max ⇒ ν′ 0 > 2 ν 0
(B) Photo-electric experiment is rn = =
associated with particle nature of light. qB (2πr n ) q B Hence, (KE) max and stopping potential
nh are linearly dependent on the frequency
(C) Davisson-Germer experiment is ⇒ r n2 =
associated with wave nature of electron. 2π q B of incident light.
∴ r n ∝ n1 /2
348 40 DAYS ~ JEE MAIN PHYSICS DAY THIRTY ONE

27 Since, the frequency of ultraviolet light 4 Given, wavelength 1 1 1


: : = 1 :2 : 4
is less than the frequency of X-rays, the λ1 = 600 nm = 600 × 10−9 m λ 01 λ 02 λ 03
energy of each incident photon will be Energy correspond to λ1 = E1 1
more for X-rays = 0.001 nm −1
Again, wavelength λ0
KE photoelectron = hν − φ λ2 = 400 nm = 400 × 10−9 m
°
Stopping potential is to stop the fastest Energy correspond to λ2 = E2 λ 0 = 10,000 A
photoelectron Let the work function of metal is φ. 1
hν φ = 0.002 nm −1
V0 = − According to question 2E1 = E2 λ 02
e e
 hc  hc
So, KE max and V 0 both increases. i.e. 2 − φ0  = − φ0 or λ 02 = 5000 A
°
λ
 1  λ2
But KE ranges from zero to KE max 1
because of loss of energy due to or
2 hc

hc
= 2φ 0 − φ 0 = 0.004 nm −1
λ1 λ2 λ 03
subsequent collisions before getting
ejected and not due to range of 2 1  °
− λ 03 = 2500 A
frequencies in the incident light. or hc   = φ0
 λ1 λ2 
°
φ 0 = 6.63 × 10−34 × 3 × 108 Violet colour has wavelength 4000A.
SESSION 2
  So, violet colour can eject
1 Here, the initial velocity is u = 0 2 1
 −9
− −9 
photoelectrons from metal 1 and
eE  600 × 10 400 × 10  metal 2.
Since, a = , v = ? at t = t
m 6.63 × 10−34 × 3 × 108  1 1 
eE or = 3 − 4 
7 We have, E = qV , we know that
So we get, using v = u + at = 0 + t 10−7  
1 2E
m
−19  4 − 3 
E = mv 2 ⇒ v =
h h h or = 6.63 × 3 × 10  12  J 2 m
This gives λ = = =  
mv m(eEt /m ) eEt h h
as, λ= =
6.63 × 3 × 10−19 mv 2E
The rate of change of de-Broglie = = 1.02 eV m
wavelength is 1.6 × 10−19 × 12 m
dλ h  −1  −h ⇒ λ=
h
= ×  = 5 According to the Einstein’s photoelectric …(i)
dt eE  t 2  eEt 2 effect 2mqV
h
2 From Einstein’s photoelectric equation, hν − hν 0 =
1
mv 2 = eV For electron, λe = …(ii)
we have 2 2me qV

1 1   hc hc  For protron λ p =
h
eV 0 = hc  − …(i) ⇒  −  = eV
  λ λ0  2mp qV
 λ λ0 
1 where, λ 0 = threshold wavelength. h
eV 0 1  ∴ λp =
= hc  −  …(ii)
Now for the first case, 2mp q ⋅ 4V
6  3λ λ 0 
hc hc (Q V = 4 V) …(iii)
On dividing Eq. (i) by Eq. (ii), we get − = e (3V 0 ) …(i)
λ λ0 On dividing Eq. (ii) by Eq. (iii), we get
1 1 
 − 
 λ λ0  For the second case, λe mp
6= hc hc =2
 1 1  − = e (V 0 ) …(ii) λp me
 −  2λ λ0
 3λ λ 0  8 If wavelength of n
On dividing Eq. (i) by Eq.(ii), we get
6 6 1 1 1 5 emitted photon in
− = − ⇒ = 1 1 
<

n
3λ λ 0 λ λ 0 λ λ0 hc  −  de-excitation is
 λ λ0  3eV 0 1
λ0 = Λn;
⇒ = 5 ⇒ λ 0 = 5λ  1 1  eV 0
λ hc  −  hc
Then, = En − Eg
 2λ λ 0  Λn
3 Photoelectric current I is directly λ0 − λ
proportional to intensity of light and hc p2 p2g  p2 
λλ 0 3eV 0 = n − Q E = 
intensity ∝
1 ⇒ = Λ n 2m 2m  2m 
λ 0 − 2λ eV 0
(distance)2
2λ ⋅ λ 0 As energies are negative, we get
1
I ∝ 2
p2g  p  
2
λ 0 = 4λ p2g p2
1−  n 
r hc
= − n =
1 Λ n 2m 2m 2m   p  
 g 
I25 ∝ …(i) 6 From the relation, 
(25)2 hc  hc  1 φ h2  λg 
2
1 eV = − φ or V =   − = 1 − 2  [Q p ∝ λ−1 , p = ]
h
I100 ∝ [1 m = 100 cm]…(ii) λ  e λ e
(100)2 2mλ2g  λ n  λ
This is the equation of straight line −1
I25 (100)2 I 2mλ2g c  λ2 
∴ = = 16 ⇒ I100 = 25 slope is tan θ = hc / e 1 − g 
I100 (25)2 16 ⇒ Λn =
hc hc hc h  λ2n 
φ1 : φ2 : φ3 = : :
λ 01 λ 02 λ 03
DAY THIRTY ONE DUAL NATURE OF MATTER 349

2mλ2g c  λ2  vB − v A 12 When an electron moves in a circular


⇒ Λn = 1 + g  ∴1= ⇒ v = vB − v A K (ii)
h  λ2n 
v path, then
From Eq. (i) and Eq. (ii), mv r 2e 2 B 2 m2 v 2
[Q (1 − x )− n = 1 + nx] v 4v Radius, r = ⇒ =
vA = and v B = eB 2 2
B 3 3 (mv )2 r 2e 2 B 2
⇒ Λn ~
− A+ KE max = ⇒ = (KE )max
λ2n  h 
  2m 2m
 2mcλ2g  λ  mV A 
Hence, A = Work function of the metal (W),
where, A =   and B λB  h 
 h    i.e. W = hν − KE max
 m 
 . V  r 2e 2 B 2 1
 2mcλ4g  
 2
B
 1.89 − φ = eV

=  are constants. 2m 2
 h  VB 4/3 r 2eB 2
= = =2 = eV
hc 2V A 2 / 3 2m
9 λ min = [hν → 1.89 eV, for the transition on
eV
11 According to Einstein’s photoelectric from third to second orbit of H-atom]
 hc  emission of light, E = (KE) max + φ
log (λ min ) = log   − log V 100 × 10−6 × 1.6 × 10−19 × 9 × 10−8
 e  hc =
As, = (KE ) max + φ 2 × 91 . × 10−31
y = c − mx λ 1.6 × 9
So, the required graph is given in If the wavelength of radiation is changed φ = 1.89 −
option (d). 3λ 2 × 91.
to , then = 1.89 − 0.79 = 1.1 eV
10 For elastic collision, 4
4 hc  4 φ 13 Q KE max = eV 0
p before collision = pafter collision . ⇒ =  (KE ) max +  + φ
3 λ 3 3 1
mv = mv A +
m
vB ⇒ mv 2 = eV 0 = 1.68 eV
2 For fastest emitted electron, 2
1 hc 1240 eV -nm
2v = 2v A + v B K (i) (KE ) max. = mv ′ 2 + φ ⇒ hν = = = 3.1 eV
Now, coefficient of restitution, 2 λ 400 nm
v − vA 1 41  φ ⇒ 3.1 eV = W 0 + 1.68 eV
e = B ⇒ mv ′2 =  mv 2  +
uA − vB 2 3 2  3 [From Einstein equation, E = W 0 + K max ]
 4
1 /2 W 0 = 1.42 eV
Here, u B = 0 (Particle at rest) and for i.e. v′ > v 
elastic collision e = 1  3
DAY THIRTY TWO

Atoms
Learning & Revision for the Day
u Scattering of α-particles u Bohr’s Model
u Rutherford‘s Model of an Atom u Energy Levels and Hydrogen Spectrum

Atom is the smallest particle of an element which contains all properties of element.
Molecule is a single atom or a group of atoms joined by chemical bonds. It is the smallest
unit of a chemical compound that can have an independent existence. Nuclei refers to a
nucleus of an atom, having a given number of nucleons. It is a general term referring to
all known isotopes–both stable and unstable of the chemical elements. Thus, O16 and
O17 are different nuclides.

Scattering of α-particles
In 1911, Rutherford successfully explained the scattering of α-particles on the basis of
nuclear model of the atom.
Number of α-particles scattered through angle θ is given by
Z2
N (θ) ∝
sin (θ /2) K 2
4

where, K is the kinetic energy of α-particle and Z is the atomic number of the metal.
At distance of closest approach the entire initial kinetic energy is converted into
potential energy, so
1 1 Ze(2e)
mv2 =
2 4πε 0 r0
Ze2 4KZe2
⇒ r0 = =
mv πε 0
2
mv2 PREP
MIRROR
Rutherford’s Model of an Atom Your Personal Preparation Indicator

On the basis of scattering of α-particles, Rutherford postulated the following model of the u No. of Questions in Exercises (x)—
atom u No. of Questions Attempted (y)—
l
Atom is a sphere of diameter about 10 −10 m. Whole of its positive charge and most of u No. of Correct Questions (z)—
its mass is concentrated in the central part called the nucleus. (Without referring Explanations)
−5
l
The diameter of the nucleus is of the order of 10 m.
u Accuracy Level (z / y × 100)—
l
The space around the nucleus is virtually empty with electrons revolving around the u Prep Level (z / x × 100)—
nucleus in the same way as the planets revolve around the sun.
l
The electrostatic attraction of the nucleus provides centripetal force to the orbiting In order to expect good rank in JEE,
your Accuracy Level should be above
electrons. 85 & Prep Level should be above 75.
DAY THIRTY TWO ATOMS 351

l
Total positive charge in the nucleus is equal to the total l
The total energy of the orbital electron is
negative charge of the orbiting electrons.  me 4 Z 2 
E = − 2 2 2 
l
Rutherford’s model suffers from the following drawbacks  8ε 0 h n 
(a) stability of the atomic model.
(b) nature of energy spectrum.  me 4  Z 2
= −  2 3  ch 2
 8ε 0ch  n

Bohr’s Model Z2 Z2
= − Rch = − 13.6 eV
n2 n2
Bohr added the following postulates to the Rutherford’s model
of the atom me 4 Z 2 me 4 Z 2
KE = , PE = −
l
The electrons revolve around the nucleus only in certain 8 n2 h2 ε20 4 n2 h2 ε20
permitted orbits, in which the angular momentum of the l
The kinetic, potential and total energies of the electron
electron is an integral multiple of h /2 π , where h is the with r as the radius of the orbit are as follows
Planck’s constant
1  1 Ze2 
 nh KE =
 L = mvnrn =

 2  4πε 0 r 
2π 
1 Ze2
l
The electrons do not radiate energy while revolving in the PE = −
permitted orbits. That is, the permitted orbits are stationary, 4πε 0 r
non-radiating orbits. 1  1 Ze2 
and E =−
The energy is radiated only when the electron jumps from 2  4πε 0 r 
l

an outer permitted orbit to some inner permitted orbit.


(Absorption of energy makes the electron jump from inner Therefore, they are related to each other as follows
orbit to outer orbit). KE = − E and PE = 2 E
l
If energy of the electron in nth and mth orbits be E n and E m 1 1
l
For a hydrogen atom, rn ∝ n2, vn ∝ and | E | ∝ 2
respectively, and when the electron jumps from nth to mth n n
orbit the radiation frequency ν is emitted, such that l
The difference in angular momentum associated with the
E n − E m = hν. This is called the Bohr’s frequency equation.
electron in the two successive orbits of hydrogen atom is
h nh h
∆L = (n + 1) − =
NOTE • Radius of the orbit of electron in a hydrogen atom in its 2π 2π 2π
stable state, corresponding to n = 1, is called Bohr‘s radius.
Value of Bohr‘s radius is r0 = 0 . 529 Å ≈ 0.53 Å .
• The time period of an electron in orbital motion in the
Bohr’s orbit is
Energy Levels and Hydrogen
T=
2 π r 2 π × 0.53Å
= = 1.52 × 10 − 6 s Q v =
c  Spectrum
v c  137  Hydrogen spectrum consists of spectral lines classified as five
137 spectral series of hydrogen atom. Out of these five, Lyman
1
and the frequency of revolution is f = = 6.5757 × 10 15 cps series lies in the ultraviolet region of spectrum, Balmer series
T
lies in the visible region and the remaining three series, lie in
the infrared region of spectrum.
Some Characteristics of an Atom
n=∞
l
The orbital radius of an electron is 0
– 0.28 n=7
2 2 2
nh n n=6
rn = 4πε 0 = 0.53 Å – 0.38
Pfund
4 π Zme
2 2
Z – 0.54 n=5
E (eV)

Brackett n=4
l
The orbital velocity of an electron is – 0.85
–1.51
Paschen n=3
1 2 Zπ e 2
 c  Z 6 Z  Infrared
vn = =  = 2.2 × 10   m/s
4πε 0 nh  137 n  n Balmer n=2
–3.40
Visible light
l
Orbital frequency is given by
Lyman
1 v me 4 series
f = = = n=1
T 2πr 4 ε20 n3 h3 –13.60
Ultraviolet
Emission spectrum of H-atom
352 40 DAYS ~ JEE MAIN PHYSICS DAY THIRTY TWO

Total number of emission spectral lines from some excited state 1  1 1


n, to another energy n2 (< n1 ) is given by = ν = R  2 − 2
λ  (5) n 
(n1 − n2 )(n1 − n2 + 1)
where, n = 6, 7, 8, ...
2
n (n − 1) • Energy of emitted radiation,
e.g. total number of lines from n1 = n to n2 = 1 are . NOTE
2
 1 1  1 1
The five spectral series of hydrogen atom are given below ∆E = E 2 − E 1 = RchZ 2  2 − 2  = 136
. Z2  2 − 2 
 n1 n2   n1 n2 
1. Lyman Series ∆E  1 1
• Frequency, ν = = RcZ 2  2 − 2 
Spectral lines of Lyman series correspond to the transition of h  n1 n2 
electron from higher energy levels (orbits) ni = 2, 3, 4, … to
ground energy level (1st orbit) nf = 1.
Ionisation Energy and Potential
1  1 1
For Lyman series, = v = R  2 − 2  Ionisation energy of an atom is defined as the energy
λ (1) n  required to ionise it i.e. to make the electron jump from its
where, n = 2, 3, 4, … present orbit to infinity.
It is found that a term Rch = 13.6 eV = 2.17 × 10 −18 J. The term Thus, ionisation energy of hydrogen atom in the ground
state = E ∞ − E1
Rch is known as Rydberg’s energy.
= 0 − (−13.6 eV) = + 13.6 eV
2. Balmer Series The potential through which an electron is to be accelerated
Electronic transitions from ni = 3, 4, 5, … to nf = 2, give rise to so that it acquires energy equal to the ionisation energy is
called the ionisation potential.
spectral lines of Balmer series.
For a Balmer series line, Therefore, ionisation potential of hydrogen atom in its
ground state is 13.6V.
1  1 1
= ν = R  2 − 2 z2 E
λ  (2) n  In general, E ion = 13.6 2 eV or Vion = ion
n e
where, n = 3, 4, 5, …

3. Paschen Series Excitation Energy and Ionisation


Lines of this series lie in the infrared region and correspond to Potential
electronic transition from ni = 4, 5, 6,… to nf = 3. Excitation energy is the energy required to excite an
1  1 1 electron from a lower energy level to a higher energy level.
= ν = R 2 − 2 The potential through which an electron is accelerated so as
λ (3) n 
to gain requisite ionisation energy is called the ionisation
where, n = 4, 5, 6, ... potential.
Thus, first excitation energy of hydrogen atom
4. Brackett Series
= E2 − E1
Spectral lines in the infrared region which corresponds to
transition from ni = 5, 6, 7, … to nf = 4. = − 3.4 − (− 13.6) eV
For Brackett series, = + 10.2 eV
1  1 1 Similarly, second excitation energy of hydrogen atom
= ν = R 2 − 2 = E3 − E1
λ  (4) n 
= − 1.51 − (−13.6)
where, n = 5, 6, 7, …
= 12.09 eV
5. Pfund Series
NOTE • Total energy of a closed system is always negative and
It lies in the far infrared region of spectrum and corresponds to its magnitude is the binding energy of the system.
electronic transitions from higher orbits
• Kinetic energy of a particle can’t be negative, while the
ni = 6, 7, 8, ... to orbit having nf = 5. potential energy can be zero, positive or negative.
For a spectral line in Pfund series,
DAY THIRTY TWO ATOMS 353

DAY PRACTICE SESSION 1

FOUNDATION QUESTIONS EXERCISE


1 An α-particle of energy 5 MeV is scattered through 180° 10 As an electron makes a transition from an excited state to
by a fixed uranium nucleus. The distance of the closest the ground state of a hydrogen like atom/ion
approach is of the order of ª JEE Main 2015
− 10
(a) 1Å (b) 10 cm (a) its kinetic energy increases but potential energy and
(c) 10− 12 cm (d) 10−15 cm total energy decrease
(b) kinetic energy, potential energy and total energy
2 To explain theory of hydrogen atom, Bohr considered
decrease
(a) quantisation of linear momentum (c) kinetic energy decreases, potential energy increases but
(b) quantisation of angular momentum total energy remains same
(c) quantisation of angular frequency
(d) kinetic energy and total energy decrease but potential
(d) quantisation of energy
energy increases
3 For the Bohr’s first orbit of circumference 2πr, the
11 An electron jumps from the 4th orbit to the 2nd orbit of
de-Broglie wavelength of revolving electron will be
hydrogen atom. Given the Rydberg’s constant
1 1
(a) 2 πr (b) πr (c) (d) R = 107 cm −1, the frequency (in hertz) of the emitted
2 πr 4 πr
radiation will be
4 Which of the following transitions in hydrogen atoms emit 3 3 9 3
(a) × 105 (b) × 1015 (c) × 1015 (d) × 1015
photons of highest frequency? 16 16 16 4
(a) n = 2 to n = 6 (b) n = 6 to n = 2
12 In figure, the energy levels of the hydrogen atom have
(c) n = 2 to n = 1 (d) n = 1to n = 2
been shown along with some transitions marked
5 In the Bohr’s model of the hydrogen atom, let r, V and E A, B and C. The transitions A, B and C, respectively
represents the radius of the orbit, the speed of electron represents
and the total energy of the electron, respectively. Which
Continuum 0 eV
of the following quantities is proportional to the quantum
number n? n=5 –0.54 eV
(a) E/ v (b) r/E (c) vr (d) rE n=4 –0.85 eV
C
6 The ratio of the kinetic energy to the energy of an n=3 –1.51 eV
B
electron in a Bohr’s orbit is n=2 –3.40 eV
(a) − 1 (b) 2
A
(c) 1 : 2 (d) None of these
7 If the atom 100Fm 257 follows the Bohr’s model and the n=1 –13.60 eV
257
radius of last orbit of 100Fm is n times the Bohr’s (a) the first member of the Lyman series, third member of
radius, then find the value of n ? Balmer series and second member of Paschen series
(a) 100 (b) 200 (c) 4 (d) 1/4 (b) the ionisation potential of H, second member of Balmer
8 Taking the Bohr’s radius as a 0 = 53 pm, the radius of Li 2+ series and third member of Paschen series
ion in its ground state, on the basis of Bohr’s model, will (c) the series limit of Lyman series, second member of
be about Balmer series and second member of Paschen series
(d) the series limit of Lyman series, third member of Balmer
(a) 53 pm (b) 27 pm
series and second member of Paschen series
(c) 18 pm (d) 13 pm
13 The given diagram indicates the 2E
9 In a hypothetical Bohr’s hydrogen atom, the mass of the 4/3E
energy levels of a certain atom.
electron is doubled. The energy E 0 and radius r0 of the E
When the system moves from 2E
first orbit will be (a 0 is the Bohr radius) 0
level to E, a photon of wavelength
(a) E 0 = − 27. 2 eV ; r0 = a0 / 2 λ is emitted. The wavelength of
(b) E 0 = − 27. 2 eV ; r0 = a0 photon produced during its transition from 4E/3 level to E
(c) E 0 = − 13.6 eV ; r0 = a0 / 2 is
(d) E 0 = − 13 .6 eV ; r0 = a0 (a) λ / 3 (b) 3 λ / 4 (c) 4 λ / 3 (d) 3λ
354 40 DAYS ~ JEE MAIN PHYSICS DAY THIRTY TWO

14 Energy E of a hydrogen atom with principal quantum 2 3


(a) r = (b) r =
13.6 3 4
number n is given by E = − 2 eV. The energy of a 1 4
n (c) r = (d) r =
3 3
photon ejected when the electron jumps from n = 3 state
to n = 2 state of hydrogen, is approximately 21 Energy required for the electron excitation in Li2+ from the
first to the third Bohr orbit is ª AIEEE 2011
(a) 1.5 eV (b) 0.85 eV (c) 3.4 eV (d) 1.9 eV
(a) 36.3 eV (b) 108.8 eV
15 The ionisation potential of H-atom is 13.6 V. When it is (c) 122.4 eV (d) 12.1 eV
excited from ground state by monochromatic radiations
of 970.6 Å, the number of emission lines on deexcitation 22 In hydrogen atom, if the difference in the energy of the
will be (according to Bohr’s theory) electron in n = 2 and n = 3 orbits is E, the ionisation
energy of hydrogen atom is
(a) 10 (b) 3 (c) 6 (d) 4
(a) 13.2 E (b) 7.2 E
16 In a hydrogen like atom electron makes transition from an (c) 5.6 E (d) 3.2 E
energy level with quantum number n to another with
quantum number (n −1). If n >> 1, the frequency of 23 Excitation energy of a hydrogen like ion in its first
radiation emitted is proportional to ª JEE Main 2013 excitation state is 40.8 eV. Energy needed to remove the
1 1 1 1 electron from the ion in ground state is
(a) (b) (c) (d)
n n2 n3 / 2 n3 (a) 54.4 eV (b) 13.6 eV
(c) 40.8 eV (d) 27.2 eV
17 Hydrogen atom is excited from ground state to another
state with principal quantum number equal to 4. Then, Direction (Q. Nos. 24-25) Each of these questions contains
the number of spectral lines in the emission spectra will two statements : Statement I and Statement II. Each of these
be ª AIEEE 2012 questions also has four alternative choices, only one of which
(a) 2 (b) 3 (c) 5 (d) 6 is the correct answer. You have to select one of the codes (a),
(b), (c) and (d) given below
18 The wavelength of the first spectral line in the Balmer (a) Statement I is true; Statement II is true; Statement II is
series of hydrogen atom is 6561 Å. The wavelength of the correct explanation for Statement I
the second spectral line in Balmer series of single (b) Statement I is true; Statement II is true; Statement II is
ionised helium atom is ª AIEEE 2011 not the correct explanation for Statement I
(a) 1215 Å (b) 1640 Å (c) 2450 Å (d) 4687 Å (c) Statement I is true; Statement II is false
19 The transition from the state n = 4 to n = 3 in a (d) Statement I is false; Statement II is true
hydrogen-like atom results in ultraviolet radiation. 24 Statement I Bohr had to postulate that the electrons in
Infrared radiation will be obtained in the transition from stationary orbits around the nucleus do not radiate.
ª AIEEE 2010
Statement II According to classical physics all moving
(a) 2 → 1 (b) 3 → 2 (c) 4 → 2 (d) 5 → 3
electrons radiate electromagnetic radiation.
20 Some energy levels of a –E
λ2 25 Statement I The different lines of emission spectra (like
molecule are shown in the –4/3E Lyman, Balmer, etc) of atomic hydrogen gas are
figure. The ratio of the λ1 produced by different atoms.
wavelengths r = λ 1 / λ 2 is given –2E
by ª JEE Main 2017 (Offline)
Statement II The sample of atomic hydrogen gas
–3E consists of millions of atoms.

DAY PRACTICE SESSION 2

PROGRESSIVE QUESTIONS EXERCISE


1 If the series limit frequency of the Lyman series is νL, the number of particles scattered through an angle of 60°
then the series limit frequency of the Pfund series is per minute by the same nucleus is
ª JEE Main 2018 (a) 28 per minute (b) 112 per minute
ν ν (c) 12.5 per minute (d) 7 per minute
(a) 25 νL (b) 16 νL (c) L (d) L
16 25 3 A hydrogen like ion having wavelength difference
2 In Rutherford’s experiment, the number of alpha particles between first Balmer and Lyman series equal 593 Å has
scattered through an angle of 90° is 28 per minute. Then, Z equal to
(a) 2 (b) 3 (c) 4 (d) 1
DAY THIRTY TWO ATOMS 355

4 The largest wavelength in the ultraviolet region of the 10 A hydrogen atom moves with a velocity u and makes a
hydrogen spectrum is 122 nm. The smallest wavelength head on inelastic collision with another stationary H-atom.
in the infrared region of the hydrogen spectrum (to the Both atoms are in ground state before collision.The
nearest integer) is minimum value of u if one of them is to be given a
(a) 802 nm (b) 823 nm (c) 1882 nm (d) 1648 nm minimum excitation energy is
(a) 2.64 × 104 ms −1 (b) 6.24 × 104 ms −1
5 Energy levels A, B and C of a certain atom (c) 2.02 × 106 ms −1 (d) 6.24 × 108 ms −1
corresponding to increasing values of energy i.e.
E A < E B < E C . If λ 1, λ 2 and λ 3 are the wavelengths of 11 In the Bohr’s model an electron moves in a circular orbit
radiations corresponding to the transitions C to B, B to A around the proton. Considering the orbiting electron to be a
circular current loop, the magnetic moment of the hydrogen
and C to A respectively, which of the following
atom, when the electron is in nth excited state, is
statements is correct?
e  n 2h e n 2h
(a)  (b)   (c) 
e  nh
(d)  
e nh
C  
λ1  2m  π m 2π  2m  2 π m 2π
B
λ2 λ3
12 A diatomic molecule is made of two masses m1 and m2
which are separated by a distance r. If we calculate its
A rotational energy by applying Bohr’s rule of angular
λ1λ 2 momentum quantisation, its energy will be given by (n is
(a) λ 3 = λ1 + λ 2 (b) λ 3 =
λ1 + λ 2 an integer) ª AIEEE 2012
(c) λ1 + λ 2 + λ 3 = 0 (d) λ23 = λ21 + λ22 (m1 + m2 )2 n 2h 2 n 2h 2
(a) (b)
6 Hydrogen (1H1), deuterium (1H2 ), singly ionised helium 2m12m22r 2 2 (m1 + m2 )r 2
( 2He4 )+ and doubly ionised lithium ( 3Li6 )2+ all have one
2 2
2n h (m1 + m2 )n 2h 2
(c) (d)
electron around the nucleus. Consider an electron (m1 + m2 )r 2 2m1m2r 2
transition from n = 2 to n = 1. If the wavelengths of
13 In the Bohr’s model of hydrogen-like atom the force
emitted radiations are λ 1, λ 2, λ 3 and λ 4 respectively, then
between the nucleus and the electron is modified as
approximately which one of the following is correct?
e2  1 β
ª JEE Main 2014 F =  +  , where β is a constant. For this atom,
4πε 0  r 2 r 3 
(a) 4 λ1 = 2 λ 2 = 2 λ 3 = λ 4 (b) λ1 = 2 λ 2 = 2 λ 3 = λ 4
(c) λ1 = λ 2 = 4 λ 3 = 9λ 4 (d) λ1 = 2 λ 2 = 3 λ 3 = 4 λ 4 the radius of the nth orbit in terms of the Bohr’s radius
 ε h2 
7 The potential energy between a proton and an electron is a 0 = 0 2  is
 m πe  ª AIEEE 2010
e2
PE = , then the radius of the Bohr’s orbit is (a) rn = a0 n − β (b) rn = a0 n 2 + β
4πε 0( 3R 3 )
(c) rn = a0 n 2 − β (d) rn = a0 n + β
4 π 2e 2m 6 π 2e 2m
(a) (b)
4 πε0n h
2 2
4 πε0n 3h 3 Direction (Q. Nos. 14-15) Each of these questions contains
e 2m 2 πe 2m two statements : Statement I and Statement II. Each of these
(c) (d) questions also has four alternative choices, only one of which
4 πε0h 3 4 πε0nh 3
is the correct answer. You have to select one of the codes (a),
8 A small particle of mass m moves such that potential (b), (c) and (d) given below
1 (a) Statement I is true; Statement II is true; Statement II is
energy PE = mr 2ω 2. Assuming Bohr’s model of
2 the correct explanation for Statement I
quantisation of angular momentum and circular orbit, (b) Statement I is true; Statement II is true; Statement II is
radius of nth orbit is proportional to not the correct explanation for Statement I
1 1 (c) Statement I is true; Statement II is false
(a) n (b) n 3 (c) (d)
n (d) Statement I is false; Statement II is true
n3
9 The electric potential between a proton and an electron 14 Statement I Balmer series lies in the visible region of
r electromagnetic spectrum.
is given by V = V0 ln , where r0 is a constant. Assuming 1  1 1
r0 Statement II = R  2 − 2  , where n = 3 , 4 , 5 ,……
λ 2 n 
Bohr’s model to be applicable, write variation of rn with n,
n being the principal quantum number. 15 Statement I The ionisation potential of hydrogen is found
1 to be 13.6 eV, the ionisation potential of doubly ionised
(a) rn ∝ n (b) rn ∝ lithium is 122.4 eV.
n
1 Statement II Energy in the nth state of hydrogen atom is
(c) rn ∝ n 2 (d) rn ∝ 13.6
n2 En = − 2 .
n
356 40 DAYS ~ JEE MAIN PHYSICS DAY THIRTY TWO

ANSWERS

SESSION 1 1 (c) 2 (b) 3 (a) 4 (b) 5 (c) 6 (a) 7 (d) 8 (c) 9 (a) 10 (a)
11 (c) 12 (d) 13 (d) 14 (d) 15 (c) 16 (d) 17 (d) 18 (a) 19 (d) 20 (c)
21 (b) 22 (b) 23 (a) 24 (b) 25 (b)

SESSION 2 1 (d) 2 (b) 3 (b) 4 (b) 5 (b) 6 (c) 7 (a) 8 (a) 9 (a) 10 (b)
11 (c) 12 (d) 13 (c) 14 (a) 15 (b)

Hints and Explanations


SESSION 1 6 The kinetic energy and total energy of æ ö
11 n = c = c . R ç 12 - 12 ÷
q q an electron are related as, KE = -E
1 Here, 1 mv 2 = 1 1 2 l è n1 n2 ø
2 4p e0 r KE 1 1
\ = -1 = 3 ´ 10 ´ 10 ç 2 - 2 ö÷
8 7æ
9 ´ 109 ´ (2e ) ´ (92e ) E è 2 4 ø
\ 5 MeV =
r æ 2ö 9
æQ 1 mv 2 = 5 MeV ö 7 rm = ç m ÷ (0.53 Å ) = (n ´ 0.53) Å = ´ 1015 Hz
ç ÷ è Z ø 16
è 2 ø
m2 12 A represents series limit of Lyman series,
9 ´ 109 ´ 2 ´ 92 ´ (1.6 ´ 10-19 )2 \ =n B represents third member of Balmer
Þr = Z
5 ´ 106 ´ 1.6 ´ 10– 19 series and C represents second member
m = 5 (for 100 fm257 the outermost shell)
of Paschen series.
. ´ 10- 14 m
r = 53 and Z = 100
-12 (5) 2 1 13 l µ 1
» 10 cm \ n= = DE
100 4 l¢ (2E - E ) 1
2 While proposing his theory of hydrogen \ = = =3
8 On the basis of Bohr’s model, l (4E /3 - E ) 1/3
atom, Bohr considered quantisation of
n2 l2 a0n2 \ l ¢ = 3l
angular momentum as the essential r = 2 2
=
4p mKZe Z
condition for the stationary orbits.
Let Li 2+ ion, Z = 3, 14 Given, E n = - 132.6 eV
n
3 According to Bohr’s first postulate, n = 1 for ground state. 13.6 13.6
nh E3 = - eV = -eV
mvr = Given, a 0 = 53 pm (3) 2 9
2p
h ö 53 ´ (1)2 and E2 = -
13.6
eV = -
13.6
eV
2pr = n æç r =
\ ÷ = nl 3 (2) 2 4
è mv ø
~ 13.6 æ 13.6 ö
For n = 1, l = 2pr - 18 pm So, DE = E3 - E2 = - - ç- ÷
9 è 4 ø
4 Emission spectrum would rises when 9 As, r µ 1 = 1.9 eV (approximately)
electron makes a jump from higher m
1 hc 12400
energy level to lower energy level. \ r 0 = a0 15 Using E = = eV = 1277 . eV
2 l 970.6
Frequency of emitted photon is
As, E µ m So, electron is excited upto n = 4
proportional to change in energy of two
\ E 0 = 2 (- 13.6) \ n2 = 4
energy levels, i.e.
æ1 = - 27.2 eV On deexcitation number of emission
1 ö
n = RcZ 2 ç 2 - 2 ÷ lines produced =
n (n - 1)
=6
n
è 1 n 2ø 10 As we know that, kinetic energy of an 2
electron is KEµ (Z /n )2 . When the
5 According to the Bohr’s postulate. 16 D E = hn
nh h ö electron makes transition from an
mvr = Þ vr = n æç ÷ excited state to the ground state, then n DE é 1 1 ù
2p è 2pm ø n= = Kê - 2ú
decreases and KE increases. We know h 2
Þ vr µ n ë (n - 1) n û
that PE is lowest for ground state. K 2n 2K
Thus, vr is directly proportional to the = 2 = 3
Also, TE = - KE. TE also decreases. n (n - 1)2 n
principal quantum number.
DAY THIRTY TWO ATOMS 357

17 In emission spectrum, number of bright 25 A single atom can have only one 36 4
593 = -
lines is given by transition at time, we are observing 5RZ 2 3RZ 2
n(n - 1 ) 4(4 - 1 ) different lines due to large number of 88
= =6 Z2 =
transitions taking place simultaneously 15R Dl
2 2
that occurred in different atoms of the
1 1 1 5R 88
18 = R æç - ö÷ = sample. =
6561 è 4 9 ø 36 15 (1.097 ´ 107 ) (593 ´ 10-10 )
1 1 1
= 4R æç - ö÷ =
3R ´ 4 SESSION 2
è 4 16 ø =9
l 16
1 Series limit occurs in the transition
l = 1215Å n2 = ¥ to n1 = 1 in Lyman series and ÞZ = 3

19 Infrared radiation corresponds to least n2 = ¥ to n1 = 5 in Pfund series. For


Lyman series, 4 The series in UV-region is Lyman series.
æ1 1 ö Longest wavelength corresponds to
value of ç 2 - 2 ÷, i.e. from Paschen, n2=∞
è n1 n2 ø minimum energy which occurs in
transition from n = 2 to n = 1.
Brackett and Pfund series. Thus, the hνL = Eg= E0 12 1
transition corresponds to 5 ® 3. 1 ∞ 1
= 13.6 eV \ 122 = R …(i)
20 We have, l = hc n 1= 1 1
- 2
1
DE 2
hn L = 13.6 …(i) (1) (2)
\ So, ratio of wavelengths
In Pfund series The smallest wavelength in the infrared
æ4 E - Eö
ç ÷ n2= ∞ region corresponds to maximum energy
l1 hc / DE1 DE2 è 3 ø 1
= = = = of Paschen series.
l2 hc / DE2 DE1 2E - E 3
13.6 1
hνp = E0 12 1
= 2
æ ö ∞
21 DE = 13.6Z 2 ç 12 - 12 ÷ 5 5 \ l=
1
R
1
…(ii)
è n1 n2 ø n1 = 5 -
(3)2 ¥
é æ1 1 öù 13.6
êQ DE = Rhc ç 2 - 2 ÷ ú hn p = …(ii) Solving Eqs. (i) and (ii), we get
ë è n1 n 2 øû 52
l = 823.5 nm » 823
1 1 From Eqs. (i) and (ii), we get
= 13.6(3)2 æç 2 - 2 ö÷ = 108.8 eV
è1 3 ø 25hn p = hn L 5 Let energy corresponding to state A, B
22 E3 - E2 = E n and C be E A, E B and EC .
\ np = L
25 So, from figure
E1 E1
or - =E ( EC - E B ) + ( E B - E A ) = ( EC - E A )
9 4 2 According to Rutherford’s scattering
or E1 = -72
. E formula, if the a-particles scattered at hc hc hc
or + =
an angle q is directly proportional to l1 l2 l3
\ Ionisation energy of hydrogen atom
1 K
is 7.2 E. , then N q =
sin 4 (q /2) sin 4 (q / 2) Þ l3 =
l1 l2
23 Excitation energy, l1 + l2
when q = 90° ,
1 1
DE = E2 - E1 = 13.6 Z 2 æç 2 - 2 ö÷ N q = 28 min -1 C
è1 2 ø
Þ 28 =
K
= 4K Þ K = 7 λ1
3
40.8 = 13.6 Z 2 ´ sin 4 (45° ) B
4 λ2 λ3
7
\ Z =2 Thus Nq = A
sin 4 (q / 2)
So, required energy to remove the
electron from ground state Hence, the number of a-particles 6 As we know that,
13.6 Z 2 scattered at an angle of 60° per minute 1 1 1
=+ = 13.6(Z )2 = 54.4 eV = RZ 2 æç 2 - 2 ö÷
7 7 l è1 2 ø
(1)2 is N q¢ = =
sin 4 30° (1 / 2)4 4
24 Bohr’s postulated that, electron instead l=
= 7 ´ 16 = 112 per minute 3RZ 2
of revolving in any orbit around the
1 æ1 1 ö 4 4
nucleus, revolves only in some specific 3 = RZ 2 ç 2 - 2 ÷, l1 = , l2 =
orbits. These orbits are called the l è n1 n2 ø 3R 3R
non-radiating orbits or the stationary 1 1
Dl = - 4 4
orbits. The electrons revolving in these 2 æ1 1ö 2 æ1 1 l3 = , l4 =
orbits do not radiate any energy. They RZ ç - ÷ RZ ç - ö÷ 12R 27R
è 4 9ø è1 4 ø
radiate only when they go from one
Þ l1 = l2 = 4l3 = 9 l 4
orbit to the next lower orbit.
358 40 DAYS ~ JEE MAIN PHYSICS DAY THIRTY TWO

- dPE e2 mu2 e 2 æ r + b ö mv 2
7 F = =- = ç ÷= …(i)
dR 4pe0R 4 4 4p e0 è r 3 ø r
mv 2 e2 1 1 1
Þ = . mu2 = 13.6 æç 2 - 2 ö÷ h
4 è1 2 ø Also, mvr = …(ii)
R 4pe0R 4 2p
1
nh (1.0078) (1.66 ´ 10-27 )u2 From Eqs. (i) and (ii), we get
Also, mvR = 4
2p . ´ 1.6 ´ 10-19
= 102 e2 æ r + b ö mn2 h2
m æ nh ö
2
e2 ç 3 ÷=
\ ç ÷ = Þ 4
u = 6.24 ´ 10 ms -1 è
4p e0 r ø 4p2 m2 r 3
R è 2pmR ø 4pe0R 4
e0n2 h2
4p2 e 2 m 11 As, i = e Þ r+b =
Þ R= T p me 2
4pe0n2 h2 and magnetic moment M = iA
Þ r n = a0n2 - b
- dPE (Q A = pr 2 )
8 F = = - mw2 r . æ e h2 ö
dr e çQ a0 = 0 2 ÷
\ M = × pr 2 …(i)
nh T è mp e ø
Since, mvr = 2pr
2p Now, T = 14 The wavelength in Balmer series is given
v
nh by
or mr 2 w = [Q v = rw] It becomes,
2p e × pr 2 1 1 1
M = =
evr
…(ii) = R æç 2 - 2 ö÷, n = 3, 4, 5¼
nh l è2 n ø
Þ r2 = 2pr / v 2
2pmw nh 1 1 1 36
Also, mvr = = R æç 2 - 2 ö÷ Þ l max =
nh 2p l max è2 3 ø 5R
Þ r =
2pmw nh
vr = 36 ´ 1
2pm l max = = 6563 Å
Þ r µ n 5 ´ 1.097 ´ 107
Putting this value in Eq. (ii), we get
1 1 1
e × nh and = R æç 2 - 2 ö÷
9 U = eV = eV 0 ln ç r ÷
æ ö M =
2 × 2pm l min è2 ¥ ø
è r0 ø
e ö nh 4 4
= æç ÷ l min = =
and |F | = -
dU
=
eV 0 è 2m ø 2 p R 1.097 ´ 107
dr r = 3646 Å
12 Rotational kinetic energy of the two
This force will provide the necessary body system rotating about their centre The wavelengths 6563 Å and 3646 Å lie
centripetal force. Hence, of mass is
in visible region. Therefore, Balmer
mv 2 eV 0 1
= RKE = mw2 r 2 , series lies in visible region.
r r 2
æ nh ö 2
2 15 From Bohr’s theory, the energy of
eV 0 1
or v = …(i) \ RKE = m×ç 2
÷ r hydrogen atom in the nth state is given
m 2 è 2 pmr ø 13.6
by E n = - 2 eV. For an atom of atomic
nh æm = reduced mass = m1 m2 ö n
Moreover, mvr = …(ii) ç ÷
2p m1 + m2 number Z, with one electron in the outer
ç nh ÷
On dividing Eq. (ii) by Eq. (i), çL = = mω r 2
÷ orbit (singly ionised He or doubly
è 2p ø
we have 2 2 2 2 13.6 Z 2
n h n h lithium) we use. E n = - eV,
nh m
= = n2
mr = æç ö÷ 8p2mr 2 2mr 2
è 2p ø eV 0 where Z is the atomic number. Hence,
(m1 + m2 )n2 h2
= ground state energy of doubly ionised
or rn µ n 2m1 m2 r 2
- 13.6 ´ 9
10 Momentum, mu = 2mv é here, h 2 = l ù lithium is = - 122.4eV
êë (1)2
u 4p úû
Þ v =
2 13 According to question, the force Ionisation potential (potential to be
2 between nucleus and electron provide applied to electron to overcome this
1 1 u
DE = mu2 - (2m ) æç ö÷ necessary centripetal force, energy) is 122.4 V.
2 2 è2ø
EXAM BITES

This Pdf Is
Downloaded From
www.exambites.in

Visit www.exambites.in for


More Premium Stuffs,Latest
Books,Test Papers,Lectures etc.
jeeneetadda
jeeneetadda_official
jeeneetadda

VISIT NOW !!
DAY THIRTY THREE

Nuclei
Learning & Revision for the Day
u Concept of Nucleus u Mass Energy Relation u Nuclear Fission
u Radioactivity u Mass Defect and Binding Energy u Nuclear Fusion

Concept of Nucleus
In every atom, the positive charge and mass is densely concentrated at the centre of the
atom forming its nucleus. In nucleus, the number of protons is equal to the atomic number
of that element and the remaining particles to fulfil the mass number are the neutrons.

Composition of Nucleus
Nucleus consists of protons and neutrons. Electrons cannot exist inside the nucleus.
A proton is a positively charged particle having mass (m p) of 1.007276 u and charge
(+ e) = +1.602 × 10 –19 C.
For a neutral atom, Number of proton ( Z ) = Number of electron
This number is called the atomic number. A neutron is a neutral particle having mass
mn = 1.008665 u. The number of neutrons in the nucleus of an atom is called the neutron
number N. The sum of the number of protons and neutrons is called the mass number A.
Thus, A = N + Z .

Properties of Nucleus
Nuclear size
(a) Size of the nucleus is of the order of fermi (1 fermi = 10 −15 m ).
(b) The radius of the nucleus is given by R = R0 A1 /3 , PREP
where, R0 = 1.3 fermi and A is the mass number. MIRROR
Your Personal Preparation Indicator
Volume
u No. of Questions in Exercises (x)—
4
The volume of nucleus is V = π (R0 A1 /3 )3 , where, R0 = radius of the nucleus. u No. of Questions Attempted (y)—
3
u No. of Correct Questions (z)—
Density (Without referring Explanations)

Mass of nucleus Am p mp
(a) Density = = = u Accuracy Level (z / y × 100)—
Volume of the nucleus 4 4 Prep Level (z / x × 100)—
π (R0 A1 /3)3 πR03 u

3 3
where, m p = 1.6 × 10 −27 kg = mass of proton and R0 = 1.3 fermi. In order to expect good rank in JEE,
your Accuracy Level should be above
(b) Density of nuclear matter is of the order of 1017 kg/m3 . 85 & Prep Level should be above 75.
(c) Density of nuclear matter is independent of the mass number.
360 40 DAYS ~ JEE MAIN PHYSICS DAY THIRTY THREE

dN dN
Isotopes, Isobars and Isotones Mathematically,
dt
∝ N or
dt
= − λN

Isotopes Here, λ is a proportionality constant, known as the decay


Isotopes of an element are nuclides having same atomic constant (or disintegration constant). Unit of λ is s–1 or day –1
number Z, but different mass number A (or different neutron or year –1, etc.
number N) is called isotopes. 11 H, 21H, 31H and 611 C, 612C, 614C, etc., It can be shown that number of nuclei present after time t is
are isotopes. given by
N = N 0 e − λt
Isobars
where, N 0 = number of nuclei present at time t = 0.
Nuclides having same mass number A, but different atomic
Again, number of nuclei decayed in time t will be
number Z are called isobars. In isobars number of protons Z
as well as number of neutrons N differ but total nucleon (or N − N 0 = N 0 [e − λt − 1]
mass) number A = N + Z is the same. 31 H, 32He and 614 C, 14
7 N are = number of daughter nuclei produced at time t .
isobars.

Isotones Half-Life Period (T1/2 )


It is the time in which, activity of the sample falls to one-half of
Nuclides with different atomic number Z and different mass
its initial value.
number A, but same neutron number are called isotones.
T N R
3 4 198 197
1 H, 2He and 80 Hg, 79 Au are examples of isotones. Thus, for t = , N = 0 and R = 0
2 2 2
l
The half-life period is related to decay constant λ as
Radioactivity T1 /2 =
0.693
λ
Radioactivity is the phenomenon of spontaneous emission of l
After n half-lives, the quantity of a radioactive substance left
radiations by heavier nucleus. Some naturally occurring
intact (undecayed) is given by
radioactive substances are uranium, thorium, polonium, t
n
radium, neptunium, etc. In fact, all elements having atomic  1  1 T
number Z > 82 are radioactive in nature. N = N 0   = N 0   1 /2
 2  2
Radiations emitted by radioactive substances are of three
types, namely (i) α-particles, (ii) β-particles and (iii) γ- rays.
l
α-particles are positively charged particles with charge
Mean Life Period ( τ )
q α = + 2 e and mass mα = 4m p. Thus, α-particles may be l
Mean life of a radioactive sample is the time, at which both
1
considered as helium nuclei (or doubly charged helium N and R have been reduced to or e −1 or 36.8% of their
ions). Ionising power of α-particles is maximum, but their e
1
penetrating power is minimum. initial values. It is found that τ = .
λ
l
β-particles are negatively charged particles with rest mass
as well as charge same as that of electrons. But origin of l
Half-life T1 /2 and mean life τ of a radioactive sample are
β-particles is from the nucleus. Their ionising power is correlated as, T1/2 = 0.693 τ or τ = 1.44T1/2 .
lesser than that of α-particles, but speed as well as
penetrating power is much greater than that of α-particles.
Generally, β-decay means β − - decay. Activity
l
γ-rays are electromagnetic radiations of extremely short The activity of a radioactive substance is defined as the rate of
wavelengths. Thus, γ-rays travel with the speed of light. disintegration (or the count rate) of that substance.
Their ionising power is least, but penetrating power is Mathematically, activity is defined as
extremely high. These are not deflected either in an dN
R=− = λN = λN 0 e − λt = R0 e − λt
electric or a magnetic field. dt
where, R0 = λN 0 = initial value of activity.
Law of Radioactive Decay Units of activity are
According to Rutherford-Soddy’s law for radioactive decay,
l
1 becquerel = Bq = 1 disintegration per second (SI unit)
‘The rate of decay of a radioactive material at any instant is l
1 curie = 1 Ci = 3 . 7 × 1010 Bq
proportional to the quantity of that material actually present
at that time.’
l
1 rutherford = 1 Rd = 10 6 Bq
DAY THIRTY THREE NUCLEI 361

The shown figure show binding energy per nucleon versus


Mass Energy Relation
l

mass number. The nuclides showing binding energy per


In nuclear physics, mass is measured in unified atomic mass nucleon greater than 7.5 MeV/nucleon are stable.
units (u), 1 u being one-twelfth of the mass of carbon-12 atom
and equals 1.66 × 10–27 kg. It can readily be shown using NOTE • Nucleons attract each other when they are separated by a
E = mc2 that, 1 u mass has energy 931.5 MeV distance of 10 –14 m.
Thus, 1 u ≡ 931.5 MeV or c 2 = 931.5 MeV/u • The density of nucleus is of the order of 10 17 .

A unit of energy may therefore be considered to be a unit of


mass. For example, the electron has a rest mass of about Nuclear Fission
0.5 MeV.
Nuclear fission is the process of splitting of a heavy nucleus
If the principle of conservation of energy is to hold for nuclear
(235 239
92 U or 94 Pu) into two lighter nuclei of comparable
reactions it is clear that mass and energy must be regarded as
equivalent. The implication of E = mc2 is that any reaction masses along with the release of a large amount of energy
producing an appreciable mass decrease is a possible source − 200 MeV ) after bombardment by slow neutrons.
(~
of energy. A characteristic nuclear fission reaction equation for 235
92 U is
l
At the rest, mass energy of each of electron and positron, is 1
0 n(slow ) + 235
92 U → 236
92 U → 144
56Ba + 89
36 Kr + 3 10 n + Q
−31
E 0 = m0 c = 9.1 × 10
2
× (3 × 10 ) J = 0.51 MeV
82
In the fission of uranium, the percentage of mass converted
Therefore, an energy of atleast 1.02 MeV is needed for pair into energy is about 0.1% .
production.
Controlled Chain Reaction
Mass Defect and Binding Energy and Nuclear Reactor
The difference in mass of a nucleus and its constituent 235
l l
In the fission of one nucleus of 92 U, on an average,
nucleons is called the mass defect of that nucleus. Thus, 1
Mass defect, ∆M = Zm p + ( A − Z )mn − M 2 neutrons are released. These released neutrons may
2
where, M is the mass of a given nucleus.
further, trigger more fissions causing more neutrons being
l
Packing fraction of an atom is the difference between mass formed, which in turn may cause more fission. Thus, a self
of nucleus and its mass number per nucleon. Thus, sustained nuclear chain reaction is formed. To maintain
M−A
Packing fraction = . the nuclear chain reaction at a steady (sustained) level, the
A
extra neutrons produced, are absorbed by suitable neutron
l
The energy equivalent of the mass defect of a nucleus is absorbents like cadmium or boron.
called its binding energy. l
Neutrons formed as a result of fission have an energy of
Thus, binding energy, ∆Eb = ∆M c2
about 2 MeV, whereas for causing further fission, we need
= [Zm p + ( A − Z )mn − M ] c2 slow thermal neutrons having an energy of about 0.3 eV.
If masses are expressed in atomic mass units, then For this purpose, suitable material called a moderator is
∆Eb = ∆M × 931.5 MeV used, which slow down the neutrons. Water, heavy water
and graphite are commonly used as moderators.
= [Zm p + ( A − Z )mn − M ] × 931.5 MeV
l
A nuclear reactor is an arrangement in which nuclear
l
Binding energy per nucleon (∆E bn ) is the average energy
fission can be carried out through a sustained and a
needed to separate a nucleus into its individual nucleons. controlled chain reaction and can be employed for
∆Eb ∆M × 931 MeV producing electrical power, for producing different isotopes
Thus, ∆E bn = =
A A Nucleon and for various other uses.
10 nE
Power of a reactor, P = , where n = number of atoms
Building energy per nucleon (MeV)

l
32 56 100
16 S Fe Mo 12
12
O I t
C
8
18
O 184 undergone fission in time t seconds and E = energy
4 W 197
An 238
He 14 U released in each fission.
N
6
6
Li
4 Reproduction Factor
3
H Reproduction factor (k ) of a nuclear chain reaction is defined
2 as
2
H Rate of production of neutrons
0
k =
0 50 100 150 200 250 Rate of loss / Absorption of neutrons
Mass number (A)
362 40 DAYS ~ JEE MAIN PHYSICS DAY THIRTY THREE

l
If k = 1, then the chain reaction will be steady and the The mass of the single nucleus, so formed is less than the sum
reactor is said to be critical. of the masses of parent nuclei and this difference in mass,
l
If k > 1, then the chain reaction is accelerated and it may results in the release of tremendously large amount of energy.
cause explosion in the reactor. Such a reactor is called The fusion reaction going on in the central core of sun is a
super-critical. multistep process, but the net reaction is
l
If k < 1, then chain reaction gradually slows down and 4 11H + 2 ε – → 42He + 2 ν + 6γ + 26.7 MeV
comes to a halt. Such a reactor is called sub-critical. When two positively charged particles (protons or deuterons)
The reactors giving fresh nuclear fuel which often exceeds the combine to form a larger nucleus, the process is hindered by
nuclear fuel used is known as breeder reactor. the Coulombian repulsion between them.
To overcome the Coulombian repulsion, the charged particles
are to be given an energy of atleast 400 keV.
Nuclear Fusion For this, proton/deuterons must be heated to a temperature of
Nuclear fusion is the process, in which two or more light about 3 × 10 9 K. Nuclear fusion reaction is therefore, known as
nuclei combine to form a single large nucleus. thermo nuclear fusion reaction.

DAY PRACTICE SESSION 1

FOUNDATION QUESTIONS EXERCISE


1 Two nucleons are at a separation of 1 fm. The net force 6 A radioactive sample decays by two different processes.
between them is F1 if both are neutrons, F2 if both are Half-life for the first process is t1 and for the second
protons and F3 if one is a proton and the other is a process is t 2. The effective half-life is
neutron. t1 t 2
(a) t1 + t 2 (b) t1 − t 2 (c) (t1 + t 2 ) / 2 (d)
(a) F1 > F2 > F3 (b) F2 > F1 > F3 t1 + t 2
(c) F1 = F3 > F2 (d) F1 = F2 > F3
7 Half-life of a radioactive substance A is 4 days. The
2 A radioactive nucleus (initial mass number A and atomic probability that a nucleus will decay in two half-lives is
number Z ) emits 3 α-particles and 2 positrons. The ratio of 1 3 1
(a) (b) (c) (d) 1
number of neutrons to that of protons in the final nucleus 4 4 2
will be ª AIEEE 2010 8 The half-life of a radioactive substance is 20 min. The
A−Z −8 A− Z −4 approximate time interval ( t 2 − t1) between the time t 2
(a) (b)
Z −4 Z − 8 2 1
A − Z − 12 A− Z −4
when of it has decayed and time t1 when of it had
(c) (d) 3 3
Z −4 Z − 2 decayed is ª AIEEE 2011
3 The sequence of decay of a radioactive nucleus is (a) 14 min (b) 20 min (c) 28 min (d) 7 min
α β α α
N0 → N1 → N2 → N3 → N4. If nucleon 9 A sample of a radioactive element has a mass of 10 g at
number and atomic number of N2 are 176 and 71 an instant t = 0. The approximate mass of this element in
respectively, then what are their values for N4 and N0? the sample after two mean lives is
(a) 168, 67 and 180, 71 (b) 67, 168 and 180, 72 (a) 1.35 g (b) 2.50 g (c) 3.70 g (d) 6.30 g
(c) 180, 67 and 72, 180 (d) None of these 10 When uranium is bombarded with neutrons, it undergoes
4 A radioactive nucleus undergoes a series of decays fission. The fission reaction can be written as
according to the scheme 92 U
235
+ 0 n1 → 56 Ba 141 + 36 Kr 92 + 3X + Q (energy)
α β α γ where three particles names X are produced and energy
A → A1 → A2 → A3 → A4
Q is released. What is the name of the particle X ?
If the mass number and atomic number of A are 180 and ª JEE Main (Online) 2013
72 respectively, these numbers of A 4 are
(a) electron (b) α-particle (c) neutron (d) neutrino
(a) 172, 69 (b) 177, 69 (c) 171, 69 (d) 172, 68
11 On fission of one nucleus of U235, the amount of energy
5 If Nt1 = N0 e −λ t1 , then the number of atoms decayed obtained is 200 MeV. The power obtained in a reactor is
during the time interval from t1 and t 2 ( t1 > t 2 ), will be
1000 kW. Number of nuclei fissioned per second in the
(a) Nt 1 − Nt 2 = N 0 [e −λt 1 − e −λt 2 ] reactor is
(b) Nt 2 − Nt 1 = N 0 [e −λt 2 − e −λt 1 ]
(c) Nt 2 − Nt 1 = N 0 [ e −λt 2 − e λt 1 ] (a) 3.125 × 1016 (b) 6.25 × 1010
(c) 3.125 × 1032 (d) 6.25 × 1020
(d) None of the above
DAY THIRTY THREE NUCLEI 363

12 If Mo is the mass of an oxygen isotope 8O17,M p and Mn 15 Statement I A certain radioactive substance has a
are the masses of a proton and a neutron, respectively the half-life period of 30 days. Its disintegration constant is
nuclear binding energy of the isotope is 0.0231 day −1.
(a) (Mo − 8M p )c 2 (b) (M o − 8 M p − 9Mn )c 2 Statement II Decay constant varies inversely as half-life.
(c) Mo c 2 (d) (Mo − 17Mn )c 2 16 Statement I Half-life of a certain radioactive element is
13 The binding energies per nucleon of Li 7and He 4 are 100 days. After 200 days, fraction left undecayed will be
5.6 MeV and 7.06 MeV respectively, then the energy of 50%.
the reaction Li 7 + p = 2 [ 2 He 4 ] will be N  1
n
Statement II =   , where symbols have usual
(a) 17.28 MeV (b) 39.2 MeV (c) 28.24 MeV (d) 1.46 MeV N 0  2
14 The below is a plot of binding energy per nucleon E b , meaning.
against the nuclear mass M; A, B, C, D, E, F correspond
17 Statement I In a decay, daughter nucleus shifts two
to different nuclei.
places to the left from the parent nucleus.
Statement II An alpha particle carries four units of mass.
C D
Eb B E 18 Statement I Energy is released in nuclear fission.
A F Statement II Total binding energy of the fission fragments
is larger than the total binding energy of the parent
M nucleus.
Consider four reactions ª [AIEEE 2010] 19 Statement I If half-life period and the mean-life of a
(i) A + B → C + ε (ii) C → A + B + ε radioactive element are denoted by T and Tm
(iii) D + E → F + ε and (iv) F → D + E + ε respectively, then T < Tm .
where ε is the energy released. In which reactions is ε 1
Statement II Mean-life =
positive? decay constant
(a) (i) and (iv) (b) (i) and (iii)
(c) (ii) and (iv) (d) (ii) and (iii)
20 Statement I Energy is released when heavy nuclei
undergo fission or light nuclei undergo fusion.
Direction (Q. Nos. 15-21) Each of these questions contains Statement II For heavy nuclei, binding energy for per
two statements : Statement I and Statement II. Each of these nucleon increases with increasing Z while for light nuclei.
questions also has four alternative choices, only one of which It decreases with increasing Z.
is the correct answer. You have to select one of the codes (a),
(b), (c) and (d) given below 21 Statement I A nucleus having energy E 1 decays by β −
(a) Statement I is true, Statement II is true; Statement II is emission to daughter nucleus having energy E 2, but β −
the correct explanation for Statement I rays are emitted with a continuous energy spectrum
(b) Statement I is true, Statement II is true; Statement II is having end point energy E 1 − E 2.
not the correct explanation for Statement I Statement II To conserve energy and momentum in
(c) Statement I is true; Statement II is false β-decay, atleast three particles must take part in the
(d) Statement I is false; Statement II is true transformation. ª AIEEE 2011

DAY PRACTICE SESSION 2

PROGRESSIVE QUESTIONS EXERCISE


−α −α to function for 10 yr, the total mass of uranium required is
1 Consider x → y → z , where half-lives of x and y are
(Avogadro’s number = 6.02 × 1026/K-mol,1 eV=1.6 × 10−19 J)
z year and one month. The ratio of atoms of x and y
(a) 3.84 × 104 kg (b) 9.28 × 106 kg
when transient equilibrium [T1/ 2( x ) > T1/ 2( y )] has been
(c) 3.84 × 108 kg (d) 9.28 × 104 kg
established is
(a) 1 : 22 (b) 1 : 26 (c) 26 : 1 (d) 23 : 1
3 The half-life of a radioactive sample is 10 h. The total
number of disintegration in 10th hour measured from a
2 In a nuclear reactor, U 235 undergoes fission liberating time when the activity was one Ci is
200 MeV of energy per fission. The reactor has 10%
. × 10−3
(a) 053 (b) 6.91 × 1013
efficiency and produces 1000 MW power. If the reactor is
(c) 2.63 × 10−3 (d) 9.91 × 1013
364 40 DAYS ~ JEE MAIN PHYSICS DAY THIRTY THREE

4 A piece of wood from the ruins of an ancient building was calculate how much bigger that B must E be for such a
found to have a 14C activity of 12 disintegrations per process to happen?
minute per gram of its carbon content. The 14C activity of B2 B B2 3B
(a) (b) (c) (d)
the living wood is 16 disintegrations per minute per gram. 2 mc 2
2 mc 2
4 mc 2 4 mc 2
How long ago did the tree, from which the wooden
sample came, die? Given, half-life of 14C is 5760 yr ? 8 Assume that a neutron breaks into a proton and an
(a) 2391 yr (b) 2300 yr (c) 2250 yr (d) 2261 yr electron. The energy released during this process is
(mass of neutron = 1.6725 × 10−27 kg, mass of proton
5 A radioactive sample S1 having an activity of 5 µCi has
= 1.6725 × 10−27 kg, mass of electron = 9 × 10 −31 kg)
twice the number of nuclei as another sample S 2 which
ª AIEEE 2012
has an activity of 10 µCi. The half-lives of S1 and S 2 can be
(a) 0.9 MeV (b) 7.10 MeV (c) 6.30 MeV (d) 5.4 MeV
(a) 20 yr and 5 yr, respectively
(b) 20 yr and 10 yr, respectively 9 A radioactive nucleus A with a half-life T , decays into a
(c) 10 yr each nucleus B. At t = 0, there is no nucleus B. After sometime
(d) 5 yr each t, the ratio of the number of B to that of A is 0.3. Then, t is
6 The half-life period of a radioactive element X is same as given by ª JEE Main 2017 (Offline)
the mean life time of another radioactive element Y. log 13
.
(a) t = T (b) t = T log 13
.
Initially, they have the same number of atoms. Then, loge 2
(a) X will decay faster than Y T T loge 2
(c) t = (d) t =
(b) Y will decay faster than X log 13
. 2 log 13
.
(c) Y and X have same decay rate initially 10 Half-lives of two radioactive elements A and B are
(d) X and Y decay at same rate always 20 min and 40 min, respectively. Initially, the samples
7 Deuteron is a bound state of a neutron and a proton with have equal number of nuclei. After 80 min, the ratio of
a binding energy B = 2.2 MeV. A γ-ray of energy E is decayed numbers of A and B nuclei will be
aimed at a deuteron nucleus to try to break it into a ª JEE Main 2016 (Offline)
(neutron + proton) such that the n and p move in the (a) 1 : 16 (b) 4 : 1 (c) 1 : 4 (d) 5 : 4
direction of the incident γ-ray. Where E ≠ B. Then,

ANSWERS
1 (c) 2 (b) 3 (a) 4 (a) 5 (a) 6 (d) 7 (b) 8 (b) 9 (a) 10 (c)
SESSION 1
11 (a) 12 (b) 13 (a) 14 (a) 15 (a) 16 (c) 17 (b) 18 (a) 19 (b) 20 (c)
21 (a)

SESSION 2 1 (d) 2 (a) 3 (b) 4 (a) 5 (a) 6 (b) 7 (c) 8 (a) 9 (a) 10 (d)

Hints and Explanations


SESSION 1 p + → n 0 + e + So, the ratio of number of neutrons to that
A−Z −4
1 Nuclear force of attraction between Number of neutrons initially was of protons =
Z −8
any two nucleons (n - n, p - p, p - n ) is A−Z
same. The difference comes up only 3 As mass number of each α-particle is 4
due to electrostatic force of repulsion Number of neutrons after decay is
units and its charge is 2 units, therefore for
between two protons. (A − Z)− 3 × 2
N4
∴ F1 = F3 ≠ F2 (due to α-particles) − 2 × 1
A = 176 − 8 = 168
As, F2 < F3 or F1 (due to positive β-decay) and Z = 71 − 4 = 67
∴ F1 = F3 > F2 As [3 × 2 (due to α-particles) + 2 Now, the charge of β is −1 and its mass
2 In positive β-decay a proton is (due to positive β-decay)] number is zero.
transformed into a neutron and a Hence, atomic number reduces by 8. So, A = 176 + 0 + 4 = 180
positron is emitted. and Z = 71 − 2 + 2 = 71
DAY THIRTY THREE NUCLEI 365

4 As the mass number of each a-particle 13 The reaction is 3 Li7 + 1 p1 ¾® 2 (2 He 4 ) 21 In practicle situation, atleast three
is 4 units and its charge is 2 unit. \ E p = 2E (2 He 4 ) - E ( Li) particles take place in transformation, so
Therefore for A 4 , Energy of b-particle + Energy of third
= 2 (4 ´ 7.06) - 7 ´ 5.6
Mass number = 180 - 8 = 172 particle = E1 - E2
= 56.48 - 39.2 = 17.28 MeV
and Z = 72 - 4 + 1 (due to b - ) = 69 Hence, energy of b-particle £ E1 - E2
14 Both fusion and fission reaction results
5 Since, N t1 = N 0e -lt1 and N t2 = N 0e -lt2 into tremendous amount of energy SESSION 2
Then, the number of atoms decayed release and nucleus/nuclei which has
l1 N 1 - l1 t
during the time interval t 1 to t 2 is higher binding energy per nucleon than 1 N2 = (e - e - l2 t )
parent nuclei. So, option (a) is correct. l2 - l1
= N t1 - N t2 = N 0 [e -lt1 - e -lt2 ]
When (T1 /2 )1 > (T1 /2 )2 at transient
15 Half-life and decay constant for a
6 As, l = l1 + l2 equilibrium, l1 < l2
nuclear reaction are related by a - l1 t
relation, which is e - l2 t < < e
1 1 1 t2 + t1
Þ = + = 0.693 t
t t1 t2 t1 t2 T1 /2 = l1 N 1e - l1
\ N2 =
l l2 - l1
t1 t2 0.693 0.693
or t = Þ l= = = 0.0231 day -1 l1 N 1
t1 + t2 T1 /2 30 =
l2 - l1
7 After two half-lives 1/4 th fraction of 16 Number of half-lives N1 l - l1
\ = 2
nuclei will remain undecayed. Or, 3/4th t 200 N2 l1
fraction will decay. Hence, the n= = =2
T 100 0.693 0.693
probability that a nucleus decays in two -
The fraction left undecayed is given by 1 2 ´ 12 23
half-lives is 3 / 4 . n 2 = =
N æ1 ö æ1 ö 0.693 1
8 N 1 = N 0 - 1 N 0 = 2 N 0,
\ =ç ÷ =ç ÷
N 0 è2 ø è2 ø 2 ´ 12
3 3
2 1 1
N2 = N 0 - N 0 = N 0 = = 25% 2 Energy generated by the reactor
3 3 4
1000 ´ 106 W = 109 Js -1
n
N2 æ1 ö 17 On adecay, charge number of parent Total energy generated in 10 yr is
We have, =ç ÷
N1 è 2 ø nucleus decreases by 2 units. As E = (109 Js -1 ) ´ 10 ´ 365
Here, n = 1 classification or grouping of elements is ´ 24 ´ 60 ´ 60
based on charge number, hence
\ t 2 - t 1 = one half-life = 20 min = 1.97 ´ 1030 MeV
daughter nucleus shifts two places to
9 The relation of mean-life and decay the left from the parent nucleus. In the reactor 200 MeV energy is
constant is, liberated in the fission of nucleus of U235
2 1
18 According to concept of binding energy,
t = 2t = , where t = atom.
fission can occur because the total mass
l l energy will decrease; that is DEbn \ Total number of U235 atoms required is
Then we get from the equation, (binding energy) will increase. We see
m = m 0e -lt that for high mass nuclide ( A = 240), the 1.97 ´ 1030
= 0.985 ´ 1028
binding energy per nucleon is about 200
Þ m = 10 ´ e -l ´2/l = 10 ´ e -2
7.6 MeV/nucleon. For the middle weight 1 kmol that is 235 kg of U235 has
= 10 ´ 0135
. . g
= 135 nuclides ( A = 120), it is about 6.02 ´ 1026 atoms Therefore, total mass of
10 The fission of 235 8.5 MeV/nucleon. Thus, binding energy
92 U is represented by U235 having 0.985 ´ 1028 atoms is
235 1 141 92
of fission fragments is larger than the
92 U + 0 n ® 56 Ba + 36 Kr 235
total binding energy of the parent ´ (0.985 ´ 1028 )
+3 0 n1 + Q nucleus. 6.02 ´ 1026
The name of the particle X is neutron 19 We know that half-life period T and = 3.84 ´ 103 kg
( 0 n1 ). decay constant l are related by the Since, efficiency of reactor is 10%, actual
equation. mass of U235 required is
11 Power received from the reactor is 0.6931 100
P = 1000 kW T = …(i) (3.84 ´ 103 ) ´ = 3.84 ´ 104 kg
l 10
= 1000 ´ 1000 = 106 J s -1
While mean-life T m is related with l by
Also, 1 MeV = 1.6 ´ 10-13 J 3 As, - dN = lN ,
the equation dt
Number of nuclei fissioned per second 1 37. ´ 1010 ´ 3.6 ´ 104
Tm = …(ii) N =
106 l
= 0.693
200 ´ 1.6 ´ 10-13 From Eqs. (i) and (ii), we get DN = - (N 0 e - l ´ 10 ´ 3600
= 3.125 ´ 1016 s -1 T = 0.6931 T m - N 0 e - l ´ 9 ´ 3600 )
12 Binding energy or T < Tm
2
(Q DN = N 1 - N 2 )
BE = (M nucleus nucleons )c
-M 20 Here, Statement I is correct and . ´ 1014 ´ 3.6
37
= [0.535 - 0.5]
= (Mo - 8 M p - 9 M n )c 2 Statement II is wrong, which can be 0.693
directly concluded from binding energy = 6.91 ´ 1013

nucleon curve.
366 40 DAYS ~ JEE MAIN PHYSICS DAY THIRTY THREE

4 Given, R = 12 dis/min per g, From conservation of momentum, 9 Decay scheme is ,


E
R 0 = 16 dis/min per g pn + p p = …(ii) N atoms
c
of B
T1 /2 = 5760 yr As E = B , Eq. (i), p2n + p2p = 0
Let t be the time span of the tree. It only happen if p n = p p = 0 A A, B
According to radioactive decay law, So, the Eq. (ii) cannot satisfy and the Let N atoms decays
R process cannot take place. into B in time t
R = R 0e - l t or e l t = 0 No No – N
R Let E = B + X , where X << B for the at t=0 atoms of A
Taking log on both the sides process to take place. NB 3
R Put value of p n from Eq. (ii) in Given, = 03
. =
lt log e e = log e 0 NA 10
R Eq. (i), we get
16 ö 2
NB 30
æ æE Þ =
lt = ç log 10 ÷ ´ 2.303 ö N A 100
è 12 ø ç - p p÷ p2p
èc ø
2.303 (log 4 - log 3) X = + So, N 0 = 100 + 30 = 130 atoms
t = 2m 2m
l By using N = N 0 e - lt
2 2 Ep p E 2
= 2391.20 yr » 2391 yr or 2 p p - + 2 - 2mX = 0 We have, 100 = 130e - lt
c c
1
5 We know that, Using the formula of quadratic Þ = e - lt
Activity ( A ) = lN 0 equation, we get 13.
For S 1 , A s1 = 5m Ci = l1 2 N 0 …(i) Þ log 13 . = lt
2E 4 E2 æ E2 ö If T is half-life, then
For S 2 , A s2 = 10m Ci = l2 N 0 …(ii) + - 8 ç 2 - 2m X ÷
c c2 èc ø log e 2
As we know, pp = l=
0.693 4 T
T s1 1 /2 = log e 2
l1 For the real value p p¢ the discriminant Þ log 13
. = ×t
is positive T
0.693
and T s2 1 /2 = 4 E2 æE ö T × log (13 . )
l2 = 8 ç 2 - 2mX ÷ \ t =
c2 è c2 ø log e 2
Therefore, by dividing Eqs. (i)
and (ii), we get 4 E2 10 Given, 80 min = 4 half-lives of A = 2
16 m X =
5 T s2 1 /2 2N 0 c2 half-lives of B.
=
10 T s1 1 /2 N 0 E2 B2 Let the initial number of nuclei in each
X = • 4 mc sample be N.
T s2 1 /2 4 mc 2 2
Þ =4 For radioactive element A,
T s1 1 /2 8 According to given data, mass of N
N A after 80 min = 4
So, only option (a) can be satisfied. neutron and proton are equal which do 2
not permit the breaking up of neutron Þ Number of A nuclides decayed
6 According to question,
and proton. But if we take standard N 15
T1 /2 ( X ) = t (l ) =N - = N
mass of neutron as 1.6750 ´ 10-27 kg, 16 16
0.693 1 For radioactive element B,
Þ = then
lX lY N
Energy released = mass defect ´ c 2 N B after 80 min = 2
lX 2
or lY =
0.693 = (m n - m p - me ) ´ c 2 Þ Number of B nuclides decayed
Þ lY > l X N 3
(1.6750 ´ 10-27 - 1.6725 ´ 10-27 =N - = N
So, Y will decay faster than X. 4 4
- 9 ´ 10-31 )
= \ Ratio of decayed numbers of A and B
7 Binding energy B = 2.2 MeV 1.66 ´ 10-27
nuclei will be
From the energy conservation law, ´ 931.5 MeV (15/ 16)N 5
p2 p2p » 0.9 MeV
=
E - B = Kn + K p = n + …(i) (3 / 4)N 4
2m 2m
DAY THIRTY FOUR

Electronic
Devices
Learning & Revision for the Day
u Energy Bands in Solids u I-V Characteristics Semiconductor Diode in u Special Purpose Diodes
u Semiconductors Forward and Reverse Bias u Transistor
u Semiconductor Diode u Diode as a Rectifier

Energy Bands in Solids


According to band theory of solids, in a crystalline solid due to mutual interaction
among valence electrons of neighbouring atoms, instead of sharp energy levels, energy
bands are formed. Energy bands are of the following three types
(i) Valence band It is the energy band formed by a series of energy levels of valence
electrons actually present. Ordinarily, valence band is completely filled and
electrons in this band are unable to gain energy from external electric field. The
highest energy level in a valence band at 0 K is called fermi energy level.
(ii) Conduction band The energy band having just higher energy than the valence band
is called conduction band. Electrons in conduction band are commonly called the
free electrons.
(iii) Forbidden band The energy gap between the valence band and the conduction band
of a solid is called the forbidden energy gap E g or forbidden band. Width of
forbidden energy gap depends upon the nature of substance.
PREP
MIRROR
Semiconductors Your Personal Preparation Indicator
l
In semiconducting solids, the valence band is completely filled but conduction band u No. of Questions in Exercises (x)—
is completely empty and the energy gap between them is small enough (E g < 3 eV). At u No. of Questions Attempted (y)—
absolute zero temperature, it behaves as an insulator. u No. of Correct Questions (z)—
l
A pure semiconductor, in which no impurity of any sort has been mixed, is called (Without referring Explanations)
intrinsic semiconductor. Germanium (E g = 0.72 eV) and silicon (E g = 1.1 eV) are
examples of intrinsic semiconductors.
u Accuracy Level (z / y × 100)—
u Prep Level (z / x × 100)—
l
Electrical conductivity of pure semiconductor is very small. To increase the
conductivity of a pure semiconducting material, it is doped with a controlled quantity In order to expect good rank in JEE,
(1 in 10 5 or 10 6) of suitable impurity. Such a doped semiconductor is called an your Accuracy Level should be above
85 & Prep Level should be above 75.
extrinsic semiconductor.
368 40 DAYS ~ JEE MAIN PHYSICS DAY THIRTY FOUR

l
The number of electrons reaching from valence band to This layer containing immobile ions is called depletion layer.
conduction band, The thickness of depletion layer is approximately of the order
− E / 2 kT
n = AT 3 / 2e g of 10 −6 m.
where, k = Boltzmann’s constant, T = absolute The potential difference developed across the p-n junction
temperature and A = atomic weight. due to diffusion of electrons and holes is called the potential
barrier Vb (or emf of fictitious battery). For germanium diode
Superconductors barrier potential is 0.3 V, but for Si diode, its value is 0.7 V.
The barrier electric field developed due to it, is of the order of
When few metals are cooled, then below a certain critical
10 5 Vm–1.
temperature, their electrical resistance suddenly becomes
zero. In this state, these substances are called
superconductors and this phenomena is called Mobility of Charge Carriers
superconductivity. Mercury become superconductor at 4.2 K, The mobility of a charge carrier is defined as the velocity
lead at 7.25 K and niobium at 9.2 K. gained by its per unit electric field, i.e. µ = Vd / E .
l
Current in semiconductor is, i = ie + i h = eA(ne Ve + nhVh )
Types of Extrinsic Semiconductor Conductivity, σ = =
J i
= e(ne µe + nhµ h )
According to type of doping impurities, extrinsic E AE
semiconductor are of two types (where, J = current density = nqV )

1. n-type Semiconductor
To prepare an n-type semiconductor, a pentavalent I-V Characteristics of Semiconductor
impurity, e.g. P, As, Sb is used as a dopant with Si or Ge.
Such an impurity is called donor impurity, because each
Diode in Forward and Reverse Bias
dopant atom provides one free electron. When we join an external potential source, such that p-side of
In n-type semiconductor ne > > nh , i.e. electrons are p-n junction is joined to positve terminal of voltage source and
majority charge carriers and the holes are minority charge n-side to negative terminal of voltage source, the junction is
carriers, such that ne ⋅ nh = n2i . An n-type semiconductor is said to be forward biased and applied electric field E opposes
electrically neutral and is not negatively charged. the barrier electric field Eb .
Conductivity, σ ≈ ne µe e As a result, width of depletion layer is reduced and on
applying a voltage V > Vb , a forward current begins to flow.
2. p-type Semiconductor Resistance offered by p-n junction in forward bias is small
(about 10-50 Ω).
To prepare a p-type semiconductor, a trivalent impurity, IF
E
e.g. B, Al, In, Ga, etc., is used as a dopant with Si or Ge. E Eb (mA)
Eb
Such an impurity is called acceptor impurity as each sr
r s r ss rr s
impurity atom wants to accept an electron from the crystal r s s ss rr r VR VF
s sr r r s
lattice. Thus, effectively each dopant atom provides a hole. p r ss rr s n
p r sr s n
In p-type semiconductor nh > > ne , i.e. holes are majority + – – +
charge carriers and electrons minority charge carriers, such IR (µA)
that nh ⋅ ne = n2i . A p-type semiconductor is electrically (a) FB (b) RB (c)
neutral and is not positively charged. If connections of potential source are reversed [Fig. (b)], i.e.
The number of free electrons in a semiconductor varies p -side is connected to negative terminal of battery and n-side
with temperature as T 3 / 2 . to positive terminal, the junction is said to be reverse biased
and in this case E and Eb , being in same direction, are added
Conductivity, σ ≈ nhµ he
up. So, the depletion layer broadens and potential barrier is
fortified. Consequently, an extremely small leakage current
Semiconductor Diode flows across the junction due to minority charge carriers and
junction resistance is extremely high (~ − 10 5 Ω). For a
A p-n junction is obtained by joining a small p-type crystal
sufficiently high reverse bias voltage (25 V or even more), the
with a small n-type crystal without employing any other
binding material in between them. Whenever a p-n junction is reverse current suddenly increases. This voltage is called
formed, electrons from n-region diffuse through the junction Zener voltage or breakdown voltage or avalanche voltage.
into p-region and the holes from p-region diffuse into
NOTE • A p-n junction behaves as a voltage controlled switch. In
n-region.
forward bias, it acts like ON switch and in reverse bias as
As a result of which neutrality of both n and p-regions is OFF switch.
disturbed, and a thin layer of immobile negative charged ions
appear near the junction in the p-crystal and a layer of • The p-n junction can be presumed as a capacitor, in which
positive ions appear near the junction in n-crystal. the depletion layer acts as dielectric.
DAY THIRTY FOUR ELECTRONIC DEVICES 369

Diode as a Rectifier Light Emitting Diode (LED)


Junction diode allows current to pass only when it is forward It is a specially designed diode made of GaAsP, GaP, etc.
biased. So, if an alternating voltage is applied across a diode, When used in forward biased, it emits characteristic, almost
the current flows only in that part of the cycle, when the diode monochromatic light. In reverse biased, it works like a normal
is forward biased. diode.
This property is used to rectify alternating voltages and the l
I-V Characteristics LEDs are current dependent devices
circuit used for this purpose is called a rectifier, and the with its forward voltage drop (VF ) depending on the
process is known as rectification. forward biased LED current. Characteristics of light
There are two types of rectifier diode as given below emitting diode I-V are shown below
1. Half Wave Rectifier A rectifier, which rectifies only

Infrared

Amber
Yellow

Green
one-half of each AC input supply cycle, is called a half

Blue
I (mA)

Red
wave rectifier.
A half wave rectifier gives discontinuous and pulsating
DC output. As no output is obtained corresponding to

Forward current
alternate half cycles of the AC input supply, its efficiency
is quite low.
2. Full Wave Rectifier A rectifier, which rectifies both
halves of each AC input cycle is called a full wave
rectifier. VF
Forward voltage
The output of a full wave rectifier is continuous, but
pulsating in nature. However, it can be made smooth by
using a filter circuit. Photodiode
As output is obtained corresponding to both the half It is a special diode used in reverse bias which conducts only
cycles of the AC input supply, its efficiency is more than when light of suitable wavelengths is incident on the junction
that of half wave rectifier. of diode. The energy of incident light photon must be greater
than the band gap of semiconductor (i.e. hν > E g ). Materials
NOTE • The ripple factor is defined as the ratio of rms value of AC used are Cds, Se, Zns, etc.
component in the output of the rectifier to the DC
component in the input.
Solar Cell
It is a special p-n junction, in which one of the
Special Purpose Diodes semiconductors is made extremely thin, so that solar radiation
There are few diodes which are designed to serve some special falling on it reaches junction of diode without any absorption.
purpose and application. A solar cell directly converts, solar energy into electrical
energy. Popularly used solar cells, Ni-cd, PbS cell, etc.
Zener Diode
It is a highly doped p-n junction diode which is not
damaged by high reverse current. It is always used in n Transistor
reverse bias in breakdown voltage region and is p A transistor is a combination of two p-n junctions joined in
chiefly used as a voltage regulator. series. A junction transistor is known as Bipolar Junction
l
Zener Diode as Voltage Regulator The following circuit is Transistor (BJT). It is a three terminal device.
used for stabilising voltage across a load RL . The circuit Transistors are of two types
consists of a series voltage-dropping resistance R and a (i) n-p-n transistor,
Zener diode in parallel with the load RL .
(ii) p-n-p transistor
+ I R
+ A transistor has three regions
(i) An emitter (E), which is most heavily doped, and is of
Fluctuating n Constant moderate size. It supplies large number of charge carriers,
p n Zener VZ RL DC output
DC input which are free electrons in an n-p-n transistor and holes
voltage diode voltage
p in a p-n-p transistor.
(ii) A base (B), which is very lightly doped and is very thin

– (thickness ~ 10 −5m).
The Zener diode is selected with Zener voltage Vz equal to (iii) A collector (C), which is moderately doped and is
the voltage desired across the load. thickest.
370 40 DAYS ~ JEE MAIN PHYSICS DAY THIRTY FOUR

A transistor is symbolically represented as shown in figures. l


In common base configuration, AC current gain is defined
n-p-n p-n-p ∆I
as α = C
E E C ∆I V = constant
C CE

l
Value of α is slightly less than 1. In fact, 0.95 ≤ α ≤ 1.
∆P
B B l
Power gain = o = β2AC × Resistance gain
(a) (b)
∆Pi
NOTE • Current gains α and β are correlated as
Transistor Action β=
α
or α = β
For proper functioning of a transistor, the emitter-base 1−α 1+ β
junction is forward biased, but the collector-base junction is
reverse biased. In an n-p-n transistor, electrons flow from Transistor as an Amplifier
emitter towards the base and constitute a current I E . n- p -n C
A transistor consisting of
Due to larger reverse bias at base-collector junction, most of two p-n junctions, one
these electrons further pass into the collector, constituting a B
forward biased and the
collector current IC . But a small percentage of electrons (less other reverse biased can
than 5%) may combine with holes present in base. These E Output
be used to amplify a weak Input
electrons constitute a base current I B . It is self evident, that signal. The forward
I E = IC + I B . biased junction has a low + – – +
Action of p-n-p transistor is also same, but with one difference resistance path, whereas
that holes are moving from emitter to base and then to the reverse biased
collector. junction has a high
A transistor can be connected in either of the following three resistance path. The weak input signal is applied across the
configurations forward biased junction, and the output signal is taken across
(i) Common Emitter (CE) configuration the reverse biased junction.
(ii) Common Base (CB) configuration Since, the input and output currents are almost equal, the
output signal appears with a much higher voltage. The
(iii) Common Collector (CC) configuration.
transistor, thus acts as an amplifier. Common-emitter
Generally, we prefer common emitter configuration, because configuration of transistor amplifier is given alongside.
power gain is maximum in this configuration.

Transistor as an Oscillator
Characteristics of a Transistor An electronic oscillator is a device that generates electrical
In common emitter configuration, variation of current on the oscillations of constant amplitude and of a desired frequency,
input side with input voltage (I B versus VBE ) is known as the without any external input.
input characteristics, and the variation in the output current
with output voltage (IC versus VCE ) is known as output The circuit providing such oscillation, is known as a tank
characteristics. From these characteristics, we obtain the oscillator, is using positive feedback.
values of following parameters
∆VBE
l
Input resistance, ri = L′
Inducting coupled

∆I B V = constant
CE
C
∆VCE B
l
Output resistance, ro = n- p- n
∆ IC I B = constant
B2
N
∆ IC
l
AC current gain, β= C
∆I B VCE = constant L
The current gain for common-emitter configuration
β ranges from 20 to 200.
K
∆ IC β B1
l
Transconductance, g m = =
∆VBE ri Some of the properties of the oscillator are
l
A transistor can be used as an amplifier. The voltage gain l
Oscillator is using positive feedback.
of an amplifier will be given by l
To work as an oscillator,
V R
A V = o = β⋅ C |Aβ| = 1; β → feedback factor
Vi RB
1 1
where, RC and RB are net resistances in collector and base l
f = frequency of oscillation = × .
2π LC
circuits, respectively.
DAY THIRTY FOUR ELECTRONIC DEVICES 371

DAY PRACTICE SESSION 1

FOUNDATION QUESTIONS EXERCISE


1 The conductivity of a semiconductor increases with (c) Ec and Ev decreases, but E g increases
increase in temperature because (d) All Ec , E g and Ev decreases
(a) number density of free current carriers increases 6 In an unbiased p-n junction, holes diffuse from the
(b) relaxation time increases p -region to n-region because
(c) Both number density of carriers and relaxation time (a) free electrons in the n- region attract them
increases (b) they move across the junction by the potential difference
(d) number density of current carriers increases, relaxation (c) hole concentration in p-region is more as compared to
time decreases but effect of decrease in relaxation time n- region
is much less than increase in number of density (d) All of the above
2 Carbon, silicon and germanium atoms have four valence 7 Application of a forward bias to a p-n junction
electrons each. Their valence and conduction bands are
(a) increases the number of donors on the n-side
separated by energy band gaps represented by
(b) increases the electric field in the depletion zone
(E g ) C , (E g ) Si and (E g ) Ge , respectively. Which one of the
(c) increases the potential difference across the depletion
following relationship is true in their case? zone
(a) (E g ) C > (E g ) Si (b) (E g ) C = (E g ) Si (d) widens the depletion zone
(c) (E g ) C < (E g ) Ge (d) (E g ) C < (E g ) Si
8 When forward bias is applied to a p-n junction, what
3 In n-type silicon, which of the following statement is true happens to the potential barrierVB and the width of
(a) Electrons are majority carriers and trivalent atoms are charge depleted region x ?
the dopants
(a)VB increases, x decreases
(b) Electrons are minority carriers and pentavalent atoms
(b)VB decreases, x increases
are the dopants
(c)VB increases, x increases
(c) Holes are minority carriers and pentavalent atoms are (d)VB decreases, x decreases
the dopants
(d) Holes are majority carriers and trivalent atoms are the 9 The temperature (T ) dependence of resistivity (ρ) of a
dopants semiconductor is represented by
4 Carbon, silicon and germanium have four valence ρ ρ
electrons each. At room temperature, which one of the
following statements is most appropriate? ª AIEEE 2012 (a) (b)

(a) The number of free conduction electrons is significant in O


O
C but small in Si and Ge T T
(b) The number of free conduction electrons is negligibly ρ ρ
small in all the three
(c) The number of free electrons for conduction is (c) (d)
significant in all the three
(d) The number of free electrons for conduction is O O
significant only in Si and Ge but small in C T T

5 If the lattice constant of this semiconductor is decreased, 10 In figure , V0 is the potential barrier across a p-n junction,
then which of the following is correct? ª AIEEE 2010 when no battery is connected across the junction.
Conduction 1
Ec 2
bandwidth
3
Band gap Eg
V0
Valence
Ev
bandwidth
(a) All Ec , E g and Ev increases
(b) Ec and Ev increases, but E g decreases
372 40 DAYS ~ JEE MAIN PHYSICS DAY THIRTY FOUR

(a) 1 and 3 both corresponds to forward bias of junction 15 The circuit has two oppositely connected ideal diodes in
(b) 3 corresponds to forward bias of junction and 1 parallel. What is the current flowing in the circuit?
correspond to reverse bias of junction 4Ω
(c) 1 corresponds to forward bias and 3 corresponds to D1
reverse bias of junction D2
(d) 3 and 1 both corresponds to reverse bias of junction 12 V
3Ω 2Ω
11 For the given circuit of p-n junction diode, which of the
following statements is correct?
(a) 1.71 A (b) 2.00 A (c) 2.31 A (d) 1.33 A
R p n
16 In the following circuits, which one of the diodes is
+ – reverse biased?
+ 10 V – 12V
V
(a) In forward biasing the voltage across R is V (a) (b) R
R
(b) In forward biasing the voltage across R is 2 V
(c) In reverse biasing the voltage across R is V +5V – 10 V
(d) In reverse biasing the voltage across R is 2 V
12 In the case of forward biasing of p-n junction, which one + 5V
of the following figures correctly depicts the direction of (c) (d)
the flow of charge carriers? R
V Vb R
VV Bb
pp n
n pp nn
– 10V
(a) (b)
V b
17. The forward biased diode connection is ª JEE Main 2014
(a) 2V +4V
V VV BB b
pp nn (b) –2V +2V
(c) (d) None of these
(c) +2V –2V

(d) –3V –3V


13 A junction diode is connected to a 10 V source and103 Ω
rheostat figure. The slope of load line on the 18 The reading of the ammeter for a silicon diode in the
characteristic curve of diode will be given circuit is ª JEE Main 2018
200Ω

103 Ω
10 V

+ – 3V
(a) 10−2 AV −1 (b) 10−3 AV −1 (c) 10−4 AV −1 (d) 10−5 AV −1 (a) 0 (b) 15 mA (c) 11.5 mA (d) 13.5 mA
14 In a forward biased p-n junction diode, the potential 19 In a full-wave rectifier circuit operating from 50 Hz
barrier in the depletion region willp be of the form mains frequency, the fundamental frequency in the
p nn p n output would be
p n
P o t e n t ia l
Potential P Potential
o t e n t ia l (a) 50 Hz (b) 25 Hz (c) 100 Hz (d) 70.7 Hz
b a r r ie r
barrier b barrier
a r r ie r
(a) (b) 20 A p-n junction (D) shown in the figure can act as a
rectifier. An alternating current source (V ) is connected in
the circuit. ª AIEEE 2013
p n p n
n p n
p P o t Potential
e n t ia l P Potential
o t e n t ia l D
b a r barrier
r ie r b barrier
a r r ie r
(c) (d) R
V~
DAY THIRTY FOUR ELECTRONIC DEVICES 373

I I (c) Solar cell, Light dependent resistance, Zener diode,


Simple diode
(a) (b)
(d) Zener diode, Solar cell, Simple diode, Light dependent
t t resistance
I I
24 In a common emitter amplifier circuit using an
(c) (d) n-p-n transistor, the phase difference between the input
t t and the output voltages will be ª JEE Main 2017 (Offline)
(a) 90° (b) 135° (c) 180° (d) 45°
21 Zener breakdown in a semiconductor diode occurs,
when 25 When A is the internal stage gain of an amplifier and B is
(a) forward current exceeds certain value
the feedback ratio, then the amplifier becomes as
oscillator if
(b) reverse bias exceeds certain value
(c) forward bias exceeds certain value (a) B is negative and magnitude of B = A / 2
(b) B is negative and magnitude of B = 1/ A
(d) potential barrier is reduced to zero (c) B is negative and magnitude of B = A
22 The I-V characteristics of an LED is ª AIEEE 2013 (d) B is positive and magnitude of B = 1/ A

R Y GB
R
Direction (Q. Nos. 26-30) Each of these questions contains
Y two statements : Statement I and Statement II. Each of these
I
(a) (b) G questions also has four alternative choices, only one of which
B
is the correct answer. You have to select one of the codes (a),
O V O V (b), (c) and (d) given below.
V O (a) Statement I is true, Statement II is true; Statement II is
the correct explanation for Statement I
I R (b) Statement I is true, Statement II is true; Statement II is
(c) (d) Y
G not the correct explanation for Statement I
B (c) Statement I is true; Statement II is false
O V I
(d) Statement I is false; Statement II is true

23 Identify the semiconductor devices whose 26 Statement I If forward current changes by 1.5 mA when
characteristics are as given below, in the order forward voltage in semiconductor diode is changed from
(p),(q),(r),(s). ª JEE Main 2016 (Offline) 0.5 V to 2 V, the forward resistance of diode will be 1 Ω.
∆Vf
I I Statement II The forward resistance is given by Rf =
∆If

27 Statement I A Zener diode is used to get constant


(p) V (q) V voltage at variable current under reverse bias.
Statement II The most popular use of Zener diode is as
voltage regulator.
28 Statement I Light Emitting Diode (LED) emits
I I
spontaneous radiation.
Dark Resistance
Statement II LED are forward biased p-n junctions.
(r) V (s) V 29 Statement I When base region has larger width, the
Intensity
of light
collector current increases.
Illuminated Statement II Electron-hole combination in base results in
increases of base current.
Choose the correct order
30 Statement I In a common-emitter transistor amplifier the
(a) Simple diode, Zener diode, Solar cell, Light dependent input current is much less than output current.
resistance
Statement II The common-emitter transistor amplifier has
(b) Zener diode, Simple diode, Light dependent resistance,
very high input impedance.
Solar cell
374 40 DAYS ~ JEE MAIN PHYSICS DAY THIRTY FOUR

DAY PRACTICE SESSION 2

PROGRESSIVE QUESTIONS EXERCISE


1 The input resistance of a common-emitter transistor 10 V
amplifier, if the output resistance is 500 k Ω, the current
gain α = 0.98 and the power gain is 6.0625 × 106 is
(a) (b)
(a) 198 Ω (b) 300 Ω (c) 100 Ω (d) 400 Ω
2 If the resistivity of copper is 1.7 × 10−6 Ω-m, then the
–10 V
mobility of electrons in copper, if each atom of copper
contributes one free electron for conduction is
[the atomic weight of copper is 63.54 and density is 5V
8.96 g/cc] (c) (d)
(a) 23.36 cm2 / Vs (b) 503.03 cm2 / Vs
–5 V
(c) 43.25 cm2 / Vs (d) 88 cm2 / Vs
3 A red LED emits light at 0.1 W uniformly around it. The 7 To plot forward characteristic of p-n junction diode, the
amplitude of the electric field of the light at a distance of correct circuit diagram is
1 m from the diode is ª JEE Main 2015
(a) 1.73 V/m (b) 2.45 V/m

(0 – 20) A
(c) 5.48 V/m (d) 7.75 V/m
+ +
4 A working transistor with its three legs marked P, Q and (a) E V – (0 – 1) V A

R is tested using a multimeter.
No conduction is found between P and Q. By conneting
the common (negative) terminal of the multimeter to R
and the other (positive) terminal to P or Q. Some
resistance is seen on the multimeter. Which of the

(0 – 1000) mA
following is the true for the transistor? ª AIEEE 2013
(a) It is an n-p-n transistor with R as base + +
(b) E V – (0 – 2) V A

(b) It is a p-n-p transistor with R as collector
(c) It is a p-n-p transistor with R as emitter
(d) It is an n-p-n transistor with R as collector
5 A piece of copper and another of germanium are cooled
from room temperature to 77 K, the resistance of
(a) each of them increases
(0 – 1000) mA

(b) each of them decreases


+ –
(c) copper decreases and germanium increases (c) E V – (0 – 2) V A
+
(d) copper increases and germanium decreases
6 If in a p-n junction diode, a square input signal of 10 V is
applied as shown.
5V
(0 – 1000) mA

RL
+ –
(d) E V – (0 – 2) V A
+

–5 V
Then, the output signal across RL will be
DAY THIRTY FOUR ELECTRONIC DEVICES 375

8 A figure is given below 10 The length of germanium rod is 0.928 cm and its area of
D3 cross-section is 1 mm 2. If for germanium
5Ω
ni = 2.5 × 1019m −3, µh = 019
. m 2 V −1 s −1,
2 −1 −1
D1 10Ω µe = 0.39 m V s , then resistance is
(a) 2.5 k Ω (b) 4.0 k Ω (c) 5.0 k Ω (d) 10.0 k Ω
20Ω
D2 5Ω 11 In the figure, potential difference between A and B is
A
10 k Ω
10 V
30 V 10 k Ω
The current through the battery is 10 k Ω
(a) 0.5 A (b) 1 A
(c) 1.5 A (d) 2 A
9 In the given circuit, the current through the zener diode is B

I (a) 10 V (b) 20 V (c) 30 V (d) 40 V


12 In a common-base mode of a transistor, the collector
R1 500Ω
current is 5.488 mA for an emitter current of 5.60 mA. The
value of the base current amplification factor (β) will be
15 V
ª AIEEE 2011
(a) 49 (b) 50 (c) 51 (d) 48
R2 1500Ω
Vz=10 V 13 For a common emitter configuration, if α and β have their
usual meanings, the incorrect relationship between α and
β is ª JEE Main 2016 (Offline)
(a) 10 mA (b) 6.67 mA
(c) 5 mA (d) 3.33 mA 1 1 β β β2
(a) = + 1 (b) α = (c) α = (d) α =
α β 1− β 1+ β 1+ β2

ANSWERS

SESSION 1 1 (d) 2 (a) 3 (c) 4 (d) 5 (c) 6 (c) 7 (a) 8 (d) 9 (c) 10 (b)
11 (a) 12 (c) 13 (b) 14 (d) 15 (b) 16 (d) 17 (c) 18 (c) 19 (c) 20 (c)
21 (b) 22 (a) 23 (a) 24 (c) 25 (d) 26 (d) 27 (a) 28 (b) 29 (d) 30 (c)

SESSION 2 1 (a) 2 (c) 3 (b) 4 (a) 5 (c) 6 (d) 7 (b) 8 (c) 9 (d) 10 (b)
11 (a) 12 (a) 13 (a,c)

Hints and Explanations


SESSION 1 (E g ) C = 5.2 eV, 4 The number of free electrons for
1 Based on the theory discussed, we can (E g ) Si = 1.21 eV conduction is significant only in Si and Ge
conclude that when temperature and (E g ) Ge = 0.75eV but small in C, as C is an impurity.
increases, number density of current Thus, (E g ) C > (E g ) Si 5 If lattice constant of semiconductor is
carriers increases in the and (E g ) C > (E g ) Ge decreased, then Ec and E v decreases but E g
semiconductor, relaxation time increases.
decreases but effect of relaxation time 3 n-type is obtained by doping the Ge or
will be ignored. Si with pentavalent atoms. In n-type 6 In an unbiased p -n junction, the diffusion
semiconductor, electrons are majority of charge carriers across the junction takes
2 Carbon, silicon and germanium are carriers and holes are minority place from higher concentration to lower
semiconductors. carriers, hence option (c) is correct. concentration. Thus, option (c) is correct.
376 40 DAYS ~ JEE MAIN PHYSICS DAY THIRTY FOUR

7 In forward biasing more number of 16 For reverse biasing of an ideal diode, continuously without being damaged.
electrons enter in n-side from battery the potential of n-side should be higher The Zener diode is used as a voltage
thereby increasing the number of than potential of p-side. Only option (d) regulator as constant voltage at variable
donors on the n-side. is satisfying the criterion for reverse current under reverse bias is obtained
biasing. from it.
8 In a p-n junction in forward bias
potential barrier V B as well as the width 17 For forward biased condition of a p-n 28 When a junction diode is forward biased
of charge depleted region x decreases. junction, p-junction should be at higher energy is released at the junction due to
potential and n-junction should be at recombination of electrons and holes. In
9 The resistivity of a semiconductor lower potential. So, option (c) is correct. the junction diode made up of gallium
decreases with increase in temperature arsenide or indium phosphide, the
exponentially. Hence, option (c) is 18 Potential drop in a silicon diode in energy is released in visible region. Such
correct. forward bias is around 0.7 V. a junction diode is called Light Emitting
In given circuit, potential drop across Diode or LED. The radiated energy
10 When p-n junction is forward biased, it 200 Ω resistor is emitted by LED is equal or less than band
opposes the potential barrier across ∆V net 3 − 07
. gap of semiconductor.
I = =
junction. When is reverse biased, it R 200
supports the same. 29 When base region has larger width,
⇒ I = 0.0115 A ⇒ I = 11.5 mA
electron-hole combination increases the
11 In forward biasing for an ideal diode 19 Given, f = 50 Hz and T = 1 base current. The output collector
resistance of diode is zero and whole 50 current decreases by the relation,
resistance in the circuit is R. Hence, For full-wave rectifier, T1 =
T
=
1 I E = I B + IC .
voltage across R is V. 2 100
and f1 = 100 Hz
30 The common-emitter transistor amplifier
12 Forward bias is obtained when the has input resistance equal to 1 kΩ
negative terminal of the battery is 20 Given figure is half wave rectifier as diode (approx.) and output resistance equal to
connected to the n-side and the positive conducts only for positive half-cycle. 10 kΩ (approx.). The output current in
terminal to the p -side of the Hence, output waveform is obtained for CE amplifier is much larger than the
semiconductor. Then, the negative half cycle only as in figure (c). input current.
terminal will repel free electrons in the 21 Zener breakdown in a semiconductor
n-section towards the junction and the diode occurs when reverse bias exceeds
SESSION 2
positive terminal on the p-side will certain value, which is known as 1 Power gain = Current gain × Voltage
push the holes towards the junction. breakdown or Zener or Avalanche gain
voltage. α 0.98
13 If V is the voltage across the junction Current gain = β = =
and I is the circuit current, then 22 For same value of current higher value 1 − α 1 − 0.98
E V V E of voltage is required for higher = 49
V + IR = E or I = − = − +
R R R R frequency.
 500 × 103 
 R2 
Slope of load line
23 Zener diode works in breakdown AV = 49   As, A v = β R 

1 1  R1 
 1
=− = = 10−3 AV −1 region.
R 1000
So, Simple diode → (p) Power gain = 6.0625 × 106
14 The diode is forward biased, hence the Zener diode → (q)  500 × 103 
= 49 ×   × 49
potential barrier decreases and becomes Solar cell → (r)  R1 
less. Though in both the options (c) and Light dependent resistance → (s) R1 = 198 Ω
(d). The diode is forward biased but in
24 2 Mobility of electron (µ ) = σ
Vi Vo
(d ) the barrier width is less. …(i)
ne
15 In the given circuit diode D1 is reverse t t 1
Resistivity (ρ) = …(ii)
biased while D2 is forward biased, so the (Input) (Output) σ
circuit can be redrawn as From Eqs. (i) and (ii), we get
I 4Ω In a CE n-p-n transistor amplifier output 1
is 180° out of phase with input. µ = …(iii)
neρ
D2 25 The condition for a circuit to oscillate where, n = number of free electrons per
are unit volume
12 V 2Ω
(i) feedback should be positive N0 × d
1 n=
(ii) output voltage feedback B = atomic weight
A 6.023 × 1023 × 8.96
∆V f =
(2 − 0.5)
Apply KVL to get current flowing 26 R f = = = 103 Ω 63.54
through the circuit ∆I f 1.5 × 10−3 = 8.5 × 1022 …(iv)

− 12 + 4 I + 2I = 0 = 1 kΩ From Eqs. (iii) and (iv), we get


1
12 27 Zener diodes are specially designed µ =
or I = = 2A junction diodes, which can operate in 8.5 × 1022 × 1.6 × 10−19 × 17. × 10−6
6
the reverse breakdown voltage region = 43.25cm2 / Vs
DAY THIRTY FOUR ELECTRONIC DEVICES 377

3 Consider the LED as For V i < 0, the diode is reverse biased The current through R2 is
a point source of and hence offer infinite resistance, so VR 10
I R2 = 2 =
light. circuit would be like as shown in Fig. R2 1500
r
Let power of the P (ii) and Vo = 0.
= 0.667 × 10−2 A
LED is P. For V i > 0, the diode is forward biased = 6.67 mA
Intensity at r from and circuit would be as shown in Fig.
The voltage drop across R1 is
the source (iii) and Vo = V i .
P V R1 = 15V − V R2
I = ...(i) Hence, the option (d) is correct.
= 15 − 10 = 5V
4 πr 2
1
7 For forward bias mode, the p-side of The current through R1 is
As we know that, I = ε0E 20 c ...(ii) diode has to be at higher potential than VR 5
2
n-side. The meters used are DC, so we I R1 = 1 = = 10−2 A
From Eqs. (i) and (ii), we can write R1 500
have to be careful while connecting
P 1 = 10 mA
= ε0E 20 c them w.r.t. polarity.
4 πr 2 2 The current through the zener diode is
Last point is to decide the range of I z = I R1 − I R2 = (10 − 6.67)mA
2P 2 × 01. × 9 × 109
or E 20 = = meters, the range of meters has to be in
4 πε0r c
2
1 × 3 × 108 = 333
. mA
such a way that we can have the
or E 20 = 6 ⇒ E 0 = 6 = 2.45 V/m readings which leads to plot on realistic 10 ∴ R = ρl = L Q ρ = 1 
 
scale. If we take 0-20 A ammeter, then A n i e (µ e + µ h )A  σ
4 Since, no conduction is found when reading we read from this is tending to 0 0.928 × 10−2
multimeter is connected across P and Q, =
it means either both P and Q are n-type to 5 divisions which is not fruitful. [2.5 × 1019 × 1.6 × 10−19
or p-type. So, it means R is base, when In options (c) and (d), the polarity of (0.39 + 0.19) × 10−6]
R is connected to common terminal and ammeter is not correct. Hence, (b) is
conduction is seen when other terminal = 4000 Ω or 4 k Ω
correct circuit.
is connected to P or Q. So, it means 11 For forward biased p-n junction diodes
transistor is n-p-n with R as base. 8 In the given circuit, diode D1 is reverse its resistance is zero.
biased, so it will not conduct. Diode D2
5 We know that resistance of conductor is So, net resistance of circuit
and D3 are forward biased, so they 10 × 10
directly proportional to temperature
conduct. The equivalent circuit is as = 10 + = 15kΩ
(i.e. R ∝ ∆t ), while resistance of 10 + 10
shown below: V 30
semiconductor is inversely proportional Net current I = =
D3
to temperature  i.e. R ∝
1 5Ω R 15 × 103
.
 ∆t  = 2 × 10−3 A
Therefore, it is clear that resistance of So, potential difference across
conductor decreases with decrease in AB = 2 × 10−3 × 5 × 103 = 10V
temperature or vice-versa, while in case 20Ω IC
of semiconductor, resistance increases D2 5Ω 12 ∴ β = and I E = IC + I B
with decrease in temperature or IB
vice-versa. Since, copper is pure IC
∴ β=
conductor and germanium is a I E − IC
semiconductor, hence due to decrease 10 V
Now, the equivalent resistance of circuit 5488
.
in temperature, resistance of conductor = = 49
decreases while that of semiconductor is . − 5488
560 .
(5 + 5) × 20 10 × 20
increases. Req = = 13 As, we know, in case of a
(5 + 5) + 20 10 + 20
6 During − ve cycle, diode will not allow common-emitter configuration, DC
200 20
the signal to pass. = = Ω current gain,
30 3 I
5V Then, current through the battery, α = c.
Ie
V 10 3
I = = = where, Ic is collector current and Ie is
Req 20 / 3 2
Vi RL Vo emitter current
= 1.5A and AC current gain,
I
9 The voltage drop across R2 is β = c.
Ib
–5 V V R2 = V2 = 10 V
Fig. (i) where, Ib is base current.
I Also, Ie = Ib + Ic
Dividing whole equation by Ic , we get
Ie I
R1 500Ω ⇒ = b +1
Vi Vo Vi Vo Ic Ic
15 V IR2
1 1
⇒ = +1
R2 1500Ω
Iz α β
Vz=10 V
β
⇒ α =
Fig. (ii) Fig. (iii) 1+ β
DAY THIRTY FIVE

Gate Circuit
Learning & Revision for the Day
u Logic Gates and Truth Table u The AND Gate u Combination of Logic Gates
u The OR Gate u The NOT Gate u Transistor as a Switch

Logic Gates and Truth Table


Logic Gates The digital circuit that can be analysed with the help of Boolean algebra is
called logic gate or logic circuit. A logic gate has two or more inputs but only one input.
There are primarily three logic gates namely the OR gate, the AND gate and the NOT
gate.
Truth Table The operation of a logic gate or circuit can be represented in a table which
contains all possible inputs and their corresponding outputs is called the truth table. To
write the truth table, we use binary digits 1 and 0.

The OR Gate
The OR gate is a device has two or more inputs and one output. This device combines
two inputs to give one output. The logic symbol of OR gate is
A
Y
B

The Boolean expression for OR gate is


Y =A+B
This indicates Y equals A OR B. PREP
Truth table for OR gate (Y = A + B ) MIRROR
A B Y Your Personal Preparation Indicator

0 0 0 u No. of Questions in Exercises (x)—


0 1 1 u No. of Questions Attempted (y)—
1 0 1 u No. of Correct Questions (z)—
(Without referring Explanations)
1 1 1
u Accuracy Level (z / y × 100)—
The output of an OR gate assumes 1 if one of more inputs assume 1. The output is high
when either of inputs A or B is high, but not if both A and B are low.
u Prep Level (z / x × 100)—

In order to expect good rank in JEE,


your Accuracy Level should be above
85 & Prep Level should be above 75.
DAY THIRTY FIVE GATE CIRCUIT 379

The logic symbol of NAND gate is shown as


The AND Gate
The AND gate a device has also two or more inputs and one A
output. The output of AND gate is equal to product of its Y
B
inputs. The logic symbol of AND gate is
A
Y The Boolean expression of NAND gate is Y = A ⋅ B, which
B
indicates ‘A AND B are negated’.
The logic symbol of AND gate is given as under. The Boolean Truth table for NAND gate
expression for AND gate is Y = A ⋅ B, this indicates Y equals to
A B Y′ Y
A AND B.
0 0 0 1
Truth table for AND gate (Y = A ⋅ B ) 1 0 0 1
A B Y 0 1 0 1
0 0 0 1 1 1 0
0 1 0
1 0 0
2. NOR Gate
1 1 1
In this type of gate, the output of OR gate is fed to input of the
The output of an AND gate is 1 only, when all the inputs NOT gate and final output is obtained at output of the NOT
assume 1. gate.
A y′
y
The NOT Gate B
The NOT gate is a device which has only one input and only
one output. Its output is complement of input. The logic The logic symbol of NOR gate is shown as
symbol of NOT gate is as shown in figure.
A
y=A+B
B
A Y
The Boolean expression for NOR gate is Y = A + B, which
The Boolean expression for NOT gate is indicates that ‘A OR B are negated’
Y = A,
Truth table for NOR gate
which indicates Y equals NOT A.
A B Y′ Y
Truth Table for NOT gate (Y = A ) 0 0 0 1
A Y 1 0 1 0
0 1 0 1 1 0
1 0 1 1 1 0

The output of a NOT gate assumes 1, if input is 0 and NOTE • NAND and NOR gates are known as universal gate.
vice-versa. These basic gates (OR, AND and NOT) can be
• The Boolean expressions obey the commutative law,
combined in various ways to provide large number of
associative law as well as distributive law.
complicated digital circuits.
Commutative law
(i) A + B = B + A (ii) A ⋅ B = B ⋅ A
Combination of Logic Gates Associative law
(iii) A + (B + C ) = ( A + B ) + C (iv) ( A ⋅ B ) ⋅ C = A ⋅ (B ⋅ C )
NAND gate and NOR gate are used to make any gate.
Distributive law
(v) A ⋅ (B + C ) = A ⋅ B + A ⋅ C
1. NAND Gate (vi) A + A ⋅ B = A + B
In this type of gate, the output of AND gate is fed to input of a (vii) A + A ⋅ B = A
NOT gate and final output is obtained at output of NOT gate. (viii) A ⋅ ( A + B ) = A
(ix) A ⋅ ( A + B ) = A ⋅ B
A
Y′ (x) A ⋅ B = A + B
Y
B (xi) A + B = A ⋅ B
(xii) A = A
380 40 DAYS ~ JEE MAIN PHYSICS DAY THIRTY FIVE

(Vce ) across it. In both the cut-off and saturation regions, the
Transistor as a Switch power dissipated by the transistor is at its minimum.
The circuit resembles that of the Common-Emitter circuit. Vcc
The difference this time is that to operate the transistor as a
switch the transistor needs to be turned either fully ‘‘OFF’’ Load
Flyback Relay
(Cut-off) fully ‘‘ON’’ (Saturated). Diode Output
An ideal transistor switch would have an infinite resistance
when turned ‘OFF’ resulting in zero current flow and zero Ic
RB ia Vce
resistance, when turned ‘‘ON’’, resulting in maximum current
flow. + β
Vin Ω R
In practice, when turned ‘‘OFF’, small leakage currents flow –
through the transistor and when fully “ON” the device has
a low resistance value causing a small saturation voltage

DAY PRACTICE SESSION 1

FOUNDATION QUESTIONS EXERCISE


1 The output of a two input OR gate is fed to a NOT gate, 6 Which of the following gates will have an output of 1 ?
the new gate obtained is
1 0
(a) OR gate (b) NOT gate (a) (b)
(c) NOR gate (d) NAND gate 0 1

2 Which of the following is the truth table for NOT gate? 0


(c) (d) None of these
 1 1  1 0  0 1  0 1 1
(a)   (b)   (c)   (d)  
 0 0  0 0  1 0  1 1
3 The output of OR gate is high 7 The following truth table is for
(a) if either or both the inputs are 1 A B Y
(b) only if both inputs are 1
1 1 0
(c) if either input is zero
(d) if both inputs are zero 1 0 1
0 1 1
4 To get an output 1 from the circuit as shown in the figure,
0 0 1
the input must be
A
(a) NAND (b) AND (c) XOR (d) NOT

B 8 Which of the following is the truth table for the circuit


Y
below?

A Y
C
A Y A Y
(a) A = 0, B = 1, C = 0
(b) A = 1, B = 0, C = 0 (a) 1 0 (b) 0 0
(c) A = 1, B = 0, C = 1 0 1 1 1
(d) A = 1, B = 1, C = 0
A Y A Y
5 Digital circuit can be made by the repetitive use of (c) (d) 0 1
1 1
(a) OR gates (b) AND gates 0 0
0 1
(c) NOT gates (d) NAND gates
DAY THIRTY FIVE GATE CIRCUIT 381

9 The circuit as shown in figure below will act as (c) NOR gate (d) NAND gate

A Y′ 15 The diagram of a logic circuit is given below. The output


Y F of the circuit is given by
B

(a) OR gate (b) AND gate W


(c) XOR gate (d) None of these X
10 The circuit as shown below will acts as F

A W
Y Y

B (a)W ⋅ (X + Y ) (b)W ⋅ (X ⋅Y )
(c)W + (X ⋅Y ) (d)W + (X + Y )
(a) AND gate (b) OR gate
(c) NAND gate (d) NOR gate 16 The output, Y of given logic circuit is
A
11 The circuit is equivalent to
B
NOR NAND NOT
A Y
Y
B
(a) AND gate (b) NAND gate
(c) NOR gate (d) OR gate C
12 The output of an OR gate is connected to both the inputs (a) A ⋅ (B + C) (b) A ⋅ (B ⋅ C)
of a NAND gate. The combination will serve as a (c) (A + B) ⋅ (A + C) (d) A + B + C
ª AIEEE 2011 17 What will be the input of A and B for the Boolean
(a) OR gate (b) NOT gate expression ( A + B ) ⋅ ( A ⋅ B ) = 1 ?
(c) NOR gate (d) AND gate
(a) 0, 0 (b) 0, 1
13 Truth table for system of four NAND gates as shown in (c) 1, 0 (d) 1, 1
figure is ª AIEEE 2012 18 Which of the following is not equal to 1 in Boolean
A algebra?
(a) A + 1 (b) A ⋅ A
Y
(c) A + A (d) A ⋅ A

B
Direction (Q.Nos. 19-21) Each of these questions contains
A B Y A B Y two statements: Statement I and Statement II. Each of these
0 0 0 0 0 0 questions also has four alternative choices, only one of which
(a) 0 1 1 (b) 0 1 0 is the correct answer. You have to select one of the codes
1 0 1 1 0 1
(a),(b), (c),(d) given below.
1 1 0 1 1 1 (a) Statement I is true, Statement II is true; Statement II is
the correct explanation for Statement I
(b) Statement I is true, Statement II is true; Statement II is
A B Y A B Y
not the correct explanation for Statement I
0 0 1 0 0 1
(c) Statement I is true; Statement II is false
(c) 0 1 1 (d) 0 1 0
(d) Statement I is false; Statement II is true
1 0 0 1 0 0
1 1 0 1 1 1 19 Statement I The logic gate NOT cannot be built using
diode.
14 The combination of gates shown below yields Statement II The output voltage and the input voltage of
the diode have 180° phase difference.
A
20 Statement I NOT gate is also called invertor.
X
Statement II NOT gate inverts the input signal.
B 21 Statement I NAND or NOR gates are called digital
building blocks.
ª AIEEE 2010
(a) OR gate (b) NOT gate Statement II The repeated use of NAND or NOR gates
can produce all the basic or complicated gates.
382 40 DAYS ~ JEE MAIN PHYSICS DAY THIRTY FIVE

DAY PRACTICE SESSION 2

PROGRESSIVE QUESTIONS EXERCISE


1 The output of an OR gate is connected to both the inputs 4 In the following combinations of logic gates, the outputs
of a NAND gate , the truth table is of A, B and C are respectively

A B Y A B Y (A) 1
1
0 0 1 0
1
(a) 0 0 (b)
1 1 1 0 1 0 (B) 0
0 1 0 1 0 0 1
1 1 0
A B Y 1
(C)
1
0 0 0 0
(c) (d) None of these
0 1 0
1 0 0 (a) 0,1,1
1 1 1 (b) 0,1, 0
(c) 1,1, 0
2 For the given combination of gates, if the logic states of (d) 1, 0,1
inputs A , B and C are as follows. A = B = C = 0 and
5 The truth table of the following combination of gates is
A = B = 1, C = 0, then the logic states of output D are
A
B A Y
B
D
C
(a) Inputs Outputs
(a) 0, 0 (b) 0, 1 (c) 1, 0 (d) 1, 1
A B A⋅B Y
3 The real time variation of input signals A and B are as
0 0 0 1
shown below.If the inputs are fed into NAND gate, then
0 1 1 0
select the output signal from the following
0 0 0 1
1 1 1 0
A
A
Y (b) Inputs Outputs
B B
A B A⋅ B Y

t(s) 1 1 0 1
0 1 0 1
1 0 0 0
Y Y
0 1 1 1
(a) (b)
(c) Inputs Outputs

0 2 4 6 8 t(s) 0 2 4 6 8 t(s) A B A+ B Y = A ⋅ ( A + B)
0 0 0 0
0 1 1 0
1 0 1 1
Y Y
(c) (d) 1 1 1 1

(d) None of the above


0 2 4 6 8 t(s) 0 2 4 6 8 t(s)
DAY THIRTY FIVE GATE CIRCUIT 383

6 The logic circuit shown below has the input waveforms A


and B as shown. Pick out the correct output waveform. A C
(a)
B
A
A
Y (b)
B C
B

Input A (c) A
B C
Input B
A
(d) C
B
(a)
9 Consider two n-p-n transistors as shown in figure. If 0 V
corresponds to false and 5 V corresponds to true, then the
(b) output at C corresponds to
5V

(c)
C
1
A
(d)

2
B
7 In the adjacent circuit, A and B represent two inputs and
C represents the output,
A
ª JEE Main (Online) 2013
C
(a) A NAND B (b) A OR B
B (c) A AND B (d) A NOR B
10 If a, b, c and d are inputs to a gate and x is its output
every time, then as per the following time graph, the gate
is ª JEE Main 2016 (Offline)
The circuit represents
(a) NOR gate (b) AND gate a. b.
(c) NAND gate (d) OR gate
8 Which of the following circuits has given outputs? c. d.

A B C x.
0 0 0 (a) NOT
0 1 0 (b) AND
1 0 1 (c) OR
1 1 0 (d) NAND
ª JEE Main (Online) 2013

ANSWERS

SESSION 1 1 (c) 2 (c) 3 (a) 4 (c) 5 (d) 6 (c) 7 (a) 8 (a) 9 (a) 10 (a)
11 (c) 12 (c) 13 (a) 14 (a) 15 (d) 16 (c) 17 (a) 18 (b) 19 (c) 20 (a)
21 (a)

SESSION 2 1 (b) 2 (d) 3 (b) 4 (c) 5 (c) 6 (a) 7 (d) 8 (c) 9 (a) 10 (c)
384 40 DAYS ~ JEE MAIN PHYSICS DAY THIRTY FIVE

Hints and Explanations


SESSION 1 = (A + B)+ (A + B) = A + B 17 The given Boolean expression can be
Output of NOT gate, written as
1 The combination of OR and NOT gates
is NOR gate. Y = Y2 = A + B Y = ( A + B ) × ( AB )
which is output of NOR gate. = ( A × B )× ( A + B )
2 Truth table of NOT gate is
12 A = ( A × A ) × B + A( B × B )
A B Y′ Y
B = A×B + A×B
A B
Y ¢= A + B and Y = Y ¢ = A + B = A×B
0 1
1 0 i.e. output of a NOR gate. So, the truth table is

3 OR gate output is high, if anyone or 13 Boolean expression for the given circuit A B Y
both input are high. 0 0 1
Y = (( A × ( A × B ))) × (B × ( A × B ))
4 The Boolean expression for the given 1 0 0
combination is Y = ( A + B ) × C = ( A + A × B ) × (B + A × B )
0 1 0
The truth table is = ( A + A × B ) + (B + ( A × B )) 1 1 0
A B C A+ B Y = (A + B) C = A ×(A × B) + B ×(A × B)
0 0 0 0 0 = A ×(A + B) + B ×(A + B)
18 Here, A × A = 0 always when either
A = 0 or 1.
0 0 1 0 0
= A×B + B× A
0 1 0 1 0 19 NOT gate inverts the signal applied to it.
0 1 1 1 1 But in diode, the input and output are in
A B A B A× B B× A Y
same phase. Thus, NOT gate cannot be
1 0 0 1 0 0 0 1 1 0 0 0 built using diode.
1 0 1 1 1 0 1 1 0 0 1 1
1 1 0 1 0 1 0 0 1 1 0 1 20 NOT gate inverts the input signal i.e. if
1 1 0 0 0 0 0 input is 1 then output will be zero or
1 1 1 1 1
vice-versa. Therefore, it is called as
So, option (a) is correct. invertor. NOT gate inverts the input
Hence, A = 1, B = 0 and C = 1
14 Truth table for given combination is order means that for low input, it gives
5 The repetitive use of NAND and NOR
high output or for high input, it gives low
gate gives digital circuits. A B X output.
6 For option (c), it is a NAND gate, its 0 0 0
output = 01
. = 0=1 0 1 1 21 NAND or NOR gates are called universal
1 0 1 (digital) building blocks because using
7 For NAND gate, Y = AB these two types of gates we can produce
1 1 1
8 The output of the NAND gate is all the basic gates namely OR, AND or
This comes out to be truth table of OR
Y = A× A = A + A = A other complex gates.
gate.
9 The output of NOR gate is made input 15 The output F = (W + X ) × (W + Y ) SESSION 2
for NOT gate.
= (W + X ) + (W + Y ) 1 When two inputs of a NAND gate are
Y = A + B= A+ B
=W + X +W + Y joined together, it works as a NOT gate.
10 The output of two NOT gate is input for The OR gate connected to this NOT gate
=W + X + Y
NOR gate. results is a NOR gate.
Hence, Y = A + B = A × B = A × B 16 The gate circuit is given as
2 The output D for the given combination
(AND gate) A A+B
D = ( A + B )× C = ( A + B ) + C
11 The gate circuit can be shown by given B
two points A and B. If A = B = C = 0, then
Y
NOR NAND NOT D = (0 + 0) + 0
A Y1 Y2
Y A+C = 0+ 0
B
C =1+ 1=1
Output of NOR gate, Y1 = A + B
If A = B = 1, C = 0, then
Output of NAND gate, Y =(A+B) (A+C)
D = (1 + 1) + 0
Y2 = Y1 × Y1 = A + B × A + B For this circuit, output
=1+ 0
= A+ B+ A+ B Y = (A + B)(A + C)
= 0+ 1 = 1
DAY THIRTY FIVE GATE CIRCUIT 385

3 From real time variation of input Inputs Outputs and NOT gate
signals,we can from truth table for A X = NOT A
A B A+B Y = A × (A + B)
and B and conclude output from NAND Rs
gate. 0 0 0 0

Inputs Output 0 1 1 0

A B Y 1 0 1 1
0 0 1 1 1 1 1
1 0 1 6 Truth table V=6V
0 0 1
A B Y Clearly, the function X = NOT (A AND
1 1 0
0 0 0 B) of the logical variables A AND B is
0 0 1
called NAND gate.
0 1 0
From output, we can show real time 10 Output of OR gate is 0 when all inputs
1 0 0
variation of output signal as below.
1 1 1 are 0 and output is 1 when atleast one of
0 0 0 the inputs is 1.
Observing output x It is 0 when all
Y 7 If we give the following inputs to A and
B, then corresponding output is shows inputs are 0 and it is 1 when atleast one
in table of the inputs is 1.
t(s) A B C \ The gate is OR.
0 0 0 Alternative Method
4 A. NAND operation on (1, 1) = 0 0 1 1
NOT operation on (0) = 1 1 0 1 OR Gate
OR operation on (1, 0) = 1 1 1 1 a b c d x
B. NOT operation on (0, 1) = (1, 0) The above table is similar to OR gate. 0 0 0 0 0
NAND operation on (1, 0) = 1
8 Observing the given gate we observe 0 0 0 1 1
C. OR operation on (1, 1) = 1 that gate would be same as given in
0 0 1 0 1
AND operation on (1, 0) = 0 option in which.
The values A = 0, B = 0 gives output 0 0 0 1 1 1
5 Let us draw the given combination The values A = 0, B = 1 gives output 0
pointing out that the first gate is OR 0 1 0 0 1
The values A = 1, B = 0 gives output 1
gate second gate is AND gate. The
The value A = 1, and B = 1 gives output 0 0 1 0 1 1
inputs of the OR gate, are A and B ,
and its output is A + B that is A OR B. 9 From the figure of AND gate 0 1 1 0 1
The inputs of the AND gate are 0 1 1 1 1
A
A and A + B and its output is
A × ( A + B ) that is A AND (A or B ). The X = A and B 1 0 0 0 1
truth table for the output is Y
= A.( A + B ) is as follows B

A V = 5V
A Y
AND
OR V=0
A·(A + B)
A+B
B
DAY THIRTY SIX

Communication
Systems
Learning & Revision for the Day
u Basic Elements of a u Optical Communication u Propagation of
Communication System u Modulation Electromagnetic Waves
u Line Communication u Demodulation or Detection u Satellite Communication

Basic Elements of a Communication System


A communication system is a set up used to transmit information from one point to
another. The essential parts of a communication system are transmitter, communi-
cation channel and receiver as shown in the following block diagram.
Encoding Decoding demodulation
modulation (distortion)
Information
source Transmitter Channel Receiver Destination

Noise
(i) Transmitter Transmitter converts the message signal produced by information
source into a form (e. g., electrical signal) that is suitable for transmission through
the channel to the receiver.
(ii) Communication Channel Communication channel is medium (transmission line,
an optical fibre or free space) which connects a receiver and a transmitter. It carries
the modulated wave from the transmitter to the receiver.
(iii) Receiver This set up receives the signals from the communication channel and PREP
converts these signals into their original form. MIRROR
Your Personal Preparation Indicator

Important Terms Used in Communication System u No. of Questions in Exercises (x)—


Signal Signal represents the electrical analog of the information. It can be analog or u No. of Questions Attempted (y)—
digital. u No. of Correct Questions (z)—
(Without referring Explanations)
(a) Analog Signal Signal which varies continuously with respect to time is called
analog signal. u Accuracy Level (z / y × 100)—
u Prep Level (z / x × 100)—

y
In order to expect good rank in JEE,
t your Accuracy Level should be above
85 & Prep Level should be above 75.
DAY THIRTY SIX COMMUNICATION SYSTEMS 387

(b) Digital Signal A digital signal has two levels of  n2 


current or voltage represented by 0 or 1. It is usually in i = sin −1  
the form of pulses.
 n1 
1
Buffer coating 10-100 mm
100 to
400 µm n1
y n2
Core
0 t Cladding
Transducer A device that converts the message signal For above condition to be fulfilled the light ray must enter the
into electrical signal before feeding it to transmitter. In optical fibre at a maximum acceptance angle θ 0 from the axis of
other words, transducer converts one form of energy into fibre such that
another.
 n2 − n2 
Noise It refers to the unwanted signals that tend to (θ 0 ) max = sin −1  1 2

disturb the transmission and hence a distorted version of  n 0 
the transmitted signal reaches at receiver.
Bandwidth Bandwidth refers to the range over, which where, n0 is the refractive index of the outer medium. For air,
the frequencies in a signal vary. n0 = 1 and then
Amplification It is the process of increasing the (θ 0 ) max = sin −1 ( n21 − n22 )
amplitude and thus strength of an electrical signal.
NOTE • Numerical Aperture (NA) = n0 sin(θ0 ) max = n12 − n22
• All the information in optical fibre is carried out by the principal
Line Communication of total internal reflection and all the information is carried in
In line communication, there is a physical connection core of the optical fibre.
between source and destination. The wired connections • If angle of incidence is greater than (θo )max , then total internal
between two points are known as transmission lines. reflection will not take place and some information will be
The wires that are most popular for wired communication lossed.
or line communication are
(i) Co-axial (ii) Parallel wire lines (iii) Twisted pair cables Modulation
The phenomenon of superposition of information signal over a
Optical Communication high frequency carrier wave is called modulation. In this process,
amplitude, frequency or phase angle of a high frequency carrier
Optical communication uses light waves in the frequency wave is modified in accordance with the instantaneous value of
range 1012 to 1016 Hz as the guided wave medium for the low frequency information.
propagation of audio frequency signal.
Main advantage of optical communication system lies in
the fact that here very high bandwidths of MHz and even Need for Modulation
GHz are possible. Consequently, a large number of Digital and analog signals to be transmitted are usually of low
messages can be transmitted through a single cable frequency and hence, cannot be transmitted as such. These
without any risk of their intermixing. signals require some carrier to be transported.
Moreover due to very little line loss the quality of (i) Frequency of Signal The audio frequency signals (20 Hz
reception is vastly superior. to 20 kHz) cannot be transmitted without distortion over
An optical communication system consists of mainly three long distances due to less energy carried by low frequency
parts which are audio waves. The energy of a wave is directly proportional
to square of its frequency. Even if the audio signal is
(i) optical source and modulator
converted into electrical signal, the later cannot be sent
(ii) optical fibre cable, and (iii) optical signal detector. very far without employing large amount of power.
(ii) Height of antenna For efficient radiation and reception, the
Optical Fibre height of transmitting and receiving antennas should be
Optical fibre make use of the principle of total internal comparable to a quarter wavelength of the frequency used.
reflection of light. The refractive index n1 of central core is velocity 3 × 10 8
Wavelength = = metre
higher than refractive index n2 of cladding. frequency frequency (Hz)
Total internal reflection will take place at core-cladding For 1 MHz it is 75 m and for 15 kHz frequency, the height of
interface if angle of incidence there is antennas has to be about 5 km which size is unthinkable.
388 40 DAYS ~ JEE MAIN PHYSICS DAY THIRTY SIX

(iii) Number of channels Audio frequencies are l


Total bandwidth of modulated signal = 2n ⋅ ω m,
concentrated in the range 20 Hz to 20 kHz. This range is where, n = number of significant side band pairs.
so narrow that there will be overlapping of signals. In l
FM technique is more complex and costly. However,
order to separate, the various signals it is necessary to
efficiency is more and audio quality is vastly improved.
convert all of them to different portions of the
Noise level is negligible.
electromagnetic spectrum.
δ
There are two types of modulation In the FM wave, modulation index m f =
νm

1. Amplitude Modulation (AM) where, δ = maximum frequency of deviation


= (νc − ν min ) or (ν max − νc )
l
Amplitude Modulation (AM) is the process of changing the
amplitude (A c) of a carrier wave linearly in accordance ν m = modulation frequency.
with the amplitude of message signal ( Am).
A A − Amin
l
The ratio µ = m = max
Ac Amax + Amin
is called the modulation Demodulation or Detection
The process of recovering the original audio signal from the
index and in practice we maintain µ ≤ 1, so as to avoid
modulated wave is called demodulation.
distortion.
Demodulation can be done by using a diode and a capacitor
l
In AM modulated wave signal we have carrier wave of
filter as shown under
frequency ωc and two side bands of frequencies (ωc − ω m)
and (ωc + ω m), respectively. Thus, total bandwidth of AM A A′
signals is 2ω m.
l
Upper Side Band Frequency (USB) = νc + ν m AM R Output
where, νc = carrier frequency, ν m = signal frequency. Input
l
Lower Side Band Frequency (LSB) = νc − ν m B B′
where, νc = carrier frequency, ν m = signal frequency Working of R-C Circuits The value of R-C is so selected such
l
Bandwidth = ν USB − ν LSB = (νc + ν m) − (νc − ν m) = 2 ν m 1
that << RC, where fc = frequency of carrier wave.
fc
( A c / 2)2 A c2
l
Power of carrier, PC = = For AM modulated wave, a p-n junction diode or a vacuum
R 2R
2 2
tube diode is used as a demodulator. A diode basically acts as
1  µ A c 1  µ A c µ2 A c2 a rectifier and thus, reduces the modulated carrier wave into
l
Power of side band, Psb =   +   =
R 2 2 R 2 2 8R positive envelope only. This positive envelope is sent through
a R-C circuit. Carrier wave passes through the capacitor and
l
AM technique is simpler and cost effective. However, it
AF signal is regenerated across R.
suffers from noisy reception, low efficiency, small
operating range and poor audio quality.
Propagation of Electromagnetic
 µ2 
l
Power dissipated in AM wave, P = Pc 1 +  where,
2
Waves

2 It is that category of communication in which no line or cable
A
Pc = c is power dissipated by unmodulated carrier is used as a communication channel and the modulated signal
2R is propagated through free space.
wave and µ = modulation index.
Different types of propagation depending upon properties are

2. Frequency Modulation (FM) 1. Ground Wave Propagation


l
Frequency modulation is the process of changing the In this type of communication, transmitting and receiving
frequency of a carrier wave in accordance with the antenna are close to surface of the earth. This type of
frequency of message signal. propagation can be sustained only at low frequency (≈ 500 kHz
l
In FM modulated wave the amplitude of wave and total to 1500 kHz). Due to such less frequency range, it is also called
transmitted power remains constant. medium wave propagation.
Frequency of modulated signal consists of central band of
l

frequency ωc and side bands of frequencies


2. Sky Wave Propagation
(ωc ± ω m),(ωc ± 2ω m),(ωc ± 3 ω m) ..... Sky wave is the radiowave which is directed towards the sky
and reflected back by the ionosphere towards the desired
The number of side bands depends on the modulation
location on the earth. Radiowaves of frequencies 2 MHz to 20
index.
MHz can be reflected by the ionosphere.
DAY THIRTY SIX COMMUNICATION SYSTEMS 389

l
Critical Frequencies It is the maximum frequency of the l
Skip distance When the sky wave is reflected back from
radiowaves which can be reflected from ionosphere and the ionosphere having a constant frequency, but less than
returns to the earth. The radiowave will penetrate the that of the critical frequency, then the smallest distance
ionosphere above this frequency. It is given by from the transmitter to the earth’s surface covered by the
fc = 9(N max )1 /2 sky wave is known as skip distance.

where, N max is the maximum electron density of the


ionosphere. 3. Space Wave Propagation
The sky waves being electromagnetic in nature, changes the The transmitted signal is received by the direct interception
dielectric constant and refractive index of the ionosphere. of the signal by receiving antenna. The, frequency range is
The effective refractive index of ionosphere is (100 MHz to 220 MHz). The maximum range of this
transmission depends upon the height of transmitting antenna
81.45 N
µ = µ0 1 − and is given by,
v2
d = 2 hR, R >> h.
where, N = electron density of ionosphere,
where, h = height of the antenna and R = radius of the earth.
ν = frequency of electromagnetic wave in Hz and
µ 0 = refractive index of free space
Satellite Communication
l
Bandwidth of a Communication Channel The difference l
It is mainly done with the help of a geostationary satellite
between the highest and lowest frequencies that a
orbiting the earth in the equatorial plane from West to East
communication channel allows to pass through it is called
at a height of about 36000 km above the surface of the
its bandwidth.
earth, so that its revolution time is 24 h.
Total bandwidth of channel
Number of channels = l
The transmitted signal from the earth station is uplinked to
Bandwidth per channel
satellite. The satellite receives it, demodulate it, amplify it
l
Maximum Usable Frequency (MUF) In this the sky waves and remodulate it and transmit it back. Now it is
with maximum frequencies are sent at some angles towards downlinked to the earth station. A radio transponder does
the ionosphere. Then these rays will again be reflected by all these jobs in a satellite.
the ionosphere to the earth. l
Uplink frequency and downlink frequency are kept
Critical Frequency (CF) widely different, so as to avoid their interference in free
MUF = = CF sec θ
cosθ space.
This is the angle which is formed along the direction of the l
For world wide coverage three geostationary satellites are
incident wave and the normal. required at 120° apart from each other.

DAY PRACTICE SESSION 1

FOUNDATION QUESTIONS EXERCISE


1 The minimum length of antenna required to transmit a 4 If µ1 and µ 2 are the refractive indices of the materials of
radio signal of frequency 20 MHz is core and cladding of an optical fibre, then the loss of
(a) 5 m (b) 7.5 m (c) 2 m (d) 3.75 m light due to its leakage can be minimised by having
2 Repeaters are required for transmitting microwave (a) µ 1 > µ 2
terrestrial communication system over a 40-50 km distance (b) µ 1 < µ 2
(c) µ 1 = µ 2
because
(d) None of the above
(a) microwave power decreases rapidly with distance
(b) the curvature of the earth limits the distance over which 5 Which of the following four alternatives is not correct?
the line of sight can be established We need modulation ª AIEEE 2011
(c) signal to noise ratio decreases rapidly with distance (a) to increase the selectivity
(d) signal distortion creeps in rapidly with distance (b) to reduce the time lag between transmission and
3 The characteristic impedance of a coaxial cable is of the reception of the information signal
order of (c) to reduce the size of antenna
(a) 50 Ω (b) 200 Ω (d) to reduce the fractional band width, i.e. the ratio of the
(c) 270 Ω (d) None of these signal band width to the centre frequency
390 40 DAYS ~ JEE MAIN PHYSICS DAY THIRTY SIX

6 A message signal of frequency ωm is superposed on a 14 The electron density of E , F1 and F2 layers of ionosphere is
carrier wave of frequency ωc to get an Amplitude 2 × 1011m −3, 5 × 1011m −3 and 8 × 1011 m −3, respectively.
Modulated wave (AM). The frequency of the AM wave What is the ratio of critical frequency for reflection of
will be radiowaves?
ωc + ωm ωc − ωm (a) 2 : 4 : 3 (b) 4 : 3 : 2 (c) 2 : 3 : 4 (d) 3 : 2 : 4
(a) ωm (b) ωc (c) (d)
2 2
15 On a particular day, the maximum frequency reflected from
7 A speech signal of 3 kHz is used to modulate a carrier the ionosphere is 9 MHz. On another day, it was found to
signal of frequency 1 MHz, using amplitude modulation.
increase by 1MHz. What is the ratio of the maximum
The frequencies of the side bands will be
electron densities of the ionosphere on the two days?
(a) 1⋅ 003 MHz and 0 ⋅ 997 MHz
(a) 1.23 (b) 1.0 (c) 1.43 (d) 0.75
(b) 3001 kHz and 2997 kHz
(c) 1003 kHz and 1000 kHz 16 Maximum Usable Frequency (MUF) in F-region layers is
(d) 1 MHz and 0.997 MHz x, when the critical frequency is 60 MHz and the angle of
8 In an amplitude modulated wave for audio frequency of incidence is 70°, then x is
500 cycle/s, the appropriate carrier frequency will be (a) 150 MHz (b) 170 MHz (c) 175 MHz (d) 190 MHz
(a) 50 cycle/s (b) 100 cycle/s 17 Frequencies higher than 10 MHz were found not being
(c) 500 cycle/s (d) 50000 cycle/s reflected by the ionosphere on a particular day at a place.
9 For what value of ma will the total power per cycle be The maximum electron density of the ionosphere on the
maximum in the modulated wave? day was near to
(a) 0 (b) 1 (c) 1/2 (d) greater than 1 (a) 1.5 × 1010 m−3 (b) 1.24 × 1012 m−3
(c) 3 × 1012 m−3 (d) None of these
10 In amplitude modulation, sinusoidal carrier frequency used
is denoted by ωc and the signal frequency is denoted by 18 How the sound waves can be sent from one place to
ωm . The bandwidth ( ∆ωm ) of the signal is such that another in space communication?
∆ωm << ωc . Which of the following frequencies is not (a) Through wires
contained in the modulated wave? ª JEE Main 2017 (Offline) (b) Through space
(c) By superimposing it on undamped electromagnetic waves
(a) ωc (b) ωm + ωc
(d) By superimposing it on damped electromagnetic waves
(c) ωc − ωm (d) ωm
11 Choose the correct statement. ª JEE Main 2016 (Offline)
19 To cover a population of 20 lakh, a transmitter tower
should have a height of (Given radius of the
(a) In amplitude modulation, the amplitude of the high
earth = 6400 km, population per square km = 1000) is
frequency carrier wave is made to vary in proportion to
the amplitude of the audio signal. (a) 25 m (b) 50 m (c) 75 m (d) 100 m
(b) In amplitude modulation, the frequency of the high 20 The TV transmission tower in Delhi has a height of 240 m.
frequency carrier wave is made to vary in proportion to The distance up to which the broadcast can be received.
the amplitude of the audio signal.
(Taking the radius of the earth to be 6.4 × 106 m) is
(c) In frequency modulation, the amplitude of the high
(a) 100 km (b) 60 km (c) 55 km (d) 50 km
frequency carrier wave is made to vary in proportion to
the amplitude of the audio signal. 21 A radar has a power of 1 kW and is operating at a
(d) In frequency modulation, the amplitude of the high frequency of 10 GHz. It is located on a mountain top of
frequency carrier wave is made to vary in proportion to height 500 m. The maximum distance upto which it can
the frequency of the audio signal. detect object located on the surface of the earth (Radius
12 A signal of 5 kHz frequency is amplitude modulated on a of earth = 6.4 × 106 m) is ª AIEEE 2012
carrier wave of frequency 2MHz. The frequencies of the (a) 80 km (b) 16 km (c) 40 km (d) 64 km
resultant signal is/are ª JEE Main 2015
Direction (Q. Nos. 22-25) Each of these questions contains
(a) 2 MHz only two statements : Statement I and Statement II. Each of these
(b) 2005 kHz and 1995 kHz questions also has four alternative choices, only one of which
(c) 2005 kHz, 2000 kHz and 1995 kHz is the correct answer. You have to select one of the codes (a),
(d) 2000 kHz and 1995 kHz (b), (c) and (d) given below
13 An EM wave of maximum frequency 300 kHz and critical (a) Statement I is true, Statement II is true; Statement II is
frequency 100 kHz is to be transmitted to a height equal the correct explanation for Statement I
(b) Statement I is true, Statement II is true; Statement II is
to 150 km. Calculate the skip distance. not the correct explanation for Statement I
(a) 624 km (b) 849 km (c) Statement I is true; Statement II is false
(c) 636 km (d) 942 km (d) Statement I is false; Statement II is true
DAY THIRTY SIX COMMUNICATION SYSTEMS 391

22 Statement I Optical fibre communication has immunity to 24 Statement I FM broadcast is preferred over AM
cross-talk. broadcast.
Statement II Optical interference between fibres is zero. Statement II Process of combining the message signals
with carrier wave is called demodulation.
23 Statement I Transducer in communication system
converts electrical signal into a physical quantity. 25 Statement I Modem is a demodulator.
Statement II For information signal is to be transmitted Statement II It works only in a transmitting and receiving
directly to long distances, modulation is necessary. mode.

DAY PRACTICE SESSION 2

PROGRESSIVE QUESTIONS EXERCISE


1 A telephonic communication service is working at carrier 7 A diode detector is used to detect an amplitude
frequency of 10 GHz. Only 10% of it is utilised for modulated wave of 60% modulation by using a
transmission. How many telephonic channels can be condenser of capacity 250 pF in parallel with a load
transmitted simultaneously, if each channel requires a resistance 100 kΩ. Find the maximum modulated
bandwidth of 5 kHz? ª JEE Main 2018 frequency which could be detected by it. ª JEE Main 2013
(a) 2 × 103 (b) 2 × 104 (c) 2 × 105 (d) 2 × 106 (a) 10.61 MHz (b) 10.61 kHz (c) 5.31 MHz (d) 5.31 kHz
2 A diode AM detector with the output circuit consisting of 8 What is the modulation index if an audio signal of
R = 1 kΩ and C = 1µF would be more suitable for amplitude one half of the carrier amplitude is used in AM?
detecting a carrier signal of (a) 1 (b) 0
(a) 10 kHz (b) 1 kHz (c) 0.75 kHz (d) 0.5 kHz (c) 0.5 (d) greater than 1
3 In optical communication system operating at 1200 nm, 9 For 100% modulation, the power carried by the side
only 2% of the source frequency is available for TV bands (PSB ) is given by
transmission having a bandwidth of 5 MHz. The number 1
(a) PSB = P (b) PSB = 3 P (c) PSB = P (d) PSB = 0
of TV channels that can be transmitted is 3
(a) 2 million (b) 10 million (c) 0.1 million (d) 1 million Direction (Q. Nos. 10-12) Each of these questions contains
two statements : Statement I and Statement II. Each of these
4 If sky wave with a frequency of 50 MHz is incident on D questions also has four alternative choices, only one of which
region at an angle of 30°, then angle of refraction is is the correct answer. You have to select one of the codes (a),
(a) 15° (b) 30° (c) 60° (d) 45° (b), (c) and (d) given below
(a) Statement I is true, Statement II is true; Statement II is
5 Three waves A , B and C of frequencies 1600 kHz, 5 MHz the correct explanation for Statement I
and 60 MHz, respectively are to be transmitted from one (b) Statement I is true, Statement II is true; Statement II is
place to another. Which of the following is the most not the correct explanation for Statement I
appropriate mode of communication? (c) Statement I is true; Statement II is false
(a) A is transmitted via space wave while B and C are (d) Statement I is false; Statement II is true
transmitted via sky wave
10 Statement I Sky wave signals are used for long
(b) A is transmitted via ground wave, B via sky wave and C
distance radio communication. These signals are in
via space wave
general, less stable than ground wave signals.
(c) B and C are transmitted via ground wave while A is
transmitted via sky wave Statement II The state of ionosphere varies from hour to
(d) B is transmitted via ground wave while A and C are
hour, day to day and season to season. ª AIEEE 2011
transmitted via space wave 11 Statement I Higher the modulation index, the reception
6 Consider telecommunication through optical fibres. will be strong and clear.
Which of the following statements is not true? Statement II The degree, to which the carrier wave is
(a) Optical fibres can be graded refractive index modulated is called modulation index.
(b) Optical fibres are subjected to electromagnetic 12 Statement I Television signals are received through sky
interference from outside wave propagation.
(c) Optical fibres have extremely low transmission loss Statement II The ionosphere reflects electromagnetic
(d) Optical fibres may have homogeneous core with a waves frequencies less than a certain critical frequency.
suitable cladding
392 40 DAYS ~ JEE MAIN PHYSICS DAY THIRTY SIX

ANSWERS

SESSION 1 1 (d) 2 (b) 3 (a) 4 (a) 5 (b) 6 (b) 7 (a) 8 (d) 9 (b) 10 (d)
11 (a) 12 (c) 13 (b) 14 (c) 15 (a) 16 (c) 17 (b) 18 (c) 19 (b) 20 (c)
21 (a) 22 (a) 23 (d) 24 (c) 25 (d)

SESSION 2 1 (c) 2 (a) 3 (d) 4 (b) 5 (b) 6 (b) 7 (b) 8 (c) 9 (c) 10 (b)
11 (b) 12 (d)

Hints and Explanations


SESSION 1  2
9 Since, P = Pc 1 + m a   
2

2  13 ∴ D skip = 2h  ν  − 1
1 ν = 20 MHz = 20 × 10 Hz
6
  νc 
Wavelength of antenna is, Power will be maximum, if m a = 1 2
 300 
Therefore, Pmax = Pc 1 +  = 2 × 150   −1
c 3 × 108 1
λ= = = 15 m  100 
ν 20 × 106  2
3 = 2 × 150 × 2 2
The minimum length of antenna = Pc = 1.5 Pc
λ 15 2 = 300 × 2 × 1.414
= = = 3.75 m = 2.828 × 300 = 848.4 ~ 849 km
4 4 10 Frequency spectrum of modulated wave
is 14 Critical frequency for reflection of
2 To increases the range of transmission
of microwaves, a number of antennas radiowaves is given by
are erected in between the transmitting LSB USB νc ∝ N 1/2
and receiving antennas. Such antennas νCE : νCF1 : νCF2
in between the transmitting and ωm ωc–ωm ωc ωc+ωm
receiving antennas are known as = (2 × 1011 )1/ 2 : (5 × 1011 )1 / 2 : (8 × 1011 )1/ 2
Bandwidth
repeaters. = 2:3: 4
Clearly, ω m is not included in the
3 Coaxial cable have a characteristic  
2
9 + 1
2
15 ∴ N ′max =  V ′c  = 
spectrum.
impedance from 40 Ω to 150 Ω. So, 
11 In amplitude modulation, the amplitude N max  Vc   9 
option (a) is correct.
2
of the high frequency carrier wave is  10 
4 Refractive index of core is always =   = 1.23
greater than refractive index of
made to vary in proportion to the  9
amplitude of the audio signal.
cladding, to minimise the loss of light. 16 ∴ MUF = v c / cos i = 60 × 106 / cos 70°
In frequency modulation, the frequency
5 Modulation does not change time lag of the high frequency carrier signal = 60 × 106 ×
1
between transmission and reception. varies with the frequency of audio 0342
.
6 In amplitude modulation the frequency signal. = 17543
. × 106
of modulated wave is equal to the 12 Frequency associated with AM are = 175.43 MHz ≈ 175MHz
frequency of carrier wave. Thus, option fc − f m , f , fc + f m
(b) is correct.
17 The critical frequency of a sky wave for
According to the question reflection from a layer of atmosphere is,
7 Here, ∆ν = 3 kHz = 0.003 MHz fc = 2 MHz = 2000 kHz νc = 9(N max )1 /2
Using amplitude modulation, the f m = 5kHz N max = number density of
frequencies of the side band Thus, frequency of the resultant ionosphere
= (ν + ∆ν) and (ν − ∆ν) signal is/are carrier frequency ν2c (10 × 106 )2 1014 −3
fc = 2000 kHz, ⇒ N max = = = m
= (1 + 0.003 ) and (1 − 0.003) 81 81 81
= 1. 003 MHz and 0.997 MHz LSB frequency = 124
. × 10 m12 −3

Thus, option (a) is correct. fc − f m = 2000 kHz −5 kHz


= 1995 kHz 18 In space communication signals are sent
8 Carrier frequency is always greater than directly from transmitting antenna to
modulating frequency (i.e. audio and USB frequency
receiving antenna by superimposing it on
frequency), so option (d) is appropriate fc + f m = 2005kHz
undamped electromagnetic waves.
carrier frequency.
DAY THIRTY SIX COMMUNICATION SYSTEMS 393

19 Area of region covered = π(2hR ) SESSION 2 6 Some of the characteristics of an optical


In 1 km2 = 1000 people fibre are as follows
1 Only 10% of 10 GHz is utilised for
1 (i) It works on the principle of total
× 20 × 106 = 2 × 103 = A transmission.
1000 internal reflection.
∴ Band available for transmission
2 × 103 = π (2 × h × 6400) 10 (ii) It consists of core made up of
= × 10 × 109 Hz
2 × 103 100 glass/silica/plastic with refractive
⇒ h= = 0.0497 km index n1 , which is surrounded by
π × 2 × 6400 = 109 Hz
a glass or plastic cladding with
= 497
. m ≈ 50 m Now, if there are n channels each using 5 refractive index n2 (n2 > n1 ). The
kHz, then refractive index of cladding can
20 ∴ d = 2Rh n × 5 × 103 = 109 be either changing abruptly or
= 2 × 6.4 × 106 × 240 m = 55km ⇒ n = 2 × 105 gradually changing (graded
index fibre).
21 Range of radar on earth surface (optical 2 Given, R = 1 k Ω
(iii) There is a very little transmission
distance, for space wave, i.e. line of R = 1 × 103 Ω, C = 1µF = 1 × 10−6 F loss through optical fibres.
view).
In this condition frequency of carrier (iv) There is no interference from
Ra 1
h ng signal, < < fc stray electric and magnetic fields
e RC
1
to the signals through optical
R < < fc fibres.
1 × 103 × 10−6
R
⇒ f c > > 1 kHz 7 D
Range = (R + h ) − R
2 2 Because frequency is greater than 1
fc = 10 kHz
− 2Rh
= 2Rh + h2 ~ Signal C R
3 The frequency optical communication
1 c
= 2 × 6400 × km = 80 km ν=
2 λ
3 × 108 τ = RC = 100 × 103 × 250 × 10−12 s
22 Optical communication is a system by ⇒ ν= −9
which we transfer the informations on 1200 × 10 = 2.5 × 107 × 1012 s
any distance from one location to other = 25 × 10 Hz 13
= 2.5 × 10−5 s
through optical range of frequency But only 2% of the source frequency is The highest frequency which can be
using optical fibre. The optical available for TV transmission detected with tolerable distortion is
interference between fibres is zero. ν ′ = 2.5 × 1014 × 2% 1
Hence, optical fibre communication has f =
2 2πm aRC
immunity to cross-talk. ν ′ = 2.5 × 1014 ×
100 [where, m a is modulation]
23 In any communication system ν ′ = 5 × 1012 Hz 1
= Hz
information (a physical quantity) is first
Number of channels =
ν′ 2 π × 0.6 × 2.5 × 10−5
converted into an electrical signal by a
bandwidth 100 × 104 4
device called transducer. Most of the = Hz = × 104 Hz
5 × 1012 25 × 1.2 π 1.2 π
speech or information signal cannot be Number of channels = = 106
directly transmitted to long distances. 5 × 106 = 10.61 kHz
For this an intermediate step of = 1 million This condition is obtained by applying the
modulation is necessary in which the
4 For D-region, N = 109 m − 3 condition that rate of decay of capacitor
information signal is loaded or
superimposed on a high frequency wave voltage must be equal or less than the rate
81.45 N
which acts as a carrier wave. µ = 1− of decay modulated signal voltage for
ν2 proper detection of modulated signal.
24 In AM modulation, the amplitude of the 81.45 × 109
carrier signal varies in accordance with = 1− ≈1 8 Here, E m = 1 Ec
(50 × 106 )2 2
the information signal. AM signals are
noisy because electrical noise signals sin i Therefore,
µ = =1
significantly affect this. In FM sin r 1
E max = Ec + E m = Ec + Ec = 1.5 Ec
modulation, amplitude of carrier wave 2
or sin r = sin i
is fixed while its frequency is changing. 1
FM gives better quality transmission. It or r = i = 30° E min = Ec − E m = Ec − Ec = 0.5 Ec
is preferred for transmission of music. 2
5 For ground wave propagation, the E max − E min
Demodulation is the process in which Also, m a =
frequency range is 530 kHz to 1710 kHz. E max + E min
the original modulating voltage is For sky wave propagation, the 1.5 Ec − 0.5 Ec
recovered from the modulated wave. frequency range is 1710 kHz to 40 MHz. =
1.5Ec + 0.5 Ec
25 Modem is a modulating and For space wave propagation, the
Ec
demodulating device. It acts as a frequency range is 54 MHz to 4.2 GHz. ma = = 0.5
modulator in transmitting mode and as Thus, option (b) is correct. 2.0Ec
demodulator in receiving mode.
394 40 DAYS ~ JEE MAIN PHYSICS DAY THIRTY SIX

2 2
reflected or refracted back towards earth Hence, for high modulation index or
9 PSB = 1  m aEc  + 1  m aEc  from the ionosphere. greater degree of modulation, the audio

R 2 2  R 2 2 
Since, it is not limited by the curvature signal reception will be clear and strong.
 Ec2 
Q Pc =  of the earth, sky wave propagation can 12 In sky wave propagation, the radiowaves
 2R 
be used to communicate beyond which have frequency between 2 MHz to
m2a Ec2 m2aPc P horizon. Ionosphere is a region of upper 30 MHz, are reflected back to the ground
PSB = = = c [∴m a = 1]
4R 2 2 atmosphere and induces the by the ionosphere. But radio waves
 m2  thermosphere and parts of mesosphere having frequency greater than 30 MHz
Also, P = Pc 1 + a 
2  and exosphere. It is distinguished cannot be reflected by the ionosphere

because it is ionised by solar radiation. because at this frequency they penetrates
Here, ma = 1 the ionosphere. It makes the sky wave
It plays an important part in
 1 3
⇒ P = Pc 1 +  = Pc atmospheric electricity. propagation less reliable for propagation
 2 2 of TV signal having frequency greater
PSB Pc / 2 1 11 The modulation index determines the than 30 MHz.
Hence, = =
P 3 3 strength and quality of the transmitted
Pc Critical frequency is defined as the
2 signal.
highest frequency that is returned to the
1
or PSB = P If the modulation index is small the earth by the ionosphere. Thus, about this
3 amount of variation in the carrier frequency a wave whether it is
amplitude will be small consequently electromagnetic will penetrate the
10 In radio communication, sky wave refers
the audio signal being transmitted will ionosphere and is not reflected by it.
to the propagation of radio waves
not be strong. Hence, option (d) is correct.
DAY THIRTY SEVEN

Unit Test 7
(Modern Physics)
1 The wavelength of incident light falling on a V (Volts) Input B
photosensitive surface is charged from 2000 Å to 2100 Å
16 V
the corresponding change in stopping potential is
(a) 0. 03 V (b) 0. 3 V 8V
(c) 3 V (d) 3 . 3 V
t(s)
2 Ultraviolet light of wavelength 350 nm and intensity 2 4 6 8 10 12
–2
1.00 Wm is incident on a potassium surface. If 0.5% of
Output of a NAND gate on these inputs will be
the photons participate in ejecting the photoelectrons,
V V
how many photoelectrons, are emitted per second, if the
potassium surface has an area of1 cm 2 ?
(a) (b)
(a) 1.76 × 1018 photoelectrons/s
(b) 1.76 × 1014 photoelectrons/s t t
6 8 10 12 6 8 10 12
(c) 8.8 × 1011photoelectrons/ s V V
(d) The value of work function is required to complete the
value of emitted photoelectrons/s (c) (d)
3 Electric field of an electromagnetic wave in vacuum is; t t
 N  rad  8 rad $ 6 8 10 12 6 8 10 12
E =  31
.  . cos 1.8 m  y +  5.4 × 10 s  . t  i
 C   6 A neutron collides with a hydrogen atom in its ground
Wavelength of this wave as it passes through a medium state and excites it to n = 3. The energy given to
3 hydrogen atom in this inelastic collision is (Neglect the
of refractive index will be
2 recoiling of hydrogen atom and assume that energy is not
(a) 3.55 m (b) 2.33 m absorbed as KE of H-atom)
(c) 1.44 m (d) 3.22 m (a) 10.2 eV (b) 12.1 eV (c) 12.5 eV (d) None of these
4 Taking the Bohr radius as a 0 = 53 pm, the radius of 7 An X-ray tube operates at 50 kV. Consider that at each
Li + + ion in its ground state, on the basis of Bohr’s model, collision, an electron converts 50% of its energy into
will be about photons and 10% energy would be dissipated as thermal
(a) 53 pm (b) 27 pm (c) 18 pm (d) 13 pm energy due to the collision then the wavelength of emitted
by photons during 2nd collision is
5 Consider inputs A and B;
(Take, hc = 1242 eV-nm)
V (Volts) Input A
(a) 1.242 nm (b) 1.242 Å (c) 4.968 nm (d) 4.968 Å
10 V 8 For the nuclear reaction, → 86 Rn + 2 He 4 the
88 Ra
226 222

5V
radium nucleus is initially at rest and the alpha particle
carries the energy 5.3 MeV. The energy released in the
t(s) reaction is
2 4 6 8 10 12
(a) 5.4 MeV (b) 5.0 MeV (c) 300 MeV (d) 286 MeV
396 40 DAYS ~ JEE MAIN PHYSICS DAY THIRTY SEVEN

9 The combinations (M) and (N) of the NAND gates are as Point C is kept at a constant voltage of 5 V and point D is
shown below. earthed. If input given at A is
VA
A
Y

B t(s)
2 5
(M)
Then output obtained at B is
A Y VB VB
B
(N) (a) (b)
The output (Y ) of (M) and (N) are equivalent to the t(s) t(s)
output of 2 5 2 5
(a) OR gate and AND gate respectively
(b) AND gate and NOT gate respectively VB VB
(c) AND gate and OR gate respectively
(c) (d)
(d) OR gate and NOT gate respectively
10 The binding energy of a H-atom, considering an electron t(s) t(s)
2 5 2 5
me 4
moving around a fixed nuclei (proton), is B = −
4 n 2 ε 02 h 2 13 The simple Bohr’s model cannot be directly applied to
(m = electron mass). calculate the energy levels of an atom with many
If one decides to work in a frame of reference where the electrons. This is because
electron is at rest, the proton would be moving around it.
(a) of the electrons not being subject to a central force
By similar arguments, the binding energy would be
(b) of the electrons colliding with each other
Me 4
B= = (M = proton mass) (c) of screening effects
8 n 2 ε 02 h 2
(d) the force between the nucleus and an electron will no
This last expression is not correct because longer be given by Coulomb’s law
(a) n would not be integral 14 An electron is trapped in a one dimensional infinite well of
(b) Bohr-quantisation applies only to electron width 250 pm and is in its ground state. What is the
(c) the frame in which the electron is at rest is non-inertial longest wavelengths of light that can excite the electron
(d) the motion of the proton would not be in circular orbits,
from the ground state via a single photon absorption?
even approximately
4 mL2c 2 mLc
11 Light strikes a sodium surface, causing photoelectric (a) λ = (b) λ =
h (ni2 − nf2 ) h (nf2 − ni2 )
emission. The stopping potential for the ejected electrons
8 mL2c 8 mL
is 5.0 V, and the work function of sodium is 2.2 eV. What (c) λ = (d) λ =
h (nf2 − ni2 ) h (nf2 − ni2 )
is the wavelength of the incident light?
(a) 100 nm (b) 170 nm 15 What is the ratio of the shortest wavelength of the Balmer
(c) 150 nm (d) 200 nm series to the shortest wavelength of the Lyman series?
12 Consider a given circuit (a) 1 : 4 (b) 2 : 4
(c) 4 : 1 (d) 1 : 1
C
VCC =+ 5V
16 In the ground state of the hydrogen atom, the electron
has a total energy of −13.6 eV, its kinetic energy is
(a) 12.5 eV (b) 13.6 eV
B (c) 14.9 eV (d) −27. 2 eV
17 A particle of mass m at rest decays into two particles of
A m1 and m2 having non-zero velocities. The ratio of
de-Broglie wavelengths of particles λ 1 / λ 2 is
m1 m2
(a) (b) (c) 1 (d) m2 / m1
m2 m1
D
DAY THIRTY SEVEN UNIT TEST 7 (MODERN PHYSICS) 397

18 If there are 2 bulbs of same power, one of them gives (b) Readings of A are correct
red colour light, while other gives blue colour light. (c) Readings of B are correct
(d) Reading of Both A and B are incorrect
If nr and nb are the number of photons per unit time
emitted by bulbs, then choose correct option; 23 Let potential energy of electron in Bohr’s first orbit of
(r : red; b : blue) hydrogen atom is zero.
(a) nr = nb (b) nr < nb (c) nr > nb (d) nr ⋅ nb = c 2 Then, total energy of electron in IInd orbit is
19 An α-particle makes an elastic head-on collision with a (a) 23.80 eV (b) 27.20 eV (c) 13.6 eV (d) 26.25 eV
proton initially at rest. 24 Consider the process;
Ratio of de-Broglie wavelength associated with α-particle → 228
232
92 U
4
90 Th + 2He
and proton after collision will be Energy released in above process is 5.40 MeV. If this
(a) 2 : 1 (b) 4 : 3 (c) 1 : 2 (d) 2 : 3 energy remains mainly with ‘α’ and daughter nucleus
20 A beam of light is allowed to fall over cathode of a then, kinetic energy of 228
90 Th, nucleus will be
photocell after passing through two polaroids. None of (a) 5.4 MeV (b) 5.3 MeV (c) 0.1 MeV (d) 0.4 MeV
the polaroid is rotated keeping other fixed. 25 The following circuit represents
Variation of photocurrent is best given by
I I A
B
Y
A
(a) (b)
B

θ θ B

(a) OR gate (b) XOR gate (c) AND gate (d) NAND gate
I I
26 Let a sample of a radioactive substance contains N0
number of active nuclei at t = 0. Then, probability that a
(c) (d) randomly choosen nucleus is disintegrated in time t is
e −λt
θ θ (a) 1 − e − λt (b) N 0 e −λt (c) (d) e −λt
N0

21 Two particles A1 and A2 of masses m1, m2 (m1 < m2 ) have 27 Match Column I and Column II and mark correct option.
the same de-Broglie wavelength. Then Column I Column II
(a) then masses are the same A. α-decay p. Large nucleus.
(b) thin energies are the same B. β + decay q. More neutrons in nucleus.
(c) energy of A1 is less than the energy of A2
(d) energy of A1 is more than the energy of A2 C. β − decay r. More protons in nucleus.
D. γ - decay s. More energy in nucleus.
22 Two students makes observations of stopping potentials
E. k - capture t. Proton number is more than 83 in
(V0 ) and frequencies (f ) and plotted their observations
nucleus.
graphically as shown below
V0 A B C D E
(volts) B (10×1014, 5) (a) p q r s t
(b) p,t r,t q,t s r,t
Observations (c) p t q s r
of student (d) t s r q p
A
28 On a particular day, the maximum frequency reflected
f (Hz)
14
A (5×10 , 0) from the ionosphere is 10 MHz. On another day, it was
found to decrease to 8 MHz. What is the ratio of the
V0 maximum electron densities of the ionosphere on the two
(volts) Q (15×1014, 2.5) days?
(a) 20 : 10 (b) 30 : 15 (c) 25 : 16 (d) 24 : 11
Observations
of student 29 A transmitting antenna at the top of a tower has a height
B 32 m and that of the receiving antenna is 50 m. What is
14
f (Hz) the maximum distance between them for satisfactory
P (10×10 , 0) communication in line of sight mode? Given radius of the
Now, choose the correct option; earth 6.4 × 106m.
(a) Both students gives accurate observation (a) 50.0 km (b) 45.5 km (c) 35.5 km (d) 30.2 km
398 40 DAYS ~ JEE MAIN PHYSICS DAY THIRTY SEVEN

Direction (Q. Nos. 30-31) According to Einstein, when a 34 Statement I Targets in X-ray tubes are made from high
photon or light of frequency ν or wavelength λ is incident on melting point metals.
photosensitive metal surface of work function φ 0, where
Statement II Most of the energy of striking electrons is lost
φ 0 < hν (here h is planck’s constant) then the emission of
photoelectrons takes place.The maximum kinetic energy of into collisions which simply appears as thermal energy.
the emitted photoelectrons is given by K max = hν − φ 0 If the 35 Statement I The different lines of emission spectra
frequency of the incident light is ν 0 called threshold (like Lyman, Balmer etc) of atomic hydrogen gas are
frequency. The photoelectrons are emitted from metal without
produced by different atoms.
any kinetic energy.So hν 0 = φ 0
Statement II The sample of atomic hydrogen gas consists
30 Stopping potential of emitted photoelectron is given by
of millions of atoms.
hν − φ0 hν φ0 + h ν
(a) (b) hν − φ0 (c) (d)
e e e Direction (Q. Nos. 36-37) A beam of light has three
31 The variation of maximum kinetic energy (Kmax ) of the wavelengths 440 nm, 495 nm and 660 nm with a total
emitted photoelectrons with frequency ( ν ) of the incident intensity of 3.24 × 10−3 Wm−2 equally distributed amongst the
radiations can be represented by three wavelengths. The beam falls normally on an area of
1.0 cm2 of a clean metallic surface of work function 2.2 eV.
Kmax Kmax
Assume that there is no loss of light by reflection and each
energetically capable photon ejects one electron and take,
h = 6.6 × 10−34 J-s.
(a) (b) 36 Photoelectric emission is caused by
(a) light of wavelength 440 nm alone
(b) light of wavelength 660 nm alone
ν ν (c) lights of wavelengths 440 nm and 495 nm
Kmax Kmax
(d) lights of wavelengths 495 nm and 660 nm
37 The incident energy (in Js −1) of each wavelength is
(c) (d) (a) 3.24 × 10−7 . × 10−7
(b) 162
. × 10−7
(c) 108 (d) 0.81 × 10−7

ν ν Direction (Q. Nos. 38-40) Carbon-14 (symbol 146 C) is


produced by the bombardment of atmospheric nitrogen with
32 Frequencies higher than 10 MHz are found not to be high energy neutrons according to the equation.
7 N + 0 n → 6 C +1 H
14 1 14 1
reflected by the ionosphere on a particular day at a place.
What is the maximum electron density of the ionosphere? Radiocarbon is unstable and decays to nitrogen with a half-life
1014 of 5600 yr. The carbon-14 is incorporated into atmospheric
(a) em−3 (b) 10 em 14 −3
carbon dioxide molecules which are taken in by plants when
9
they breathe in carbon dioxide. Animals which eat the plants
1014 1014
(c) em−3 (d) em−3 also take in carbon-14. By measuring the ratio of the
81 7 concentration of 14C to 12C in any ancient organism, say a tree,
one can determine the date when the organism died.
Direction (Q. Nos. 33-35) Each of these questions
contains two statements : Statement I and Statement II. 38 A capsule contains 8 g of 146 C whose half-life is
Each of these questions also has four alternative choices, only 5600 yr. After 16800 yr, the amount of 146 C left in the
one of which is the correct answer. You have to select one of capsule will be
the codes (a), (b), (c), (d) given below 8
(a) 4 g (b) 2 g (c) g (d) 1 g
(a) Statement I is true, Statement II is true; Statement II is 3
the correct explanation for Statement I
39 Radiocarbon is produced in the atmosphere as a result of
(b) Statement I is true, Statement II is true; Statement II is
not the correct explanation for Statement I (a) collisions between fast neutrons and nitrogen nuclei
(b) the action of cosmic rays on atmospheric oxygen
(c) Statement I is true; Statement II is false
(c) the action of X-rays on carbon
(d) Statement I is false; Statement II is true (d) lighting discharge in atmosphere
33 Statement I As intensity of incident light 40 Choose the only incorrect statement. In radioactive decay
(in photoelectric effect) increases, the number of of an element
photoelectrons emitted per unit time increases. (a) α-particles may be emitted
(b) β-particles may be emitted
Statement II More intensity of light means more energy
(c) γ-rays may be emitted
per unit area per unit time.
(d) the nucleus does not undergo any change
DAY THIRTY SEVEN UNIT TEST 7 (MODERN PHYSICS) 399

ANSWERS
1. (b) 2. (c) 3. (b) 4. (c) 5. (a) 6. (b) 7. (b) 8. (a) 9. (a) 10. (c)
11. (b) 12. (d) 13. (a) 14. (c) 15. (c) 16. (d) 17. (c) 18. (c) 19. (d) 20. (a)
21. (d) 22. (d) 23. (a) 24. (c) 25. (c) 26. (a) 27. (b) 28. (c) 29. (b) 30. (a)
31. (c) 32. (c) 33. (b) 34. (a) 35. (d) 36. (c) 37. (c) 38. (d) 39. (a) 40. (d)

Hints and Explanations


1 Given, λ1 = 2000 Å = 2 × 10−7 m When this wave passes through a 1242
So, required wavelength, λ = nm
−7
3 10000
λ2 = 2100Å = 2.1 × 10 m medium of refractive index, ; its
2 = 1.242 Å
hc
= W + eV1 …(i) wavelength will be
λ1 λ 2 π /1.8 2 22 10 8 As parent nucleus is at rest and emitted
λ2 = 1 = = ×2× × particle (α ) carries some energy, daughter
hc n 3/2 3 7 18
= W + eV2 …(ii) nucleus (Rn) recoils to conserve the
λ2 = 233
. m
momentum.
Subtracting Eq. (ii) from Eq. (i) 4 On the basis of Bohr’s model, The energy released in the reaction
 1 1  n2 h2 n2
hc  −  = e (V1 − V2 ) r = = a0 appears in the form of kinetic energy of
 λ1 λ2  2
4π m KZe 2
Z α-particle and the daughter nucleus.
Change in stopping potential, For Li + + ion, Z = 3; n = 1 for ground Q = Kα + KD
hc  1 1  state.
∆V = V1 − V2 =  −  From momentum conservation,
e  λ1 λ2  Given, a0 = 53 pm pα = pD
6. 6 × 10−34 × 3 × 108 ∴ r =
53 × 12
= 18 pm
Solving above equation, we have
= MD
1. 6 × 10−19 3 Kα = ×Q
MD + Mα
 1

1  5 For a NAND gate, output is
  M + Mα
 2 × 10−7 2.1 × 10−7  ⇒ Q = D × Kα
A B Y MD
6 . 6 × 3 × 0 .1
= V = 0 .3 V 0 0 1 222 + 4
1 . 6 × 2 × 2 .1 = × 5.3
1 0 1 222
2 Energy of photon 0 1 1 = 5.4 MeV
hc 1242 1 1 0
E = = eV = 3.55 eV 9 It follows from the logic symbol (A) that
λ 350 So, output waveform is like option (a).
= 5.68 × 10−19 J X = AB
Let n photons, per unit area per unit
6 The energy taken by hydrogen atom for which the truth table is as follows
corresponds to its transition from n = 1
time are reaching the potassium surface,
then to n = 3 state. A B A B A⋅B X = AB
1.00 ∆E (given to hydrogen atom)
n= = 1.76 × 1018
= 13.6  1 −  = 13.6 × = 12.1 eV
1 8 0 0 1 1 1 0
5.68 × 10−19
 9 9 1 0 0 1 0 1
So, number of photons received by
potassium surface per unit time is, 7 During first collision, Initial energy of 0 1 1 0 0 1
n × Area of potassium surface electron = 50 keV
= 1.76 × 1018 × 1 × 10− 4 = 1.76 × 1014 Energy appearing as photon
1 1 0 0 0 1
Required number of photoelectrons = 50% of 50 keV = 25keV This truth table satisfies the Boolean
emitted per unit time
Energy lost in collision expression X = A + B , which is the OR
0.5
= 1.76 × 1014 × = 8.8 × 1011 = 10% of 50 keV = 5 keV gate. Hence, the logic symbol (A) is
100
Energy left for second collision equivalent to an OR gate. It follows from
3 Phase of wave is ‘ky + ωt ’, = (50 − 25 − 5) keV = 20 keV logic symbol (B) that
2π For second collision, Initial energy
So, k = 1.8 or = 1.8 X = A⋅B = A⋅B
λ = 20 keV
2π which is the Boolean expression for AND
⇒ λ1 = Energy of the emitted photon
gate.
1.8 = 50% of 20 keV = 10000 eV
400 40 DAYS ~ JEE MAIN PHYSICS DAY THIRTY SEVEN

10 If electron is considered at rest, then So, number of photons, 23 PE of electron in Ist orbit = − 27.20 eV
photons are accelerating around COM E P
n= = Now, in IInd orbit,
of system. Thus, with respect to COM hf × ∆t hf 13.6
electrons are also accelerating and Total energy of electron = −
As, f blue > f red n2
hence frame is non-inertial. ⇒ n red > n blue 13.6
=−
11 The energy of an incident photon 19 Let initial speed of α is v 0 and final 22
E = hf = hc /λ, kinetic energy of the = − 3.4 eV
speeds of α and proton are v α and v p .
most energetic electron emitted In IInd orbit, KE = 3.4 eV and
K m = E − φ = (hc /λ ) − φ, eV 0 is related Then, momentum conservation gives
4m pv i = 4m pv α + m pv p PE = − 2 × 3.4 = − 6.8 eV
to kinetic energy by eV 0 = K m ,
{Q m α = 4m p} To make PE = 0 in Ist orbit, energy must
so, eV 0 = (hc /λ ) − φ and be increased by 27.20 eV.
hc Also, e = 1; elastic collision
λ= So, PE in IInd orbit = 2720
. + (−6.8)
eV 0 + φ ⇒ vi − 0 = v p − vα
= 20.40 eV
Elimination of v i gives,
6.6 × 10-34 × 3 × 108 Hence total energy = PE + KE
= 3
5 × 1.6 × 10−19 + 22
. × 1.6 × 10−19 v α = vp = 20.40 + 3.4
8
= 171.8 × 10−9 m λ h m pv p = 23.80 eV.
= 171.8 nm Now, α = ×
λ p mα v α h 24 Energy is distributed in inverse ratio of
≈ 170 nm masses of products,
mp vp
12 Given circuit is a transistorized ‘NOT’ = . k 228
mα v α So, α =
gate. When A is made positive, k Th 4
1 8 2
transistor is ON and it draws maximum = × = where (228 + 4)x = 540
. MeV
current to collector. 4 3 3 540
.
∴ x=
So, V B = 0 for the time A remains 20 In figure (a) following Malus’ law, 232
positive. π 228
intensity reduces upto θ = and then So, k α = × 5.40
14 Energy levels are E n = n h 8 / mL ,
2 2 2 2 232
increases. = 5.30 MeV
f = ∆E / h = (h/8mL2 )(n2f − n2i ) and the Also, intensity ∝ photocurrent. So, k Th = 01. MeV
wavelength of the light is
21 Masses of particles A1 and A2 are 25 Output of upper AND gate = AB
c 8mL2c
λ= = m1 and m2 . Output of lower AND gate = BA,
f h(n2f − n2i )
where, m1 < m2 Output Y = A B + BA
m1
15 The energy E of the photon emitted ⇒ <1 …(i) 26 Probability of surviving after time t
m2
 1 1  =
Number undecayed in time (t )
E = 13.6  2 − 2  Since, both particles have same
 1 Total number
n n 2 de-Broglie wavelength.
N 0 e − λt
Frequency f of the electromagnetic h = = e − λt
Hence, momentum, p = will be same N0
wave f = E /h and the wavelength λ
λ = c/f . for both particles. ∴ Probability of decay
Thus,
1 f
= =
E p2 = 1 − Probability of survival
∴ Energy, E =
λ c hc 2m = 1 − e − λt
13.6  1 1  E2 m 27 For α-decay, electrostatic repulsion must
=  − 2 = 1
hc  n12 n2  E1 m2 be greater than nuclear force; this
E2 happens when nucleus is large.
For which n2 = ∞. For the Balmer <1 [from Eq. (i)] In β + decay, a proton is changed to a
series, n1 = 2 and the shortest E1
neutron. This occurs when protons are
wavelength is λ B = 4hc /13.6. For the E2 < E1 more.
Lyman series, n1 = 1 and the shortest In β − decay, a neutron is changed to a
wavelength is λ L = hc / 13.6. The ratio is
22 As slope of V 0 and f graph = h
e proton. This happens when neutrons are
λ B/ λ L = 4 So, h = e × slope of graph. more.
From readings of A, In k-capture, if protons are large, a
17 Initial momentum = 0 proton and an electrons forms a neutron.
∴ Final momentum = 0 1.6 × 10−19 × (5 − 0)
h= 28 f c = 10 MHz, f ′c = 8 MHz
p+ p = 0 (10 − 5) × 1014
2 2
p1 = p2 = 16 × 10−34 J-s  f 
=  c  =   = 25 : 16
(numerically) N max 10
λ=
h

1 N ′ max  f ′c   8
From readings of B,
p p
1.6 × 10−19 × (2.5 − 0) 29 d m = 2 × 64 × 105 × 32 +
λ1 p2 h=
⇒ = =1 (15 − 10) × 1014
λ2 p1 2 × 64 × 105 × 50 m
= 8 × 10−34 J-s
Energy radiated = 64 × 10 × 10 + 8 × 103 × 10 m
2
18 Power, P = Hence, both of the readings are
time incorrect. = 144 × 102 10 m = 45.5 km
DAY THIRTY SEVEN UNIT TEST 7 (MODERN PHYSICS) 401

32 Critical frequency, 35 In one particular sample, atoms can be = 1.08 × 10−3 Wm −2 .


fc = 10 MHz = 10 Hz 7
excited only upto a particular level. Area of metal surface is
fc = 9(N max )1 /2 A = 1cm2
36 The threshold wavelength is
f hc = 1 × 10−4 m2 .
or (N max )1 /2 = c λ0 =
9 W0 Therefore, energy of each wavelength is
2
(6.6 × 10−34 ) × (3 × 108 ) E =I×A
=  c 
f
or N max = = 1.08 × 10−7 J - s −1
 9 . × 1.6 × 10−19
22
 107 
2 = 6 × 10−7 m 38 Number of half-lives n = 16800 = 3
−3
or N max =   em = 600 nm 5600
 9 
Out of the three given wavelength, two Therefore, the amount of C - 14 left
1014 wavelengths λ1 = 440 nm and λ2 = 495 8g
= em −3 = n
81 nm will cause photoelectric emission as (2)
these wavelengths are less than λ 0. 8g
33 Intensity ∝ number of photons. = 3
37 Intensity of each wavelength is (2)
34 When energy lost in a collision is less, it 1
appears in form of heat radiation. I = . × 10−3
× 324 =1g
3
402 40 DAYS ~ JEE MAIN PHYSICS DAY THIRTY EIGHT

DAY THIRTY EIGHT

Mock Test 1
(Based on Complete Syllabus)

Instructions
1. This question paper contains of 30 Questions of Physics, divided into two Sections :
Section A Objective Type Questions and Section B Numerical Type Questions.
2. Section A contains 20 Objective questions and all Questions are compulsory (Marking Scheme : Correct + 4, Incorrect –1) .
3. Section B contains 10 Numerical value questions out of which only 5 questions are to be attempted (Marking Scheme : Correct + 4, Incorrect 0) .

k1 k1 k2 k2
Section A : Objective Type Questions (a) (b) (c) (d)
k2 k2 k1 k1
1 A running man has half the kinetic energy of a boy of
half his mass. The man speeds up by 1.0 ms −1 and 4 The length of a simple pendulum executing simple
then has the same kinetic energy as the boy. The harmonic motion is increased by 21%. The percentage
original speed of the boy was increase in the time period of the pendulum of increased
(a) 2.4 ms −1 (b) 9.6 ms −1 (c) 4.8 ms −1 (d) 7.2 ms −1 length is
2 The length of the string of a simple pendulum is (a) 11% (b) 21% (c) 42% (d) 10%
measured with a meter scale, is found to be 92.0 cm, 5 Assertion Thin prisms do not deviate light much.
the radius of the bob plus the hook is measured with Reason Thin prism have small angle A and hence, Dm
the help of vernier calliper to be 2.17 cm. Mark out the (minimum deviation) is also very small as Dm = [(1µ 2 − 1) A ],
correct statement. where 1µ 2 is the refractive index of prism w.r.t. medium 1.
(a) Least count of meter scale is 0.1 cm In the light of the above statements, choose the most
(b) Least count of vernier callipers is 0.01 cm appropriate answer from the options given below.
(c) Effective length of simple pendulum is 94.2 cm
(d) All of the above (a) Both A and R are correct and R is the correct explanation
of A.
3 Two bodies A and B of equal mass are suspended (b) Both A and R are correct but R is not the correct
from two separate massless springs of spring explanation of A.
constants k1 and k 2 respectively. If the bodies oscillate (c) A is correct but R is not correct.
vertically such that their maximum velocities are equal, (d) A is not correct but R is correct.
the ratio of the amplitudes of A to that of B is
DAY THIRTY EIGHT MOCK TEST 1 403

6 Two bodies of different masses has been released from (c) 2 photons of energy 3.4 eV
the top of tower. One is thrown in the horizontal direction (d) 1 photons of energy 3.4 eV and 1 electron of 1.4 eV
while other is dropped, then which will reach the ground 11 A non-conducting plate (infinite plane plate) is given a
first? charge in such a way that Q1 appears on one side and Q 2
(a) The body which has been thrown horizontally on other side. The face area of plate is A. Find the electric
(b) The body which has been dropped field at points 1 and 2.
(c) Both will reach the ground simultaneously Q2 Q1
(d) Depends on the velocity with which the first body has
been projected horizontally
7 In a hall, a person receives direct sound waves from a 1 2
source 120 m away. He also receives waves from the same
source which reach him after being reflected from the 25 m
high ceiling at a point half-way between them. The two Q1 + Q2 Q2 − Q1
(a) ,
waves interfere constructively for wave lengths (in metre) of 2 ε0 A 2 ε0 A
5
(a) 10, 5, , … (b) 20,
20 20
, , ... Q1 − Q2 Q1 + Q2
(b) ,
2 3 5 2 ε0 A 2 ε0 A
(c) 30, 20, 10, … (d) 35, 25, 15, …
Q1 + Q2 Q2 − Q1
(c) ,
8 An AC source producing emf ε0 A ε0 A
e = e 0 [cos (100 π s −1) t + cos ( 500 π s −1) t ] Q1 − Q2 Q1 + Q2
(d) ,
is connected in series with a capacitor and resistor. The ε0 A ε0 A
steady state current in the circuit is found to be
12 The emf and internal resistance of the battery as shown in
I = I1 cos [(100 πs −1 ) t + φ1] + I2 cos [(500 π s −1 ) t + φ 2 ]
figure are 4.3 V and 1 Ω respectively. The external
(a) I1 > I 2
resistance R is 50 Ω. The resistance of the voltmeter and
(b) I1 = I 2
ammeter are 200 Ω and 2 Ω respectively. Find the
(c) I1 < I 2
readings of the two meters.
(d) The information is insufficient to find the relation
between I1 and I 2
4.3 V 1Ω
9 In the diagram, a plot between δ (deviation) versus i
(angle of incidence) for a triangular prism is given. From
the observed plot, some conclusions can be drawn. A
50 Ω
Mark out the correct conclusions. 2Ω
δ
V
δ0 200 Ω
(a) 0.1 A, 2 V (b) 0.1 A, 4 V
δm (c) 0. 4 A, 1 V (d) 0.4 A, 4 V
13 Statement I The rocket works on the principle of
i conservation of linear momentum.
i0 π/2
Statement II Whenever there is the change in
(a) The range of deviation for which two angles of momentum of one body, the same change occurs in the
incidence are possible for same deviation is δ 0 − δm momentum of the second body of the same system
(b) The curve is unsymmetrical about i 0 (having two bodies only) but in opposite direction.
(c) For a given δ, i is unique In the light of the above statements, choose the most
(d) Both (a) and (b) are correct appropriate answer from the options given below.
10 A photon of 10.2 eV energy collides with hydrogen atom (a) Statement I is true but Statement II is false.
in ground state inelastically. After few microseconds one (b) Both Statement I and Statement II are true.
more photon of energy 15 eV collides with same (c) Both Statement I and Statement II are false.
hydrogen atom. Then what can be detected by a suitable (d) Statement I is false but Statement II is true.
detector? 14 All the accelerations as shown in figure are with respect
(a) 1 photon of 10.2 eV and an electron of energy 1.4 eV to ground, find acceleration of B.
(b) 2 photons of energy 10.2 eV
404 40 DAYS ~ JEE MAIN PHYSICS DAY THIRTY EIGHT

2 ms–2 18 A hole is bored along the diameter of the earth and a


particle is dropped into it. If R is the radius of the earth
and g is the acceleration due to gravity at the surface of
P the earth, then the time period of oscillation of the particle
is
R R 2R R
(a) 2 π (b) 2 π (c) 2 π (d) 2 π
A g 2g g 3g

3 ms–2
19 Five rods of same dimensions are arranged as shown in
B
the figure. They have thermal conductivities K1, K 2, K 3, K 4
and K 5. When the points A and B are maintained at
(a) 3 ms −2 , upward (b) 5 ms −2 , upward different temperatures, no heat flows through the central
(c) 3 ms −2 , downward (d) None of these rod if
15 Light is incident at an angle α on one planer end of a C
transparent cylindrical rod of refractive index n. The least
value of n for which the light entering the rod will not K1 K2
emerge from the curved surface of rod, irrespective of
value of α is A K5 B
1 1
(a) (b) 2 (c) (d) 3
2 3 K4
K3
16 A galvanometer has resistance100 Ω and it requires
current 100 µA for full scale deflection. A resistor 0 .1 Ω is D
connected to make it ammeter. The smallest current in
(a) K1 K 4 = K 2 K 3
circuit to produce the full scale deflection is
(b) K1 = K 4 and K 2 = K 3
(a) 1000.1 mA (b) 1.1 mA (c) 10.1 mA (d) 100.1 mA
K K
(c) 1 = 2
17 A rod AB of uniform cross-section consists of four section K4 K3
AC, CD, DE and EB of different metals with thermal
(d) K1K 2 = K 3K 4
conductivities K, (0.8) K, (1.2) K and (1.50) K,
respectively. Their lengths are respectively L, (1.2) L, 20 A body dropped from a height H reaches the ground
(1.5) L and (0.6) L. They are joined rigidly in succession with a speed of 1.2 gH . Calculate the work done by
at C, D and E to form the rod AB. The end A is air-friction.
maintained at 100°C and the end B is maintained at 0°C. (a) 2.8 mgH (b) –1.3 mgH
The steady state temperatures of the joints C, D and E (c) 1.3 mgH (d) − 0.28 mgH
are respectively TC , TD and TE . Column I lists the Section B : Numerical Type Questions
temperature differences (TA − TC ), (TC − TD ), (TD − TE ) and
21 A raft of wood of density 600 kgm −3 and mass 120 kg
(TE − TB ) in the four sections and column II their values
floats in water. How much weight (in kg) can be put on
jumbled up. Match each item in column I with its correct
the raft to make it just sink?
value in column II.
22 A ‘double star’ is a composite system of two stars rotating
A C D E B about their centre of mass under their mutual gravitational
attraction. Let us consider such a ‘double star’ which has
Column I Column II two stars of masses m and 2m at separation l. If T is the
A. (TA − TC ) 1. 9.6 time period of rotation about their centre of mass, is
B. (TC − TD ) 2. 30.1 l3
found to be 2π , then the value of x is… .
x Gm
C. (TD − TE ) 3. 24.1
D (TE − TB ) 4. 36.2 23 A SONAR system fixed in a submarine operates at a
A B C D A B C D frequency 40.0 kHz. An enemy submarine moves towards
(a) 3 4 2 1 (b) 1 2 4 3 the SONAR with a speed of 360 km/h. What is the
(c) 3 4 1 2 (d) 3 2 1 4 frequency of sound (in kHz) reflected by the submarine?
Take the speed of sound in water to be 1450 m/s.
DAY THIRTY EIGHT MOCK TEST 1 405

24 If electric potential due to some charge distribution is 27 A sky wave with a frequency 55 MHz is incident on D-region
given by V = 3 /r 2, where r is radial distance, then electric of earth’s atmosphere at 45°. Find the angle of refraction in
2 $ $ $ degree. (Electron density for D-regions is 400 electron/cm 3.)
field at (1, 1, 1) is found to be ( i + j + k ), then the value
n
28 A 20 g bullet pierces through plate of mass m1 = 1kg and
of n is …… .
then comes to rest inside a second plate of mass
25 Three dielectric slabs of thickness d/4 , d/ 7 and d/2 having m2 = 2.98 kg. It is found that the two plates, initially at rest,
dielectric constants 2, 8/7 and 4 respectively are inserted now move with equal velocities. Find the percentage loss
between the plates of a parallel plate capacitor having in the initial velocity of bullet when it is between m1 and m2
plate separation d and plate area A. If the net capacitance (neglect any loss of material of the bodies, due to action
p ε 0A of bullet).
of the system is equal to , then the value of p is …….
75 d 29 A block of wood has a mass of 25 g. When a 5 g metal
piece with a volume of 2 cm 3 is attached to the bottom of
26 An electron of hydrogen atom is considered to be
the block, the wood barely floats in water. What is the
revolving around the proton in the circular orbit of radius
volume V (in cm 3) of the wood?
h2 2 πe 2
2 2
with velocity . The equivalent current due to
4π me h 30 If the half-life of a radioactive substance is 40 days, then
x π 2me 5 find the duration in days in which it will decay 75% of its
circulating charge is found to be , then the value
h3 initial amount.
of x …… .

ANSWERS
1. (c) 2. (c) 3. (d) 4. (d) 5. (a) 6. (c) 7. (b) 8. (c) 9. (d) 10. (a)
11. (a) 12. (b) 13. (b) 14. (d) 15. (b) 16. (d) 17. (a) 18. (a) 19. (a) 20. (d)
21. (80) 22. (3) 23. (45.93) 24. (3) 25. (284) 26. (4) 27. (45) 28. (20) 29. (28) 30. (80)

Hints and Explanations


1 x2
1 (KE) man = (KE) boy 3 Maximum velocity = aω = a k 21 = 2 x +
2 m 100
⇒ mv2m =  ×
1 1 1 m 2 k1 k2 On solving, x = 10%
vb  Given that, a 1 = a2
2 2 2 2  m m [by cross-check method]
v
⇒ vm = b …(i) a1 k2 5 For thin prisms, angle of prisms A is
2 ⇒ =
Further, a2 k1 small.
m ( vm + 1 )2 =   v2b
1 1 m l 2 l For small A , Dmin (minimum deviation) is
4 Q T = 2π , T = 4 π2 ,
2 2 2  g g also small.
v
⇒ vm + 1 = b A + Dmin 
sin 
…(ii)
2  g  
l =  2  T2  2 
From Eqs. (i) and (ii), we get 4π  So, µ = …(i)
sin ( A / 2 )
vb = 2( 2 + 1 ) = 4.82 ms −1 xy
∴ % change = x + y +
vm = 2 + 1 = 2.41 ms −1 100  A + Dmin  ≈ A + Dmin
sin  
Vaild only for two variables in terms of  2  2
2 Effective length of the simple pendulum
percentage. (Qsinθ ≈ θ for small θ)
is (92.0 + 2.17) cm
x → % change in first variable
= 94.2 cm after rounding off to 3 A A
x → % change in second variable and sin ≈
significant digits. 2 2
x2
% increase in length = x + x +
100
406 40 DAYS ~ JEE MAIN PHYSICS DAY THIRTY EIGHT

Using above approximations, 8 I1 = e0


=
e0
, 3 = a AP − 2
A + Dmin  1 
2 Z1 ⇒ a AP = 5 ms −2
µ = ⇒ Dmin = (µ − 1 ) A R2 +  
2
 ω1C  ∴ a BG = a BP + a PG
A/2 = − 5 − 2 = − 7 ms −2
where, ω1 = 100 π
Hence, it can be seen that if A is small, e0 e 15 Q r + i = 90 °
I2 = = 0,
Dmin is also small. 2 Z2
 1  ⇒ i = 90 ° − r
6 Since, vertical displacement is same, R2 +  
 ω2C 
as well as initial velocity in vertical i
downward direction is zero for both the where, ω2 = 500 π r
So, Z1 > Z2 , therefore I1 < I2 . α
bodies. n
v 10 When photon strickes the hydrogen
atom, the photon is absorbed and H For ray not to emerge from curved
atom reaches in (n=2 state) or first surface,
h excited state, emitting a photon of i>C
energy 10.2 eV. Ionisation energy of ⇒ sin i > sin C
H-atom =13.6 eV, so the second photon ⇒ sin (90° − r ) > sin C
of energy 15 eV will ionise the H atom
⇒ cos r > sin C
and extra energy (15 -13.6)eV = 1.4 eV
1
g t 21
will be retained by the electron. Thus ⇒ 1 − sin 2 r >
So, h = (for horizontal finally we have one photon of energy n
2 10.2 eV and one electron of energy Qsin C = 1 
throwing) 1.4eV.  n 
gt 22 Q + Q2
h =
2
(for dropping) 11 At 1; E1 = σ 2 + σ 1 = 1 ⇒
1 − sin 2 i
>
1
2 ε0 2 ε0 2 A ε0 n2 n2
∴ t1 = t2 1
towards left ⇒ 1> (1 + sin 2 i )
7 ∴Path difference, σ σ Q − Q1 n2
At 2; E2 = 2 − 1 = 2
∆x = (SA + AP ) − SP 2 ε0 2 ε0 2 A ε0 ⇒ n 2 > 1 + sin 2 i ⇒ n > 2
= (65 + 65 ) − 120 [Q sin i = 1 for i = 90 ° ]
towards right
⇒ ∆x = 10 m
Q2 Q1 ∴ Least value is 2.
A where, σ 2 = and σ 1 =
A A ig S
16 =
12 First of all draw the equivalent circuit i S+G
65m 65m diagram, current flowing through circuit S+G
25m i = ⋅ ig
4.3 S
60m 60m = = 0.1 A
(50 || 200 + 2 +1) 0 . 1 + 100
= × 100 × 10 −6 A
S P 0.1
120m 4.3 V
= 100 . 1 × 10 −3 A = 100 . 1 mA
But at A, the wave suffers reflection at 50 Ω
the surface of rigid/fixed end or denser 17 A → 3; B → 4; C → 2; D → 1
medium. Hence, the wave must suffer
λ We have four sections, AB,BC , CD and
an additional path change of or a 2Ω 1Ω DE with (dQ / dt ) as the steady state
2 200 Ω thermal energy transmitted per second
phase change of π.
Voltmeter reading = 4.3 − 0.1 × 3 = 4 V ( A being the areas of cross-section)
λ
⇒ Net path difference =  10 −  dQ KA (100 − Tc )
 2 13 Since, in the rocket fuel is undergoing =
For maxima (constructive interference), dt L
combustion, the gases produced in this
λ A (0.8 ) K ( TC − T D )
Net path difference = (2 n ) ; process leave the body of the rocket =
2 with large velocity and produce (1.5 ) L
where, n = 0, 1, 2, . . . upthrust to the rocket. Let us assume
λ λ
⇒ 10 − = 2 n   ; that the fuel is undergoing combustion (1.2 ) KA ( T D − T E ) (1.5 ) KA T E
2 2 at the constant rate, then rate of change = =
(1.5 ) L (0.6 ) L
where, n = 0, 1, 2, . . . of momentum of the rocket will be
⇒ constant. Since, more and more fuel These given
λ will be burnt the mass of rocket will go
(100 − TC ) = 
10 = (2 n +1 ) ; where, n = 0, 1, 2, . . . 8.0 
2 on decreasing, so it will lead to increase  ( TC − T D )
 1.2 
⇒ the velocity of the rocket more and
= 
λ = 20 (2 n + 1 ) ; where, n = 0, 1, 2, . . . more rapidly. 1.2   1.5  T
 ( TD − TE ) =   E
14 Consider downward direction as  1.5   0.6 
20 20 positive
or λ = 20, , , ... 6 (100 − TC ) = 4 ( TC − T D )
3 5 a AP = − a BP
= (4 .8 )( T D − T E ) = 15 T E
a AG = a AP + a PG
DAY THIRTY EIGHT MOCK TEST 1 407

Solving for the differences 21 Volume of raft = 120 = 1 m3 Apparent frequency received by the
(100 − TC ), ( TC − T D ), ( T D − T E ) and 600 5 submarine,
Fraction of volume inside water is v + ve 
ν′ = 
TE
ρwood 600 3  νs
remaining that the sum of these = Relative density = =  v 
ρwater 1000 5
differences is 100, we obtain 1450 + 100 
So, fraction of volume outside water is =   × 40 × 10
3
( T A − TC ) = 24.1 ,  1450 
=  1 −  =
3 2
( TC − T D ) = 36.2  5 5 = 42.76 × 103 Hz
( T D − T E ) = 30.1 ⇒ Volume outside water is, Now, the reflected waves have a different
and ( T E − T B ) = 9.6 2 1
Vout = × =
2
m3 frequency,
GM ′ m 5 5 25  v 
18 F=− ν′′ =   ν′
x2 When the raft just sinks, the additional  v − vs 
upthrust is Here, vs = 100 m/s is velocity of enemy
G  π x3ρ m
4
2
3  U= × 103 × g submarine,
⇒F = − 25
x2  1450 
The weight m put on the raft is ν′′ =   × 42.76 × 10
3

= −  πGρm  x  1450 − 100 


4
2
3  mg = × 103 × g
25 = 45.93 × 103 Hz = 45.93 kHz
⇒ x ′ = −  π Gρ x
4
∴ m = 80 kg
3  24 As, V = 32
22 The system will revolve/rotate about an r
∂ 3 
∴ E = − 
axis passing through the centre of mass dV  6
 r =− r = 3 r
of the combined system. Considering  dr  ∂ r  r 2  r
m origin at the particle of mass 2m, we have
( i$ + $j + k$ ) 2 $ $ $
x the centre of mass at a distance l/3 from ⇒ E=6 = ( i + j + k)
M′ ( 3 )3 3
O 2l
2l and from m 2 $ $
3 = (i + j + k$ )
n
(given)
∴ n =3
Time period of oscillation,
x 3 CM 25 Here, three slabs are in series
T = 2π = 2π 2m m
| x ′| 4 πGρ
R
⇒ T = 2π
g 2 8 4
7
Qg = Gm = 4 πRρG 
 R2 3  l/3 2l/3
The gravitational force of attraction d d d
19 The arrangement of rods is analogous between 2m and m provides the 4 7 2
to the arrangement of resistances in a ε0 A
necessary centripetal force to the mass to C1 =
Wheatstone bridge balanced condition. 2l l
revolve in a circle of radius for m or  d /4 d / 7 d /2 
Thus, no heat flows through the rod  + + 
3 3  2 8/ 7 4 
conductivity K5 , then
for 2m.
K1 K2 ε0 A 8ε A
= = = 0
⇒ m   ω2 =
2l Gm (2 m )
K3 K4
3 d + d + d  3d
l2  
⇒ K1 K4 = K2 K3 8 8 8
3Gm l3 ε0 A 28 ε0 A
⇒ ω= ⇒ T = 2π C2 = =
20 The forces acting on the body are force l 3
3Gm   d d d   25 d
of gravity and air-friction d − + + 
On comparing with    
According to work-energy theorem,  4 7 2 
total work done on the body = Gain in l3 284 ε0 A
T = 2π , we get Now, C eq = C 1 + C 2 =
Kinetic energy x Gm 75 d
1 1
W = mv2 = m (1.2 gH ) 2 x =3 pε0 A
= (given)
2 2 75 d
23 SONAR frequency,
= 0.72 mgH
νs = 40 kHz = 40 × 103 Hz ∴ p = 284
As work done by gravity, 2 πr 2 π × h 2 h
W1 = mgH Speed of enemy submarine 26 As, T = = ×
5 v 4 π2 me 2 2 πe 2
∴ Work done by friction, ve = 360 km / h = 360 × m/s
W2 = W − W1 18 h3
=
= 0.72 mgH − mgH = 100 m / s Q 1 km / h = 5 m / s  4 π me 4 2
 
 18  e 4 π2 me 5
= −0.28 mgH ∴ Current, I = =
Speed of sound in water = 1450 m/s T h3
408 40 DAYS ~ JEE MAIN PHYSICS DAY THIRTY EIGHT

x π2 me 5 Solving this equation, we get t /T


=  
= N 1
(given) or …(i)
Percentage loss in N0  2 
3
h
∴ x =4 ( u − u1 )
u = × 100% = 20% N
u where, T is the half-life period and is
1 − 
80.5 N  N0
27 n eff = n 0 
 ν2  29 Let volume of wood is V cm 3 , then total fraction of atoms left after time t. Here,
N 25
80.5 × (400 × 10 ) 6 volume of displaced water is T = 40 days and =
= 1 1− =1
(55 × 106 )2 ( V + 2 ) cm 3 , then for translational N0 100
equilibrium, ( V + 2 ) ρg = (25 g + 5 g ) 1
sin i = = 0. 25
Also, n eff = ⇒ sin i = sin r 4
sin r Wood N
⇒ r = i = 45 ° Putting the values of T and in Eq. (i),
Water N0
28 The situation is as shown in figure. Metal we get
Firstly take first sheet and bullet as the 1 1
t /40
system, = 
where all the quantities are in CGS unit 4 2
mu = mu 1 + m 1 v 2 t /40
and ρ is the density of water. 1 = 1
m1 m2 or    
⇒ ( V + 2 ) × 1 = 30 2 2
m t
∴ V = 28 cm 3 or = 2 or t = 80 days
u u1 40
t /T
N = N0  
1
v v 30 Here,
2
Now, take second sheet and bullet as
the system,
mu 1 = ( m 1 + m 2 ) v
DAY THIRTY NINE MOCK TEST 2 409

DAY THIRTY NINE

Mock Test 2
(Based on Complete Syllabus)

Instructions
1. This question paper contains of 30 Questions of Physics, divided into two Sections :
Section A Objective Type Questions and Section B Numerical Type Questions.
2. Section A contains 20 Objective questions and all Questions are compulsory (Marking Scheme : Correct + 4, Incorrect –1) .
3. Section B contains 10 Numerical value questions out of which only 5 questions are to be attempted (Marking Scheme : Correct + 4, Incorrect 0) .

Section A : Objective Type Questions (a)


2. 29
× 1017 kgm−3 (b) 2. 29 × 79 × 1017 kgm−3
1 A particle projected with velocity v 0 strikes at right angles 79
2. 29
a plane passing through the point of projection and (c) 2. 29 × 1017 kgm−3 (d) × 1017 kgm−3
79
having inclination β with the horizontal. Find the height
(from horizontal plane) of the point, where the particle 5 If mass of a proton is 1.007825 amu and mass of a
strikes the plane neutron is 1.008665 amu, then mass of 3 Li 7 nucleus
approximately be
2v02 v 02
(a) y = (b) y = (a) 7.058075 amu (b) 7.000000 amu
g (4 + cot β)
2
g (4 + cot 2 β)
(c) 7.023475 amu (d) 7.034600 amu
v 02 v 02
(c) y = (d) y = 6 Two steel balls A and B are
(4 + cot 2 β) 2g (4 + cot 2 β)
placed inside a right circular P A
2 If the charge of 10 µC and −2 µC are given to two plates cylinder, of diameter 54 cm B R
of a capacitor, which are connected across a battery of making contacts at points P,Q and
12 V, find the capacitance of the capacitor. R as shown in the figure. The Q
(a) 0.33 µF . µF
(b) 05 (c) 0.41 µF (d) 0.66 µF radius rA = 12 cm and rB = 18 cm. 54 cm
3 A body when projected vertically up, covers a total The masses are mA = 15 kg and
distance s, during its time of flight. If we neglect gravity mB = 60 kg. The force exerted by the floor at the point Q
then how much distance the particle will travel during the and the wall at R are respectively (taking, g = 10 ms −2)
same time. Will it fall back? (a) 600 N,150 N
(a) s, Yes (b) s, No (c) 2s, Yes (d) 2s, No (b) 750 N, 150 N
(c) 600 N, 200 N
4 The density of hydrogen nucleus with Z =1 is (d) 750 N, 200 N
2.29 × 1017 kgm −3. The density of gold nucleus Z = 79
would be
410 40 DAYS ~ JEE MAIN PHYSICS DAY THIRTY NINE

7 A particular piano string is supposed to vibrate at a Region containing


frequency of 440 Hz. In order to check its frequency, a Magnetic field
tuning fork known to vibrate at a frequency of 440 Hz is
sounded at the same time the piano key is struck, and a
1 2 3 4
beat frequency of 4 beats/s is heard. Find the possible r2 r1
frequencies at which the string could be vibrating.
r4 r3
(a) 444 Hz, 436 Hz (b) 440 Hz, 436 Hz
(c) 444 Hz, 440 Hz (d) 449 Hz, 440 Hz
The table below gives the masses and charges of the ions
8 A uniform rectangular marble slab is 3.4 m long and
ION MASS CHARGE
2.0 m wide. It has a mass of 180 kg. If it is originally lying
A 2m +e
on the flat ground, how much work is needed to stand it
B 4m −e
on one end? C 2m −e
(a) 2.0 kJ (b) 3.0 J (c) 3.0 kJ (d) 3000 kJ D m +e
9 Statement I When the range of projectile is maximum, The ions fall at different positions 1,2,3 and 4 as shown.
the time of flight is the largest. Correctly match the ions with respective falling positions
Statement II Range is maximum, when angle of
Column I Column II
projection is 45°.
A 1
In the light of above statements, choose the most
B 2
appropriate answer from the options given below.
C 3
(a) Statement I is true but Statement II is false.
D 4
(b) Both Statement I and Statement II are true.
(c) Both Statement I and Statement II are false. A B C D A B C D
(d) Statement I is false but Statement II is true. (a) 4 3 2 1 (b) 1 2 3 4
(c) 4 1 2 3 (d) 3 4 1 2
10 The two blocks as shown in the figure, have equal
masses and µs = µk = 0.3 for both blocks. Wedge W is 13 A small conducting circular loop is placed inside a long
fixed and block A is given initial speed of 1 ms −1, down solenoid carrying a current. The plane of the loop
the plane. How far will it move before coming to rest, if contains the axis of the solenoid. If the current in the
inclines and strings are quite long? (take, g = 10 ms −2) solenoid is varied, the current induced in the loop is
(a) clockwise
B (b) anti-clockwise
(c) zero
s –1

A
1m

(d) clockwise or anti-clockwise depending on whether the


m current is increased or decreased
m
53° W 37° 14 Two wires, each having a weight per unit length of
(a) 0.45 m (b) 0.9 m 1.0 × 10−4 Nm −1, are strung parallel to one another above
(c) + 4.8 m (d) Question is irrelevant earth’s surface, one directly above the other. The wires
11 As shown in figure, a uniform solid sphere rolls on a are aligned in a North-South direction, so that earth’s
horizontal surface at 20 ms −1. It then rolls up the incline magnetic field will not affect them. When their distance of
shown. What will be the value of h, where the ball stops? separation is 0.10 m, what must be the current in each in
order for the lower wire to levitate the upper wire?
Assume that the wires carry the same currents, travelling
v = 20 ms–1
in opposite directions.
h
(a) 2.7 A (b) 0.1 A
30°
(c) 3.5 A (d) 7.1 A
(a) 28.6 m (b) 8.6 m (c) 6 m (d) 18.6 m
15 Calculate the minimum thickness of a soap-bubble film
12 A beam consisting of four types of ions a,b,c and d (n = 1.33) that will result in constructive interference in the
enters a region that contains a uniform magnetic field as reflected light, if the film is illuminated by light with a
shown in figure. The field is perpendicular to the plane of wavelength in free space of 602 nm.
the paper, but its precise direction is not given. All ions in (a) 98 nm (b) 113 nm
the beam travel with the same speed. (c) 125 nm (d) 25 nm
DAY THIRTY NINE MOCK TEST 2 411

16 A ground receiver station is receiving a signal at locate the final image formed of an object 30.0 cm in front
100 MHz, transmitted from a ground transmitter at a height of the first lens.
of 300 m located at a distance of 100 km. Then, 30.0 cm 20.0 cm
(Nmax = 1012 per m3 )
(a) signal is coming via space wave
(b) signal is coming via sky wave
(c) signal is coming via satellite transponder
(d) None of the above Object

17 The output waveform (Y ) of AND gate for the following


inputs A and B given below is
f1 = 10.0 cm f2 = 20.0 cm
t1 t2 t3 t5 t6 (a) 6.67 cm left (b) 6.67 cm right
(c) 15.0 cm left (d) 15.0 cm right
t4
20 Assertion In Young’s double slit experiment, the fringe
A width for dark fringes is same as that for white fringes.
inputs Reason In Young’s double slit experiment, when the
B fringes are performed with a source of white light, then
only dark and bright fringes are observed.
t1 t2 t3 t4 t5 t6 In the light of the above statements, choose the most
appropriate answer from the options given below.
(a) (a) Both A and R are correct and R is the correct
explanation of A.
t1 t2 t3 t4 t5 t6 (b) Both A and R are correct but R is not the correct
explanation of A.
(b) (c) A is correct but R is not correct
(d) A is not correct but R is correct

t1 t2 t3 t4 t5 t6 Section B : Numerical Type Questions


21 A man can swim with a speed of 4 Kmh−1 in still water.
(c) How long does he take to cross a river 1 km wide, if the
river flows steadily 3 Kmh−1 and he makes his strokes
t1 t2 t3 t4 t5 t6 normal to the river current. How far (in m) down the river
does he go when he reaches the other bank?
(d)
A D

β
18 Satellite dishes do not have to change directions in order
β
to stay focussed on a signal from a satellite. This means
that the satellite always has to be found at the same C O
B
location with respect to the surface of the earth. For this
to occur, the satellite must be at a height such that its 22 The equivalent thermal resistance of the combination of
revolution period is the same as that of earth, 24 h. At xl
rods shown below is , then find the value of x, if rod
what height must the satellite be so to achieve this? 12kA
1/ 3 has the same length l and cross-sectional area A.
 T2   T2 
(a)  GMe  (b)  GMe  Thermal conductivities are mentioned in figure. Assume
4π 4π
2 2
  that there is no heat loss due to radiation or convection.
1/ 2 1/ 3
 T2   T 
(c)  GMe  (d)  GMe 
4π
2
 4π
2
 1 k
3
4k
2
19 Two converging lenses are placed 20.0 cm apart, as 2k
shown in the figure. If the first lens has a focal length of T2
10.0 cm and the second has a focal length of 20.0 cm, T1
412 40 DAYS ~ JEE MAIN PHYSICS DAY THIRTY NINE

23 In J J Thomson’s experiment, a potential difference of 10.0 m. Use conservation of mechanical energy to find
320 V is accelerating the electron. The electron beam is the speed (in ms −1) of the sledge at the bottom of the hill,
entering a region having uniform magnetic field assuming the rider pushes off with an initial speed of
6 × 10−5 T, acting perpendicular to it. Find the value of 5.00 ms −1. Neglect air resistance,
electric field (in Vm −1) in this region, so that the electron
does not experience any deflection. 28 Water with a mass of 2.0 kg is held at constant volume in
(Take, me = 9.1 × 10−31 kg ) a container while 10.0 kJ of energy is slowly added by a
flame. The container is not well insulated, and as a result
24 A wire of length 100 cm is connected to a cell of emf 2V 2.0 kJ of energy leaks out to the surroundings. What is
and negligible internal resistance. The resistance of the the temperature (in °C) increase of water?
wire is 3 Ω. The additional resistance required to produce
a potential difference of 1 mV/cm is x Ω, then find the 29 A 20 g bullet is fired horizontally with a speed of 600 ms −1
value of x. into a 7 kg block on a table top. The bullet b lodges in the
block B. If the coefficient of kinetic friction between the
25 A 1.6 kg block on a horizontal surface is attached to a block and the table top is 0.4, what is the distance (in cm)
spring with a spring constant of 1. 0 × 103 Nm −1. The the block will slide?
spring is compressed to a distance of 2.0 cm, and the
block is released from rest. Calculate the speed (in ms −1) b B µ = 0.4
of the block as it passes through the equilibrium position,
x = 0, if the surface is frictionless.

26 An inductor coil joined to a 6 V battery draws a steady 30 A uniform rope, of mass m per unit length, hangs
current of 12 A. This coil is connected to a capacitor and vertically from a support, so that the lower end just
an AC source of rms voltage 6 V in series. If the current touches the table top. If it is released, then at the time a
in the circuit is in phase with the emf, then find the rms length y of the rope has fallen, the force on the table is
current (in A). equivalent to the weight of the length k y of the rope. Find
the value of k.
27 A sledge and its rider together weight 800 N. They move
down on a frictionless hill through a vertical distance of

ANSWERS
1. (a) 2. (b) 3. (d) 4. (c) 5. (a) 6. (d) 7. (a) 8. (c) 9. (d) 10. (a)
11. (a) 12. (c) 13. (c) 14. (d) 15. (b) 16. (c) 17. (b) 18. (a) 19. (a) 20. (c)
21. (750) 22. (7) 23. (637) 24. (57) 25. (0.5) 26. (12) 27. (14.9) 28. (0.96) 29. (37) 30. (3)
DAY THIRTY NINE MOCK TEST 2 413

Hints and Explanations


1 Let α be the angle between the velocity If there is no gravity, then s ′ = u × T the frequency of the string (the second
of projection and the inclined plane 2 u2 source) were either 444 Hz or 436 Hz.
= = 2s
v0 g 8 The work done by gravity is the work
Y′ If gravity is not there, it will never fall done, as if all the mass were
X′ back. concentrated at the centre of mass. The
work necessary to lift the object can be
4 Density of every nucleus is same thought of as the work done against
α = 2 . 29 × 1017 kgm −3 gravity and is just W = mgh , where h is
β the height through which the centre of
X′ mass is raised.
5 In 3 Li 7 , Z = 3 ; N = A − Z = 7 − 3 = 4
v0 x ′ = v0 cos α, v0 y ′ = v0 sin α W = 180 (9.8) (1.7) = 3.0 kJ
a x′ = −g sin β, a y ′ = −g cos β ∴ Mass of nucleus = Zmp + ( A − Z) m n
u 2 sin 2 θ
⇒ vx ′t = v0 cos α − g sin β t 9 The horizontal range, R =
= 3 × 1. 007825 + 4 × 1. 008665 g
At the point of impact vx′ = 0
= 7. 058075 amu 2 u sin θ
v cos α Time of flight, T =
⇒ t = 0 …(i) The actual mass of nucleus is slightly g
g sin β
less than this calculated value.
Also, y at this point is zero Range is maximum, when θ = 45 °
6 Free body diagrams of balls A and B,
⇒ v0 sin α t − 1 / 2g cos β t 2 = 0 So that, sin 2 θ = sin 90 ° = 1
2 v sin α u2
t2 = 0 …(ii)
NP R max =
g cos β g
NR Time of flight is maximum when θ = 90 °
From Eqs. (i) and (ii), we get 600
cot β So that, sin θ = sin 90 ° = 1
150
tan α = 2u
2 Tmax =
g
x = v0 cos (α + β ) t
 
2  10 Find acceleration,
 2  cot β  Na
2 ma = mg sin 53 ° − µ mg cos 53 °
v0   4 + cot 2 β 
2 
 NQ=750=0
= − mg sin 37 ° − µ mg cos 37 °
g  cot β

12
− ⋅
2  Then, use v2 = u 2 + 2as, v = 0,
 2 
cm
 4 + cot β 4 + cot β  u = 1 ms −1
2

v0
2
2 cot β 18 On solving, we get s = 0.45 m
= cm
g (4 + cot 2 β ) 11 The rotational and translational kinetic
2
q
v 2 cot β energy of the ball at the bottom will be
∴ y = y tan β = 0 ⋅ tan β 24cm
changed to gravitational potential energy,
g 4 + cot 2 β cos q = 24/30 = 4/5
2
when the sphere stops. We therefore
2 v0 Free body diagram of ball B, write
=
g(4 + cot 2 β ) N  Mv2 Iω2 
 +  = ( Mgh ) end
 2 2  start
2 Charge of capacitor is the charge on
facing surfaces of the plates of capacitor NR 2 Mr 2
600 For a solid sphere, I =
q − q 2  [10 − ( − 2 )]
Q =  1 = = 6 µC 5
 2  2 v
Also, ω = . Then, above equation
Potential difference across the capacitor r
= 12 V NQ = 750 becomes
Q  (6 × 10 −6 )  Nsin θ + 600 − 750 = 0 2
Mv2 +  Mr 2   v  = Mgh
So, C = = 1 1 2
V   F = 0.5 µF 3N  
12  = 150 2 2 5  r
5 1 2 1 2
3 Let particle is projected with speed u, so 150 × 5 or v + v = (9 . 8 ) h
⇒ N= = 250 2 5
total time of flight, 3
2 u  N cos θ − N R = 0 Using v = 20 ms–1, gives h = 28 . 6 m
T = 
 g  N R = 200 N 12 A → 4, B → 1, C → 2, D → 3
and s = 2 × maximum height 7 The number of beats per second is equal R = mv / qB
u2 u2 to the difference in frequency between
=2 × = RB > RA
2g g the two sound sources. In this case,
because one of the source frequencies is and R A = R C (in opposite sense)
440 Hz, 4 beats s −1 would be heard, if and R D is smallest.
414 40 DAYS ~ JEE MAIN PHYSICS DAY THIRTY NINE

18 The force that produces the centripetal B vr C

13 The angle between magnetic field and acceleration of the satellite is the
area vector is 90°, so the flux associated gravitational force, so
1 km v
M m m v2
vm β
with coil is zero. Although magnetic ∴ G e2 = …(i)
field is changing but flux is remaining r r
constant equal to zero, so emf induced where, Me is earth’s mass and r is the Time taken by the man to cross the river
and hence, current in the loop is equal satellite’s distance from the centre of Width of the river 1 km
to zero. t= =
the earth. Speed of the man 4 Kmh −1
14 If the upper wire is to float, it must be Also, we find the speed of the satellite 1 1
= h = × 60 = 15 min
in equilibrium under the action of two to be 4 4
forces : the force of gravity and d 2πr Distance travelled along the river = vr × t
magnetic repulsion. The weight per unit v= = …(ii)
T T 1 3
length here1 . 0 × 10 −4 Nm −1 must be = 3 × = km
where, T is the orbital period of the 4 4
equal and opposite the magnetic force 3000
per unit length. Because the currents
satellite. = = 750 m
4
are the same, we have Solving Eqs. (i) and (ii) simultaneously
F1 mg µ 0 I 2 for r yields, 22 R 1 = l , R 2 = l , R 3 = l
= = 1 /3 KA 2 KA 4 KA
I I 2 πd  T2 
r = G Me   R1R2 
(4 π × 10 −7 ) ( I 2 ) 4 π
2
 R eq =  + R3 
⇒ 1.0 × 10 −4 =  R1 + R2 
(2 π )(0 .10 )
19 First we make ray diagrams roughly to  l ⋅ l 
scale to see where the image from the  KA 2 KA l 
We solve for the current to find
first lens falls and how it acts as the R eq =  + 
I = 7.1 A object for the second lens. The location  l + l 4 KA 
15 Because 2 n t = λ , we have of the image formed by the first lens is  KA 2 KA 
2 found via the thin lens equation
1 1 1  l2 
λ 602 + = , v1 = + 15.0 cm  2 2 
t = = = 113 nm 30.0 v1 10.0 2 K A
= 
l
4 n (4 )(1.33 ) +
 2l + l 4 KA 
16 Maximum distance covered by space  
 2 KA 
wave communication
= 2 Rh l l 4 l+ 3 l 7l
F1 I2 I1 = + = =
O1 F1 F2 3 KA 4 KA 12 KA 12 KA
= 2 × 6 . 4 × 10 × 300 = 62 km
6
F2 xl
= (given)
Since, receiver-transmitter distance is 15.0 cm 12 KA
10.0 cm
100 km, this is ruled out for signal 6.67 cm ∴ x =7
frequency. 30.0 cm 20.0 cm
Lens 2 (as rods 1 and 2 are in parallel and
Lens 1
Further fc for ionospheric propagation is equivalent is in series with 3).
fc = 9 ( N max )1 /2 = 9 × (1012 )1 /2 The image formed by this lens becomes
2q v
the object for the second lens. Thus, the 23 E = ×B
= 9 MHz M
object distance for the second lens is
So, the signal of 100 MHz (7fc ) comes 20.0 cm − 15.0 cm = 5.00 cm . We again 2 × 1 . 60218 × 10 −19 × 320
= × 6 × 10 −5
via the satellite mode. apply the thin lens equation to find the 9 . 1 × 10 −31
17 For t ≤ t 1 ; A = 0, B = 0 ; Hence Y = 0 location of the final image.
1 1 1 = 112 . 680 × 1012 × 6 × 10 −5
For t 1 to t 2 ; A = 1, B = 0 ; Hence Y = 0 + = , v2 = − 6 . 67 cm
5 . 00 v2 20.0 = 10 . 61512 × 106 × 6 × 10 −5
For t 2 to t 3 ; A = 1, B = 1 ; Hence Y = 1
For t 3 to t 4 ; A = 0, B = 1 ; Hence Y = 0 Thus, the final image is 6.67 cm to the = 63 . 69072 × 101
left of the second lens. = 636 . 9072 Vm −1 ≈ 637 Vm −1
For t 4 to t 5 ; A = 0, B = 0 ; Hence Y = 0
For t 5 to t 6 ; A = 1, B = 0 ; Hence Y = 0 20 In Young’s double slit experiment fringe
24 Let a resistance R is connected in series
For t > t 6 ; A = 0, B = 1 ; Hence Y = 0 width for dark and white fringes are
same while in the same experiment, with the battery and wire.
Based on the above, the output 100 cm, 3 W
when a white light as source is used,
waveform for AND gate can be drawn as
the central fringe is white around while
given below.
few coloured fringes are observed on
t2 t3 either side. R
t1 t4 t5 t6
21 Given, speed of man ( vm ) = 4 kmh −1
Speed of river ( vr ) = 3 kmh −1 2V
Width of the river (d ) = 1 km Voltage drop across wire
= 1 × 10 −3 × 100 = 0.1 V
DAY THIRTY NINE MOCK TEST 2 415

Let current in circuit is I. 1 2 1 p2 ( mv) 2


or vi + g yi = v2f + g yf K= = ...(i)
2 2 2 2 (M + m) 2 (M + m)
∴ 0 .1 = l × 3 = 3 × ⇒ R = 57 Ω
R +3 If we set the origin of our coordinates at The friction force does work
the bottom of the incline, the initial and Wf = − fs = − µ k ( m + M ) g s ...(ii)
25 The initial elastic potential energy of final y-coordinates of the sledge are
the compressed spring is in stopping the block where s is the
yi = 10.0 m and yf = 0.
distance traversed by block-bullet system
1 Thus, we get on the table top.
PEs = kx i2 1 2 1
2 vi + g yi = v2f + 0 From work-energy theorem,
Because the block is always at the same 2 2 ∆K = Wf
height above earth’s surface, the v2f = vi2 + 2g yi 0 − K = − µk ( m + M) gs
gravitational potential energy of the = (5.00) 2 + 2 (9.80)(10.0) Substituting values in Eqs.(i) and (ii), we
system remains constant. Hence, the get
vf = 14 . 9 ms −1
initial potential energy stored in the ∴ s = 0.37 m = 37 cm
spring is converted to kinetic energy at 28 Recall that an isovolumetric process is Also the rest mass of photon is zero.
x = 0. i.e. one that takes place at constant volume.
1 1 30 The descending part of the rope is in free
k x2i = m v2f In such a process the work done is
2 2 equal to zero because there is no change fall. It has speed v = 2 gy at the instant
k in volume. Thus, the first law of all its points have descended a distance
Solving for vf gives, vf = xi
m thermodynamics gives y. The length of the rope which lands on
1.0 × 103 ∆U = Q the table during an interval dt following
= (2.0 × 10 −2 ) this instant is vdt. The increment of
1.6 This indicates that the net energy Q
momentum imparted to the table by this
= 0 .50 ms −1 added to the water goes into increasing
the internal energy of the water. The net length in coming to rest is m ( v dt ) v.
Thus, the rate at which momentum is
26 Resistance of coil = VDC = 6 energy added to the water is
I DC 12 transferred to the table is
Q = 10.0 − 2.0 = 8.0 kJ
In an AC circuit, the current is in phase Because Q = m c ∆T , the temperature y
with emf. This means that the net increase of the water is v
reactance of the circuit is zero. The Q 8.0 × 103
impedance is equal to the resistance, i.e. ∆T = = F
mc (2.0)(4.186 × 103 )
Z = 0.5 Ω
rms voltage = 0.96 ° C
Rms current =
Z 29 By conservation of momentum, the dp
= m v2 = (2 m y ) g
6 momentum of the block bullet system dt
= = 12 A
0.5 just after the interaction is p = mv. and this is the force arising from
where, m is the mass of bullet and v is stopping the downward fall of the rope.
27 The initial energy of the its velocity before striking the block. Since, a length of rope y of the weight
sledge-rider-earth system includes ( my ) g , already lies on the tabletop, the
kinetic energy because of the initial Hence, the kinetic energy of the system
just after the lodging of bullet into the total force on the tabletop is
speed (2 m y ) g + ( m y ) g = (3 m y ) g , or the
1 1 block is
m v2i + mg yi = m v2f + mg yf weight of a length 3y of rope.
2 2 So, k =3
416 40 DAYS ~ JEE MAIN PHYSICS DAY FOURTY

DAY FOURTY

Mock Test 3
(Based on Complete Syllabus)

Instructions
1. This question paper contains of 30 Questions of Physics, divided into two Sections :
Section A Objective Type Questions and Section B Numerical Type Questions.
2. Section A contains 20 Objective questions and all Questions are compulsory (Marking Scheme : Correct + 4, Incorrect –1) .
3. Section B contains 10 Numerical value questions out of which only 5 questions are to be attempted (Marking Scheme : Correct + 4, Incorrect 0) .

Section A : Objective Type Questions 3 A thin wire of length L and uniform linear mass density ρ is
1 An organ pipe of length L 0, open at both ends is bound bent into a circular loop with centre at O as shown. The
to vibrate in its first harmonic, when sounded with a moment of inertia of the loop about the axis XX ′ is
tuning fork of 480 Hz. What should be the length of a X X′
pipe closed at one end, so that it also vibrates in its first 90°
harmonic with the same tuning fork?
O
L0 L0 2L 0
(a) Lc = 2L 0 (b) Lc = (c) Lc = (d) Lc =
3 2 3
2 Choose the correct alternative. ρL3 ρL3
(a) (b)
(a) Gravitational potential at curvature centre of a thin 8π 2
16 π 2
hemispherical shell of radius R and mass M is equal to 5 ρL3 3 ρL3
GM /R (c) (d)
16 π 2
8 π2
(b) Gravitational field strength at a point lying on the axis of
a thin uniform circular ring of radius R and mass M is 4 If a drop of liquid breaks into smaller droplets, it results in
GM lowering of temperature of the droplets. Let a drop of
equal to 2 , where x is distance of that point
(R + x 2 ) 3 / 2 radius R, break into N small droplets each of radius r.
from centre of the ring Estimate the temperature in drop.
(c) Newton’s law of gravitation for gravitational force S  1 2S 1 − 1 
between two bodies is applicable only, when bodies (a) (b)
ρs  R  ρs  r R 
have spherically symmetric distribution of mass 3 S  1 1
(c) − (d)
2S  1 − 1
(d) None of the above
ρs  R r  ρs  R r 
DAY FOURTY MOCK TEST 3 417

5 The smiling face in figure consists of three items (i), A the axis is moved downwards, moment of inertia of the
thin rod of charge –3.0 µC that forms a full circle of radius slab will first decreases, then increases.
6 cm. (ii) second thin rod of charge 2.0 µC that forms a Reason (R) Axis is first moving towards its centre of mass,
circular arc of radius 4.0 cm, subtending an angle of 20° then it is receding from it.
about the centre of the full circle and (iii) electric dipole A
with a dipole moment that is ⊥ to a radial line and has a
magnitude 1. 28 × 10−21cm, what is the net electric
potential of the centre.
B C

In the light of the above statements, choose the most


appropriate answer from the options given below.
(a) Both A and R are correct and R is the correct
explanation of A.
(b) Both A and R are correct but R is not the correct
explanation of A.
(a) zero (b) 2 . 3 × 10−3 (c) A is correct but R is not correct.
(c) 1. 28 × 1021 (d) None of these (d) A is not correct but R is correct.

6 Pressure versus temperature graphs of an ideal gas are 9 A solid conducting sphere of radius a has a net positive
as shown in figure. Choose the incorrect statement. charge 2 Q. A conducting spherical shell of inner radius b
p p and outer radius c is concentric with the solid sphere and
has a net charge − Q.
The surface charge density on the inner and outer
surfaces of the spherical shell will be

T T
(i) (ii)
p
a
b

T 2Q Q Q Q
(a) − , (b) − ,
(iii) 4 πb 2
4 πc 2
4 πb 2
4 πc 2
(a) Density of gas is increasing in graph (i) Q
(c) 0, (d) None of these
(b) Density of gas is increasing in graph (ii) 4 πc 2
(c) Density of gas is constant in graph (iii)
(d) None of the above 10 Statement I Time period of oscillation of two magnets,
when like poles are in same direction (in a vibration
7 Assertion (A) A constant force F is applied on the two
magnetometer) is smaller, than the period of vibration
blocks and one spring system as shown in the figure.
when like poles are in opposite direction.
Velocity of centre of mass increases linearly with time.
Statement II Moment of inertia increases in same
F position.
m 2m
In the light of the above statements, choose the most
Smooth
appropriate answer from the options given below.
Reason (R) Acceleration of centre of mass is constant. (a) Statement I is true but Statement II is false.
(a) Both A and R are correct and R is the correct (b) Both Statement I and Statement II are true.
explanation of A. (c) Both Statement I and Statement II are false.
(b) Both A and R are correct but R is not the correct (d) Statement I is false but Statement II is true.
explanation of A.
11 In the circuit shown, the coil has inductance and
(c) A is correct but R is not correct.
resistance. When X is joined to Y , the time constant is τ
(d) A is not correct but R is correct.
during growth of current. When the steady state is
8 Assertion (A) There is a triangular plate as shown in the reached, heat is produced in the coil at a rate P. X is now
figure A dotted axis is lying in the plane of the slab. As joined to Z
418 40 DAYS ~ JEE MAIN PHYSICS DAY FOURTY

B B

D X¢ X D
(a) (b)
Y X A C A C X¢
X d d d d
Z
B B
(a) the total heat produced in the coil is P τ
1 X X¢ X A D X¢
(b) the total heat produced in the coil is P τ (c) (d)
2 A C D C
(c) the total heat produced in the coil is 2P τ d d d d
(d) the data given is not sufficient to reach a conclusion
12 Nickle shows ferromagnetic property at room 16 Two radioactive nuclei A and B have their disintegration
temperature. If the temperature is increased beyond constant λ A and λ B , respectively. Initially, NA and NB
curie temperature, then it will show number of nuclei are taken, then the time after which their
(a) anti-ferromagnetism undisintegrated nuclei are same is
(b) no magnetic property N  N 
λ A λB 1
(c) diamagnetism (a) ln  B  (b) ln  B 
(d) paramagnetism (λ A − λB )  N A  (λ A + λB )  N A 
1 N  1 N 
13 A glass prism ABC (refractive index 1.5), immersed in (c) ln  B  (d) ln  B 
water (refractive index 4/3). A ray of light is incident (λB − λ A )  N A  (λ A − λB )  N A 
normally on face AB. If it is totally reflected at face AC, 17 A student constructed a vernier callipers as shown in the
then figure. He used two identical inclines and tried to
measure the length of line PQ. For this instrument
determine the least count.
B A
θ

P
C
Q
l
8
(a) sinθ ≥
9 l (1 − cos θ)
(a) units
2 cos θ
(b) sinθ ≥ l
3 (b) units
cos θ
3
(c) sinθ = (c) l (1 − cos θ) units
2
2 8 1 − cos θ
(d) < sinθ < (d) units
3 9 l

14 In a given process of an ideal gas, dW = 0 and dQ < 0. 18 A double star consists of two stars having masses
Then, for the gas M and 2 M . The distance between their centres is equal to
(a) the temperature will decrease r. They revolve under their mutual gravitational interaction.
(b) the volume will increase Then, which of the following statement(s) is/are correct?
(c) the pressure will remain constant (a) Heavier star revolves in orbit of radius 2r/3
(d) the temperature will increase (b) Both of the stars revolve with the same period which is

15 Two long parallel wires are at a distance 2d apart. They equal to r 3/ 2
2 GM /3
carry steady equal currents flowing out of the plane of
the paper, as shown. The variation of the magnetic field (c) Kinetic energy of heavier star is twice that of the other
B along the line XX ′ is given by star
(d) None of the above
DAY FOURTY MOCK TEST 3 419

19 To plot forward characteristic of p-n junction diode, the heard distinctly, when the rate of beating of the drum is
correct circuit diagram is 1 per sec. What is the original distance (in m) of the man
from the mountain?

(0-1000) mA
24 A metal wire of linear mass density of 9.8 gm −1 is
(0- 20) A

(a) E V+

(0-1)V A+– (b) E + +
V– (0-2)V A – stretched with a tension of 10 kg-wt between two rigid
supports which are 1m apart. The wire passes through
the middle points between the poles of a permanent
magnet and it vibrates in resonance, when carrying on
(0-1000) mA

(0-1000) mA
alternating current of frequency n. The frequency n of the
+ – +
(c) E V– (0-2)V A+ (d) E V– (0-2)V A–+ alternating current (in Hz) is …… .

25 The circuit shown in figure, contains a resistance of


R = 6 Ω connected with a battery of emf 6 V.
20 For atomic model of hydrogen atom given by Niels Bohr,
match the following proportionalities. R
I
Column I Column II
A. Angular momentum 1. 1/ n
6V
B. Velocity of electron 2. n2
Given, n = number of electrons per volume = 1029 / m 3,
C. Radius of electron 3. 1 / n2 length of circuit = 10 cm, cross-section A = 1 mm 2. The
D. Energy of electron 4. n energy absorbed by electrons from initial state of no
current (ignore thermal motion) to the state of drift
A B C D velocity is found to be x × 10−17 J, then the value of x is
(a) 1 2 3 4 …… .
(b) 4 3 2 1
(c) 4 1 3 2 26 The potential energy of a particle of mass m is given by
(d) 4 1 2 3 E 0 0 ≤ x ≤1
U(x) = 
Section B : Numerical Type Questions 0 x >1
21 A hill is 500 m high. Supplies are to be sent across the
λ 1 and λ 2 are the de-Broglie wavelengths of the particle,
hill using a canon that can hurl packets at a speed of 125
when 0 ≤ x ≤ 1 and x > 1, respectively. If the total energy of
ms −1 over the hill. The canon is located at a distance of
λ x
800 m from the foot of hill and can be moved on the particle is 2 E 0. If the value of the ratio 1 is , then the
ground at a speed of 2 ms −1, so that its distance from the λ2 1
hill can be adjusted. What is the shortest time (in s) in value of x is …… .
which a packet can reach on the ground across the hill?
27 A cubical block of side L rests on a rough horizontal
(Take, g = 10 ms −2)
surface with coefficient of friction µ. A horizontal force F is
22 A motor cyclist starts from the bottom of a slope of angle applied on the block as shown in the figure. If the
45° and travels along the slope to jump clear of the valley coefficient of friction is sufficiently high, so that the block
AB as shown in figure. The width of the valley is 160 m does not slide before toppling, the minimum force
and the length of the slope is160 2 m. The minimum required to topple the block is found to be nmg, then the
velocity with which he should leave the bottom O, so that value of n is …… .
he can clear the valley, is (nearest to in ms −1)
F
A B L
45° µ
O
28 The fundamental frequency of a sonometer wire of length
l is f0. A bridge is now introduced at a distance of ∆l from
23 A man beats a drum at a certain distance from a the centre of the wire ( ∆l << l ′ ). The number of beats
mountain. He slowly increase the rate of beating and heard, if both sides of the bridges are set into vibration in
finds that the echo is not heard distinctly, when the drum bf ∆l
their fundamental modes, are found to be 0 , then find
beating is at the rate of 40 per min. He moves by 80 m l
towards the mountain and finds that the echo is again not the value of b.
420 40 DAYS ~ JEE MAIN PHYSICS DAY FOURTY

29 A mason is supplied with bricks by his assistant who is block then moves on similar smooth horizontal surface
3 m below him, the assistant is tossing the brick vertically with a velocity v. then the velocity v in terms of v 0 is found
up. The speed of the brick, when it reaches the mason is 2v
to be 0 , then find the value of m.
2 ms −1. What percentage of energy used up by the m
servant serves no useful purpose ? m vo m v

30 A block of mass m moves with a velocity vo on a smooth O


horizontal surface. If that passes over a cylinder of radius R
R and mass m, capable of rotating about its own fixed m
axis through O the block, while passing over, slips on the
cylinder. The slipping stops before, it loses contact. The

ANSWERS
1. (c) 2. (c) 3. (d) 4. (c) 5. (a) 6. (a) 7. (a) 8. (a) 9. (a) 10. (a)
11. (b) 12. (d) 13. (a) 14. (a) 15. (b) 16. (c) 17. (a) 18. (b) 19. (b) 20. (d)
21. (45) 22. (70) 23. (240) 24. (50) 25. (2) 26. (2) 27. (0.5) 28. (8) 29. (5.6) 30. (3)
DAY FOURTY MOCK TEST 3 421

Hints and Explanations


1 The fundamental frequency for an open 4 4 F
πR 3 = N × πr 3 ∴ a CM =
ν 3 3 3m
pipe, ν0 =
2L0 R3 = constant as F is constant
or R 3 = Nr 3 or N =
r3 ∴ vCM = a CM × t
Now, change in surface area F
or vCM = × t or vCM ∝ t
Lc = 4 πR 2 − N4 πr 2 3m
= 4 π ( R 2 − Nr 2 ) Hence, both Assertion and Reason are
L0 correct and the Reason is the correct
Energy released = S × ∆ A explanation of Assertion.
Closed organ pipe (where, S = surface tension)
= S × 4 π( R 2 − Nr 2 ) 8 Moment of inertia ( I ) = mr2 , where r is
Due to release of this energy, the distance from the axis of rotation to the
temperature is lowered. centre of mass. When dotted axis moved
Open organ pipe downward (towards centre of mass), r
If ρ is the density and s is specific heat
Fundamental frequency for closed pipe, decreases result moment of inertia
ν of liquid and its temperature is lowered
ν0′ = decrease and when dotted axis cross the
by ∆θ then,
4Lc centre of mass and moved further
Energy released = ms∆θ
downwards, then r increases result

S × 4 π ( R 2 − N r 2 ) =  × R 3 × ρ s∆θ
According to the question,
moment of inertia increases.
ν0 = ν0′  3  Hence, both Assertion and Reason are
ν ν L
= ⇒ Lc = 0 S × 4 π ( R 2 − Nr 2 ) correct and the Reason is the correct
2 L0 4 L0 2 ∆θ =
4 explanation of Assertion.
πR 3ρ × s
2 Because every element of hemispherical 3 9 Due to induction, inner surface of
shell is at a distance R from centre of 3 S  R2 Nr 2  spherical shell has charge − 2 Q,
=  − 3 
curvature, therefore gravitational ρs  R 3
R 
GM
potential at its centre = − , 3S  1 ( R 3 / r3 ) × r2 
R =  − 
–2Q
ρs  R R3 
i.e. option (a) is incorrect. a
3S  1
− 
1 +2Q b
Gravitational field strength at a point, =
ρs  R r 
lying on the axis of a thin uniform c
circular ring of radius R is 2
GMx 5 The dipole potential (θ = 90 ° ) is given
( R + x2 )3 /2 by the equation,
p cos θ p cos 90 ° So, surface charge density on inner side
So, option (b) is incorrect. V= = =0
4 πεο r 2 4 πεo r 2 −2 Q
Newton’s law of gravitation is σ inner =
applicable to only those bodies which [Q cos 90 ° = 0 ] 4 πb2
have spherically symmetric distribution Also, the potential due to the short arc and surface charge density on outer side
Q
of mass. So, option (c) is correct. is q 1 / 4 πε0 r1 and that caused by the σ outer =
long arc is q 2 / 4 πε0 r 2 . 4 πc 2
3 Mass of the ring, M = ρL
Since, q 1 = + 2 µC, r1 = 4 cm, 10 Case I When the like poles of two
Let R be the radius of the ring.
q 2 = −3 µC and r2 = 6 cm, the potential magnets are placed in same direction,
L
Then, L = 2 πR or R = of the arcs cancel. then the time period of vibration is
2π expressed as
Thus, the result is zero.
Moment of inertia about X X′ (from I1 + I2
pM
parallel axis theorem) will be given by 6 As, ρ = T′ = 2 π …(i)
RT ( M1 + M2 ) B
1 3
I XX′ = MR 2 + MR 2 = MR 2 Density ρ remains constant, when p / T
2 2 Case II When the like poles of two
or volume remains constant. In graph (i)
Putting values of M and R , magnets are placed in opposite direction,
volume is decreasing, hence density is
 L2  3 ρL3 then period of vibration is expressed as
3 increasing; while in graphs (ii) and (iii)
I XX′ = ( ρL )  2  =
2  4 π  8 π2 temperature is increasing, hence, I1 + I2
T ′′ = 2 π …(ii)
density is decreasing. Note that, volume ( M1 − M2 ) B
4 When a big drop of radius R, break into would have been constant in case the
straight line in graph (iii) had passed It is clear from Eqs. (i) and (ii) that,
N droplets each of radius r, the volume
through origin. T ′ < T ′ ′.
remains constant.
∴ Volume of big drop 7 Total mass of the system 11 As, P = ( I0 ) 2 ⋅ R
= N × volume of small drop = m + 2m = 3m P
i.e. ( I0 ) 2 =
Force applied on the system is F. R
422 40 DAYS ~ JEE MAIN PHYSICS DAY FOURTY

LI02 = ( τR )   = P τ me 4
1 1 P 1 18 The centre of mass of the double star
∴ U= Potential energy i. e . , En = −
2 2 R 2 system remains stationary and both the 8 n 2 ε20 h 2
stars revolve round in circular orbits,
12 The curie temperature is defined as the which are concentric with the centre of ∴ En ∝ 1 / n 2
temperature beyond which the mass.
ferromagnetic material shows 21 Given, height of the hill (h) = 500 m
The distance of centre of mass from the
paramagnetic behaviour. Mr + 2 M × 0 r Velocity of canon, u = 125 ms −1
heavier star = =
13 For total internal reflection at AC-face M + 2M 3 To cross the hill, the vertical component
µ of the velocity should be sufficient to
sin i ≥ w Hence, the heavier star revolves in a
µg r cross such height.
circle of radius while the lighter star
4 3 ∴ u y ≥ 2gh ≥ 2 × 10 × 500
sin θ ≥ 2r
3 × 1.5 in a circle of radius . ≥ 100 ms −1
8 3
sin θ ≥ M ⋅2 M But u =2
u 2x + u 2y
9 Reduced mass of the system =
M + 2M ∴ Horizontal component of initial
14 From first law of thermodynamics 2M velocity,
dQ = dU + dW =
3 ux = u 2 − u 2y = (125 )2 − (100 )2
dQ = dU (QdW = 0)
Since, dQ < 0
Period of revolution of the double star = 75 ms −1

Therefore, dU < 0 system = r 3 /2 Time taken to reach the top of the hill,
2 GM
or Ufinal < Uinitial 2h 2 × 500
3 t = = = 10 s
or temperature will decrease. g 10
where, r is the distance between two
15 If the current flows out of the paper, the stars. Time taken to reach the ground from the
magnetic field at points to the right of 1 top of the hill,
KE of a star = mv2 t ′ = t = 10 s
the wire will be upwards and to the left 2
will be downward. Now, magnetic field Horizontal distance travelled in 10 s,
KE of heavier star
at C is zero. The field in the region DX′ 2 x = ux × t
× 2 M ×  ω 
1 r = 75 × 10 = 750 m
will be upwards (+ve), because all E1 =
points existing in this region are to the 2 3  ∴ Distance through which canon has to
right of both the wires. Similarly, and that of lighter star be removed = 800 − 750 = 50 m
magnetic field in the region AX will be 2 Speed with which canon can move
M 
1 2r 
downwards (–ve). The field in the E2 = ω = 2 ms −1
region AC will be upwards (+ve), 2  3 
50
because points are closer to A as So, kinetic energy of lighter star is two ∴ Time taken by canon =
compared to D. Similarly, magnetic field 2
times that of heavier star. t ′′ = 25 s
in region DC will be downward (–ve).
19 For forward bias mode the p-side of Hence, total time taken by a packet to
16 After disintegration, diode has to be at higher potential than reach on the ground = t ′′ + t + t ′
N A e − λ A t = N B e − λ B t (for n-side. The meters used are DC, so we = 25 + 10 + 10
0 ≤ x ≤ 1) have to be careful while connecting = 45 s
NB them w.r.t. polarity.
or e ( λ B −λA ) t
= …(i) 22 Velocity to take-off from A to clear the
NA Last point is to decide the range of
meters, the range of meters has to be in valley is given by
N  u2
∴ ( λ B − λ A ) t = In  B  such a way that we can have the R = sin 2 α
 NA  readings which leads to plot on realistic g
1 N  scale. If we take 0-20 A ammeter, then
α = 45 ° , u = gR = 40 ms −1
⇒ t = In  B 
λB − λA  NA  reading we read from this is tending to 0
to 5 divisions which is not fruitful. Velocity to start from lowest point (due
17 Let θ be the angle of incline. Here, the to retardation on inclined plane, g sin α),
20 A → 4, B → 1, C → 2, D → 3 v20 = u 2 + 2 g sin α × s
incline kept horizontally is working as
main scale while the other incline kept As we know, angular momentum 1
on horizontally placed incline is treated nh (40 ) 2 + 2 × 10 ×
( L) = ⇒L ∝ n ∴ v0 = 2
as vernier scale. 2π
From the figure, it is clear that, 2 × 160 × 2
n2  h 
Radius of an electron = rn =  
1 MSD =
l
unit and 1 VSD = l unit m 2π = 4800 ~ 70 ms −1
cos θ
4 πε0 23 The echo is not heard distinctly, when
So, LC of instrument is, ⇒ rn ∝ n 2
e2 the echo and the next beat fall on the ear
LC = 1MSD − 1 VSD simultaneously, i.e. time per beat = time
Velocity of electrons
 l  l (1 − cos θ)
= − l = units 1 e2 1 taken by the reflected beat to reach the
 cos θ  cos θ vn = ⋅ man.
n 4 πε0 ( h / 2 π ) 2 d 60 3
Hence, = =
vn ∝ 1 / n v 40 2
DAY FOURTY MOCK TEST 3 423

2 (d − 80 ) From Eqs. (i) and (ii), we get used to describe the motion. The initial
and =1
v velocity v0 is found by putting known
This gives, d = 240 m λ1 values in the above equation,
= 2 = x (given)
λ2 v20 = 36 + 2 × 32 × 10 = 676
24 Since, the tension,
T = 10 kg-wt = 10 × 9.8 = 98 N ∴ x =2 ⇒ v0 = 26 fts −1
and m = 9.8 × 10 −3 kgm −1 , L = 1m The kinetic energy given to each brick
27 At the critical condition, normal and supplied by the assistant is
So, we get reaction N will pass through point P. 1
1 T 1 98 E1 = mv02
n = = × N 2
2L m 2 × 1 9.8 × 10 −3 1
= × m × 676
= 50 Hz F 2
25 Given, V = 6 V, R = 6 Ω , G = 338 m ft 2s −1
L
−6
L/2 If the brick assistant supplied is only just
A = 1 × 10 m 2
f enough energy to reach the required level
and l = 10 cm = 0.1 m P
and no more, the initial velocity being u,
The current in the circuit, mg they would have zero velocity at the
V 6
I = = = 1A The block will topple when Mason’s hand.
R 6 ∴ u 2 = 0 + 2 g ( x − x0 )
L
Use the relation I = ne A vd τ F > τmg or FL > ( mg )
2 = 2 × 32 × 10 = 640
Drift velocity of electrons,
mg ⇒ u = 8 10 fts −1
I ∴ F>
vd = 2
neA KE supplied in this case,
1 Therefore, the minimum force required
= 29 mg E2 =
1
mu 2 = 320 m ft 2s −2
10 × 1.6 × 10 −19 × 1 × 10 −6 to topple the block is F = .
2 2
1
= × 10 −4 ms −1 = 0.5 mg ∴ Wasted energy = E1 − E2
1.6 = n mg (given)
1 E1 − E2
The energy of electrons, (KE) = mv2 ∴ n = 0.5 % waste = × 100
2 E1
1
= × m e × vd2 × volume × number of
28 As, f0 = v 338 − 320
2l = × 100
2 320
electrons per volume Beat frequency = f1 − f2 = 5.6%
2 v v
1  10 −4  = −
= × 9.1 × 10 −31 ×  ×A ×I ×n 30 Using conservation of angular
2  − ∆l 2
  l + ∆l 
l
2  1.6    momentum about the axis of cylinder for
2  2 
the (block + cylinder) system
 2 2 
(QMass of electron m e = 9.1 × 10 −31 ) = ( f0 l)  −  MR 2ω
mvo R = mvR +
 l − 2 ∆l l + 2 ∆l  2
9.1 × 10 −39 4 ∆l 8 f ∆l
= × 10 −6 × 0.1 × 1029 = 2 f0 l  2  ≈ 0 3
2 × 1.6 × 1.6  l  ⇒ mvo R = mvR
l 2
= 2 × 10 −17 J bf0 ∆ l (Q v = ωR )
= (given)
∴x =2 l (when slipping stops)
∴ b=8 ⇒ v=
2 vo
26 KE = 2 E0 − E0 = E0 (for 0 ≤ x ≤ 1 )
3
h 29 Once the bricks leave the assistant’s
So, λ1 = …(i) hands the only force that acts on them =
2 v0
(given)
2 mE0 3
is gravitational force. Since this
Again KE = 2 E0 (for x > 1 ) produces a constant acceleration ∴ m =3
∴ λ2 =
h
…(ii) a = − g = − 32 fts −2 , the kinematic
2 m 2 E0 equation, v2 = v20 − 2 a ( x − x0 ), can be
EXAM BITES

This Pdf Is
Downloaded From
www.exambites.in

Visit www.exambites.in for


More Premium Stuffs,Latest
Books,Test Papers,Lectures etc.
jeeneetadda
jeeneetadda_official
jeeneetadda

VISIT NOW !!
February Attempt Online JEE Main 2021 1

JEE Main 2021


(February, March, July & August Attempt)

Instructions
This question paper contains of 30 Questions of Physics, divided into two Sections : Section A Objective Type
Questions and Section B Numerical Type Questions.
Section A contains 20 questions and all Questions are compulsory (Marking Scheme : Correct + 4, Incorrect –1) .
Section B contains 10 questions out of which only 5 questions are to be attempted
(Marking Scheme : Correct + 4, Incorrect 0) .

24 FEBRUARY SHIFT I
Section A : Objective Type Questions 5. Consider two satellites S1 and S 2 with periods of
1. The work done by a gas molecule in an isolated system is revolution 1 h and 8 h respectively, revolving around a
− x
2 planet in circular orbits. The ratio of angular velocity of
given by, W = αβ 2e αKT , where x is the displacement, k is satellite S1 to the angular velocity of satellite S 2 is
the Boltzmann constant and T is the temperature, α and a. 8 : 1 b. 1 : 8
β are constants. c. 2 : 1 d. 1 : 4
Then, the dimensions of β will be 6. Each side of a box made of metal sheet in cubic shape is
2
a. [M L T ] 2 0
b. [M L T ] 0 a at room temperature T , the coefficient of linear
expansion of the metal sheet is α. The metal sheet is
c. [M L T − 2 ] d. [M L 2 T − 2 ] heated uniformly, by a small temperature ∆T , so that its
2. Two stars of masses m and 2m at a distance d rotate new temperature is T + ∆T . Calculate the increase in the
about their common centre of mass in free space. The volume of the metal box.
period of revolution is a. 4 πa 3α∆T b. 4a 3α∆T
1 3Gm d3 4
a. b. 2 π c. πa 3α∆T d. 3a 3α∆T
2π d3 3Gm 3
3Gm 1 d3 7. If Y, K and η are the values of Young’s modulus, bulk
c. 2 π d.
d 3
2π 3Gm modulus and modulus of rigidity of any material,
respectively. Choose the correct relation for these
3. Four identical particles of equal masses 1 kg made to parameters.
move along the circumference of a circle of radius 1 m
9Kη 9Kη
under the action of their own mutual gravitational a. Y = N/m 2 b. Y = N/m 2
attraction. The speed of each particle will be 2η + 3K 3K − η
Yη 3YK
(1 + 2 2 )G G c. K = N/m 2 d. η = N/m 2
a. b. (1 + 2 2 ) 9η − 3Y 9K + Y
2 2
c. G(1 + 2 2 ) d.
G
(2 2 − 1) 8. If the velocity-time graph has the shape AMB, what would
2 be the shape of the corresponding acceleration-time
4. Moment of inertia (MI) of four bodies, having same mass graph ?
and radius, are reported as Velocity (v)
I1 = MI of thin circular ring about its diameter,
I2 = MI of circular disk about an axis perpendicular to the A B
disk and going through the centre,
I3 = MI of solid cylinder about its axis Time (t)
and I4 = MI of solid sphere about its diameter. Then,
M
5
a. I1 + I 2 = I 3 + I4 b. I1 + I 3 < I 2 + I 4
2
c. I1 = I 2 = I 3 < I 4 d. I1 = I 2 = I 3 > I 4
2 Online JEE Main 2021 February Attempt

a a

a. t c. t m

k
a a
M
b.
d. t
t

9. n mole of a perfect gas undergoes a cyclic process ABCA M+ m M M− m M


a. A b. A c. A d. A
(see figure) consisting of the following processes. M M+ m M M−m
A → B : Isothermal expansion at temperature T , so that
the volume is doubled from V1 to V2 = 2V1 and pressure
12. A cell E 1 of emf 6 V and internal resistance 2 Ω is
changes from p1 to p2. connected with another cell E 2 of emf 4 V and internal
resistance 8Ω (as shown in the figure). The potential
B → C : Isobaric compression at pressure p2 to initial difference across points X and Y is
volume V1. E1 E2
P X Y
C → A : Isochoric change leading to change of pressure
from p2 to p1. 6 V, 2 Ω 4 V, 8 Ω

Total work done in the complete cycle ABCA is


p

A a. 2.0 V b. 3.6 V
p1
c. 5.6 V d. 10.0 V
13. A current through a wire depends on time as
i = α 0t + βt 2, where α 0 = 20 A/s and
p2
C
B
β = 8 As − 2. Find the charge crossed through a section of
the wire in 15 s.
a. 260 C b. 2100 C
V c. 11250 C d. 2250 C
V1 V2=2V1
14. Two equal capacitors are first connected in series and
a. 0 b. nRT ln 2 then in parallel. The ratio of the equivalent capacities in
c. nRT  ln 2 +  d. nRT  ln 2 − 
1 1 the two cases will be
 2  2 a. 1 : 2 b. 2 : 1 c. 4 : 1 d. 1 : 4
10. Match List-I with List-II. 15. A cube of side a has point charges
+ Q located at each of its vertices except at the origin,
List-I List-II
where the charge is − Q. The electric field at the centre of
A. Isothermal 1. Pressure constant cube is
x
B. Isochoric 2. Temperature constant
+Q +Q
C. Adiabatic 3. Volume constant

D. Isobaric 4. Heat content is constant +Q +Q

Choose the correct answer from the options given below. +Q


A B C D z
–Q
(a) 1 3 2 4 +Q +Q
a
(b) 3 2 1 4
(c) 2 4 3 1 y
(d) 2 3 4 1
−Q $ + y$ + z$ ) Q $ + y$ + z$ )
a. (x b. (x
11. In the given figure, a mass M is attached to a horizontal 3 3 πε0a 2 3 3 πε0a 2
spring which is fixed on one side to a rigid support. The −2Q 2Q
c. $ + y$ + z$ )
(x d. (x$ + y$ + z$ )
spring constant of the spring is k. The mass oscillates on
3 3 πε0a 2 3 3 πε0a 2
a frictionless surface with time period T and amplitude A.
When the mass is in equilibrium position as shown in the 16. If an emitter current is changed by 4 mA, the collector
figure, another mass m is gently fixed upon it. The new current changes by 3.5 mA. The value of β will be
amplitude of oscillation will be a. 7 b. 0.875 c. 0.5 d. 3.5
February Attempt Online JEE Main 2021 3

17. In the given figure, the energy levels of hydrogen atom 22. An unpolarised light beam is incident on the polariser of a
have been shown alongwith some transitions marked polarisation experiment and the intensity of light beam
A, B, C, D and E. The transitions A, B and C respectively emerging from the analyser is measured as 100 lumens.
represent Now, if the analyser is rotated around the horizontal axis
+eV (direction of light) by 30° in clockwise direction, the
Continuum intensity of emerging light will be ....... lumens.
0 eV
n=5 –0.54 eV 23. A ball with a speed of 9 m/s collides with another identical
n=4 – 0.85 eV ball at rest. After the collision, the direction of each ball
n=3 –1.51 eV makes an angle of 30° with the original direction. The ratio
B C D of velocities of the balls after collision is x : y , where x is
n=2 –3.4 eV ........... .
A E
24. A hydraulic press can lift 100 kg when a mass m is placed
n=1 –13.6 eV on the smaller piston. It can lift ...... kg when the diameter
a. The first member of the Lyman series, third member of of the larger piston is increased by 4 times and that of the
Balmer series and second member of Paschen series. smaller piston is decreased by 4 times keeping the same
b. The ionisation potential of hydrogen, second member of mass m on the smaller piston.
Balmer series and third member of Paschen series. 25. An inclined plane is bent in such a way that the vertical
c. The series limit of Lyman series, second member of x2
Balmer series and second member of Paschen series. cross-section is given by y = where, y is in vertical and x
4
d. The series limit of Lyman series, third member of Balmer
in horizontal direction. If the upper surface of this curved
series and second member of Paschen series.
plane is rough with coefficient of friction µ = 0.5, the
18. Given below are two statements : maximum height in cm at which a stationary block will not
Statement I Two photons having equal linear slip downward is .......... cm.
momenta have equal wavelengths. 26. A resonance circuit having inductance and resistance
Statement II If the wavelength of photon is decreased, 2 × 10− 4 H and 6.28 Ω respectively oscillates at 10 MHz
then the momentum and energy of a photon will also frequency. The value of quality factor of this resonator is
decrease. .......... .
In the light of the above statements, choose the correct [Take, π = 3.14]
answer from the options given below. 27. An audio signal v m = 20 sin 2π(1500 t) amplitude modulates
a. Both Statement I and Statement II are true.
a carrier v c = 80 sin 2π(100000 t)
b. Both Statement I and Statement II are false.
c. Statement I is true but Statement II is false. The value of per cent modulation is ......... .
d. Statement I is false but Statement II is true. 28. In connection with the circuit drawn below, the value of
19. The focal length f is related to the radius of curvature r current flowing through 2kΩ resistor is ....... × 10− 4 A.
of the spherical convex mirror by
r r 1kΩ
a. f = r b. f = − r c. f = − d. f = +
2 2 2 kΩ 5V
20. In a Young’s double slit experiment, the width of the
one of the slit is three times the other slit. The
amplitude of the light coming from a slit is proportional 10 V
to the slit-width. Find the ratio of the maximum to the
minimum intensity in the interference pattern.
29. An electromagnetic wave of frequency
5 GHz, is travelling in a medium whose relative electric
a. 4 : 1 b. 2 : 1 c. 1 : 4 d. 3 : 1
permittivity and relative magnetic permeability both are 2.
Section B : Numerical Type Questions Its velocity in this medium is ...... × 107 m/s.
21. The coefficient of static friction between a wooden 30. A common transistor radio set requires 12 V (DC) for its
block of mass 0.5 kg and a vertical rough wall is 0.2. operation. The DC source is constructed by using a
The magnitude of horizontal force that should be transformer and a rectifier circuit, which are operated at
applied on the block to keep it adhere to the wall will 220 V (AC) on standard domestic AC supply. The number
be ........ N. [Take, g = 10 ms − 2] of turns of secondary coil are 24, then the number of
turns of primary are ............ .

Answers For solution scan


the QR code
1. (c) 2. (b) 3. (*) 4. (d) 5. (a) 6. (d) 7. (c) 8. (a) 9. (d) 10. (d)
11. (b) 12. (c) 13. (c) 14. (d) 15. (c) 16. (a) 17. (d) 18. (c) 19. (d) 20. (a)
21. (25) 22. (75) 23. (1) 24. (25600) 25. (25) 26. (2000) 27. (25) 28. (25) 29. (15) 30. (440)

Note (*) None of the option is correct.


4 Online JEE Main 2021 February Attempt

24 FEBRUARY SHIFT II
Section A : Objective Type Questions 7. Given below are two statements:
1. When a particle executes SHM, the nature of graphical Statement I p-n junction diodes can be used to function as
representation of velocity as a function of displacement transistor, simply by connecting two diodes, back to back,
is which acts as the base terminal.
a. circular b. elliptical Statement II In the study of transistor, the amplification
c. parabolic d. straight line factor β indicates ratio of the collector current to the base
2. Two electrons each are fixed at a distance 2d. A third current.
charge proton placed at the mid-point is displaced In the light of the above statements, choose the correct
slightly by a distance x( x < < d) perpendicular to the line answer from the options given below.
joining the two fixed charges. Proton will execute simple a. Statement I is false but Statement II is true.
harmonic motion having angular frequency? b. Both Statement I and Statement II are true.
(m = mass of charged particle) c. Both Statement I and Statement II are false.
1 1
 2q 2  2  πε md 3  2 d. Statement I is true but Statement II is false.
a.   b.  0 2 
3
 πε0 md   2q  8. In the given figure, a body of mass M is held between two
1 1 massless springs, on a smooth inclined plane. The free
 q2  2  2 πε0 md 3  2 ends of the springs are attached to firm supports. If each
c.  3
 d.  
 2πε0 md   q2  spring has spring constant k, then the frequency of
oscillation of given body is
3. On the basis of kinetic theory of gases, the gas exerts
pressure because its molecules
k
a. continuously lose their energy till it reaches wall
b. are attracted by the walls of container
k M
c. continuously stick to the walls of container
d. suffer change in momentum when impinge on the walls of α
container
4. A soft ferromagnetic material is placed in an external 1 k 1 2k
magnetic field. The magnetic domains a. b.
2π 2M 2π Mg sin α
a. increase in size but no change in orientation
b. have no relation with external magnetic field 1 2k 1 k
c. d.
c. decrease in size and changes orientation 2π M 2π Mg sin α
d. may increase or decrease in size and change its orientation
9. Figure shows a circuit that contains four identical
5. A resistors with resistance R = 20. Ω, two identical
C
B inductors with inductance L = 2.0 mH and an ideal
The logic circuit shown above is equivalent to battery with electromotive force E = 9 V. The current i just
after the switch S is closed will be
A
a. C
B
S
A L R
b. A
C +
B R
E=9 V – i
R L
A
c. C
B
R
a. 2.25 A b. 3.0 A c. 3.37 A d. 9 A
A
d. C 10. The de-Broglie wavelength of a proton and α-particle are
B
equal. The ratio of their velocities is
6. The period of oscillation of a simple pendulum is a. 4 : 3 b. 4 : 1 c. 4 : 2 d. 1: 4
L
T = 2π . Measured value of L is 1.0 m from metre scale 11. If one mole of an ideal gas at ( p1, V1) is allowed to expand
g
reversibly and isothermally
having a minimum division of 1 mm and time of one
complete oscillation is 1.95 s measured from stopwatch (A to B ), its pressure is reduced to one-half of the original
of 0.01 s resolution. The percentage error in the pressure (see figure). This is followed by a constant
determination of g will be volume cooling till its pressure is reduced to one-fourth
a. 1.13% b. 1.03% c. 1.33% d. 1.30% of the initial value (B → C ). Then, it is restored to its initial
February Attempt Online JEE Main 2021 5

state by a reversible adiabatic compression (C to A). c. lightly doped and have narrow depletion layer
The net work done by the gas is equal to d. heavily doped and have wide depletion layer
P 18. Match List-I with List-II.

A
List-I List-II
p1
A. Source of microwave 1. Radioactive decay of nucleus
p1 frequency
B
2 B. Source of infrared 2. Magnetron
p1
C frequency
4
C. Source of gamma rays 3. Inner shell electrons
V D. Source of X-rays 4. Vibration of atoms and
V1 2V1 molecules
 1  RT
a. RT  ln 2 −  b. − 5. LASER
 2(γ − 1)  2(γ − 1)
6. R - C circuit
c. 0 d. RT ln2
12. An X-ray tube is operated at Choose the correct answer from the options given below.
1.24 million volt. The shortest wavelength of the A B C D A B C D
produced photon will be a. 6 4 1 5 b. 6 5 1 4
a. 10− 3 nm b. 10− 1 nm c. 2 4 6 3 d. 2 4 1 3
c. 10− 2 nm d. 10− 4 nm 19. A particle is projected with velocity v 0 along X-axis. A
13. Which of the following equations represents a damping force is acting on the particle which is proportional
travelling wave? to the square of the distance from the origin, i.e.
a. y = A sin(15x − 2t )
2
b. y = Ae − x (vt + θ) ma = − αx 2. The distance at which the particle stops is
1 1
c. y = Ae x cos(ωt − θ) d. y = A sin x cosωt  3 mv 02  2
b. 
2 mv 0  3
a.   
14. According to Bohr atom model, in which of the  2α   3α 
following transitions will the frequency be maximum 1 1
?  2 mv 02  2  3 mv 02  3
c.   d.  
a. n = 4 to n = 3 b. n = 2 to n = 1  3α   2α 
c. n = 5 to n = 4 d. n = 3 to n = 2
20. A body weights 49 N on a spring balance at the North pole.
15. If the source of light used in a Young's double slit What will be its weight recorded on the same weighing
experiment is changed from red to violet, then machine, if it is shifted to the equator ?
a. the consecutive fringe lines will come closer GM
(Use, g = 2 = 9.8 ms − 2 and radius of earth, R = 6400 km)
b. the central bright fringe will become a dark fringe R
c. the fringes will become brighter
a. 49 N b. 48.83 N
d. the intensity of minima will increase
c. 49.83 N d. 49.17 N
16. A circular hole of radius   is cut out of a circular
a
2 Section B : Numerical Type Questions
disc of radius a as shown in figure. The centroid of 21. A uniform metallic wire is elongated by
the remaining circular portion with respect to point O 0.04 m when subjected to a linear force F . The elongation, if
will be its length and diameter is doubled and subjected to the
Y-axis same force will be ……… cm.

22. A cylindrical wire of radius 0.5 mm and conductivity 5 × 107


S/m is subjected to an electric field of 10 mV/m. The
expected value of current in the wire will be x 3π mA. The
a/2 value of x is ……… .
O a X–axis
23. A uniform thin bar of mass 6 kg and length 2.4 m is bent to
make an equilateral hexagon. The moment of inertia about
an axis passing through the centre of mass and
perpendicular to the plane of hexagon is ………
1 10 5 2 × 10− 1 kg - m2.
a. a b. a c. a d. a
6 11 6 3
24. Two solids A and B of mass 1 kg and 2 kg respectively are
17. Zener breakdown occurs in a p-n junction having p moving with equal linear momentum. The ratio of their
and n both A
kinetic energies (KE) A : (KE) B will be , so the value of A will
a. lightly doped and have wide depletion layer 1
b. heavily doped and have narrow depletion layer be ……… .
6 Online JEE Main 2021 February Attempt

25. The root mean square speed of molecules of a given 27. A signal of 0.1 kW is transmitted in a cable. The
mass of a gas at 27°C and 1 atmosphere pressure is 200 attenuation of cable is − 5 dB per km and cable length is
ms − 1. The root mean square speed of molecules of the 20 km. The power received at receiver is 10−x W. The
x
gas at 127°C and 2 atmosphere pressure is ms − 1. The value of x is ……… .
3 P 
value of x will be ……… . [Gain in dB = 10 log10  0  ]
 Pi 
26. A point charge of + 12 µC is at a distance 6 cm vertically
above the centre of a square of side 12 cm as shown in 28. A series L-C -R circuit is designed to resonate at an
figure. The magnitude of the electric flux through the angular frequency ω 0 = 105 rad/s. The circuit draws 16 W
square will be ……… × 103 N - m2 / C . power from 120 V source at resonance. The value of
resistance R in the circuit is ……… Ω.
+q
29. Two cars are approaching each other at an equal speed
of 7.2 km/h. When they see each other, both blow horns
6 cm
having frequency of 676 Hz. The beat frequency heard
by each driver will be ……… Hz. [Velocity of sound in air is
340 m/s.]
30. An electromagnetic wave of frequency 3 GHz enters a
12 dielectric medium of relative electric permittivity 2.25
cm
cm

from vacuum. The wavelength of this wave in that


12

medium will be ……… × 10− 2 cm.

Answers For solution scan


the QR code
1. (b) 2. (c) 3. (d) 4. (d) 5. (d) 6. (a) 7. (a) 8. (c) 9. (a) 10. (b)
11. (a) 12. (a) 13. (a) 14. (b) 15. (a) 16. (c) 17. (b) 18. (d) 19. (d) 20. (b)
21. (2) 22. (5) 23. (8) 24. (2) 25. (400) 26. (226) 27. (8) 28. (900) 29. (8) 30. (667)

25 FEBRUARY SHIFT I
Section A : Objective Type Questions km, respectively. If TA and TB are the time periods of A
and B respectively, then the value of TB − TA is
1. Given below are two statements: one is labelled as
Assertion A and the other is labelled as Reason R.
Assertion (A) When a rod lying freely is heated, no
thermal stress is developed in it. B
A
Reason (R) On heating, the length of the rod increases. E
In the light of the above statements, choose the correct
answer from the options given below
a. Both A and R are true but R is not the correct explanation
of A. (Given, radius of Earth = 6400 km, mass of Earth
b. A is false but R is true. = 6 × 1024 kg)
c. A is true but R is false. a. 1.33 × 103 s b. 3.33 × 102 s
d. Both A and R are true and R is the correct explanation of A. c. 4.24 × 103 s d. 4.24 × 102 s
2. A student is performing the experiment of resonance 4. The angular frequency of alternating current in an L-C-R
column. The diameter of the column tube is 6 cm. The circuit is 100 rad/s. The components connected are
frequency of the tuning fork is 504 Hz. Speed of the shown in the figure. Find the value of inductance of the
sound at the given temperature is 336 m/s. The zero of coil and capacity of condenser.
the meter scale coincides with the top end of the
resonance column tube. The reading of the water level in R=60 Ω
the column when the first resonance occurs is 15 V
a. 13 cm b. 16.6 cm
c. 18.4 cm d. 14.8 cm R′=40 Ω L 20 V
C 10 V
3. Two satellites A and B of masses 200 kg and 400 kg are
revolving around the Earth at height of 600 km and 1600
February Attempt Online JEE Main 2021 7

a. 0.8 H and 150 µF b. 0.8 H and 250 µF 12. An engine of a train moving with uniform acceleration,
c. 1.33 H and 250 µF d. 1.33 H and 150 µF passes the signal-post with velocity u and the last
5. A proton, a deuteron and an α-particle are moving with compartment with velocity v. The velocity with which
same momentum in a uniform magnetic field. The ratio middle point of the train passes the signal post is
of magnetic forces acting on them is ……… and their v2 + u2 v −u u+v v2 − u2
speed is ……… in the ratio. a. b. c. d.
2 2 2 2
a. 1 : 2 : 4 and 2 : 1 : 1 b. 2 : 1 : 1 and 4 : 2 : 1
c. 4 : 2 : 1 and 2 : 1 : 1 d. 1 : 2 : 4 and 1 : 1 : 2 13. Match List-I with List-II
6. Given, below are two statements List-I List-II
Statement I A speech signal of 2 kHz is used to modulate a A. h (Planck's constant) 1. [M L T −1]
carrier signal of 1 MHz. The
bandwidth requirement for the signal is B. E (kinetic energy) 2. [M L2 T −1]
4 kHz.
C. V (electric potential) 3. [M L2 T −2 ]
Statement II The side band frequencies are 1002 kHz and
998 kHz. In the light of the above statements, choose the D. P (linear momentum) 4. [M L2I−1T −3 ]
correct answer from the options given below
a. Statement I is true but Statement II is false. Choose the correct answer from the options given
b. Statement I is false but Statement II is true. below.
A B C D
c. Both Statement I and Statement II are true.
a. 3 4 2 1
d. Both Statement I and Statement II are false.
b. 2 3 4 1
7. If the time period of a 2 m long simple pendulum is 2 s, c. 1 2 4 3
the acceleration due to gravity at the place, where d. 3 2 4 1
pendulum is executing SHM is 14. Magnetic fields at two points on the axis of a circular coil
a. π 2 ms −2 b. 9.8 ms −2 c. 2 π 2 ms −2 d. 16 ms −2
at a distance of 0.05 m and 0.2 m from the centre are in
8. The pitch of the screw gauge is 1 mm and there are 100 the ratio 8 : 1. The radius of coil is
divisions on the circular scale. When nothing is put in a. 0.2 m b. 0.1 m
between the jaws, the zero of the circular scale lies 8 c. 0.15 m d. 1.0 m
divisions below the reference line. When a wire is placed
between the jaws, the first linear scale division is clearly 15. A solid sphere of radius R gravitationally attracts a
visible while 72nd division on circular scale coincides particle placed at 3R from its centre with a force F1. Now,
with the reference line. The radius of the wire is R
a spherical cavity of radius   is made in the sphere (as
a. 1.64 mm b. 0.82 mm
 2
c. 1.80 mm d. 0.90 mm shown in figure) and the force becomes F 2. The value of
F1 : F 2 is
9. A 5 V battery is connected across the points X and Y.
Assume D1 and D 2 to be normal silicon diodes. Find the
current supplied by the battery, if the positive terminal
of the battery is connected to point X.
B A
D1
10 Ω O 2R m

D2
5Ω

a. 25 : 36 b. 36 : 25
X Y
c. 50 : 41 d. 41 : 50
a. ~ 0.5 A b. ~ 1.5 A 16. Two radioactive substances X and Y originally have N1
c. ~ 0.86 A d. ~ 0.43 A and N2 nuclei, respectively. Half-life of X is half of the
10. An α-particle and a proton are accelerated from rest by a half-life of Y. After three half-lives of Y, number of nuclei
N
potential difference of 200 V. After this, their de-Broglie of both are equal. The ratio 1 will be equal to
λp N2
wavelengths are λ α and λ p, respectively. The ratio is 1 3
λα a. b.
8 1
a. 3.8 b. 8 c. 7.8 d. 2.8
8 1
7 5 c. d.
11. A diatomic gas having C p = R and C V = R , is heated at 1 3
2 2
constant pressure. The 17. In an octagon ABCDEFGH of equal side, what is the sum
ratio dU : dQ : dW is of AB + AC + AD + AE + AF + AG + AH
a. 5 : 7 : 3 b. 5 : 7 : 2 c. 3 : 7 : 2 d. 3 : 5 : 2 if, AO = 2$i + 3$j − 4k$ ?
8 Online JEE Main 2021 February Attempt

A B
23. In a certain thermodynamical process, the pressure of a
gas depends on its volume as kV 3. The work done when
H C
the temperature changes from 100°C to 300°C will be
O ……… nR, where n denotes number of moles of a gas.
G D
24. A small bob tied at one end of a thin string of length 1m
is describing a vertical circle, so that the maximum and
F E
minimum tension in the string are in the ratio 5 : 1. The
a. −16$i − 24 $j + 32k$ b. 16 $i + 24 $j − 32k$ velocity of the bob at the highest position is ……… m/s.
c. 16 i$ + 24 $j + 32k$ d. 16 i$ − 24 $j + 32k$
(Take, g = 10 m/s 2)
18. Given below are two statements: one is labelled as
Assertion A and the other is labelled as Reason R.
25. In the given circuit of potentiometer, the potential
difference E across AB (10 m length) is larger than E 1 and
Assertion A The escape velocities of planet A and B are E 2 as well. For key K1(closed), the jockey is adjusted to
same. But A and B are of unequal mass. touch the wire at point J1, so that there is no deflection in
Reason R The product of their mass and radius must be the galvanometer. Now, the first battery (E 1) is replaced
same, M1R1 = M 2R 2 by second battery (E 2) for working by making K1 open and
K 2 closed. The galvanometer gives then null deflection at
In the light of the above statements, choose the most
J 2. The value of E 1 /E 2 is a / b,, where a = ……… .
appropriate answer from the options given below.
K1
(a) Both A and R are correct but R is not the correct
explanation of A. E1
(b) A is correct but R is not correct.
(c) Both A and R are correct and R is the correct explanation of G
A. E2 K2
E
(d) A is not correct but R is correct.
A
19. The current ( i) at time t = 0 and t = ∞ respectively for the K
given circuit is 20 cm J1
5Ω 5Ω
i
E J2
Rh 60 cm

1Ω 4Ω
B 1m

L
26. The same size images are formed by a convex lens when
the object is placed at 20 cm or at 10 cm from the lens.
18E 5E 10E 5E 5E 18E 5E 10E The focal length of convex lens is ………… cm.
a. , b. , c. , d. ,
55 18 33 18 18 55 18 33 27. 512 identical drops of mercury are charged to a potential
20. Two coherent light sources having intensity in the ratio of 2 V each. The drops are joined to form a single drop.
Imax − Imin The potential of this drop is ………… V.
2x produce an interference pattern. The ratio
Imax + Imin 28. A coil of inductance 2H having negligible resistance is
will be connected to a source of supply whose voltage is given
2 2x 2x 2x 2 2x by V = 3t V (where, t is in second). If the voltage is applied
a.
x+1
b.
2x + 1
c.
x+1
d.
2x + 1
when t = 0, then the energy stored in the coil after 4 s is
………… J.
Section B : Numerical Type Questions 29. A monoatomic gas of mass 4.0 u is kept in an insulated
21. A transmitting station releases waves of wavelength container. Container is moving with velocity 30 m/s. If
960 m. A capacitor of 2.56 µF is used in the resonant container is suddenly stopped, then change in
x
circuit. The self-inductance of coil necessary for temperature of the gas (R = gas constant) is . Value of
3R
resonance is ……… × 10−8 H.
x is …… .
22. The electric field in a region is given 30. The potential energy (U) of a diatomic molecule is a
3 4  N function dependent on r (interatomic distance) as
E =  E 0$i + E 0$j . The ratio of flux of reported field α β
5 5  C U = 10 − 5 − 3
through the rectangular surface of area 0.2 m2 (parallel r r
to YZ-plane) to that of the surface of area 0.3 m 2 (parallel where, α and β are positive constants. The equilibrium
a/ b
to XZ- plane) is a : b, where a = ……… .  2α 
distance between two atoms will be   , where a =
$ $j and k$ are unit vectors along X, Y and Z-axes,
[Here i,  β
respectively] ……… .
February Attempt Online JEE Main 2021 9

Answers For solution scan


the QR code

1. (a) 2. (d) 3. (a) 4. (b) 5. (b) 6. (c) 7. (c) 8. (b) 9. (d) 10. (d)
11. (b) 12. (a) 13. (b) 14. (b) 15. (c) 16. (c) 17. (b) 18. (b) 19. (d) 20. (d)
21. (10) 22. (1) 23. (50) 24. (5) 25. (1) 26. (15) 27. (128) 28. (144) 29. (3600) 30. (1)

25 FEBRUARY SHIFT II
Section A : Objective Type Questions 5. Thermodynamic process is shown below on a p-V
1. If e is the electronic charge, c is the speed of light in free diagram for one mole of an ideal gas.
2
1 |e | T2
space and h is Planck's constant, the quantity If V2 = 2V1, then the ratio of temperature is
4 πε 0 hc T1
has dimensions of p
a. [MLT 0 ] b. [MLT −1] 1(p1, V1, T1)
c. [M 0L0 T 0 ] d. [LC −1]
pV1/2=constant

2. A stone is dropped from the top of a building. When it


crosses a point 5 m below the top, another stone starts 2(p2, V2, T2)
to fall from a point 25 m below the top. Both stones
reach the bottom of building simultaneously. The height V
of the building is V1 V2
a. 45 m b. 25 m c. 35 m d. 50 m 1
a. b. 2
3. A sphere of radius a and mass m rolls along a horizontal 2
plane with constant speed v 0. It encounters an inclined 1
c. d. 2
plane at angle θ and climbs upwards. Assuming that it 2
rolls without slipping, how far up the sphere will travel? 6. Given below are two statements:
Statement I In a diatomic molecule, the rotational
energy at a given temperature obeys Maxwell's
distribution.
a v0 Statement II In a diatomic molecule, the rotational energy
θ at a given temperature equals the translational kinetic
energy for each molecule.
v 02 v 02
a. b. In the light of the above statements, choose the correct
2 g sin θ 5 g sin θ
answer from the options given below.
10 v 02 2 v 02 a. Both Statement I and Statement II are true.
c. d.
7 g sin θ 5 g sin θ b. Both Statement I and Statement II are false.
c. Statement I is true but Statement II is false.
4. The point A moves with a uniform speed along the d. Statement I is false but Statement II is true.
circumference of a circle of radius 0.36 m and covers 30°
in 0.1 s. The perpendicular projection P from A on the 7. Two identical springs of spring constant 2k are attached
diameter MN represents the simple harmonic motion of to a block of mass m and to fixed support (see
P. The restoration force per unit mass when P touches M figure).When the mass is displaced from equilibrium
will be position on either side, it executes simple harmonic
motion. The time period of oscillations of this system is
A

0.1s
0.36 m 30° 2k 2k
M N
O P m

m m
a. 2 π b. 2π
2k k
a. 100 N b. 9.87 N c. π
m
d. π
m
c. 50 N d. 0.49 N k 2k
10 Online JEE Main 2021 February Attempt

8. Y = A sin (ωt + φ0) is the time-displacement equation of diameter of the pinhole is slightly increased, it will affect
A the diffraction pattern such that
SHM. At t = 0, the displacement of the particle is Y = a. its size increases and intensity increases
2
and it is moving along negative x-direction. Then, the b. its size increases, but intensity decreases
initial phase angle φ0 will be c. its size decreases, but intensity increases
π 5π π 2π d. its size decreases and intensity decreases
a. b. c. d.
3 6 6 3
15. An electron of mass m e and a proton of mass
9. A charge q is placed at one corner of a cube as shown in m p = 1836 m e are moving with the same speed.
figure. The flux of electrostatic field E through the λ
The ratio of their de-Broglie wavelength electron will be
shaded area is λproton
Z 1
a. 1 b. 1836 c. d. 918
1836
16. The wavelength of the photon emitted by a hydrogen
atom when an electron makes a transition from n = 2 to
n = 1state is
a. 121.8 nm b. 194.8 nm
c. 490.7 nm d. 913.3 nm
q Y 17. If a message signal of frequency fm is amplitude
modulated with a carrier signal of frequency f c and
X radiated through an antenna, the wavelength of the
q q q q corresponding signal in air is
a. b. c. d. c c c c
48ε0 4 ε0 8ε0 24 ε0 a. b. c. d.
fc − fm fc + fm fc fm
10. An electron with kinetic energy K1 enters between
18. For extrinsic semiconductors when doping level is
parallel plates of a capacitor at an angle α with the increased,
plates. It leaves the plates at angle β with kinetic energy
a. Fermi level of p-type semiconductor will go upward and
K 2. Then, the ratio of kinetic energies K1 : K 2 will be Fermi level of n-type semiconductors will go downward
cos β cos β b. Fermi level of p-type semiconductors will go downward
a. b.
cos α sin α and Fermi level of n-type semiconductor will go upward
sin 2 β cos 2 β c. Fermi level of p and n-type semiconductors will not be
c. 2
d. affected
cos α cos 2 α
d. Fermi level of both p-type and n-type semiconductors will
11. In a ferromagnetic material, below the Curie go upward for T > TF K and downward for T < TF K, where TF
is Fermi temperature
temperature, a domain is defined as
a. a macroscopic region with zero magnetisation 19. Match List-I with List-II.
b. a macroscopic region with saturation magnetisation
c. a macroscopic region with randomly oriented magnetic List-I List-II
dipoles A. Rectifier 1. Used either for stepping up or
d. a macroscopic region with consecutive magnetic dipoles stepping down the AC voltage
oriented in opposite direction
B. Stabiliser 2. Used to convert AC voltage into DC
12. An L-C-R circuit contains resistance of 110 Ω and a supply voltage
of 220 V at 300 rad/s angular frequency. If only
C. Transformer 3. Used to remove any ripple in the
capacitance is removed from the circuit, current lags rectified output voltage
behind the voltage by 45°. If on the other hand, only
inductor is removed the current leads by 45° with the D. Filter 4. Used for constant output voltage
applied voltage. The rms current flowing in the circuit will even when the input voltage or load
be current change
a. 1A b. 1.5 A c. 2A d. 2.5 A
Choose the correct answer from the options given
13. The stopping potential for electrons emitted from a below.
photosensitive surface illuminated by light of wavelength A B C D A B C D
491 nm is 0.710 V. When the incident wavelength is a. 2 1 3 4 b. 2 4 1 3
changed to a new value, the stopping potential is 1.43 V. b. 2 1 4 3 c. 3 4 1 2
The new wavelength is
20. The truth table for the followng logic circuit is
a. 309 nm b. 329 nm
c. 382 nm d. 400 nm A
14. Consider the diffraction pattern obtained from the
sunlight incident on a pinhole of diameter 0.1µm. If the Y
B
February Attempt Online JEE Main 2021 11

A B Y reduced by 52 K, its efficiency is doubled. The


0 0 0 temperature in kelvin of the source will be …………… .

a. 0 1 1 26. Two small spheres each of mass 10 mg are suspended


1 0 1 from a point by threads 0.5 m long. They are equally
charged and repel each other to a distance of 0.20 m.
1 1 0 a
The charge on each of the sphere is × 10−8 C. The
21
A B Y value of a will be …………… .
0 0 1
[Given, g = 10 ms −2]
b. 0 1 0
27. Two identical conducting spheres with negligible volume
1 0 0 have 2.1 nC and – 0.1 nC charges, respectively. They are
1 1 1 brought into contact and then separated by a distance of
0.5 m. The electrostatic force acting between the spheres
A B Y is …………… × 10 −9 N.
0 0 1 1
[Given, 4 πε 0 = SI unit]
c.
0 1 0 9 × 109
1 0 1
28. The peak electric field produced by the radiation coming
1 1 0 x µ 0c V
from the 8 W bulb at a distance of 10 m is .
10 π m
A B Y
The efficiency of the bulb is 10% and it is a point source.
0 0 0 The value of x is ………… .
0 1 1
d. 29. A current of 6 A enters one corner P of an equilateral
1 0 0 triangle PQR having three wires of resistance 2 Ω each
1 1 1 and leaves by the corner R. The currents i l in ampere is
………… .
Section B : Numerical Type Questions
21. Two particles having masses 4 g and 16 g respectively 6A
are moving with equal kinetic energies. The ratio of the
2Ω
magnitudes of their linear momentum is n : 2. The value
of n will be ………… .
P
22. The initial velocity v i required to project a body vertically i1 I2
upward from the surface of the Earth to reach a height
of 10R, where R is the radius of the Earth, may be 2Ω 2Ω
described in terms of escape velocity v e such that
x
vi = × v e . The value of x will be ………… .
y 2Ω
Q R
23. The percentage increase in the speed of transverse
waves produced in a stretched string, if the tension is 30. The wavelength of an X-ray beam is 10 Å. The mass of a
increased by 4%, will be ………… %. fictitious particle having the same energy as that of the
x
X-ray photons is h kg. The value of x is ………… .
24. If P × Q = Q × P, the angle between P and Q is 3
θ (0° < θ < 360°). The value of θ will be ……………°. (h = Planck's constant)

25. A reversible heat engine converts one-fourth of the heat


input into work. When the temperature of the sink is

Answers For solution scan


the QR code

1. (c) 2. (a) 3. (*) 4. (b) 5. (b) 6. (c) 7. (c) 8. (c) 9. (d) 10. (d)
11. (b) 12. (c) 13. (c) 14. (c) 15. (b) 16. (a) 17. (c) 18. (b) 19. (b) 20. (b)
21. (1) 22. (10) 23. (2) 24. (180) 25. (104) 26. (630) 27. (7.56) 28. (2) 29. (2) 30. (10)
12 Online JEE Main 2021 February Attempt

26 FEBRUARY SHIFT I
Objective Type Questions a.
2πR
b.
g
c.
1 g
d. 2π
R
1. Find the gravitational force of attraction between the g 2πR 2π R g
ring and sphere as shown in the figure, where the plane 7. An alternating current is given by the equation
of the ring is perpendicular to the line joining the i = i1 sin ωt + i 2 cos ωt. The rms current will be
centres. If 8 R is the distance between the centres of a 1 2 1
a. (i1 + i 22 )1/ 2 b. (i1 + i 2 ) 2
ring (of mass m) and a sphere (of mass M), where both 2 2
have equal radius R. 1 1
c. (i12 + i 22 )1/ 2 d. (i1 + i 2 )
2 2
8. The normal density of a material is ρ and its bulk
m M modulus of elasticity is K. The magnitude of increase in
R R density of material, when a pressure p is applied
uniformly on all sides, will be
X Y ρK ρp K pK
a. b. c. d.
p K ρp ρ

8 GmM 2 2 GMm 1 GMm 8 GmM 9. A particle is moving with uniform speed along the
a. ⋅ b. ⋅ 2 c. ⋅ d. ⋅ circumference of a circle of radius R under the action of a
9 R 3 R 3 8 R2 27 R 2
central fictitious force F which is inversely proportional to
2. Consider the combination of two capacitors C 1 and C 2, R 3. Its time period of revolution will be given by
with C 2 > C 1, when connected in parallel, the equivalent a. T ∝ R 2 b. T ∝ R 3 / 2
capacitance is 15/4 time the equivalent capacitance of c. T ∝ R 5 / 2 d. T ∝ R 4 / 3
the same connected in series. Calculate the ratio of 10. A planet revolving in elliptical orbit has
C
capacitors 2 . I. a constant velocity of revolution
C1
15 111 29 15 II. has the least velocity when it is
a. b. c. d. nearest to the Sun
11 80 15 4
III. its areal velocity is directly
3. In a typical combustion engine,
2
the work done by a gas proportional to its velocity
− βx
molecule is given W = α 2βe kT , where x is the IV. areal velocity is inversely
proportional to its velocity.
displacement, k is the Boltzmann constant and T is the
V. to follow a trajectory such that
temperature. If α and β are constants, dimensions of α
the areal velocity is constant.
will be
Choose the correct answer from the options given below.
a. [MLT −2 ] b. [M 0 LT 0 ] c. [M 2 LT −2 ] d. [MLT −1]
a. Only I b. Only IV
4. If λ 1 and λ 2 are the wavelengths of the third member of c. Only III d. Only V
Lyman and first member of the Paschen series
11. Given below are two statements : one is labelled as
respectively, then the value of λ 1 : λ 2 is
Assertion A and the other is labelled as Reason R.
a. 1 : 9 b. 7 : 108 c. 7 : 135 d. 1 : 3
Assertion A Body P having mass M moving with speed u
5. A short straight object of height 100 cm lies before the has head-on collision elastically with another body Q
central axis of a spherical mirror, whose focal length has having mass m initially at rest. If m << M, body Q will have a
absolute value f = 40 cm. The image of object produced maximum speed equal to 2u after collision.
by the mirror is of height 25 cm and has the same Reason R During elastic collision, the momentum and
orientation of the object. One may conclude from the kinetic energy are both conserved.
information.
In the light of the above statements, choose the most
a. Image is real, same side of concave mirror
appropriate answer from the options given below.
b. Image is virtual, opposite side of concave mirror
a. A is not correct but R is correct.
c. Image is real, same side of convex mirror
b. Both A and R are correct but R is not the correct
d. Image is virtual, opposite side of convex mirror
explanation of A.
6. Assume that a tunnel is dug along a chord of the earth, c. Both A and R are correct and R is the correct explanation of
R A.
at a perpendicular distance   from the earth's centre,
 2 d. A is correct but R is not correct.
where R is the radius of the earth. The wall of the tunnel 12. Four identical solid spheres each of mass m and radius a
is frictionless. If a particle is released in this tunnel, it will are placed with their centres on the four corners of a
execute a simple harmonic motion with a time period? square of side b. The moment of inertia of the system
February Attempt Online JEE Main 2021 13

about one side of square, where the axis of rotation is In the light of the above statements, choose the correct
parallel to the plane of the square is answer from the options given below.
4 8 a. A is true but R is false.
a. ma 2 + 2mb 2 b. ma 2 + mb 2
5 5 b. Both A and R are true and R is the correct explanation of A.
8 4 c. Both A and R are true but R is not the correct explanation
c. ma 2 + 2mb 2 d. ma 2
5 5 of A.
d. A is false but R is true.
13. In a Young's double slit experiment, two slits are
separated by 2 mm and the screen is placed one metre
18. LED is constructed from GaAsP semiconducting material.
The energy gap of this LED is 1.9 eV. Calculate the
away. When a light of wavelength 500 nm is used, the
wavelength of light emitted and its colour.
fringe separation will be
[h = 6.63 × 10−34 Js and c = 3 × 108 ms −1]
a. 0.25 mm b. 0.50 mm
a. 1046 nm and red colour b. 654 nm and orange colour
c. 0.75 mm d. 1 mm
c. 1046 nm and blue colour d. 654 nm and red colour
14. Find the electric field at point P (as shown in figure) on
the perpendicular bisector of a uniformly charged thin
19. A large number of water drops, each of radius r,
combine to have a drop of radius R. If the surface
wire of length L carrying a charge Q. The distance of the
3 tension is T and mechanical equivalent of heat is J, the
point P from the centre of the rod is a = L. rise in heat energy per unit volume will be
2
2T  1 1  2T 3T 3T  1 1 
a.  −  b. c. d.  − 
J  r R rJ rJ J  r R

20. Five equal resistances are connected in a network as


a E shown in figure. The net resistance between the points A
L and B is
O P
D

Q R
3Q Q R R
a. b.
4 πε0L2 3 πε0L2
R A
Q Q B
c. d.
2 3 π ε0L2 4 πε0L2
E C
R
15. If two similar springs each of spring constant K1 are R 3R
a. 2R b. c. d. R
joined in series, the new spring constant and time period 2 2
would be changed by a factor
1
a. , 2 b.
1
, 2
Section B : Numerical Type Questions
2 4
1 1 21. A person standing on a spring balance inside a stationary
c. , 2 2 d. , 2 2 lift measures 60 kg. The weight of that person, if the lift
4 2
descends with uniform downward acceleration of 1.8
16. The temperature θ at the junction of two insulating m/s 2 will be ……… N. [g = 10 m/s 2]
sheets, having thermal resistances R1 and R 2 as well as
top and bottom temperatures θ1 and θ 2 (as shown in 22. In an electrical circuit, a battery is connected to pass 20 C
figure) is given by of charge through it in a certain given time. The potential
difference between two plates of the battery is maintained
θ2 at 15 V. The work done by the battery is ……… J.
R2
23. The circuit contains two diodes each with a forward
θ
R1
resistance of 50 Ω and with infinite reverse resistance. If
the battery voltage is 6V, the current through the 120 Ω
θ1 resistance is ……… mA.
θ 2 R 2 − θ1 R1 θ R − θ 2R1 θ1 R 2 + θ 2R1 θ1 R1 + θ 2R 2 D1 130Ω
a. b. 1 2 c. d.
R 2 − R1 R 2 − R1 R1 + R 2 R1 + R 2

17. Given below are two statements: One is labelled as


Assertion A and the other is labelled as Reason R. D2
100Ω
Assertion A An electron microscope can achieve better
resolving power than an optical microscope.
Reason R The de-Broglie's wavelength of the electrons 120Ω
emitted from an electron gun is much less than
wavelength of visible light. 6V
14 Online JEE Main 2021 February Attempt

24. A radiation is emitted by 1000 W bulb and it generates 27. A container is divided into two chambers by a partition.
an electric field and magnetic field at P, placed at a The volume of first chamber is 4.5 L and second
distance of 2 m. The efficiency of the bulb is 1.25%. The chamber is 5.5 L. The first chamber contain 3.0 mol of
value of peak electric field at P is x × 10−1 V/m. Value of x gas at pressure 2.0 atm and second chamber contain 4.0
is ……… . mol of gas at pressure 3.0 atm. After the partition is
removed and the mixture attains equilibrium, then the
(Rounded-off to the nearest integer)
common equilibrium pressure existing in the mixture is
[Take, ε 0 = 8.85 × 10−12 C 2N–1m–2 , c = 3 × 108 ms −1] x × 10−1 atm. Value of x is ……… .
25. A boy pushes a box of mass 2 kg with a force 28. The mass per unit length of a uniform wire is 0.135 g/cm.
F = (20i$ + 10$j) N on a frictionless surface. If the box was A transverse wave of the form y = − 0.21sin ( x + 30t) is
initially at rest, then ……… m is displacement along the produced in it, where x is in metre and t is in second.
X-axis after 10 s. Then, the expected value of tension in the wire is
26. As shown in the figure, a block of mass 3 kg is kept on a x × 10−2 N. Value of x is ……… (Round-off to the nearest
horizontal rough surface of coefficient of friction 1/ 3 3. integer)
The critical force to be applied on the vertical surface as 29. In a series L-C-R resonant circuit, the quality factor is
shown at an angle 60° with horizontal such that it does measured as 100. If the inductance is increased by two
not move, will be 3x. The value of x will be ……… fold and resistance is decreased by two fold, then the
3 1 quality factor after this change will be ……… .
[ g = 10 ms −2; sin 60° = ; cos 60° = ]
2 2 30. The maximum and minimum amplitude of an amplitude
modulated wave is 16 V and 8 V, respectively. The
modulation index for this amplitude modulated wave is
µ = 1/3√3 m= √3kg x × 10−2. The value of x is ……… .

60°

Answers For solution scan


the QR code

1. (d) 2. (*) 3. (b) 4. (c) 5. (d) 6. (d) 7. (a) 8. (b) 9. (a) 10. (d)
11. (c) 12. (c) 13. (a) 14. (c) 15. (a) 16. (c) 17. (b) 18. (d) 19. (d) 20. (d)
21. (492) 22. (300) 23. (20) 24. (137) 25. (500) 26. (3.33) 27. (25.5) 28. (1215) 29. (400) 30. (33.33)
Note (*) None of the option is correct.

26 FEBRUARY SHIFT II
Objective Type Questions 4. A tuning fork A of unknown frequency produces 5
1. If C and V represent capacity and voltage respectively, beats/s with a fork of known frequency 340 Hz. When
C fork A is filled, the beat frequency decreases to 2 beats/s.
then what are the dimensions of λ , where = λ ? What is the frequency of fork A?
V
a. [M −2L−3I2 T 6 ] b. [M −3L−4I3 T 7 ] a. 342 Hz b. 345 Hz c. 335 Hz d. 338 Hz
c. [M −1L−3I−2 T −7 ] d. [M −2L−4I3 T 7 ] 5. A particle executes SHM, the graph of velocity as a
2. The length of metallic wire is l1 when tension in it is T1. It function of displacement is
a. a circle b. a parabola
is l 2 when the tension is T2. The original length of the
c. an ellipse d. a helix
wire will be
l1 + l 2 T2 l1 + T1l 2 T2 l1 − T1l 2 T1l1 − T2 l 2 6. The trajectory of a projectile in a vertical plane is
a. b. c. d.
2 T1 + T2 T2 − T1 T2 − T1 y = αx − βx 2 , where α and β are constants and x and y are
respectively the horizontal and vertical distances of the
3. An aeroplane with its wings spread 10 m, is flying at a projectile from the point of projection. The angle of
speed of 180 km/h in a horizontal direction. The total projection θ and the maximum height attained H are
intensity of Earth's field at that part is 2.5 × 10−4 Wb/m 2 respectively given by
and the angle of dip is 60°. The emf induced between the α2 α2
tips of the plane wings will be a. tan −1 α , b. tan −1 β ,
4β 2β
a. 108.25 mV b. 54.125 mV
−1 4α 2 −1  β  α
2
c. 88.37 mV d. 62.50 mV c. tan α , d. tan   ,
β α β
February Attempt Online JEE Main 2021 15

7. A cord is wound round the circumference of wheel of charge 5 mC is allowed to slide down from rest at a
radius r. The axis of the wheel is horizontal and the height of 1m. If the coefficient of friction is 0.2, find the
moment of inertia about it is I. A weight mg is attached to time taken by the body to reach the bottom.
the cord at the end. The weight falls from rest. After 1 3
[Take, g = 9.8 m/s 2, sin 30° = , cos 30° = ]
falling through a distance h, the square of angular 2 2
velocity of wheel will be
2mgh 2mgh 2 gh
a. b. c. 2gh d. C
I + 2mr 2 I + mr 2 I + mr 2 5m
/C g,
0N 1k 1m
8. The internal energy (U), pressure (p) and volume (V) of an 2 0
ideal gas are related as U = 3pV + 4. The gas is E=
a. diatomic only b. polyatomic only 30°
c. Either monoatomic or diatomic d. monoatomic only a. 0.92 s b. 0.46 s c. 2.3 s d. 1.3 s
9. Given below are two statements: one is labelled as
Assertion (A) and the other is labelled as Reason (R).
14. Two masses A and B, each of mass M are fixed together
by a massless spring. A force acts on the mass B as
Assertion (A) For a simple microscope, the angular size of shown in figure. If the mass A starts moving away from
the object equals the angular size of the image. mass B with acceleration a, then the acceleration of
Reason (R) Magnification is achieved as the small object mass B wil be
can be kept much closer to the eye than 25 cm and hence,
it subtends a large angle. F B A
In the light of the above statements, choose the most Ma − F MF
appropriate answer from the options given below. a. b.
M F + Ma
a. A is true but R is false.
F + Ma F − Ma
b. Both A and R are true but R is not the correct explanation c. d.
of A. M M
c. Both A and R are true and R is the correct explanation of A. 15. Draw the output signal Y in the given combination of
d. A is false but R is true. gates
10. Given below are two statements:
Statement I An electric dipole is placed at the centre of a
hollow sphere. The flux of electric field through the sphere A 0 1 2 3 4 5 t(s)
is zero but the electric field is not zero anywhere in the
sphere.
Statement II If R is the radius of a solid metallic sphere B
and Q be the total charge on it. The electric field at any 0 1 2 3 4 5 t(s)
point on the spherical surface of radius r (< R ) is zero but
the electric flux passing through this closed spherical
surface of radius r is not zero. A
Y
In the light of the above statements, choose the correct
answer from the options given below. B
a. Both Statement I and Statement II are true.
b. Statement I is true but Statement II is false.
c. Both Statement I and Statement II are false.
a.
d. Statement I is false but Statement II is true. 0 1 2 3 4 5 t(s)
11. The recoil speed of a hydrogen atom after it emits a
photon in going from n = 5 state to n = 1state will be
a. 4.17 m/s b. 2.19 m/s c. 3.25 m/s d. 4.34 m/s
12. Find the peak current and resonant frequency of the
following circuit (as shown in figure). b.
0 1 2 3 4 5 t(s)
100 mH 100 µF

V=30sin100 t
c.
0 1 2 3 4 5 t(s)
120 Ω
a. 0.2 A and 50 Hz b. 0.2 A and 100 Hz
c. 2 A and 100 Hz d. 2 A and 50 Hz
13. An inclined plane making an angle of 30° with the d.
horizontal is placed in a uniform horizontal electric field 0 1 2 3 4 5 t(s)
200 N/C as shown in the figure. A body of mass 1kg and
16 Online JEE Main 2021 February Attempt

16. A radioactive sample is undergoing α-decay. At any time


A
t1, its activity is A and another time t 2 , the activity is .
5
What is the average life time for the sample? 50 cm S
60 cm
ln 5 t −t t −t ln (t 2 + t1)
a. b. 1 2 c. 2 1 d.
t 2 − t1 ln 5 ln 5 2

17. A scooter accelerates from rest for time t1 at constant 1.2 m


rate a1 and then retards at constant rate a 2 for time t 2
25. A particle executes SHM with amplitude a and time
and comes to rest. The correct value of t / t will be
a1 + a 2 a2 a1
1 2
a1 + a 2 period T . The displacement of the particle when its speed
a. b. c. d. is half of maximum speed is xa /2. The value of x is
a2 a1 a2 a1
........... .
18. Given below are two statements:
Statement I A second's pendulum has a time period of 1 s. 26. 27 similar drops of mercury are maintained at 10 V each.
All these spherical drops combine into a single big drop.
Statement II It takes precisely one second to move
The potential energy of the bigger drop is ............ times
between the two extreme positions.
that of a smaller drop.
In the light of the above statements, choose the correct
answer from the options given below. 27. Time period of a simple pendulum is T. The time taken to
a. Both Statement I and Statement II are false. complete 5/8 oscillations starting from mean position is
α
b. Statement I is false but Statement II is true. T. The value of α is ......... .
β
c. Statement I is true but Statement II is false.
d. Both Statement I and Statement II are true. 28. In the reported figure of Earth, the value of acceleration
19. A wire of 1 Ω has a length of 1m. It is stretched till its due to gravity is same at point A and C but it is smaller
than that of its value at point B (surface of the Earth). The
length increases by 25%. The percentage change in
value of OA : AB will be x : y. The value of x is .......... .
resistance to the nearest integer is
a. 56% b. 25% c. 12.5% d. 76% C
20. The incident ray, reflected ray and the outward drawn 3200 km
normal are denoted by the unit vectors a, b and c, B
respectively. Then, choose the correct relation for these
vectors. A
a. b = a + 2c b. b = 2a + c c. b = a − 2(a ⋅ c )c d. b = a − c
Earth O
Numerical Type Questions
21. The volume V of a given mass of monoatomic gas R=6400 km
changes with temperature T according to the relation
V = kT 2/ 3. The work done when temperature changes by
90 K will be xR. The value of x is……… . 29. 1 mole of rigid diatomic gas performs a work of Q/5
[R = universal gas constant] when heat Q is supplied to it. The molar heat capacity of
xR
22. If the highest frequency modulating a carrier is 5 kHz, the gas during this transformation is , The value of x is
8
then the number of AM broadcast stations ........... .
accommodated in a 90 kHz bandwidth are ......... .
[R = universal gas constant]
23. Two stream of photons, possessing energies equal to
twice and ten times the work function of metal are 30. The Zener diode has a Vz = 30 V. The current passing
incident on the metal surface successively. The value of through the diode for the following circuit is ......... mA.
ratio of maximum velocities of the photoelectrons
4kΩ
emitted in the two respective cases is x : y. The value of x
is .............. .
24. A point source of light S, placed at a distance 60 cm in
front of the centre of a plane mirror of width 50 cm, 90 V 5kΩ
hangs vertically on a wall. A man walks in front of the
mirror along a line parallel to the mirror at a distance 1.2
m from it (see in the figure). The distance between the
extreme points, where he can see the image of the light
source in the mirror is .......... cm.

Answers For solution scan


the QR code

1. (d) 2. (c) 3. (a) 4. (c) 5. (c) 6. (a) 7. (b) 8. (b) 9. (c) 10. (b)
11. (a) 12. (a) 13. (d) 14. (d) 15. (d) 16. (c) 17. (b) 18. (b) 19. (a) 20. (c)
21. (60) 22. (9) 23. (1) 24. (150) 25. (3) 26. (243) 27. (7) 28. (4) 29. (25) 30. (9)
March Attempt Online JEE Main 2021 17

16 MARCH SHIFT I
Section A : Objective Type Questions C. angle of incidence is equal to that of the angle of
emergence
1. One main scale division of a vernier callipers is a cm and
nth division of the vernier scale coincide with (n − 1)th D. angle of emergence is double the angle of incidence
division of the main scale. The least count of the callipers Choose the correct answer from the options given below.
(in mm) is a. Statements (A), (B) and (C) are true.
10 na 10 a  n − 1 10a b. Only statement (D) is true.
a. b. c.  a d.
(n − 1) (n − 1)  10 n  n c. Only statements (A) and (B) are true.
d. Statements (B) and (C) are true.
2. For changing the capacitance of a given parallel plate
capacitor, a dielectric material of dielectric constant K is 6. A plane electromagnetic wave of frequency 500 MHz is
used, which has the same area as the plates of the travelling in vacuum along y-direction. At a particular
3 point in space and time, B = 80 . × 10−8 z$ T . The value of
capacitor. The thickness of the dielectric slab is d, electric field at this point is
4
where d is the separation between the plates of parallel (speed of light = 3 × 108 ms −1;
plate capacitor. The new capacitance $ z$ are unit vectors along x , y and z-direction).
$ y,
x,
(C′) in terms of original capacitance (C 0) is given by the a. −24 x$ V/m b. 2.6x$ V/m c. 24 x$ V/m d. −2.6x$ V/m
following relation
3+ K 4+K 7. The maximum and minimum distance of a comet from
a. C ′ = C0 b. C ′ = C0 the Sun are 16 . × 1012 m and 80
. × 1010 m, respectively. If
4K 3
4K 4 the speed of the comet at the nearest point is
c. C ′ = C0 d. C ′ = C0 6 × 104ms −1, then the speed at the farthest point is
K+3 3+ K
. × 103 m/s
a. 15 . × 103 m/s
b. 60
3. A block of mass m slides along a floor, while a force of . × 103 m/s
c. 30 d. 4.5 × 103 m/s
magnitude F is applied to it at an angle θ as shown in
figure. The coefficient of kinetic friction is µ k. Then, the 8. A bar magnet of length 14 cm is placed in the magnetic
block's acceleration a is given by meridian with its North pole pointing towards the
(g is acceleration due to gravity) geographic North pole. A neutral point is obtained at a
distance of 18 cm from the centre of the magnet. If
F B H = 0.4 G, then the magnetic moment of the magnet is
(1 G = 10−4T)
a. 2.88 × 103 J T −1 b. 2.88 × 102 J T −1
θ c. 2.88 J T −1 d. 28.8 J T −1
9. The volume V of an enclosure contains a mixture of three
gases, 16 g of oxygen, 28 g of nitrogen and 44 g of
carbon dioxide at absolute temperature T . Consider R as
universal gas constant. The pressure of the mixture of
cos θ − µ k  g − sin θ cos θ − µ k  g − sin θ
F F F F gases is
a. − b.
m  m  m  m  88RT 3RT 5 RT 4RT
a. b. c. d.
F  F
c. cos θ − µ k  g + sin θ  d. cos θ + µ k  g − sin θ
F  F V V 2V V
m  m  m  m  10. In thermodynamics, heat and work are
4. The pressure acting on a submarine is 3 × 10 Pa at a 5 a. path functions
certain depth. If the depth is doubled, the percentage b. intensive thermodynamic state variables
increase in the pressure acting on the submarine would c. extensive thermodynamic state variables
be (Assume that atmospheric pressure is 1× 105 Pa, d. point functions
density of water is 103kg m−3, g = 10 ms −2) 11. Four equal masses, m each are placed at the corners of a
200 200 5 3 square of length (l) as shown in the figure. The moment
a. % b. % c. % d. %
3 5 200 200 of inertia of the system about an axis passing through A
5. The angle of deviation through a prism is minimum and parallel to DB would be
when D l C
m m

δ
l l

m m
A. incident ray and emergent ray are symmetric to the prism A l B
B. the refracted ray inside the prism becomes parallel to its a. 1ml 2 b. 2ml 2 c. 3ml 2 d. 3ml 2
base
18 Online JEE Main 2021 March Attempt

12. A conducting wire of length l, area of cross-section A and a(ms–2)


electric resistivity ρ is connected between the terminals
of a battery. A potential difference V is developed
between its ends, causing an electric current. If the
length of the wire of the same material is doubled and d.
the area of cross-section is halved, the resultant current 18
would be 2
1 VA 3 VA 1 ρl VA
a. b. c. d. 4 0
4 ρl 4 ρl 4 VA ρl 200 400 x(m)
13. Time period of a simple pendulum is T inside a lift, when 15. A 25 m long antenna is mounted on an antenna tower.
the lift is stationary. If the lift moves upwards with an The height of the antenna tower is 75 m. The wavelength
acceleration g/2, then the time period of pendulum will (in m) of the signal transmitted by this antenna would be
be
T a. 300 b. 400 c. 200 d. 100
a. 3T b.
3 16. For an electromagnetic wave travelling in free space, the
3 2 relation between average energy densities due to
c. T d. T
2 3 electric (U e ) and magnetic (U m ) fields is
14. The velocity-displacement graph describing the motion a. Ue = Um b. Ue > Um
of a bicycle is shown in the following figure. c. Ue < Um d. Ue ≠ Um
17. An R-C circuit as shown in the figure is driven by an AC
v(ms-1) source generating a square wave. The output wave
pattern monitored by CRO would look close to
50
R

10 C CRO

0 200 400 x(m)

The acceleration-displacement graph of the bicycle’s


motion is best described by
a(ms-2)
a.

b.
a.
18
2 c.
0 200 400 x(m)

a(ms-2) d.

18. The stopping potential in the context of photoelectric


b. effect depends on the following property of incident
18 electromagnetic radiation
a. phase b. intensity
2
c. amplitude d. frequency
0 200 400 x(m) 19. A block of 200 g mass moves with a uniform speed in a
a(ms ) -2 horizontal circular groove, with vertical side walls of radius
20 cm. If the block takes 40 s to complete one round, the
normal force by the side walls of the groove is
a. 0.0314 N b. 9.859 × 10−2 N
c. c. 6.28 × 10−3 N d. 9.859 × 10−4 N
18 20. A conducting bar of length L is free to slide on two
2 parallel conducting rails as shown in the figure
0 200 400 x(m)
March Attempt Online JEE Main 2021 19

24. The first three spectral lines of H-atom in the Balmer


v series are given λ 1 , λ 2 , λ 3 considering the Bohr atomic
model, the wavelengths of first and third spectral lines
× ×
R1
B
R2 ( λ 1 / λ 3 ) are related by a factor of approximately x × 10−1.
The value of x to the nearest integer, is ……… .
× ×
25. The value of power dissipated across the Zener diode
(Vz = 15 V) connected in the circuit as shown in the figure
is x × 10−1 W.
Two resistors R1 and R2 are connected across the ends of
Rs=35 Ω
the rails. There is a uniform magnetic field B pointing into
the page. An external agent pulls the bar to the left at a
constant speed v.
The correct statement about the directions of induced 22 V
Vz=15 V RL=90 Ω
currents I1 and I 2 flowing through R1 and R2 respectively is
a. both I1 and I 2 are in anti-clockwise direction.
b. both I1 and I 2 are in clockwise direction.
c. I1 is in clockwise direction and I 2 is in anti-clockwise The value of x, to the nearest integer, is ……… .
direction.
26. A sinusoidal voltage of peak value 250 V is applied to a
d. I1 is in anticlockwise direction and I 2 is in clockwise
direction.
series L-C-R circuit, in which R = 8 Ω, L = 24 mH and
C = 60µF. The value of power dissipated at resonant
Section B : Numerical Type Questions condition is x kW. The value of x to the nearest integer is
21. In the figure given, the electric current flowing through …… .
the 5 kΩ resistor is x mA. 27. In the logic circuit shown in the figure, if input A and B
are 0 to 1 respectively, the output at Y would be x. The
3 kΩ
value of x is …… .
5 kΩ 3 kΩ
A
3 kΩ

21 V, 1 kΩ
28. The resistance R = V /I, where V = (50 ± 2) V and
The value of x to the nearest Integer is …3… . I = (20 ± 02
. ) A. The percentage error in R is
x %. The value of x to the nearest integer is……… .
22. A fringe width of 6 mm was produced for two slits
separated by 1 mm apart. The screen is placed 10 m
29. Consider a frame that is made up of two thin massless
rods AB and AC as shown in the figure. A vertical force P
away. The wavelength of light used is x nm. The value of
of magnitude 100 N is applied at point A of the frame.
x to the nearest integer is ……… .
23. Consider a 20 kg uniform circular disc of radius 0.2 m. It
is pin supported at its centre and is at rest initially. The
disc is acted upon by a constant force F = 20 N through a A
massless string wrapped around its periphery as shown
in the figure. 70°

P B

C
145°

F = 20 N

Suppose the disc makes n number of revolutions to attain Suppose the force is P resolved parallel to the arms AB and
an angular speed of 50 rad s −1. The value of n to the AC of the frame. The magnitude of the resolved component
nearest integer, is ………. . along the arm AC is x N. The value of x, to the nearest
(Given, in one complete revolution, the disc rotates by 6.28 integer, is ……… .
rad.) [Given, sin(35° ) = 0. 573, cos(35° ) = 0.819, sin(110° ) = 0.939,
cos(110° ) = − 0. 342]
20 Online JEE Main 2021 March Attempt

30. A ball of mass 10 kg moving with a velocity 10 3 ms −1 After collision


Y
along X-axis, hits another ball of mass 20 kg, which is at
rest. After collision, the first ball comes to rest and the
second one disintegrates into two equal pieces. One of
the pieces starts moving along Y-axis at a speed of 10 X-axis
m/s. The second piece starts moving at a speed of 20 θ
m/s at an angle θ (degree) with respect to the X-axis.
The configuration of pieces after collision is shown in the
figure. The value of θ to the nearest integer is ……… .

Answers For solution scan


the QR code

1. (d) 2. (c) 3. (b) 4. (a) 5. (a) 6. (a) 7. (c) 8. (c) 9. (c) 10. (a)
11. (c) 12. (a) 13. (d) 14. (a) 15. (d) 16. (a) 17. (c) 18. (d) 19. (d) 20. (c)
21. (3) 22. (600) 23. (20) 24. (15) 25. (5) 26. (4) 27. (0) 28. (5) 29. (82) 30. (30)

16 MARCH SHIFT II
Section A : Objective Type Questions L/min ? The radius of the tap and viscosity of water are
0.5 cm and 10−3 Pa-s, respectively.
1. The following logic gate is equivalent to
(Density of water = 10 3 kg/m 3 )
A a. Unsteady to steady flow b. Remains steady flow
Y c. Remains turbulent flow d. Steady flow to unsteady flow
B . t 2$i + 3t $j + 9k$ )
5. A mosquito is moving with a velocity v = (05
a. NOR Gate b. OR Gate m/s and accelerating in uniform conditions. What will be
c. AND Gate d. NAND Gate the direction of mosquito after 2s?
a. tan −1  from X-axis b. tan −1  from Y-axis
2 2
. kg is hanging
2. A large block of wood of mass M = 599  3  3
from two long massless cords. A bullet of mass m = 10 g
c. tan −1  from Y-axis d. tan −1  from X-axis
5 5
is fired into the block and gets embedded in it. The  2  2
system (block + bullet) then swing upwards, their centre
of mass rising a vertical distance h = 98
. cm before the 6. Find out the surface charge density at the intersection of
(block + bullet) pendulum comes momentarily to rest at point X = 3 m plane and X-axis, in the region of uniform
the end of its arc. The speed of the bullet just before line charge of 8 nC/m lying along the Z-axis in free space.
collision is a. 0.424 nC m−2 b. 47.88 nC m−2
(Take g = 9.8 ms −2) c. 0.07 nC m−2 d. 4.0 nC m−2
7. The de-Broglie wavelength associated with an electron
and a proton were calculated by accelerating them
through same potential of 100 V. What should nearly be
the ratio of their wavelengths? (m p = 100727
. u,
m e = 000055
. u)
a. 1860 : 1 b. (1860) 2 : 1 c. 41.4 : 1 d. 43 : 1
h 8. For the given circuit, comment on the type of
m M transformer used.
v
Il IL 0.11 A
a. 841.4 m/s b. 811.4 m/s
c. 831.4 m/s d. 821.4 m/s L
p s O
3. A charge Q is moving dI distance in the magnetic field B . 220 V 60 W V2
A
Find the value of work done by B. D
a. 1 b. Infinite
c. Zero d. −1
a. Auxilliary transformer b. Auto transformer
4. What will be the nature of flow of water from a circular
tap, when its flow rate increased from 0.18 L/min to 0.48 c. Step-up transformer d. Step down transformer
March Attempt Online JEE Main 2021 21

9. The half-life of Au198 is 2.7 days. The activity of 1.50 mg of 16. Two identical antennas mounted on identical towers are
Au198 if its atomic weight is 198 g mol−1 is (NA = 6 × 1023 / separated from each other by a distance of 45 km. What
mol) should nearly be the minimum height of receiving
a. 240 Ci b. 357 Ci c. 535 Ci d. 252 Ci antenna to receive the signals in line of sight ?
10. Calculate the value of mean free path ( λ ) for oxygen (Assume, radius of earth is 6400 km.)
a. 19.77 m b. 39.55 m c. 79.1 m d. 158.2 m
. × 105
molecules at temperature 27°C and pressure 101
17. The magnetic field in a region is given by B = B 0   k$ . A
Pa. Assume the molecular diameter 0.3 nm and the gas x
is ideal. a
. × 10−23 JK −1)
(k = 138 square loop of side d is placed with its edges along the X
a. 58 nm b. 32 nm c. 86 nm d. 102 nm and Y-axes. The loop is moved with a constant velocity
v = v 0i$. The emf induced in the loop is
11. The refractive index of a converging lens is 1.4. What will
be the focal length of this lens if it is placed in a medium Y
of same refractive index ? (Assume the radii of curvature Z
of the faces of lens are R1 and R 2 respectively)
R1R 2 d
a. 1 b. Infinite c. d. Zero
R1 − R 2 d
X
12. In order to determine the Young's modulus of a wire of B v 2d Bv d B v d2 B0v 0d 2
radius 0.2 cm (measured using a scale of least count a. 0 0 b. 0 0 c. 0 0 d.
2a 2a a 2a
= 0001
. cm) and length 1m (measured using a scale of
least count = 1mm), a weight of mass 1kg (measured 18. Amplitude of a mass-spring system, which is executing
using a scale of least count = 1g) was hanged to get the simple harmonic motion decreases with time. If mass =
elongation of 0.5 cm (measured using a scale of least 500 g, decay constant = 20 g/s, then how much time is
count 0.001 cm). What will be the fractional error in the required for the amplitude of the system to drop to half
value of Young's modulus determined by this experiment? of its initial value ? (ln 2 = 0.693)
a. 0.14% b. 0.9% c. 9% d. 1.4% a. 34.65 s b. 17.32 s c. 0.034 s d. 15.01 s
13. A bimetallic strip consists of metals A and B. It is
mounted rigidly as shown. The metal A has higher 19. Calculate the time interval between 33% decay and 67%
coefficient of expansion compared to that of metal B. decay if half-life of a substance is 20 min.
When the bimetallic strip is placed in a cold bath, it will a. 60 min b. 20 min c. 40 min d. 13 min

20. Red light differs from blue light as they have


a. different frequencies and different wavelengths
b. different frequencies and same wavelengths
A B
c. same frequencies and same wavelengths
d. same frequencies and different wavelengths

Section B : Numerical Type Questions

a. bend towards the right b. not bend but shrink


21. The energy dissipated by a resistor is 10 mJ in 1s when
an electric current of 2 mA flows through it. The
c. Neither bend nor shrink d. bend towards the left
resistance is ……… Ω.
14. A resistor develops 500 J of thermal energy in 20 s, when (Round off to the nearest integer)
a current of 1.5 A is passed through it. If the current is
increased from 1.5 A to 3 A, what will be the energy 22. In a parallel plate capacitor set up, the plate area of
developed in 20 s? capacitor is 2 m2 and the plates are separated by 1m. If
a. 1500 J b. 1000 J c. 500 J d. 2000 J the space between the plates are filled with a dielectric
material of thickness 0.5 m and area 2 m2 (see figure) the
15. Statement I A cyclist is moving on an unbanked road capacitance of the set-up will be …… ε 0.
with a speed of 7 kmh−1 and takes a sharp circular turn (Dielectric constant of the material = 3.2)
along a path of radius of 2 m without reducing the
speed. The static friction coefficient is 0.2. The cyclist will (Round off to the nearest integer)
not slip and pass the curve ( g = 9.8 m / s 2) d
Statement II If the road is banked at an angle of 45°,
cyclist can cross the curve of 2 m radius with the speed of
18.5 kmh−1 without slipping.
In the light of the above statements, choose the correct K
answer from the options given below.
a. Statement I is false and statement II is true.
b. Statement I is true and statement II is false.
c. Both statement I and statement II are false. d/2
d. Both statement I and statement II are true.
22 Online JEE Main 2021 March Attempt

23. A force F = 4 i$ + 3$j + 4k$ is applied on an intersection other bank just opposite to his starting point is ………° (in
point of x = 2 plane and X-axis. The magnitude of torque degree).
of this force about a point (2, 3, 4) is …… . (Round off to the nearest integer)
(Round off to the nearest integer) 28. A closed organ pipe of length L and an open organ pipe
contain gases of densities ρ1 and ρ2 respectively. The
24. If one wants to remove all the mass of the earth to
compressibility of gases are equal in both the pipes.
infinity in order to break it up completely. The amount of
2 Both the pipes are vibrating in their first overtone with
energy that needs to be supplied will be x GM , where x x ρ
5 R same frequency. The length of the open pipe is L 1
3 ρ2
is ……… .
(Round off to the nearest integer) where x is …… .
(M is the mass of earth, R is the radius of earth and G is the (Round off to the nearest integer)
gravitational constant.) 29. A solid disc of radius a and mass m rolls down without
25. A deviation of 2° is produced in the yellow ray when slipping on an inclined plane making an angle θ with the
2
prism of crown and flint glass are achromatically horizontal. The acceleration of the disc will begsinθ,
combined. Taking dispersive powers of crown and flint b
glass are 0.02 and 0.03 respectively and refractive index where b is ……… .
for yellow light for these glasses are 1.5 and 1.6, (Round off to the nearest integer)
respectively. The refracting angles for crown glass prism (g = acceleration due to gravity)
will be ……° (in degree). (θ = angle as shown in figure)
(Round off to the nearest integer)
26. A body of mass 2 kg moves under a force of a
(2i$ + 3$j + 5k$ )N. It starts from rest and was at the origin P
initially. After 4 s, its new coordinates are (8, b, 20). The
value of b is ……… .
(Round off to the nearest integer) θ
27. A swimmer can swim with velocity of 12 km/h in still 30. For an ideal heat engine, the temperature of the source
water. Water flowing in a river has velocity 6 km/h. The is 127°C. In order to have 60% efficiency the temperature
direction with respect to the direction of flow of river of the sink should be …… °C.
water he should swim in order to reach the point on the (Round off to the nearest integer)

Answers For solution scan


the QR code
1. (a) 2. (c) 3. (c) 4. (d) 5. (*) 6. (a) 7. (d) 8. (c) 9. (b) 10. (d)
11. (b) 12. (d) 13. (d) 14. (d) 15. (d) 16. (b) 17. (c) 18. (a) 19. (b) 20. (a)
21. (2500) 22. (3) 23. (20) 24. (3) 25. (12) 26. (12) 27. (120) 28. (4) 29. (3) 30. (113)

17 MARCH SHIFT I
Section A : Objective Type Questions a. −15 − 20 3 , 15 − 20 3 b. 15 + 20 3 , 15 − 20 3
1. A triangular plate is shown below. A force F = 4 i$ − 3$j is c. 15 − 20 3 , 15 + 20 3 d. −15 + 20 3 , 15 + 20 3

applied at point P. The torque at point P with respect to 2. When two soap bubbles of radii a and b( b > a) coalesce,
point O and Q are the radius of curvature of common surface is
Y ab a+ b b −a ab
a. b. c. d.
b −a ab ab a+ b

3. A polyatomic ideal gas has 24 vibrational modes. What is


F P the value of γ?
a. 1.03 b. 1.30
c. 1.37 d. 10.3
10
cm

cm

4. If an electron is moving in the nth orbit of the hydrogen


10

atom, then its velocity v n for the nth orbit is given as


60° 60° 1 1
X a. v n ∝ n b. v n ∝ c. v n ∝ n 2 d. v n ∝
O 10 cm Q n n2
March Attempt Online JEE Main 2021 23

5. An electron of mass m and a photon have same energy 11. A solenoid of 1000 turns per metre has a core with
E. The ratio of wavelength of electron to that of photon is relative permeability 500. Insulated windings of the
(c being the velocity of light) solenoid carry an electric current of 5 A. The magnetic
1/ 2 1/ 2
flux density produced by the solenoid is
a. 
1 2m 
b. 
1 E 
 
c E  c  2m  (Permeability of free space = 4 π × 10−7 H/m)
π
1/ 2 a. πT b. 2 × 10−3 πT c. T d. 10−4 πT
c. 
E 
 d. c (2mE )1/ 2 5
 2m 
12. A mass M hangs on a massless z ω
6. Two identical metal wires of thermal conductivities K1 rod of length l which rotates at
and K 2 respectively are connected in series. The effective a constant angular frequency. B
α
thermal conductivity of the combination is The mass M moves with steady
2K 1K 2 K1 + K 2
a. b. speed in a circular path of I
K1 + K 2 2K 1K 2 constant radius. Assume that
K1 + K 2 K K the system is in steady circular
c. d. 1 2
K 1K 2 K1 + K 2 motion with constant angular
7. The vernier scale used for measurement has a positive velocity ω. The angular
zero error of 0.2 mm. If while taking a measurement, it momentum of M about point A
is LA which lies in the positive A r M
was noted that '0' on the vernier scale lies between 8.5
cm and 8.6 cm, vernier coincidence is 6, then the correct z-direction and the angular
value of measurement is ……… cm. momentum of M about B is LB .
a. 8.36 b. 8.54 The correct statement for this
c. 8.58 d. 8.56 system is
8. An AC current is given by I = I1sinωt + I2 cos ωt. A hot wire a. LA and L B are both constant in magnitude and direction
b. LB is constant in direction with varying magnitude
ammeter will give a reading c. LB is constant, both in magnitude and direction
I12 − I 22 I12 + I 22 d. LA is constant, both in magnitude and direction
a. b.
2 2 13. For what value of displacement the kinetic energy and
I1 + I 2 I1 + I 2 potential energy of a simple harmonic oscillation
c. d.
2 2 2 become equal?
A A
9. A modern grand-prix racing car of mass m is travelling on a. x = 0 b. x = ± A c. x = ± d. x =
2 2
a flat track in a circular arc of radius R with a speed v. If
the coefficient of static friction between the tyres and 14. A Carnot's engine working between 400 K and 800 K has
the track is µ s , then the magnitude of negative lift f L a work output of 1200 J per cycle. The amount of heat
acting downwards on the car is energy supplied to the engine from the source in each
(Assume forces on the four tyres are identical and g = cycle is
acceleration due to gravity) a. 3200 J b. 1800 J c. 1600 J d. 2400 J
v 15. The thickness at the centre of a plano convex lens is 3
mm and the diameter is 6 cm. If the speed of light in the
material of the lens is 2 × 108ms −1, then the focal length
of the lens is
a. 0.30 cm b. 15 cm c. 1.5 cm d. 30 cm
R
16. The output of the given combination gates represents
A
Y
B
 v2   v2 
a. m  + g b. m  − g a. XOR gate b. NAND gate
µ
 s R  µ
 s R  c. AND gate d. NOR gate
 v2   v2 
c. m  g −  d. − m  g +  17. A boy is rolling a 0.5 kg ball on the frictionless floor with
 µ sR   µ sR  the speed of 20 ms −1. The ball gets deflected by an
10. A car accelerates from rest at a constant rate α for some obstacle on the way. After deflection it moves with 5% of
time after which it decelerates at a constant rate β to its initial kinetic energy. What is the speed of the ball
come to rest. If the total time elapsed is t seconds, the now ?
total distance travelled is a. 19.0 ms −1 b. 4.47 ms −1 c. 14.41 ms −1 d. 1.00 ms −1
4αβ 2 2αβ 2 αβ αβ 18. Which level of the single ionized carbon has the same
a. t b. t c. t2 d. t2
(α + β) (α + β) 2(α + β) 4(α + β) energy as the ground state energy of hydrogen atom?
a. 1 b. 6 c. 4 d. 8
24 Online JEE Main 2021 March Attempt

19. Two ideal polyatomic gases at temperatures T1 and T2 are (Round off to the nearest integer)
mixed so that there is no loss of energy. If f1 and f 2, m1
and m 2 , n1 and n2 be the degrees of freedom, masses,
number of molecules of the first and second gas
respectively, the temperature of mixture of these two
gases is Ta Tb
n T + n 2T2 n f T + n 2 f 2T2
a. 1 1 b. 1 1 1 M
n1 + n 2 n1 f1 + n 2 f 2
Fig. 1
n1 f1T1 + n 2 f 2T2 n1 f1T1 + n 2 f 2T2
c. d.
f1 + f 2 n1 + n 2

20. A current of 10 A exists in a wire of cross sectional area M


of 5 mm2 with a drift velocity of 2 × 10−3 ms −1. The number Fig. 2
of free electrons in each cubic metre of the wire is
a. 2 × 106 b. 625 × 1025 c. 2 × 1025 d. 1 × 1023 27. The following bodies,
1. a ring 2. a disc
Section B : Numerical Type Questions 3. a solid cylinder 4. a solid sphere
of same mass m and radius R are allowed to roll down
21. For VHF signal broadcasting, ……… km 2 of maximum without slipping simultaneously from the top of the
service area will be covered by an antenna tower of
inclined plane. The body which will reach first at the
height 30 m, if the receiving antenna is placed at ground.
bottom of the inclined plane is ……… .
Let radius of the Earth be 6400 km.
(Round off to the nearest integer). (Take π as 3.14) (Mark the body as per their respective numbering given
in the question)
22. The angular speed of truck wheel is increased from 900
rpm to 2460 rpm in
26 s. The number of revolutions by the truck engine
during this time is ……… .
(Assuming the acceleration to be uniform).
23. The equivalent resistance of series combination of two S h
resistors is s. When they are connected in parallel, the
equivalent resistance is p. If s = np, then the minimum
value for n is ……… .
(Round off to the nearest integer) θ

24. Four identical rectangular plates with length, l = 2 cm and 28. A parallel plate capacitor whose capacitance C is 14 pF is
breadth, b = 3 / 2 cm are arranged as shown in figure. charged by a battery to a potential difference V = 12 V

The equivalent capacitance between A and C is 0 . The between its plates. The charging battery is now
d disconnected and a porcelain plate with
value of x is ……… . K = 7 is inserted between the plates, then the plate would
(Round off to the nearest integer) oscillate back and forth between the plates with a
constant mechanical energy of ……… pJ.
(Assume no friction)
A B C
D 29. Two blocks ( m = 0.5 kg and M = 4 .5 kg) are arranged on a
horizontal frictionless table as shown in figure. The
coefficient of static friction between the two blocks is
3/7. Then, the maximum horizontal force that can be
applied on the larger block so that the blocks move
together is …… N.
d d d (Round off to the nearest integer. Take, g = 9.8 ms −2)

25. The radius in kilometre to which the present radius of m


Earth (R = 6400 km) to be compressed so that the escape F
M
velocity is increased 10 times is ……… .

26. Consider two identical springs each of spring constant k 30. If 2.5 × 10−6 N average force is exerted by a light wave on
and negligible mass compared to the mass M as shown. a non-reflecting surface of 30 cm2 area during 40 min of
Fig.1 shows one of them and Fig.2 shows their series time span, the energy flux of light just before it falls on
combination. The ratios of time period of oscillation of the surface is …… W / cm2.
T
the two SHM is b = x , where value of x is ……… . (Round off to the nearest integer. Assume complete
Ta
absorption and normal incidence conditions are there.)
March Attempt Online JEE Main 2021 25

Answers For solution scan


the QR code

1. (b) 2. (a) 3. (a) 4. (b) 5. (b) 6. (a) 7. (b) 8. (b) 9. (b) 10. (c)
11. (a) 12. (d) 13. (c) 14. (d) 15. (d) 16. (b) 17. (b) 18. (b) 19. (b) 20. (b)
21. (1206) 22. (728) 23. (4) 24. (2) 25. (64) 26. (2) 27. (4) 28. (864) 29. (21) 30. (25)

17 MARCH SHIFT II
Section A : Objective Type Questions a. y (x , t ) = 003
. [sin 51
. x– . × 103 )t ]
(02
1. A rubber ball is released from a height of 5 m above the b. y (x , t ) = 006
. [sin 51
. x– . × 103 )t ]
(15
floor. It bounces back repeatedly, always rising to 81/100 c. y (x , t ) = 006
. [sin 08
. x– . × 103 )t ]
(05
of the height through which it falls. Find the average d. y (x , t ) = 003
. [sin 51
. x– . × 103 )t ]
(15
speed of the ball. 8. Which one is the correct option for the two different
(Take, g = 10 ms −2 ) thermodynamic processes?
a. 3.0 ms −1 b. 3.5 ms −1 c. 2.0 ms −1 d. 2.5 ms −1
2. If one mole of the polyatomic gas is having two Adiabatic
vibrational modes and β is the ratio of molar specific p
 C p
heats for polyatomic gas  β =  , then the value of β is (A)
 CV  Isothermal
a. 1.02 b. 1.2 c. 1.25 d. 1.35
3. A block of mass 1 kg attached to a spring is made to V
oscillate with an initial amplitude of 12 cm. After 2 min,
the amplitude decreases to 6 cm. Determine the value of Isothermal
the damping constant for this motion. p
(Take, ln 2 = 0693
. )
. × 102 kg/s
a. 069 b. 3.3 × 102 kg/s (B)
. × 10−2 kg/s
c. 116 d. 5.7 × 10−3 kg/s
Adiabatic
4. Which one of the following will be the output of the given
circuit ? T

A
Y V Isothermal

B (C)
Adiabatic
a. NOR Gate b. NAND Gate c. AND Gate d. XOR Gate
5. An object is located at 2 km beneath the surface of the
water. If the fractional compression ∆V / V is 1.36% , the T
ratio of hydraulic stress to the corresponding hydraulic Isothermal
strain will be …………… . p
(Take, density of water is 1000 kg m −3 and g = 981
. ms −2 ) (D)
. × 107 Nm−2
a. 196 . × 107 Nm−2
b. 144
. × 109 Nm−2
c. 226 . × 109 Nm−2
d. 144 Adiabatic

6. A geostationary satellite is orbiting around an arbitrary


planet P at a height of 11R above the surface of P, R being T
the radius of P. The time period of another satellite in a. C and A b. C and D
hours at a height of 2R from the surface of P is ………… . c. Only A d. B and C
P has the time period of 24 h.
a. 6 2 b. 6/ 2 c. 3 d. 5
9. The velocity of a particle is v = v 0 + gt + Ft 2. Its position is
x = 0 at t = 0, then its displacement after time (t = 1) is
7. A sound wave of frequency 245 Hz travels with the speed g F
of 300 ms −1 along the positive X-axis. Each point of the a. v 0 + g + F b. v 0 + +
2 3
wave moves to and fro through a total distance of 6 cm. g
c. v 0 + +F d. v 0 + 2 g + 3F
What will be the mathematical expression of this 2
travelling wave ?
26 Online JEE Main 2021 March Attempt

10. A carrier signal C (t) = 25sin(2512


. × 1010t) is amplitude 15. Match List-I with List-II
. × 108t)
modulated by a message signal m(t) = 5sin(157 List-I List-II
and transmitted through an antenna. What will be the
bandwidth of the modulated signal ? A. Phase difference between 1. π
; current leads voltage
a. 8 GHz b. 2.01 GHz current and voltage in a 2
c. 1987.5 MHz d. 50 MHz purely resistive AC circuit

11. Two cells of emf 2E and E with internal resistance r1 and B. Phase difference between 2. zero
current and voltage in a
r2 respectively are connected in series to an external
pure inductive AC circuit
resistor R (see figure). The value of R, at which the
potential difference across the terminals of the first cell C. Phase difference between 3. π
; current lags voltage
becomes zero is current and voltage in a 2
2E E pure capacitive AC circuit

D. Phase difference between 4. tan −1 XC − X L 


 
current and voltage in an  R 
L-C-R series circuit

Choose the most appropriate answer from the options


given below.
R A B C D
r r a. 1 3 4 2
a. r1 + r2 b. 1 − r2 c. 1 + r2 d. r1 − r2
2 2 b. 2 4 3 1
c. 2 3 4 1
12. A hairpin like shape as shown in figure is made by d. 2 3 1 4
bending a long current carrying wire. What is the
magnitude of a magnetic field at point P which lies on the 16. Two identical blocks A and B each of mass m resting on
centre of the semicircle? the smooth horizontal floor are connected by a light
spring of natural length L and spring constant k. A third
I block C of mass m moving with a speed v along the line
joining A and B collides with A. The maximum
r compression in the spring is
P I
C A B
I m m m
µ I µ I µ I µ I
a. 0 (2 − π ) b. 0 (2 + π ) c. 0 (2 + π ) d. 0 (2 − π ) m mv
4 πr 4 πr 2 πr 2 πr a. v b.
2k 2k
13. The four arms of a Wheatstone bridge have resistances mv m
as shown in the figure. A galvanometer of 15 Ω c. d.
k 2k
resistance is connected across BD. Calculate the current
through the galvanometer when a potential difference of 17. The atomic hydrogen emits a line spectrum consisting of
10 V is maintained across AC . various series. Which series of hydrogen atomic spectra
is lying in the visible region ?
B
Ω 10 a. Brackett series b. Paschen series
0 Ω c. Lyman series d. Balmer series
10
A C
G
18. Two identical photocathodes receive the light of
60 frequencies f1 and f 2, respectively. If the velocities of the
Ω Ω
5 photoelectrons coming out are v1 and v 2 respectively,
D then
2h 2h
a. v12 − v 22 = [ f1 − f 2 ] b. v12 + v 22 = [ f1 + f 2 ]
m m
1 1

c. v1 − v 2 =  ( f1 + f 2 )  d. v1 − v 2 =  ( f1− f 2 ) 
2h 2 2h 2
10V
 m   m 
a. 2.44 µA b. 2.44 mA
19. What happens to the inductive reactance and the
c. 4.87 mA d. 4.87 µA
current in a purely inductive circuit, if the frequency is
14. Two particles A and B of equal masses are suspended halved ?
from two massless springs of spring constants k1 and k 2, a. Both inductive reactance and current will be halved.
respectively. If the maximum velocities during b. Inductive reactance will be halved and current will be
oscillations are equal, the ratio of the amplitude of A and doubled.
B is c. Inductive reactance will be doubled and current will be
k2 k1 k1 k2 halved.
a. b. c. d. d. Both inductive reactance and current will be doubled.
k1 k2 k2 k1
March Attempt Online JEE Main 2021 27

20. A sphere of mass 2 kg and radius 0.5 m is rolling with an 2 3


−1
26. The electric field in a region is given by E = E 0 $i + E 0$j
initial speed of 1 ms goes up an inclined plane which 5 5
makes an angle of 30° with the horizontal plane, without with E 0 = 4.0 × 103 N/C. The flux of this field through a
slipping. How long will the sphere take to return to the rectangular surface area 0.4 m 2 parallel to the
starting point A ? yz-plane is ………… N-m 2 C −1.

27. The disc of mass M with uniform surface mass density σ


is shown in the figure. The centre of mass of the quarter
xa x a
disc (the shaded area) is at the position , , where
3π 3 π
x is ………… .
30° (Round off to the nearest integer)
A
(a is an area as shown in the figure)
a. 0.60 s b. 0.52 s c. 0.57 s d. 0.80 s
Y
Section B : Numerical Type Questions
21. The electric field intensity produced by the radiation a
coming from a 100 W bulb at a distance of 3 m is E. The
electric field intensity produced by the radiation coming θ
x X
from 60 W at the same distance is E , where the value
5
of x is …………… .
22. A body of mass 1 kg rests on a horizontal floor with
1
. It is
which it has a coefficient of static friction
3 28. The image of an object placed in air formed by a convex
desired to make the body move by applying the refracting surface is at a distance of 10 m behind the
minimum possible force F newton. The value of F will be surface. The image is real and is at 2/3 of the distance of
……………… .
the object from the surface .The wavelength of light
(Round off to the nearest integer)
inside the surface is 2/3 times the wavelength in air. The
(Take, g = 10 ms −2 ) x
radius of the curved surface is m. The value of x is
23. A boy of mass 4 kg is standing on a piece of wood having 13
mass 5 kg . If the coefficient of friction between the wood ……………… .
and the floor is 0.5, the maximum force that the boy can
29. A 2 µF capacitor C 1 is first charged to a potential
exert on the rope, so that the piece of wood does not
move from its place is …………… N. (Round off to the difference of 10 V using a battery. Then, the battery is
nearest integer) (Take, g = 10 ms −2) removed and the capacitor is connected to an
uncharged capacitor C 2 of 8 µF. The charge in C 2 on
equilibrium condition is ………… µC.
T
(Round off to the nearest integer)
T S1 S2
f
R T

T 2 µF 8 µF
10V C1 C2

24. Suppose you have taken a dilute solution of oleic acid in


such a way that its concentration becomes 0.01 cm 3 of 30. Seawater at a frequency f = 9 × 102 Hz, has permittivity
oleic acid per cm 3 of the solution. Then, you make a thin ε = 80ε 0 and resistivity r = 0 . 25 Ω-m. Imagine a parallel
film of this solution (monomolecular thickness) of area 4
plate capacitor is immersed in seawater and is driven by
cm 2 by considering
1 an alternating voltage source V (t) = V0 sin(2πft). Then, the
 3 3 −3 2 conduction current density becomes 10x times the
100 spherical drops of radius   × 10 cm . Then,
 40π  1
displacement current density after timet = s. The
the thickness of oleic acid layer will be x × 10–14 m, 800
where x is ……………… . value of x is …………… .
 1 
25. A particle of mass m moves in a circular orbit in a central  Take, = 9 × 109N - m2C −2
potential field U( r) = U 0r 4. If Bohr's quantisation  4 πε 0 
conditions are applied, radii of possible orbitals rn vary
with n1/ α , where α is ………………… .
28 Online JEE Main 2021 March Attempt

Answers For solution scan


the QR code

1. (d) 2. (b) 3. (c) 4. (d) 5. (d) 6. (c) 7. (d) 8. (b) 9. (b) 10. (d)
11. (b) 12. (b) 13. (c) 14. (d) 15. (d) 16. (a) 17. (d) 18. (a) 19. (b) 20. (c)
21. (3) 22. (5) 23. (30) 24. (25) 25. (3) 26. (640) 27. (4) 28. (30) 29. (16) 30. (6)

18 MARCH SHIFT I
Section A : Objective Type Questions B

1. An oil drop of radius 2 mm with a density A C


3 g cm−3 is held stationary under a constant electric field i
. × 105 Vm −1 in the Millikan’s oil drop experiment.
355
What is the number of excess electrons that the oil drop D
. m/s 2)
will possess? (Take, g = 981
a. 12T b. 6T c. 9T d. 1T
a. 48.8 × 1011 b. 1.73 × 1010 c. 17.3 × 1010 d. 1.73 × 1012
2. Match List-I with List-II. 7. The time period of a simple pendulum is given by
T = 2π l / g . The measured value of the length of
List-I List-II pendulum is 10 cm known to a 1mm accuracy. The time
A. 10 km height over Earth’s (i) Thermosphere for 200 oscillations of the pendulum is found to be 100 s
surface using a clock of 1s resolution. The percentage accuracy
in the determination of g using this pendulum is x. The
B. 70 km height over Earth’s (ii) Mesosphere value of x to the nearest integer is
surface
a. 2% b. 3% c. 5% d. 4%
C. 180 km height over Earth’s (iii) Stratosphere 8. A constant power delivering machine has towed a box,
surface
which was initially at rest, along a horizontal straight line.
D. 270 km height over Earth’s (iv) Troposphere The distance moved by the box in time t is proportional
surface to
a. t 2 / 3 b. t 3 / 2 c. t d. t 1/ 2
A B C D
a. (iv) (iii) (ii) (i) b. (i) (iv) (iii) (ii) 9. What will be the average value of energy along one
c. (iii) (ii) (i) (iv) d. (ii) (i) (iv) (iii) degree of freedom for an ideal gas in thermal equilibrium
at a temperature T ? (k B is Boltzmann constant)
3. Imagine that the electron in a hydrogen atom is replaced
1 2 3
by a muon (µ). The mass of muon particle is 207 times a. kBT b. kBT c. kBT d. kBT
that of an electron and charge is equal to the charge of 2 3 2
an electron. The ionisation potential of this hydrogen 10. A radioactive sample disintegrates via two independent
atom will be decay processes having half-lives T1(/12) and T1(/22) ,
a. 13.6 eV b. 2815.2 eV c. 331.2 eV d. 27.2 eV respectively. The effective half-life T1/ 2 of the nuclei is
4. A plane electromagnetic wave of frequency 100 MHz is T1(/12) + T1(/22)
a. T1/ 2 = b. T1/ 2 = T1(/12) + T1(/22)
travelling in vacuum along the x-direction. At a particular T1(/12) − T1(/22)
. × 10−8 kT
point in space and time, B = 20 $ (where, k$ is unit
T1(/12) T1(/22)
vector along z-direction). What is E at this point? c. T1/ 2 = d. None of these
T1(/12) + T1(/22)
a. 0.6 $j V/m b. 6.0 k$ V/m c. 6.0 $j V/m d. 0.6 k$ V/m
11. The p-V diagram of a diatomic ideal gas system going
5. A thin circular ring of mass M and radius r is rotating under cyclic process as shown in figure. The work done
about its axis with an angular speed ω. Two particles during an adiabatic process CD is (use, γ = 14
. )
having mass m each are now attached at diametrically
opposite points. The angular speed of the ring will A D
200N/m2
become
M M + 2m M M − 2m 100N/m2 C
a. ω b. ω c. ω d. ω B
M+ m M M + 2m M + 2m
p
6. Four identical long solenoids A, B, C and D are connected
to each other as shown in the figure. If the magnetic field
at the centre of A is 3T, the field at the centre of C would 1 3 4
be (Assume that, the magnetic field is confined with in V(m3)
the volume of respective solenoid) a. − 500 J b. − 400 J c. 400 J d. 200 J
March Attempt Online JEE Main 2021 29

12. In Young’s double slit arrangement, slits are separated 18. Your friend is having eye sight problem. She is not able
by a gap of 0.5 mm, and the screen is placed at a to see clearly a distant uniform window mesh and it
distance of 0.5 m from them. The distance between the appears to her as non-uniform and distorted. The doctor
first and the third bright fringe formed when the slits are diagnosed the problem as
illuminated by a monochromatic light of 5890 Å is a. astigmatism
a. 1178 × 10−9 m b. 1178 × 10−6 m b. myopia with astigmatism
c. 1178 × 10−12 m d. 5890 × 10−7 m c. presbyopia with astigmatism
d. myopia and hypermetropia
13. A particle is travelling 4 times as fast as an electron.
Assuming the ratio of de-Broglie wavelength of a particle 19. A loop of flexible wire of irregular shape carrying current
to that of electron is 2 : 1, the mass of the particle is is placed in an external magnetic field.
1
a. times the mass of electron Identify the effect of the field on the wire.
16
a. Loop assumes circular shape with its plane normal to the
b. 8 times the mass of electron field.
c. 16 times the mass of electron b. Loop assumes circular shape with its plane parallel to the
1 field.
d. times the mass of electron
8 c. Wire gets stretched to become straight.
14. The position, velocity and acceleration of a particle moving d. Shape of the loop remains unchanged.
with a constant acceleration can be represented by
20. The time period of a satellite in a circular orbit of radius R
is T . The period of another satellite in a circular orbit of
Acceleration
Position

Velocity

x(t) v(t) a(t) radius 9R is


a.
a. 9 T b. 27 T
c. 12 T d. 3 T

t t t Section B : Numerical Type Questions


Acceleration

21. A particle performs simple harmonic motion with a


Position

Velocity

x(t) v(t) a(t)


period of 2 s. The time taken by the particle to cover a
b. displacement equal to half of its amplitude from the
mean position is 1/ a s. The value of a to the nearest
integer is ……… .
t t t
22. The circuit shown in the figure consists of a charged
Acceleration

capacitor of capacity 3 µF and a charge of 30 µC. At time


Position

x(t) v(t) a(t)


Velocity

c. t = 0, when the key is closed, the value of current flowing


through the 5 MΩ resistor is x µA. The value of x to the
nearest integer is ……… .
t t t
C = 3µF 5MΩ
Acceleration
Position

Velocity

x(t) v(t) a(t)


q = 30µC
d.

t t t

15. In the experiment of Ohm’s law, a potential difference of 23. The voltage across the 10Ω resistor in the given circuit is
5.0 V is applied across the end of a conductor of length x volt.
10.0 cm and diameter of 5.00 mm. The measured current 50Ω
in the conductor is 2.00 A. The maximum permissible
percentage error in the resistivity of the conductor is 10Ω
20Ω
a. 3.9 b. 8.4 c. 7.5 d. 3.0
16. In a series L-C-R resonance circuit, if we change the
170 V
resistance only, from a lower to higher value,
a. the bandwidth of resonance circuit will increase
b. the resonance frequency will increase
The value of x to the nearest integer is …… .
c. the quality factor will increase
d. the quality factor and the resonance frequency will remain 24. Two separate wires A and B are stretched by 2 mm and 4
constant mm respectively, when they are subjected to a force of 2
N. Assume that both the wires are made up of same
17. An AC source rated 220 V, 50 Hz is connected to a material and the radius of wire B is 4 times that of the
resistor. The time taken by the current to change from
radius of wire A. The length of the wires A and B are in
its maximum to the rms value is
the ratio of a : b. Then, a / b can be expressed as 1/ x ,
a. 2.5 ms b. 25 ms c. 2.5 s d. 0.25 ms
where x is ……… .
30 Online JEE Main 2021 March Attempt

25. A person is swimming with a speed of 10 m/s at an angle 28. As shown in the figure, a particle of mass 10 kg is placed
of 120° with the flow and reaches to a point directly at a point A. When the particle is slightly displaced to its
opposite on the other side of the river. The speed of the right, it starts moving and reaches the point B. The speed
flow is x m/s. The value of x to the nearest integer of the particle at B is x m/s. (Take, g = 10 m/s 2)
is ……… .
The value of x to the nearest integer is ….… .
26. A parallel plate capacitor has plate area 100 m2 and
A C
plate separation of 10 m. The space between the plates
is filled upto a thickness 5 m with a material of dielectric
constant of 10. The resultant capacitance of the system B
Horizontal
. × 10−12 fm −1.
is x pF. The value of ε 0 = 885 surface
The value of x to the nearest integer is …… . 10 m
5m
27. A ball of mass 10 kg moving with a velocity 10 3 m/s
along the X-axis, hits another ball of mass 20 kg which is
at rest. After the collision, first ball comes to rest while 29. An n-p-n transistor operates as a common emitter
the second ball disintegrates into two equal pieces. One
piece starts moving along Y-axis with a speed of 10 m/s. amplifier with a power gain of 10 6. The input circuit
resistance is 100Ω and the output load resistance is 10 k
The second piece starts moving at an angle of 30° with Ω. The common emitter current gain β will be ......... .
respect to the X-axis. The velocity of the ball moving at
(Round off to the nearest integer)
30° with X-axis is x m/s. The configuration of pieces after
collision is shown in the figure below. The value of x to 30. A bullet of mass 0.1 kg is fired on a wooden block to
the nearest integer is .......... . pierce through it, but it stops after moving a distance of
50 cm into it. If the velocity of bullet before hitting the
Y-axis Piece-1 wood is 10 m/s and it slows down with uniform
deceleration, then the magnitude of effective retarding
v1 = 10 m/s force on the bullet is x N. The value of x to the nearest
integer is ......... .
X-axis
30°

v2

Answers For solution scan


the QR code

1. (b) 2. (a) 3. (b) 4. (c) 5. (c) 6. (d) 7. (b) 8. (b) 9. (a) 10. (c)
11. (a) 12. (b) 13. (d) 14. (b) 15. (a) 16. (a) 17. (a) 18. (b) 19. (a) 20. (b)
21. (6) 22. (2) 23. (70) 24. (32) 25. (5) 26. (161) 27. (20) 28. (10) 29. (100) 30. (10)

18 MARCH SHIFT II
Section A : Objective Type Questions 2. An object of mass m1 collides with another object of
1. Which of the following statements are correct? mass m 2, which is at rest. After the collision, the objects
A. Electric monopoles do not exist, whereas magnetic move with equal speeds in opposite direction. The ratio
monopoles exist. of the masses m 2 : m1 is
B. Magnetic field lines due to a solenoid at its ends and a. 3 : 1 b. 2 : 1 c. 1 : 2 d. 1 : 1
outside cannot be completely straight and confined. 3. For an adiabatic expansion of an ideal gas, the fractional
C. Magnetic field lines are completely confined within a change in its pressure is equal to (where, γ is the ratio of
toroid. specific heats)
D. Magnetic field lines inside a bar magnet are not parallel. dV V 1 dV dV
a. − γ b. − γ c. − d.
E. χ = − 1is the condition for a perfect diamagnetic material, V dV γ V V
where χ is its magnetic susceptibility.
4. A proton and an α-particle, having kinetic energies K p and
Choose the correct answer from the options given below.
a. C and E b. B and D K α , respectively, enter into a magnetic field at right angles.
c. A and B d. B and C The ratio of the radii of trajectory of proton to that of
α-particle is 2 : 1. The ratio of K p : K α is
a. 1: 8 b. 8 : 1 c. 1 : 4 d. 4 : 1
March Attempt Online JEE Main 2021 31

5. A plane electromagnetic wave propagating along 11. The decay of a proton to neutron is
y-direction can have the following pair of electric field (E) a. not possible as proton mass is less than the neutron mass
and magnetic field (B) components. b. possible only inside the nucleus
a. E y , B y or E z , Bz b. E y , Bx or E x , B y c. not possible but neutron to proton conversion is possible
c. E x , Bz or E z , Bx d. E x , B y or E y , Bx d. always possible as it is associated only with β + decay

6. Consider a uniform wire of mass M and length L. It is bent 12. In a series L-C -R circuit, the inductive reactance ( X L) is 10
into a semicircle. Its moment of inertia about a line Ω and the capacitive reactance ( X C ) is 4 Ω. The resistance
perpendicular to the plane of the wire passing through (R) in the circuit is 6 Ω. The power factor of the circuit is
the centre is a.
1
b.
1
c.
1
d.
3
1 ML2 2 ML2 ML2 1 ML2 2 2 2 2 2
a. b. c. d.
4 π2 5 π2 π2 2 π2
13. The angular momentum of a planet of mass M moving
7. The velocity-displacement graph of a particle is shown in around the Sun in an elliptical orbit is L . The magnitude
the figure. of the areal velocity of the planet is
4L L 2L L
v a. b. c. d.
M M M 2M
v0
14. The function of time representing a simple harmonic
π
motion with a period of is
ω
a. sin (ωt ) = cos (ωt ) b. cos (ωt ) + cos (2ωt ) + cos (3ωt )
π
O x0
x c. sin 2 (ωt ) d. 3cos  − 2ωt 
4 
The acceleration-displacement graph of the same
particle is represented by
15. A solid cylinder of mass m is wrapped
with an inextensible light string and, is
a a
placed on a rough inclined plane as
shown in the figure. The frictional force
a. b. acting between the cylinder and the
inclined plane is
O x O x
(The coefficient of static friction, µ s , is 0.4)
7
a. mg b. 5 mg
a a 2 60°
O x mg
c. d. c. d. 0
5

O x
16. The time taken for the magnetic energy to reach 25% of
its maximum value, when a solenoid of resistance R,
inductance L is connected to a battery, is
8. The correct relation between α (ratio of collector current a.
L
ln 5 b. infinite c.
L
ln 2 d.
L
ln 10
to emitter current) and β (ratio of collector current to R R R
base current) of a transistor is
α β 1 β 17. A particle of mass m moves in a circular orbit under the
a. β = b. α = c. β = d. α = central potential field, U( r) = − C / r , where C is a positive
1+ α 1− α 1− α 1+ β
constant. The correct radius-velocity graph of the
9. Three rays of light, namely P particle’s motion is
red (R), green (G) and blue (B) r r
are incident on the face PQ
B
of a right angled prism PQR
a. b.
as shown in figure G
The refractive indices of the R v v
material of the prism for red, O O
green and blue wavelength R r r
Q
are 1.27, 1.42 and 1.49,
respectively. The colour of the ray(s) emerging out of the
face PR is c. d.
a. green b. red c. blue and green d. blue
v v
10. If the angular velocity of Earth’s spin is increased such O O
that the bodies at the equator start floating, the duration 18. An ideal gas in a cylinder is separated by a piston in such
of the day would be approximately a way that the entropy of one part is S1 and that of the
(Take 2 g = 10 ms −2 , the radius of Earth, R = 6400 × 103 m, other part is S 2. Given that S1 > S 2. If the piston is
take π = 314
. ) removed, then the total entropy of the system will be
a. 60 min b. does not change S1
a. S1 × S 2 b. S1 − S 2 c. d. S1 + S 2
c. 1200 min d. 84 min S2
32 Online JEE Main 2021 March Attempt

19. Consider a sample of oxygen behaving like an ideal gas. (Take value of g to be 10 ms −2)
At 300 K, the ratio of root mean square (rms) velocity to
the average velocity of gas molecule would be 24 kg
(Molecular weight of oxygen is 32 g/mol; R = 8. 3 J K −1 A
mol −1)
3 8 3π 8π
a. b. c. d.
3 3 8 3
20. The speed of electrons in a scanning electron
microscope is 1 × 107 ms −1. If the protons having the
B
same speed are used instead of electrons, then the
resolving power of scanning proton microscope will be 25. A TV transmission tower antenna is at a height of 20 m.
changed by a factor of Suppose that the receiving antenna is at
1 1
a. 1837 b. c. 1837 d. (i) ground level
1837 1837
(ii) a height of 5 m.
Secton B : Numerical Type Questions The increase in antenna range in case (ii) relative to case (i)
21. The projectile motion of a particle of mass 5 g is shown is n%.
in the figure. The value of n, to the nearest integer, is ……… .
26. The radius of a sphere is measured to be (7.50 ± 0.85) cm.
45° 45°
Suppose the percentage error in its volume is x. The
A B value of x to the nearest x, is …………… .

The initial velocity of the particle is 5 2 ms −1 and the air 27. An infinite number of point charges, each carrying 1 µC
resistance is assumed to be negligible. The magnitude of charge, are placed along the Y-axis at y = 1m, 2m, 4 m, 8m.
the change in momentum between the points A and B is The total force on a 1 C point charge, placed at the origin, is
x × 10–2 kg-ms −1. The value of x to the nearest integer, is x × 103 N. The value of x to the nearest integer, is …………… .
……………… . 1
(Take, = 9 × 109 N-m 2 /C 2 )
22. A ball of mass 4 kg, moving with a velocity of 10 ms −1, 4 πε0
collides with a spring of length 8 m and force constant 28. Consider a 72 cm long wire AB as shown in the figure. The
100 Nm −1. The length of the compressed spring is x m. galvanometer jockey is placed at P on AB at a distance x
The value of x to the nearest integer, is ………… . cm from A. The galvanometer shows zero deflection.
23. The typical output characteristics curve for a transistor 12 Ω C 6Ω
working in the common-emitter configuration is shown
in the figure.
Ic(mA) G

8 IB=40 µA x
A B
6 IB=30 µA P
4 IB=20 µA
2 IB=10 µA The value of x, to the nearest integer, is ……… .
0 29. Two wires of same length and thickness having specific
VCE(V) resistances 6 Ω- cm and 3 Ω-cm respectively are connected
The estimated current gain from the figure is ……… . in parallel. The effective resistivity is ρ Ω-cm. The value of ρ
to the nearest integer, is …………… .
24. Consider a water tank as shown in the figure. It’s
cross-sectional area is 0.4 m 2. The tank has an opening 30. A galaxy is moving away from the Earth at a speed of 286
B near the bottom whose cross-section area is 1 cm 2. A kms −1. The shift in the wavelength of a red line at 630 nm
load of 24 kg is applied on the water at the top when the is
height of the water level is 40 cm above the bottom, the x × 10–10 m. The value of x to the nearest integer, is
velocity of water coming out the opening B is v ms −1. The ………… .
value of v, to the nearest integer, is …………… . (Take the value of speed of light c, as 3 × 108 ms −1)

Answers For solution scan


the QR code

1. (a) 2. (a) 3. (a) 4. (d) 5. (c) 6. (c) 7. (c) 8. (d) 9. (b) 10. (d)
11. (b) 12. (c) 13. (d) 14. (d) 15. (c) 16. (c) 17. (a) 18. (d) 19. (c) 20. (a)
21. (5) 22. (6) 23. (200) 24. (3) 25. (50) 26. (34) 27. (12) 28. (48) 29. (4) 30. (6)
July Attempt Online JEE Main 2021 33

20 JULY SHIFT I
Section A : Objective Type Questions 6. Region I and II are separated by a spherical surface of
1. The value of current in the 6 Ω resistance is radius 25 cm. An object is kept in region I at a distance of
40 cm from the surface. The distance of the image from
20Ω 5Ω the surface is
I II
25cm
140V 6Ω 90V
O C µI=1.25 µII=1.4

a. 55.44 cm b. 9.52 cm
c. 18.23 cm d. 37.58 cm
a. 4 A b. 8 A
c. 10 A d. 6 A 7. A person whose mass is 100 kg travels from Earth to
Mars in a spaceship. Neglect all other objects in sky and
2. The normal reaction N for a vehicle of 800 kg mass, take acceleration due to gravity on the surface of the
negotiating a turn on a 30° banked road at maximum
Earth and Mars as 10 m/s2 and 4 m/s2, respectively.
possible speed without skidding is …… × 103 kg-m/s2.
[Take, µ s = 02
. ] Identify from the below figures, the curve that fits best
a. 10.2 b. 7.2
for the weight of the passenger as a function of time.
c. 12.4 d. 6.96
1000N A
3. A radioactive material decays by simultaneous emission
of two particles with half-lives of 1400 yr and 700 yr, I
respectively. What will be the time after the which Weight II
one-third of the material remains ?
[Take, In 3 = 1.1] B
400N
a. 1110 yr b. 700 yr III
c. 340 yr d. 740 yr IV Time
4. A steel block of 10 kg rests on a horizontal floor as a. III b. I
shown. When three iron cylinders are placed on it as c. IV d. II
shown, the block and cylinders go down with an
acceleration 0.2 m/s2. 8. The amount of heat needed to raise the temperature of
4 moles of a rigid diatomic gas from 0°C to 50°C when no
The normal reaction R by the floor, if mass of the iron work is done is ……… . (R is the universal gas constant)
cylinders are equal and of 20 kg each, is …… N. a. 250 R b. 750 R
[Take, g = 10 m / s 2 and µ s = 02
. ] c. 175 R d. 500 R

3
9. If A and B are two vectors satisfying the relation
A ⋅ B = |A × B|. Then, the value of|A − B| will be
1 2 a. A 2 + B 2 b. A 2 + B 2 + 2AB
c. A 2 + B 2 + 2AB d. A 2 + B 2 − 2AB

10. A deuteron and an α-particle having equal kinetic energy


enter perpendicular into a magnetic field. Let rd and rα
a = 0.2 m/s2
be their respective radii of circular path. The value of
a. 716 b. 686 rd / rα is equal to
c. 714 d. 684 a. 1 / 2 b. 2

5. AC voltage V (t) = 20 sinωt volt of frequency c. 1 d. 2

50 Hz is applied to a parallel plate capacitor. The 11. A nucleus of mass M emits γ- ray photon of frequency ν.
separation between the plates is 2 mm and the area is The loss of internal energy by the nucleus is
1 m2. The amplitude of the oscillating displacement [Take, c is the speed of electromagnetic wave.]
current for the applied AC voltage is …… . a. hν b. zero
. × 10−12 F/m)
(Take, ε0 = 885  hν  hν 
c. hν 1 − d. hν 1 +
a. 21.14 µA b. 83.37 µA  2Mc 2   2Mc 2 
c. 27.79 µA d. 55.58 µA
34 Online JEE Main 2021 July Attempt

12. A certain charge Q is divided into two parts q and (Q − q).


How should the charges Q and q be divided, so that q and
(Q − q) placed at a certain distance apart experience
maximum electrostatic repulsion ? P
q
a. Q = b. Q = 2q
2
c. Q = 4q d. Q = 3q Q
13. A current of 5 A is passing through a non-linear
magnesium wire of cross-section
0.04 m2. At every point, the direction of current density is x=0 x=b x=2b
at an angle of 60° with the unit vector of area of
cross-section. The magnitude of electric field at every A
point of the conductor is
(Take, resistivity of magnesium, ρ = 44 × 10−8 Ω -m)
B
a. 11 × 10−2 V/m b. 11 × 10−7 V/m
c. 11 × 10−5 V/m d. 11 × 10−3 V/m
14. Consider a mixture of gas molecule of types A, B and C
having masses m A < m B < mC . The ratio of their root
mean square speeds at normal temperature and x=0 C
pressure is b 2b b
1 1 1
a. v A = v B = vC = 0 b. > >
v A v B vC a. A - flux, B - power dissipated, C - emf
1 1 1 b. A - power dissipated, B - flux, C - emf
c. v A = v B ≠ vC d. < <
v A v B vC c. A - flux, B - emf , C - power dissipated
d. A - emf , B - power dissipated, C - flux
15. A butterfly is flying with a velocity 4 2 m/s in North-East
direction. Wind is slowly blowing at 1 m/s from North to 19. The entropy of any system is given by
South. The resultant displacement of the butterfly in 3 s  µkR 
S = α 2β ln  2 + 3
is  J β 
a. 3 m b. 20 m c. 12 2 m d. 15 m where, α and β are the constants; µ, J , k and R are number of
moles, mechanical equivalent of heat, Boltzmann constant
16. The value of tension in a long thin metal wire has been and gas constant, respectively.
changed from T1 to T2. The lengths of the metal wire at  Take, S = dQ 
two different values of tension T1 and T2 are l1 and l 2,
 T 
respectively. The actual length of the metal wire is
T1l 2 − T2 l1 T1l1 − T2 l 2 Choose the incorrect option.
a. b. a. α and J have the same dimensions.
T1 − T2 T1 − T2
l1 + l 2 b. S , β , k and µR have the same dimensions.
c. d. TT
1 2 l1l 2 c. S and α have different dimensions.
2
d. α and k have the same dimensions.
17. For the circuit shown below, calculate the value of Iz.
20. The radiation corresponding to 3 → 2 transition of a
Rs=1000Ω hydrogen atom falls on a gold surface to generate
photoelectrons. These electrons are passed through a
R
Iz Iz magnetic field of 5 × 10−4 T. Assume that the radius of the
largest circular path followed by these electrons is 7 mm,
Vi 100V R = 2000Ω the work-function of the metal is (Take, mass of electron
Vz=50V
. × 10−31 kg)
= 91
a. 1.36 eV b. 1.88 eV
c. 0.16 eV d. 0.82 eV
a. 25 mA b. 0.15 A
c. 0.1 A d. 0.05 A Section B : Numerical Type Questions
18. The arm PQ of a rectangular conductor is moving from 21. In a spring gun having spring constant
x = 0 to x = 2b outwards and then inwards from x = 2b to 100 N/m a small ball B of mass 100 g is put in its barrel
x = 0 as shown in the figure. A uniform magnetic field (as shown in figure) by compressing the spring through
perpendicular to the plane is acting from x = 0 to x = b. 0.05 m. There should be a box placed at a distance d on
Identify the graph showing the variation of different the ground, so that the ball falls in it. If the ball leaves the
quantities with distance.
July Attempt Online JEE Main 2021 35

gun horizontally at a height of 2 m above the ground. 25. In the reported figure, heat energy absorbed by a system
The value of d is ……… m. in going through a cyclic process is …… πJ.
(Take, g =10m / s 2) p(kPa)
Gun
40
Ball

20

2m
L
20 40

22. In an L-C-R series circuit, an inductor 30 mH and a 26. A circular disc reaches from top to bottom of an inclined
resistor 1Ω are connected to an AC source of angular plane of length L. When it slips down the plane, it takes
frequency 300 rad/s. The value of capacitance for which, time t1. When it rolls down the plane, it takes time t 2. The
1 t 3
the current leads the voltage by 45° is × 10−3 F. Then, value of 2 is . The value of x will be …… .
x t1 x
the value of x is …… .
27. A rod of mass M and length L is lying on a horizontal
23. The amplitude of wave disturbance propagating in the frictionless surface. A particle of mass m travelling along
1
positive x-direction is given by y = at time t = 0 the surface hits at one end of the rod with a velocity u in
(1 + x) 2 a direction perpendicular to the rod. The collision is
1 completely elastic. After collision, particle comes to rest.
and y = at t = 1s, where x and y are in metre.
1 + ( x − 2) 2 1
The ratio of masses   is . The value of x will be …… .
m
M x
The shape of wave does not change during the
propagation. The velocity of the wave will be …… m/s. 28. An object viewed from a near point distance of 25 cm,
24. A body having specific charge 8 µC/g is resting on a using a microscopic lens with magnification 6, gives an
frictionless plane at a distance 10 cm from the wall (as unresolved image. A resolved image is observed at
shown in the figure). It starts moving towards the wall infinite distance with a total magnification double the
when a uniform electric field of 100 V/m is applied earlier using an eyepiece along with the given lens and a
horizontally towards the wall. If the collision of the body tube of length 0.6 m, if the focal length of the eyepiece is
with the wall is perfectly elastic, then the time period of equal to ……… cm.
the motion will be …… s. 29. The frequency of a car horn encountered a change from
400 Hz to 500 Hz, when the car approaches a vertical
wall. If the speed of sound is 330 m/s, then the speed of
car is …… km/h.
30. A carrier wave VC (t) = 160sin(2π × 106t) V is made to vary
between Vmax = 200 V and Vmin = 120 V by a message
Body
100 V/m signal Vm (t) = Am sin(2π × 103t) V. The peak voltage Am of
the modulating signal is …… .

Answers For solution scan


the QR code

1. (c) 2. (a) 3. (d) 4. (b) 5. (c) 6. (d) 7. (a) 8. (d) 9. (d) 10. (b)
11. (d) 12. (b) 13. (c) 14. (d) 15. (d) 16. (a) 17. (a) 18. (c) 19. (d) 20. (d)
21. (0.003) 22. (3) 23. (2) 24. (1) 25. (100) 26. (2) 27. (4) 28. (25) 29. (132) 30. (40)
36 Online JEE Main 2021 July Attempt

20 JULY SHIFT II
Section A : Objective Type Questions 9. A particle is making simple harmonic motion along the
1. If the kinetic energy of a moving body becomes four times X-axis. If at a distances x1 and x 2 from the mean position,
of its initial kinetic energy, then the percentage change in the velocities of the particle are v1 and v 2 respectively,
its momentum will be then the time period of its oscillation is given as
a. 100% b. 200% c. 300% d. 400% x 22 + x12 x 22 + x12
a. T = 2 π b. T = 2 π
2. A boy reaches the airport and finds that the escalator is v12 − v 22 v12 + v 22
not working. He walks up the stationary escalator in time x 22 − x12 x 22 − x12
t1. If he remains stationary on a moving escalator, then c. T = 2 π d. T = 2 π
v12 + v 22 v12 − v 22
the escalator takes him up in time t 2. The time taken by
him to walk up on the moving escalator will be 10. An electron having de-Broglie wavelength λ is incident
t1t 2 t1 + t 2 t1t 2 on a target in a X-ray tube. Cut-off wavelength of
a. b. c. d. t 2 − t1
t 2 − t1 2 t 2 + t1 emitted X-ray is
2m 2c 2 λ 2 2mcλ 2 hc
3. A satellite is launched into a circular orbit of radius R a. Zero b. 2
c. d.
h h mc
around Earth, while a second satellite is launched into a
circular orbit of radius 1.02 R. The percentage difference 11. A body rolls down an inclined plane without slipping. The
in the time periods of the two satellites is kinetic energy of rotation is 50% of its translational
a. 1.5 b. 2.0 c. 0.7 d. 3.0 kinetic energy. The body is
4. With what speed should a galaxy move outward with a. solid sphere b. solid cylinder
respect to Earth, so that the sodium-D line at wavelength c. hollow cylinder d. ring
5890 Å is observed at 5896 Å ? 12. If time (t), velocity (v ) and angular momentum ( l) are
a. 306 km/s b. 322 km/s c. 296 km/s d. 336 km/s taken as the fundamental units, then the dimension of
5. The length of a metal wire is l1, when the tension in it is T1 mass ( m) in terms of t ,v and l is
and is l 2 when the tension is T2. The natural length of the a. [ t −1v1l −2 ] b. [ t 1v 2 l −1]
wire is −2 −1 1
c. [ t v l ] d. [ t −1v −2 l1]
l T − l 2T1 l T + l 2T1 l +l
a. l1 l 2 b. 1 2 c. 1 2 d. 1 2
T2 − T1 T2 + T1 2 13. The correct relation between the degrees of freedom f
and the ratio of specific heat γ is
6. In an electromagnetic wave, the electric field vector and 2 2 γ+1 1
a. f = b. f = c. f = d. f =
magnetic field vector are given as E = E 0$i and B = B 0k$ , γ −1 γ+1 2 γ+1
respectively. The direction of propagation of
electromagnetic wave is along 14. For a certain radioactive process, the graph between InR
a. k$ b. $j c. (−k$ ) d. (− $j ) and t (sec) is obtained as shown in the figure. Then, the
value of half-life for the unknown radioactive material is
7. For a series L-C-R circuit with R = 100 Ω, approximately
L = 05
. mH and C = 01
. pF connected across 220 V-50 Hz
AC supply, the phase angle between current and
supplied voltage and the nature of the circuit is 8 R = Decay rate
a. 0°, resistive circuit 6
In R

b. ≈ 90°, predominantly inductive circuit


4
c. 0° resonance circuit
d. ≈ 90°, predominantly capacitive circuit 2
8. Which of the following graphs represent the behaviour
of an ideal gas ? (Symbols have their usual meanings.) 10 20 30 40 50 60
pV pV Time, t(s)
a. 9.15 s b. 6.93 s c. 2.62 s d. 4.62 s
a.
b. 15. Consider a binary star system of star A and star B with
masses m A and m B revolving in a circular orbit of radii rA
T T and rB , respectively. If TA and TB are the time period of
pV pV star A and star B respectively, then
3

c. T r 2
a. A =  A  b. TA = TB
d. TB  rB 
T T c. TA > TB (if m A > m B ) d. TA > TB (if rA > rB )
July Attempt Online JEE Main 2021 37

16. At an angle of 30° to the magnetic meridian, the 24. Two bodies, a ring and a solid cylinder of same material
apparent dip is 45°. Find the true dip. are rolling down without slipping an inclined plane. The
−1 −1
1  −1
2  −1
3 radii of the bodies are same. The ratio of velocity of the
a. tan ( 3 ) b. tan   c. tan   d. tan   centre of mass at the bottom of the inclined plane of the
 3  3  2 
ring to that of the cylinder is x / 2. Then, the value of x is
17. A body at rest is moved along a horizontal straight line …… .
by a machine delivering a constant power. The distance
25. For the forward biased diode characteristics shown in
moved by the body in time t is proportional to
3 1 1 3 the figure, the dynamic resistance at ID = 3 mA will be
a. t 2 b. t 2 c. t 4 d. t 4 ……… Ω.
18. Two vectors P and Q have equal magnitudes. If the

ID(mA)
magnitude of P + Q is n times the magnitude of P − Q,
then angle between P and Q is 8
−1
n − 1 −1
n − 1 7
a. sin   b. cos  
 n + 1  n + 1 6
−1
n 2 − 1 −1
n 2 − 1 5
c. sin  2  d. cos  2 
 n + 1  n + 1 4
19. Two small drops of mercury each of radius R coalesce to 3
form a single large drop. The ratio of total surface energy 2
before and after the change is 1
a. 21/ 3 : 1 b. 1: 21/ 3 c. 2 : 1 d. 1 : 2
0.1 0.2 0.3 0.4 0.5 0.6 0.7 0.8 0.9 VD(V)
20. The magnetic susceptibility of a material of a rod is 499.
−7
Permeability in vacuum is 4 π × 10 H/m. Absolute 26. A series L-C-R circuit of R = 5 Ω, L = 20 mH and C = 05
. µF is
permeability of the material of the rod is
a. 4 π × 10 H/m−4
b. 2 π × 10 H/m −4 connected across an AC supply of 250 V, having variable
frequency. The power dissipated at resonance condition
c. 3 π × 10−4 H/m d. π × 10−4 H/m
is …… × 102 W .
Numerical Type Questions 27. One mole of an ideal gas at 27°C is taken from A to B as
21. A Zener diode having Zener voltage 8 V and power shown in the given p-V indicator diagram. The work done
dissipation rating of 0.5 W is connected across a by the system will be …… × 10−1 J .
potential divider arranged with maximum potential drop [Take, R = 8.3 J/ mol-K, In 2 = 06931
. ]
across Zener diode is as shown in the diagram. The value (Round off to the nearest integer)
of protective resistance R p is ...........Ω.
Rp 200 A(p1V1)
n
p(N/m2)

p
Vz = 8V
100 B(p2V2)

– + 2 4
V(m 3)
20 V
22. A body of mass m is launched up on a rough inclined 28. A certain metallic surface is illuminated by monochromatic
plane making an angle of 30° with the horizontal. The radiation of wavelength λ. The stopping potential for
coefficient of friction between the body and plane is photoelectric current for this radiation is 3V0.
x / 5. if the time of ascent is half of the time of descent. If the same surface is illuminated with a radiation of
The value of x is …… . wavelength 2λ, the stopping potential is V0. The
threshold wavelength of this surface for photoelectric
23. In the given figure, switches S1 and S 2 are in open effect is …… λ.
condition. The resistance across ab when the switches S1
and S 2 are closed is …… Ω. 29. A body rotating with an angular speed of
600 rpm is uniformly accelerated to 1800 rpm in 10 s.
12 Ω 4Ω 6Ω The number of rotations made in the process is …… .
30. A radioactive substance decays to (1/16)th of its initial
a S1 S2 b
activity in 80 days. The half-life of the radioactive
substance expressed in days is …… .
6Ω 4Ω 12 Ω
38 Online JEE Main 2021 July Attempt

Answers For solution scan


the QR code

1. (a) 2. (c) 3. (d) 4. (a) 5. (b) 6. (d) 7. (d) 8. (c) 9. (d) 10. (c)
11. (b) 12. (d) 13. (a) 14. (d) 15. (b) 16. (d) 17. (a) 18. (d) 19. (a) 20. (b)
21. (192) 22. (3) 23. (10) 24. (3) 25. (25) 26. (125) 27. (17258) 28. (4) 29. (32) 30. (20)

22 JULY SHIFT II
Section A : Objective Type Questions shown in figure. Find the resistance of the combination
1. In a circuit consisting of a capacitance and a generator between the ends A and B.
with alternating emf E g = E g 0 sinωt, VC and IC are the (Take, resistivity of copper = 1.7 × 10−8 Ω-m, resistivity of
voltage and current. Correct phasor diagram for such aluminium = 2.6 × 10−8 Ω-m)
circuit is
Cu
A B
Al

Eg IC C V a. 2.170 m Ω b. 1.420 m Ω
C
c. 0.0858 m Ω d. 0.858 m Ω
3. What will be the projection of vector A = i$ + $j + k$ on
vector B = i$ + $j ?
a. 2 ($i + $j + k$ ) b. 2($i + $j + k$ )
VC c. 2($i + $j ) d. ($i + $j )
4. A porter lifts a heavy suitcase of mass 80 kg and at the
ωt destination lowers it down by a distance of 80 cm with a
a. constant velocity. Calculate the work done by the porter
in lowering the suitcase.
[Take, g = 9.8 ms −2 ]
IC
a. − 627200. J b. − 6272
. J
c. + 6272
. J d. 784.0 J
VC
IC 5. T0 is the time period of a simple pendulum at a place. If
the length of the pendulum is reduced to 1/16 times of
b. its initial value, then the modified time period is
ωt 1
a. T0 b. 8 πT0 c. 4T0 d. T0
4
6. A ray of light passes from a denser medium to a rarer
medium at an angle of incidence i. The reflected and
IC VC refracted rays make an angle of 90° with each other. The
angle of reflection and refraction are respectively r and
ωt r'. The critical angle is given by
c.

i r
VC
r′
ωt
d.

a. sin −1(cot r ) b. tan −1(sin i ) c. sin −1(tan r ′ ) d. sin −1(tan r )

IC
7. Statement I The ferromagnetic property depends on
temperature. At high temperature, ferromagnet
2. A copper (Cu) rod of length 25 cm and cross-sectional becomes paramagnet.
area 3 mm2 is joined with a similar aluminium (Al) rod as
July Attempt Online JEE Main 2021 39

Statement II At high temperature, the domain wall area 14. Match List I with List II.
of a ferromagnetic substance increases.
In the light of the above statements, choose the most List I List II
appropriate answer from the options given below. A. 1 (i) Current is in phase with EMF
ωL >
a. Statement I is true but Statement II is false ωC
b. Both Statement I and Statement II are true B. 1 (ii) Current lags behind the
ωL =
c. Both Statement I and Statement II are false ωC applied EMF
d. Statement I is false but Statement II is true 1
C. ωL < (iii) Maximum current occurs
ωC
8. A bullet of 4 g mass is fired from a gun of mass 4 kg. If
the bullet moves with the muzzle speed of 50 ms −1, the D. Resonant frequency (iv) Current leads the EMF
impulse imparted to the gun and velocity of recoil of gun
are Choose the correct answer from the options given below.
a. 0.4 kg-ms −1, 0.1 ms −1 b. 0.2 kg-ms −1, 0.05 ms −1 Codes
c. 0.2 kg-ms −1, 0.1 ms −1 d. 0.4 kg-ms −1 , 0.05 ms −1 A B C D A B C D
a. (ii) (i) (iv) (iii) b. (ii) (i) (iii) (iv)
9. Choose the correct option. c. (iii) (i) (iv) (ii) d. (iv) (iii) (ii) (i)
a. True dip is not mathematically related to apparent dip.
b. True dip is less than apparent dip. 15. What should be the height of transmitting antenna and
c. True dip is always greater than the apparent dip. the population covered, if the television telecast is to
d. True dip is always equal to apparent dip. cover a radius of 150 km ? The average population
density around the tower is 2000/km2 and the value of R e
10. Consider a situation in which a ring, a solid cylinder and
. × 106 m.
= 65
a solid sphere roll down on the same inclined plane
a. Height = 1731m, Population covered = 1413 × 105
without slipping. Assume that they start rolling from rest
b. Height = 1241m, Population covered = 7 × 105
and having identical diameter.
c. Height = 1600 m, Population covered = 2 × 105
The correct statement for this situation. d. Height = 1800 m, Population covered = 1413 × 108
a. The sphere has the greatest and the ring has the least
velocity of the centre of mass at the bottom of the inclined
16. What will be the average value of energy for a monoatomic
plane. gas in thermal equilibrium at temperature T ?
2 3 1
b. The ring has the greatest and the cylinder has the least a. K BT b. K BT c. K BT d. K BT
velocity of the centre of mass at the bottom of the inclined 3 2 2
plane. 17. Intensity of sunlight is observed as
c. All of them will have same velocity. 0.092 Wm−2 at a point in free space. What will be the
d. The cylinder has the greatest and the sphere has the least peak value of magnetic field at that point?
velocity of the centre of mass at the bottom of the inclined
plane. (ε 0 = 8.85 × 10−12C –2N–1m–2)
a. 2.77 × 10−8 T b. 1.96 × 10−8 T
11. Consider a situation in which reverse biased current of a c. 8.31 T d. 5.88 T
particular p-n junction increases when it is exposed to a 18. The motion of a mass on a spring, with spring constant k
light of wavelength ≤ 621 nm. During this process, is as shown in figure.
enhancement in carrier concentration takes place due to
generation of hole-electron pairs. The value of band gap
is nearly
a. 2 eV b. 4 eV c. 1 eV d. 0.5 eV

12. A nucleus with mass number 184 initially at rest emits an


α-particle. If the Q-value of the reaction is 5.5 MeV, x
calculate the kinetic energy of the α-particle.
a. 5.0 MeV b. 5.5 MeV
c. 0.12 MeV d. 5.38 MeV The equation of motion is given by
13. An electron of mass m e and a proton of mass m p are k
x(t) = A sinωt + B cos ωt with ω =
m
accelerated through the same potential difference. The
ratio of the de-Broglie wavelength associated with the Suppose that at time t = 0, the position of mass is x(0)
electron to that with the proton is and velocity v(0), then its displacement can also be
a.
mp
b. 1 represented as x(t) = C cos(ωt − φ), where C and φ are
me
2v (0) 2  v (0) 
mp m a. C = + x (0) 2 , φ = tan −1 
c. d. e ω2  x (0)ω 
me mp
2v (0) 2  x (0)ω 
b. C = + x (0) 2 , φ = tan −1 
ω2  2v (0) 
40 Online JEE Main 2021 July Attempt

v (0) 2  x (0)ω  24. The total charge enclosed in an incremental volume of


c. C = + x (0) 2 , φ = tan −1 
ω2  v (0)  2 × 10−9m3 located at the origin is …… nC, if electric flux
v (0) 2
 v (0)  density of its field is found as
d. C = + x (0) 2 , φ = tan −1  D = e − x sin y $i − e − x cos y$j + 2zk$ C/m2.
ω2  x (0)ω 
25. Three particles P, Q and R are moving along the vectors
19. An electric dipole is placed on X-axis in proximity to a line
−6 A = $i + $j, B = $j + k$ and C = − i$ + $j, respectively. They
charge of linear charge density 30 . × 10 C/m. Line
strike on a point and start to move in different
charge is placed on Z-axis and positive and negative
directions. Now, particle P is moving normal to the plane
charge of dipole is at a distance of 10 mm and 12 mm
which contain vectors A and B. Similarly, particle Q is
from the origin, respectively. If total force of 4 N is
moving normal to the plane which contain vectors A and
exerted on the dipole, find out the amount of positive or
C. The angle between the direction of motion of P and Q
negative charge of the dipole.
 1
a. 815.1 nC b. 8.8 µC c. 0.485 nC d. 4.44 µC is cos −1  . Then, the value of x is ……… .
 x
20. A body is projected vertically upwards from the surface
of Earth with a velocity sufficient enough to carry it to 26. The centre of a wheel rolling on a plane surface moves
infinity. The time taken by it to reach height h is ……… s. with a speed v 0. A particle on the rim of the wheel at the
same level as the centre will be moving at a speed x v 0.

Re h
3/ 2  
2R e h
3/ 2 
a. 1+  − 1 b. 1+  − 1 Then, the value of x is ……… .
 
2g Re    
g Re   27. A ray of light passing through a prism (µ = 3) suffers
1 Re  h
3/ 2  1 2R e  h
3/ 2  minimum deviation. It is found that the angle of
c. 1+  − 1 d. 1+  − 1
3 2g  Re   3 g  Re   incidence is double the angle of refraction within the
 
prism. Then, the angle of prism is ……… (in degrees).
Section B : Numerical Type Questions 28. The area of cross-section of a railway track is 0.01 m2.
The temperature variation is 10°C. Coefficient of linear
21. In a given circuit diagram, a 5 V Zener diode along with a
expansion of material of track is 10−5/°C. The energy
series resistance is connected across a 50 V power
stored per metre in the track is …… J/m.
supply. The minimum value of the resistance required, if
the maximum Zener current is 90 mA will be ……… Ω. (Take, Young’s modulus of material of track is 1011Nm−2)

I I2 29. Three students S1, S 2 and S 3 perform an experiment for


determining the acceleration due to gravity (g) using a
R
I1 simple pendulum. They use different lengths of
pendulum and record time for different number of
RL Vz oscillations. The observations are as shown in the table.
V =50V
Z
Student Length of No. of Total time Time period
No. pendulum oscillations for n (s)
(cm) (n) oscillations
1. 64.0 8 128.0 16.0
22. The position of the centre of mass of
2. 64.0 4 64.0 16.0
a uniform semi-circular wire of radius R placed in
XY-plane with its centre at the origin and the line joining 3. 20.0 4 36.0 9.0
its ends as X-axis is given by (0, xR / π ). Then, the value of
|x| is ……… . (Least count of length = 01
. m, least count for time = 01
. s)

23. In an electric circuit, a cell of certain EMF provides a If E 1 ,E 2 and E 3 are the percentage errors in g for
potential difference of 1.25 V across a load resistance of students 1, 2 and 3 respectively, then the minimum
5 Ω. However, it provides a potential difference of 1 V percentage error is obtained by student number ……… .
across a load resistance of 2 Ω. The emf of the cell is 30. In 5 min, a body cools from 75°C to 65°C at room
x temperature of 25°C. The temperature of body at the
given by V. Then, the value of x is ……… .
10 end of next 5 min is ……… °C.

Answers For solution scan


the QR code

1. (c) 2. (d) 3. (d) 4. (b) 5. (d) 6. (d) 7. (a) 8. (b) 9. (b) 10. (a)
11. (a) 12. (d) 13. (c) 14. (a) 15. (a) 16. (c) 17. (a) 18. (d) 19. (d) 20. (d)
21. (500) 22. (2) 23. (15) 24. (4) 25. (9) 26. (4) 27. (60) 28. (5) 29. (1) 30. (57)
July Attempt Online JEE Main 2021 41

25 JULY SHIFT I
Section A : Objective Type Questions 7. Match List I with List II.
1. For a gas C p − C V = R in a state P and C P − C V = 1.10 R in a List I List II
state Q. TP and TQ are the temperatures in two different C
states P and Q, respectively. Then, (A) C − A −B=0 (i) A
a. TP = TQ b. TP < TQ B
c. TP = 09
. TQ d. TP > TQ
C
2. Assertion A Moment of inertia of a circular disc of mass (B) A −C −B=0 (ii)
B
M and radius R about X , Y -axes (passing through its
plane) and Z-axis which is perpendicular to its plane A
were found to be Ix , I y and Iz , respectively. The respective C
radii of gyration about all the three axes will be the (C) B− A −C =0 (iii)
same. A
B
Reason R A rigid body making rotational motion has
fixed mass and shape. A
In the light of the above statements, choose the most (D) A + B= −C (iv) C
appropriate answer from the options given below. B
a. Both A and R are correct and R is the correct explanation
of A. Choose the correct answer from the options given below.
b. Both A and R are correct but R is not the correct a. (A) → (iv), (B) → (i) , (C) → (iii), (D) → (ii)
explanation of A. b. (A) → (iv), (B) → (iii) , (C) → (i), (D) → (ii)
c. A is correct but R is not correct. c. (A) → (iii), (B) → (ii) , (C) → (iv), (D) → (i)
d. A is not correct but R is correct. d. (A) → (i), (B) → (iv) , (C) → (ii), (D) → (iii)
3. What should be the order of arrangement of de-Broglie 8. A parallel plate capacitor with plate area A and distance
wavelength of electron (λ e ), an α-particle (λ α ) and proton of separation d is filled with a dielectric. What is the
(λ p) given that all have the same kinetic energy ? capacity of the capacitor when permittivity of the
a. λ e = λ p = λ α b. λ e < λ p < λ α dielectric varies as
c. λ e > λ p > λ α d. λ e = λ p > λ α  d
ε( x) = ε 0 + kx , for  0 < x ≤ 
 2
4. Identify the logic operation carried out.
d 
ε( x) = ε 0 + k(d − x), for  ≤ x ≤ d
A 2 
2 / kA
Y
a.  ε0 +
kd  kA
 b.
B  2  2ε0 + kd 
2 ln  
 2ε0 
a. OR b. AND
kA  2ε0 
c. NOR d. NAND c. zero d. ln  
2  2ε0 − kd 
5. A particle of mass 4M at rest disintegrates into two
particles of masses M and 3M respectively having 9. A monoatomic ideal gas, initially at temperature T1 is
non-zero velocities. The ratio of de-Broglie wavelength of enclosed in a cylinder fitted with a frictionless piston.
particle of mass M to that of mass 3M will be The gas is allowed to expand adiabatically to a
a. 1 : 3 b. 3 : 1 temperature T2 by releasing the piston suddenly. If l1 and
c. 1 : 3 d. 1 : 1 l 2 are the lengths of the gas column, before and after the
T
6. Some nuclei of a radioactive material are undergoing expansion respectively, then the value of 1 will be
T2
radioactive decay. The time gap between the instances 2 2
when a quarter of the nuclei have decayed and when  l 3 l 3 l2 l1
a.  1  b.  2  c. d.
half of the nuclei have decayed is given as  l2   l1  l1 l2
(where, λ is the decay constant)
a.
1 ln 2
b.
ln2 10. A ray of laser of a wavelength 630 nm is incident at an
2 λ λ angle of 30° at the diamond-air interface. It is going from
3 diamond to air. The refractive index of diamond is 2.42
ln
2ln 2 2 and that of air is 1. Choose the correct option.
c. d.
λ λ a. Angle of refraction is 24.41° b. Angle of refraction is 30°
c. Refraction is not possible d. Angle of refraction is 53.4°
42 Online JEE Main 2021 July Attempt

11. Two wires of same length and radius are joined 16. A linearly polarised electromagnetic wave in vacuum is
end-to-end and loaded. The Young's moduli of the . cos[(1.8) z − (5.4 × 106) t ] $i N/C is incident normally
E = 31
materials of the two wires are Y1 and Y2. The combination on a perfectly reflecting wall at z = a .
behaves as a single wire, then its Young's modulus is Choose the correct option.
2YY 2YY a. The wavelength is 5.4 m
a. Y = 12 b. Y = 12
3(Y1 + Y2 ) Y1 + Y2 b. The frequency of electromagnetic wave is 54 × 104 Hz.
YY YY c. The transmitted wave will be 31. cos[(18. ) z − (5.4 × 106 )t ] $i N/C
c. Y = 12 d. Y = 12
d. The reflected wave will be 31 . ) z + (5.4 × 106 )t ] $i N/C
. cos[(18
2(Y1 + Y2 ) Y1 + Y2
17. In the given figure, there is a circuit of potentiometer of
12. The half-life of 198 Au is 3 days. If atomic weight of 198 Au is length AB = 10 m. The resistance per unit length is 0.1 Ω
198 g/mol, then the activity of 2 mg of 198 Au is [in per cm. Across AB, a battery of EMF E and internal
disintegration/s] resistance r is connected. The maximum value of emf
. × 1012
a. 267 . × 1018
b. 606 measured by this potentiometer is
c. 32.36 × 1012 . × 1012
d. 1618
E r
13. Two billiard balls of equal mass 30 g strike a rigid wall + –
with same speed of 108 km/h (as shown) but at different
G
angles. If the balls get reflected with the same speed,
then the ratio of the magnitude of impulses imparted to
ball a and ball b by the wall along, x. direction is A B
J
y y 550 cm 450cm

u 45º
x x
x′ x′ + –
20W K
y′ y′ 6V
Ball (a) Ball (b)
a. 5 V b. 2.25 V c. 6 V d. 2.75 V
a. 1 : 1 b. 2 : 1
18. In amplitude modulation, the message signal
c. 2 : 1 d. 1 : 2
Vm (t ) = 10sin(2 π × 105 t ) volts
14. In the Young’s double slit experiment, the distance and carrier signal
between the slits varies in time as d(t) = d 0 + a 0 sinωt, V c (t ) = 20sin(2 π × 107 t ) volts
where d 0, ω and a 0 are constants. The difference
The modulated signal now contains the message signal with
between the largest fringe width and the smallest fringe lower side band and upper side band frequency.
width obtained over time is given as Therefore, the bandwidth of modulated signal is α kHz. The
2λD (d 0 ) 2λDa 0 value of α is
a. b.
(d 02 − a 02 ) (d 02 − a 02 ) a. 200 kHz b. 50 kHz
λD λD c. 100 kHz d. zero
c. 2 a 0 d.
d0 d0 + a 0 19. Water droplets are coming from an open tap at a
15. Two different metal bodies A and B of equal mass are particular rate. The spacing between a droplet observed
heated at a uniform rate under similar conditions. The at 4th second after its fall to the next droplet is 34.3 m.
variation of temperature of the bodies is graphically At what rate, the droplets are coming from the tap ?
represented as shown in the figure. The ratio of specific (Take, g = 9.8 m/s2)
heat capacities is a. 3 drops / 2 s b. 2 drops / s
c. 1 drop / s d. 1 drop / 7 s
20. The minimum and maximum distances of a planet
150
revolving around the Sun are x1 and x 2. If the minimum
A
speed of the planet on its trajectory is v 0, then its
Temperature T(°C)

120
B maximum speed will be
90 v 0 x12 v 0 x 22 v 0 x1 v 0 x2
a. b. c. d.
60 x 22 x12 x2 x1

30 Section B : Numerical Type Questions


21. A body of mass 2 kg moving with a speed of 4 m/s makes
1 2 3 4 5 6 7 8
Time t(s) an elastic collision with another body at rest and
continues to move in the original direction but with
8 3 3 4 one-fourth of its initial speed. The speed of the two body
a. b. c. d. x
3 8 4 3 centre of mass is m/s. Then, the value of x is ……… .
10
July Attempt Online JEE Main 2021 43

22. Student A and student B used two screw gauges of equal Springs are kept in a stretched position with some
pitch and 100 equal circular divisions to measure the extension when the system is released. The horizontal
radius of a given wire. The actual value of the radius of surface is assumed to be frictionless. The angular
the wire is 0.322 cm. The absolute value of the difference frequency will be ……… rad/s when k = 20 N/m.
between the final circular scale readings observed by the
k 4k
students A and B is ……… .
A B
[Figure shows position of reference O when jaws of S1 S2
screw gauge are closed]
Given, pitch = 0.1 cm.
26. The value of aluminium susceptibility is 2.2 × 10−5. The
O O percentage increase in the magnetic field, if space within
a current carrying toroid is filled with aluminium is
x
, then the value of x is ……… .
104
27. A particle of mass 1 mg and charge q is lying at the
mid-point of two stationary particles kept at a distance 2
0 10 m when each is carrying same charge q. If the free
5 90 92 95
charged particle is displaced from its equilibrium
Screw gauge Screw gauge position through distance x (x << 1 m), the particle
(A) (B) executes SHM. Its angular frequency of oscillation will be
…… × 105 rad/s, if q 2 =10C 2.
23. An inductor of 10 mH is connected to a 20 V battery
through a resistor of 10 kΩ and a switch. After a long 28. An electric bulb rated as 200 W at 100 V is used in a
time, when maximum current is set up in the circuit, the circuit having 200 V supply. The resistance R that must
current is switched off. The current in the circuit after be put in series with the bulb, so that the bulb delivers
x the same power is …… Ω.
1 µs is mA. Then, x is equal to …… . (Take, e −1 = 037
. )
100
29. A pendulum bob has a speed of 3 m/s at its lowest
24. A circular conducting coil of radius 1 m is being heated position. The pendulum is 50 cm long. The speed of bob
by the change of magnetic field B passing perpendicular when the length makes an angle of 60° to the vertical will
to the plane in which the coil is laid. The resistance of the be ……. m/s. ( Take, g = 10 m / s 2)
coil is 2 µΩ. The magnetic field is slowly switched off such
that its magnitude changes in time as
30. A particle of mass m is moving in time t on a trajectory
given by
4
B = × 10−3  1 −
t 
T r = 10α t 2$i + 5β(t − 5) $j
π  100
where, α and β are dimensional constants. The angular
The energy dissipated by the coil before the magnetic
momentum of the particle becomes the same as it was
field is switched off completely is E = …… mJ.
for t = 0 at time t is …… s.
25. In the reported figure, two bodies A and B of masses 200
g and 800 g are attached with the system of springs.

Answers For solution scan


the QR code

1. (d) 2. (d) 3. (c) 4. (b) 5. (d) 6. (d) 7. (b) 8. (b) 9. (b) 10. (c)
11. (b) 12. (d) 13. (b) 14. (b) 15. (b) 16. (d) 17. (a) 18. (a) 19. (c) 20. (d)
21. (25) 22. (13) 23. (74) 24. (80) 25. (10) 26. (22) 27. (6000) 28. (50) 29. (2) 30. (10)

25 JULY SHIFT II
Section A : Objective Type Questions 2. In a simple harmonic oscillation, what fraction of total
1. The relation between time t and distance x for a moving mechanical energy is in the form of kinetic energy, when
body is given as t = mx 2 + nx , where m and n are the particle is midway between mean and extreme
constants. The retardation of the motion is (when v position.
stands for velocity) a. 1/2 b. 3/4
a. 2 mv 3 b. 2 mnv 3 c. 2 nv 3 d. 2n 2v 3 c. 1/3 d. 1/4
44 Online JEE Main 2021 July Attempt

3. A force F = ( 40i$ + 10$j) N acts on a body of mass 5 kg. If 10. If q f is the free charge on the capacitor plates and q b is
the body starts from rest its position vector r at time the bound charge on the dielectric slab of dielectric
t = 10 s, will be constant K placed between the capacitor plates, then
a. (100$i + 400$j ) m b. (100$i + 100$j ) m bound charge q b can be expressed as
c. (400$i + 100$j ) m d. (400$i + 400$j ) m
a. q b = q f  1 −
1 
b. q b = q f  1 − 
1

4. A prism of refractive index µ and angle of prism A is  K  K
c. q b = q f  1 +
1 
d. q b = q f  1 + 
placed in the position of minimum angle of deviation. If 1

minimum angle of deviation is also A, then in terms of  K  K
refractive index, 11. Consider a planet in some solar system which has a
−1 µ −1 µ
a. 2cos   b. sin   mass double the mass of Earth and density equal to the
 2  2 average density of Earth. If the weight of an object on
 µ − 1 µ Earth is w, the weight of the same object on that planet
c. sin −1  d. cos −1 
 2   2 will be
1
a. 2 w b. w c. 2 3 w d. 2 w
5. A heat engine has an efficiency of 1/6 . When the
temperature of sink is reduced by 62°C, its efficiency get 12. Two ideal electric dipoles A and B, having their dipole
doubled. The temperature of the source is moment p1 and p2 respectively, are placed on a plane
a. 124°C b. 37°C with their centres at O as shown in the figure. At point C
c. 62°C d. 99°C on the axis of dipole A, the resultant electric field is
making an angle of 37° with the axis. The ratio of the
6. In the given potentiometer circuit arrangement, the p
balancing length AC is measured to be 250 cm. When the dipole moment of A and B, 1 is
p2
galvanometer connection is shifted from point (1) to
3
point (2) in the given diagram, the balancing length (Take, sin 37° = )
ε 5
becomes 400 cm. The ratio of the EMF of two cells, 1 is
ε2 –
B
K V A– O
C + C
A B
+
G
3 3 2 4
a. b. c. d.
1 2 8 2 3 3
13. Two spherical soap bubbles of radii r1 and r2 in vacuum
E1 E2
combine under isothermal conditions. The resulting
a. 5/3 b. 8/5 bubble has a radius equal to
c. 4/3 d. 3/2 r1r2 r1 + r2
a. b. r1r2 c. r12 + r22 d.
7. Two ions having same mass have charges in the ratio 1 : r1 + r2 2
2. They are projected normally in a uniform magnetic 14. The force is given in terms of time t and displacement x
field with their speeds in the ratio 2 : 3. The ratio of the by the equation
radii of their circular trajectories is F = A cos Bx + C sin Dt
a. 1 : 4 b. 4 : 3 AD
The dimensional formula of is
c. 3 : 1 d. 2 : 3 B
8. A 10 Ω resistance is connected across a. [M 0LT −1 ] b. [ML2T −3 ] c. [M1L1T −2 ] d. [M 2L2T −3 ]
220 V - 50Hz AC supply. The time taken by the current to 15. The given potentiometer has its wire of resistance 10 Ω.
change from its maximum value to the rms value is When the sliding contact is in the middle of the
a. 2.5 ms b. 1.5 ms potentiometer wire, the potential drop across 2 Ω
c. 3.0 ms d. 4.5 ms resistor is
9. A balloon was moving upwards with a uniform velocity of 20 V
10 m/s. An object of finite mass is dropped from the
balloon when it was at a height of 75 m from the ground
level. The height of the balloon from the ground when A B
object strikes the ground was around, is 2Ω
(Take, the value of g = 10 m/s 2)
a. 300 m b. 200 m a. 10 V b. 5 V
c. 125 m d. 250 m 40 40
c. V d. V
9 11
July Attempt Online JEE Main 2021 45

16. An electron moving with speed v and a photon moving Section B : Numerical Type Questions
with speed c, have same de-Broglie wavelength. The ratio
of kinetic energy of electron to that of photon is 21. A system consists of two types of gas molecules A and B
3c v having same number density 2 × 1025 /m3. The diameter
a. b.
v 3c of A and B are 10 Å and 5 Å, respectively. They suffer
c.
v
d.
2c collision at room temperature. The ratio of average
2c v distance covered by the molecule A to that of B between
two successive collisions is …… ×10−2.
17. The instantaneous velocity of a particle moving in a
straight line is given as v = αt + βt 2, where α and β are 22. A light beam of wavelength 500 nm is incident on a metal
constants. The distance travelled by the particle between having work-function of 1.25 eV, placed in a magnetic
1s and 2s is field of intensity B. The electrons emitted perpendicular
3 7 to the magnetic field B, with maximum kinetic energy are
a. 3α + 7β b. α + β
2 3 bent into circular arc of radius 30 cm. The value of B is
α β 3 7
c. + d. α + β …… ×10−7 T.
2 3 2 2
(Take, hc = 20 × 10−26 J-m, mass of electron = 9 × 10−31 kg)
18. A ray of light entering from air into a denser medium of
refractive index 4/3, as shown in figure. The light ray 23. A message signal of frequency 20 kHz and peak voltage
suffers total internal reflection at the adjacent surface as of 20 V is used to modulate a carrier wave of frequency 1
shown. The maximum value of angle θ should be equal MHz and peak voltage of 20 V. The modulation index will
to be …… .

24. A 16 Ω wire is bend to form a square loop. A 9 V supply


having internal resistance of 1 Ω is connected across one
θ
of its sides. The potential drop across the diagonals of
the square loop is …… ×10−1 V.
θ′ 25. Two circuits are shown in the figures (a) and (b). At a
frequency of ……… rad/s, the average power dissipated in
one cycle will be same in both the circuits.
5Ω 40µF 5Ω 0.1 H
θ′′ µ=4/3 R C R L

220 V 220 V
7 5
a. sin −1 b. sin −1 Fig. (a) Fig. (b)
3 4
c. sin −1
7
d. sin −1
5 26. From the given data, the amount of energy required to
4 3 break the nucleus of aluminium 13
27
Al is x × 10−3 J.
19. When radiation of wavelength λ is incident on a metallic Mass of neutron = 100866
. u
surface, the stopping potential of ejected photoelectrons Mass of proton = 100726
. u
is 4.8 V. If the same surface is illuminated by radiation of Mass of aluminium nucleus = 2718846
. u
double the previous wavelength, then the stopping
potential becomes 1.6 V. The threshold wavelength of (Assume 1 u corresponds to x joule of energy)
the metal is (Round off to the nearest integer)
a. 2 λ b. 4 λ c. 8 λ d. 6 λ 27. A force of F = (5 y + 20) $j N acts on a particle. The work
20. Two vectors x and y have equal magnitude. The done by this force when the particle is moved from y = 0
magnitude of ( x − y ) is n times the magnitude of ( x + y ). m to y10 m is 450.
The angle between
x and y is 28. A solid disc of radius 20 cm and mass 10 kg is rotating
 − n − 1
2  n −1 
2 with an angular velocity of 600 rpm, about an axis
a. cos −1  2  b. cos −1  2
 normal to its circular plane and passing through its
 n −1   − n − 1
centre of mass. The retarding torque required to bring
 n2 + 1   n 2 + 1
c. cos −1   d. cos −1  2  the disc at rest in 10 s is π × 10−1 N-m.
2
 − n − 1  n − 1
46 Online JEE Main 2021 July Attempt

29. In a semiconductor, the number density of intrinsic 30. The nuclear activity of a radioactive element becomes
charge carriers at 27°C is 1.5 × 1016/ m3. If the (1/ 8) th of its initial value in 30 yr. The half-life of
semiconductor is doped with impurity atom, the hole radioactive element is yr.
density increases to 4.5 × 1022 / m3. The electron density
in the doped semiconductor is …… ×109 /m3.

Answers For solution scan


the QR code

1. (a) 2. (b) 3. (c) 4. (a) 5. (d) 6. (a) 7. (b) 8. (a) 9. (c) 10. (b)
11. (c) 12. (c) 13. (c) 14. (b) 15. (c) 16. (c) 17. (b) 18. (a) 19. (b) 20. (b)
21. (25) 22. (125) 23. (1) 24. (45) 25. (500) 26. (27.16) 27. (450) 28. (4) 29. (5) 30. (10)

27 JULY SHIFT I
Section A : Objective Type Questions
1. In the given figure, a battery of emf E is connected across List I List II
a conductor PQ of length l and different area of C. Moment of inertia of the rod III. ML2
cross-sections having radii r1 and r2( r2 < r1). (length 2L, mass M, about an axis 12
perpendicular to the rod passing
r1 r2 Q
P through its midpoint)
D. Moment of inertia of the rod IV. 2 ML2
– (length 2L, mass 2M, about an axis 3
perpendicular to the rod passing
+
through one of its end)
+ –

E K Choose the correct answer from the options given below.


A B C D A B C D
Choose the correct option as one moves from P to Q. a. (II) (III) (I) (IV) b. (II) (I) (III) (IV)
a. Drift velocity of electron increases c. (III) (IV) (II) (I) d. (III) (IV) (I) (II)
b. Electric field decreases 5. Three objects A , B and C are kept in a straight line on a
c. Electron current decreases
frictionless horizontal surface. The masses of A, B and C
d. All of the above
are m , 2m and 2m, respectively. A moves towards B with a
2. The number of molecules in 1 L of an ideal gas at 300 K speed of 9 m/s and makes an elastic collision with it.
and 2 atm pressure with mean kinetic energy 2 × 10−9 J There after B makes a completely inelastic collision with
per molecules is C . All motions occur along same straight line. The final
a. 0.75 × 1011 b. 3 × 1011 speed of C is
. × 1011
c. 15 d. 6 × 1011 A B C
3. The relative permittivity of distilled water is 81. The m 2m 2m
velocity of light in it will be
(Take, µ r = 1) a. 6 m/s b. 9 m/s c. 4 m/s d. 3 m/s
a. 4.33 × 107 m/s 7
b. 2.33 × 10 m/s 6. A capacitor of capacitance C = 1µF is suddenly connected
c. 3.33 × 107 m/s d. 5.33 × 107 m/s to a battery of 100 V through a resistance R = 100 Ω. The
4. time taken for the capacitor to be charged to get 50 V is
List I List II
R=100Ω
A. Moment of inertia of the rod I. 8 ML2
(length L, mass M, about an axis 3
perpendicular to the rod passing 100 V C=1µF
through the mid-point)
B. Moment of inertia of the rod II. ML2
(length L, mass 2M, about an axis 3
perpendicular to the rod passing [Take, In 2 = 069
. ]
through one of its end) . × 10−4 s
a. 144 b. 3.33 × 10−4 s
. × 10−4 s
c. 069 d. 0.30 × 10−4 s
July Attempt Online JEE Main 2021 47

7. In the reported figure, a capacitor is formed by placing a Choose the correct option out of the following for work done,
compound dielectric between the plates of parallel plate if processes BC and DA are adiabatic.
capacitor. The expression for the capacity of the said a. WAB = WDC b. WAD = WBC
capacitor will be (Take, area of plate = A) c. WBC + WDA > 0 d. WAB < WCD
11. Assertion A If A , B , C , D are four points on a semi-circular
C1 C2 C3 arc with centre at O such that|AB| =|BC| =|CD|, then
AB + AC + AD = 4 AO + OB + OC
K 3K 5K Reason R Polygon law of vector addition yields
AB + BC + CD + AD = 2AO
d 2d 3d
O
15 K ε0 A 15 K ε0 A 25 K ε0 A 9 K ε0 A A D
a. b. c. d.
34 d 6 d 6 d 6 d
8. The figure shows two solid discs with radius R and r, B C
respectively. If mass per unit area is same for both, what
is the ratio of MI of bigger disc around axis AB (which is In the light of the above statements, choose the most
perpendicular to the plane of the disc and passing appropriate answer from the options given below.
through its centre) of MI of smaller disc around one of its a. Both A and R are correct and R is the correct explanation of A.
diameters lying on its plane? b. Both A and R are correct but R is not the correct
Given, M is the mass of the larger disc. (MI stands for explanation of A.
moment of inertia) c. A is correct but R is not correct.
d. A is not correct but R is correct.
A
12. A light cylindrical vessel is kept on a horizontal surface.
Area of base is A. A hole of cross-sectional area a is made
just at its bottom side. The minimum coefficient of
friction necessary to prevent sliding the vessel due to the
C r D impact force of the emerging liquid is (a << A) .
M R
A

B
a. R 2 : r 2 b. 2r 4 : R 4 c. 2R 2 : r 2 d. 2R 4 : r 4
9. In Young’s double slit experiment, if the source of light
changes from orange to blue, then a
a. the central bright fringe will become a dark fringe
b. the distance between consecutive fringes will decrease
c. the distance between consecutive fringes will increase A
a. b. None of these
d. the intensity of the minima will increase 2a
10. In the reported figure, there is a cyclic process ABCDA on 2a a
c. d.
a sample of 1 mol of a diatomic gas. The temperature of A A
the gas during the process A → B and C → D are T1 and T2 13. A particle starts executing simple harmonic motion (SHM)
(T1 > T2) , respectively. of amplitude a and total energy E. At any instant, its kinetic
3E
p energy is , then its displacement y is given by
4
5p0 A a
a. y = a b. y =
2
B a 3 a
c. y = d. y =
2 2
14. If f denotes the ratio of the number of nuclei decayed (
Nd ) to the number of nuclei at t = 0
(N0), then for a collection of radioactive nuclei, the rate of
p0 C
D change of f with respect to time is given as
[λ is the radioactive decay constant]
V a. − λ (1 − e − λt ) b. λ (1− e − λt )
O V0 1.5V0 3.5V0 5.5V0
− λt
c. λe d. − λe − λt
48 Online JEE Main 2021 July Attempt

15. Two capacitors of capacities 2C and C are joined in Section B : Numerical Type Questions
parallel and charged upto potential V. The battery is 21. Consider an electrical circuit containing a two way switch
removed and the capacitor of capacity C is filled S. Initially S is open and then T1 is connected to T2. As the
completely with a medium of dielectric constant K. The current in R = 6 Ω attains a maximum value of steady
potential difference across the capacitors will now be state level, T1 is disconnected from T2 and immediately
V V
a. b. connected to T3. Potential drop across r = 3 Ω resistor
K+2 K
immediately after T1 is connected to T3 is ……… V.
3V 3V
c. d. (Round off to the nearest integer)
K+2 K
R = 6Ω T2 T3
16. A ball is thrown up with a certain velocity, so that it S T1
reaches a height h. Find the ratio of the two different
h L
times of the ball reaching in both the directions. r = 3Ω
3 6V
2−1 1
a. b.
2+1 3
3− 2 3−1
22. Suppose two planets (spherical in shape) of radii R and
c. d. 2R, but mass M and 9M respectively have a centre to
3+ 2 3+1
centre separation 8R as shown in the figure. A satellite of
17. A 0.07 H inductor and a 12 Ω resistor are connected in mass m is projected from the surface of the planet of
series to a 220 V, 50 Hz AC source. The approximate mass M directly towards the centre of the second planet.
current in the circuit and the phase angle between current The minimum speed v required for the satellite to reach
22 a GM
and source voltage are, respectively. [Take, π as ] the surface of the second planet is , then the
7 7 R
a. 8.8 A and tan −1  b. 88 A and tan −1 
11 11
value of a is …………… .
 6  6
[Take, the two planets are fixed in their position]
c. 0.88 A and tan −1  d. 8.8 A and tan −1 
11 6
 6  11

18. Two identical tennis balls each having mass 2R


R
m and charge q are suspended from a fixed point by 9M
M
threads of length l. What is the equilibrium separation
when each thread makes a small angle θ with the vertical
? 8R
1 1
 q2l  2  q2l  3 23. In Bohr’s atomic model, the electron is assumed to
a. d =   b. d =  
 2πε0 mg   2πε0 mg  revolve in a circular orbit of radius 0.5 Å. If the speed of
1 1 . × 166 m/s, then the current associated with
electron is 22
 q2l 2  3  q2l 2  3 the electron will be …………… × 10−2 mA.
c. d =  2
 d. d =  2 2

 2πε0 m g   2πε 0 m g  22
[Take, π as ]
19. Assertion A If in five complete rotations of the circular 7
scale, the distance travelled on main scale of the screw 24. A radioactive sample has an average life of 30 ms and is
gauge is 5 mm and there are 50 total divisions on decaying. A capacitor of capacitance 200 µF is first
circular scale, then least count is 0.001 cm. charged and later connected with resistor R. If the ratio
Pitch of charge on capacitor to the activity of radioactive
Reason R Least count =
Total divisions on circular scale sample is fixed with respect to time, then the value of R
In the light of the above statements, choose the most should be ……… Ω.
appropriate answer from the options given below. . × 10–31 kg travels in a medium with
25. A particle of mass 91
a. Both A and R are correct and R is the correct explanation of
a speed of 106 m/s and a photon of a radiation of linear
A.
b. Both A and R are correct and R is not the correct
momentum 10–27 kg-m/s travels in vacuum. The
explanation of A. wavelength of photon is ……… times the wavelength of
c. A is correct but R is not correct. the particle.
d. A is not correct but R is correct. 26. A prism of refractive index n1 and another prism of
20. A body takes 4 min to cool from 61° C to 59°C. If the refractive index n2 are stuck together (as shown in the
temperature of the surroundings is 30°C, then the time figure). n1 and n2 depend on λ, the wavelength of light,
taken by the body to cool from 51°C to 49° C is according to the relation
a. 4 min b. 3 min . × 10−14
108
c. 8 min d. 6 min n1 = 12
. +
λ2
July Attempt Online JEE Main 2021 49

. × 10−14
18 . × 109 N/m2)
(Take, Young’s modulus of rubber = 05
and n2 = 145
. +
λ2 28. A transistor is connected in common emitter circuit
The wavelength for which rays incident at any angle on configuration, the collector supply voltage is 10 V and the
the interface BC pass through without bending at that voltage drop across a resistor of 1000 Ω in the collector
interface will be …………… nm. circuit is 0.6 V. If the current gain factor (β) is 24, then the
base current is …………… µA. (Round off to the nearest
D integer)
90° 29. The amplitude of upper and lower side bands of AM
C 70° wave, where a carrier signal with frequency 11.21 MHz,
peak voltage 15 V is amplitude modulated by a 7.7 kHz
n2 a b
N sine wave of 5 V amplitude are V and V,
i 10 10
a
n1 respectively. Then, the value of is ………… .
b
20

60°
°

40°
A B 30. In a uniform magnetic field, the magnetic needle has a
magnetic moment 985 . × 10−2 A/m2 and moment of
27. A stone of mass 20 g is projected from a rubber catapult inertia 5 × 10−6 kg-m2. If it performs 10 complete
of length 0.1 m and area of cross-section 10–6 m2 oscillations in
stretched by an amount 0.04 m. The velocity of the 5 s, then the magnitude of the magnetic field is ……… mT.
projected stone is ………… m/s. [Take, π 2 as 9.85]

Answers For solution scan


the QR code

1. (a) 2. (c) 3. (c) 4. (c) 5. (d) 6. (c) 7. (a) 8. (d) 9. (b) 10. (b)
11. (c) 12. (c) 13. (d) 14. (c) 15. (c) 16. (c) 17. (a) 18. (b) 19. (d) 20. (d)
21. (3) 22. (4) 23. (112) 24. (150) 25. (910) 26. (600) 27. (20) 28. (25) 29. (1) 30. (8)

27 JULY SHIFT II
Objective Type Questions Y Y
1. An electron and proton are separated by a large
distance. The electron starts approaching the proton
c. d.
with energy 3 eV. The proton captures the electrons and δ δ
forms a hydrogen atom in second excited state. The
resulting photon is incident on a photosensitive metal of X X
i i
threshold wavelength 4000 Å. What is the maximum
kinetic energy of the emitted photoelectron? 3. A raindrop with radius R = 02
. mm falls from a cloud at a
a. 7.61 eV height h = 2000 m above the ground. Assume that, the
drop is spherical throughout its fall and the force of
b. 1.41 eV
buoyance may be neglected, then the terminal speed
c. 3.3 eV attained by the raindrop is
d. No photoelectron would be emitted (Take, density of water, ρ w =1000 kg m −3 and density of
2. The expected graphical representation of the variation of air, ρ a = 1.2 kg m −3, g = 10 m/s2, coefficient of viscosity of
angle of deviation δ with angle of incidence i in a prism is air, η = 1.8 × 10–5 N-s m −2)
Y Y a. 250.6 ms −1 b. 43.56 ms −1
c. 4.94 ms −1 d. 14.4 ms −1

a. b.
4. One mole of an ideal gas is taken through an adiabatic
δ δ process, where the temperature rises from 27°C to 37°C.
If the ideal gas is composed of polyatomic molecule that
X X has 4 vibrational modes, which of the following is true?
i i
[Take, R = 8314
. J mol −1 K −1]
50 Online JEE Main 2021 July Attempt

a. Work done by the gas is close to 332 J


b. Work done on the gas is close to 582 J
c. Work done by the gas is close to 582 J Medium /
d. Work done on the gas is close to 332 J (K) –V1
m
5. An object of mass 0.5 kg is executing simple harmonic +V2 +q
motion. It amplitude is 5 cm and time period (T) is 0.2 s. Air
What will be the potential energy of the object at an
instant t = T s starting from mean position ? Assume
t
4
that, the initial phase of the oscillation is zero. d
a. 0.62 J b. 6.2 × 10−3 J c. 1.2 × 103 J d. 6.2 × 103 J  q C1 (V 2 − V1) 
a. tan −1 × 
 mg (C 1 + C 2 ) (d − t ) 
6. Match List I with List II.
 q C 2 (V 2 − V1) 
b. tan −1 × 
List I List II  mg (C 1 + C 2 ) (d − t ) 
A. Capacitance, C I. M1 L1 T −3 A −1  q C 2 (V1 + V 2 ) 
c. tan −1 × 
B. Permittivity of free space, ε0 II. M −1 L −3 T 4 A 2  mg (C 1 + C 2 ) (d − t ) 

C. Permeability of free space, µ 0 III. M −1 L −2 T 4 A 2  q C1 (V1 + V 2 ) 


d. tan −1 × 
D. Electric field, E IV. M1 L1 T −2 A −2  mg (C1 + C 2 ) (d − t ) 
10. Two Carnot engines A and B operate in series such that
Choose the correct answer from the options given below.
engine A absorbs heat at T1 and rejects heat to a sink at
A B C D A B C D
temperature T . engine B absorbs half of the heat rejected
a. III II IV I b. III IV II I
by engine A and rejects heat to the sink at T3. When work
c. IV II III I d. IV III II I
done in both the cases is equal, then the value of T is
7. Given below is the plot of a potential energy function U( x) 2 3 1 2
a. T1 + T3 b. T1 + T3
for a system, in which a particle is in one-dimensional 3 2 3 3
motion, while a conservative force F ( x) acts on it. Suppose 3 1 2 1
c. T1 + T3 d. T1 + T3
that Emech = 8 J, the incorrect statement for this system is 2 3 3 3

U(J) 11. Find the truth table for the function Y of A and B
represented in the following figure.
10
Emech=8J A
8

6 Y
B
4 a. A B Y b. A B Y
2 0 0 0 0 0 1
0 1 1 0 1 0
0 x 1 0 0 1 0 1
x1 x2 x3 x4
1 1 0 1 1 1
[where, KE = kinetic energy]
a. at x > x 4 , KE is constant throughout the region. c. A B Y d. A B Y
b. at x < x1, KE is smallest and the particle is moving at the
0 0 0 0 0 0
slowest speed.
0 1 0 0 1 1
c. at x = x 2 , KE is greatest and the particle is moving at the
fastest speed. 1 0 0 1 0 1
d. at x = x 3 , KE = 4 J. 1 1 1 1 1 1

8. A 100 Ω resistance, a 0.1 µF capacitor and an inductor 12. Figures A and B shown two long straight wires of circular
are connected in series across a 250 V supply at variable cross-section (a and b with a < b), carrying current I which
frequency. Calculate the value of inductance of inductor is uniformly distributed across the cross-section. The
at which resonance will occur. Given that the resonant magnitude of magnetic field B varies with radius r and
frequency is 60 Hz. can be represented as
a. 0.70 H b. 70.3 mH
. × 10−5 H
c. 703 d. 70.3 H
9. A simple pendulum of mass m, length l and charge + q a b
suspended in the electric field produced by two I I
conducting parallel plates as shown. The value of
Fig. A Fig. B
deflection of pendulum in equilibrium position will be
July Attempt Online JEE Main 2021 51

a. 5 and ± 2 b. 4 and ± 3
B B 16 3
c. and ± d. 8 and ± 2
3 2
a. a b. a 17. An automobile of mass m accelerates starting from
b origin and initially at rest, while the engine supplies
b
O r O r constant power P. The position is given as a function of
time by
1 3 1 2
B B
a. 
9P  2 2
b. 
8P  2 3
 ⋅t  ⋅t
 8m   9m 
c. d. 1 1
a b 3 3
c. 
9m  2 2
d. 
8P  2 2
 ⋅t  ⋅t
b a  8P   9m 
O r O r
18. The planet Mars has two Moons, if one of them has a
13. Two identical particles of mass 1 kg each go round a . × 103 km.
period 7 h, 30 min and an orbital radius of 90
circle of radius R, under the action of their mutual Find the mass of Mars.
gravitational attraction. The angular speed of each  4π2 11 −1 −2 2
particle is  Take, G = 6 × 10 N m kg 
 
G 1 G 1 1 2G
a. b. c. d. . × 1019 kg
a. 596 . × 1021 kg
b. 325
2R 3 2 R3 2R G R3
. × 1025 kg
c. 702 . × 1023 kg
d. 600
14. Consider the following statements. 19. A particle of mass M originally at rest is subjected to a
A. Atoms of each element emit characteristics spectrum. force whose direction is constant but magnitude varies
B. According to Bohr's postulate, an electron in a with time according to the relation
hydrogen atom, revolves in a certain stationary orbit.  t − T  2
F = F 0 1 −   
C. The density of nuclear matter depends on the size of
  T  
the nucleus.
D. A free neutron is stable but a free proton decay is where, F 0 and T are constants. The force acts only for the
possible. time interval 2T . The velocity v of the particle after time
E. Radioactivity is an indication of the instability of 2T is
2F0T F0T
nuclei. a. b.
M 2M
Choose the correct answer from the options given 4F T FT
below. c. 0 d. 0
3M 3M
a. A, B, C, D and E b. A, B and E
c. B and D d. A, C and E 20. The resistance of a conductor at 15°C is 16 Ω and at
15. What will be the magnitude of electric field at point O as 100°C is 20 Ω. What will be the temperature coefficient
shown in figure? Each side of the figure is l and of resistance of the conductor?
perpendicular to each other. a. 0.010°C −1
b. 0.033°C −1
A(–q) C l D
(+q) c. 0.003°C −1
(2q) d. 0.042°C −1
l l
(+)q O G(2q) Section B : Numerical Type Questions
B
l l 21. In the given figure, two wheels P and Q are connected by
l l a belt B. The radius of P is three times as that of Q. In
(2q) l (q) (–q) case of same rotational kinetic energy, the ratio of
E F H I 
rotational inertias  1  will be x : 1. The value of x will be
 I2 
1 q 1 q
a. b. (2 2 − 1)
4 πε0 l 2 4 πε0 (2l 2 ) ………… .
q 1 2q
c. d. ( 2)
4 π ε0 (2l ) 2 4 πε0 2l 2

16. A physical quantity y is represented by the formula P Q


y = m 2r −4 g x l −3/ 2. If the percentage errors found in 3R
R
y , m , r , l and g are 18, 1, 0.5, 4 and p respectively, then
B
find the value of x and p.
52 Online JEE Main 2021 July Attempt

22. The difference in the number of waves when yellow light 27. For the circuit shown, the value of current at time t = 32
. s
propagates through air and vacuum columns of the will be ………… A.
same thickness is one. The thickness of the air column is
………… mm.
10
[Take, refractive index of air = 10003
. , wavelength of
V (t)
yellow light in vacuum = 6000 Å]
5
23. The maximum amplitude for an amplitude modulated
wave is found to be 12 V, while the minimum amplitude 0
is found to be 3 V. The modulation index is 06
. x, where x 1 2 3 4
Fig. (1) t (s)
is ………… .
R=1Ω
24. In the given figure, the magnetic flux through the loop
increases according to the relation φ B (t) = 10t 2 + 20t,
where φ B is in milliwebers and t is in seconds.
V (t) I 5V
The magnitude of current through R = 2 Ω resistor at
t = 5 s is ………… mA.
Fig. (2)
R=2Ω
[Voltage distribution V (t) is shown by Fig. (1) and the
circuit is shown in Fig. (2).]
28. A small block slides down from the top of hemisphere of
radius R = 3 m as shown in the figure. The height h at
which the block will lose contact with the surface of the
sphere is ………… m. (Assume there is no friction
25. A particle executes simple harmonic motion represented between the block and the hemisphere)
by displacement function as
x(t) = A sin (ωt + φ)
If the position and velocity of the particle at t = 0 s are 2 (R–h)
A
cm and 2ω cms −1 respectively, then its amplitude is x 2 R
cm, where the value of x is ………… . θ h

26. A swimmer wants to cross a river from point A to point B. O


Line AB makes an angle of 30° with the flow of river.
Magnitude of velocity of the swimmer is same as that of 29. The K α X-ray of molybdenum has wavelength 0.071 nm.
the river. The angle θ with the line AB should be ……°, so If the energy of a molybdenum atom with a K electron
that the swimmer reaches point B. knocked out is 27.5 keV, the energy of this atom when an
L electron is knocked out will be ……… keV.
B
(Round off to the nearest integer)
[Take, h = 4.14 × 10−15 eV-s, c = 3 × 108 ms −1]
30. The water is filled upto height of 12 m in a tank having
θ vertical sidewalls. A hole is made in one of the walls at a
30º depth h below the water level. The value of h for which
A the emerging stream of water strikes the ground at the
maximum range is ………… m.

Answers For solution scan


the QR code
1. (b) 2. (b) 3. (c) 4. (b) 5. (a) 6. (a) 7. (b) 8. (d) 9. (c) 10. (d)
11. (b) 12. (c) 13. (b) 14. (b) 15. (d) 16. (c) 17. (d) 18. (d) 19. (c) 20. (c)
21. (9) 22. (2) 23. (1) 24. (60) 25. (2) 26. (30) 27. (1) 28. (2) 29. (10) 30. (6)
August Attempt Online JEE Main 2021 53

26 AUGUST SHIFT I
Section A : Objective Type Questions 6. An electric appliance supplies 6000 J/min heat to the
1. The fractional change in the magnetic field intensity at a system. If the system delivers a power of 90W. How long
distance r from centre on the axis of current carrying coil it would take to increase the internal energy by
of radius a to the magnetic field intensity at the centre of . × 103 J?
25
the same coil is (Take, r < a ) . × 102 s
a. 25 b. 4.1 × 101 s
c. 2.4 × 103 s . × 101 s
d. 25
3 a2 2 a2 2 r2 3 r2
a. b. c. d.
2 r2 3 r2 3 a2 2 a2 7. An inductor coil stores 64 J of magnetic field energy and
dissipates energy at the rate of 640 W when a current of
2. The magnitude of vectors OA, OB, and OC in the given 8A is passed through it. If this coil is joined across an ideal
figure are equal. The direction of OA + OB − OC with
battery, find the time constant of the circuit in seconds.
X-axis will be
a. 0.4 b. 0.8 c. 0.125 d. 0.2
Y
8. A series L-C-R circuit driven by 300 V at a frequency of 50
C
A Hz contains a resistance R = 3 kΩ, an inductor of
inductive reactance X L = 250 πΩ and an unknown
capacitor. The value of capacitance to maximise the
45° 30°
O 60°
X average power should be (Take, π 2 = 10)
a. 4 µF b. 25 µF c. 400 µF d. 40 µF

B 9. Identify the logic operation carried out by the given


circuit.
−1 (1 − 3 − 2 ) −1 ( 3 − 1 + 2)
a. tan b. tan X
(1 + 3 + 2) (1 + 3 − 2) A
( 3 − 1 + 2) (1 + 3 − 2) Z
c. tan −1 d. tan −1
(1 − 3 + 2 ) (1 − 3 − 2 )
B
Y
3. Car B overtakes another car A at a relative speed of 40
ms −1. How fast will the image of car B appear to move in a. OR b. AND c. NOR d. NAND
the mirror of focal length 10 cm fitted in car A, when the 10. A particular hydrogen like ion emits radiation of
car B is 1.9 m away from the car A? frequency 292. × 1015 Hz when it makes transition from
a. 4 ms −1 b. 0.2 ms −1 n = 3 to n = 1. The frequency in Hz of radiation emitted in
c. 40 ms −1 d. 0.1 ms −1 transition from n = 2 to n = 1will be
a. 0.44 × 1015 . × 1015
b. 657 c. 4.38 × 1015 d. 2.46 × 1015
4. Inside a uniform spherical shell
I. the gravitational field is zero.
11. In a photoelectric experiment ultraviolet light of
wavelength 280 nm is used with lithium cathode having
II. the gravitational potential is zero. work-function φ = 25 . eV. If the wavelength of incident
III. the gravitational field is same everywhere. light is switched to 400 nm, find out the change in the
. × 10−34 Js,
stopping potential. (h = 663
IV. the gravitation potential is same everywhere.
and c = 3 × 108 ms −1)
V. All of the above
a. 1.3 V b. 1.1 V c. 1.9 V d. 0.6 V
Choose the most appropriate answer from the options
given below .
12. In the given figure, the emf of the cell is 2.2 V and if
. Ω. Calculate the power
internal resistance is 06
a. I, III and IV b. Only V dissipated in the whole circuit
c. I, II and III d. II, III and IV
4W
5. Two narrow bores of diameter 5.0 mm and 8.0 mm are
4W 4W
joined together to form a U-shaped tube open at both
A B
ends. If this U-tube contains water, what is the difference
in the level of two limbs of the tube. 2W 8W
[Take surface tension of water T = 73. × 10−2 Nm−1, angle
−2
of contact = 0, g = 10 ms and density of water 8W
. × 103 kg m −3]
= 10
a. 3.62 mm b. 2.19 mm
c. 5.34 mm d. 4.97 mm 2.2 V, r =0.6W
a. 1.32 W b. 0.65 W c. 2.2 W d. 4.4 W
54 Online JEE Main 2021 August Attempt

13. A solid metal sphere of radius R having charge q is (Given, resistivities of iron and copper-nickel alloy wire are
enclosed inside the concentric spherical shell of inner 12 µΩ cm and 51 µΩ cm respectively)
radius a and outer radius b as shown in figure. The a. 82 m b. 97 m c. 110 m d. 90 m
approximate variation electric field as a function of
17. The initial mass of a rocket is 1000 kg. Calculate at what
distance r from centre O is given by
rate the fuel should be burnt, so that the rocket is given
an acceleration of 20 ms −1. The gases come out at a
relative speed of 500 ms −1 with respect to the rocket
[Use, g = 10 m/s 2]
b . × 102 kg s −1
a. 60 b. 500 kg s −1
c. 10 kg s −1 d. 60 kg s −1
a 18. If E , L, M and G denote the quantities as energy, angular
momentum, mass and constant of gravitation
respectively, then the dimension of P in the formula
a. E P = EL2M −5G −2 is
a. [M 0L1T 0 ] b. [M −1L−1T 2 ] c. [M1L1T −2 ] d. [M 0L0 T 0 ]
19. The material filled between the plates of a parallel plate
capacitor has resistivity
200 Ωm. The value of capacitance of the capacitor is 2pF.
R a R
r If a potential difference of 40 V is applied across the
plates of the capacitor, then the value of leakage current
b. E flowing out of the capacitor is (Given, the value of
relative permittivity of material is 50.)
a. 9.0 µA b. 9.0 mA c. 0.9 mA d. 0.9 µA
20. Statement I By doping silicon semiconductor with
pentavalent material, the electrons density increases.
R a b
r Statement II The n-type semiconductor has net negative
c. E charge.
In the light of the above statements, choose the most
appropriate answer from the options given below.
a. Statement I is true but statement II is false.
b. Statement I is false but statement II is true.
R a b
r
c. Both statement I and statement II are true.
d. E d. Both statement I and statement II are false.

Section B : Numerical Type Questions


21. A uniform chain of length 3 m and mass 3 kg overhangs
a smooth table with 2 m laying on the table. If k is the
R a b r
kinetic energy of the chain in joule as it completely slips
14. The rms speeds of the molecules of hydrogen, oxygen off the table, then the value of k is ……… .
and carbondioxide at the same temperature are vH , v O (Take, g = 10 m/s 2)
and VCO 2 respectively, then 22. The electric field in a plane electromagnetic wave is given
a. vH > v O > v CO2 b. v CO2 > v O > vH
by
c. vH = v O > v CO2 d. vH = v O = v CO2
  05
. × 103   rad   V $
15. In a screw gauge, 5th division of the circular scale E = 200cos   . × 1011
 x −  15 × t  j
  m
coincides with the reference line when the ratchet is  m  s 
closed. There are 50 divisions on the circular scale, and If this wave falls normally on a perfectly reflecting surface
the main scale moves by 0.5 mm on a complete rotation. having an area of 100 cm 2. If the radiation pressure
For a particular observation the reading on the main
exerted by the EM wave on the surface during a 10 min
scale is 5 mm and the 20th division of the circular scale x
coincides with reference line. Calculate the true reading. exposure is 9 ⋅ Find the value of x.
10
a. 5.00 mm b. 5.25 mm c. 5.15 mm d. 5.20 mm
23. A source and a detector move away from each other in
16. What equal length of an iron wire and a copper-nickel absence of wind with a speed of 20 m/s with respect to
alloy wire, each of 2 mm diameter connected parallel to the ground. If the detector detects a frequency of 1800
give an equivalent resistance of 3Ω? Hz of the sound coming from the source, then the
August Attempt Online JEE Main 2021 55

original frequency of source considering speed of sound 29. White light is passed through a double slit and
in air 340 m/s will be ........ Hz. interference is observed on a screen 1.5 m away. The
24. Two spherical balls having equal masses with radius of 5 separation between the slits is 0.3 mm. The first violet
cm each are thrown upwards along the same vertical and red fringes are formed 2.0 mm and 3.5 mm away
direction at an interval of 3s with the same initial velocity from the central white fringes. The difference in
of 35 m/s, then these balls collide at a height of .......... m. wavelengths of red and violet light is ......... nm.
(Take, g = 10 m/s 2) 30. Consider a badminton racket with length scales as
shown in the figure.
25. A soap bubble of radius 3 cm is formed inside the
another soap bubble of radius 6 cm. The radius of an
equivalent soap bubble which has the same excess
pressure as inside the smaller bubble with respect to the P
atmospheric pressure is ........ cm.
26. An amplitude modulated wave is represented by A
. cos 12560t) sin(111 × 104t) V.
C m (t) = 10 (1 + 02
The modulating frequency in kHz will be …… . r
2
27. Two short magnetic dipoles m1 and m 2 each having
magnetic moment of 1 Am2 are placed at point O and P, 6r 2r
respectively. The distance between OP is 1 m. The torque If the mass of the linear and circular portions of the
experienced by the magnetic dipole m 2 due to the badminton racket are same (M) and the mass of the
presence of m1 is …… × 10−7 Nm. threads are negligible, the moment of inertia of the
28. Two travelling waves produces a standing wave racket about an axis perpendicular to the handle and in
r
represented by equation. the plane of the ring at, distance from the end A of the
2
y = 10 . cm−1 ) x sin(785
. mm cos (157 . s −1 ) t. handle will be ........ Mr 2.
The node closest to the origin in the region x > 0 will be at x
is ……… cm.

Answers For solution scan


the QR code

1. (d) 2. (a) 3. (d) 4. (a) 5. (b) 6. (a) 7. (d) 8. (a) 9. (c) 10. (d)
11. (a) 12. (c) 13. (a, b) 14. (a) 15. (c) 16. (b) 17. (d) 18. (d) 19. (c) 20. (a)
21. (40) 22. (354) 23. (2025) 24. (50) 25. (2) 26. (2) 27. (1) 28. (1) 29. (300) 30. (52)

26 AUGUST SHIFT II
Objective Type Questions 3. A particle of mass m is suspended from a ceiling through
1. The temperature of equal masses of three different a string of length L. The particle moves in a horizontal
L
liquids x , y and z are 10ºC, 20ºC and 30ºC, respectively. circle of radius r such that r = . The speed of particle
The temperature of mixture when x is mixed with y is 2
16ºC and that when y is mixed with z is 26°C. The will be
temperature of mixture when x and z are mixed will be a. rg b. 2rg c. 2 rg d. rg / 2
−3
a. 28.32ºC b. 25.62ºC c. 23.84ºC d. 20.28ºC 4. A cylindrical container of volume 4.0 × 10 m3 contains
2. The de-Broglie wavelength of a particle having kinetic one mole of hydrogen and two moles of carbon dioxide.
energy E is λ. How much extra energy must be given to Assume the temperature of the mixture is 400 K. The
this particle, so that the de-Broglie wavelength reduces pressure of the mixture of gases is
to 75% of the initial value ? [Take, gas constant = 8.3 J mol− 1K − 1]
1 7 16 a. 249 × 101 Pa b. 24.9 × 103 Pa
a. E b. E c. E d. E
9 9 9 c. 24.9 × 105 Pa d. 24.9 Pa
56 Online JEE Main 2021 August Attempt

5. The angle between vector A and ( A − B) is a. 14.9 kg / m3 b. 75. × 101 kg / m3


. × 102 kg / m3
c. 75 . × 102 kg / m3
d. 149
10. A parallel-plate capacitor with plate area A has
B separation d between the plates. Two dielectric slabs of
dielectric constant K1 and K 2 of same area A / 2 and
120º thickness d / 2 are inserted in the space between the
A
plates. The capacitance of the capacitor will be given by
β
–B
+Q

K1
 B  d
 − 
−1 2  − 1
A 
a. tan  b. tan   K2
 3  0.7B 
A −B 
 2 
–Q
 3B   B cos θ 
c. tan − 1  d. tan − 1 
 2A − B   A − B sin θ 
ε0 A  1 K 1K 2  ε0 A  1 K 1K 2 
 +  +
6. A light beam is described by E = 800 sin  ωt −  . An  
x a. b.
d  2 K1 + K 2  d  2 2 (K 1 + K 2 ) 
c
electron is allowed to move normal to the propagation of ε0 A  1 K 1 + K 2  ε0 A  1 2(K 1 + K 2 ) 
c.  +  d.  + 
light beam with a speed 3 × 107 ms − 1. What is the d 2 K 1K 2  d 2 K 1K 2 
maximum magnetic force exerted on the electron?
11. A bomb is dropped by fighter plane flying horizontally.
a. 1.28 × 10− 17 N b. 1.28 × 10− 18 N
To an observer sitting in the plane, the trajectory of the
c. 12.8 × 10− 17 N d. 12.8 × 10− 18 N
bomb is a
7. The two thin co-axial rings, each of radius a and having a. hyperbola
charges + Q and − Q respectively, are separated by a b. parabola in the direction of motion of plane
distance of s. The potential difference between the c. straight line vertically down the plane
centres of the two rings is d. parabola in a direction opposite to the motion of plane
1  1 
a.
Q
 +
1
 b.
Q
 +
1

12. At time t = 0, a material is composed of two radioactive
2 πε0  a 2
s + a  2 4 πε0  a 2
s + a  2
atoms A and B, where NA (0) = 2NB (0). The decay constant
of both kind of radioactive atoms is λ. However, A
Q 1 1  Q 1 1 
disintegrates to B and B disintegrates to C . Which of the
c.  −  d.  − 
4 πε0  a s 2 + a 2  2 πε0  a s 2 + a 2  following figures represents the evolution of NB (t) / NB (0)
 
with respect to time t ?
8. If you are provided a set of resistances 2Ω , 4 Ω , 6Ω and
NA (0) = Number of A atoms at t = 0 
8Ω. Connect these resistances, so as to obtain an N (0) = Number of B atoms at t = 0
46  B 
equivalent resistance of Ω.
3
a. 4 Ω and 6 Ω are in parallel with 2 Ω and 8 Ω in series. 1
b. 6 Ω and 8 Ω are in parallel with 2 Ω and 4 Ω in series.
c. 2 Ω and 6 Ω are in parallel with 4 Ω and 8 Ω in series. NB(t)
a.
d. 2 Ω and 4 Ω are in parallel with 6 Ω and 8 Ω in series. NB(0)
9. The solid cylinder of length 80 cm and mass M has a
radius of 20 cm. Calculate the density of the material t
used, if the moment of inertia of the cylinder about an 1/2λ
axis CD parallel to AB as shown in figure is 2.7 kg m2.
A C
1

b. NB(t)
L M NB(0)
L/2

t
1/λ
r B D
August Attempt Online JEE Main 2021 57

17. Match List-I with List-II.


c. 1 List-I List-II

NB(t) A. Magnetic induction 1. [ML2 T − 2 A − 1]


NB(0) B. Magnetic flux 2. [ML−1A]
C. Magnetic permeability 3. [MT − 2 A − 1]
t
1/2λ D. Magnetisation 4. [MLT − 2 A − 2 ]

Choose the most appropriate answer from the options


given below.
1 A B C D A B C D
a. 2 4 1 3 b. 2 1 4 3
d. NB(t) c. 3 2 4 1 d. 3 1 4 2
NB(0) 18. In the given circuit the AC source has ω = 100 rad s − 1.
Considering the inductor and capacitor to be ideal, what
t will be the current I flowing through the circuit?
1/2λ
100 µF 100 Ω
13. A transmitting antenna at top of a tower has a height of
50 m and the height of receiving antenna is 80 m. What
is range of communication for line of sight (LOS) mode ?
[Use radius of Earth = 6400 km]
I
a. 45.5 km b. 80.2 km c. 144.1 km d. 57.28 km 0.5 H 50 Ω
14. A refrigerator consumes an average 35 W power to
operate between temperature − 10º C to 25ºC. If there is
no loss of energy, then how much average heat per 200 V
second does it transfer ?
a. 5.9 A b. 4.24 A c. 0.94 A d. 6 A
a. 263 J/s b. 298 J/s c. 350 J/s d. 35 J/s
15. An electric bulb of 500 W at 100 V is used in a circuit 19. If the length of the pendulum in pendulum clock
having a 200 V supply. Calculate the resistance R to be increases by 0.1%, then the error in time per day is
connected in series with the bulb, so that the power a. 86.4 s b. 4.32 s
delivered by the bulb is 500 W. c. 43.2 s d. 8.64 s
a. 20 Ω b. 30 Ω c. 5 Ω d. 10 Ω 20. Two blocks of masses 3 kg and 5 kg are
16. Four NOR gates are connected as shown in figure. connected by a metal wire going over a
smooth pulley. The breaking stress of the
The truth table for the given figure is
metal is (24 / π ) × 102 Nm− 2. What is the
A minimum radius of the wire?
3kg
(Take, g = 10 ms − 2)
Y a. 125 cm b. 1250 cm
5kg
c. 12.5 cm d. 1.25 cm

Section B : Numerical Type Questions


B
a. b. 21. Two waves are simultaneously passing through a string
and their equations are y1 = A1sin k( x − vt),
A B Y A B Y
y 2 = a 2 sin k( x − vt + x 0).
0 0 1 0 0 0 [Given, amplitudes A1 = 12 mm and
0 1 0 0 1 1 A2 = 5 mm, x 0 = 35 . cm and wave number k = 6.28 cm− 1].
1 0 1 1 0 1 The amplitude of resulting wave will be .......... mm.
1 1 0 1 1 0
22. A source of light is placed in front of a screen. Intensity
c. d. of light on the screen is I. Two polaroids P1 and P2 are so
A B Y A B Y placed in between the source of light and screen that the
intensity of light on screen is I / 2. P2 should be rotated by
0 0 0 0 0 1
an angle of ........... (degrees), so that the intensity of light
0 1 1 0 1 0 on the screen becomes 3I / 8.
1 0 0 1 0 0
1 1 1 1 1 1
58 Online JEE Main 2021 August Attempt

23. If the maximum value of accelerating potential provided 28. For the given circuit, the power across Zener diode is
by a radio frequency oscillator is 12 kV. The number of ............ mW.
revolution made by a proton in a cyclotron to achieve
1 kΩ
one sixth of the speed of light is ........... .
[Given, mp = 1.67 × 10 − 27 kg, e = 1.6 × 10− 19 C,
c = 3 × 108 m/s] Iz

24 V R L = 5kΩ
24. The acceleration due to gravity is found upto an accuracy V z = 10 V
of 4% on a planet. The energy supplied to a simple
pendulum to known mass m to undertake oscillations of
time period T is being estimated. If time period is
measured to an accuracy of 3%, the accuracy to which E
29. An object is placed at a distance of 12 cm from a convex
lens. A convex mirror of focal length 15 cm is placed on
is known as ..........%.
other side of lens at 8 cm as shown in the figure. Image
25. A circular coil of radius 8.0 cm and 20 turns is rotated of object coincides with the object.
about its vertical diameter with an angular speed of 50
rad s − 1 in a uniform horizontal magnetic field of
Image
30. × 10− 2 T. The maximum emf induced in the coil will be
.......... × 10− 2 V. Object Image in
the absence
(rounded off to the nearest integer.) of mirror
12 cm 8 cm
26. Two simple harmonic motions are represented by the
equations When the convex mirror is removed, a real and inverted
π image is formed at a position. The distance of the image
x1 = 5sin 2 πt +  and x2 = 5 2(sin2 πt + cos 2 πt ) from the object will be ........ cm.
 4
The amplitude of second motion is .............. times the 30. The coefficient of static friction between two blocks is 0.5
amplitude in first motion. and the table is smooth. The maximum horizontal force
that can be applied to move the blocks together is .......N.
27. A coil in the shape of an equilateral triangle of side 10 cm
(Take, g = 10 ms − 2)
lies in a vertical plane between the pole pieces of
permanent magnet producing a horizontal magnetic Table 1 kg µ=0.5
field 20 mT. The torque acting on the coil when a current
2kg F
of 0.2 A is passed through it and its plane becomes
parallel to the magnetic field will be x × 10− 5 Nm. The
value of x is.......... .

Answers For solution scan


the QR code

1. (c) 2. (b) 3. (a) 4. (c) 5. (c) 6. (a,d) 7. (d) 8. (d) 9. (d) 10. (a)
11. (c) 12. (c) 13. (d) 14. (a) 15. (a) 16. (d) 17. (*) 18. (*) 19. (c) 20. (c)
21. (7) 22. (30) 23. (543) 24. (14) 25. (60) 26. (2) 27. (3) 28. (120) 29. (50) 30. (15)

27 AUGUST SHIFT I
Section A : Objective Type Questions 2. There are 1010 radioactive nuclei in a given radioactive
element. Its half-life time is 1min.
1. A uniformly charged disc of radius R having surface How many nuclei will remain after 30 s? ( 2 = 1414)
.
charge density σ is placed in the xy-plane with its centre
a. 2 × 1010 b. 7 × 109 c. 105 d. 4 × 1010
at the origin. Find the electric field intensity along the
Z-axis at a distance Z from origin 3. Which of the following is not a dimensionless quantity ?
σ  Z  σ  Z 
a. E = 1−  b. E = 1+  a. Relative magnetic permeability (µ r )
2ε0  2 2 
(Z + R )  2ε0  2 2 
(Z + R )  b. Power factor
 
   c. Permeability of free space (µ 0 )
2ε0  1 σ 1 1 
c. E = + Z d. E =  + d. Quality factor
σ  (Z 2 + R 2 )  2ε0  (Z 2 + R 2 ) Z 2 
   
August Attempt Online JEE Main 2021 59

4. If E and H represent the intensity of electric field and 11. For a transistor in CE mode to be used as an amplifier, it
magnetising field respectively, then the unit of E / H will must be operated in
be a. both cut-off and saturation b. saturation region only
a. ohm b. mho c. joule d. newton c. cut-off region only d. the active region only
5. The resultant of these forces OP, OQ, OR, OS and OT is 12. An ideal gas is expanding such that pT 3 = constant. The
approximately ...... N. coefficient of volume expansion of the gas is
[Take, 3 = 1.7, 2 = 1. 4 and given $i and $j unit vectors a. 1/ T b. 2 / T c. 4 / T d. 3 / T
along X , Y axis] 13. In a photoelectric experiment, increasing the intensity of
P incident light
20N
T Y a. increases the number of photons incident and also
15N
increases the KE of the ejected electrons.
Q b. increases the frequency of photons incident and increases
60° 30° the KE of the ejected electrons.
10N
c. increases the frequency of photons incident and the KE of
30° the ejected electrons remains unchanged.
X' 45° 45° X d. increases the number of photons incident and the KE of the
O
ejected electrons remains unchanged.
14. A bar magnet is passing through a conducting loop of
15N radius R with velocity v. The radius of the bar magnet is
20N R
S Y' such that it just passes through the loop. The induced
emf in the loop can be represented by the approximate
a. 9.25i$ + 5$j b. 3i$ + 15$j c. 2.5$i − 14 .5$j d. − 1.5i$ − 155
. $j
curve
6. A balloon carries a total load of 185 kg at normal l
pressure and temperature of 27ºC. What load will the R
balloon carry on rising to a height at which the
barometric pressure is 45 cm of Hg and the temperature N S
is − 7ºC? [Assuming, the volume constant.]
a. 181.46 kg b. 214.15 kg c. 219.07 kg d. 123.54 kg loop
v
7. An object is placed beyond the centre of curvature C of
the given concave mirror. If the distance of the object is
d1 from C and the distance of the image formed is d 2 emf
from C , the radius of curvature of this mirror is
2d1d 2 2d1d 2 d1d 2 d1d 2 a. t
a. b. c. d.
d1 − d 2 d1 + d 2 d1 + d 2 d1 − d 2 I/v

8. A huge circular arc of length 4.4 ly subtends an angle 4s


at the centre of the circle. How long it would take for a
body to complete 4 rev if its speed is 8 AU per sec ?
[Given, 1 ly = 9.46 × 1015 m,1 AU = 1.5 × 1011 m]
a. 4.1 × 108 s b. 4 .5 × 1010 s c. 3.5 × 106 s d. 7.2 × 108 s emf
9. Calculate the amount of charge on capacitor of 4 µF. The b.
internal resistance of battery is 1 Ω. t
I/v
2µF
4µF 6Ω

5V
2µF
A B
emf
4Ω I/v
c.
a. 8 µC b. zero c. 16 µC d. 4 µC t

10. Moment of inertia of a square plate of side l about the


axis passing through one of the corner and
perpendicular to the plane of square plate is given by
Ml 2 Ml 2 2 2
a. b. Ml 2 c. d. Ml
6 12 3
60 Online JEE Main 2021 August Attempt

x
emf
A B
I/v O t
d. C
t

The potential energy U( x) versus time (t) plot of the


particle is correctly shown in figure :

15. Two ions of masses 4 amu and 16 amu have charges +2e U(x)
and +3e, respectively. These ions pass through the
a. O t
region of constant perpendicular magnetic field. The A B C
kinetic energy of both ions is same. Then,
a. lighter ion will be deflected less than heavier ion
b. lighter ion will be deflected more than heavier ion
U(x)
c. both ions will be deflected equally A B
d. no ion will be deflected b. O t
C
16. Find the distance of the image from object O, formed by
the combination of lenses in the figure.

f =+10cm f =–10cm f =+30cm U(x)


c. A B
O t
C

O U(x)
A B
d. O t
C

30cm 5cm 10cm

a. 75 cm b. 10 cm Section B : Numerical Type Questions


c. 20 cm d. infinity
21. A body of mass 2M splits into four masses
17. In Millikan’s oil drop experiment, what is viscous force {m, M − m , m , M − m}, which are rearranged to form a
acting on an uncharged drop of radius 2.0 × 10− 5 m and square as shown in the figure. The ratio of
M
for which,
. × 103kgm− 3? Take viscosity of liquid
density 12 m
. × 10− 5Nsm− 2. (Neglect buoyancy due to air).
= 18 the gravitational potential energy of the system becomes
a. 38. × 10− 11N . × 10− 10 N
b. 39
maximum is x : 1. The value of x is……… .
. × 10− 10 N
c. 18 . × 10− 10 N
d. 58 m M–m
18. Electric field in a plane electromagnetic wave is given by
E = 50sin(500x − 10 × 1010t) V/m.
d
The velocity of electromagnetic wave in this medium is
(Given, c = speed of light in vacuum)
3 m
a. c b. c M–m
2 d
2 c
c. c d.
3 2 22. The alternating current is given by

19. Five identical cells each of internal resistance 1 Ω and i =  42 sin t  + 10 A
  T  
emf 5V are connected in series and in parallel with an
The rms value of this current is ........ A.
external resistance R . For what value of R , current in
series and parallel combination will remain the same ? 23. A uniform conducting wire of length is 24 a, and
a. 1Ω b. 25 Ω resistance R is wound up as a current carrying coil in the
c. 5 Ω d. 10 Ω shape of an equilateral triangle of side a and then in the
form of a square of side a . The coil is connected to a
20. The variation of displacement with time of a particle
voltage source V0. The ratio of magnetic moment of the
executing free simple harmonic motion is shown in the
coils in case of equilateral triangle to that for square is
figure.
1: y . The value of where y is ……… .
August Attempt Online JEE Main 2021 61

24. A circuit is arranged as shown in figure. The output 27. If the velocity of a body related to displacement x is given
voltage V0 is equal to ....... V. by v = 5000 + 24 x m/ s , then the acceleration of the
body is ...... m / s 2.
V0
D1 28. A rod CD of thermal resistance 10.0 kW − 1 is joined at the
R middle of an identical rod AB as shown in figure. The end
R
5V A , B and D are maintained at 200ºC, 100ºC and 125ºC
D2 respectively. The heat current in CD is P watt. The value
R of P is ....... .
5V A B

200°C C 100°C

25. First, a set of n equal resistors of 10 Ω each are


connected in series to a battery of emf 20 V and internal 125°C D
resistance 10 Ω. A current l is observed to flow. Then, the
n resistors are connected in parallel to the same battery.
29. Two persons A and B perform same amount of work in
moving a body through a certain distance d with
It is observed that the current is increased
application of forces acting at angle 45º and 60º with the
20 times, then the value of n is ......... .
direction of displacement respectively. The ratio of force
26. Two cars X and Y are approaching each other with applied by person A to the force applied by person B is
velocities 36 km/h and 72 km/h respectively. The 1/ x . The value of x is ....... .
frequency of a whistle sound as emitted by a passenger
30. A transmitting antenna has a height of 320 m and that of
in car X, heard by the passenger in car Y is 1320 Hz. If the
receiving antenna is 2000 m. The maximum distance
velocity of sound in air is 340 m/s, the actual frequency
between them for satisfactory communication in line of
of the whistle sound produced is ........ Hz.
sight mode is d. The value of d is ........ km.

Answers For solution scan


the QR code

1. (a) 2. (b) 3. (c) 4. (a) 5. (a) 6. (d) 7. (a) 8. (b) 9. (a) 10. (d)
11. (d) 12. (c) 13. (d) 14. (c) 15. (b) 16. (a) 17. (b) 18. (c) 19. (a) 20. (d)
21. (2) 22. (11) 23. (3) 24. (5) 25. (20) 26. (1210) 27. (12) 28. (2) 29. (2) 30. (224)

27 AUGUST SHIFT II
Section A : Objective Type Questions 2. The boxes of masses 2 kg and 8 kg are connected by a
1. Curved surfaces of a plano-convex lens of refractive massless string passing over smooth pulleys. Calculate
index µ1 and a plano-concave lens of refractive index µ 2 the time taken by box of mass 8 kg to strike the ground
have equal radius of curvature as shown in figure. Find starting from rest.
the ratio of radius of curvature to the focal length of the (Use, g = 10 m/s 2)
combined lenses.

µ1 µ2

8kg
1
a. b. µ 1 − µ 2
µ 2 − µ1 20cm
1 2kg
c. d. µ 2 − µ 1
µ1 − µ 2
a. 0.34 s b. 0.2 s c. 0.25 s d. 0.4 s
62 Online JEE Main 2021 August Attempt

IC I 10. A constant magnetic field of 1 T is applied in the x > 0


3. For a transistor α and β are given as α = and β = C .
IE IB
region. A metallic circular ring of radius 1 m is moving
Then, the correct relation between α and β will be with a constant velocity of 1 m/s along the X-axis. At t = 0
1− β α β s, the centre of O of the ring is at x = − 1m. What will be
a. α = b. β = c. αβ = 1 d. α =
β 1− α 1− β the value of the induced emf in the ring at t = 1s?
(Assume the velocity of the ring does not change.)
4. Water drops are falling from a nozzle of a shower onto
the floor, from a height of 9.8 m. The drops fall at a
regular interval of time. When the first drop strikes the
floor, at that instant, the third drop begins to fall. Locate O v
the position of second drop from the floor when the first
drop strikes the floor.
a. 4.18 m b. 2.94 m c. 2.45 m d. 7.35 m x=0 x
5. Two discs have moments of inertia I1 and I2 about their
a. E
respective axes perpendicular to the plane and passing
through the centre. They are rotating with angular
speeds, ω1 and ω 2 respectively and are brought into
contact face to face with their axes of rotation co-axial.
The loss in kinetic energy of the system in the process is
given by R a R
r
2
II (I − I ) ω1ω 2
a. 1 2 (ω1 − ω 2 ) 2 b. 1 2 b. E
(I1 + I 2 ) 2(I1 + I 2 )
I1I 2 (ω 1 − ω 2 ) 2
c. (ω1 − ω 2 ) 2 d.
2(I1 + I 2 ) 2(I1 + I 2 )

6. Three capacitors C 1 = 2µF, C 2 = 6µF andC 3 = 12µF are


R a b
connected as shown in figure. Find the ratio of the r
charges on capacitors C 1, C 2 and C 3, respectively. a. 1 V b. 2π V c. 2 V d. 0

D 11. A mass of 50 kg is placed at the centre of a uniform


spherical shell of mass 100 kg and radius 50 m. If the
A C2 C3 B gravitational potential at a point, 25 m from the centre is
V kg/m. The value of V is
C1 a. − 60 G b. + 2 G c. − 20 G d. − 4 G

V
12. For full scale deflection of total 50 divisions, 50 mV
voltage is required in galvanometer. The resistance of
a. 2 : 1 : 1 b. 2 : 3 : 3
galvanometer if its current sensitivity is 2 div/mA will be
c. 1 : 2 : 2 d. 3 : 4 : 4
a. 1 Ω b. 5 Ω c. 4 Ω d. 2 Ω
7. The colour coding on a carbon resistor is shown in the 13. A monochromatic neon lamp with wavelength of 670.5
given figure. The resistance value of the given resistor is nm illuminates a photo-sensitive material which has a
stopping voltage of 0.48 V. What will be the stopping
voltage if the source light is changed with another source
Gold
of wavelength of 474.6 nm?
a. 0.96 V b. 1.25 V c. 0.24 V d. 1.5 V
Red
Green 14. Match List-I with List-II.
Violet
List-I List-II
a. (5700 ± 285) Ω b. (7500 ± 750) Ω
A. R H (Rydberg constant) 1. kg m − 1s − 1
c. (5700 ± 375) Ω d. (7500 ± 375) Ω
B. h (Planck’s constant) 2. kg m 2 s − 1
8. An antenna is mounted on a 400 m tall building. What
will be the wavelength of signal that can be radiated C. µ B (Magnetic field energy density) 3. m − 1
effectively by the transmission tower upto a range of 44 D. η (Coefficient of viscosity) 4. kg m − 1s − 2
km?
a. 37.8 m b. 605 m c. 75.6 m d. 302 m Choose the most appropriate answer from the options
given below.
9. If the rms speed of oxygen molecules at 0°C is 160 m/s,
A B C D A B C D
find the rms speed of hydrogen molecules at 0°C.
a. 2 3 4 1 b. 3 2 4 1
a. 640 m/s b. 40 m/s c. 80 m/s d. 332 m/s
c. 4 2 1 3 d. 3 2 1 4
August Attempt Online JEE Main 2021 63

15. If force (F ), length (L) and time (T ) are taken as the Section B : Numerical Type Questions
fundamental quantities. Then what will be the dimension
of density : 21. A heat engine operates between a cold reservoir at
a. [FL T ]−4 2 −3
b. [FL T ] 2 temperature T2 = 400 K and a hot reservoir at
c. [FL−5 T 2 ] d. [FL−3 T 3 ] temperature T1. It takes 300 J of heat from the hot
reservoir and delivers 240 J of heat to the cold reservoir
16. A co-axial cable consists of an inner wire of radius a in a cycle. The minimum temperature of the hot
surrounded by an outer shell of inner and outer radii b reservoir has to be ............ K.
and c, respectively. The inner wire carries an electric
current i 0, which is distributed uniformly across 22. Two simple harmonic motion, are represented by the
π
cross-sectional area. The outer shell carries an equal equations y1 = 10 sin 3πt +  and
 3
current in opposite direction and distributed uniformly.
What will be the ratio of the magnetic field at a distance y 2 = 5 (sin 3πt + 3 cos 3πt)
x from the axis when (i) x < a and (ii) a < x < b? Ratio of amplitude of y1 to y 2 = x : 1. The value of x is
x2 a2 x2 b2 − a2 ............ .
a. b. c. d.
a2 x2 b2 − a2 x2
23. X different wavelengths may be observed in the
17. The height of victoria falls is 63 m. What is the difference spectrum from a hydrogen sample if the atoms are
in temperature of water at the top and at the bottom of exited to states with principal quantum number n = 6?
fall ? The value of X is ........ .
[Given, 1 cal = 4.2 J and specific heat of water 24. A Zener diode of power rating 2W is to be used as a voltage
= 1 cal g − 1ºC − 1] regulator. If the Zener diode has a breakdown of 10 V
a. 0.147º C b. 14.76º C and it has to regulate voltage fluctuated between 6 V and
c. 1.476º C d. 0.014º C 14 V, the value of R S for safe operation should be ........ Ω
18. A player kicks a football with an initial speed of 25 ms − 1 Rs
at an angle of 45º from the ground. What are the
maximum height and the time taken by the football to
reach at the highest point during motion ? Unregulated Regulated
(Take, g = 10 ms − 2) voltage voltage
a. hmax = 10 m, T = 2.5 s b. hmax = 15.625 m, T = 3.54 s
c. hmax = 15.625 m, T = 1.77 s d. hmax = 3.54 m, T = 0.125 s
25. Wires W1 and W2 are made of same
19. The light waves from two coherent sources have same W1
material having the breaking stress of
intensity I1 = I2 = I0. In interference pattern the intensity 20 kg
. × 109 N/m 2. W1 and W2 have
125
of light at minima is zero. What will be the intensity of
cross-sectional area of 8 × 10− 7 m 2 W2
light at maxima ?
and 4 × 10 − 7m 2, respectively. Masses 10 kg
a. I 0 b. 2 I 0
of 20 kg and 10 kg hang from them as
c. 5 I 0 d. 4 I 0
shown in the figure. The maximum Pan
20. Figure shows a rod AB, which is bent in a 120º circular arc mass that can be placed in the pan
of radius R. A charge (− Q) is uniformly distributed over without breaking the wires is .......... kg.
rod AB. What is the electric field E at the centre of (Use, g = 10 m/s 2)
curvature O?
26. A bullet of 10 g, moving with velocity v, collides head-on
y
with the stationary bob of a pendulum and recoils with
A
velocity 100 m/s. The length of the pendulum is 0.5 m
and mass of the bob is 1 kg. The minimum value of v
.......... m/s, so that the pendulum describes a circle.
(Assume, the string to be inextensible and g = 10 m/s 2)
120º
60º
O
60º x
0.5 m
R
v
10 g 1 kg
B
27. An AC circuit has an inductor and a resistor of resistance
3 3Q $ 3 3Q R in series, such that X L = 3R . Now, a capacitor is added in
a. (i ) b. ($i )
8 πε0R 2 8 π 2ε0R 2 series such that X C =2R . The ratio of new power factor
3 3Q 3 3Q with the old power factor of the circuit is 5: x The value
c. ($i ) d. (−i$ )
16 π 2 ε0R 2 8 π 2 ε0R 2 of x is.
64 Online JEE Main 2021 August Attempt

28. The ratio of the equivalent resistance of the network 29. A plane electromagnetic wave with frequency of 30 MHz
(shown in figure) between the points a and b when travels in free space. At particular point in space and
switch is open and switch is closed is x : 8. The value of x time, electric field is 6 V/m. The magnetic field at this
is point will be x × 10− 8 T. The value of x is.
R 2R
30. A tuning fork is vibrating at 250 Hz. The length of the
shortest closed organ pipe that will resonate with the
a S b tuning fork will be ......... cm.
(Take, speed of sound in air as 340 ms − 1)
2R R

Answers For solution scan


the QR code

1. (b) 2. (d) 3. (b) 4. (d) 5. (c) 6. (c) 7. (d) 8. (b) 9. (a) 10. (c)
11. (d) 12. (d) 13. (b) 14. (b) 15. (a) 16. (a) 17. (a) 18. (c) 19. (d) 20. (d)
21. (500) 22. (1) 23. (15) 24. (20) 25. (40) 26. (400) 27. (1) 28. (9) 29. (2) 30. (34)

31 AUGUST SHIFT I
Objective Type Questions 1
4. A reversible engine has an efficiency of . If the
4
1. A helicopter is flying horizontally with a speed v at an
temperature of the sink is reduced by 58°C, its efficiency
altitude h has to drop a food packet for a man on the
becomes double. Calculate the temperature of the sink.
ground. What is the distance of helicopter from the man
a. 174°C b. 280°C
when the food packet is dropped?
c. 180.4°C d. 382°C
2 ghv 2 + 1
a. b. 2 ghv 2 + h 2 5. An object is placed at the focus of concave lens having
h2
focal length f . What is the magnification and distance of
2
2v h 2 gh the image from the optical centre of the lens?
c. + h2 d. +h 2
g v2 a. 1, ∞ b. Very high, ∞
1 f 1 f
2. In the following logic circuit, the sequence of the inputs c. , d. ,
A, B are (0, 0), (0,1), (1, 0) and (1, 1). The output Y for this 2 2 4 4
sequence will be
6. A sample of a radioactive nucleus A disintegrates to
A another radioactive nucleus B, which in turn
B disintegrates to some other stable nucleus C . Plot of a
P
Y graph showing the variation of number of atoms of
Q nucleus B versus time is
(Assume that at t = 0, there are no B atoms in the sample)
a. 1, 0, 1, 0 b. 0, 1, 0, 1
c. 1, 1, 1, 0 d. 0, 0, 1, 1
No. of atoms

No. of atoms

3. Two particles A and B having charges 20 µC and − 5 µC a. b.


respectively are held fixed with a separation of 5 cm. At
what position a third charged particle should be placed,
so that it does not experience a net electric force? Time Time
20µC –5µC
No. of atoms

A 5 cm B
No. of atoms

a. At 5 cm from 20 µC on the left side of system c. d.


b. At 5 cm from – 5 µC on the right side
c. At 1.25 cm from – 5 µC between two charges
Time Time
d. At mid-point between two charges
August Attempt Online JEE Main 2021 65

V
7. A coil having N turns is wound tightly in the form of a
spiral with inner and outer radii a and b, respectively.
+3
Find the magnetic field at centre, when a current I passes c. t
through coil –3
µ 0IN µ 0I  a + b 
ln  
b
a. b. ln  
2(b − a )  a  8 a − b 
µ 0I µ 0I  a − b 
ln  − 
1 1 V
c. d. ln  
4 (a − b )  a b  8 a + b
+3
d. t
8. A body of mass M moving at speed v 0 collides elastically –3
with a mass m at rest. After the collision, the two masses
move at angles θ1 and θ 2 with respect to the initial
direction of motion of the body of mass M. The largest
possible value of the ratio M/m, for which the angles θ1
12. A uniform heavy rod of weight 10 kg ms −2, cross-sectional
area 100 cm 2 and length 20 cm is hanging from a fixed
and θ 2 will be equal, is
support. Young’s modulus of the material of the rod is
a. 4 b. 1 c. 3 d. 2
2 × 1011 Nm − 2. Neglecting the lateral contraction, find the
9. The masses and radii of the Earth and Moon are (m1, R1) elongation of rod due to its own weight.
a. 2 × 10− 9 m b. 5 × 10− 8 m
and (m 2, R 2), respectively. Their centres are at a distance
r apart. Find the minimum escape velocity for a particle c. 4 × 10− 8 m d. 5 × 10− 10 m
of mass m to be projected from the middle of these two 13. Two plane mirrors M1 and M 2 are at right angle to each
masses.
other shown. A point source P is placed at a and 2a
1 4G (m1 + m 2 ) 4G (m1 + m 2 )
a. v = b. v = meter away from M1 and M 2, respectively. The shortest
2 r r
distance between the images thus formed is
1 2G (m1 + m 2 ) 2G (m1 + m 2 )
c. v = d. v = (Take 5 = 2.3)
2 r r
a
P
10. A small square loop of side a and one turn is placed
inside a larger square loop of side b and one turn (b >>
a). The two loops are coplanar with their centres M1
coinciding. If a current l is passed in the square loop of 2a
side b, then the coefficient of mutual inductance
between the two loops is
µ0 a2 µ0 8 2 µ0 b2 µ0 8 2
a. 8 2 b. c. 8 2 d.
4π b 4π a 4π a 4π b M2
11. Choose the correct wave form that can represent the a. 3a b. 4.6 a c. 2.3 a d. 2 10 a
voltage across R of the following circuit, assuming the
diode is ideal one.
14. Match List-I with List-II.
D List-I List-I
A. Torque 1. MLT − 1
B. Impulse 2. MT − 2
R
V i =10 sinωt ~ C. Tension 3. ML2 T − 2
+ D. Surface tension 4. MLT − 2
3V

Choose the most appropriate answer from the options
V given below.
A B C D A B C D
+3 a. 3 1 4 2 b. 2 1 4 3
a. t c. 1 3 4 2 d. 3 4 1 2
–3
15. For an ideal gas the instantaneous change in pressure p
dp
with volume V is given by the equation = − ap. If p = p0
V dV
at V = 0 is the given boundary condition, then the
+3 maximum temperature one mole of gas can attain is
b. t
–3 (Here R is the gas constant)
p0 ap 0
a. b. c. infinity d. 0ºC
aeR cR
66 Online JEE Main 2021 August Attempt

16. Which of the following equations is dimensionally two diametrically opposite points of the circle in unit of
incorrect ? Ω will be
Where, t = time, h = height, s = surface tension, θ = angle, ρ 25. A wire having a linear mass density
= density, a, r = radius, g = acceleration due to gravity, V = 9.0 × 10− 4 kg/m is stretched between two rigid supports
volume , p = pressure, W = work done, τ = torque, ε = with a tension of 900 N. The wire resonates at a
permittivity, E = electric field, J = current density, L = frequency of 500 Hz. The next higher frequency at which
length. the same wire resonates is 550 Hz. The length of the wire
πpa 4 2s cos θ is ......... m.
a. V = b. h =
8ηL ρrg 26. The voltage drop across 15Ω resistance in the given
∂E figure will be ......... V.
c. J = ε d. W = τθ
∂t
4Ω 15Ω
17. Angular momentum of a single particle moving with 2Ω
constant speed along circular path 4Ω 10Ω
a. changes in magnitude but remains same in the direction
a b
b. remains same in magnitude and direction 8Ω 12Ω
c. remains same in magnitude but changes in the direction
d. is zero 8Ω 12Ω
18. In an AC-circuit, an inductor, a capacitor and a resistor
are connected in series with X L = R = X C . Impedance of 12 V 1Ω
this circuit is
2
a. 2R b. Zero
c. R d. R 2 27. A block moving horizontally on a smooth surface with a
19. A moving proton and electron have the same de-Broglie speed of 40 ms − 1 splits into two equal parts. If one of
wavelength. If k and p denote the KE and momentum, the parts moves at 60 ms − 1 in the same direction, then
respectively. Then, choose the correct option. the fractional change in the kinetic energy will be x : 4,
a. K p < K e and p p = p e b. K p = K e and p p = p e where x is
c. K p < K e and p p < p e d. K p > K e and p p = p e 28. The electric field in an electromagnetic wave
20. Consider a galvanometer shunted with 5 Ω resistance is given by E = (50 NC − 1) sin ω  t −  .
x
and 2% of current passes through it. What is the  c
resistance of the given galvanometer ? . × 10− 12
The energy contained in a cylinder of volume V is 55
a. 300 Ω b. 344 Ω J. The value of V is .......... cm 3 .
c. 245 Ω d. 226 Ω (Given ε0 = 8.85 × 10− 12 C 2 N − 1 m − 2 ).
Section B : Numerical Type Questions 29. A capacitor of 50 µF is connected in a circuit as shown in
figure. The charge on the upper plate of the capacitor is
21. When a rubber ball is taken to a depth of ...... m in deep .............. µC.
sea, its volume decreases by 0.5%. (The bulk modulus of
rubber = 9.8 × 108 Nm − 2. Density of sea water = 103 kg m
−3 2 kΩ
,
6V
g = 9.8 m/s 2)
2 kΩ
22. A particle of mass 1 kg is hanging from a spring of force
constant 100 Nm − 1. The mass is pulled slightly
downward and released, so that it executes free simple 2 kΩ C=50 µF
harmonic motion with time period T . The time when the
kinetic energy and potential energy of the system will
become equal, is T / x . The value of x is.
23. If the sum of the heights of transmitting and receiving 30. A car is moving on a plane inclined at 30º to the
antennas in the line of sight of communication is fixed at horizontal with an acceleration of
160 m, then the maximum range of LOS communication 10 ms −2 parallel to the plane upward. A bob is
is .......... km. suspended by a string from the roof of the car. The angle
(Take, radius of Earth = 6400 km) in degrees which the string makes with the vertical is
.........
24. A square shaped wire with resistance of each side 3Ω is
bent to form a complete circle. The resistance between (Take, g = 10 ms − 2)
August Attempt Online JEE Main 2021 67

Answers
For solution scan
the QR code
1. (c) 2. (c) 3. (b) 4. (a) 5. (c) 6. (b) 7. (a) 8. (c) 9. (b) 10. (a)
11. (*) 12. (d) 13. (b) 14. (a) 15. (a) 16. (a) 17. (b) 18. (c) 19. (a) 20. (c)
21. (500) 22. (8) 23. (64) 24. (3) 25. (10) 26. (6) 27. (5) 28. (500) 29. (100) 30. (30)

Note (*) None of the option is correct.

31 AUGUST SHIFT II
Section A : Objective Type Questions 5. A free electron of 2.6 eV energy collides with a H+ ion.
1. Four identical hollow cylindrical columns of mild steel This results in the formation of a hydrogen atom in the
3
support a big structure of mass 50 × 10 kg. The inner first excited state and a photon is released. Find the
and outer radii of each column are 50 cm and 100 cm, frequency of the emitted photon.
respectively. Assuming, uniform local distribution, (h = 6.6 × 10− 34 Js)
calculate the compression strain of each column. a. 1.45 × 1016 MHz b. 0.19 × 1015 MHz
[Use, Y = 2.0 × 1011 Pa, g = 9.8 m/s 2]. c. 1.45 × 109 MHz d. 9.0 × 1027 MHz
a. 3.60 × 10−8 b. 2.60 × 10−7 6. Two thin metallic spherical shells of radii r1 and r2 (r1 < r2)
c. 1.87 × 10−3 d. 7.07 × 10−4
are placed with their centres coinciding. A material of
2. A current of 1.5 A is flowing through a triangle, of side 9 thermal conductivity K is filled in the space between the
cm each. The magnetic field at the centroid of the shells. The inner shell is maintained at temperature θ1
triangle is and the outer shell at temperature θ 2(θ1 < θ 2). The rate at
(Assume that, the current is flowing in the clockwise which heat flows radially through the material is
direction.) 4 πKr1r2 (θ 2 − θ1) πr1r2 (θ 2 − θ1)
a. b.
a. 3 × 10− 7 T, outside the plane of triangle r2 − r1 r2 − r1
b. 2 3 × 10− 7 T, outside the plane of triangle K (θ 2 − θ1) K (θ 2 − θ1) (r2 − r1)
c. d.
c. 2 3 × 10− 5 T, inside the plane of triangle r2 − r1 4 π r1r2
d. 3 × 10− 5 T, inside the plane of triangle
7. If VA and VB are the input voltages (either 5V or 0V) and V0
3. A system consists of two identical spheres each of mass is the output voltage then the two gates represented in
1.5 kg and radius 50 cm at the end of light rod. The
the following circuit ( A) and (B) are
distance between the centres of the two spheres is 5 m.
What will be the moment of inertia of the system about V=5V
an axis perpendicular to the rod passing through its D1 RC=1 kΩ
mid-point ? VA
V0
2
a. 18.75 kgm
V0 VB β=150
b. 1.905 × 105 kgm 2
RB=100 kΩ n- p-n
c. 19.05 kgm 2 VB R=1kΩ
d. 1.875 × 105 kgm 2 D2
4. Statement I Two forces (P + Q) and (P − Q) where P ⊥ Q, (B)
(A )
when act at an angle θ1 to each other, the magnitude of
a. AND and OR gate b. OR and NOT gate
their resultant is 3(P 2 + Q 2) , when they act at an angle c. NAND and NOR gate d. AND and NOT gate
θ 2, the magnitude of their resultant becomes 8. Consider two separate ideal gases of electrons and
protons having same number of particles. The
2(P 2 + Q 2) . This is possible only when θ1 < θ 2. temperature of both the gases are same. The ratio of the
Statement II In the situation given above. θ1 = 60 ° and θ2 uncertainty in determining the position of an electron to
= 90°. that of a proton is proportional to
3
In the light of the above statements, choose the most m 2 me
appropriate answer from the options given below. a.  p  b.
 me  mp
a. Statement I is false but statement II is true.
b. Both statement I and statement II are true. mp mp
c. d.
c. Statement I is true but statement II is false. me me
d. Both statement I and statement II are false.
68 Online JEE Main 2021 August Attempt

9. A bob of mass m suspended by a thread of length l 15. A mixture of hydrogen and oxygen has volume 500 cm 3,
undergoes simple harmonic oscillations with time period temperature 300 K, pressure 400 kPa and mass 0.76 g.
T . If the bob is immersed in a liquid that has density 1/4 The ratio of masses of oxygen to hydrogen will be
times that of the bob and the length of the thread is a. 3 : 8 b. 3 : 16 c. 16 : 3 d. 8 : 3
increased by 1/3rd of the original length, then the time
period of the simple harmonic oscillations will be 16. A block moving horizontally on a smooth surface with a
a. T
3
b. T c.
3
T d.
4
T
speed of 40 m/s splits into two parts with masses in the
2 4 3 ratio of 1: 2. If the smaller part moves at 60 m/s in the same
direction, then the fractional change in kinetic energy is
10. Statement I If three forces F1 ⋅ F2 and F3 are represented
a. 1/3 b. 2/3 c. 1/8 d. 1/4
by three sides of a triangle and F1 + F2 = − F3, then these
three forces are concurrent forces and satisfy the 17. A coil is placed in a magnetic field B as shown below.
condition for equilibrium.
Induced
Statement II A triangle made up of three forces F1, F2 and F3 Coil current
as its sides taken in the same order, satisfy the condition
B
for translatory equilibrium.
In the light of the above statements, choose the most
appropriate answer from the options given below.
a. Statement I is false but statement II is true. A current is induced in the coil because B is
b. Statement I is true but statement II is false. a. outward and decreasing with time
c. Both statement I and statement II are false. b. parallel to the plane of coil and decreasing with time
d. Both statement I and statement II are true. c. outward and increasing with time
d. parallel to the plane of coil and increasing with time
11. If velocity [v ], time [T ] and force [F ] are choosen as the
base quantities, the dimensions of the mass will be 18. For a body executing SHM
a. [ FT − 1v − 1] b. [ FTv − 1] A. potential energy is always equal to its kinetic energy.
c. [ FT 2v ] d. [ FvT − 1] B. average potential and kinetic energy over any given time
interval are always equal.
12. The magnetic field vector of an electromagnetic wave is C. sum of the kinetic and potential energy at any point of time
$i + $j
given by B = B is constant.
0 cos( kz − ωt) T where $i, $j represents
2 D. average kinetic energy in one time period is equal to
average potential energy in one time period.
unit vector along X and Y-axis respectively. At t = 0, two
electric charges q1 of 4 π C and q 2 of 2π C located at Choose the most appropriate option from the options
 0,0, π  and  0,0, 3π  respectively, have the same velocity given below.
  
k  k a. (C) and (D) b. Only (C) c. (B) and (C) d. Only (B)
of 0.5 c i. $ (where, c is the velocity of light). The ratio of the
19. Statement I To get a steady DC output from the pulsating
force acting on charge q1 to q 2 is voltage received from a full wave rectifier we can
a. 2 2 : 1 b. 1: 2 c. 2 : 1 d. 2 : 1 connect a capacitor across the output parallel to the load
13. The equivalent resistance of the given circuit between R L.
the terminals A and B is Statement II To get a steady DC output from the pulsating
2Ω 2Ω voltage received from a full wave rectifier we can connect
A an inductor in series with RL.
In the light of the above statements, choose the most
5Ω 2Ω 3Ω 3Ω appropriate answer from the options given below.
a. Statement I is true but statement II is false.
B b. Statement I is false but statement II is true.
c. Both statement I and statement II are false.
a. 0 b. 3 Ω c. 9 / 2 Ω d. 1 Ω
d. Both statement I and statement II are true.
14. Choose the incorrect statement.
A. The electric lines of force entering into a Gaussian surface 20. If R E be the radius of Earth, then the ratio between the
provide negative flux.
acceleration due to gravity at a depth r below and a
B. A charge q is placed at the centre of a cube. The flux
height r above the Earth surface is (Given, r < R E )
through all the faces will be the same.
r r2 r3 r r2 r3
C. In a uniform electric field net flux through a closed a. 1 − − − b. 1 + + +
Gaussian surface containing no net charge is zero. R E R E2 R E3 R E R E2 R E3
D. When electric field is parallel to a Gaussian surface, it r r2 r3 r r2 r3
c. 1 + − + d. 1 + − −
provides a finite non-zero flux. R E R E2 R E3 R E R E2 R E3
Choose the most appropriate answer from the options
given below.
a. (C) and (D) b. (B) and (D) c. Only (D) d. (A) and (C)
August Attempt Online JEE Main 2021 69

Section B : Numerical Type Questions One main scale division is 1 mm. The main scale reading
is 10 mm and 8th division of vernier scale was found to
21. A bandwidth of 6 MHz is available for AM transmission. If
coincide exactly with one of the main scale division. If the
the maximum audio signal frequency used for
given vernier callipers has positive zero error of 0.04 cm,
modulating the carrier wave is not to exceed 6 kHz. The
then the radius of the bob is .......... × 10− 2 cm.
number of stations that can be broadcasted within this
band simultaneously without interfering with each other 27. A sample of gas with γ = 1.5 is taken through an adiabatic
will be ............ . process in which the volume is compressed from 1200 cm
3
22. A parallel plate capacitor of capacitance to 300 cm 3. If the initial pressure is 200 kPa. The
200 µF is connected to a battery of 200 V. A dielectric absolute value of the work done by the gas in the
slab of dielectric constant 2 is now inserted into the process is ......... J.
space between plates of capacitor while the battery 28. At very high frequencies, the effective impedance of the
remain connected. The change in the electrostatic given circuit will be .........Ω.
energy in the capacitor will be ......... J.
1Ω 0.5F 2Ω 0.5F
23. A long solenoid with 1000 turns/m has a core material
with relative permeability 500 and volume 10 3cm 3. If the
core material is replaced by another material having 0.5F
relative permeability of 750 with same volume 1Ω 20H 2Ω 0.5F
maintaining same current of 0.75 A in the solenoid, the
fractional change in the magnetic moment of the core
χ 
would be approximately   . Find the value of χ.
 499 220V

24. A particle is moving with constant acceleration a. 29. Cross–section view of a prism is the equilateral triangle
Following graph shows v 2 versus x (displacement) plot. ABC in the figure. The minimum deviation is observed
The acceleration of the particle is ........ m/s 2 using this prism when the angle of incidence is equal to
the prism angle. The time taken by light to travel from P
(mid-point of BC ) to A is ..... × 10− 10 s.
80 C
(Given, speed of light in vacuum = 3 × 108 m/s and
v2 (m/s)2

60 3
B cos30 ° = )
40 2
A
A
20

0 10 20 30
x (m) 10 cm 10 cm

25. In a Young’s double slit experiment, the slits are


separated by 0.3 mm and the screen is 1.5 m away from
the plane of slits. Distance between fourth bright fringes B P C
on both sides of central bright is 2.4 cm. The frequency 30. A resistor dissipates 192 J of energy in 1 s when a current
of light used is ........ × 1014 Hz. of 4 A is passed through it. Now, when the current is
26. The diameter of a spherical bob is measured using a doubled, the amount of thermal energy dissipated in 5 s
Vernier callipers. 9 divisions of the main scale, in the is ........ J.
vernier calipers, are equal to 10 divisions of vernier scale.

Answers For solution scan


the QR code

1. (b) 2. (d) 3. (c) 4. (b) 5. (c) 6. (a) 7. (b) 8. (c) 9. (d) 10. (d)
11. (b) 12. (c) 13. (d) 14. (c) 15. (c) 16. (c) 17. (a) 18. (a) 19. (d) 20. (d)
21. (500) 22. (4) 23. (250) 24. (1) 25. (5) 26. (52) 27. (480) 28. (2) 29. (5) 30. (3840)
70 Online JEE Main 2021 September Attempt

1 SEPTEMBER SHIFT II
Section A : Objective Type Questions 6. A block of mass m slides on m
1. A cube is placed inside an electric field,E = 150 y 2$j. The the wooden wedge, which
in turn slides backward on M
side of the cube is 0.5 m and is placed in the field as the horizontal surface. The
shown in the given figure. The charge inside the cube is acceleration of the block
y with respect to the wedge is
[Given, m = 8 kg, M = 16 kg] 30°
Assume all the surfaces
shown in the figure to be frictionless.
4 6 3 2
x a. g b. g c. g d. g
3 5 5 3

7. Due to cold weather a 1 m water pipe of cross–sectional


z area 1 cm 2 is filled with ice at –10°C. Resistive heating is
used to melt the ice. Current of 0.5 A is passed through
a. 3.8 × 10−11 C b. 8.3 × 10−11 C
4 kΩ resistance. Assuming that, all the heat produced is
−12
c. 3.8 × 10 C d. 8.3 × 10−12 C used for melting, what is the minimum time required ?
. × 105 J kg
[Given, latent heat of fusion for water/ice = 333
2. A square loop of side 20 cm and resistance −1 3 −1
, specific heat of ice = 2 × 10 J kg and density of ice
1 Ω is moved towards right with a constant speed v 0. The
= 103 kg/m 3]
right arm of the loop is in a uniform magnetic field of 5
T. The field is perpendicular to the plane of the loop and a. 0.353 s b. 35.3 s c. 3.53 s d. 70.6 s
is going into it. The loop is connected to a network of
8. A student determined Young’s modulus of elasticity
resistors each of value 4 Ω. What should be the value of MgL3
v 0, so that a steady current of 2 mA flows in the loop ? using the formula Y = . The value of g is taken to
4 bd 3δ
×××××××× 2
be 9.8 m/s , without any significant error, his
××××××××
4Ω 4Ω ×××××××× observations are as following.
××××××××
P Physical quantity Least count of the Observed
××××××××
4Ω 4Ω ×××××××× equipment used for value
Q
×××××××× measurement
×××××××× Mass (M) 1g 2 kg
××××××××
v0 Length of bar (L) 1 mm 1m
Breadth of bar (b) 0.1 mm 4 cm
a. 1 m/s b. 1 cm/s c. 102 m/s d. 10−2 cm/s
Thickness of bar (d ) 0.01 mm 0.4 cm
3. The temperature of an ideal gas in 3-dimensions is 300 Depression (δ) 0.01 mm 5 mm
K. The corresponding de-Broglie wavelength of the
electron approximately at 300 K, is Then, the fractional error in the measurement of Y is
[m e = mass of electron = 9 × 10−31 kg a. 0.0083 b. 0.0155 c. 0.155 d. 0.083
h =Planck’s constant = 6.6 × 10−34 Js
K B = Boltzmann constant = 1.38 × 10−23 JK −1] 9. Two resistors R1 = ( 4 ± 0.8) Ω and R 2 = ( 4 ± 0.4) Ω are
a. 6.26 nm b. 8.46 nm c. 2.26 nm d. 3.25 nm connected in parallel. The equivalent resistance of their
parallel combination will be
4. A body of mass m dropped from a height h reaches the a. (4 ± 0.4 ) Ω b. (2± 0.4 ) Ω
ground with a speed of 0.8 gh. The value of workdone c. (2± 0.3) Ω d. (4 ± 0.3) Ω
by the air-friction is
a. − 0.68 mgh b. mgh c. 1.64 mgh d. 0.64 mgh 10. The half life period of radioactive element x is same as
the mean life time of another radioactive element y.
5. The ranges and heights for two projectiles projected Initially they have the same number of atoms. Then,
with the same initial velocity at angles 42° and 48° with a. x will decay faster than y
the horizontal are R1, R 2 and H1, H 2, respectively. Choose b. y will decay faster than x
the correct option. c. x and y have same decay rate initially and later on different
a. R1 > R 2 and H1 = H2 b. R1 = R 2 and H1 < H2 decay rate
c. R1 < R 2 and H1 < H2 d. R1 = R 2 and H1 = H2 d. x and y decay at the same rate always
September Attempt Online JEE Main 2021 71

11. Following plots show magnetisation (M) versus 15. An object of mass m is being moved
magnetising field (H) and magnetic susceptibility (χ) with a constant velocity under the
action of an applied force of 2 N m
versus temperature (T ) graph θ θ
along a frictionless surface with
M M D
following surface profile.
The correct applied force versus distance graph will be
(A) (B)
H H F
F
2N

χ χ 2N
x
a. b.
(C) T (D) x
T –2N
D

D
Which of the following combination will be represented
F
by a diamagnetic material? F
2N
a. (A) and (C) b. (A) and (D) c. (B) and (D) d. (B) and (C)
D x
12. A glass tumbler having inner depth of 17.5 cm is kept on c. x b.
a table. A student starts pouring water (µ = 4 / 3) into it
while looking at the surface of water from the above. –2N
When he feels that the tumbler is half filled, he stops –2N
pouring water. Up to what height, the tumbler is actually D
filled?
a. 11.7 cm b. 10 cm c. 7.5 cm d. 8.75 cm 16. A mass of 5 kg is
connected to a spring. 10
13. In the given figure, each diode has a forward bias The potential energy
resistance of 30 Ω and infinite resistance in reverse bias. curve of the simple

U(J)
harmonic motion 5
The current I1 will be
executed by the system is
130 Ω shown in the figure. A
simple pendulum of 0 2 4
length 4 m has the same
130 Ω x(m)
period of oscillation as
the spring system. What
130 Ω is the value of acceleration due to gravity on the planet
where these experiments are performed?
I1
20 Ω a. 10 m/s 2 b. 5 m/s 2 c. 4 m/s 2 d. 9.8 m/s 2

200 V 17. A capacitor is connected to a 20 V battery through a


resistance of 10 Ω. It is found that the potential
a. 3.75 A b. 2.35 A c. 2 A d. 2.73 A difference across the capacitor rises to 2 V in 1 µs. The
capacitance of the capacitor in .……… µF.
14. For the given circuit the current i through the battery  10
[Given, ln   = 0.105 ]
when the key in closed and the steady state has been  9
reached is
a. 9.52 b. 0.95 c. 0.105 d. 1.85
2Ω
18. Four particles each of mass M, M M
move along a circle of radius R
i R R
0.5mH 0.2 H under the action of their mutual
3Ω gravitational attraction as shown
30V in figure. The speed of each 90° R
3Ω 3Ω particle is R
M M
1 GM 1 GM
a. b. (2 2 + 1)
2 R(2 2 + 1) 2 R
a. 6 A b. 25 A c. 10 A d. 0 1 GM GM
c. (2 2 − 1) d.
2 R R
72 Online JEE Main 2021 September Attempt

19. Electric field of plane electromagnetic wave propagating 24. When a body slides down from rest along a smooth
through a non–magnetic medium is given by inclined plane making an angle of 30° with the
E = 20cos(2 × 1010t − 200x) V/m. The dielectric constant of horizontal, it takes time T . When the same body slides
down from the rest along a rough inclined plane making
the medium is equal to
the same angle and through the same distance, it takes
(Take, µ r = 1)
time αT , where α is a constant greater than 1. The
a. 9 b. 2 c. 1/3 d. 3
coefficient of friction between the body and the rough
1  α − 1
2
20. There are two infinitely long straight current carrying plane is   , where x is ……… .
conductors and they are held at right angles to each x  α2 
other so that their common ends meet at the origin as
shown in the figure given below. The ratio of current in 25. The average translational kinetic energy of N2 gas
both conductors is 1 : 1. The magnetic field at point P is
molecules at ...........°C becomes equal to the KE of an

y electron accelerated from rest through a potential
difference of 0.1 V.
P(x,y) [Given, K B = 1.38 × 10−23 J/K]
θ2
26. A uniform heating wire of resistance 36 Ω is connected
θ1
I
across a potential difference of 240 V. The wire is then
cut into half and potential difference of 240 V is applied
across each half separately. The ratio of power
O ∞x dissipation in first case to the total power dissipation in
I
the second case would be 1: x, where x is ……… .
µ 0I µ 0I
a. [ x 2 + y 2 + (x + y )] b. [ x 2 + y 2 − (x + y )]
4 πxy 4 πxy 27. A steel rod with Y = 2.0 × 1011 Nm −2 and α = 105 °C −1 of
µ Ixy µ Ixy length 4 m and area of cross-section 10 cm 2 is heated
c. 0 [ x 2 + y 2 − (x + y )] d. 0 [ x 2 + y 2 + (x + y )]
4π 4π from 0°C to 400°C without being allowed to extend. The
tension produced in the rod is x × 105 N, where the value
Section B : Numerical Type Questions
of x is ............. .
21. The temperature of 3.00 mol of an ideal diatomic gas is
increased by 40.0 °C without changing the pressure of 28. A 2 kg steel rod of length 0.6 m is clamped on a table
the gas. The molecules in the gas rotate but do not vertically at its lower end and is free to rotate in vertical
oscillate. If the ratio of change in internal energy of the plane. The upper end is pushed so that the rod falls
x under gravity. Ignoring the friction due to clamping at its
gas to the amount of workdone by the gas is , then lower end, the speed of the free end of rod when it
10
the value of x (round off to the nearest integer) is ……… . passes through its lowest position is ........... ms −1.
(Given, R = 8.31 J mol −1 K −1) (Take, g = 10 ms −2)
29. A carrier wave with amplitude of 250 V is amplitude
22. The width of one of the two slits in a Young’s double slit
modulated by a sinusoidal base band signal of
experiment is three times the other slit. If the amplitude
amplitude 150 V. The ratio of minimum amplitude to
of the light coming from a slit is proportional to the
maximum amplitude for the amplitude modulated wave
slit-width, the ratio of minimum to maximum intensity in
is 50 : x, then value of x is ........... .
the interference pattern is x : 4 where x is ……… .
30. An engine is attached to a wagon through a shock
23. Two satellites revolve around a planet in coplanar
absorber of length 1.5 m. The system with a total mass
circular orbits in anti-clockwise direction. Their period of
of 40000 kg is moving with a speed of 72 kmh −1, when
revolutions are 1 h and 8 h, respectively. The radius of
the brakes are applied to bring it to rest. In the process
the orbit of nearer satellite is 2 × 103 km. The angular
of the system being brought to rest, the spring of the
speed of the farther satellite as observed from the
shock absorber gets compressed by 1.0 m. If 90% of
nearer satellite at the instant when both the satellites
π energy of the wagon is lost due to friction, the spring
are closest is rad h −1, where x is ……… . constant is ……… × 105 N/m.
x

Answers For solution scan


the QR code

1. (b) 2. (b) 3. (a) 4. (a) 5. (b) 6. (d) 7. (b) 8. (b) 9. (c) 10. (b)
11. (a) 12. (b) 13. (c) 14. (c) 15. (b) 16. (c) 17. (b) 18. (b) 19. (a) 20. (a)
21. (25) 22. (1) 23. (3) 24. (3) 25. (500) 26. (4) 27. (8) 28. (6) 29. (4) 30. (16)
EXAM BITES

This Pdf Is
Downloaded From
www.exambites.in

Visit www.exambites.in for


More Premium Stuffs,Latest
Books,Test Papers,Lectures etc.
jeeneetadda
jeeneetadda_official
jeeneetadda

VISIT NOW !!

You might also like